4D 4G Labrev Volume 1

You might also like

Download as pdf or txt
Download as pdf or txt
You are on page 1of 1094

4D – ANTONIO, Marielle

PAGE
CASE TITLE
NO.

Citibank v. CA, November 27, 1998 1


PAL v. NLRC, March 20, 1998 2
San Miguel Corporation v. Bersamira, June 13, 1990 7
Penaranda v. Bagong Plywood Corp, May 3, 2006 10
Pondoc v. NLRC, October 3, 1996 12
Villamaria v. CA, April 19, 2006 15
Mendoza v. Officers of Manila Water, January 25, 2016 19
Atlas Farms v. NLRC, November 18, 2002 23
Negros Metal v. Lamayo, August 25, 2010 26
Vivero v. CA, October 24, 2000 28
University of Immaculate Concepcion v. NLRC, January 26, 2011 32
Austria v. NLRC, August 16, 1999 34
Reyes v. RTC Makati Branch 42, August 11, 2008 36

AQUINO, Gem Edward


PAGE
CASE TITLE
NO.
Locsin v. Nissan Lease Philippines October 20, 2010 38
Weslayan University v. Maglaya, January 23, 2017 41
Cacho v. Balagtas, February 7, 2018 43
Paredes v. Feed the Children Philippines, September 9, 2015 46
Lunzaga v. Albar Shipping, April 18, 2012 48
Santos v. Servier Philippines Inc. November 28, 2008 50
World’s Best Gas v. Vital, September 9, 2015 52
Halaguena v. PAL October 2, 2009 54
Pepsi Cola v. Gal-lang, September 24, 1991 57
Banez v. Valdevilla, May 9, 2000 60
Milan v. NLRC, February 4, 2015 63
Amecos Innovations v. Lopez, July 2, 2014 66
PAL v. ALPAP, February 26, 2018 68

ASUNCION, Mar Lorenzo B.


PAGE
CASE TITLE
NO.
Dai-chi Electronics v. Villarama, November 21, 1994 71
People’s Broadcasting Service v. Sec of Labor, March 6, 2012 73
Ex-Bataan Veterans Security v. Laguesma, November 20, 2007 75
Meteoro v. Creative Creatures, July 13, 2009 77
Okol v. Slimmers World December 11, 2009 79
Santiago v. CF Sharp Crew Management , July 10, 2007 81
Industrial Personnel and Management v. De Vera, March 7, 2016 84
Ace Navigation v. Fernandez, October 10, 2012 88
Estate of Nelson Dulay v. Aboitiz Jebsen Maritime 90
LRTA v. Alvarez, November 18, 2016 92
GSIS v. NLRC, November 17, 2010 95
Duty Free Phils. V. Mojica, September 30, 2005 98
WPP Marketing Comunications v. Galera, March 25, 2010 100

CHAVEZ, Jordan
PAGE
CASE TITLE
NO.

Pakistan International Airlines v. Ople, September 28, 1990 104


PNB v. Cabansag, June 21, 2005 107
Manila Hotel v. NLRC, October 13, 2000 109
Saudi Arabian Airlines v. Rebesencio, January 14, 2015 111
Continental Micronesia v. Basco, September 23, 2015 113
Kawachi v. Del Quero, March 27, 2007 115
Perpetual Help Credit Cooperative v. Faburada, October 8, 2001 117
7K Corp. V. Albanco, June 26, 2013 120
Kawachi et. al. v. Del Quero, March 27, 2007 122
San Miguel v. Semillano, July 5, 2010 124
Ellao v. BATELEC, July 9, 2010 126
Comscentre v. Ricio, January 22, 2020 128
Gemudiano v. Naess Shipping, January 20, 2020 130

CRUZ, Asher Grace M.


PAGE
CASE TITLE
NO.
Pasay City Alliance Church v. Benito, November 28, 2019 132
Tumaodos v. San Miguel, February 15, 2020 135
Zonio v. 88 Aces Maritime Services , October 16, 2019 138
PAL v. NLRC, March 20, 1998 141
Landbank of the Phils. v. Listana, August 5, 2008 143
Robosa v. NLRC, February 8, 2012. 146
Jolo’s Kiddie v. Cabilla, November 29, 2017 148
Frondozo v. Meralco, August 22, 2017 151
Bisig Mangagawa sa Concrete v. NLRC, September 16, 1993 156
Meralco v. Gala, March 7, 2012 159
Nationwide Security and Allied Services v. CA, July 14, 2008 162
Diamond Taxi v. Llamas, March 12, 2014 164
Sara Lee v. Macatlang, January 14, 2015 167

CUNANAN, Cyrus S.
PAGE
CASE TITLE
NO.

Dela Rosa Liner v. Borela, July 29, 2015 170


Magsaysay Maritime v. De Jesus, August 30, 2017. 173
GBMLT Manpower v. Malinao, July 6, 2015 176
Orozco v. CA, April 29, 2005 179
Lepanto Consolidated Mining v. Icao, January 15, 2014 182
Forever Security v. Flores, September 7, 2007 184
UERM- Memorial Medical Center v. NLRC, March 3, 1997 187
Manila Mining v. Amor , April 20, 2015 189
Banahaw Broadcasting v. Pacana, May 30, 2011 191
Mc Burnie v. Ganzon, October 17, 2013 195
Sara Lee Phils. V. Macatlang, January 14, 2015 198
AFP General Insurance v. Molina, June 30, 2008 201
Islriz Trading v. Capade et. al. January 31, 2011 204

DAQUIOAG, Alecx Frances S.


PAGE
CASE TITLE
NO.
FSFI v. NLRC, December 11, 2003 207
Buenaobra v. Lim King Guan, January 20, 2004 209
Bergonio v. SEAIR, April 21, 2014 211
Loon v. Power Master , December 11, 2013. 213
Waterfront Cebu City Casino v. Ledesma, March 25, 2015 215
Balite v. SSS Ventures, February 4, 2015 217
Turks Shawarama v. Pajaron, January 16, 2017 220
Pioneer Texturizing Corporation v. NLRC, 1997 case 222
Roquero v. PAL, 2 April 2004 224
Air Phil Corp. v. Zamora, August 7, 2004 226
Lansangan v. Amkor Technology Philippines, January 30, 2009 228
Genuino v. NLRC, December 4, 2007 230
Garcia et al. v. PAL, January 20, 2009 232
DE LOS REYES, Jana Marie F.
PAGE
CASE TITLE
NO.

Mt. Carmel College v. Resuena October 10, 2007 234


Buenviaje v. CA, November 12, 2002 236
Pfizer Inc. v. Velasco, March 9, 2011 238
Wenphil Corp. v. Abing, April 7, 2014 240
Smart Communications v. Solidum, April 15, 2015 242
Manila Doctors College v. Oloroes, October 3, 2016 243
Sy et. al., v. Fairland Knitcraft Co., December 12, 2011 245
IBM Nestle v. Nestle Phils, September 23, 2015 247
Yupangco Cotton Mills v. CA, January 16, 2002 249
Ando v. Campo, February 16, 2011 251
PAL v. Bischara, September 2, 2015 253
Guillermo v. Uson, March 7, 2016 255
Dutch Movers Inc. v. Lequin et al, April 25, 2017 257

DUTERTE, Camille Anne E.


PAGE
CASE TITLE
NO.

Genuino Agro v. Romano, September 18, 2019 259


Carag v. NLRC, April 2, 2007 261
Roca v. Dabuyan, March 5, 2018 263
Fernandez et. al. v. NewfieldStaff Solution, July 10, 2013 265
St. Martin Funeral Homes v. NLRC, September 16, 1998 267
Veloso v. China Airlines, July 14, 1999 270
Stanfilco v. Tequillo, July 17, 2019 272
Hanjin Engineering v. CA, April 10, 2006 274
Employees Union of Bayer Phils. v. Bayer Phils, Dec. 6, 2010 277
Montaño v. Verceles, July 26, 2010 280
Diokno et al., v. Cacdac, July 4, 2007 282
La Tondeña Workers v. Secretary of Labor, December 9, 1994 284
Abbot Laboratories v. Abbot Laboratories Employee Union 286

FAN, Paulon L.
PAGE
CASE TITLE
NO.

Takata Corporation v. BLR, June 4, 2014 288


Magbuana v. Uy, May 6, 2005 290
Philippine Transmarine Carrier v. Pelagio, August 12, 2015 293
Magsaysay Maritime v. De Jesus, August 30, 2017 296
Solomon et . al. v. Powertech Corp., January 22, 2008 298
Philippine Journalists Inc., v. NLRC, September 5, 2006 301
Periquet v. NLRC, June 22, 1990 304
Aujero v. Philcomsat, January 18, 2012 306
Carolina’s Lace Shoppe v. Maquilan, April 10, 2019 309
Colegio De San Juan De Letran v. Association of Employees and
312
Faculty of Letran et. al, September 18, 2000
Mariwasa Siam Ceramics v. Sec of Labor, December 21, 2009 315
Electromat Manufacturing v. Lagunzad et. al., July 27, 2011 317
Eagle Ridge Golf and Country Club v. CA, March 18, 2010 320

FLORALDE, Genevieve E.
PAGE
CASE TITLE
NO.
Tagaytay Highlands International Golf Club Inc. v. Tagaytay
323
Highlands Employees Union PGTWO, January 22, 2003
SS Ventures v. SS Ventures Labor Union, July 23, 2008 325
Heritage Hotel v. NUWHRAIN-HHMSC, January 12, 2011 328
Republic of the Phils. v. Kawashima Textile July 23, 2008 331
Del Pilar Academy et. al. v. Del Pilar Academy’s Employee’s
334
Union, April 30, 2008
Edgardo Marino Jr. et. al. v. Gamilia, July 7, 2009 337
Ergonomic Systems Phil Inc. v. Enase, December 13, 2017 343
Heritage Hotel v. NUWHRAIN-HHMSC, January 12, 2011 346
Abaria v. NLRC, December 7, 2011 349
Peninsula Employees Union v. Esquivel, December 1, 2016 352
Ergonomic Systems v. Enaje, December 13, 2017 355
Weslayan University Phils. v. Weslayan University Philippines
358
Faculty and Staff Association, March 12, 2014
Samahan ng Mangagawa sa Hanjin v. BLR, October 14, 2015 360

GINGOYON, Cordell Jericho M.


PAGE
CASE TITLE
NO.
United Pepsi Cola Supervisory v. Laguesma, March 25, 1998 363
Metrolab Industries v. Confesor, February 28, 1996 365
San Miguel Foods v. San Miguel Corporation Supervisors and
368
Exempt Union, August 1, 2011
SMCC v. Charter Chemical and Coating Corp., March 16, 2011 371
Cathay Pacific Steel Corp. V. CA, August 30, 2003 373
AIM v. AIM Faculty Association, January 23, 2017 375
Holy Child Catholic School v. Sto. Tomas, July 23, 2013 377
T&H Shopfitters v. T&H Shopfitters Union, February 26, 2014 379
Ren Transport v. NLRC, June 27, 2016 381
Digital Telecommunications v. Digitel Employees Union, October
383
10, 2012
General Santos Coca-Cola Plant Free Workers Union Tupas v.
385
Coca-Cola Bottlers, Phils. Inc., February 13, 2009
UST Faculty Union v. UST April 7, 2009 387
Phil. Skylanders Inc. v. NLRC Jan 31, 2002 390

GIRON, Jose Justin C.


PAGE
CASE TITLE
NO.
Tropical Hut Employee’s Union v. Tropical Hut January 20, 1990 393
Purefoods Corp. v. Nagkakaisang Samahang Mangagawa ng
397
Purefoods Rank and File, August 28, 2008
De la Salle University v. DLSUEA-NAFTEU, April 7, 2009 400
MSMG-UWP v. Ramos, February 28, 2000 403
Alabang Country Club v. Nlrc, February 14, 2008 408
Standard Chartered Bank Employees union v. Confesor, June
412
16, 2004
GMC v. CA, February 11, 2004 417
Hacienda Fatima v. NFSW Food, January 28, 2003 420
St. John Colleges Inc. v. John Academy Faculty and Employees
423
union October 27, 2006
Central Azucarera De Bais Employees Union NFL v. Central
427
Azucarera De Bais Inc. Nov 17, 2010
UFE-DFA-KMU v. Nestle Phils. Inc. March 3, 2008 430
Malayang Mangagawa ng Stayfast v. NLRC, August 28, 2014. 433
Holy Child Catholic School v. Sec of Labor, July 23, 2013. 436

GOLANGCO, Claudine Gayle S.


PAGE
CASE TITLE
NO.

Employee of Bayer Phils. v Bayer Phils. 440


SACORU v. CCBPI, October 4, 2017 442
SWOFLU v. Universal Robina Corp, July 5, 2017 444
PEU v. Esquivel et al, December 1, 2016 445
UFE-DFA-KMU v. Nestle Phils. Inc. March 3, 2008 446
UST Faculty Union v. UST April 7, 2009 448
General Milling Corporation v. CA, February 2004 450
Kiok loy v. NLRC, January 22, 1986 452
Colegio de San Juan de Letran Case, September 18, 2000 454
PAL v. PALEA, March 12, 2008 455
FVC Labor Union v. SANAMA-FVC-SIGLO, November 27, 2009 457
SMCEU-PTGWO v. Confesor, September 19, 1996

GONZALEZ, Jaymee Samantha O.


PAGE
CASE TITLE
NO.

HBILU v. HSBC, February 28, 2018 459


Manila Electric Co. v. Quisumbing, February 22 & August 1, 2000 462
International School Alliance v. Quisumbing, June 1, 2000 465
National Association of Free Trade Unions v. Mainit Lumber
468
Development Company Workers Union, December 21, 1990
Picop Resources Inc. v. Dequilla et al. December 7, 2011 470
National Union of Workers in Hotels, Restaurants and Allied
473
Industries- Manila Pavillon Hotel v. Sec of Labor, July 31, 2009
Coca Cola Bottlers v. Ilocos Professional and Technical
476
Employees Union, September 9, 2015
Santuyo et. al. v. Remerco Garmets Manufacturing Inc. March
478
22, 2010
Teng v. Pahagac, Nov. 17, 2010 480
Samahan ng mga Mangagawa sa Hyatt v. Magsalin, June 6 483
Baronda v. CA, October 14, 2015 485
NYK-FIL Ship Management v. Dabu, September 13, 2017 488
Guagua National Colleges v. CA, August 28, 2018 491

GUEVARRA, Joy L.
PAGE
CASE TITLE
NO.

Octavio v. PLDT, February 27, 2013 494


Abaria v. NLRC, December 7, 2011 496
YSS Employees Union v. YSS Laboratories, December 4, 2009 499
NUWHRAIN-APL-IUF Dusit Hotel Nikko Chapter v. CA,
502
November 11, 2008
Jackbilt Industries Inc., v. Jackbilt Employees Workers Union-
505
NAFLU-KMU, March 20, 2009
APAP v. PAL, June 6, 2011 508
Olisa et. al. v. Escario Et. al. VCMC v. Yballe, January 15, 2014 511
Tavangao Shell Refinery Employees Association v. Pilipinas
515
Shell, April 7, 2014.
Asia Brewery Inc. v. TPMA, September 18m, 2013. 518
Escario Et. al. v NLRC 2010 case 520
University of San Augustin Employees Union v. CA, (2006 case) 523
Phil Diamond Hotel and Resort Inc. v. Manila Diamond Hotel
527
Employees Union
JACABA, Joyce B.
PAGE
CASE TITLE
NO.

Solidbank v. Gamier, November 15, 2010 530


C Alcantara & Sons v. CA, September 29, 2010 and March 2012 533
Sukhothai Cuisine and Restaurant v. CA, 495 SCRA 336 536
Biflex Phils. Inc. Labor Unions v. Filflex Industrial Manufacturing 539
Sta. Rosa Coca Cola Plant v. Coca-cola Bottlers Phils. Inc 541
Manila Hotel Employee’ Association v. Manila Hotel Corporation 544
G&S Transport Corporation v. Infante, 533 SCRA 289 546
Steel Corporation of the Phils. v. SCP Employees Union National 548
Chris Garments Case (January 2009) 551
University of Immaculate Concepcion v. Sec of Labor, September
554
14, 2006
Ramirez v. Polyson Industries, October 19, 2016 556
Bigg’s v. Boncacas, March 6, 2019 558
St. Lukes Medical Center Inc. v. Notario, October 20, 2010 561
Aliling v. World Express Corp., April 25, 2012 563

NG, Kristoffer Monico S.


PAGE
CASE TITLE
NO.
Perez v. PT&T April 9, 2009 566
Distribution and Control Products Inc. v. Santos, July 10, 2017 568
Agabon v, NLRC, November 17, 2004 570
King of Kings Transport v. Mamac, June 29, 2007 572
Puncia v. Toyota Shaw, June 28, 2019 574
Jaka Food Processing v. Pacot, March 28, 2005 576
Abbot Laboratories v. Alcaraz, July 23, 2013 578
Agabon v. NLRC, November 17, 2004 580
Jaka Food Processing Corp. v. Pacot, March 28, 2005 582
Culili v. Eastern Telecommunications Phils. February 9, 2011 584
Serrano v. Gallant Maritime, March 24, 2009 586
Yap v. Thenamaris Ship’s Management May 30, 2011 588
Bank of Lucbao v. Manabat, Feb 1, 2012 590
St. Mary’s Academy v. Palacio et. al. September 8, 2010 592
PAJARILLAGA, Jan Laurence
PAGE
CASE TITLE
NO.
Toyota Motor Phils. Corp. Workers Association v. NLRC, October
594
19, 2007
Bristol Myers Squibb Inc. v. Haban, December 17, 2008 601
Yrasuegui v. PAL, October 17, 2008 604
Dreamland Hotel Resort v. Johnson, March 12, 2014 608
Manila Water v. Del Rosario, January 29, 2014 613
Nacar v. Gallery Frames, August 13, 2013 617
Bani Rural Bank Inc. v. De Guzman et. al., November 13, 2013 621
Lara’s Gift & Decors v. Midtown Industrial Sales, August 28, 2019 626
Session Delights v. CA, February 8, 2010 629
United Coconut Chemicals v. Almores, July 12, 2017 632
Universal Robina Corp. v. Castillo, July 10, 2013. 636
Baptista v. Villanueva, July 31, 2013. 639
BPI Employees Union Davao City v. BPI , July 24, 2013. 643
Integrated Microelectronics v. Pionelles, August 28, 2013. 647

PALOMAR, Andrei Anne U.


PAGE
CASE TITLE
NO.

Golden Ace Builders v. Talde, May 5, 2010. 649


Metroguards Security Agency Corp. v. Hilongo, March 9, 2015. 651
Maersk-Filipinas Crewing Inc. v. Avestruz, February 18, 2015 653
Villena v. Batangas II Electric, February 4, 2015. 655
Sangwoo Phils. v. SangWoo Philippines Employees Union, 658
Papertech v. Kando, January 8, 2020 660
Olympia Housing v. Pastora, January 13, 2016 662
Claudia’s Kitchen v. Tanguin, June 28, 2017 664
Dumapis v. Lepanto, September 15, 2020 667
Lynvil Fishing Enterprises Inc. v. Ariola, February 1, 2012 670
Sonza v. ABS CBN Broadcasting corp, June 10, 2004 673
Consolidated Broadcasting Systems Inc. v. Oberio June 8, 2007 676
Orozco v. CA, August 13, 2008 678
William Uy Construction Corp. v. Trinidad, March 10, 2010 680
PARIL, Joshua F.
PAGE
CASE TITLE
NO.

DM Consunji Inc. v. Jamin April 18, 2012 682


Aro et. al. v. NLRC March 7, 2012 684
Universal Robina Sugar Milling Corp v. Acibo, January 15, 2014 687
GMA Network Inc. v. Pabriga, November 27, 2013. 689
Pasas v. PNCC, July 3, 2013. 692
Gapayao v. Fulo, June 13, 2013 694
Millennium Erectors Corp. v. Magallanes, November 15, 2010 696
Caparoso et. al. v. CA, February 15, 2007 698
Spouses Lim v. Legaspi Hope Christian School, March 31, 2009 700
DM Consunji v. Gobres et. al, August 8, 2010 702
Mercado et. al. v. Ama Computer College, April 13, 2010 705
Brent School v. Zamora 708
Pure Foods Corporation v. NLRC December 12, 1997 711
Leyte Geothermal Power v. PNOC, March 30, 2011 714

RAFAEL, John Ranier A.


PAGE
CASE TITLE
NO.

Salazar v. NLRC (1996 CASE) 716


Fonterra Brands Phil v. Lagardo, March 18, 2015 717
Basan v. Coca Cola Bottles, February 4, 2015. 720
Convoy Marketing Corp. v. Albia, October 7, 2015. 722
Jamias v. NLRC, March 9, 2016 724
Gadia v. Sykes Asia, January 28, 2015 726
Innodata Knowledge Services v. Inting, December 6, 2017 728
Pacific Metal Case, December 5, 2019 730
Claret School of QC v. Sinday, October 9, 2019 732
Gapayao v. Fulo, June 13, 2013 735
Paz v. Northern Tobacco Redrying Co. 738
Kimberly Independent Labor Union v. Drilon, May 9, 1990 741
Philippine Geothermal Inc. v. NLRC, August 30, 1990 744
Samonte v. La Salle Greenhills, February 10, 2016 746

ROSALES, Mikhaila Klaudine A.


PAGE
CASE TITLE
NO.

St. Luke’s Medical Center v. Sanchez, March 11, 2015 749


Philippine Span Asia Carriers v. Pelayo, February 28, 2018 751
Zuellig Freight and Cargo System v. NLRC, July 22, 2013. 753
Peckson v. Robinsons Supermarket Corp. , July 3, 2013. 755
Gatbonton v. NLRC, January 23, 2006 757
Automatic Appliances v. Deguidez, December 4, 2019 759
Telus case, December 4, 2019 761
Puncia v. Toyota Shaw, June 28, 2016 763
Duncan Association of Detailman-PTGWO v. Glaxo Wellcome,
765
September 17, 2004
Tiu v. Platinum Plans, February 28, 2007 767
The Phil. Geothermal Inc v. Unocal Phil, September 28, 2016 769
SME Bank v. Peregrin, October 8, 2013 771
Tamson’s Enterprises Inc.et. al. v. CA, March 16, 2011 773
Hacienda Primera Development v. Villegas, April 11, 2011 775

ROSARIO, Lex Angelo A.


PAGE
CASE TITLE
NO.
Universidad De Sta. Isabel v. Sambajon, April 2, 2014 777
Univac Development v. Soriano, June 19, 2013. 781
Abbott Laboratories v. Alcaraz, July 23, 2013. 784
Colegio de Santisimo Rosario v. Rojo, September 4, 2013 788
Phil. Daily Inquirer v. Magtibay, July 24, 2007 791
Alcira v. NLRC, June 9, 2004 794
Mercado v. Ama Computer College 797
Oyster Plaza Hotel v. Melivo, October 5, 2016 801
Imasen Philippine Corporation v. Alcon, 22 October 2014 803
Northwest Airlines v. Del Rosario, 10 September 2014 806
Citibank v. NLRC, 6 February 2008 808
Mirant Philippines v. Caro, 23 April 2014 810
Nacague v. Sulpicio Lines, 9 August 2010 813
Leus v. St. Scholastica’s College, 28 January 2015 815

TANGHAL, Juan Miguel P.


PAGE
CASE TITLE
NO.
Cadiz v. Brent Hosptial, February 24, 2016 818
Sterling Paper Products v. KMM-Katipunan, August 2, 2017 822
Maribago Resort v. Dual, July 20, 2010 824
Benitez v. Santa Fe Moving and Relocation, April 20, 2015. 826
Domingo v. Rayala 2008 case 829
Coca-Cola Export Corporation v. Gacayan, December 15, 2010 831
Holcim Phil. v. Obna, August 8, 2016 835
Waterfront Cebu City Casino Hotel v. Ledesma, March 25, 2015 838
Naguit v. San Miguel, June 22, 2015 840
Adamson University Faculty v. Adamson, March 9, 2020 842
Coffee Bean and Tea Leaf v. Arenas, 11 March 2015 844
Lores Realty Enterprises v. Pacia, 9 March 847
ePacific Global Contact Center v. Cabansay, 23 November 2007 850
Tongko v. The Manufacturer’s Life Insurance, November 7, 2008 854

TORRES, Paula Gail I.


PAGE
CASE TITLE
NO.
Pharmacia and Upjohn Inc. v. Albayda Jr., August 23, 2010 857
St. Luke’s Medical Center v. Sanchez, March 11, 2015 859
St. Luke’s Medical Center v. Notario, 20 October 2010 861
LBC Express v. Mateo, 9 June 2009 863
Mansion Printing Center v. Bitara, 15 January 2012 865
Cavite Apparel v. Marquez, 6 February 2013 867
Manarpiis v. Texan Philippines, 28 January 869
School of Holy Spirit of Quezon City v. Taguiam, July 14, 2008 871
Philippine Airlines v. NLRC, 16 March 2000 873
Sta. Ana v. Manila Jockey Club, 15 February 2017 875
Hormillosa v. Coca-Cola Bottlers Philippines, 9 October 877
Manese v. Jollibee Foods Corporation, 11 October 2012 879
Grand Asian Shipping Lines Inc., v. Galvez, January 29, 2014 881
Bluer than blue Joint Ventures Co. v. Esteban, April 7, 2014. 883

VERGARA, Jean Colleen M.


PAGE
CASE TITLE
NO.

Concepcion v. Minex Import Corporation, January 24, 2012 885


Manila Jockey Club v. Trajano, June 26, 2013. 887
Reno v. Nagkakaisang Lakas ng Manggagawa, March 2010 889
Lynvil Fishing Enterprises v. Ariola, 1 February 2012 891
John Hancock Life Insurance v. Davis, 3 September 2008 893
Yrasuegui v. PAL, October 17, 2008 895
Son et al. v. UST, April 18, 2018 (other causes) 897
Mallo v. Southeast Asian College, October 14, 2015 899
Manarpiis v. Texas Phils, January 28, 2015 901
Arabit v. Jardine Pacific, 21 April 2014 903
Shimizu Philippines v. Callanta, 29 September 2010 906
SPI Technologies Inc. v. Mapua, April 7, 2014 908
Culili v. Eastern Telecommunication, February 9, 2011 910
Asufrin v. San Miguel Corporation, March 10, 2004 912

4G – BACOLOD, Jamielou
PAGE
CASE TITLE
NO.
Ocean East Agency Corp v. Lopez, October 14, 2015 914
Asian Alcohol Corporation v. NLRC, 25 March 1999 917
FASAP v. Philippine Airlines, 13 March 2018 920
Sebuguero v. NLRC, 27 September 1995 922
Blue Eagle v. Naval, April 19, 2016 925
La Consolacion College v. Pascua, March 14, 2018 928
Sangwoo v. Sangwoo Philippines Employees, 9 December 2013 930
Navotas Shipyard v. Montallana, 24 March 2014 933
Veterans Federation. v. Montenegro, November 29, 2017 935
PNCC Skyway Corp. v. Secretary of Labor, February 6, 2017 937
Penafrancia Tours v. Sarmiento October 20, 2010 939
Sy et. al. v. CA, February 27, 2003 941
Union Motors Corp v. NLRC, December 9, 2004 944
Villaruel v. Yeo Han Guan, June 1, 2011 946

BRODIT, Vyel Marie C.


PAGE
CASE TITLE
NO.
Deoferio v. Intel Technology Philippines, June 18, 2014
Malig-on v. Equitable General Services Inc. June 29, 2010 948
Chang Kai Shek College v. Torres, April 2, 2014. 950
Opinaldo v. Ravina, October 16, 2013. 952
Willi Hahn Enterprises v. Maghuyop, December 17, 2004 955
Skippers United Pacific et. al. v. Doza et. al., February 8, 2012 957
Morales v. Harbour Centre Port Terminal Inc., January 25, 2012 960
SHS Perforated Material Inc. v. Diaz, October 13, 2010 963
San Miguel Properties Phils. v. Gucaban, July 18, 2011 967
BMG Records Phils. Inc. v. Aparecio, September 5, 2007 970
Tatel v. JLFP Investigations, February 25, 2015 974
Paredes v. Feed the Children Phils., September 9, 2015 977
Silvertex Weaving Corp. v. Campo, Mach 16, 2016 980
Divine World College of Laoag v. Mina, April 13, 2016 982
CALVO, Krissy Angela A.
PAGE
CASE TITLE
NO.

Cokia Industries v. Beatriz Bug-os, November 27, 2019 985


Kondo v. Toyota, September 11, 2019 987
LBC v. Palco, February 12, 2020 990
SKM Article Craft Corporation v. Bauca, November 27, 2013 993
Emeritus Security and Maintenance v. Dailig, April 2, 2014. 995
Nippon Housing Phil. Inc. et. al v. Leynes, August 3, 2011 996
Mayon Hotel and Restaurant et. al. v. Adana May 16, 2005 999
ICT Marketing v. Sales, September 9, 2015 1001
Carique v. Phil. Scout Veterans Security September 16, 2015 1004
Daaboy v. Coca-Cola , August 19, 2013. 1006
Pinero v. NLRC, August 20, 2004 1008
Sta. Catalina College v. NLRC, November 19, 2003 1010
Pantranco North Express Inc. v. NLRC, July 24, 1996 1012
R and E Transport v. Latag, February 13, 2004 1014

CERERA, Florence Diane N.


PAGE
CASE TITLE
NO.

Serrano v. Severino Santos Transit, August 9, 2010 1016


Obusan v. PNB, July 26, 2010 1020
Kimberly Clark Phils. v. Dimayuga, September 18, 2009 1023
Magdadaro v. PNB, July 17, 2009. 1028
Laya v. CA, January 10, 2018. 1030
Tolentino v. PAL, January 24, 2018 1034
De La Salle Araneta University v. Bernardo, February 13, 2017 1037
UDMC v. Bernadas, December 13, 2017 1040
PAL v. Hassaram, June 5, 2017 1044
Carissa E. Santo v. University of Cebu, August 28, 2019 1048
Padillo v. Rural Bank of Nabunturan, January 21, 2013 1051
Philippine Journalist Inc. v. De Guzman et. al., April 1, 2019 1054
PLDT v. Pingol, September 10, 2008 1057
Serrano v. CA, August 15, 2001 1060
JAZMIN, Jendy F.
PAGE
CASE TITLE
NO.

IBC v. Panganiban, February 6, 2007 1063


Accessories Specialist v. Albanza July 23, 2008 1065
Autobus Transport System v. Bautista May 16, 2007 1067
Montero v. Times Transport, March 16, 2015 1070
Callanta v. Carnation Phils., October 28, 1986 1072
Arriola v. Pilipino Star Ngayon, August 13, 2014 1074
Caladin v. POEA, December 5, 1994 1076
Page 1

Case Digests by: MARIELLE B. ANTONIO

CITIBANK V. CA
G.R. No.108961, November 27, 1998
Pardo, J.

DOCTRINE:
DEFINITION OF LABOR DISPUTE
Labor dispute "includes any controversy or matter concerning
terms or conditions of employment or the association or representation
of persons in negotiating, fixing, maintaining, changing or arranging the
terms and conditions of employment, regardless of whether the
disputants stand in the proximate relation of employer and employee.

FACTS:
Citibank and El Toro Security Agency entered into a contract in
which El Toro obligated itself to provide the services of security guards
to safeguard and protect the premises and property of Citibank against
theft, robbery or any other unlawful acts committed by any person or
persons, and assumed responsibility for losses and/or damages that
may be incurred by Citibank due to or as a result of the negligence of
El Toro or any of its assigned personnel.

After the contract expired, Citibank hired another security agency,


the Golden Pyramid Security Agency, to render security services at
Citibank's premises. Security guards of El Toro who were replaced by
guards of the Golden Pyramid Security Agency considered the non-
renewal of El Toro's service agreement with Citibank as constituting a
lockout and/or a mass dismissal. They threatened to go on strike
against Citibank and picket its premises. In fact, security guards
formerly assigned to Citibank under the expired agreement loitered
around and near the Citibank premises in large groups of from twenty
(20) and at times fifty (50) persons.

Citibank filed for injunction and damages before the RTC.


Petitioner Citibank contends that there is no employer-employee
relationship between Citibank and the security guards represented by
respondent CIGLA and that there is no "labor dispute" in the subject
controversy. The security guards were employees of El Toro security
agency, not of Citibank. Its service contract with Citibank had expired
and not renewed.

ISSUE:
Whether there is a labor dispute.

1
Page 2

RULING:
NO. In this case, it was the security agency El Toro that recruited,
hired and assigned the watchmen to their place of work. It was the
security agency that was answerable to Citibank for the conduct of its
guards. Article 212, paragraph 1 of the Labor Code provides the
definition of a "labor dispute". It "includes any controversy or matter
concerning terms or conditions of employment or the association or
representation of persons in negotiating, fixing, maintaining, changing
or arranging the terms and conditions of employment, regardless of
whether the disputants stand in the proximate relation of employer and
employee.

If at all, the dispute between Citibank and El Toro security agency


is one regarding the termination or non-renewal of the contract of
services. This is a civil dispute. El Toro was an independent contractor.
Thus, no employer-employee relationship existed between Citibank
and the security guard members of the union in the security agency
who were assigned to secure the bank's premises and property.
Hence, there was no labor dispute and no right to strike against the
bank.

2
Page 3

Case Digests by: MARIELLE B. ANTONIO

PAL v. NLRC
G.R. No.120567, March 20, 1998
Martinez, J.

DOCTRINE:
DEFINITION OF LABOR DISPUTE
The term "labor dispute" is defined as "any controversy or matter
concerning terms and conditions of employment or the association or
representation of persons in negotiating, fixing, maintaining, changing,
or arranging the terms and conditions of employment regardless of
whether or not the disputants stand in the proximate relation of
employers and employees."

The term "controversy" is likewise defined as "a litigated question;


adversary proceeding in a court of law; a civil action or suit, either at
law or in equity; a justiciable dispute."

FACTS:
Private respondents are flight stewards of the petitioner. Both were
dismissed from the service for their alleged involvement in the April 3,
1993 currency smuggling in Hong Kong. Aggrieved by said dismissal,
private respondents filed with the NLRC a petition for injunction and
damages.

ISSUE:
Whether there is a labor dispute.

RULING:
NO. In labor cases, Article 218 of the Labor Code empowers the
NLRC-(e) To enjoin or restrain any actual or threatened commission of
any or all prohibited or unlawful acts or to require the performance of a
particular act in any labor dispute which, if not restrained or performed
forthwith, may cause grave or irreparable damage to any party or
render ineffectual any decision in favor of such party; Complementing
the quoted provision, Sec. 1, Rule XI of the New Rules of Procedure of
the NLRC, pertinently provides as follows:"Section 1. Injunction in
Ordinary Labor Dispute.-A preliminary injunction or a restraining order
may be granted by the Commission through its divisions pursuant to
the provisions of paragraph (e) of Article 218 of the Labor Code, as
amended, when it is established on the bases of the sworn allegations
in the petition that the acts complained of, involving or arising from any
labor dispute before the Commission, which, if not restrained or

3
Page 4

performed forthwith, may cause grave or irreparable damage to any


party or render ineffectual any decision in favor of such party.

The foregoing ancillary power may be exercised by the Labor


Arbiters only as an incident to the cases pending before them in order
to preserve the rights of the parties during the pendency of the case,
but excluding labor disputes involving strikes or lockout.

From the foregoing provisions of law, the power of the NLRC to


issue an injunctive writ originates from "any labor dispute" upon
application by a party thereof, which application if not granted "may
cause grave or irreparable damage to any party or render ineffectual
any decision in favor of such party."

It is an essential requirement that there must first be a labor


dispute between the contending parties before the labor arbiter. In the
present case, there is no labor dispute between the petitioner and
private respondents as there has yet been no complaint for illegal
dismissal filed with the labor arbiter by the private respondents against
the petitioner.

The petition for injunction directly filed before the NLRC is in reality
an action for illegal dismissal. This is clear from the allegations in the
petition which prays for: reinstatement of private respondents; award
of full backwages, moral and exemplary damages; and attorney's fees.
As such, the petition should have been filed with the labor arbiter who
has the original and exclusive jurisdiction to hear and decide the cases
involving all workers, whether agricultural or non-agricultural:

The jurisdiction conferred by the foregoing legal provision to the


labor arbiter is both original and exclusive, meaning, no other officer or
tribunal can take cognizance of, hear and decide any of the cases
therein enumerated. The only exceptions are where the Secretary of
Labor and Employment or the NLRC exercises the power of
compulsory arbitration, or the parties agree to submit the matter to
voluntary arbitration pursuant to Article 263 (g) of the Labor Code, the
pertinent portions of which reads:(g) When, in his opinion, there exists
a labor dispute causing or likely to cause a strike or lockout in an
industry indispensable to the national interest, the Secretary of Labor
and Employment may assume jurisdiction over the dispute and decide
it or certify the same to the Commission for compulsory arbitration.
Such assumption or certification shall have the effect of automatically
enjoining the intended or impending strike or lockout as specified in the
assumption or certification order. If one has already taken place at the
time of assumption or certification, all striking or locked out employees
shall immediately resume operations and readmit all workers under the

4
Page 5

same terms and conditions prevailing before the strike or lockout. The
Secretary of Labor and Employment or the Commission may seek the
assistance of law enforcement agencies to ensure compliance with this
provision as well as with such orders as he may issue to enforce the
same.

On the other hand, the NLRC shall have exclusive appellate


jurisdiction over all cases decided by labor arbiters as provided in
Article 217(b) of the Labor Code. In short, the jurisdiction of the NLRC
in illegal dismissal cases is appellate in nature and, therefore, it cannot
entertain the private respondents' petition for injunction which
challenges the dismissal orders of petitioner. Article 218(e) of the
Labor Code does not provide blanket authority to the NLRC or any of
its divisions to issue writs of injunction, considering that Section 1 of
Rule XI of the New Rules of Procedure of the NLRC makes injunction
only an ancillary remedy in ordinary labor disputes"

Thus, the NLRC exceeded its jurisdiction when it issued the


assailed Order granting private respondents' petition for injunction and
ordering the petitioner to reinstate private respondents. The argument
of the NLRC in its assailed Order that to file an illegal dismissal suit
with the labor arbiter is not an "adequate" remedy since it takes three
(3) years before it can be disposed of, is patently erroneous. An
"adequate" remedy at law has been defined as one "that affords relief
with reference to the matter in controversy, and which is appropriate to
the particular circumstances of the case." It is a remedy which is
equally beneficial, speedy and sufficient which will promptly relieve the
petitioner from the injurious effects of the acts complained of.

Under the Labor Code, the ordinary and proper recourse of an


illegally dismissed employee is to file a complaint for illegal dismissal
with the labor arbiter. In the case at bar, private respondents
disregarded this rule and directly went to the NLRC through a petition
for injunction praying that petitioner be enjoined from enforcing its
dismissal orders. Moreover, the preliminary mandatory injunction
prayed for by the private respondents in their petition before the NLRC
can also be entertained by the labor arbiter who, as shown earlier, has
the ancillary power to issue preliminary injunctions or restraining orders
as an incident in the cases pending before him in order to preserve the
rights of the parties during the pendency of the case.

Furthermore, an examination of private respondents' petition for


injunction reveals that it has no basis since there is no showing of any
urgency or irreparable injury which the private respondents might
suffer.

5
Page 6

Here, the alleged injury which private respondents stand to suffer


by reason of their alleged illegal dismissal can be adequately
compensated and therefore, there exists no "irreparable injury," as
defined above which would necessitate the issuance of the injunction
sought for. Article 279 of the Labor Code provides that an employee
who is unjustly dismissed from employment shall be entitled to
reinstatement, without loss of seniority rights and other privileges, and
to the payment of full backwages, inclusive of allowances, and to other
benefits or their monetary equivalent computed from the time his
compensation was withheld from him up to the time of his actual
reinstatement.

Finally, an injunction, as an extraordinary remedy, is not favored


in labor law considering that it generally has not proved to be an
effective means of settling labor disputes. It has been the policy of the
State to encourage the parties to use the non-judicial process of
negotiation and compromise, mediation and arbitration. Thus,
injunctions may be issued only in cases of extreme necessity based on
legal grounds clearly established, after due consultations or hearing
and when all efforts at conciliation are exhausted which factors,
however, are clearly absent in the present case.

6
Page 7

Case Digests by: MARIELLE B. ANTONIO

SAN MIGUEL CORPORATION EMPLOYEES UNION v.


BERSAMIRA
G.R. No. 87700, June 13, 1990
Melencio-Herrera, J.

DOCTRINE:
DEFINITION OF LABOR DISPUTE
A "labor dispute" as defined in Article 212 (1) of the Labor Code
includes "any controversy or matter concerning terms and conditions
of employment or the association or representation of persons in
negotiating, fixing, maintaining, changing, or arranging the terms and
conditions of employment, regardless of whether the disputants stand
in the proximate relation of employer and employee."

FACTS:
SanMig entered into contracts with licensed independent
contractors (Lipercon and D'Rite) to maintain its competitive position
and in keeping with the imperatives of efficiency, business expansion
and diversity of its operation. In said contracts, it was expressly
understood and agreed that the workers employed by the contractors
were to be paid by the latter and that none of them were to be deemed
employees or agents of SanMig. There was to be no employer-
employee relation between the contractors and/or its workers, on the
one hand, and SanMig on the other.

The Union advised SanMig that some Lipercon and D'Rite workers
had signed up for union membership and sought the regularization of
their employment with SMC. The Union alleged that this group of
employees, while appearing to be contractual workers supposedly
independent contractors, have been continuously working for SanMig
for a period ranging from six (6) months to fifteen (15) years and that
their work is neither casual nor seasonal as they are performing work
or activities necessary or desirable in the usual business or trade of
SanMig. Thus, it was contended that there exists a "labor-only"
contracting situation. It was then demanded that the employment
status of these workers be regularized. Subsequently, the Union filed
a notice of strike. Series of pickets were staged by Lipercon and D'Rite
workers in various SMC plants and offices.

After several hearings on SanMig's application for injunctive relief,


where the parties presented both testimonial and documentary
evidence on 25 March 1989, respondent Court issued the questioned
Order stating that absence of employer-employee relationship negates

7
Page 8

the existence of labor dispute. Verily, this court has jurisdiction to take
cognizance of plaintiff's grievance. The evidence so far presented
indicates that plaintiff has contracts for services with Lipercon and
D'Rite. The application and contract for employment of the defendants'
witnesses are either with Lipercon or D'Rite. What could be discerned
is that there is no employer-employee relationship between plaintiff
and the contractual workers employed by Lipercon and D'Rite. This,
however, does not mean that a final determination regarding the
question of the existence of employer-employee relationship has
already been made. To finally resolve this dispute, the court must
extensively consider and delve into the manner of selection and
engagement of the putative employee; the mode of payment of wages;
the presence or absence of a power of dismissal; and the Presence or
absence of a power to control the putative employee's conduct. This
necessitates a full-blown trial. If the acts complained of are not
restrained, plaintiff would, undoubtedly, suffer irreparable damages.
Upon the other hand, a writ of injunction does not necessarily expose
defendants to irreparable damages.

ISSUE:
Whether there is a labor dispute.

RULING:
YES. While it is SanMig's submission that no employer-employee
relationship exists between itself, on the one hand, and the contractual
workers of Lipercon and D'Rite on the other, a labor dispute can
nevertheless exist "regardless of whether the disputants stand in the
proximate relationship of employer and employee" provided the
controversy concerns, among others, the terms and conditions of
employment or a "change" or "arrangement" thereof. Put differently,
and as defined by law, the existence of a labor dispute is not negative
by the fact that the plaintiffs and defendants do not stand in the
proximate relation of employer and employee.

That a labor dispute, as defined by the law, does exist herein is


evident. At bottom, what the Union seeks is to regularize the status of
the employees contracted by Lipercon and D'Rite in effect, that they
be absorbed into the working unit of SanMig. This matter definitely
dwells on the working relationship between said employees vis-a-vis
SanMig. Terms, tenure and conditions of their employment and the
arrangement of those terms are thus involved bringing the matter
within the purview of a labor dispute. Further, the Union also seeks to
represent those workers, who have signed up for Union membership,
for the purpose of collective bargaining. SanMig, for its part, resists that
Union demand on the ground that there is no employer-employee
relationship between it and those workers and because the demand
violates the terms of their CBA. Obvious then is that representation and

8
Page 9

association, for the purpose of negotiating the conditions of


employment are also involved. In fact, the injunction sought by SanMig
was precisely also to prevent such representation. Again, the matter of
representation falls within the scope of a labor dispute. Neither can it
be denied that the controversy below is directly connected with the
labor dispute already taken cognizance of by the NCMB-DOLE.

The precedent in Layno vs. de la Cruz (G.R. No. L-29636, 30 April


1965, 13 SCRA 738) relied upon by SanMig is not controlling as in that
case there was no controversy over terms, tenure or conditions, of
employment or the representation of employees that called for the
application of labor laws. In that case, what the petitioning union
demanded was not a change in working terms and conditions, or the
representation of the employees, but that its members be hired as
stevedores in the place of the members of a rival union, which
petitioners wanted discharged notwithstanding the existing contract of
the arrastre company with the latter union. Hence, the ruling therein,
on the basis of those facts unique to that case, that such a demand
could hardly be considered a labor dispute.

As the case is indisputably linked with a labor dispute, jurisdiction


belongs to the labor tribunals. As explicitly provided for in Article 217
of the Labor Code, prior to its amendment by R.A. No. 6715 on 21
March 1989, since the suit below was instituted on 6 March 1989,
Labor Arbiters have original and exclusive jurisdiction to hear and
decide the following cases involving all workers including "1. unfair
labor practice cases; 2. those that workers may file involving wages,
hours of work and other terms and conditions of employment; ... and
5. cases arising from any violation of Article 265 of this Code, including
questions involving the legality of strikes and lockouts. ..." Article 217
lays down the plain command of the law.

The claim of SanMig that the action below is for damages under
Articles 19, 20 and 21 of the Civil Code would not suffice to keep the
case within the jurisdictional boundaries of regular Courts. That claim
for damages is interwoven with a labor dispute existing between the
parties and would have to be ventilated before the administrative
machinery established for the expeditious settlement of those disputes.
To allow the action filed below to prosper would bring about "split
jurisdiction" which is obnoxious to the orderly administration of justice
(Philippine Communications, Electronics and Electricity Workers
Federation vs. Hon. Nolasco, L-24984, 29 July 1968, 24 SCRA 321).

9
Page 10

Case Digests by: MARIELLE B. ANTONIO

PENARANDA v. BAGONG PLYWOOD CORP.


G.R. No. 159577, May 3, 2006
Panganiban, C.J.

DOCTRINE:
MANAGERIAL EMPLOYEE
Managerial employees and members of the managerial staff are
exempted from the provisions of the Labor Code on labor standards.
Since petitioner belongs to this class of employees, he is not entitled
to overtime pay and premium pay for working on rest days.

FACTS:
Petitioner Charlito Peñaranda was hired as an employee of
Baganga Plywood Corporation (BPC) to take charge of the operations
and maintenance of its steam plant boiler. Eventually, Peñaranda filed
a Complaint for illegal dismissal with money claims against BPC and
its general manager before the NLRC. He alleges that his services
were terminated without the benefit of due process and valid grounds
in accordance with law. Furthermore, he was not paid his overtime pay,
premium pay for working during holidays/rest days, night shift
differentials and finally claims for payment of damages and attorney’s
fees having been forced to litigate the present complaint.

Respondents allege that complainant’s separation from service


was done pursuant to Art. 283 of the Labor Code. The respondent was
on temporary closure due to repair and general maintenance and it
applied for clearance with the DOLE to shut down and to dismiss
employees, and due to the insistence of herein complainant he was
paid his separation benefits. Consequently, when respondent partially
reopened in January 2001, Peñaranda failed to reapply. Hence, he
was not terminated from employment much less illegally. He opted to
severe employment when he insisted payment of his separation
benefits. Furthermore, being a managerial employee, he is not entitled
to overtime pay and if ever he rendered services beyond the normal
hours of work, there was no office order/or authorization for him to do
so.

The labor arbiter ruled that there was no illegal dismissal, and that
petitioner is entitled to overtime pay, premium pay for working on rest
days, and attorney’s fees. The NLRC ruled that petitioner was not
entitled to these awards because he was a managerial employee. The
CA dismissed the Petition.

10
Page 11

ISSUE:
Whether petitioner is a managerial employee.

RULING:
NO, he is not a managerial employee but a member of the
managerial staff, which also takes him out of the coverage of labor
standards. Like managerial employees, officers and members of the
managerial staff are not entitled to the provisions of law on labor
standards. The Implementing Rules of the Labor Code define
members of a managerial staff as those with the following duties and
responsibilities:(1) The primary duty consists of the performance of
work directly related to management policies of the employer; (2)
Customarily and regularly exercise discretion and independent
judgment;(3) (i) Regularly and directly assist a proprietor or a
managerial employee whose primary duty consists of the management
of the establishment in which he is employed or subdivision thereof; or
(ii) execute under general supervision work along specialized or
technical lines requiring special training, experience, or knowledge; or
(iii) execute under general supervision special assignments and tasks;
and(4) who do not devote more than 20 percent of their hours worked
in a workweek to activities which are not directly and closely related to
the performance of the work described in paragraphs (1), (2), and (3)
above."

The following duties and responsibilities illustrate that petitioner


was a member of the managerial staff: 1.To supply the required and
continuous steam to all consuming units at minimum cost; 2. To
supervise, check and monitor manpower workmanship as well as
operation of boiler and accessories; 3. To evaluate performance of
machinery and manpower; 4. To follow-up supply of waste and other
materials for fuel. 5. To train new employees for effective and safety
while working; and To recommend personnel actions such as:
promotion, or disciplinary action. His duties and responsibilities
conform to the definition of a member of a managerial staff under the
Implementing Rules.

Petitioner supervised the engineering section of the steam plant


boiler. His work involved overseeing the operation of the machines and
the performance of the workers in the engineering section. This work
necessarily required the use of discretion and independent judgment
to ensure the proper functioning of the steam plant boiler. As
supervisor, petitioner is deemed a member of the managerial staff.

His classification as supervisor is further evident from the manner


his salary was paid. He belonged to the 10% of respondent’s 354
employees who were paid on a monthly basis; the others were paid
only on a daily basis.

11
Page 12

Case Digests by: MARIELLE B. ANTONIO

PONDOC v. NLRC
G.R. No.116347, October 3, 1996
Davide, Jr., J.

DOCTRINE:
JURISDICTION OF LABOR ARBITER
Under paragraph (b) of Article 217, the NLRC has exclusive
appellate jurisdiction over all cases decided by the Labor Arbiters. This
simply means that the NLRC does not have original jurisdiction over
the cases enumerated in paragraph (a) and that if a claim does not fall
within the exclusive original jurisdiction of the Labor Arbiter, the NLRC
cannot have appellate jurisdiction thereon.

FACTS:
Private respondent Eulalio Pondoc is the owner-proprietor of
Melleonor General Merchandise and Hardware Supply. Respondent is
engaged, among others, in the business of buying and selling copra,
rice, corn, "binangkol," junk iron and empty bottles. He has in his
employ more than twenty (20) regular workers. Andres Pondoc was
employed by Eulalio Pondoc as a laborer from October 1990 up to
December 1991, receiving a wage rate of P20.00 per day. He was
required to work twelve (12) hours a day from 7:00 AM to 8:00 PM,
Monday to Sunday. Despite working on his rest days and holidays, he
was not paid his premium pay as required by law.

Consequently, on May 14, 1992, Natividad Pondoc, on behalf of


her husband, filed a complaint for salary differential, overtime pay, 13th
month pay, holiday pay and other money claims before the NLRC.
Private respondent questioned, among others, the existence of
employer-employee relationship between them. He further averred
that Melleonor General Merchandise and Hardware Supply is a
fictitious establishment.

Labor Arbiter rendered a Decision finding the existence of


employer-employee relationship between the parties. On his last day
to perfect an appeal, private respondent filed a Manifestation before
the Labor Arbiter praying that his liabilities be set-off against
petitioner’s alleged indebtedness to him which the LA denied but the
NLRC granted and approved respondent’s independent petition for
injunction and damages.

ISSUE:

12
Page 13

Whether the NLRC has jurisdiction over the injunction filed.

RULING:
NO. The proceedings before the NLRC were fatally flawed. In the
first place, the NLRC should not have entertained the private
respondent’s separate or independent petition for Injunction and
Damages. It was obvious that the petition was a scheme to defeat or
obstruct the enforcement of the judgment where, in fact, a writ of
execution had been issued. Article 218(e) of the Labor Code does not
provide blanket authority to the NLRC or any of its divisions to issue
writs of injunction, while Rule XI of the New Rules of Procedure of the
NLRC makes injunction only an ancillary remedy in ordinary labor
disputes such as the one brought by the petitioner. This is clear from
Section 1 of the said Rule which pertinently provides as follows:
Section 1. Injunction in Ordinary Labor Disputed. — A preliminary
injunction or a restraining order may be granted by the Commission
through its divisions pursuant to the provisions of paragraph (e) of
Article 218 of the Labor Code, as amended, when it is established on
the bases of the sworn allegations in the petition that the acts
complained of, involving or arising from any labor dispute before the
Commission, which, if not restrained or performed forthwith, may
cause grave or irreparable damage to any party or render ineffectual
any decision in favor of such party.

The foregoing ancillary power may be exercised by the Labor


Arbiters only as an incident to the cases pending before them in order
to preserve the rights of the parties during the pendency of the case,
but excluding labor disputes involving strike or lockout.

Secondly, the appeal of the private respondent in NLRC was not


from the decision therein, but from the order of the Labor Arbiter
denying the set-off insisted upon by the private respondent and
directing the execution of the judgment. Therefore, the private
respondent admitted the final and executory character of the judgment.

The Labor Arbiter, in denying the set-off, reasoned "It could have
been considered if it was presented before the decision of this case."
While this is correct, there are stronger reasons why the set-off should,
indeed, be denied. As correctly contended by the Office of the Solicitor
General, there is a complete want of evidence that the indebtedness
asserted by the private respondent against Andres Pondoc arose out
of or was incurred in connection with the employer-employee
relationship between them. The Labor Arbiter did not then have
jurisdiction over the claim as under paragraph (a) of Article 217 of the
Labor Code.

13
Page 14

On the other hand, under paragraph (b) thereof, the NLRC has
exclusive appellate jurisdiction over all cases decided by the Labor
Arbiters. This simply means that the NLRC does not have original
jurisdiction over the cases enumerated in paragraph (a) and that if a
claim does not fall within the exclusive original jurisdiction of the Labor
Arbiter, the NLRC cannot have appellate jurisdiction thereon.

The conclusion then is inevitable that the NLRC was without


jurisdiction, either original or appellate, to receive evidence on the
alleged indebtedness, render judgment thereon, and direct that its
award be set-off against the final judgment of the Labor Arbiter.

Finally, even assuming arguendo that the claim for the alleged
indebtedness fell within the exclusive original jurisdiction of the Labor
Arbiter, it was deemed waived for not having been pleaded as an
affirmative defense or barred for not having been set up as a
counterclaim before the Labor Arbiter at any appropriate time prior to
the rendition of the decision. Under the Rules of Court, which is
applicable in a suppletory character in labor cases before the Labor
Arbiters or the NLRC pursuant to Section 3, Rule I of the New Rules of
Procedure of the NLRC, defenses which are not raised either in a
motion to dismiss or in the answer are deemed waived and
counterclaims not set up in the answer are barred. Set-off or
compensation is one of the modes of extinguishing obligations and
extinguishment is an affirmative defense and a ground for a motion to
dismiss.

The NLRC acted without jurisdiction or with grave abuse of


discretion in entertaining an independent action for injunction and in
receiving evidence and rendering judgment on the alleged
indebtedness of Andres Pondoc, and in ordering such judgment to
offset the final award of the labor Arbiter.

14
Page 15

Case Digests by: MARIELLE B. ANTONIO

VILLAMARIA v. CA
G.R. No.165881, April 19, 2006
Callejo, Sr., J.

DOCTRINE:
JURISDICTION OF LABOR ARBITER
The jurisdiction of Labor Arbiters and the NLRC under Article 217
of the Labor Code is limited to disputes arising from an employer-
employee relationship which can only be resolved by reference to the
Labor Code, other labor statutes or their collective bargaining
agreement.

FACTS:
Petitioner is the owner of Villarama Motors. Villamaria verbally
agreed to sell a jeepney to Bustamante under the "boundary-hulog
scheme," where Bustamante would remit to Villarama P550.00 a day
for a period of four years; Bustamante would then become the owner
of the vehicle and continue to drive the same under Villamaria’s
franchise. It was also agreed that Bustamante would make a
downpayment of P10,000.00. Villamaria executed a contract entitled
"Kasunduan ng Bilihan ng Sasakyan sa Pamamagitan ng Boundary-
Hulog". The parties agreed that if Bustamante failed to pay the
boundary-hulog for three days, Villamaria Motors would hold on to the
vehicle until Bustamante paid his arrears, including a penalty of P50.00
a day; in case Bustamante failed to remit the daily boundary-hulog for
a period of one week, the Kasunduan would cease to have legal effect
and Bustamante would have to return the vehicle to Villamaria Motors.
Certain conditions were imposed on Bustamante under the
Kasunduan.

When he failed to pay the boundary-hulog and the petitioner took


back the jeep, Bustamante filed for illegal dismissal and alleged that
he was employed by Villamaria in July 1996 under the boundary
system, where he was required to remit P450.00 a day. After one year
of continuously working for them, the spouses Villamaria presented the
Kasunduan for his signature, with the assurance that he (Bustamante)
would own the jeepney by March 2001 after paying P550.00 in daily
installments and that he would thereafter continue driving the vehicle
along the same route under the same franchise. He further narrated
that in July 2000, he informed the Villamaria spouses that the surplus
engine of the jeepney needed to be replaced, and was assured that it
would be done. However, he was later arrested and his driver’s license
was confiscated because apparently, the replacement engine that was

15
Page 16

installed was taken from a stolen vehicle. Due to negotiations with the
apprehending authorities, the jeepney was not impounded. The
Villamaria spouses took the jeepney from him on July 24, 2000, and
he was no longer allowed to drive the vehicle since then unless he paid
them P70,000.00.

Villamaria admitted the existence of the Kasunduan, but alleged


that Bustamante failed to pay the P10,000.00 downpayment and the
vehicle’s annual registration fees. They further alleged that
Bustamante eventually failed to remit the requisite boundary-hulog of
P550.00 a day, which prompted them to issue the Paalaala. Instead of
complying with his obligations, Bustamante stopped making his
remittances despite his daily trips and even brought the jeepney to the
province without permission. Worse, the jeepney figured in an accident
and its license plate was confiscated; Bustamante even abandoned
the vehicle in a gasoline station in Sucat, Parañaque City for two
weeks. Villamaria argued that Bustamante was not illegally dismissed
since the Kasunduan executed on August 7, 1997 transformed the
employer-employee relationship into that of vendor-vendee. Hence,
the spouses concluded, there was no legal basis to hold them liable for
illegal dismissal.

The LA ruled in favor of petitioner. The NLRC dismissed


respondent’s appeal. The NLRC ruled that under the Kasunduan, the
juridical relationship between Bustamante and Villamaria was that of
vendor and vendee, hence, the Labor Arbiter had no jurisdiction over
the complaint. The CA ruled that the LA has jurisdiction because the
relationship between Bustamante and Villamaria was dual: that of
vendor-vendee and employer-employee.

ISSUE:
Whether the Labor Arbiter has jurisdiction over the illegal
dismissal case.

RULING:
YES. The jurisdiction of Labor Arbiters and the NLRC under Article
217 of the Labor Code is limited to disputes arising from an employer-
employee relationship which can only be resolved by reference to the
Labor Code, other labor statutes or their collective bargaining
agreement. Not every dispute between an employer and employee
involves matters that only the Labor Arbiter and the NLRC can resolve
in the exercise of their adjudicatory or quasi-judicial powers. Actions
between employers and employees where the employer-employee
relationship is merely incidental is within the exclusive original
jurisdiction of the regular courts. When the principal relief is to be
granted under labor legislation or a collective bargaining agreement,
the case falls within the exclusive jurisdiction of the Labor Arbiter and

16
Page 17

the NLRC even though a claim for damages might be asserted as an


incident to such claim.

Under the boundary-hulog scheme incorporated in the


Kasunduan, a dual juridical relationship was created between
petitioner and respondent: that of employer-employee and vendor-
vendee. The Kasunduan did not extinguish the employer-employee
relationship of the parties extant before the execution of said deed. As
early as 1956, the Court ruled in National Labor Union v. Dinglasan
that the jeepney owner/operator-driver relationship under the boundary
system is that of employer-employee and not lessor-lessee. This
doctrine was affirmed, under similar factual settings, in Magboo v.
Bernardo and Lantaco, Sr. v. Llamas, and was analogously applied to
govern the relationships between auto-calesa owner/operator and
driver, bus owner/operator and conductor, and taxi owner/operator and
driver.

The boundary system is a scheme by an owner/operator engaged


in transporting passengers as a common carrier to primarily govern the
compensation of the driver, that is, the latter’s daily earnings are
remitted to the owner/operator less the excess of the boundary which
represents the driver’s compensation. Under this system, the
owner/operator exercises control and supervision over the driver. It is
unlike in lease of chattels where the lessor loses complete control over
the chattel leased but the lessee is still ultimately responsible for the
consequences of its use. The management of the business is still in
the hands of the owner/operator, who, being the holder of the
certificate of public convenience, must see to it that the driver follows
the route prescribed by the franchising and regulatory authority, and
the rules promulgated with regard to the business operations. The fact
that the driver does not receive fixed wages but only the excess of the
"boundary" given to the owner/operator is not sufficient to change the
relationship between them. Indubitably, the driver performs activities
which are usually necessary or desirable in the usual business or trade
of the owner/operator.

Under the Kasunduan, respondent was required to remit P550.00


daily to petitioner, an amount which represented the boundary of
petitioner as well as respondent’s partial payment (hulog) of the
purchase price of the jeepney. Respondent was entitled to keep the
excess of his daily earnings as his daily wage. Thus, the daily
remittances also had a dual purpose: that of petitioner’s boundary and
respondent’s partial payment (hulog) for the vehicle. This dual purpose
was expressly stated in the Kasunduan. The well-settled rule is that an
obligation is not novated by an instrument that expressly recognizes
the old one, changes only the terms of payment, and adds other
obligations not incompatible with the old provisions or where the new

17
Page 18

contract merely supplements the previous one. The two obligations of


the respondent to remit to petitioner the boundary-hulog can stand
together.

Under the Kasunduan, petitioner retained supervision and control


over the conduct of the respondent as driver of the jeepney. Under the
boundary-hulog scheme, petitioner retained ownership of the jeepney
although its material possession was vested in respondent as its driver.
In case respondent failed to make his P550.00 daily installment
payment for a week, the agreement would be of no force and effect
and respondent would have to return the jeepney to petitioner; the
employer-employee relationship would likewise be terminated unless
petitioner would allow respondent to continue driving the jeepney on a
boundary basis of P550.00 daily despite the termination of their
vendor-vendee relationship. The juridical relationship of employer-
employee between petitioner and respondent was not negated by the
foregoing stipulation in the Kasunduan, considering that petitioner
retained control of respondent’s conduct as driver of the vehicle.

The exercise of control by private respondent over petitioner’s


conduct in operating the jeepney he was driving is inconsistent with
private respondent’s claim that he is, or was, not engaged in the
transportation business; that, even if petitioner was allowed to let some
other person drive the unit, it was not shown that he did so; that the
existence of an employment relation is not dependent on how the
worker is paid but on the presence or absence of control over the
means and method of the work; that the amount earned in excess of
the "boundary hulog" is equivalent to wages; and that the fact that the
power of dismissal was not mentioned in the Kasunduan did not mean
that private respondent never exercised such power, or could not
exercise such power. Requiring petitioner to drive the unit for
commercial use, or to wear an identification card, or to wear decent
attire, or to park the vehicle in Villamaria Motors garage, or to inform
Villamaria Motors about the fact that the unit would be going out to the
province for two days of more, or to drive the unit carefully, etc.
necessarily related to control over the means by which the petitioner
was to go about his work; that the ruling applicable here is not Singer
Sewing Machine but National Labor Union since the latter case
involved jeepney owners/operators and jeepney drivers, and that the
fact that the "boundary" here represented installment payment of the
purchase price on the jeepney did not withdraw the relationship from
that of employer-employee, in view of the overt presence of
supervision and control by the employer.

18
Page 19

Case Digests by: MARIELLE B. ANTONIO

MENDOZA v. OFFICERS OF MANILA WATER


EMPLOYEE’S UNION
G.R. No. 201595, January 25, 2016
Del Castillo, J.

DOCTRINE:
JURISDICTION OF LABOR ARBITER (ULP)
"Inter-Union Dispute" refers to any conflict between and among
legitimate labor unions involving representation questions for purposes
of collective bargaining or to any other conflict or dispute between
legitimate labor unions. "Intra-Union Dispute" refers to any conflict
between and among union members, including grievances arising from
any violation of the rights and conditions of membership, violation of or
disagreement over any provision of the union’s constitution and by-
laws, or disputes arising from chartering or affiliation of union. On the
other hand, the circumstances of unfair labor practices (ULP) of a labor
organization are stated in Article 249 of the Labor Code.

FACTS:
Petitioner was charged for non-payment of union dues twice and
was suspended for 30 days. Meanwhile, MWEU scheduled an election
of officers on September 14, 2007. Petitioner filed his certificate of
candidacy for Vice-President, but he was disqualified for not being a
member in good standing on account of his suspension. For the third
time, petitioner was charged for non-payment of union dues again and
was expelled.

In 2008, during the freedom period and negotiations for a new


collective bargaining agreement (CBA) with MWC, petitioner joined
another union, the Workers Association for Transparency,
Empowerment and Reform, All-Filipino Workers Confederation
(WATER-AFWC). He was elected union President. Other MWEU
members were inclined to join WATER-AFWC, but MWEU director
Torres threatened that they would not get benefits from the new CBA.
The MWEU leadership submitted a proposed CBA which contained
provisions to the effect that in the event of retrenchment, non-MWEU
members shall be removed first, and that upon the signing of the CBA,
only MWEU members shall receive a signing bonus.

Petitioner filed a Complaint against respondents for unfair labor


practices. Respondents claimed that the Labor Arbiter had no
jurisdiction over the dispute, which is intra-union in nature; that the
Bureau of Labor Relations (BLR) was the proper venue, in accordance

19
Page 20

with Article 226 of the Labor Code and Section 1, Rule XI of


Department Order 40-03, series of 2003, of the DOLE.

The LA ruled that the filing of the instant case is still premature.
Section 5, Article X-Investigation Procedures and Appeal Process of
the Union Constitution and By-Laws provides that: Section 5. Any
dismissed and/or expelled member shall have the rights to appeal to
the Executive Board within seven (7) days from the date of notice of
the said dismissal and/or expulsion, which in [turn] shall be referred to
the General Membership Assembly. In case of an appeal, a simple
majority of the decision of the Executive Board is imperative. The same
shall be approved/disapproved by a majority vote of the general
membership assembly in a meeting duly called for the purpose. On the
basis of the foregoing, the parties shall exhaust first all the
administrative remedies before resorting to compulsory arbitration.
Thus, instant case is referred back to the Union for the General
Assembly to act or deliberate complainant’s appeal on the decision of
the Executive Board.

ISSUE:
Whether the Labor Arbiter has jurisdiction over ULP.

RULING:
YES. It is true that some of petitioner’s causes of action constitute
intra-union cases cognizable by the BLR under Article 226 of the Labor
Code. However, petitioner’s charge of unfair labor practices falls within
the original and exclusive jurisdiction of the Labor Arbiters, pursuant to
Article 217 of the Labor Code. In addition, Article 247 of the same Code
provides that "the civil aspects of all cases involving unfair labor
practices, which may include claims for actual, moral, exemplary and
other forms of damages, attorney’s fees and other affirmative relief,
shall be under the jurisdiction of the Labor Arbiters."

Unfair labor practices may be committed both by the employer


under Article 248 and by labor organizations under Article 249 of the
Labor Code. The provisions of the preceding paragraph
notwithstanding, only the officers, members of governing boards,
representatives or agents or members of labor associations or
organizations who have actually participated in, authorized or ratified
unfair labor practices shall be held criminally liable.

Under Article VI, Section 2(a) of MWEU’s Constitution and By-


Laws, the general membership assembly has the power to "review
revise modify affirm or repeal resolution and decision of the Executive
Board and/or committees upon petition of thirty percent (30%) of the
Union in good standing," and under Section 2(d), to "revise, modify,

20
Page 21

affirm or reverse all expulsion cases." Under Section 3 of the same


Article, "[t]he decision of the Executive Board may be appealed to the
General Membership which by a simple majority vote reverse the
decision of said body. If the general Assembly is not in session the
decision of the Executive Board may be reversed by a petition of the
majority of the general membership in good standing." And, in Article
X, Section 5, "[a]ny dismissed and/or expelled member shall have the
right to appeal to the Executive Board within seven days from notice of
said dismissal and/or expulsion which, in [turn] shall be referred to the
General membership assembly. In case of an appeal, a simple majority
of the decision of the Executive Board is imperative. The same shall
be approved/disapproved by a majority vote of the general
membership assembly in a meeting duly called for the purpose."

In regard to suspension of a union member, MWEU’s Constitution


and By-Laws provides under Article X, Section 4 thereof that "[a]ny
suspended member shall have the right to appeal within three (3)
working days from the date of notice of said suspension. In case of an
appeal a simple majority of vote of the Executive Board shall be
necessary to nullify the suspension." Thus, when an MWEU member
is suspended, he is given the right to appeal such suspension within
three working days from the date of notice of said suspension, which
appeal the MWEU Executive Board is obligated to act upon by a simple
majority vote. When the penalty imposed is expulsion, the expelled
member is given seven days from notice of said dismissal and/or
expulsion to appeal to the Executive Board, which is required to act by
a simple majority vote of its members. The Board’s decision shall then
be approved/ disapproved by a majority vote of the general
membership assembly in a meeting duly called for the purpose.

The documentary evidence is clear that when petitioner received


Borela’s August 21, 2007 letter informing him of the Executive Board’s
unanimous approval of the grievance committee recommendation to
suspend him for the second time effective August 24, 2007, he
immediately and timely filed a written appeal. However, the Executive
Board did not act thereon. Then again, when petitioner was charged
for the third time and meted the penalty of expulsion from MWEU by
the unanimous vote of the Executive Board, his timely appeal was
again not acted upon by said board.

Thus, contrary to respondents’ argument that petitioner lost his


right to appeal when he failed to petition to convene the general
assembly through the required signature of 30% of the union
membership in good standing pursuant to Article VI, Section 2(a) of
MWEU’s Constitution and By-Laws or by a petition of the majority of
the general membership in good standing under Article VI, Section 3,
this Court finds that petitioner was illegally suspended for the second

21
Page 22

time and thereafter unlawfully expelled from MWEU due to


respondents’ failure to act on his written appeals. The required petition
to convene the general assembly through the required signature of
30% (under Article VI, Section 2[a]) or majority (under Article VI,
Section 3) of the union membership does not apply in petitioner’s case;
the Executive Board must first act on his two appeals before the matter
could properly be referred to the general membership. Because
respondents did not act on his two appeals, petitioner was
unceremoniously suspended, disqualified and deprived of his right to
run for the position of MWEU Vice-President in the September 14,
2007 election of officers, expelled from MWEU, and forced to join
another union, WATER-AFWC. For these, respondents are guilty of
unfair labor practices under Article 249 (a) and (b) – that is, violation of
petitioner’s right to self-organization, unlawful discrimination, and
illegal termination of his union membership – which case falls within
the original and exclusive jurisdiction of the Labor Arbiters, in
accordance with Article 217 of the Labor Code.

"In essence, [unfair labor practice] relates to the commission of


acts that transgress the workers’ right to organize." "All the prohibited
acts constituting unfair labor practice in essence relate to the workers’
right to self-organization." "The term unfair labor practice refers to that
gamut of offenses defined in the Labor Code which, at their core,
violates the constitutional right of workers and employees to self-
organization."

The right of self-organization includes the right to organize or


affiliate with a labor union or determine which of two or more unions in
an establishment to join, and to engage in concerted activities with co-
workers for purposes of collective bargaining through representatives
of their own choosing, or for their mutual aid and protection, i.e., the
protection, promotion, or enhancement of their rights and interests.

As members of the governing board of MWEU, respondents are


presumed to know, observe, and apply the union’s constitution and by-
laws. Thus, their repeated violations thereof and their disregard of
petitioner’s rights as a union member – their inaction on his two
appeals which resulted in his suspension, disqualification from running
as MWEU officer, and subsequent expulsion without being accorded
the full benefits of due process – connote willfulness and bad faith, a
gross disregard of his rights thus causing untold suffering, oppression
and, ultimately, ostracism from MWEU.

22
Page 23

Case Digests by: MARIELLE B. ANTONIO

ATLAS FARMS v. NLRC


G.R. No. 142244, November 18, 2002
Quisumbing, J.

DOCTRINE:
JURISDICTION OF LABOR ARBITER (TERMINATION DISPUTE)
Where the dispute is just in the interpretation, implementation or
enforcement stage, it may be referred to the grievance machinery set
up in the CBA, or brought to voluntary arbitration. But, where there was
already actual termination, with alleged violation of the employee’s
rights, it is already cognizable by the labor arbiter.

FACTS:
Private respondent Jaime O. dela Peña was employed as a
veterinary aide by petitioner in December 1975. He was among several
employees terminated in July 1989. On July 8, 1989, he was re-hired
by petitioner and given the additional job of feedmill operator. He was
instructed to train selected workers to operate the feedmill.

On March 13, 1993, Peña was allegedly caught urinating and


defecating on company premises not intended for the purpose. The
farm manager of petitioner issued a formal notice directing him to
explain within 24 hours why disciplinary action should not be taken
against him for violating company rules and regulations.

Co-respondent Abion was a carpenter/mason and a maintenance


man whose employment by petitioner commenced on October 8, 1990.
Allegedly, he caused the clogging of the fishpond drainage resulting in
damages worth several hundred thousand pesos when he improperly
disposed of the cut grass and other waste materials into the pond’s
drainage system.

Both respondents refused to receive the notice requiring them to


explain and both were subsequently terminated. Peña and Abion filed
separate complaints for illegal dismissal that were later consolidated.

The labor arbiter dismissed their complaints on the ground that the
grievance machinery in the collective bargaining agreement (CBA) had
not yet been exhausted. Private respondents availed of the grievance
process, but later on refiled the case before the NLRC in Region IV.
They alleged "lack of sympathy" on petitioner’s part to engage in
conciliation proceedings.

23
Page 24

Petitioner contends that the dismissal of private respondents was


for a just and valid cause, pursuant to the provisions of the company’s
rules and regulations. It also alleges lack of jurisdiction on the part of
the labor arbiter, claiming that the cases should have been resolved
through the grievance machinery, and eventually referred to voluntary
arbitration, as prescribed in the CBA.

The LA dismissed the complaint finding that the case was one of
illegal dismissal and did not involve the interpretation or
implementation of any CBA provision. The NLRC ruled that
respondents were illegally dismissed.

ISSUE:
Whether the Labor Arbiter has jurisdiction over the illegal
dismissal case.

RULING:
YES. Article 217 of the Labor Code provides that labor arbiters
have original and exclusive jurisdiction over termination disputes. A
possible exception is provided in Article 261 of the Labor Code, which
provides that — The Voluntary Arbitrator or panel of voluntary
arbitrators shall have original and exclusive jurisdiction to hear and
decide all unresolved grievances arising from the interpretation or
implementation of the Collective Bargaining Agreement and those
arising from the interpretation or enforcement of company personnel
policies referred to in the immediately preceding article. Accordingly,
violations of a Collective Bargaining Agreement, except those which
are gross in character, shall no longer be treated as unfair labor
practice and shall be resolved as grievances under the Collective
Bargaining Agreement. For purposes of this article, gross violations of
Collective Bargaining Agreement shall mean flagrant and or malicious
refusal to comply with the economic provisions of such agreement.

The Commission, its Regional Offices and the Regional Directors


of the Department of Labor and Employment shall not entertain
disputes, grievances or matters under the exclusive and original
jurisdiction of the Voluntary Arbitrator or panel of Voluntary Arbitrators
and shall immediately dispose and refer the same to the grievance
Machinery or Arbitration provided in the Collective Bargaining
Agreement.

The instant case is a termination dispute falling under the original


and exclusive jurisdiction of the Labor Arbiter, and does not specifically
involve the application, implementation or enforcement of company
personnel policies contemplated in Policy Instruction No. 56.

24
Page 25

Records show, however, that private respondents sought without


success to avail of the grievance procedure in their CBA. On this point,
petitioner maintains that by so doing, private respondents recognized
that their cases still fell under the grievance machinery. According to
petitioner, without having exhausted said machinery, the private
respondents filed their action before the NLRC, in a clear act of forum-
shopping. However, it is worth pointing out that private respondents
went to the NLRC only after the labor arbiter dismissed their original
complaint for illegal dismissal. Under these circumstances private
respondents had to find another avenue for redress.

It was petitioner who failed to show proof that it took steps to


convene the grievance machinery after the labor arbiter first dismissed
the complaints for illegal dismissal and directed the parties to avail of
the grievance procedure under Article VII of the existing CBA. They
could not now be faulted for attempting to find an impartial forum, after
petitioner failed to listen to them and after the intercession of the labor
arbiter proved futile.

Pursuant to Article 260 of the Labor Code, the parties to a CBA


shall name or designate their respective representatives to the
grievance machinery and if the grievance is unsettled in that level, it
shall automatically be referred to the voluntary arbitrators designated
in advance by the parties to a CBA. Consequently, only disputes
involving the union and the company shall be referred to the grievance
machinery or voluntary arbitrators. In these termination cases of
private respondents, the union had no participation, it having failed to
object to the dismissal of the employees concerned by the petitioner.
It is obvious that arbitration without the union’s active participation on
behalf of the dismissed employees would be pointless, or even
prejudicial to their cause.

Without proof of notice, private respondents who were


subsequently dismissed without hearing were also deprived of a
chance to air their side at the level of the grievance machinery. Given
the fact of dismissal, it can be said that the cases were effectively
removed from the jurisdiction of the voluntary arbitrator, thus placing
them within the jurisdiction of the labor arbiter. Where the dispute is
just in the interpretation, implementation or enforcement stage, it may
be referred to the grievance machinery set up in the CBA, or brought
to voluntary arbitration. But, where there was already actual
termination, with alleged violation of the employee’s rights, it is already
cognizable by the labor arbiter. The labor arbiter and then the NLRC
had jurisdiction over the cases involving private respondents’
dismissal, and no error was committed by the appellate court in
upholding their assumption of jurisdiction.

25
Page 26

Case Digests by: MARIELLE B. ANTONIO

NEGROS METAL v. LAMAYO


G.R. No. 186557, August 25, 2010
Carpio Morales, J.

DOCTRINE:
JURISDICTION OF LABOR ARBITER (TERMINATION DISPUTE)
As a general rule then, termination disputes should be brought
before a labor arbiter, except when the parties, under Art. 262,
unmistakably express that they agree to submit the same to voluntary
arbitration.

FACTS:
While respondent was at the company's foundry grinding some
tools he was using, William Uy, Sr., company manager, called his
attention why he was using the grinder there to which he replied that
since the machine there was bigger, he would finish his work faster.
Respondent's explanation was found unsatisfactory, hence, he was,
via memorandum, charged of loitering and warned. He reported for
work after three days but was suspended again for another 10 days.

After serving the two suspensions, his services was terminated


prompting him to file an illegal dismissal case. Petitioner contends that
the LA has no jurisdiction over the case as it should first go through the
grievance procedure under their CBA. However, respondent claims
that he ceased to be a member of the union which is why he did not
opt to go through the grievance procedure.

ISSUE:
Whether the illegal dismissal case should first go through
grievance procedure.

RULING:
NO. Articles 217, 261, and 262 of the Labor Code outline the
jurisdiction of labor arbiters and voluntary arbitrators. As a general rule
then, termination disputes should be brought before a labor arbiter,
except when the parties, under Art. 262, unmistakably express that
they agree to submit the same to voluntary arbitration.

In the present case, the CBA provision on grievance machinery


being invoked by petitioner does not expressly state that termination
disputes are included in the ambit of what may be brought before the
company's grievance machinery. Thus, the pertinent provision in the

26
Page 27

parties' CBA reads:The parties hereto agree on principle that all


disputes between labor and management may be settled through
friendly negotiations that the parties have the same interest in the
continuity of work until all points in dispute shall have been discussed
and settled. For this purpose, a grievance is defined as any
disagreement between the UNION and the EMPLOYER or between a
worker or group of workers on one hand and the EMPLOYER on the
one hand as to the application and interpretation of any of the
provisions of this contract. Other matters subject of collective
bargaining or regulated by existing labor laws shall not be considered
as grievances.

Even assuming, however, that the suspension of an employee


may be considered as a "disagreement" which bears on the
"application and interpretation of any of the provisions" of the CBA,
respondent could not have bound himself to bring the matter of his
suspension to grievance procedure or voluntary arbitration in light of
the documented fact that he had resigned from the union more than a
year before his suspension, not to mention the fact that he denied
having a hand in the preparation of the union president Ronquillo's
letter invoking the grievance procedure. In fine, the labor tribunal had
original and exclusive jurisdiction over respondent's complaint for
illegal dismissal.

The Court sustains the Labor Arbiter's ruling that respondent was
illegally dismissed absent a showing that he was accorded due
process when he was summarily terminated. The Court is not a trier of
facts. It is not tasked to review the evidence on record, documentary
and testimonial, and reassess the probative weight thereof, especially
in view of the well-entrenched rule that findings of fact of administrative
officials, such as labor arbiters, who have acquired expertise on
account of their specialized jurisdiction are accorded by the courts not
only respect but, most often, with finality, particularly when affirmed on
appeal.

27
Page 28

Case Digests by: MARIELLE B. ANTONIO

VIVERO v. CA
G.R. No.138938, October 24, 2000
Bellosillo, J.

DOCTRINE:
JURISDICTION OF LABOR ARBITER (TERMINATION DISPUTE)
There is a need for an express stipulation in the CBA that illegal
termination disputes should be resolved by a Voluntary Arbitrator or
Panel of Voluntary Arbitrators, since the same fall within a special class
of disputes that are generally within the exclusive original jurisdiction
of Labor Arbiters by express provision of law.

FACTS:
Complainant was hired by respondent as Chief Officer of the
vessel. On grounds of very poor performance and conduct, refusal to
perform his job, refusal to report to the Captain or the vessel’s
Engineers or cooperate with other ship officers about the problem in
cleaning the cargo holds or of the shipping pump and his dismal
relations with the Captain of the vessel, complainant was repatriated
on 15 July 1994.

He filed a case for illegal dismissal with the Union and went
through the grievance procedure as stated in the CBA. The parties
failed to reach an amicable settlement. Complainant then filed the case
with the POEA.

While the case was pending before the POEA, private


respondents filed a Motion to Dismiss on the ground that the POEA
had no jurisdiction over the case considering petitioner Vivero’s failure
to refer it to a Voluntary Arbitration Committee in accordance with the
CBA between the parties. Upon the enactment of RA 8042, the Migrant
Workers and Overseas Filipinos Act of 1995, the case was transferred
to the Adjudication Branch of the NLRC.

According to the Labor Arbiter, since the CBA of the parties


provided for the referral to a Voluntary Arbitration Committee should
the Grievance Committee fail to settle the dispute, and considering the
mandate of Art. 261 of the Labor Code on the original and exclusive
jurisdiction of Voluntary Arbitrators, the Labor Arbiter clearly had no
jurisdiction over the case.

28
Page 29

The NLRC set aside the decision of the Labor Arbiter on the
ground that the record was clear that petitioner had exhausted his
remedy by submitting his case to the Grievance Committee of
AMOSUP(Union). The NLRC further held that the contested portion in
the CBA providing for the intercession of a Voluntary Arbitrator was not
binding upon petitioner since both petitioner and private respondents
had to agree voluntarily to submit the case before a Voluntary
Arbitrator or Panel of Voluntary Arbitrators.

The CA set aside the NLRC decision and reinstated the LA ruling.
Petitioner should have followed the provision in the CBA requiring the
submission of the dispute to the Voluntary Arbitration Committee once
the Grievance Committee failed to settle the controversy.

The fact that private respondents filed their Position Paper first
before filing their Motion to Dismiss was immaterial and did not operate
to confer jurisdiction upon the Labor Arbiter, following the well-settled
rule that jurisdiction is determined by law and not by consent or
agreement of the parties or by estoppel.

ISSUE:
Whether the case is a termination dispute which the Adjudication
Branch of the NLRC, has jurisdiction or whether the NLRC is deprived
of jurisdiction over illegal dismissal cases whenever a CBA provides
for grievance machinery and voluntary arbitration proceedings.

RULING:
YES. The case is primarily a termination dispute. It is clear from
the claim/assistance request form submitted by petitioner to AMOSUP
that he was challenging the legality of his dismissal for lack of cause
and lack of due process.

It is not sufficient to merely say that parties to the CBA agree on


the principle that "all disputes" should first be submitted to a Voluntary
Arbitrator. There is a need for an express stipulation in the CBA that
illegal termination disputes should be resolved by a Voluntary
Arbitrator or Panel of Voluntary Arbitrators, since the same fall within a
special class of disputes that are generally within the exclusive original
jurisdiction of Labor Arbiters by express provision of law. Absent such
express stipulation, the phrase "all disputes" should be construed as
limited to the areas of conflict traditionally within the jurisdiction of
Voluntary Arbitrators, i.e., disputes relating to contract-interpretation,
contract-implementation, or interpretation or enforcement of company
personnel policies. Illegal termination disputes — not falling within any
of these categories — should then be considered as a special area of
interest governed by a specific provision of law.

29
Page 30

In this case, however, while the parties did agree to make


termination disputes the proper subject of voluntary arbitration, such
submission remains discretionary upon the parties.

The use of the word "may" shows the intention of the parties to
reserve the right to submit the illegal termination dispute to the
jurisdiction of the Labor Arbiter, rather than to a Voluntary Arbitrator.
Petitioner validly exercised his option to submit his case to a Labor
Arbiter when he filed his Complaint before the proper government
agency.

In the case at bar, the dispute was never brought to a Voluntary


Arbitrator for resolution; in fact, petitioner precisely requested the Court
to recognize the jurisdiction of the Labor Arbiter over the case. The
Court had held in San Miguel Corp. v. NLRC that neither officials nor
tribunals can assume jurisdiction in the absence of an express legal
conferment. In the same manner, petitioner cannot arrogate into the
powers of Voluntary Arbitrators the original and exclusive jurisdiction
of Labor Arbiters over unfair labor practices, termination disputes, and
claims for damages, in the absence of an express agreement between
the parties in order for Art. 262 of the Labor Code to apply in the case
at bar. In other words, the Court of Appeals is correct in holding that
Voluntary Arbitration is mandatory in character if there is a specific
agreement between the parties to that effect. It must be stressed
however that, in the case at bar, the use of the word "may" shows the
intention of the parties to reserve the right of recourse to Labor Arbiters.

The CBA clarifies the proper procedure to be followed in situations


where the parties expressly stipulate to submit termination disputes to
the jurisdiction of a Voluntary Arbitrator or Panel of Voluntary
Arbitrators. For when the parties have validly agreed on a procedure
for resolving grievances and to submit a dispute to voluntary arbitration
then that procedure should be strictly observed. Non-compliance
therewith cannot be excused, as petitioner suggests, by the fact that
he is not well-versed with the "fine prints" of the CBA. It was his
responsibility to find out, through his Union, what the provisions of the
CBA were and how they could affect his rights. As provided in Art. 241,
par. (p), of the Labor Code.

It may be observed that under Policy Instruction No. 56 of the


Secretary of Labor, dated 6 April 1993, "Clarifying the Jurisdiction
Between Voluntary Arbitrators and Labor Arbiters Over Termination
Cases and Providing Guidelines for the Referral of Said Cases
Originally Filed with the NLRC to the NCMB," termination cases arising
in or resulting from the interpretation and implementation of collective
bargaining agreements and interpretation and enforcement of
company personnel policies which were initially processed at the

30
Page 31

various steps of the plant-level Grievance Procedures under the


parties’ collective bargaining agreements fall within the original and
exclusive jurisdiction of the voluntary arbitrator pursuant to Art. 217 (c)
and Art. 261 of the Labor Code; and, if filed before the Labor Arbiter,
these cases shall be dismissed by the Labor Arbiter for lack of
jurisdiction and referred to the concerned NCMB Regional Branch for
appropriate action towards an expeditious selection by the parties of a
Voluntary Arbitrator or Panel of Arbitrators based on the procedures
agreed upon in the CBA.

This case is a termination dispute falling under the original and


exclusive jurisdiction of the Labor Arbiter, and does not specifically
involve the application, implementation or enforcement of company
personnel policies contemplated in Policy Instruction No. 56.
Consequently, Policy Instruction No. 56 does not apply in the case at
bar. In any case, private respondents never invoked the application of
Policy Instruction No. 56 in their Position Papers, neither did they raise
the question in their Motion to Dismiss which they filed nine (9) months
after the filing of their Position Papers. At this late stage of the
proceedings, it would not serve the ends of justice if this case is
referred back to a Voluntary Arbitrator considering that both the
AMOSUP and private respondents have submitted to the jurisdiction
of the Labor Arbiter by filing their respective Position Papers and
ignoring the grievance procedure set forth in their CBA.

After the grievance proceedings have failed to bring about a


resolution, AMOSUP, as agent of petitioner, should have informed him
of his option to settle the case through voluntary arbitration. Private
respondents, on their part, should have timely invoked the provision of
their CBA requiring the referral of their unresolved disputes to a
Voluntary Arbitrator once it became apparent that the grievance
machinery failed to resolve it prior to the filing of the case before the
proper tribunal. The private respondents should not have waited for
nine (9) months from the filing of their Position Paper with the POEA
before it moved to dismiss the case purportedly for lack of jurisdiction.
As it is, private respondents are deemed to have waived their right to
question the procedure followed by petitioner, assuming that they have
the right to do so. Under their CBA, both Union and respondent
companies are responsible for selecting an impartial arbitrator or for
convening an arbitration committee; yet, it is apparent that neither
made a move towards this end. Consequently, petitioner should not be
deprived of his legitimate recourse because of the refusal of both Union
and respondent companies to follow the grievance procedure.

31
Page 32

Case Digests by: MARIELLE B. ANTONIO

UNIVERSITY OF IMMACULATE CONCEPCION v. NLRC


G.R. No. 181146, January 26, 2011
Carpio, J.

DOCTRINE:
JURISDICTION OF LABOR ARBITER (TERMINATION DISPUTE)
No less than Section 3, Article XIII of the Constitution declares
as state policy the preferential use of voluntary modes in settling
disputes, to wit: The State shall promote the principle of shared
responsibility between workers and employers and the preferential use
of voluntary modes in settling disputes, including conciliation, and shall
enforce their mutual compliance therewith to foster industrial peace.

FACTS:
From 18 November to 22 November 2002, private respondent
Axalan, regular faculty and union president, attended a seminar in
Quezon City on website development. Axalan claimed that she held
online classes while attending the seminar. She explained that she was
under the impression that faculty members would not be marked
absent even if they were not physically present in the classroom as
long as they conducted online classes. Dean Celestial relayed to
Axalan the message of the university president that no administrative
charge would be filed if Axalan would admit having been absent
without official leave and write a letter of apology seeking forgiveness.

From 28 January to 3 February 2003, Axalan attended a seminar


in Baguio City on advanced paralegal training. Dean Celestial wrote
Axalan informing her that her participation in the paralegal seminar in
Baguio City was the subject of a second AWOL charge.

After conducting hearings and receiving evidence, the ad hoc


grievance committee found Axalan to have incurred AWOL on both
instances and recommended that Axalan be suspended without pay
for six months on each AWOL charge. Thus, Axalan filed for
constructive dismissal. The university moved to dismiss the complaint
on the ground that the Labor Arbiter had no jurisdiction over the subject
matter of the complaint. The university maintained that jurisdiction lay
in the voluntary arbitrator.

The Labor Arbiter held that there being no existing collective


bargaining agreement between the parties, no grievance machinery
was constituted, which barred resort to voluntary arbitration. The LA,
NLRC and CA ruled that Axalan was illegally dismissed.

32
Page 33

ISSUE:
Whether the Voluntary Arbitrator has jurisdiction over the case.

RULING:
YES. As can be gleaned from the transcript of stenographic notes
of the administrative hearing held on 20 February 2003, the parties in
this case clearly agreed to resort to voluntary arbitration. To quote the
exact words of the parties’ counsels: Atty. Dante Sandiego: x x x So,
are we to understand that the decision of the President shall be without
prejudice to the right of the employees to contest the validity or legality
of his dismissal or of the disciplinary action imposed upon him by
asking for voluntary arbitration under the Labor Code or when
applicable availing himself of the grievance machinery under the Labor
Code which ends in voluntary arbitration. That will be the steps that we
will have to follow. Atty. Sabino Padilla, Jr. agreed.

Thus, the Labor Arbiter should have immediately disposed of the


complaint and referred the same to the voluntary arbitrator when the
university moved to dismiss the complaint for lack of jurisdiction.

It is unrefuted that Axalan promptly resumed teaching at the


university right after the expiration of the suspension period. In other
words, Axalan never quit. Hence, Axalan cannot claim that she was left
with no choice but to quit, a crucial element in a finding of constructive
dismissal. Thus, Axalan cannot be deemed to have been constructively
dismissed.

Significantly, at the time the Labor Arbiter rendered his Decision


on 11 October 2004, Axalan had already returned to her teaching job
at the university on 1 October 2004. The Labor Arbiter’s Decision
ordering the reinstatement of Axalan, who at the time had already
returned to work, is thus absurd. There being no constructive
dismissal, there is no legal basis for the Labor Arbiter’s order of
reinstatement as well as payment of backwages, salary differentials,
damages, and attorney’s fees.

Note that on the first AWOL incident, the university even offered
to drop the AWOL charge against Axalan if she would only write a letter
of contrition. But Axalan adamantly refused knowing fully well that the
administrative case would take its course leading to possible
sanctions. She cannot now be heard that the imposition of the penalty
of six-month suspension without pay for each AWOL charge is
unreasonable.

33
Page 34

Case Digests by: MARIELLE B. ANTONIO

AUSTRIA v. NLRC
G.R. No. 124382, August 16, 1999
Kapunan, J.

DOCTRINE:
JURISDICTION OF LABOR ARBITER (TERMINATION DISPUTE;
PRIEST)
While the State is prohibited from interfering in purely
ecclesiastical affairs, the Church is likewise barred from meddling in
purely secular matters.

FACTS:
Petitioner was a Pastor of the Seventh-Day Adventists (SDA) until
31 October 1991, when his services were terminated. Petitioner
received a letter of dismissal citing misappropriation of denominational
funds, willful breach of trust, serious misconduct, gross and habitual
neglect of duties, and commission of an offense against the person of
employers duly authorized representative, as grounds for the
termination of his services.

Reacting against the adverse decision of the SDA, petitioner filed


a complaint before the Labor Arbiter for illegal dismissal against the
SDA and its officers and prayed for reinstatement with backwages and
benefits, moral and exemplary damages and other labor law benefits.
The Labor Arbiter rendered a decision in favor of petitioner. The NLRC
dismissed the case for lack of jurisdiction.

Private respondents contend that by virtue of the doctrine of


separation of church and state, the Labor Arbiter and the NLRC have
no jurisdiction to entertain the complaint filed by petitioner. Since the
matter at bar allegedly involves the discipline of a religious minister, it
is to be considered a purely ecclesiastical affair to which the State has
no right to interfere.

ISSUE:
Whether the Labor Arbiter/NLRC has jurisdiction over the case.

RULING:
YES. The case at bar does not concern an ecclesiastical or purely
religious affair as to bar the State from taking cognizance of the same.
An ecclesiastical affair is one that concerns doctrine, creed, or form or
worship of the church, or the adoption and enforcement within a
religious association of needful laws and regulations for the

34
Page 35

government of the membership, and the power of excluding from such


associations those deemed unworthy of membership. Based on this
definition, an ecclesiastical affair involves the relationship between the
church and its members and relate to matters of faith, religious
doctrines, worship and governance of the congregation. To be
concrete, examples of this so-called ecclesiastical affairs to which the
State cannot meddle are proceedings for excommunication,
ordinations of religious ministers, administration of sacraments and
other activities with which attached religious significance. The case at
bar does not even remotely concern any of the abovecited examples.
While the matter at hand relates to the church and its religious minister
it does not ipso facto give the case a religious significance. Simply
stated, what is involved here is the relationship of the church as an
employer and the minister as an employee. It is purely secular and has
no relation whatsoever with the practice of faith, worship or doctrines
of the church. In this case, petitioner was not excommunicated or
expelled from the membership of the SDA but was terminated from
employment. Indeed, the matter of terminating an employee, which is
purely secular in nature, is different from the ecclesiastical act of
expelling a member from the religious congregation.

Under the Labor Code, the provision which governs the dismissal
of employees, is comprehensive enough to include religious
corporations, such as the SDA, in its coverage. Article 278 of the Labor
Code on post-employment states that the provisions of this Title shall
apply to all establishments or undertakings, whether for profit or not.
Obviously, the cited article does not make any exception in favor of a
religious corporation.

With this clear mandate, the SDA cannot hide behind the mantle
of protection of the doctrine of separation of church and state to avoid
its responsibilities as an employer under the Labor Code.

35
Page 36

Case Digests by: MARIELLE B. ANTONIO

REYES v. RTC MAKATI BRANCH 42


G.R. NO. 165744, August 11, 2008
Brion, J.

DOCTRINE:
JURISDICTION OF LABOR ARBITER (TERMINATION DISPUTE)
The nature of the present controversy is not one which may be
classified as an intra-corporate dispute and is beyond the jurisdiction
of the special commercial court to resolve.

FACTS:
Petitioner Oscar and private respondent Rodrigo are two of the
four children of the spouses Pedro and Anastacia Reyes. Pedro,
Anastacia, Oscar, and Rodrigo each owned shares of stock of Zenith
Insurance Corporation (Zenith), a domestic corporation established by
their family. With Anastacia's estate, which included her shareholdings
in Zenith, no settlement and partition appear to have been made.
Zenith and Rodrigo filed a complaint with the Securities and Exchange
Commission (SEC) against Oscar, stating that it is "a derivative suit
initiated and filed by the complainant Rodrigo to obtain an accounting
of the funds and assets of Zenith which are now or formerly in the
control, custody, and/or possession of respondent [herein petitioner
Oscar] and to determine the shares of stock of deceased spouses
Pedro and Anastacia Reyes that were arbitrarily and fraudulently
appropriated [by Oscar] for himself [and] which were not collated and
taken into account in the partition, distribution, and/or settlement of the
estate of the deceased spouses, for which he should be ordered to
account for all the income from the time he took these shares of stock,
and should now deliver to his brothers and sisters their just and
respective shares."

Oscar questioned the SEC's jurisdiction to entertain the complaint


because it pertains to the settlement of the estate of Anastacia Reyes.
When R.A. No. 87997 took effect, the SEC's exclusive and original
jurisdiction over cases enumerated in Section 5 of P.D. No. 902-A was
transferred to the RTC designated as a special commercial court. The
records of Rodrigo's SEC case were thus turned over to the RTC,
Branch 142, Makati. Oscar claimed that the complaint is a mere
nuisance or harassment suit and should, according to the Interim Rules
of Procedure for Intra-Corporate Controversies, be dismissed; and that
it is not a bona fide derivative suit as it partakes of the nature of a
petition for the settlement of estate of the deceased Anastacia that is
outside the jurisdiction of a special commercial court. The RTC denied
the motion in part.

36
Page 37

ISSUE:
Whether the trial court, sitting as a special commercial court, has
jurisdiction over the subject matter of Rodrigo's complaint

RULING:
NO. A review of relevant jurisprudence shows a development in
the Court's approach in classifying what constitutes an intra-corporate
controversy. Initially, the main consideration in determining whether a
dispute constitutes an intra-corporate controversy was limited to a
consideration of the intra-corporate relationship existing between or
among the parties.

The Court then combined the two tests: relationship and nature of
the controversy tests and declared that jurisdiction should be
determined by considering not only the status or relationship of the
parties, but also the nature of the question under controversy.

Insofar as the subject shares of stock (i.e., Anastacia's shares)


are concerned - Rodrigo cannot be considered a stockholder of Zenith.
Consequently, it cannot be declared that an intra-corporate
relationship exists that would serve as basis to bring this case within
the special commercial court's jurisdiction under Section 5(b) of PD
902-A, as amended. Rodrigo's complaint, therefore, fails the
relationship test.

More than the matters of injury and redress, what Rodrigo clearly
aims to accomplish through his allegations of illegal acquisition by
Oscar is the distribution of Anastacia's shareholdings without a prior
settlement of her estate - an objective that, by law and established
jurisprudence, cannot be done. The RTC of Makati, acting as a special
commercial court, has no jurisdiction to settle, partition, and distribute
the estate of a deceased.

It is clear that trial courts trying an ordinary action cannot resolve


to perform acts pertaining to a special proceeding because it is subject
to specific prescribed rules.

The nature of the present controversy is not one which may be


classified as an intra-corporate dispute and is beyond the jurisdiction
of the special commercial court to resolve. In short, Rodrigo's
complaint also fails the nature of the controversy test.

37
Page 38

Case Digests by: Gem Edward Aquino

Arsenio Locsin v. Nissan Lease Phil. Inc.


G.R. No. 185567. October 20, 2010.
Brion, J.

DOCTRINE:
JURISDICTION OF LABOR ARBITERS; INTRACORPORATE
DISPUTE
A corporate officer’s dismissal is always a corporate act, or an
intra-corporate controversy, so that the RTC should exercise
jurisdiction pursuant to Sec.5 of RA. 8799.

FACTS:
Petitioner Locsin was elected as Executive Vice President and
Treasurer (EVP/Treasurer) of respondent NCLPI. He held his position
for 13years, having been re-elected every year since 1992, until
January of 2005 where he was elected as the Chairman of the Board.
On August of 2005, the NCLPI Board held a special meeting for the
election of a new set of officers. Unfortunately, petitioner was neither
re-elected as Chairman nor as EVP/Treasurer. Petitioner then filed a
complaint for illegal dismissal with prayer of reinstatement against
respondent before the Labor Arbiter. Instead of filling its position paper,
respondent filed a motion to dismiss on the ground that the LA did not
have jurisdiction over the case since it involves an intra-corporate
dispute. The LA denied the motion of respondent, holding that it has
jurisdiction to decide the complaint finding extant an employer-
employee relationship. Respondent, thus, elevated the case to the CA
through a Rule 65 Petition.

In its decision, the CA reversed and set aside the LA’s order and
ruled that petitioner was a corporate officer citing PD 902-A which
defined corporate officers as those officers of a corporation who are
given that character either by the Corporation Code or the
corporation’s by-laws, and that the position of EVP/Treasurer is
specifically enumerated as an office in the corporation’s by-laws, and
that he only those functions that were specifically set forth in the By-
laws or required of him by the Board of Directors. Hence, this petition.
Petitioner submits that respondent wrongfully filed a Petition for
Certiorari before the CA, for the proper remedy is to appeal to the
NLRC the LA’s order, and that his relationship with respondent meets
the four-fold test.

ISSUE:
I.) Whether or not the Rule 65 Petition of respondent is proper.

38
Page 39

RULING:
YES. Prefatorily, we agree with Locsin's submission that the
NCLPI incorrectly elevated the Labor Arbiter's denial of the Motion to
Dismiss to the CA. As a general rule, an aggrieved party's proper
recourse to the denial is to file his position paper, interpose the grounds
relied upon in the motion to dismiss before the labor arbiter, and
actively participate in the proceedings. Thereafter, the labor arbiter's
decision can be appealed to the NLRC, not to the CA. The NLRC Rules
are clear: the denial by the labor arbiter of the motion to dismiss is not
appealable because the denial is merely an interlocutory order. From
this perspective, the CA clearly erred in the application of the
procedural rules by disregarding the relevant provisions of the NLRC
Rules, as well as the requirements for a petition for certiorari under the
Rules of Court. However, A strict implementation of the NLRC Rules
and the Rules of Court would cause injustice to the parties because
the Labor Arbiter clearly has no jurisdiction over the present intra-
corporate dispute. It was held in Lazaro v. CA that, “rules may be
relaxed only in exceptionally meritorious cases.”

Due to existing exceptional circumstances, the ruling on the merits


that Locsin is an officer and not an employee of Nissan must take
precedence over procedural considerations. First, the parties have
sufficiently ventilated their positions on the disputed employer-
employee relationship and have, in fact, submitted the matter for the
CA's consideration. Second, the CA correctly ruled that no employer-
employee relationship exists between Locsin and Nissan. Locsin was
undeniably Chairman and President, and was elected to these
positions by the Nissan board pursuant to its By-laws. As such, he was
a corporate officer, not an employee. Third. Even as Executive Vice-
President/Treasurer, Locsin already acted as a corporate officer
because the position of Executive Vice- President/Treasurer is
provided for in Nissan's By-Laws. Article IV, Section 4 of these By-
Laws specifically provides for this position.

An “office” is created by the charter of the corporation and the


officer is elected by the directors or stockholders. On the other hand,
an “employee” usually occupies no office and generally is employed
not by action of the directors or stockholders but by the managing
officers of the corporation who also determines the compensation to
be paid to such employee. In this case, petitioner was elected by the
NCLPI Board, in accordance with the Amened By-laws of the
corporation. Therefore, petitioner, at the time of his severance from the
NCLPI, was the latter’s corporate officer. Given Locsin's status as a
corporate officer, the RTC, not the Labor Arbiter or the NLRC, has
jurisdiction to hear the legality of the termination of his relationship with
Nissan. A corporate officer’s dismissal is always a corporate act, or an
intra-corporate controversy, so that the RTC should exercise

39
Page 40

jurisdiction pursuant to Sec.5 of RA. 8799. Based on the above


jurisdictional considerations, we would be forced to remand the case
to the Labor Arbiter for further proceedings if we were to dismiss the
petition outright due to the wrongful use of Rule 65.

We cannot close our eyes, however, to the factual and legal


reality, established by evidence already on record, that Locsin is a
corporate officer whose termination of relationship is outside a labor
arbiter's jurisdiction to rule upon. Under these circumstances, we have
to give precedence to the merits of the case, and primacy to the
element of jurisdiction. Jurisdiction is the power to hear and rule on a
case and is the threshold element that must exist before any quasi-
judicial officer can act. In the context of the present case, the Labor
Arbiter does not have jurisdiction over the termination dispute Locsin
brought, and should not be allowed to continue to act on the case after
the absence of jurisdiction has become obvious, based on the records
and the law. In more practical terms, a contrary ruling will only cause
substantial delay and inconvenience as well as unnecessary
expenses, to the point of injustice, to the parties.

40
Page 41

Case Digests by: Gem Edward Aquino

Wesleyan University-Philippines v. Guillermo Maglaya,


Sr.
G.R. No. 212774. January 23, 2017.
Peralta, J.

DOCTRINE:
JURISDICTION OF LABOR ARBITER; TERMINATION DISPUTE.
We held that one who is included in the by-laws of a corporation
in its roster of corporate officers is an officer of said corporation and
not a mere employee. The alleged "appointment" of Maglaya instead
of "election" as provided by the by-laws neither convert the president
of university as a mere employee, nor amend its nature as a corporate
officer.

FACTS:
Respondent Maglaya was appointed as a corporate member and
elected as a member of the Board of Trustees of WUP on January
2004, both for a period of 5years. On 2005, he was elected as the
President of the University for a five-year term, and was re-elected as
a trustee on 2007. However, on 2008, the incumbent Bishops of the
United Methodist Church appraised all the corporate members of the
expiration of their terms, unless renewed by the former, and on 2009,
respondent learned that the Bishops created an Ad Hoc Committee to
plan the efficient and orderly turnover of the administration of WUP and
the appointment of incoming corporate members and trustees.
Respondent was informed of the termination of his services and
authority as the President of the University on April 2009.

Respondent filed an illegal dismissal case against WUP before the


Labor Arbiter, claiming that he was unceremoniously dismissed in a
wanton, reckless, oppressive and malevolent manner and in evident
bad faith in disregarding his five-year term of office. Petitioner, on the
other hand, claimed that the dismissal of respondent, being a corporate
officer, is an intra-corporate controversy under the jurisdiction of the
RTC. In its decision, the LA, ruled in favor of petitioner and held that
the employer-employee relation between the parties is merely
incidental and that the case involves an intra-corporate dispute beyond
the jurisdiction of the labor tribunal. The NLRC, on appeal, reversed
and set aside the LA’s decision on the ground that the Board did not
elect respondent, but merely appointed him, and that based on the
control test, Maglaya is an employee of petitioner. On appeal before
the CA, the petition was dismissed for procedural matters. Hence, this
petition.

41
Page 42

ISSUE:
Whether or not respondent Maglaya is a corporate officer.

RULING:
YES. "Corporate officers" in the context of Presidential Decree No.
902-A are those officers of the corporation who are given that character
by the Corporation Code or by the corporation's bylaws. The president,
vice-president, secretary and treasurer are commonly regarded as the
principal or executive officers of a corporation, and they are usually
designated as the officers of the corporation. However, other officers
are sometimes created by the charter or by-laws of a corporation, or
the board of directors may be empowered under the bylaws of a
corporation to create additional offices as may be necessary. This
Court expounded that an "office" is created by the charter of the
corporation and the officer is elected by the directors or stockholders,
while an "employee" usually occupies no office and generally is
employed not by action of the directors or stockholders but by the
managing officer of the corporation who also determines the
compensation to be paid to such employee. From the foregoing, that
the creation of the position is under the corporation's charter or by-
laws, and that the election of the officer is by the directors or
stockholders must concur in order for an individual to be considered a
corporate officer, as against an ordinary employee or officer.

Under Sec. 2, Art. VI of the By-Laws of WUP, it is provided that


the President of the Wesleyan University-Philippines shall be honorary
members of the Board, and Sec.6, Art. VIII specifically mentions the
President as an officer of the corporation. It is apparent from the By-
laws of WUP that the president was one of the officers of the
corporation, and was an honorary member of the Board. He was
appointed by the Board and not by a managing officer of the
corporation. We held that one who is included in the by-laws of a
corporation in its roster of corporate officers is an officer of said
corporation and not a mere employee. The alleged "appointment" of
Maglaya instead of "election" as provided by the by-laws neither
convert the president of university as a mere employee, nor amend its
nature as a corporate officer. With the office specifically mentioned in
the by-laws, the NLRC erred in taking cognizance of the case, and in
concluding that Maglaya was a mere employee. A corporate officer's
dismissal is always a corporate act, or an intracorporate controversy
which arises between a stockholder and a corporation, and the nature
is not altered by the reason or wisdom with which the Board of
Directors may have in taking such action. To emphasize, the
determination of the rights of a corporate officer dismissed from his
employment, as well as the corresponding liability of a corporation, if
any, is an intra-corporate dispute subject to the jurisdiction of the
regular courts.

42
Page 43

Case Digests by: Gem Edward Aquino

Norma Cacho and North Star International Travel, Inc. v.


Virginia Balagtas
G.R. No. 202974 . February 7, 2018.
Leonardo-De Castro. J.

DOCTRINE:
JURISDICTION OF LABOR ARBITER; TERMINATION DISPUTE
A two-tier test must be employed to determine whether an intra-
corporate controversy exists in the present case, viz.: (a) the
relationship test, and (b) the nature of the controversy test.

FACTS:
Respondent Balagtas filed a complaint for constructive dismissal
against petitioner North Star and its President Cacho before the Labor
Arbiter. Respondent alleged that she was one of the original
incorporators-directors of North Star, and when it started its operations
in 1990, she was the General Manager and later became the Executive
Vice President/Chief Executive Officer. After 14 years of service,
respondent was placed under 30days of preventive suspension
pursuant to a Board Resolution due to her alleged questionable
transactions. While under preventive suspension, respondent wrote a
letter to petitioner informing the latter that she was assuming her
position as the EVP/CEO, however, she was prevented for doing so.
Respondent also inquired about the status of investigation but was
ignored. Thus, respondent filed a complaint claiming that she was
constructively and illegally dismissed. Petitioner, for their part, asserted
that respondent was not illegally dismissed but was merely placed
under preventive suspension.

The LA, in its decision, found that respondent was illegally


dismissed from service, and ordered petitioner to pay separation pay
and damages. On appeal to the NLRC, petitioner maintained that
respondent was never dismissed and that she was a corporate officer,
and a member of North Star’s Board of Directors, making the case an
intracorporate controversy, which falls within the jurisdiction of the
ordinary courts. The NLRC, in its decision, ruled in favor of petitioner
and dismissed the complaint for lack of jurisdiction. However, on
appeal before the CA, the CA reversed and set aside the NLRC’s
decision and found that respondent is a mere employee of petitioner
applying the two-tier test. The CA held that petitioners By-Laws
indicated the position of “vice president” as one of its corporate officers,
however, respondent was a “executive vice president”. Hence, this
petition.

43
Page 44

ISSUE:
Whether or not respondent the present case is an intra-corporate
controversy.

RULING:
YES. At the onset, We agree with the appellate court's ruling that
a two-tier test must be employed to determine whether an intra-
corporate controversy exists in the present case, viz.: (a) the
relationship test, and (b) the nature of the controversy test.

Under the relationship test, a dispute is considered an intra-


corporate controversy under the relationship test when the relationship
between or among the disagreeing parties is any one of the following:
(a) between the corporation, partnership, or association and the public;
(b) between the corporation, partnership, or association and its
stockholders, partners, members, or officers; (c) between the
corporation, partnership, or association and the State as far as its
franchise, permit or license to operate is concerned; and (d) among the
stockholders, partners, or associates themselves. One shall be
considered a corporate officer only if two conditions are met, viz.: (1)
the position occupied was created by charter/by-laws, and (2) the
officer was elected (or appointed) by the corporation's board of
directors to occupy said position.

The rule is that corporate officers are those officers of a


corporation who are given that character either by the Corporation
Code or by the corporation's by-laws. Under Sec. 1, Art. IV of North
Star’s By-Laws, it states that there shall be “one or more Vice-
President”. Clearly, there may be one or more vice president positions
in petitioner North Star and, by virtue of its by-laws, all such positions
shall be corporate offices. Consequently, the next question that begs
to be asked is whether or not the phrase "one or more vice president"
in the above-cited provision of the by-laws includes the Executive Vice
President position held by respondent Balagtas. The CA is of the
opinion that the position should be the exact and complete name of the
position that appears in the by-laws. The appellate court's
interpretation of the phrase "one or more vice president" unduly
restricts one of petitioner North Star's inherent corporate powers, viz.:
to adopt its own by-laws, provided that it is not contrary to law, morals,
or public policy for its internal affairs. Thus, by name, the Executive
Vice President position is embraced by the phrase "one or more vice
president" in North Star's by-laws.

Under the Nature of Controversy test, the disagreement must not


only be rooted in the existence of an intra-corporate relationship, but
must as well pertain to the enforcement of the parties' correlative rights
and obligations under the Corporation Code and the internal and intra-

44
Page 45

corporate regulatory rules of the corporation. Verily, in a long line of


cases, the Court consistently ruled that a corporate officer's dismissal
is always a corporate act, or an intra-corporate controversy which
arises between a stockholder and a corporation. However, a closer
look at these cases will reveal that the intra-corporate nature of the
disputes therein did not hinge solely on the fact that the subject of the
dismissal was a corporate officer. In other words, the dismissal must
relate to any of the circumstances and incidents surrounding the
parties' intra-corporate relationship.

To be considered an intra-corporate controversy, the dismissal of


a corporate officer must have something to do with the duties and
responsibilities attached to his/her corporate office or performed in
his/her official capacity. In respondent Balagtas's Position Paper filed
before the Labor Arbiter she alleged, among others, that: i) the Board
did not authorize either her suspension and removal from office; and
(ii) as a result of her illegal dismissal, she is entitled to separation pay
in lieu of her reinstatement to her previous positions, plus back wages,
allowances, and other benefits. The foregoing allegations mainly relate
to incidents involving her capacity as Executive Vice President. On the
other hand, petitioners Cacho and North Star terminated respondent
Balagtas for the following reasons: (a) for allegedly appropriating
company funds for her personal gain; (b) for abandonment of work; (c)
violation of a lawful order of the corporation; and (d) loss of trust and
confidence. From these, it is clear that the termination complained of
is intimately and inevitably linked to respondent Balagtas's role as
petitioner North Star's Executive Vice President. That all these
incidents are adjuncts of her corporate office lead the Court to
conclude that respondent Balagtas's dismissal is an intra-corporate
controversy, not a mere labor dispute.

45
Page 46

Case Digests by: Gem Edward Aquino

Rosalinda Paredes v. Feed the Children Philippine, Inc.


G.R. No. 184397. September 9, 2015.
Peralta. J.

DOCTRINE:
JURISDICTION OF LABOR ARBITERS; DAMAGES.
Money claims of workers" referred to in Article 217 of the Labor
Code embraces money claims which arise out of or in connection with
the employer-employee relationship, or some aspect or incident of
such relationship.

FACTS:
Petitioner Paredes was FTCP’s National Director and an ex officio
member of the Board, thus was always present in every meeting. On
August 2005, 42 FTCP employees signed a petition letter addressed
to the Board expressing their complaints against alleged detestable
practices of petitioner. When the Board convened to discuss the
animosity between petitioner and the staff it decided that there shall be
a supervisory team that shall draw a definite work plan and an
independent professional management and financial auditor shall be
hired. Petitioners’ lawyers questioned the legality of the audit
demanded that they should desist from conducting the same. While
petitioner was at an orientation, she received a phone call from her
staff that the auditors were already at her office, and was instructed to
meet the auditors. Petitioner refrained from obeying the order and
demanded that due process be observed. The Board, thus, resolved
to suspend petitioner because of her indifferent attitude and unjustified
refusal to submit an audit. However, before it could be implemented,
respondent received petitioners’ resignation letter where she reasoned
that she found it no longer tenable to work with the Board. Thereafter,
petitioner filed a complaint for illegal dismissal, claiming that she was
forced to resign, thus, was constructively dismissed.

The LA, in its decision, dismissed the complaint for lack of merit
and ordered petitioner to pay its accountabilities, moral damages, and
exemplary damages to respondent. The NLRC, on appeal, reversed
and set aside the LA’s decision and ruled in favor of petitioner.
However, on appeal before the CA, the CA reversed the NLRC’s order
and ruled in favor of respondent, ordering petitioner to pay to the former
her unpaid loans, disbursements, and withdrawals. Hence, this
petition. Petitioner questions, among others, the order of the CA
directing her to pay damages to respondent that clearly did not arise
from an employer-employee relationship

46
Page 47

ISSUE:
Whether or not petitioner may be ordered to pay the money
claims of respondent.

RULING:
NO. This Court held that the "money claims of workers" referred
to in Article 217 of the Labor Code embraces money claims which arise
out of or in connection with the employer-employee relationship, or
some aspect or incident of such relationship. Applying the rule of
noscitur a sociis in clarifying the scope of Article 217, it is evident that
paragraphs 1 to 5 refer to cases or disputes arising out of or in
connection with an employer-employee relationship. In other words,
the money claims within the original and exclusive jurisdiction of labor
arbiters are those which have some reasonable causal connection with
the employer-employee relationship. This claim is distinguished from
cases of actions for damages where the employer-employee
relationship is merely incidental and the cause of action proceeds from
a different source of obligation. Thus, the regular courts have
jurisdiction where the damages claimed for were based on: tort,
malicious prosecution, or breach of contract, as when the claimant
seeks to recover a debt from a former employee or seeks liquidated
damages in the enforcement of a prior employment contract. By the
designating clause "arising from the employer-employee relations,"
Article 217 applies with equal force to the claim of an employer for
actual damages against its dismissed employee, where the basis for
the claim arises from or is necessarily connected with the fact of
termination, and should be entered as a counterclaim in the illegal
dismissal case.

In this case, the CA erred in awarding for petitioner's unpaid debt


to respondents. The claim for recovery of a debt has no reasonable
causal connection with any of the claims provided for in Article 217.
The fact that the transaction happened at the time they were employer
and employee did not negate the civil jurisdiction of trial court. Hence,
it is erroneous for the LA and the CA to rule on such claim arising from
a different source of obligation and where the employer-employee
relationship was merely incidental.

47
Page 48

Case Digests by: Gem Edward Aquino

Gilda Lunzaga v. Albar Shipping and Trading Corp.


G.R. No. 200476. April 18, 2012.

DOCTRINE:
JURISDICTION OF LABOR ARBITER; DAMAGES.
And while the Labor Arbiter has no jurisdiction to determine who
among the alleged heirs is entitled to receive Romeo's death benefits,
it should have made a ruling holding Albar liable for the claim.

FACTS:
Rome Lunzaga was a seaman working for respondent Albar.
Romeo was assigned as the Chief Engineer on board the vessel MV
Lake Aru, however, Romeo suffered a heart attack and was repatriated
to the Philippines only to die later on. Petitioner Gilda, claiming to be
the surviving spouse of Romeo filed with the NLRC a complaint for
payment of death benefits, damages and attorney’s fees against
respondent. However, Romeo’s children from his first marriage
claimed that Gilda is not entitled to the death benefits of Romeo,
because their marriage was bigamous. Respondent signified its
willingness to pay the death benefits, however, Gilda and the children
of Romeo could not agree as to the sharing of the benefits.

The LA issued an Order temporarily dismissing the complaint and


directing the parties to filed their case with the regular courts. Petitioner
filed an appeal before the NLRC but was dismissed for having been
filed beyond the reglementary period. On appeal, the CA dismissed
petitioners’ appeal for being devoid of merit. Hence, this petition.

ISSUE:
Whether or not the LA erred in dismissing the complaint of
petitioner.

RULING:
YES. We agree with the pronouncement of the Labor Arbiter and
the CA that the issue of who is the proper beneficiary of Romeo is
properly within the jurisdiction of the regular courts. However, this is
not the only issue in the instant petition. A review of the records of the
case reveals that the main issue in the complaint before the Labor
Arbiter was whether the heirs of Romeo are entitled to receive his
death benefits from Albar. Clearly, the Labor Arbiter has jurisdiction
over this issue and the case itself, involving as it does a claim arising

48
Page 49

from an employer-employee relationship. And while the Labor Arbiter


has no jurisdiction to determine who among the alleged heirs is entitled
to receive Romeo's death benefits, it should have made a ruling
holding Albar liable for the claim.

Verily, Albar is liable to the heirs of Romeo. Albar hereby is


ordered to deposit this amount in an escrow account under the control
of the NLRC in order to protect the interests of Romeo's heirs. The
parties claiming to be the beneficiaries of Romeo are directed to file
the appropriate action with a trial court to determine the true and legal
heirs of Romeo entitled to receive the disability benefits. The amount
in the escrow account will only be released to the legal heirs per the
decision of a trial court

49
Page 50

Case Digests by: Gem Edward Aquino

Ma. Isabel Santos v. Servier Philippines, Inc.


G.R. No. 166377. November 28, 2008.
Nachura, J.

DOCTRINE:
JURISDICTION OF LABOR ARBITER; TAX DEDUCTION
The issue of deduction for tax purposes is intertwined with the
main issue of whether or not petitioner's benefits have been fully given
her. It is, therefore, a money claim arising from the employer-employee
relationship, which clearly falls within the jurisdiction of the Labor
Arbiter and the NLRC

FACTS:
Petitioner Santos was the Human Resource Manager of
respondent Servier. On 1998, petitioner attended a meeting of all
human resource managers of respondent, held in Paris, France. While
in Paris, petitioner, together with her husband and son, had dinner in a
restaurant which was known for mussels as their specialty. However,
petitioner was brought to the hospital and fell into coma for 21 days
and stayed at the ICU for 52 days due to alimentary allergy. All
expenses for the hospitalization as well as the expenses for their stay
were paid by respondent, who also continued to pay petitioners
salaries. When petitioner was then transferred to the St. Luke’s
Medical Center, respondents’ physicians concluded that petitioner had
not fully recovered and is not fit to resume for work, prompting
respondent to terminate petitioners’ services. As a consequence of
such termination, respondent offered a retirement package, where the
benefits amounted to P1,063,842. However, only P701,454.89 was
released to petitioner, the balance thereof was withheld by respondent
allegedly for tax purposes. Petitioner, thus, instituted a complaint for,
among others, the unpaid balance of the retirement package.

The LA, in its decision, dismissed petitioners’ complaint. The


arbiter refused to rule on the legality of the deductions made by
respondent, on the ground that the issue was beyond the jurisdiction
of the NLRC. Hence, this petition.

ISSUE:
Whether or not the LA has jurisdiction over deductions made in a
retirement package for tax purposes.

RULING:

50
Page 51

YES. Contrary to the Labor Arbiter and NLRC's conclusions,


petitioner's claim for illegal deduction falls within the tribunal's
jurisdiction. It is noteworthy that petitioner demanded the completion of
her retirement benefits, including the amount withheld by respondent
for taxation purposes. The issue of deduction for tax purposes is
intertwined with the main issue of whether or not petitioner's benefits
have been fully given her. It is, therefore, a money claim arising from
the employer-employee relationship, which clearly falls within the
jurisdiction of the Labor Arbiter and the NLRC.

51
Page 52

Case Digests by: Gem Edward Aquino

World’s Best Gas, Inc. v. Henry Vital


G.R. No. 211588. September 9, 2015.
Perlas-Bernabe, J.

DOCTRINE:
JURISDICTION OF LABOR ARBITER; DAMAGES
Having no subject matter jurisdiction to resolve claims arising from
employer-employee relations, the RTC's ruling on Vital's claim of
P845,000.00 and P250,000.00 in unpaid salaries and separation pay
is, thus, null and void, and therefore, cannot perpetuate even if affirmed
on appeal.

FACTS:
Respondent Vital was on of the incorporators of petitioner WBGI,
holding P500,000 worth of shares of stock therein. As a separate
business venture, respondent sourced LPG from petitioner and
distributed the same through ERJ Enterprises owned by them. As of
respondents last statement of account, their outstanding balance with
petitioner for unpaid LPG amounted to P923,843. On 2003, respondent
reached the age of mandatory retirement, petitioner thus computed his
retirement benefits at P82,500 and agreed to acquire Vitals’ P500,000
shares of stock at par value. Respondent claimed that he is entitled to
the unpaid salaries and separation pay due to him amounting to
P845,000 and P250,000. After offsetting the P500,000 due from ERJ’s
outstanding balance, respondent claimed that the net amount of
P671,156 was payable to him. However, petitioner rejected such claim
and contended that it is Vital who actually owns them P369,156.
Respondent, then, filed a complaint before the NLRC for non-payment
of separation and retirement benefits, and illegal reduction of salary,
benefits, and damages. For its part, petitioner argued that the LA had
no jurisdiction over the case because respondent is not an employee,
but a mere incorporator.

The LA, in its decision, found that the case is in the nature of
intracorporate controversy and dismissed the same for lack of
jurisdiction. This prompted respondent to file his complaint before the
RTC. In its decision, the RTC declared that it had jurisdiction to rule on
the main intracorporate controversy together with the question of
employment benefits and found that Vital was actually an employee of
petitioner and upheld his claims. However, the RTC offset the claims
against the amount payable to petitioner from ERJ enterprises. On
appeal, the CA affirmed the RTC’s decision. Hence, this petition.

52
Page 53

ISSUE:
Whether or not the RTC had jurisdiction over respondents claim
of separation pay and unpaid salaries.

RULING:
NO. At the outset, it should be pointed out that the instant case
actually involves three (3) distinct causes of action, namely, (1) Vital's
claim for P845,000.00 and P250,000.00 in unpaid salaries and
separation pay; (2) the P923,843.59 in arrearages payable to WBGI
from ERJ Enterprises; and (3) Vital's claim of P500,000.00 due from
WBGI's acquisition of Vital's shares of stocks. All of the foregoing were
threshed out by the RTC in its Decision, and effectively upheld by the
CA on appeal. However, the RTC's adjudication of the first cause of
action was improper since the same is one which arose from Vital and
WBGI's employer-employee relations, involving an amount exceeding
P5,000.00, hence, belonging to the jurisdiction of the labor arbiters
pursuant to Article 217 (6) of the Labor Code.

Having no subject matter jurisdiction to resolve claims arising from


employer-employee relations, the RTC's ruling on Vital's claim of
P845,000.00 and P250,000.00 in unpaid salaries and separation pay
is, thus, null and void, and therefore, cannot perpetuate even if affirmed
on appeal, rendering the CA's ratiocination that it "has the eventual
authority to review the labor courts' decision on the matter" direly infirm.
Contrary to its lack of jurisdiction over claims arising from employer-
employee relations, the RTC has: (a) general jurisdiction to adjudicate
on the P923,843.59 in arrearages payable to WBGI from ERJ
Enterprises, which was admitted by Vital but not claimed by WBGI; and
(b) special jurisdiction, as a special commercial court, to adjudicate on
Vital's claim of P500,000.00 from WBGI's acquisition of his shares of
stocks.

53
Page 54

Case Digests by: Gem Edward Aquino

Patricia Halaguena, Et Al., v. PAL


G.R. No. 172013. October 2, 2009.
Peralta. J.

DOCTRINE:
JURISDICTION OF LABOR ARBITER; DAMAGES.
Thus, where the principal relief sought is to be resolved not by
reference to the Labor Code or other labor relations statute or a
collective bargaining agreement but by the general civil law, the
jurisdiction over the dispute belongs to the regular courts of justice and
not to the labor arbiter and the NLRC.

FACTS:
Petitioners were employed as female flight attendants of
respondent and members of Flight Attendants and Stewards
Association of the Philippines (FASAP), the sole and exclusive
bargaining representative for flight attendants of respondent. On 2001,
FASAP and respondent entered into a CBA, Sec. 144 Part A thereof,
provides that the compulsory age of retirement for females shall be 55
and 60 for males. On 2003, petitioners manifested that the
aforementioned provision in their CBA is discriminatory and demanded
an equal treatment with their male counterparts. Unheeded, petitioners
filed a Special Civil Action for Declaratory Relief with prayer for the
issuance of TRO/WPI before the RTC for the invalidity of the subject
provision.

The RTC, then issued an Order upholding its jurisdiction over the
case. It held that the allegations in the petition do not make out a labor
dispute. Thereafter, it issued another Order directing the issuance of a
WPI enjoining respondent from implementing Sec. 144 of the CBA.
Aggrieved, respondent filed a Petition for Certiorari and Prohibition
before the CA alleging that the RTC had no jurisdiction over the case.
In its decision, the CA granted the petition and dismissed the case.
Hence, this petition.

ISSUE:
I) Whether or not the regular courts have jurisdiction over the
present case.

II) Whether or not the case should first go through the agreed
grievance machinery

54
Page 55

RULING:
I) YES. Jurisdiction of the court is determined on the basis of the
material allegations of the complaint and the character of the relief
prayed for irrespective of whether plaintiff is entitled to such relief.
From the petitioners' allegations and relief prayed for in its petition, it is
clear that the issue raised is whether Section 144, Part A CBA is
unlawful and unconstitutional. The subject of litigation is incapable of
pecuniary estimation, exclusively cognizable by the RTC, pursuant to
Section 19 (1) of Batas Pambansa Blg. 129, as amended. Being an
ordinary civil action, the same is beyond the jurisdiction of labor
tribunals. The said issue cannot be resolved solely by applying the
Labor Code. Rather, it requires the application of the Constitution,
labor statutes, law on contracts and the Convention on the Elimination
of All Forms of Discrimination Against Women, and the power to apply
and interpret the Constitution and CEDAW is within the jurisdiction of
trial courts, a court of general jurisdiction. Not every controversy or
money claim by an employee against the employer or vice-versa is
within the exclusive jurisdiction of the labor arbiter. Actions between
employees and employer where the employer-employee relationship
is merely incidental and the cause of action precedes from a different
source of obligation is within the exclusive jurisdiction of the regular
court. Here, the employer-employee relationship between the parties
is merely incidental and the cause of action ultimately arose from
different sources of obligation, i.e., the Constitution and CEDAW.

Thus, where the principal relief sought is to be resolved not by


reference to the Labor Code or other labor relations statute or a
collective bargaining agreement but by the general civil law, the
jurisdiction over the dispute belongs to the regular courts of justice and
not to the labor arbiter and the NLRC. In such situations, resolution of
the dispute requires expertise, not in labor management relations nor
in wage structures and other terms and conditions of employment, but
rather in the application of the general civil law. Clearly, such claims
fall outside the area of competence or expertise ordinarily ascribed to
labor arbiters and the NLRC and the rationale for granting jurisdiction
over such claims to these agencies disappears.

II) NO. This Court holds that the grievance machinery and
voluntary arbitrators do not have the power to determine and settle the
issues at hand. They have no jurisdiction and competence to decide
constitutional issues relative to the questioned compulsory retirement
age. Their exercise of jurisdiction is futile, as it is like vesting power to
someone who cannot wield it.

Although the CBA provides for a procedure for the adjustment of


grievances, such referral to the grievance machinery and thereafter to
voluntary arbitration would be inappropriate to the petitioners, because

55
Page 56

the union and the management have unanimously agreed to the terms
of the CBA and their interest is unified. In the same vein, the dispute in
the case at bar is not between FASAP and respondent PAL, who have
both previously agreed upon the provision on the compulsory
retirement of female flight attendants as embodied in the CBA. The
dispute is between respondent PAL and several female flight
attendants who questioned the provision on compulsory retirement of
female flight attendants. Thus, applying the principle in the
aforementioned case cited, referral to the grievance machinery and
voluntary arbitration would not serve the interest of the petitioners.

56
Page 57

Case Digests by: Gem Edward Aquino

Pepsi Cola Distributors of the Philippines, Inc. v. Hon.


Lolita Gal-Lang
G.R. No. 89621. September 24, 1991.
Cruz, J.

DOCTRINE:
JURISDICTION OF LABOR ARBITER; DAMAGES.
It must be stressed that not every controversy involving workers
and their employers can be resolved only by the labor arbiters. This
will be so only if there is a "reasonable causal connection" between the
claim asserted and employee-employer relations to put the case under
the provisions of Article 217. Absent such a link, the complaint will be
cognizable by the regular courts of justice in the exercise of their civil
and criminal jurisdiction.

FACTS:
Private respondents were employees of petitioner who were
suspected of complicity in the irregular disposition of empty Pepsi Cola
bottles. Petitioners then filed a criminal complaint theft against private
respondents but was later substituted with a criminal complaint for
falsification of private documents. After preliminary investigation, the
complaint was dismissed by the MTC and was affirmed by the Office
of the Provincial Prosecutor. In the meantime, allegedly after an
administrative investigation, private respondents were dismissed by
petitioner. As a result, they filed a complaint for illegal dismissal before
the NLRC and a separate civil complaint for damages for malicious
prosecution. Petitioners moved to dismiss the civil complaint on the
ground that the trial court had no jurisdiction over the case because it
involved employee-employer relations that were exclusively
cognizable by the LA.

The motion was initially granted, but was reversed on a motion for
reconsideration. The trial court held that the civil case was distinct from
the labor case pending before the labor courts. Hence, this petition for
relief.

ISSUE:
Whether or not the labor courts have jurisdiction over the civil
complaint of private respondents.

RULING:

57
Page 58

NO. It must be stressed that not every controversy involving


workers and their employers can be resolved only by the labor arbiters.
This will be so only if there is a "reasonable causal connection"
between the claim asserted and employee-employer relations to put
the case under the provisions of Article 217. Absent such a link, the
complaint will be cognizable by the regular courts of justice in the
exercise of their civil and criminal jurisdiction.

In Medina v. Castro-Bartolome, it was held that: “It is obvious from


the complaint that the plaintiffs have not alleged any unfair labor
practice. Theirs is a simple action for damages for tortuous acts
allegedly committed by the defendants. Such being the case, the
governing statute is the Civil Code and not the Labor Code.” The latest
ruling on this issue is found in San Miguel Corporation v. NLRC, where
it was held that:

“It is the character of the principal relief sought that appears


essential, in this connection. Where such principal relief is to be
granted under labor legislation or a collective bargaining agreement,
the case should fall within the jurisdiction of the Labor Arbiter and the
NLRC, even though a claim for damages might be asserted as an
incident to such claim. xxx xxx xxx Where the claim to the principal
relief sought is to be resolved not by reference to the Labor Code or
other labor relations statute or a collective bargaining agreement but
by the general civil law, the jurisdiction over the dispute belongs to the
regular courts of justice and not to the Labor Arbiter and the NLRC. xxx
xxx xxx While paragraph 3 above refers to "all money claims of
workers," it is not necessary to suppose that the entire universe of
money claims that might be asserted by workers against their
employers has been absorbed into the original and exclusive
jurisdiction of Labor Arbiters. xxx xxx xxx The Court, therefore, believes
and so holds that the "money claims of workers" referred to in
paragraph 3 of Article 217 embraces money claims which arise out of
or in connection with the employer- employee relationship, or some
aspect or incident of such relationship.”

The case now before the Court involves a complaint for damages
for malicious prosecution which was filed with the Regional Trial Court
by the employees of the defendant company. It does not appear that
there is a "reasonable causal connection" between the complaint and
the relations of the parties as employer and employees. The complaint
did not arise from such relations and in fact could have arisen
independently of an employment relationship between the parties. No
such relationship or any unfair labor practice is asserted. What the
employees are alleging is that the petitioners acted with bad faith when
they filed the criminal complaint which the Municipal Trial Court said
was intended "to harass the poor employees". This is a matter which

58
Page 59

the labor arbiter has no competence to resolve as the applicable law is


not the Labor Code but the Revised Penal Code.

59
Page 60

Case Digests by: Gem Edward Aquino

Bebiano Bañez v. Hon. Downey Valdevilla & Oro


Marketing Inc.
G.R. No. 128024. May 9, 2000.
Gonzaga-Reyes, J.

DOCTRINE:
JURISDICTION OF LABOR ARBITER; DAMAGES.
Whereas this Court in a number of occasions had applied the
jurisdictional provisions of Article 217 to claims for damages filed by
employees, we hold that by the designating clause "arising from the
employer-employee relations" Article 217 should apply with equal force
to the claim of an employer for actual damages against its dismissed
employee, where the basis for the claim arises from or is necessarily
connected with the fact of termination, and should be entered as a
counterclaim in the illegal dismissal case.

FACTS:
Petitioner was the sales manages of private respondent in its
branch in Iligan City. Petitioner was indefinitely suspended, prompting
him to file a complaint for illegal dismissal before the NLRC against
private respondent. In its decision, the LA found petitioner to have been
illegally dismissed and ordered private respondent to pay the former
separation pay. The case was appealed to the NLRC but was later
dismissed, thus was elevated to the SC through a petition for certiorari.
The SC, in its decision, dismissed the appeal on technical grounds but
pointed out that there was no grave abuse of discretion on the part of
the NLRC. Thereafter, private respondent filed a complaint for
damages before the RTC against petitioner for interest as loss of profit
and/or unearned income and estimated cost of office supplies.
Petitioner filed a motion to dismiss the complaint arguing that the action
having arisen from an employer-employee relationship, was squarely
under the exclusive original jurisdiction of the NLRC and is barred by
final judgement in the labor case.

Respondent judge, in ruling upon the motion to dismiss ipheld its


jurisdiction over the subject matter. Respondent held that a perusal of
the complaint which is for damages does not ask for any relief under
the Labor Code. It seeks to recover damages as redress for petitioners
breach of his contractual obligation to respondent who was damaged
and prejudiced. Hence, this petition.

ISSUE:

60
Page 61

Whether or not the RTC have jurisdiction over the complaint for
damages.

RULING:
NO. Article 217(a), paragraph 4 of the Labor Code, was already in
effect at the time of the filing of this case. The above provisions are a
result of the amendment by Section 9 of R.A. No. 6715, which took
effect on March 21, 1989. Presently, the jurisdiction of Labor Arbiters
and the NLRC in Article 217 is comprehensive enough to include
claims for all forms of damages "arising from the employer-employee
relations. Whereas this Court in a number of occasions had applied the
jurisdictional provisions of Article 217 to claims for damages filed by
employees, we hold that by the designating clause "arising from the
employer-employee relations" Article 217 should apply with equal force
to the claim of an employer for actual damages against its dismissed
employee, where the basis for the claim arises from or is necessarily
connected with the fact of termination, and should be entered as a
counterclaim in the illegal dismissal case. There is no mistaking the
fact that in the case before us, private respondent's claim against
petitioner for actual damages arose from a prior employer-employee
relationship. In the first place, private respondent would not have taken
issue with petitioner's "doing business of his own" had the latter not
been concurrently its employee. Thus, the damages alleged in the
complaint below are: first, those amounting to lost profits and earnings
due to petitioner's abandonment or neglect of his duties as sales
manager, having been otherwise preoccupied by his unauthorized
installment sale scheme; and second, those equivalent to the value of
private respondent's property and supplies which petitioner used in
conducting his "business".

Second, and more importantly, to allow respondent court to


proceed with the instant action for damages would be to open anew
the factual issue of whether petitioner's installment sale scheme
resulted in business losses and the dissipation of private respondent's
property. This issue has been duly raised and ruled upon in the illegal
dismissal case, where private respondent brought up as a defense the
same allegations now embodied in his complaint, and presented
evidence in support thereof. The Labor Arbiter, however, found to the
contrary — that no business losses may be attributed to petitioner as
in fact. In other words, the issue of actual damages has been settled
in the labor case, which is now final and executory. In Ebon vs. de
Guzman, this Court discussed: Evidently, the lawmaking authority had
second thoughts about depriving the Labor Arbiters and the NLRC of
the jurisdiction to award damages in labor cases because that setup
would mean duplicity of suits, splitting the cause of action and possible
conflicting findings and conclusions by two tribunals on one and the
same claim.” Clearly, respondent court's taking jurisdiction over the

61
Page 62

instant case would bring about precisely the harm that the lawmakers
sought to avoid in amending the Labor Code to restore jurisdiction over
claims for damages of this nature to the NLRC.

This is, of course, to distinguish from cases of actions for damages


where the employer-employee relationship is merely incidental and the
cause of action proceeds from a different source of obligation. Thus,
the jurisdiction of regular courts was upheld where the damages,
claimed for were based on tort, malicious prosecution, or breach of
contract, as when the claimant seeks to recover a debt from a former
employee or seeks liquidated damages in enforcement of a prior
employment contract. Neither can we uphold the reasoning of
respondent court that because the resolution of the issues presented
by the complaint does not entail application of the Labor Code or other
labor laws, the dispute is intrinsically civil. Article 217(a) of the Labor
Code, as amended, clearly bestows upon the Labor Arbiter original and
exclusive jurisdiction over claims for damages arising from employer-
employee relations — in other words, the Labor Arbiter has jurisdiction
to award not only the reliefs provided by labor laws, but also damages
governed by the Civil Code.

62
Page 63

Case Digests by: Gem Edward Aquino

Emer Milan, Et Al., v. NLRC & Solid Mills, Inc.


G.R. No. 202961. February 4, 2015.
Leonen, J.

DOCTRINE:
JURISDICTION OF LABOR ARBITER; DAMAGES.
As a general rule, therefore, a claim only needs to be sufficiently
connected to the labor issue raised and must arise from an employer-
employee relationship for the labor tribunals to have jurisdiction.

FACTS:
Petitioners are respondents’ employees. Being as such,
petitioners and their families were allowed to occupy SMI Village, a
property owned by respondent, out of liberality and for the convenience
of petitioners, on the condition that they would vacate the premises
anytime respondent deems fit. However, on 2003, respondent was
forced to cease its operations due to serious business losses, and was
recognized by petitioners bargaining unit, thus, a memorandum of
agreement provided for respondents grant of separation pay less
accountabilities to petitioners. Later, respondent sent to petitioners
notices to vacate SMI village, and were required to sign a
memorandum with release and quitclaim before their separation pay
and other benefits would be released. Employees who sign the
memorandum were considered to have agreed to vacate SMI village
as a condition for the release of their separation pay. Petitioner refused
to sign the documents and demanded to be paid their separation pay,
thus, they filed complaints before the LA for alleged non-payment of
separation pay and other benefits.

In its decision, the LA ruled in favor of petitioners. According to the


LA, respondent illegally withheld petitioners’ separation pay and
benefits. It also held that, Petitioners' possession should not be
construed as petitioners' "accountabilities" that must be cleared first
before the release of benefits. Their possession "is not by virtue of any
employer-employee relationship." It is a civil issue, which is outside the
jurisdiction of the Labor Arbiter. On appeal before the NLRC, the NLRC
ruled that because of petitioners' failure to vacate Solid Mills' property,
Solid Mills was justified in withholding their benefits and separation
pay. On appeal before the CA, the petition was dismissed. Hence, this
petition.

ISSUE:

63
Page 64

I.) Whether or not the NLRC have jurisdiction over the


“accountabilities” of petitioners.

II) Whether or not respondent had the right to withhold the


payment of separation pay and other benefits from petitioners.

RULING:
I. YES. The National Labor Relations Commission has jurisdiction
to determine, preliminarily, the parties' rights over a property, when it
is necessary to determine an issue related to rights or claims arising
from an employer-employee relationship. Article 217(6) provides that
the Labor Arbiter, in his or her original jurisdiction, and the National
Labor Relations Commission, in its appellate jurisdiction, may
determine issues involving claims arising from employer-employee
relations. Petitioners' claim that they have the right to the immediate
release of their benefits as employees separated from respondent
Solid Mills is a question arising from the employer-employee
relationship between the parties. Claims arising from an employer-
employee relationship are not limited to claims by an employee.
Employers may also have claims against the employee, which arise
from the same relationship.

As a general rule, therefore, a claim only needs to be sufficiently


connected to the labor issue raised and must arise from an employer-
employee relationship for the labor tribunals to have jurisdiction. In this
case, respondent Solid Mills claims that its properties are in petitioners'
possession by virtue of their status as its employees. Respondent Solid
Mills allowed petitioners to use its property as an act of liberality. Put
in other words, it would not have allowed petitioners to use its property
had they not been its employees. The return of its properties in
petitioners' possession by virtue of their status as employees is an
issue that must be resolved to determine whether benefits can be
released immediately. The issue raised by the employer is, therefore,
connected to petitioners' claim for benefits and is sufficiently
intertwined with the parties' employer-employee relationship. Thus, it
is properly within the labor tribunals' jurisdiction.

II. YES. Requiring clearance before the release of last payments


to the employee is a standard procedure among employers, whether
public or private. As a general rule, employers are prohibited from
withholding wages from employees. However, our law supports the
employers' institution of clearance procedures before the release of
wages. As an exception to the general rule that wages may not be
withheld and benefits may not be diminished. Art. 113(3) of the Labor
Code provides, “In cases where the employer is authorized by law or
regulations issued by the Secretary of Labor and Employment”

64
Page 65

The Civil Code provides that the employer is authorized to


withhold wages for debts due: Article 1706. “Withholding of the wages,
except for a debt due, shall not be made by the employer”. "Debt" in
this case refers to any obligation due from the employee to the
employer. It includes any accountability that the employee may have
to the employer. There is no reason to limit its scope to uniforms and
equipment, as petitioners would argue. More importantly, respondent
Solid Mills and the union representing petitioners, agreed that the
release of petitioners' benefits shall be "less accountabilities."
"Accountability," in its ordinary sense, means obligation or debt. The
ordinary meaning of the term "accountability" does not limit the
definition of accountability to those incurred in the worksite. As long as
the debt or obligation was incurred by virtue of the employer-employee
relationship, generally, it shall be included in the employee's
accountabilities that are subject to clearance procedures. It may be
true that not all employees enjoyed the privilege of staying in
respondent Solid Mills' property. However, this alone does not imply
that this privilege when enjoyed was not a result of the employer-
employee relationship. Those who did avail of the privilege were
employees of respondent Solid Mills. Petitioners' possession should,
therefore, be included in the term "accountability."

65
Page 66

Case Digests by: Gem Edward Aquino

AMECOS Innovations, Inc. v. Eliza Lopez


G.R. No. 178055. July 2, 2014.
Del Castillo, J.

DOCTRINE:
JURISDICTION OF LABOR ARBITER; DAMAGES.
Since there is no longer any dispute regarding coverage, benefits,
contributions and penalties to speak of (SSS), the Social Security
Commission need not be unnecessarily dragged into the picture.
Besides, it cannot be made to act as a collecting agency for petitioners'
claims against the respondent; the Social Security Law should not be
so interpreted, lest the SSC be swamped with cases of this sort.

FACTS:
On May 2003, petitioner received a subpoena from the Office of
the Prosecutor in connection with a complaint filed by the Social
Security System against it for alleged delinquency in the remittance of
SSS contributions and penalty liabilities. By way of explanation,
petitioner attributed the such acts to respondent Lopez, arguing that
respondent refused to provide it with her SSS number and to be
deducted her contributions. Petitioner eventually settled its obligations
with SSS, thus the complaint against it was withdrawn. Thereafter,
petitioner sent a demand letter to respondent demanding payment
representing her share in the SSS contributions and expenses, but to
not avail. Thus, petitioners filed a complaint for sum of money and
damages against respondent before the MeTC. Respondent, for her
part, filed a motion to dismiss, claiming that the regular courts do not
have jurisdiction over the instant case as it arose out of their employer-
employee relationship.

In its decision, the MeTC dismissed the complaint for lack of


jurisdiction. Such decision was affirmed by the RTC and CA on appeal.
Hence, this petition. Petitioner argues that the employer-employee
relationship between them is merely incidental, and does not
necessarily place their dispute within the exclusive jurisdiction of the
labor tribunals; the true source of respondent's obligation is derived
from Articles 19, 22, and 2154 of the Civil Code.,
ISSUE:
Whether or not the regular courts have jurisdiction over
petitioners complaint.

RULING:

66
Page 67

NO. This Court holds that as between the parties, Article 217 (a)
(4) of the Labor Code is applicable. Said provision bestows upon the
Labor Arbiter original and exclusive jurisdiction over claims for
damages arising from employer-employee relations. The observation
that the matter of SSS contributions necessarily flowed from the
employer-employee relationship between the parties — shared by the
lower courts and the CA — is correct; thus, petitioners' claims should
have been referred to the labor tribunals.

In this connection, it is noteworthy to state that "the Labor Arbiter


has jurisdiction to award not only the reliefs provided by labor laws, but
also damages governed by the Civil Code." At the same time, it cannot
be assumed that since the dispute concerns the payment of SSS
premiums, petitioners' claim should be referred to the Social Security
Commission (SSC) pursuant to Republic Act No. 1161. As far as SSS
is concerned, there is no longer a dispute with respect to petitioners'
accountability to the System; petitioners already settled their pecuniary
obligations to it. Since there is no longer any dispute regarding
coverage, benefits, contributions and penalties to speak of, the SSC
need not be unnecessarily dragged into the picture. Besides, it cannot
be made to act as a collecting agency for petitioners' claims against
the respondent; the Social Security Law should not be so interpreted,
lest the SSC be swamped with cases of this sort.

67
Page 68

Case Digests by: Gem Edward Aquino

Philippine Airlines, Inc. v. Airline Pilots Association of


the Philippines, Et Al.
G.R. No. 200088. February 26, 2018.
Martires, J.

DOCTRINE:
JURISDICTION OF LABOR ARBITER; DAMAGES.
regular courts are devoid of any jurisdiction over claims for
damages arising from a labor strike. The rule stands even if the strike
is illegal.

FACTS:
The present case arose from a labor dispute between petitioner
and respondent, the sole and exclusive bargaining agent of all
commercial pilots of PAL. On 1997, respondent filed a notice of strike
before the DOLE alleging that petitioner committed unfair labor
practice. The SOLE assumed jurisdiction over the dispute and
prohibited respondent from staging a strike. However, respondent still
commenced a strike. A return-to-work order was then issued by the
DOLE but was unheeded by respondent, thus, the SOLE issue a
resolution which declared the strike as illegal and the loss of
employment status of officers who participated in the strike. This
resolution was affirmed by the CA and SC on appeal and attained
finality. Thereafter, almost 8month from the finality of such decision,
petitioner filed before the LA a complaint for damages against
respondent alleging that on the 2nd day of the illegal strike, its striking
pilots abandoned 3 PAL aircraft which resulted in petitioner incurring
expenses by way of hotel accommodations, meals for stranded
passengers, and that its operations was crippled by such illegal strike.

In its decision, the LA ruled that it had not jurisdiction to resolve


the issue on damages. On appeal before the NLRC, the NLRC held
that the reliefs prayed for by PAL should have been ventilated before
the regular courts considering that they are based on the tortuous acts
allegedly committed by the respondents. On appeal before the CA, the
CA sustained the dismissal of the complaint for lack of jurisdiction.
Hence, this petition.

ISSUE:
Whether or not the regular courts have jurisdiction over the
complaint for damages of petitioner.

68
Page 69

RULING:
NO. To determine whether a claim for damages under paragraph
4 of Article 217 is properly cognizable by the labor arbiter,
jurisprudence has evolved the "reasonable connection rule" which
essentially states that the claim for damages must have reasonable
causal connection with any of the claims provided for in that article. A
money claim by a worker against the employer or vice-versa is within
the exclusive jurisdiction of the labor arbiter only if there is a
"reasonable causal connection" between the claim asserted and
employee-employer relations. Only if there is such a connection with
the other claims can the claim for damages be considered as arising
from employer-employee relations. Absent such a link, the complaint
will be cognizable by the regular courts.

The appellate court was of the opinion that, applying the


reasonable connection rule, PAL's claims for damages have no
relevant connection whatsoever to the employer-employee
relationship between the parties. Thus, the claim is within the exclusive
jurisdiction of the regular courts. The appellate court is mistaken. The
Court agrees with PAL that its claim for damages has reasonable
connection with its employer-employee relationship with the
respondents. Contrary to the pronouncements made by the appellate
court, PAL's cause of action is not grounded on mere acts of quasi-
delict. The claimed damages arose from the illegal strike and acts
committed during the same which were in turn closely related and
intertwined with the respondents' allegations of unfair labor practices
against PAL. Since the loss and injury from which PAL seeks
compensation have reasonable causal connection with the alleged
acts of unfair labor practice, a claim provided for in Article 217 of the
Labor Code, the question of damages becomes a labor controversy
and is therefore an employment relationship dispute.

In Goodrich Employees Association v. Hon. Flores, the Court


stressed the rule that “cases involving unfair labor practices are within
the jurisdiction of the Court of Industrial Relations (CIR), the labor
tribunal at that time.” The Court further emphasized that where the
subject matter is within the exclusive jurisdiction of the CIR, it must be
deemed to have jurisdiction over all incidental matters connected to
the main issue. In Philippine Long Distance Telephone Company v.
Free Telephone Workers Union, the Court reiterated the rule that
regular courts are devoid of any jurisdiction over claims for damages
arising from a labor strike. The rule stands even if the strike is illegal.
In Antipolo Highway Lines Employees Union v. Hon. Aquino, it was
held that, “although it was artfully made to appear that the suit was one
for damages that did not divest the Court of Industrial Relations of its
jurisdiction. The Complaint itself, in paragraph 5, adverted to an "illegal
strike" and "picket lines," which are but mere incidents or

69
Page 70

consequences of the unfair labor practice complained against by


petitioner Union.” Jurisprudence dictates that where the plaintiff's use
of action for damages arose out of or was necessarily intertwined with
an alleged unfair labor practice, the jurisdiction is exclusively with the
labor tribunal. Likewise, where the damages separately claimed by the
employer were allegedly incurred as a consequence of strike or
picketing of the union, such complaint for damages is deeply rooted in
the labor dispute between the parties and within the exclusive
jurisdiction of the labor arbiter. Consequently, the same should be
dismissed by ordinary courts for lack of jurisdiction.

70
Page 71

Case Digests by: Mar Lorenzo B. Asuncion

Dai-chi Electronics v. Villarama


G.R. No. 112940, November 21, 1994
Quiason, J.:

DOCTRINE:
CLAIM FOR DAMAGES BY THE EMPLOYEE AGAINST THE
EMPLOYER; REASONABLE CAUSAL CONNECTION
The mere fact that there was a prior or pre-existing employer-
employee relationship between the parties per se does not necessarily
mean that all claims for damages will already be within the jurisdiction
of the Labor Arbiter and would already call for the application of the
Labor Code. In other words, although a controversy is between an
employer and an employee, the Labor Arbiters have no jurisdiction if
the issue does not call for the application of the Labor Code.

In order for the claim for damages to be within the jurisdiction of


the Labor Arbiter, it must refer to a claim which has a reasonable
causal connection with the employer-employee relationship and not
merely incidental thereto. When the cause of action is based on a
quasi-delict, [contract], or tort, which has no reasonable causal
connection with any of the claims provided for in Article 217, jurisdiction
over the action is with the regular courts. In these cases. the complaint
for damages was anchored not on the termination of the employee's
services per se, but rather on the manner and consequent effects of
such termination.

In this case, the claim for damages filed by Dai-chi Electronics did
not call for the application of the Labor Code because the breach in
this case is a Civil Law issue which would call for the application of the
New Civil Code since what was involved is an alleged violation of a
non-compete clause founded upon a contract entered into between the
parties rather than a dispute arising from the parties’ prior employer-
employee relationship.

FACTS:
Dai-chi Electronics filed a complaint for damages with the RTC
against Villarama who was a former employee on the ground that
Villarama allegedly violated paragraph 5 of their Contract of
Employment which provided for a non-compete clause. Dai-chi
contends that after Villarama’s employment, the latter became an
employee of Angel Sound Philippines Corporation, a corporation which
is engaged in the same line of business as that of Dai-chi within 2 years
from Villarama’s employment thus violating the non-compete clause
and that Villarama was holding the position of Head of the Material

71
Page 72

Management Control Department, the same position he held while in


the employ of the petitioner.

The RTC dismissed the case on the ground that it had no


jurisdiction over the subject matter of the controversy because the
complaint was for damages arising from an employer-employee
relationship. Thus, under the Labor Code, it is the Labor Arbiter which
had original and exclusive jurisdiction over the subject matter of the
case.

ISSUE:
Was the RTC correct in ruling that it had no jurisdiction over the
dispute between Villarama and Dai-chi Electronics as to an alleged
violation of the former of the non-compete clause embodied in their
contract?

RULING:
No. The RTC in this case was incorrect in ruling that it had no
jurisdiction over the controversy since it does not involve the
application of the Labor Code nor was the dispute related to the
employer-employee relationship between the parties. The dispute in
this case calls for the application on the rules involving contract
relations between the parties.

Petitioner does not ask for any relief under the Labor Code of the
Philippines. It seeks to recover damages agreed upon in the contract
as redress for private respondent's breach of his contractual obligation
to its "damage and prejudice". Such cause of action is within the realm
of Civil Law, and jurisdiction over the controversy belongs to the
regular courts.

The claims for damages in this case call for the application of Civil
Law. It does not involve a claim for damages under Article 217(4).
Jurisprudence has evolved the rule that claims for damages under
paragraph 4 of Article 217, to be cognizable by the Labor Arbiter, must
have a reasonable causal connection with any of the claims provided
for in that article. Only if there is such a connection with the other claims
can the claim for damages be considered as arising from employer-
employee relations.

72
Page 73

Case Digests by: Mar Lorenzo B. Asuncion

People’s Broadcasting Service (Bombo Radyo Phils.


Inc.) v. Secretary of the Department of Labor and
Employment
G.R. No. 179652, March 6, 2012
Velasco Jr., J.:

DOCTRINE:
EXPANDED VISITORIAL AND ENFORCEMENT POWERS OF THE
DOLE
a. if a complaint is brought before the DOLE to give effect to the
labor standards provisions of the Labor Code or other labor
legislation, and there is a finding by the DOLE that there is an
existing employer-employee relationship, the DOLE exercises
jurisdiction to the exclusion of the NLRC.

b. If the DOLE finds that there is no [existing] employer-employee


relationship, the jurisdiction is properly with the NLRC.

c. If a complaint is filed with the DOLE, and it is accompanied by a


claim for reinstatement, the jurisdiction is properly with the Labor
Arbiter, under Art. 217 (3) of LC, which provides that the Labor
Arbiter has original and exclusive jurisdiction over those cases
involving wages, rates of pay, hours of work, and other terms and
conditions of employment, if accompanied by a claim for
reinstatement.

d. If a complaint is filed with the NLRC, and there is still an existing


employer-employee relationship, the jurisdiction is properly with
the DOLE. The findings of the DOLE, however, may still be
questioned through a petition for certiorari under Rule 65 of the
Rules of Court.

FACTS:
Jandeleon Juezan filed a complaint against petitioner with DOLE
Regional Office Cebu for illegal deduction and non-payment of
statutory benefits as required by law. The DOLE Regional Director
found that Juezan was an employee and entitled to his money claims.

DOLE Sec. dismissed the appeal. On appeal, CA held that DOLE


Sec. had jurisdiction as the jurisdictional limitation imposed by Art. 129
of the Labor Code of the power of the DOLE Sec. under Art. 128(b)
had been repealed by RA 7730. The CA also held that petitioner was
not denied due process.

73
Page 74

SC reversed the CA decision and found that there was no


employer-employee relationship. It held that while DOLE may make a
determination of the existence of employer-employee relationship, this
is not co extensive with the visitorial and enforcement power in Art.
128(b) as amended by RA 7730.

In its decision, the Supreme Court ruled that the NLRC was the
primary agency to determine the existence of such employer-employee
relationship. PAO and DOLE filed a motion for clarification wherein
such motion was treated as a second Motion for Reconsideration and
was granted.

ISSUE:
Does the DOLE, in the exercise of its enforcement and visitorial
powers, have no jurisdiction to determine whether there is an existing
employer-employee relationship between the parties?

RULING:
No. DOLE can make a determination as to whether there exists
an employer-employee relationship and there is no requirement that
referral must first be made to the NLRC for such determination.

No limitation in the law was placed on DOLE’s power to determine


the existence of EE relationship. No procedure was laid where DOLE
would only make a preliminary finding, that the power was primarily
held by NLRC. The law did not say that DOLE would first seek NLRC’s
determination of EE relation or that if its existence is disputed, DOLE
would refer the matter to NLRC.

DOLE must have the power to determine whether EE relation


exists and from there decide whether to issue compliance orders in
accordance with Art. 128(b) as amended. The use of the control test is
not limited to NLRC. DOLE Sec. or his representatives can use the
same test, even in the course of inspection, using the same evidence
that would have been presented with NLRC.

DOLE’s determination must be respected. The expanded visitorial


and enforcement power of DOLE under RA 7730 would be rendered
nugatory if the alleged employer could, by just disputing EE relation,
force referral to NLRC. If DOLE makes a finding that there is EE
relation, it takes cognizance of the matter to the exclusion of NLRC.
DOLE may also determine that there is no EE relation, divesting itself
of jurisdiction. Under Art. 128(b) as amended, DOLE is fully
empowered to determine the existence of EE relationship in the
exercise of its visitorial and enforcement power, subject to judicial
review NOT review by NLRC.

74
Page 75

Case Digests by: Mar Lorenzo B. Asuncion

Ex-Bataan Veterans Security Agency v. Sec. Laguesma


G.R. No. 152396, November 20, 2007,
Carpio, J.:

DOCTRINE:
EXPANDED VISITORIAL AND ENFORCEMENT POWERS OF THE
DOLE
The visitorial and enforcement powers of the DOLE Regional
Director to order and enforce compliance with labor standard laws can
be exercised even when the individual claim exceeds P5,000. RA 7730
repealed the jurisdictional limitation imposed by Article 129 on Article
128 of the Labor Code.

FACTS:
Ex-Bataan Veterans Security Agency, Inc. (EBVSAI) is in the
business of providing security services while the private respondents
in this case are EBVSAI’s employees assigned to NAPOCOR’s
Ambuklao Plant.

The private respondents headed by Pong instituted a complaint


for underpayment of wages against EBVSAI before the Regional Office
of the DOLE. The Regional Office, after inspection found several
violations of the Labor Code such as non-payment of holiday pay, rest
day premium, service incentive leave, and underpayment of night-shift
differential pay, among others. EBVSAI was ordered to pay the
computed deficiencies in favor of the private respondents for a total
amount of around P793,000.00. EBVSAI filed a motion for
reconsideration contending that the Regional Director does not have
jurisdiction over the subject matter because the money claim of each
respondents exceed P5,000. EBVSAI pointed out that the case should
have been endorsed to the Labor Arbiter.

ISSUE:
Does the Secretary of Labor or his duly authorized representatives
have jurisdiction over the money claims notwithstanding that the claim
exceeds P5,000?

75
Page 76

RULING:
Yes. The Supreme Court ruled that the visitorial and enforcement
powers of the DOLE Regional Director to order and enforce
compliance with labor standard laws can be exercised even where the
individual claim exceeds P5,000.

In this case, the Regional Director validly assumed jurisdiction


over the money claims of private respondents even if the claims
exceeded P5,000 because such jurisdiction was exercised in
accordance with Article 128(b) of the Labor Code.

In relation to People’s Broadcasting Services v. Secretary of


DOLE, RA 7730, an Act Further Strengthening the Visitorial and
Enforcement Powers of the Sec. of Labor, did away with the P5,000
limitation. The only qualification to the expanded power of DOLE was
that there still be an existing employer-employee relationship.

76
Page 77

Case Digests by: Mar Lorenzo B. Asuncion

Victor Meteoro v. Creative Creatures, Inc.


G.R. No. 171275, July 13, 2009
Nachura, J.:

DOCTRINE:
EXCEPTION TO THE RULE THAT THERE IS NO LONGER ANY
LIMIT AS TO THE MONETARY AWARD WITHIN THE
JURISDICTION OF THE DOLE
Although as a general rule, there is no longer any limit as to the
monetary award which is within the scope of jurisdiction of the DOLE,
this rule is not absolute. The last sentence of Article 128 (b) of the
Labor Code, otherwise known as the "exception clause," provides an
instance when the Regional Director or his representatives may be
divested of jurisdiction over a labor standards case provided that the
following requisites are present:

a. that the employer contests the findings of the labor regulations


officer and raises issues thereon,

b. that in order to resolve such issues, there is a need to examine


evidentiary matters; and

c. that such matters are not verifiable in the normal course of


inspection.

FACTS:
Meteoro et. al. filed their respective complaints for non-payment of
night shift differential pay, overtime pay, holiday pay, among others
before the DOLE-NRC. In its position paper, the respondent argued
that the DOLE-NCR does not have jurisdiction over the complaint of
the petitioners because of the absence of an employee-employee
relationship. Moreover, they contend that the petitioners were
freelance individuals performing skills and expertise inherently
exclusive to them like actors among others. Thus, they were treated as
special types of workers. The petitioners contend that they were
employees of Creative Creatures. Thereafter, the DOLE ordered the
respondent to pay unpaid benefits in favor of the petitioners in the
amount of around P2,600,000.00 sustaining the claim on the existence
of employer-employee relationship using the determinants as set forth
under the Labor Code.

77
Page 78

ISSUE:
Which body or tribunal has jurisdiction over petitioners’ money
claim, the DOLE Secretary or his duly authorized representatives, or
the NLRC?

RULING:
It is the NLRC which has jurisdiction over the case. R.A. 7730 was
enacted, amending Article 128 (b) to its present formulation, so as to
free it from the jurisdictional restrictions found in Articles 129 and 217.
This notwithstanding, the power of the Regional Director to hear and
decide the monetary claims of employees is not absolute. The last
sentence of Article 128 (b) of the Labor Code, otherwise known as the
"exception clause," provides an instance when the Regional Director
or his representatives may be divested of jurisdiction over a labor
standards case. Under prevailing jurisprudence, the so-called
"exception clause" has the following elements, all of which must
concur:

a. that the employer contests the findings of the labor regulations


officer and raises issues thereon;

b. that in order to resolve such issues, there is a need to examine


evidentiary matters; and

c. that such matters are not verifiable in the normal course of


inspection.

In the present case, the CA aptly applied the "exception clause."


At the earliest opportunity, respondent registered its objection to the
findings of the labor inspector. The labor inspector, in fact, noted in its
report that "respondent alleged that petitioners were contractual
workers and/or independent and talent workers without control or
supervision and also supplied with tools and apparatus pertaining to
their job." In its position paper, respondent again insisted that
petitioners were not its employees. It then questioned the Regional
Director's jurisdiction to entertain the matter before it, primarily
because of the absence of an employer-employee relationship. In the
case at bar, whether or not petitioners were independent
contractors/project employees/free-lance workers is a question of fact
that necessitates the examination of evidentiary matters not verifiable
in the normal course of inspection. Respondent contested the findings
of the labor inspector during and after the inspection and raised issues
the resolution of which necessitated the examination of evidentiary
matters not verifiable in the normal course of inspection. Hence, the
Regional Director was divested of jurisdiction and should have
endorsed the case to the appropriate Arbitration Branch of the NLRC.

78
Page 79

Case Digests by: Mar Lorenzo B. Asuncion

Leslie Okol v. Slimmers World International Behavior


Modifications, Inc.
G.R. No. 160146, December 11, 2009
Carpio, J.:

DOCTRINE
DISMISSAL OF A CORPORATE OFFICER IS AN INTRA-
CORPORATE CONTROVERSY OUTSIDE OF THE JURISDICTION
OF THE LABOR ARBITER
A corporate officer's dismissal is always a corporate act, or an
intra-corporate controversy which arises between a stockholder and a
corporation. The question of remuneration involving a stockholder and
officer, not a mere employee, is not a simple labor problem but a matter
that comes within the area of corporate affairs and management and
is a corporate controversy in contemplation of the [Revised]
Corporation Code.

FACTS:
Slimmers World International operating under the name Behavior
Modifications, Inc. (Slimmers World) employed Okol as a management
trainee. She rose up the ranks and became a Head Office Manager
and then Director and Vice President.

Prior to the dismissal of Okol, Slimmers World preventively


suspended Okol. The suspension arose from the seizure of the Bureau
of Customs of seven elliptical machines and treadmills belonging to or
consigned to Slimmers World. The shipment of the equipment was
placed under the names of Okol and two customs brokers for a value
less tha $500. For being undervalued, the equipment were seized.
Eventually, Okol’s employment with Slimmers World was terminated.

Okol filed a complaint with the Arbitration branch of the NLRC


against Slimmers World, Behavior Modifications, Inc. and Moy for
illegal suspension and illegal dismissal among others. The case was
dismissed by the Labor Arbiter, it ruled that since the dismissal
involved a corporate officer, the dispute was an intra-corporate
controversy falling outside the juridisction of the Arbitration branch.

ISSUE:
Does the NLRC have jurisdiction over the illegal dismissal case
filed by Okol?

79
Page 80

RULING:
No. The NLRC have no jurisdiction over the illegal dismissal filed
by Okol because what is involved in this case is an intra-corporate
dispute.

The dismissal of a corporate officer is always a corporate dispute.


Clearly, from the documents submitted by respondents, petitioner was
a director and officer of Slimmers World. The charges of illegal
suspension, illegal dismissal, unpaid commissions, reinstatement and
back wages imputed by petitioner against respondents fall squarely
within the ambit of intra-corporate disputes.

A corporate officer's dismissal is always a corporate act, or an


intra-corporate controversy which arises between a stockholder and a
corporation. The question of remuneration involving a stockholder and
officer, not a mere employee, is not a simple labor problem but a matter
that comes within the area of corporate affairs and management and
is a corporate controversy in contemplation of the [Revised]
Corporation Code. The determination of the rights of a director and
corporate officer dismissed from his employment as well as the
corresponding liability of a corporation, if any, is an intra-corporate
dispute subject to the jurisdiction of the regular courts. Thus, it is the
Special Commercial Courts which has jurisdiction over the dispute
between Okol and Slimmers World and not the Labor Arbiter nor the
NLRC because Okol’s position was that of a director and officer.

80
Page 81

Case Digests by: Mar Lorenzo B. Asuncion

Paul V. Santiago v. CF Sharp Crew Management, Inc.


G.R. No. 162419, July 10, 2007
Tinga, J.:

DOCTRINE
INSTANCE WHERE THE NLRC HAS JURISDICTION EVEN IF
THERE WAS NO EMPLOYER-EMPLOYEE RELATIONSHIP THAT
FORMED BETWEEN THE PARTIES
Despite the absence of an employer-employee relationship
between petitioner and respondent, the NLRC has jurisdiction over
petitioner’s complaint. Under Sec. 10 of R.A. No. 8042 or the Migrant
Workers and Overseas Filipinos Act: notwithstanding any provision of
law to the contrary, the Labor Arbiters of the National Labor Relations
Commission (NLRC) shall have the original and exclusive jurisdiction
to hear and decide, within ninety (90) calendar days after the filing of
the complaint, the claims arising out of an employer-employee
relationship or by virtue of any law or contract involving Filipino
workers for overseas deployment including claims for actual, moral,
exemplary and other forms of damages. x x x

Since the present petition involves the employment contract


entered into by petitioner for overseas employment, his claims are
cognizable by the labor arbiters of the NLRC [notwithstanding that an
employer-employee relationship between the parties was did not arise
because of the non-deployment of Santiago].

FACTS:
Santiago worked as a seafarer for Smith Bell Management for 5
years. A new contract was signed on February 3, 1998. The following
day, the contract was approved by the Philippine Overseas
Employment Administration (POEA). Santiago was to be deployed on
board the “MSV Seaspread.

A week before the scheduled date of departure, the Vice President


of CF Sharp Crew Management sent a facsimile message to the
captain of “MSV Seaspread” telling the latter that there were reports
received by the former that Santiago should not be sent to “MSV
Seaspread” because Santiago, if allowed to depart will jump ship in
Canada like his brother. Because of this message, prior to Santiago
getting aboard of “MSV Seaspread”, the contract was unilaterally
cancelled. He was then told he would not be leaving for Canada
anymore, but he was reassured that he might be considered for
deployment at some future date. Santiago filed a claim for illegal
dismissal. Santiago contends that CF Sharp Crew Management

81
Page 82

violated that Migrant Workers and Overseas Filipinos Act because it


had unilaterally and arbitrarily prevent the consummation of the POEA-
approved contract. Since it prevented his deployment without valid
basis, said deployment being a condition to the consummation of the
POEA contract, the contract is deemed consummated, and therefore
he should be awarded actual damages, consisting of the stipulated
salary and fixed overtime pay

On the other hand, respondent argues that the Labor Arbiter has
no jurisdiction to award petitioner’s monetary claims. His employment
with respondent did not commence because his deployment was
withheld for a valid reason. Consequently, the labor arbiter and/or the
NLRC cannot entertain adjudication of petitioner’s case much less
award damages to him. Moreover, what was involved was a breach of
contract, thus, the dispute is cognizable by the civil courts.

ISSUE:
a. Did the employer-employee relationship between the parties
already commence?

b. Assuming that the employer-employee relationship between the


parties has not yet commenced, can the NLRC still have
jurisdiction over the dispute?

RULING:
a. The employer-employee relationship relationship between the
parties had not yet commenced since the seafarer, Santiago,
was prevented from leaving the port of Manila and was refused
deployment and under the POEA-approved employment
contract provides that the employer-employee relationship shall
commence only upon the seafarer’s actual departure from the
port. Considering that petitioner was not able to depart from the
airport or seaport in the point of hire, the employment contract
did not commence, and no employer-employee relationship was
created between the parties

However, a distinction must be made between the perfection of


the employment contract and the commencement of the
employer-employee relationship. The perfection of the contract,
which in this case coincided with the date of execution thereof,
occurred when petitioner and respondent agreed on the object
and the cause, as well as the rest of the terms and conditions
therein. The commencement of the employer-employee
relationship, as earlier discussed, would have taken place had
petitioner been actually deployed from the point of hire. Thus,
even before the start of any employer-employee relationship,
contemporaneous with the perfection of the employment contract

82
Page 83

was the birth of certain rights and obligations, the breach of which
may give rise to a cause of action against the erring party. Thus,
if the reverse had happened, that is the seafarer failed or refused
to be deployed as agreed upon, he would be liable for damages.
While the POEA Standard Contract must be recognized and
respected, neither the manning agent nor the employer can
simply prevent a seafarer from being deployed without a valid
reason.

Thus, notwithstanding that there was no employer-employee


relationship that is present in this case, Santiago is not left
without relief and he may avail of the provisions of the Labor
Code.

b. Yes. Notwithstanding that no employer-employee relationship


was formed between the parties, Santiago is not left without
relief. The jurisdiction of labor arbiters is not limited to claims
arising from employer-employee relationships by reason of Sec.
10 of the Migrant Workers Act:

Sec. 10. Money Claims. – Notwithstanding any provision of law


to the contrary, the Labor Arbiters of the National Labor Relations
Commission (NLRC) shall have the original and exclusive
jurisdiction to hear and decide, within ninety (90) calendar days
after the filing of the complaint, the claims arising out of an
employer-employee relationship or by virtue of any law or
contract involving Filipino workers for overseas deployment
including claims for actual, moral, exemplary and other forms of
damages. x x x [Emphasis supplied]

Since the present petition involves the employment contract


entered into by petitioner for overseas employment, his claims
are cognizable by the labor arbiters of the NLRC.

83
Page 84

Case Digests by: Mar Lorenzo B. Asuncion

Industrial Personnel & Management Services, Inc.


(IPAMS) v. Jose G. De Vera
G.R. No. 205703, March 07, 2016
Mendoza, J.:

DOCTRINE:
GENERALLY, EMPLOYMENT CONTRACTS ENTERED INTO BY
OFWs ARE GOVERNED BY PHILIPPINES LAWS. BY WAY OF
EXCEPTION, FOREIGN LAW CAN APPLY PROVIDED THAT
THERE ARE CERTAIN REQUITES THAT ARE PRESENT
In the case of employment contracts entered into by Overseas
Filipino Workers, as a general rule, it is still Philippine law that will
govern such overseas employment contracts. By way of exception,
foreign law can govern the overseas employment contract of an OFW
provided the following requisites are present:

1. That it is expressly stipulated in the overseas employment


contract that a specific foreign law shall govern;

2. That the foreign law invoked must be proven before the


courts pursuant to the Philippine rules on evidence;

3. That the foreign law stipulated in the overseas employment


contract must not be contrary to law, morals, good customs,
public order, or public policy of the Philippines; and

4. That the overseas employment contract must be processed


through the POEA.

If any of the requisites are absent, notwithstanding the express


agreement by the parties, it Is still Philippine law that will govern the
overseas employment contract. contracts. This rule is rooted in the
constitutional provision of Section 3, Article XIII that the State shall
afford full protection to labor, whether local or overseas. Hence, even
if the OFW has his employment abroad, it does not strip him of his
rights to security of tenure, humane conditions of work and a living
wage under our Constitution.

84
Page 85

FACTS:
Arriola was offered by SNC-Lavalin Engineers and Contractors,
Inc. through a letter the position of Safety Officer in its Ambatovy
Project site in Madagascar for a period of nineteen months. Arriola
was then hired by SNC-Lavalin through its local manning agency,
Industrial Personnel and Management Services (IPAMS). He then
signed the contract of employment and started working in Madagascar.

After three months, Arriola received a notice of pre-termination of


employment from SNC-Lavalin stating that the employment will be pre-
terminated due to diminishing workload in the area of his expertise and
the unavailability of alternative assignments. Eventually, Arriola was
repatriated.

Aggrieved, Arriola filed a complaint against IPAMS for illegal


dismissal and non-payment of overtime pay and other benefits. He
claimed that SNC-Lavalin still owed him unpaid salaries equivalent to
the three-month unexpired portion of his contract. He asserted that
SNC-Lavalin never offered any valid reason for his early termination
and that he was not given sufficient notice regarding the same. Arriola
also insisted that the petitioners must prove the applicability of
Canadian law before the same could be applied to his employment
contract.

IPAMS contend among others that particular labor laws of a


foreign country incorporated in a contract freely entered into between
an OFW and a foreign employer through the latter’s agent was valid.
Thus, in this case, Canadian laws governed the contract.
Consequently, applying the laws of Canada, the said foreign law did
not require any ground for early termination of employment, and the
only requirement was the written notice of termination. Even assuming
that Philippine laws should apply, Arriola would still be validly
dismissed because domestic law recognized retrenchment and
redundancy as legal grounds for termination.

ISSUE:
In determining the rights and obligations of an OFW, the local
recruiter, and the foreign employer, should the foreign law
automatically apply if agreed upon by the parties?

RULING:
No. Notwithstanding such agreement, the general rule is that
Philippine laws apply even to overseas employment contracts. This
rule is rooted in the constitutional provision of Section 3, Article XIII that
the State shall afford full protection to labor, whether local or overseas.
Hence, even if the OFW has his employment abroad, it does not strip

85
Page 86

him of his rights to security of tenure, humane conditions of work and


a living wage under our Constitution. By way of exception, the parties
may agree that a foreign law shall govern the employment contract.
However, before such agreement can be given effect, the following
requisites must be present:

1. That it is expressly stipulated in the overseas employment


contract that a specific foreign law shall govern;

2. That the foreign law invoked must be proven before the


courts pursuant to the Philippine rules on evidence;

3. That the foreign law stipulated in the overseas employment


contract must not be contrary to law, morals, good customs,
public order, or public policy of the Philippines; and

4. That the overseas employment contract must be processed


through the POEA.

These four (4) requisites must be complied with before the


employer could invoke the applicability of a foreign law to an overseas
employment contract. With these requisites, the State would be able to
abide by its constitutional obligation to ensure that the rights and well-
being of our OFWs are fully protected.

In this case, the foreign law was not expressly specified in the
employment contract. No foreign law was expressly stipulated in the
overseas employment contract with Arriola. The foreign employer is
obliged to expressly declare at the onset of the labor contract that a
foreign law shall govern it. In that manner, the OFW would be informed
of the applicable law before signing the contract. Considering that no
foreign law was specified in the contract and the same was executed
in the Philippines, the doctrine of lex loci celebrationis applies and the
Philippine laws shall govern the overseas employment of Arriola.

Moreover, the foreign law invoked is contrary to the Constitution


and the Labor Code. Even if the employment contract expressly
stipulated in the overseas employment contract that a specific foreign
law shall govern, the third requisite will still be missing since some
provisions thereof is contrary to the Constitution as well as the Labor
Code. First, the ESA does not require any ground for the early
termination of employment. Second, the ESA allows the employer to
dispense with the prior notice of termination to an employee. The
employee under the ESA could be immediately dismissed without
giving him the opportunity to explain and defend himself.

86
Page 87

Thus, the provisions of the Employment Agreement are patently


inconsistent with the right to security of tenure both protected by the
Constitution as well the Labor Code. Likewise, there is a violation of
the right to due process of the employee. In fine, as the petitioners
failed to meet all the four (4) requisites on the applicability of a foreign
law, then the Philippine labor laws must govern the overseas
employment contract of Arriola.

87
Page 88

Case Digests by: Mar Lorenzo B. Asuncion

Ace Navigation Co. Inc., v. Fernandez


G.R. No. 197309, October 10, 2012
Brion, J.:

DOCTRINE:
JURISDICTION OF THE VOLUNTARY ARBITRATOR
Matters that are within the original and exclusive jurisdiction of the
Labor Arbiter may be submitted to a Voluntary Arbitrator upon express
agreement by the parties pursuant to Art. 262 of the Labor Code which
provides that the Voluntary Arbitrator or panel of Voluntary Arbitrators,
upon agreement of the parties, shall also hear and decide all other
labor disputes including unfair labor practices and bargaining
deadlocks.

A matter that is within the exclusive jurisdiction of the Labor Arbiter


can be submitted for voluntary arbitration because of the clear
language of the parties’ CBA on the matter. Moreover, and more
importantly, the jurisdiction of the voluntary arbitrator is upheld in
recognition of the State’s express preference for voluntary modes of
dispute settlement, such as conciliation and voluntary arbitration as
expressed in the Constitution, the law and the rules as provided in Sec.
3, Article XIII of the Constitution which declares:

xxxx

The State shall promote the principle of shared responsibility


between workers and employers and the preferential use of voluntary
modes in settling disputes, including conciliation, and shall enforce
their mutual compliance therewith to foster industrial peace.

When the parties have validly agreed on a procedure for resolving


grievances and to submit a dispute to voluntary arbitration then that
procedure should be strictly observed.

FACTS:
Fernandez, a seaman, filed with the NLRC a complaint for
disability benefits against Ace Navigation Co., Inc.

The petitioners moved to dismiss the complaint, contending that


the Labor Arbiter had no jurisdiction over the dispute. They argued that
exclusive original jurisdiction is with the voluntary arbitrator or panel of
voluntary arbitrators, pursuant to Section 29 of the POEA Standard
Employment Contract (POEA-SEC), since the parties are covered by

88
Page 89

the AMOSUP-TCC or AMOSUP-VELA (as later cited by the


petitioners) collective bargaining agreement (CBA). Under Section 14
of the CBA, a dispute between a seafarer and the company shall be
settled through the grievance machinery and mandatory voluntary
arbitration.

Fernandez then contended that what was involved was a money


claim. Thus, the original and exclusive jurisdiction over the case is
vested with the Labor Arbiter.

ISSUE:
Who was jurisdiction over the money claim of Fernandez, the
Labor Arbiter or the Voluntary Arbitrator?

RULING:
It is the Voluntary Arbitrator that has jurisdiction over the money
claim of Fernandez. Since the parties used unequivocal language in
their CBA for the submission of their disputes to voluntary arbitration
(a condition laid down in Vivero for the recognition of the submission
to voluntary arbitration of matters within the original and exclusive
jurisdiction of labor arbiters), the matter must be submitted for
arbitration with the Voluntary Arbitrator.

A matter that is within the exclusive jurisdiction of the Labor Arbiter


can be submitted for voluntary arbitration because of the clear
language of the parties’ CBA on the matter. Moreover, and more
importantly, the jurisdiction of the voluntary arbitrator is upheld in
recognition of the State’s express preference for voluntary modes of
dispute settlement, such as conciliation and voluntary arbitration as
expressed in the Constitution, the law and the rules. When the parties
have validly agreed on a procedure for resolving grievances and to
submit a dispute to voluntary arbitration then that procedure should be
strictly observed.

Thus, because the Collective Bargaining Agreement in this case


unequivocally provides that the matter involved in this case should first
be submitted to a Voluntary Arbitrator, the Labor Arbiter has no
jurisdiction over the subject matter pursuant to Art. 262 of the Labor
Code. Consistent with this finding, Fernandez’s contention — that his
complaint for disability benefits is a money claim that falls within the
original and exclusive jurisdiction of the labor arbiter under Section 10
of R.A. No. 8042 — is untenable.

89
Page 90

Case Digests by: Mar Lorenzo B. Asuncion

Estate of Nelson Dulay v. Aboitiz Jebsen Maritime, Inc.


G.R. No. 172642, June 13, 2012
Peralta, J.:

DOCTRINE:
VOLUNTARY ARBITRATION
When the parties have validly agreed on a procedure for resolving
grievances and to submit a dispute to voluntary arbitration then that
procedure should be strictly observed. While the suit filed by Merridy
Jane is a money claim, the same basically involves the interpretation
and application of the provisions in the subject CBA. As such,
jurisdiction belongs to the voluntary arbitrator and not the labor arbiter.

With respect to disputes involving claims of Filipino seafarers


wherein the parties are covered by a collective bargaining agreement,
the dispute or claim should be submitted to the jurisdiction of a
voluntary arbitrator or panel of arbitrators. It is only in the absence of a
collective bargaining agreement that parties may opt to submit the
dispute to either the NLRC or to voluntary arbitration.

FACTS:
Nelson was employed by General Charterers Inc., a subsidiary of
Aboitiz Jebsen Maritime, Inc. He initially worked as an ordinary
seaman and later as bosun on a contractual basis. Thereafter, Nelson
was detailed as part of the crew of MV Kickapoo Belle. 25 days after
the completion of the employment contract, Nelson died due to acute
renal failure secondary to septicemia. At the time of his death, Nelson
was a bona fide member of GCI’s collective bargaining agent. Merridy,
the widow of Nelson claimed for death benefits through the grievance
procedure under the CBA between the union and GC. Thereafter, the
grievance procedure was “declared deadlocked” as petitioners refused
to grant the benefits sought by the widow.

Merridy then filed a complaint with the NLRC Sub-Regional


Arbitration Board against GCI for death and medical benefits and
damages. Aboitiz contends that the NLRC had no jurisdiction over the
action on account of the absence of employer-employee relationship
between GCI and Nelson at the time of the latter’s death. Nelson also
had no claims against petitioners for sick leave allowance/medical
benefit by reason of the completion of his contract with GCI.

The Labor Arbiter and the NLRC granted the claim by Merridy as
to the grant of death benefits under the CBA. The CA, on a special civil
action for certiorari referred the case to the National Conciliation and

90
Page 91

Mediation Board (NCMB) for designation of a Voluntary Arbitrator. The


CA ruled that while the suit filed by Merridy Jane is a money claim, the
same basically involves the interpretation and application of the
provisions in the subject CBA. As such, jurisdiction belongs to the
voluntary arbitrator and not the labor arbiter.

ISSUE:
Does the Voluntary Arbitrator have jurisdiction over the claim?

RULING:
Yes. The parties, in the first place, really intended to bring to
conciliation or voluntary arbitration any dispute or conflict in the
interpretation or application of the provisions of their CBA. It is settled
that when the parties have validly agreed on a procedure for resolving
grievances and to submit a dispute to voluntary arbitration then that
procedure should be strictly observed.

The basic issue raised by Merridy Jane in her complaint filed with
the NLRC is: which provision of the subject CBA applies insofar as
death benefits due to the heirs of Nelson are concerned. This issue
clearly involves the interpretation or implementation of the said CBA.
Thus, the specific or special provisions of the Labor Code govern.

With respect to disputes involving claims of Filipino seafarers


wherein the parties are covered by a collective bargaining agreement,
the dispute or claim should be submitted to the jurisdiction of a
voluntary arbitrator or panel of arbitrators. It is only in the absence of a
collective bargaining agreement that parties may opt to submit the
dispute to either the NLRC or to voluntary arbitration. It is elementary
that rules and regulations issued by administrative bodies to interpret
the law which they are entrusted to enforce, have the force of law, and
are entitled to great respect. Such rules and regulations partake of the
nature of a statute and are just as binding as if they have been written
in the statute itself.

No less than the Philippine Constitution provides, under the third


paragraph, Section 3, Article XIII, thereof that “[t]he State shall promote
the principle of shared responsibility between workers and employers
and the preferential use of voluntary modes in settling disputes,
including conciliation, and shall enforce their mutual compliance
therewith to foster industrial peace.” Consistent with this constitutional
provision, Article 211 of the Labor Code provides the declared policy
of the State “[t]o promote and emphasize the primacy of free collective
bargaining and negotiations, including voluntary arbitration, mediation
and conciliation, as modes of settling labor or industrial disputes.”

91
Page 92

Case Digests by: Mar Lorenzo B. Asuncion

Light Rail Transit Authority v. Alvarez


G.R. No. 188047, November 28, 2016
Jardeleza, J.:

DOCTRINE
LIABILITY OF GOVERNMENT FOR SALARIES OF EMPLOYEES
By engaging in a particular business thru the instrumentality of a
corporation, the government divests itself pro hac vice of its sovereign
character, so as to render the corporation subject to the rules of law
governing private corporations. The LRTA must submit itself to the
provisions governing private corporations, including the Labor Code,
for having conducted business through a private corporation, in this
case, METRO.

[LRTA's] contractual commitments with [METRO] and its


employees arose out of its business relations with [METRO] which is
private in nature. Such private relation was not changed
notwithstanding the subsequent acquisition by [LRTA] of full ownership
of [METRO] and take-over of its business operations at LRT."

FACTS:
Light Rail Transit Authority (LRTA) is a government-owned and
controlled corporation (GOCC). Alvarez et. al. are former employees
of Meralco Transit Organization, Inc. (METRO)

METRO and LRTA entered into an agreement whereby LRTA


shouldered and provided for the operating expenses of METRO. Also,
METRO signed a Collective Bargaining Agreement (CBA) with its
employees wherein provisions on wage increases and benefits were
approved by LRTA's Board of Directors.

Thereafter, COA voided the provisions of the agreement. To


resolve the issue, LRTA acquired METRO by purchasing all of its
shares of stocks. METRO then became a wholly-owned subsidiary of
LRTA. Thereafter, LRTA and METRO declared and continued the
implementation of the agreement and the non-interruption of the
employment relations of the employees of METRO. hey likewise
continued the establishment and funding of the Metro, Inc. Employees
Retirement Plan which covers the past services of all METRO regular
employees from the date of their employment. They confirmed that all
CBAs remained in force and effect. LRTA then sanctioned the CBA of
the union of rank-and-file employees and the union of supervisory
employees.

92
Page 93

METRO then announced in a memorandum that its board of


directors approved the severance/resignation benefit of METRO
employees at one and a half salaries for every year of service.
Eventually, LRTA stopped the operation of METRO. COA issued an
Advisory Opinion that LRTA is liable, as owner of its wholly-owned
subsidiary METRO, to pay the severance pay of the latter's employees.
Although the first 50% of the severance pay of the private respondents
herein was paid, the balance thereon were no longer paid. Private
respondents repeatedly and formally asked LRTA, being the principal
owner of METRO, to pay the balance of their severance pay, but to no
avail.

Thus, Alvarez et. al. filed a complaint before the Arbitration Branch
of the NLRC praying for the payment of the balance of the benefits that
was not paid to them against LRTA and METRO. LRTA contends
among others that the LA and the NLRC have no jurisdiction over the
money claims of the private respondents.

ISSUE:
a. Does the LA and the NLRC have jurisdiction over the money
claims of the private respondents?

b. Can LRTA be made liable despite the absence of an employer-


employee relationship?

RULING:
a. Yes. By engaging in a particular business thru the
instrumentality of a corporation, the government divests itself
pro hac vice of its sovereign character, so as to render the
corporation subject to the rules of law governing private
corporations. The LRTA must submit itself to the provisions
governing private corporations, including the Labor Code, for
having conducted business through a private corporation, in
this case, METRO.

[LRTA's] contractual commitments with [METRO] and its


employees arose out of its business relations with [METRO]
which is private in nature. Such private relation was not
changed notwithstanding the subsequent acquisition by [LRTA]
of full ownership of [METRO] and take-over of its business
operations at LRT.

b. Yes. LRTA is solidarily liable with METRO for the payment of


the separation pay of the private respondents.

First, LRTA is contractually obligated to pay the retirement or


severance/resignation pay of METRO employees. In this case,

93
Page 94

it is the LRTA through its Board of Directors that approved the


severance/resignation benefit of METRO employees. LRTA
obligated itself to fund METRO's retirement fund to answer for
the retirement or severance/resignation of METRO employees
as part of METRO's "operating expenses."

Second, even if LRTA is not contractually liable to pay the


separation benefit, it is solidarily liable as an indirect employer
of the respondents. LRTA qualifies as an indirect employer by
contracting METRO to manage and operate the Metro Manila
light rail transit. Being an indirect employer, LRTA is solidarily
liable with METRO in accordance with Article 109 of the Labor
Code. The fact that there is no actual and direct employer-
employee relationship between LRTA and private respondents
does not absolve the former from liability for the latter's
monetary claims. The owner of the project is not the direct
employer but merely an indirect employer, by operation of law,
of his contractor's employees.

In this case, the pretermination of the contract was for reasons


not attributable to the contractor or subcontractor. The
agreement was not renewed through no fault of METRO, as it
was solely at the behest of LRTA. The fact is, under the
circumstances, METRO really had no choice on the matter,
considering that it was a mere subsidiary of LRTA.

94
Page 95

Case Digests by: Mar Lorenzo B. Asuncion

GSIS v. NLRC
G.R. No. 180045, November 17, 2010
Nachura, J.:

DOCTRINE:
SOLIDARY LIABILITY OF THE EMPLOYER OF AN INDEPENDENT
CONTRACTOR
An employer of an independent contractor can be held solidarily
liable for the monetary claims made by the employees of the latter
because he is an indirect employer. The joint and several liability of the
employer or principal was enacted to ensure compliance with the
provisions of the Code, principally those on statutory minimum wage.
The contractor or subcontractor is made liable by virtue of his or her
status as a direct employer, and the principal as the indirect employer
of the contractor’s employees. This liability facilitates, if not
guarantees, payment of the workers’ compensation, thus, giving the
workers ample protection as mandated by the 1987 Constitution. This
is not unduly burdensome to the employer. Should the indirect
employer be constrained to pay the workers, it can recover whatever
amount it had paid in accordance with the terms of the service contract
between itself and the contractor.

The liability however cannot extend to the payment of separation


pay. An order to pay separation pay is invested with a punitive
character such that an indirect employer should not be made liable
without finding that it had conspired in the illegal dismissal of the
employees.

FACTS:
Banlasan et. al., were employed as security guards by DNL
Security. DNL Security and GSIS entered into a service contract in
which the private respondents were assigned to the GSIS Tacloban
City office.

Thereafter, DNL Security informed respondents that its service


contract with GSIS was terminated. This notwithstanding, DNL
Security instructed respondents to continue reporting for work to
petitioner. Respondents worked as instructed but without receiving
their wages. Thereafter, they were terminated from employment. They
then filed a complaint against DNL Security and GSIS for illegal
dismissal. The Labor Arbiter rendered a decision against DNL Security
and GSIS finding that there was no illegal dismissal but ordered DNL
Security as well as GSIS to pay separation pay in favor of the

95
Page 96

respondents. The liability of GSIS was joint and solidary with DNL
Security.

ISSUE:
Can GSIS be held liable for the payment of the claims of the
private respondents?

RULING:
Yes. GSIS is an indirect employer. The fact that there is no actual
and direct employer-employee relationship between petitioner and
respondents does not absolve the former from liability for the latter’s
monetary claims. When petitioner contracted DNL Security’s services,
petitioner became an indirect employer of respondents, pursuant to
Article 107 of the Labor Code, which reads:

ART. 107. Indirect employer. – The provisions of the immediately


preceding Article shall likewise apply to any person, partnership,
association or corporation which, not being an employer, contracts with
an independent contractor for the performance of any work, task, job
or project.

After DNL Security failed to pay respondents the correct wages


and other monetary benefits, petitioner, as principal, became jointly
and severally liable, as provided in Articles 106 and 109 of the Labor
Code, which state:

ART. 106. Contractor or subcontractor. – Whenever an employer


enters into a contract with another person for the performance of the
former’s work, the employees of the contractor and of the latter’s
subcontractor, if any, shall be paid in accordance with the provisions
of this Code.

In the event that the contractor or subcontractor fails to pay the


wages of his employees in accordance with this Code, the employer
shall be jointly and severally liable with his contractor or subcontractor
to such employees to the extent of the work performed under the
contract, in the same manner and extent that he is liable to employees
directly employed by him. x x x.

ART. 109. Solidary liability. – The provisions of existing laws to the


contrary notwithstanding, every employer or indirect employer shall be
held responsible with his contractor or subcontractor for any violation
of any provision of this Code. For purposes of determining the extent
of their civil liability under this Chapter, they shall be considered as
direct employers.

96
Page 97

This statutory scheme is designed to give the workers ample


protection, consonant with labor and social justice provisions of the
1987 Constitution

The joint and several liability of the employer or principal was


enacted to ensure compliance with the provisions of the Code,
principally those on statutory minimum wage. The contractor or
subcontractor is made liable by virtue of his or her status as a direct
employer, and the principal as the indirect employer of the contractor’s
employees. This liability facilitates, if not guarantees, payment of the
workers’ compensation, thus, giving the workers ample protection as
mandated by the 1987 Constitution. This is not unduly burdensome to
the employer. Should the indirect employer be constrained to pay the
workers, it can recover whatever amount it had paid in accordance with
the terms of the service contract between itself and the contractor.

Note:

Although GSIS has liability that is joint and solidary with the liability
of DNL Security, it only covers salary differential and 13th month pay
during the time they worked for petitioner as well as unpaid wages from
February 1993 until April 20, 1993.

The liability however cannot extend to the payment of separation


pay. An order to pay separation pay is invested with a punitive
character such that an indirect employer should not be made liable
without finding that it had conspired in the illegal dismissal of the
employees.

97
Page 98

Case Digests by: Mar Lorenzo B. Asuncion

Duty Free Philippines v. Mojica


G.R. No. 166365, September 30, 2005
Ynares-Santiago, J.:

DOCTRINE:
DISMISSAL OF CIVIL SERVICE EMPLOYEE
If the employee alleged to have been illegally dismissed is a civil
service employee, the Labor Arbiter, the NLRC, nor the CA does not
have jurisdiction over the issue. The case is within the jurisdiction of
the Civil Service Commission. In this case, the procedure that
should’ve been followed is the procedure laid down in DFP's merit
system and the Civil Service rules and regulations by reason of his civil
service employment. Recourse to the LA and the NLRC should not
have been made.

FACTS:
Stock Clerk Mojica was found by the Discipline Committee of Duty
Free guilty of neglect of duty by causing considerable damage to or
loss of materials, assets and property DFP. Thus, Mojica was
considered forcibly resigned from the service with forfeiture of all
benefits except his salary and the monetary value of the accrued leave
credits.

Mojica was formally informed of his forced resignation. Thereafter,


he filed a complaint for illegal dismissal with prayer for reinstatement
and payment of full back wages among others with the NLRC. The
Labor Arbiter ruled that Mojica was illegally dismissed and thereafter
ordered his reinstatement which was reversed by the NLRC. The CA
reinstated the ruling of the Labor Arbiter agreeing that Mojica was not
guilty of gross negligence or habitual negligence which would warrant
his dismissal.

ISSUE:
Does the Labor Arbiter and the NLRC have jurisdiction to rule on
the legality of the dismissal of Mojica?

RULING:
No. Mojica is a civil service employee. Thus, jurisdiction is lodged
not with the NLRC but with the Civil Service Commission. Duty Free
Philippines was created under Executive Order No. 46 primary to
augment the service facilities for tourists and generate foreign
exchange and revenue for the government.

98
Page 99

Duty Free Philippines is under the exclusive authority of the


Philippine Tourism Authority and the latter is a corparte body attached
to the Department of Tourism. Thus, since DFP is under the exclusive
authority of the PTA, it follows that its officials and employees are
likewise subject to the Civil Service rules and regulations. Clearly then,
Mojica's recourse to the Labor Arbiter was not proper. He should have
followed the procedure laid down in DFP's merit system and the Civil
Service rules and regulations.

Thus, the Labor Arbiter, the NLRC, and the CA, erred in taking
cognizance of the complaint as jurisdiction over the complaint for illegal
dismissal is lodged with the Civil Service Commission since Mojica is
a civil service employee.

99
Page 100

Case Digests by: Mar Lorenzo B. Asuncion

WPP Marketing Communications, Inc. v. Galera


G.R. No. 169207, March 25, 2010
Carpio, Acting C.J.:

DOCTRINE:
DISMISSAL OF A FOREIGNER; EMPLOYMENT PERMIT
REQUIREMENT
A foreigner who is found to have been illegally dismissed is not
entitled to the monetary benefits provided for under our Labor laws if
such foreigner began employment here in the Philippines without first
complying with the obtention of an employment permit from the
Department of Labor which is required prior to employment.

Under Sec. 4 Rule XIV, Book 1 of the Implementing Rules and


Regulations: No alien seeking employment, whether as a resident or
non-resident, may enter the Philippines without first securing an
employment permit from the Ministry. If an alien enters the country
under a non-working visa and wishes to be employed thereafter, he
may only be allowed to be employed upon presentation of a duly
approved employment permit.

Thus, if there is a violation of the permit requirement, such person


cannot come to court and seek relief and obtain the benefits to which
such foreigner would have been entitled.

In this case, no employment permit was obtained by Galera prior


to employment. Galera cannot come to this Court with unclean hands.
To grant Galera’s prayer is to sanction the violation of the Philippine
labor laws requiring aliens to secure work permits before their
employment. The Supreme Court ruled that the status quo must prevail
in the present case and the Court left the parties where they are. This
ruling, however, does not bar Galera from seeking relief from other
jurisdictions.

100
Page 101

FACTS:
Galera is an American citizen who was recruited from the United
States of America by Steedman, the Chairman of WPP Worldwide and
CEO of Mindshare, Co., a corporation based in Hong Kong China to
work in the Philippines for WPP Marketing Communications, Inc.
(WPP)

Galera accepted the offer and she signed an Employment


Contract. The employment of Galera, a foreigner, became effective
solely on the instruction of the CEO and without obtention of the permit
as required for by law. She was then designated as Vice President of
the WPP. Thereafter, Galera was verbally notified by Steedman that
her services had been terminated from WPP. A termination letter
followed the next day. Galera filed a complaint for illegal dismissal. The
Labor Arbiter held the petitioners herein liable for illegal dismissal
ruling that not only was Galera illegally dismissed, Galera was also
denied due process. The NLRC reversed the ruling. The CA then
reversed the ruling of the NLRC and upheld the decision of the LA. It
rejected the contention of the NLRC that Galera was a corporate officer
for being a Vice President and thus, the dismissal was an intra-
corporate controversy.

The CA ruled that a person can be considered as a “corporate


officer” if appointed as such by a corporation’s Board of Directors, or if
pursuant to the power to them given by the Articles of Incorporation or
the By-Laws.

ISSUE:
a. Was Galera a corporate officer?
b. Assuming that Galera is not a corporate officer and the Labor
Arbiter has jurisdiction over the case and a finding of illegal
dismissal exists, should Galera be accorded the relief sought
for?

RULING:
a. No. Galera is not a corporate officer but a mere employee
although holding a high-ranking position. Corporate officers
under the [Revised] Corporation Code are expressly
enumerated. The President, Treasurer, and Secretary.
Likewise, those which are expressly created under the Articles
of Incorporation or By-laws of the corporation are considered
as corporate officers. However, for the fourth kind of corporate
officer, it is essential that there must be an express grant of
such corporate office under the Articles of Incorporation or the
by-laws of the corporation.

101
Page 102

If the person’s position is merely created under a general


provision under the Articles of Corporation or the by-laws
without expressly stating therein the position, then the person
is only an employee, albeit a high ranking one, and not a
corporate officer as contemplated under the Revised
Corporation Code, the termination of which would be an intra-
corporate controversy.

In this case, Galera is an employee. First, WPP’s by-laws


provided for only one Vice-President. At the time of
appointment of Galera, WPP already had a Vice-President in
the person of Webster. Galera cannot be said to be a director
of WPP also because all five directorship positions provided in
the by-laws are already occupied. Finally, WPP cannot rely on
its Amended By-Laws to support its argument that Galera is a
corporate officer. The Amended By-Laws provided for more
than one Vice-President and for two additional directors. Even
though WPP’s stockholders voted for the amendment on 31
May 2000, the SEC approved the amendments only on 16
February 2001. Galera was dismissed on 14 December 2000.
WPP, Steedman, Webster, and Lansang did not present any
evidence that Galera’s dismissal took effect with the action of
WPP’s Board of Directors.

Second, in applying the Control Test, Galera was under the


control and supervision of WPP not only with respect to the
results of the work done but also the manner by which such
work is to be done. Sections 1 and 4 of the employment
contract mandate where and how often she is to perform her
work; sections 3, 5, 6 and 7 show that wages she receives are
completely controlled by x x x WPP; and sections 10 and 11
clearly state that she is subject to the regular disciplinary
procedures of x x x WPP.

Third, the contract itself expressly provides that she is a


permanent employee, not a Vice-President nor a member of
the Board of Directors.

Thus, because Galera was an employee and not a corporate


officer, the illegal dismissal case is within the jurisdiction of the
Labor Arbiter and the NLRC.

b. No. Although there was a finding that there was illegal dismissal
on the part of WPP, the Supreme Court upheld the status quo
since a person who is seeking relief must come to court with
clean hands. Galera worked in the Philippines without a proper
work permit but now wants to claim employee’s benefits under
Philippine labor laws. Consequently, because Galera was

102
Page 103

working without the pre-requisite employment permit, Galera


cannot now avail of the protection accorded to employees
under the Labor Code.

Galera began employment without having any employment


permit. The law and the rules are consistent in stating that the
employment permit must be acquired prior to employment. The
Labor Code states: "Any alien seeking admission to the
Philippines for employment purposes and any domestic or
foreign employer who desires to engage an alien for
employment in the Philippines shall obtain an employment
permit from the Department of Labor.

Galera cannot come to this Court with unclean hands. To grant


Galera’s prayer is to sanction the violation of the Philippine
labor laws requiring aliens to secure work permits before their
employment. We hold that the status quo must prevail in the
present case and we leave the parties where they are. This
ruling, however, does not bar Galera from seeking relief from
other jurisdictions.

103
Page 104

Case Digests by: CHAVEZ, Jordan N.

PAKISTAN INTERNATIONAL AIRLINES CORP. vs.


Hon. BLAS F. OPLE et. al.
G.R. No. 223825. January 20, 2020.
Reyes, Jr. J.

DOCTRINE:
Art. 224 – Jurisdiction of the Labor Arbiter;
The employer-employee relationship is much affected with public
interest and that the otherwise applicable Philippine laws and
regulations cannot be rendered illusory by the parties agreeing upon
some other law to govern their relationship.

FACTS:
Petitioner Pakistan International Airlines Corp. (PIA) executed in
Manila two separate contracts of employment, one with private
respondent Ethelynne B. Farrales and the other with private
respondent Ma. M.C. Mamasig. The agreement is for a period of three
years. On August 2, 1980, roughly one year and four months prior to
the expiration of the contract of employment, petitioner PIA terminated
the contract with private respondents. Private respondents Farrales
and Mamasig jointly instituted a complaint for illegal dismissal and non-
payment of company benefits and bonuses. The Regional Director
ordered the reinstatement of private respondents with full backwages
or the payment of the amount equivalent to their salaries for the
remainder of the three-year contract. The RD ruled that private
respondents had attained the status of regular employee after they had
rendered more than a year of continued service. On appeal, the Deputy
Minister of the Ministry on Labor and Employment affirmed the RD
ruling. Hence, Petitioner PIA filed a Petition for Certiorari before the
Supreme Court. Petitioner argued, among others, that the Regional
Director, MOLE had no jurisdiction over the subject matter of the
complaint initiated by private respondents for illegal dismissal,
jurisdiction over the same being lodged in the Arbitration Branch of the
National Labor Relations Commission.

ISSUE:
(1) Whether the Regional Director, MOLE had jurisdiction over the
controversy?

(2) Whether the general provisions of the Labor Code applies in


the case at bar?

104
Page 105

RULING:

(1) Yes, the Supreme Court ruled that the Regional Director,
MOLE had jurisdiction over the case at bar.

Article 278 of the Labor Code, as it then existed, forbade the


termination of the services of employees with at least one (1) year of
service without prior clearance from the Department of Labor and
Employment. Rule XIV, Book No. 5 of the Rules and Regulations
Implementing the Labor Code made clear that in case of a termination
without the necessary clearance, the Regional Director was authorized
to order the reinstatement of the employee concerned and the payment
of backwages; necessarily, therefore, the Regional Director must have
been given jurisdiction over such termination cases.

(2) Yes, the Supreme Court ruled that the general provisions
of the Philippine Labor Code applies in the case at bar.

Petitioner PIA cannot take refuge in paragraph 10 of its


employment agreement which specifies, firstly, the law of Pakistan as
the applicable law of the agreement and, secondly, lays the venue for
settlement of any dispute arising out of or in connection with the
agreement "only in courts of Karachi, Pakistan". The first clause of
paragraph 10 cannot be invoked to prevent the application of Philippine
labor laws and regulations to the subject matter of this case, i.e., the
employer-employee relationship between petitioner PIA and private
respondents. Such relationship is much affected with public interest
and that the otherwise applicable Philippine laws and regulations
cannot be rendered illusory by the parties agreeing upon some other
law to govern their relationship. Neither may petitioner invoke the
second clause of paragraph 10, specifying the Karachi courts as the
sole venue for the settlement of disputes between the contracting
parties. Even a cursory scrutiny of the relevant circumstances of this
case will show the multiple and substantive contacts between
Philippine law and Philippine courts, on the one hand, and the
relationship between the parties, upon the other: the contract was not
only executed in the Philippines, it was also performed here, at least
partially; private respondents are Philippine citizens and residents,
while petitioner, although a foreign corporation, is licensed to do
business (and actually doing business) and hence resident in the
Philippines; lastly, private respondents were based in the Philippines
in between their assigned flights to the Middle East and Europe.

105
Page 106

All the above contacts point to the Philippine courts and


administrative agencies as a proper forum for the resolution of
contractual disputes between the parties. Under these circumstances,
paragraph 10 of the employment agreement cannot be given effect to
oust Philippine agencies and courts of the jurisdiction vested upon
them by Philippine law.

106
Page 107

Case Digests by: CHAVEZ, JORDAN N.

PHILIPPINE NATIONAL BANK


vs. FLORENCE O. CABANSAG
G.R. No. 157010. June 21, 2005.
Panganiban, J.

DOCTRINE:
Art. 224. Jurisdiction of the Labor Arbiter.
All Filipino workers, whether employed locally or overseas, enjoy the
protective mantle of the Philippine labor and social legislations. Our
labor statutes may not be rendered ineffective by laws or judgments
promulgated, or stipulations agreed upon, in a foreign country.

FACTS:
In 1998, respondent Cabansag arrived in Singapore as a tourist.
She was then accepted for an employment as a Branch Credit Officer
with the Singapore Branch of the Philippine National Bank. The
Philippine Embassy in Singapore processed the employment contract
of Cabansag. Barely three (3) months in office, she was terminated
from employment with the Bank. She filed a complaint for illegal
dismissal. The Labor Arbiter ruled the respondent was illegally
dismissed from employment. On appeal, the NLRC affirmed the
decision of the Labor Arbiter. On review, the Court of Appeals the
decisions of the NLRC and the LA. The CA ruled that respondent did
not waive Philippine labor laws and in so doing, neither did she submit
herself solely to the jurisdiction of the Ministry of Manpower of
Singapore over disputes arising from her employment.

ISSUE:
(1) Whether the Labor Arbiter of the NLRC has jurisdiction
over the case?

(2) Whether the respondent was illegally dismissed?

RULING:
(1) Yes, the Supreme Court ruled that the Labor Arbiter of the
NLRC has jurisdiction over the controversy.

Under Art. 217 (now 224) of the Labor Code and Sec. 10, R.A.
8042, the labor arbiter clearly has original and exclusive jurisdiction
over claims arising from employer-employee relations, including

107
Page 108

termination disputes involving all workers, among whom are Overseas


Filipino Workers (OFW).

Respondent applied for and secured an Overseas Employment


Certificate from the POEA through the Philippine Embassy in
Singapore. Under Philippine law, this document authorized her working
status in a foreign country and entitled her to all the benefits and
process under our statutes. Moreover, petitioner is a Philippine
corporation doing business through a branch office in Singapore. This
circumstance militates against petitioner’s contention that respondent
was locally hired in Singapore and governed only by the laws of
Singapore. Instead, with more reason does this fact reinforce the
presumption that respondent falls under the legal definition of migrant
worker, in this case one deployed in Singapore.

(2) Yes, the Supreme Court also ruled that respondent was
illegally dismissed from employment.

Petitioner PNB has not asserted any of the grounds provided by


law as a valid reason for terminating the employment of respondent. It
merely insists that her dismissal was validly effected pursuant to the
provisions of her employment Contract, which she had voluntarily
agreed to be bound to.

The appellate court was correct in holding that respondent was


already a regular employee at the time of her dismissal, because her
three-month probationary period of employment had already ended.
This ruling is in accordance with Art. 281 of the Labor Code: “An
employee who is allowed to work after a probationary period shall be
considered a regular employee”. Indeed, petitioner recognized
respondent as such at the time it dismissed her, by giving her one
month’s salary in lieu of a one-month notice, consistent with her
employment contract.

108
Page 109

Case Digests by: CHAVEZ, Jordan N.

MANILA HOTEL CORP. and NLRC


G.R. No. 120077. October 13, 2000.
Pardo, J.

DOCTRINE:

RULE OF FORUM NON CONVENIENS.


Under the rule of forum non conveniens, a Philippine court or
agency may assume jurisdiction over the case if it chooses to do so
provided: (1) that the Philippine court is one to which the parties may
conveniently resort to; (2) that the Philippine court is in a position to
make an intelligent decision as to the law and the facts; and (3) that
the Philippine court has or is likely to have power to enforce its
decision. The conditions are unavailing in the case at bar.

FACTS:

During the employment of private respondent Marcelo Santos as


an overseas worker as a printer in the Sultanate of Oman, respondent
was recruited by Mr. Shmidt to work in the Palace Hotel, Beijing,
People’s Republic of China. Private respondent Santos resigned in his
work in Oman and accepted the work in the Palace Hotel, Beijing. The
employment contract with the Palace Hotel stated that his employment
would commence September 1, 1988 for a period of two years. On
September 5, 1989, the Palace Hotel terminated the employment of
respondent Santos and paid all benefits due him, including his plane
fare back to the Philippines. Respondent, then, filed a complaint for
Illegal Dismissal with the Arbitration Branch, NCR of the NLRC. The
Palace Hotel and Mr. Shmidt were not served with summons and
neither participated in the proceedings before the Labor Arbiter. The
Labor Arbiter ruled that Santos was illegally dismissed and granted
damages. On appeal, the NLRC reversed the LA ruling.

ISSUE/S:

Whether the Labor Arbiter and the NLRC has jurisdiction over
the controversy?

109
Page 110

HELD:

No, the Supreme Court ruled that the Labor Arbiter and the
NLRC has no jurisdiction over the controversy. The NLRC was a
seriously inconvenient forum.

We note that the main aspects of the case transpired in two


foreign jurisdictions and the case involves purely foreign elements. The
only link that the Philippines has with the case is that respondent
Santos is a Filipino citizen. The Palace Hotel and MHICL are foreign
corporations. Not all cases involving our citizens can be tried here.

The Court fail to see how the NLRC is a convenient forum given
that all the incidents of the case, from the time of recruitment, to
employment to dismissal occurred outside the Philippines. The
inconvenience is compounded by the fact that the proper defendants,
the Palace Hotel and MHICL are not nationals of the Philippines.
Neither can an intelligent decision be made as to the law governing the
employment contract as such was perfected in a foreign soil. The
employment contract was not perfected in the Philippines. Respondent
Santos signified his acceptance by writing a letter while he was in the
Republic of Oman. Neither can the NLRC determine the facts
surrounding the alleged illegal dismissal as all acts complained of took
place in Beijin, People’s Republic of China. The NLRC was not an in
position to determine whether the Tiannamen Square incident truly
adversely affected operations of the Palace Hotel as to justify
respondent Santos’ retrenchment.

Even assuming that a proper decision could be reached by the


NLRC, such would not have any binding effect against the employer,
the Palace Hotel. The Palace Hotel is a corporation incorporated under
the laws of Chine and was not even served with summons. Jurisdiction
over its person was not acquired.

The Court clarified that Philippine courts and agencies have no


power to solve controversies involving foreign employers. If Santos
were an overseas contract worker, a Philippine forum, specifically the
POEA, not the NLRC, would protect him. However, private respondent
Santos was not an overseas contract worker.

110
Page 111

Case Digests by: CHAVEZ, Jordan N.

SAUDI ARABIAN AIRLINES (SAUDIA) vs.


MA. JOPETTE M. REBESENCIO, et. al.
G.R. No. 198587. January 14, 2015.
Leonen, J.

DOCTRINE:
Art. 224 – Jurisdiction of the Labor Arbiter;
While a Philippine tribunal is called upon to respect the parties
choice of governing law, such respect must not be so permissive as to
lose sight of consideration of law, morals, good customs, public order
and public policy. It is settled that contracts relating to labor and
employment are impressed with public policy. it is settled that contracts
relating to labor and employment are impressed with public policy.

FACTS:
Petitioner Saudi Arabian Airlines (Saudia) is a foreign corporation
who has an office located in Makati, Philippines. Respondents were
recruited and hired by Saudia as Temporary Flight Attendants.
Afterwards, they become Permanent Flight Attendants. They, then,
entered into Cabin Attendant contracts with Saudia. Respondents
continued their employment with Saudia until they were separated from
service on various dates in 2006. On Nov. 8, 2007, respondents filed
a complaint against Saudia for illegal dismissal. They argued that their
termination was made solely because they were pregnant. Saudia
assailed the jurisdiction of the Labor Arbiter. it claimed that all the
determining points of contract referred to a foreign law and insisted that
the complaint ought to be dismissed on the ground of forum non
conveniens. The Labor Arbiter dismissed respondents’ complaint. On
appeal, the NLRC reversed the LA ruling and ruled that considering
that complainants-appellants are OFWs, the LA and NLRC has
jurisdiction to hear and decide their complaint for termination. The
Court of Appeals affirmed the NLRC ruling with modification.

ISSUE:
(1) Whether the LA and the NLRC has jurisdiction over Saudia
and apply Philippine law in adjudicating the present dispute?

(2) Whether respondents were illegally terminated?

RULING:

111
Page 112

(1) Yes, the Supreme Court ruled that the LA and the NLRC
had jurisdiction over Saudi and Philippine law applies in
adjudicating the present dispute.

The summons were validly served and jurisdiction over it was


validly acquired. By its own admission, Saudia, while a foreign
corporation, has a Philippine office. Sec. 3 (d) of the Foreign
Investment Act leads to no other conclusion that Saudia is a foreign
corporation doing business in the Philippines. As such, Saudia maybe
sued in the Philippines and is subject to the jurisdiction of Philippine
tribunals. Contractual choice of law is not determinative of jurisdiction.
Stipulations on the laws of a given jurisdiction as the governing law of
contract does not preclude the exercise of jurisdiction by tribunals
elsewhere. A choice of law governing the validity of contracts or the
interpretation of its provision does not necessarily imply forum non
conveniens (forum is inconvenient). While a Philippine tribunal is called
upon to respect the parties choice of governing law, such respect must
not be so permissive as to lose sight of consideration of law, morals,
good customs, public order and public policy. It is settled that contracts
relating to labor and employment are impressed with public policy. it is
settled that contracts relating to labor and employment are impressed
with public policy. As the present dispute relates to the illegal
termination of respondent’s employment, the case is immutably a
matter of public interest and public policy. Philippine laws properly find
application in and govern this case. it cannot be said that the local
judicial machinery is inadequate for effectuating the rights sought to be
maintained.

(2) Yes, the Supreme Court ruled that respondents were


illegally terminated.

The termination of respondent’s employment happened when they


were pregnant and expecting to incur costs on account of child delivery
and infant rearing. A constructive dismissal is defined as the cessation
of work because continued employment is rendered impossible,
unreasonable or unlikely, as an offer involving a demotion in rank or
diminution in pay and other benefits. The threat of termination is
enough to compel a reasonable person in respondents’ position to give
up his or her employment. It is clear that respondents intended to
remain employed with Saudia. All they did was avail of their maternity
leaves. The threat of termination is enough to compel a reasonable
person in respondents' position to give up his or her employment.

112
Page 113

Case Digests by: CHAVEZ, Jordan N.

CONTINENTAL MICRONESIA INC. vs.


JOSEPH BASSO
G.R. No. 223825. January 20, 2020.
Reyes, Jr. J.

DOCTRINE:
Art. 224 – Jurisdiction of the Labor Arbiter;
That the employment contract of Basso was replete with
references to US laws, and that it originated from and was returned to
the US, do not automatically preclude our labor tribunals from
exercising jurisdiction to hear and try this case.

FACTS:
Petitioner Continental Micronesia Inc. (CMI) is a foreign
corporation licensed to do business in the Philippines. Respondent
Joseph Basso, a US citizen residing in the Philippines, was employed
as General Manager of Continental Airlines Inc. On Nov. 7, 1992,
Petitioner CMI took over the Philippine operations of Continental, with
Basso retaining his position as General Manager. In 1995, respondent
Basso was relieved as General Manager and was instead offered the
position of consultant to CMI. Basso filed a Complaint for Illegal
Dismissal. Alleging the presence of foreign elements, CMI filed a
Motion to Dismiss on the ground of lack of jurisdiction over the person
of CMI and the subject matter of the controversy. The Labor Arbiter
dismissed the case for lack of merit and jurisdiction. On appeal, the
NLRC reversed the LA ruling and ruled that the Labor Arbiter acquired
jurisdiction over the controversy when CMI voluntarily submitted to his
office’s jurisdiction by presenting evidence. The Court of Appeals
affirmed the NLRC ruling and ruled that the Labor Arbiter and NLRC
had jurisdiction over the subject matter of the case and over the parties.

ISSUE:
(1) Whether the Labor Arbiter and the NLRC had jurisdiction to
hear and try the illegal dismissal case?

(2) Whether the local forum is the convenient forum in light of the
facts of the case?

113
Page 114

RULING:

(1) Yes, the Supreme Court ruled that the Labor Arbiter and
the NLRC had jurisdiction to hear and try the illegal dismissal
case.

The Court agreed with CMI that there is a conflict of laws issue
that needs to be resolved first. Where the facts establish the existence
of foreign elements, the case presents a conflict of laws issue. The
foreign element in a case may appear in different forms, such as in this
case, where one of the parties is an alien and the other is domiciled in
another case. That the employment contract of Basso was replete with
references to US laws, and that it originated from and was returned to
the US, do not automatically preclude our labor tribunals from
exercising jurisdiction to hear and try this case. The case stemmed
from an illegal dismissal complaint. The Labor Code, under Art. 217,
clearly vests original and exclusive jurisdiction to hear and decide
cases involving termination disputes to the Labor Arbiter. Hence, the
Labor Arbiter and the NLRC have jurisdiction over the subject matter
of the case. The Labor Arbiter also acquired jurisdiction over the
person of Basso, notwithstanding his citizenship, when he filed his
complaint against CMI. Jurisdiction over the person of CMI was
acquired through the coercive process of service of summons.
Considering that the Labor Arbiter and the NLRC have jurisdiction over
the parties and the subject matter of the case, these tribunals may
proceed to try the case even if the rules of conflict of laws or the
convenience of the parties point to a foreign forum.

(2) Yes, the Supreme Court ruled that the local forum is the
convenient forum in light of the facts of the case.

Basso may conveniently resort to our labor tribunals as he and


CMI had physical presence in the Philippines during the duration of the
trial. CMI has a Philippine branch, while Basso, before his death, was
residing here. Thus, it could be reasonably expected that no
extraordinary measures were needed for the parties to make
arrangements in advocating their respective cases. The labor tribunals
can make an intelligent decision as to the law and facts. The dismissal
of Basso happened in the Philippines, the surrounding circumstances
of which can be ascertained without having to leave the Philippines.
The acts that allegedly led to loss of trust and confidence and Basso's
eventual dismissal were committed in the Philippines. As to the law,
we hold that Philippine law is the proper law of the forum, as we shall
discuss shortly. Also, the labor tribunals have the power to enforce
their judgments because they acquired jurisdiction over the persons of
both parties.

114
Page 115

Case Digests by: CHAVEZ, Jordan N.

JULIUS KAWACHI AND GAYLE KAWACHI


vs. DOMINIE DEL QUERO
G.R. No. 163768. March 27, 2007.
Tinga, J.

DOCTRINE:
Art. 224 – Jurisdiction of the Labor Arbiter; REASONABLE
CAUSAL CONNECTION RULE.
Under the reasonable causal connection rule, if there is a
reasonable causal connection between the claim asserted and the
employer- employee relations, then the case is within the jurisdiction
of our labor courts. In the absence of such nexus, it is the regular courts
that have jurisdiction.

FACTS:
Virgilio Kawachi hired private respondent Dominie Del Quero as
clerk of the pawnshop. On certain occasions, she worked beyond the
regular working hours but was not paid the corresponding overtime
pay. On August 10, 2002, petitioner Julius Kawachi scolded private
respondent Del Quero in front of many people about the way she
treated the customers of the pawnshop and afterwards terminated her
employment without affording her due process. Private respondent Del
Quero filed an action for damages against petitioners Kawachi before
the MeTC of Quezon City. Petitioners moved for the dismissal of the
complaint on the grounds of lack of jurisdiction and forum-shopping or
splitting of causes of action. The MeTC ruled in favor of private
respondent. On appeal, the RTC uphold the jurisdiction of the MeTC
over private respondent’s complaint for damages. Petitioner filed a
petition for review on certiorari before the SC. Petitioners argued that
the NLRC has jurisdiction over the action for damages because the
alleged injury is work-related. They also content that private
respondent should not be allowed to split her causes of action by filing
the action for damages separately from the labor case.

ISSUE:
Whether the MeTC has jurisdiction over the controversy?

RULING:
No, the Supreme Court ruled that the MeTC has no jurisdiction
over the controversy.

115
Page 116

Article 217(a) of the Labor Code, as amended, clearly bestows


upon the Labor Arbiter original and exclusive jurisdiction over claims
for damages arising from employer-employee relations — in other
words, the Labor Arbiter has jurisdiction to award not only the reliefs
provided by labor laws, but also damages governed by the Civil Code.

In San Miguel, the Court noted what was then the current trend,
and still is, to refer worker-employer controversies to labor courts,
unless unmistakably provided by the law to be otherwise. Because of
this trend, jurisprudence has developed the "reasonable causal
connection rule." Under this rule, if there is a reasonable causal
connection between the claim asserted and the employer- employee
relations, then the case is within the jurisdiction of our labor courts. In
the absence of such nexus, it is the regular courts that have jurisdiction.

The allegations in private respondent's complaint for damages


show that her injury was the offshoot of petitioners' immediate harsh
reaction as her administrative superiors to the supposedly sloppy
manner by which she had discharged her duties. Petitioners' reaction
culminated in private respondent's dismissal from work in the very
same incident. The allegations in private respondent's complaint
unmistakably relate to the manner of her alleged illegal dismissal.

The NLRC has jurisdiction over private respondent's complaint for


illegal dismissal and damages arising therefrom. She cannot be
allowed to file a separate or independent civil action for damages
where the alleged injury has a reasonable connection to her
termination from employment. Consequently, the action for damages
filed before the MeTC must be dismissed.

116
Page 117

Case Digests by: CHAVEZ, Jordan N.

PERPETUAL HELP CREDIT COOPERATIVE, INC.


vs. BENEDICTO FABURADA, et. al.
G.R. No. 121948. October 08, 2001.
Sandoval-Gutierrez, J.

DOCTRINE:
Art. 224 – Jurisdiction of the Labor Arbiter; IN RELATION TO
COOPERATIVES.
Art. 121 of the Cooperative Code of the Philippines and Sec. 8 of
the Cooperative Development Authority Law apply members, officers
and directors of the cooperative involved in disputes within a
cooperative or between cooperatives. There is no evidence that private
respondents are members of petitioner PHCCI and even if they are,
the dispute is about payment of wages, overtime pay, rest day and
termination of employment. Under Art. 217 of the Labor Code, these
disputes are within the original and exclusive jurisdiction of the Labor
Arbiter.

FACTS:
Private respondents Benedicto Faburada, Sisinita Vilar, Imelda
Tamayo and Harold Catipay filed a complaint against the Perpetual
Help Credit Cooperative, Inc. (PHCCI), petitioner, with the Arbitration
Branch, Department of Labor and Employment (DOLE), Dumaguete
City, for illegal dismissal, premium pay on holidays and rest days,
separation pay, wage differential, moral damages, and attorney's fees.

Petitioner PHCCI filed a motion to dismiss the complaint on the


ground that there is no employer-employee relationship between them
as private respondents are all members and co-owners of the
cooperative. The Labor Arbiter ruled that private respondent was
illegally dismissed from the cooperative. On appeal, the NLRC,
affirmed the LA ruling. Petitioner filed a petition for review on certiorari
before the Supreme Court. Petitioner PHCCI argued that labor arbiter
has no jurisdiction to take cognizance of the complaint of private
respondents considering that they failed to submit their dispute to the
grievance machinery as required by P.D. 175.

117
Page 118

ISSUE:
(1) Whether the Labor Arbiter has jurisdiction over the
controversy?

(2) Whether there is an employer-employee relationship between


PHCCI and private respondents and that private respondents were
illegally dismissed?

RULING:
(1) Yes, the Supreme Court ruled that the Labor Arbiter has
jurisdiction over the controversy.

P.D. 175 (strengthening the Cooperative Movement) and R.A.


6939 (or the Cooperative Development Authority Law) apply to
members, officers and directors of the cooperative or between
cooperatives.

There is no evidence that private respondents are members of


petitioner PHCCI and even if they are, the dispute is about payment of
wages, overtime pay, rest day and termination of employment. Under
Art. 217 of the Labor Code, these disputes are within the original and
exclusive jurisdiction of the Labor Arbiter.

(2) Yes, the Supreme Court ruled that an employer-employee


relationship exists between the parties and those private
respondents were illegally dismissed from employment in the
cooperative.

In determining the existence of an employer-employee


relationship, the following elements are considered: (1) the selection
and engagement of the worker or the power to hire; (2) the power to
dismiss; (3) the payment of wages by whatever means; and (4) the
power to control the worker's conduct, with the latter assuming primacy
in the overall consideration. Petitioner PHCCI hired private
respondents to work for it. They worked regularly on regular working
hours, were assigned specific duties, were paid regular wages and
made to accomplish daily time records just like any other regular
employee. They worked under the supervision of the cooperative
manager.

Private respondents were dismissed not for just or authorized


causes under the Labor Code. They were dismissed because
petitioner considered them to be mere voluntary workers, being its
members, and as such work at its pleasure. Procedural due process
was also violated. Only one notice was served upon private
respondents by petitioner.

118
Page 119

Article 217(a) of the Labor Code, as amended, clearly bestows


upon the Labor Arbiter original and exclusive jurisdiction over claims
for damages arising from employer-employee relations — in other
words, the Labor Arbiter has jurisdiction to award not only the reliefs
provided by labor laws, but also damages governed by the Civil Code.

In San Miguel, the Court noted what was then the current trend,
and still is, to refer worker-employer controversies to labor courts,
unless unmistakably provided by the law to be otherwise. Because of
this trend, jurisprudence has developed the "reasonable causal
connection rule." Under this rule, if there is a reasonable causal
connection between the claim asserted and the employer- employee
relations, then the case is within the jurisdiction of our labor courts. In
the absence of such nexus, it is the regular courts that have jurisdiction.

The allegations in private respondent's complaint for damages


show that her injury was the offshoot of petitioners' immediate harsh
reaction as her administrative superiors to the supposedly sloppy
manner by which she had discharged her duties. Petitioners' reaction
culminated in private respondent's dismissal from work in the very
same incident. The allegations in private respondent's complaint
unmistakably relate to the manner of her alleged illegal dismissal.

The NLRC has jurisdiction over private respondent's complaint for


illegal dismissal and damages arising therefrom. She cannot be
allowed to file a separate or independent civil action for damages
where the alleged injury has a reasonable connection to her
termination from employment. Consequently, the action for damages
filed before the MeTC must be dismissed.

119
Page 120

Case Digests by: CHAVEZ, Jordan N.

7K CORPORATION vs. EDDIE ALBARICO


G.R. No. 182295. June 26, 2013.
Sereno, C.J.

DOCTRINE:
Art. 224 – Jurisdiction of the Labor Arbiter;
Although the general rule under the Labor Code gives the labor
arbiter exclusive and original jurisdiction over termination disputes, it
also recognizes exceptions. One of the exceptions is provided in Article
262 of the Labor Code.

FACTS:
Respondent Eddie Albarico was a regular employee of petitioner
7K Corporation, a company selling water purifiers. In April 1993,
petitioner 7K Corp. terminated Albarico’s employment allegedly for his
poor sales performance. Respondent had to stop working for work. He
subsequently submitted his money claims against petitioner for
arbitration before the National Conciliation and Mediation Board
(NCMB). The issue for voluntary arbitration was whether respondent
Albarico was entitled to the payment of separation pay and the sales
commission reserved form him by the corporation. While the NCMB
arbitration case was pending, respondent Albarico filed a Complaint
against petitioner with the NLRC for illegal dismissal with money
claims. The Labor Arbiter ruled in favor of respondent Albarico. On
appeal, the NLRC reversed the LA ruling on the ground of forum
shopping on the part of respondent Albarico because the NCMB
arbitration case was still pending. In its position paper filed with the
NCMB, petitioner denied that respondent was terminated from work.
Petitioner claimed that respondent abandoned his work. Almost 12
years after the filing of the NCMB case, both parties appeared in a
hearing before the NCMB manifesting that they want to settle amicably,
however, with varying demands from each other. The NCMB voluntary
arbitrator rendered a Decision finding petitioner corporation liable for
illegal dismissal. In lieu of reinstatement, the NCMB ordered the
payment of separation pay for two years and backwages. On appeal,
the Court of Appeals affirmed the Decision of the voluntary arbitrator.

120
Page 121

ISSUE:
(1) Whether the voluntary arbitrator may assume jurisdiction over
a termination dispute.

(2) Whether the voluntary arbitrator properly assumed jurisdiction


to decide the issue of the legality of the dismissal of respondent as
well as the latter’s entitlement to backwages, even if neither the
legality nor the entitlement was expressly claimed in the Submission
Agreement by the parties.

RULING:
(1) Yes, the voluntary arbitrator may, by agreement of the
parties, assume jurisdiction over a termination dispute.

Although the general rule under the Labor Code gives the labor
arbiter exclusive and original jurisdiction over termination disputes, it
also recognizes exceptions. One of the exceptions is provided in Article
262 of the Labor Code. Art. 262 of the Labor Code provides that the
Voluntary Arbitrator or panel of Voluntary Arbitrators, upon agreement
of the parties, shall also hear and decide all other labor disputes
including unfair labor practices and bargaining deadlocks. In San Jose
vs. NLRC, the Court explained that the labor disputes referred to in the
same Art. 262 can include all those disputes mentioned in Art. 217 over
which the Labor Arbiter has original and exclusive jurisdiction.

(2) Yes, the Court ruled the voluntary arbitrator properly


assumed jurisdiction to decide the issue of the illegality of the
dismissal of respondent as well as the latter’s entitlement to back
wages.

Under the Labor Code, separation pay may be given not only
when there is illegal dismissal. In fact, it is also given to employees
who are terminated for authorized causes. The circumstances of
respondent Albarico for separation pay was premised to no other
conclusion than that the claim of respondent Albarico for separation
pay was premised on his allegation of illegal dismissal. The voluntary
arbitrator may award backwages upon a finding of illegal dismissal,
even though the issue of entitlement thereto is not explicitly claimed in
the Submission Agreement. Since arbitration is a final resort for the
adjudication of disputes, the voluntary arbitrator in the present case
can assume that he has the necessary power to make a final
settlement.

121
Page 122

Case Digests by: CHAVEZ, Jordan N.

JULIUS KAWACHI AND GAYLE KAWACHI


vs. DOMINIE DEL QUERO
G.R. No. 163768. March 27, 2007.
Tinga, J.

DOCTRINE:
Art. 224 – Jurisdiction of the Labor Arbiter; REASONABLE
CAUSAL CONNECTION RULE.
Under the reasonable causal connection rule, if there is a
reasonable causal connection between the claim asserted and the
employer- employee relations, then the case is within the jurisdiction
of our labor courts. In the absence of such nexus, it is the regular courts
that have jurisdiction.

FACTS:
Virgilio Kawachi hired private respondent Dominie Del Quero as
clerk of the pawnshop. On certain occasions, she worked beyond the
regular working hours but was not paid the corresponding overtime
pay. On August 10, 2002, petitioner Julius Kawachi scolded private
respondent Del Quero in front of many people about the way she
treated the customers of the pawnshop and afterwards terminated her
employment without affording her due process. Private respondent Del
Quero filed an action for damages against petitioners Kawachi before
the MeTC of Quezon City. Petitioners moved for the dismissal of the
complaint on the grounds of lack of jurisdiction and forum-shopping or
splitting of causes of action. The MeTC ruled in favor of private
respondent. On appeal, the RTC uphold the jurisdiction of the MeTC
over private respondent’s complaint for damages. Petitioner filed a
petition for review on certiorari before the SC. Petitioners argued that
the NLRC has jurisdiction over the action for damages because the
alleged injury is work-related. They also content that private
respondent should not be allowed to split her causes of action by filing
the action for damages separately from the labor case.

ISSUE:
Whether the MeTC has jurisdiction over the controversy?

RULING:
No, the Supreme Court ruled that the MeTC has no jurisdiction
over the controversy.

122
Page 123

Article 217(a) of the Labor Code, as amended, clearly bestows


upon the Labor Arbiter original and exclusive jurisdiction over claims
for damages arising from employer-employee relations — in other
words, the Labor Arbiter has jurisdiction to award not only the reliefs
provided by labor laws, but also damages governed by the Civil Code.

In San Miguel, the Court noted what was then the current trend,
and still is, to refer worker-employer controversies to labor courts,
unless unmistakably provided by the law to be otherwise. Because of
this trend, jurisprudence has developed the "reasonable causal
connection rule." Under this rule, if there is a reasonable causal
connection between the claim asserted and the employer- employee
relations, then the case is within the jurisdiction of our labor courts. In
the absence of such nexus, it is the regular courts that have jurisdiction.

The allegations in private respondent's complaint for damages


show that her injury was the offshoot of petitioners' immediate harsh
reaction as her administrative superiors to the supposedly sloppy
manner by which she had discharged her duties. Petitioners' reaction
culminated in private respondent's dismissal from work in the very
same incident. The allegations in private respondent's complaint
unmistakably relate to the manner of her alleged illegal dismissal.

The NLRC has jurisdiction over private respondent's complaint for


illegal dismissal and damages arising therefrom. She cannot be
allowed to file a separate or independent civil action for damages
where the alleged injury has a reasonable connection to her
termination from employment. Consequently, the action for damages
filed before the MeTC must be dismissed.

123
Page 124

Case Digests by: CHAVEZ, Jordan N.

SAN MIGUEL CORP. vs. VICENTE B. SEMILLANO, et.al.


G.R. No. 164257. July 05, 2010.
Mendoza, J.

DOCTRINE:
Art. 224 – Jurisdiction of the Labor Arbiter; IN RELATION TO
COOPERATIVES.
Although respondent Semillano is a member of AMPCO, he had
joined the others in filing the complaint because it is his position that
petitioner SMC is his true employer and liable for all his claims under
the Labor Code.

FACTS:
Alilgilan Multi-Purpose Cooperative (AMPCO) hired the services
of Vicente, et.al. on different dates in December. All of them were
assigned to work in San Miguel Corp.’s Bottling Plant. Subsequently,
SMC entered into a Contract of Services with AMPCO designating the
latter as the employer of Vicente, et. al., As a result, Vicente, et. al.,
failed to claim the rights and benefits ordinarily accorded a regular
employee of SMC. On June 6, 1995, they were not allowed to enter
the premises of SMC and was told to wait for one month. Unfortunately,
they never heard a word from SMC. Consequently, Vicente, et.al. filed
a Complaint for Illegal Dismissal against AMPCO and SMC as
respondents. Complainants alleged that they were fillers of SMC
Bottling Plant assigned to perform activities necessary and desirable
in the usual business of SMC. However, SMC utilized AMPCO making
it appear that the latter was their employer. Respondent SMC raised
the defense that it is the employer of the complainants. According to
SMC, AMPCO is the employer private respondents because the latter
is an independent contractor. The Labor Arbiter ruled the private
respondents were regular employees of San Miguel Corporation. On
appeal, the NLRC reversed the LA’s decision and ruled that AMPCO,
as employer of respondent, liable to pay for respondent’s backwages.
On review, the Court of Appeals reversed the NLRC ruling and ruled
that AMPCO was a labor-only contractor.

ISSUE:
(1) Whether the case at bar is within the jurisdiction of the labor
tribunals?

(2) Whether AMPCO is a legitimate job contractor?

124
Page 125

RULING:
(1) Yes, the Court ruled that the case at bar is within the
jurisdiction of the labor tribunals.

Although respondent Semillano is a member of AMPCO, he had


joined the others in filing the complaint because it is his position that
petitioner SMC is his true employer and liable for all his claims under
the Labor Code.

(2) Yes, the Supreme Court ruled that AMPCO is a labor-only


contractor.

The existence of an independent and permissible contractor


relationship is generally established by the following criteria: whether
or not the contractor is carrying on an independent business; the
nature and extent of the work; the skill required; the term and duration
of the relationship; the right to assign the performance of a specified
piece of work; the control and supervision of the work to another; the
employer's power with respect to the hiring, firing and payment of the
contractor's workers; the control of the premises; the duty to supply the
premises, tools, appliances, materials, and labor; and the mode,
manner and terms of payment. Petitioner did not prove that AMPCO
had substantial equipment, tools, machineries, and supplies actually
and directly used by it in the performance or completion of the
segregation and piling job.

Therefore, petitioner SMC, as principal employer, is solidarily


liable with AMPCO, the labor-only contractor, for all the rightful claims
of respondents. Under this set-up, AMPCO, as the "labor-only"
contractor, is deemed an agent of the principal (SMC). The law makes
the principal responsible over the employees of the "labor-only"
contractor as if the principal itself directly hired the employees.

125
Page 126

Case Digests by: CHAVEZ, Jordan N.

DEMETRIO ELLAO vs. BATANGAS I ELECTRIC


COOPERATIVE, INC. (BATELEC I)
G.R. No. 209166. July 9, 2018.
Tijam, J.

DOCTRINE:
Art. 224 – Jurisdiction of the Labor Arbiter; IN RELATION TO
COOPERATIVES.
Complaints for illegal dismissal filed by a cooperative officer
constitute an intra-cooperative controversy, jurisdiction over which
belongs to the regional trial courts not the Labor Arbiter nor the NLRC.

FACTS:
Respondent BATELEC I is an electric cooperative organized and
existing under P.D. 269. Petitioner Ellao was appointed as General
Manager in 2006. In March 13, 2009, BATELEC I terminated Ellao as
General Manager on the grounds of gross and habitual neglect of
duties and responsibilities. Petitioner Ellao filed a Complaint for illegal
dismissal and money claims before the Labor Arbiter against
BATELEC I. BATELEC I moved to dismiss Ellao’s complaint on the
ground that it is the NEA and not the NLRC which has jurisdiction over
the complaint.

The Labor Arbiter affirmed its jurisdiction over the complaint and
ruled that Ellao was illegally dismissed as the grounds for his dismissal
were unsubstantiated. On appeal, the NLRC ruled affirmed the LA’s
ruling. On review, the Court of Appeals ruled that Ellao, as BATELEC
I’s General Manager, is a corporate officer. As such, the CA concluded
that Ellao’s dismissal is considered an intra-corporate controversy
which falls under the jurisdiction of the SEC, not the RTC’s, and not
with the NLRC.

ISSUE:
(1) Whether the Labor Arbitral has jurisdiction over Ellao’s
complaint for illegal dismissal?

126
Page 127

RULING:
(1) No, the Supreme Court ruled that the Labor Arbiter has no
jurisdiction over Ellao’s complaint for illegal dismissal.

As a rule, the illegal dismissal of an officer or other employee of a


private employer is properly cognizable by the labor arbiter pursuant to
Article 217 (a) 238 of the Labor Code, as amended. By way of
exception, where the complaint for illegal dismissal involves a
corporate officer, the controversy falls under the jurisdiction of the
SEC. Under the Securities and Regulation Code, the SEC’s jurisdiction
over all intra-corporate disputes was transferred to the Regional Trial
Court. A position must be expressly mentioned in the By-Laws in order
to be considered as a corporate office. Only officers of a corporation
were those given that character either by the Corporation Code or by
the By-Laws so much so that the rest of the corporate officers could
be considered only as employees or subordinate officials.

In this case, the position of General Manager is expressly provided


for under BATELEC I’s By-laws. Evidently, the functions of the office
of the General Manager are specifically laid down under BATELEC I’s
By-laws itself. It is therefore beyond the cavil that Ellao’s position as
General Manager is a cooperative office. Accordingly, his complaint for
illegal dismissal partakes of the nature of an intra-cooperative
controversy. It involves a dispute between a cooperative officer on one
hand, and the Board of Directors, on the other.

Therefore, the Labor Arbiter and the NLRC were without


jurisdiction to take cognizance of Ellao’s Complaint.

127
Page 128

Case Digests by: CHAVEZ, Jordan N.

COMSCENTRE PHILS., INC. vs. CAMILLE B. ROCIO


G.R. No. 209166. July 9, 2018.
Tijam, J.

DOCTRINE:
Art. 224 – Jurisdiction of the Labor Arbiter; REFUND OF
EMPLOYMENT BOND.
The jurisdiction of labor tribunals is comprehensive enough to
include claims for all forms of damages arising from the employer-
employee relations. Labor tribunals have jurisdiction to award not only
the reliefs provide by labor laws, but also damages governed by the
Civil Code.

FACTS:
Petitioners Comscentre Phils. Inc. hired respondent Camille B.
Rocio as a Network Engineer. On August 5, 2011, respondent
informed petitioners of her intention to resign effective Sept. 9, 2011.
Prior to the effectivity of her resignation, she was informed that she had
to pay an employment bond of PhP 80,000.00 for resigning within 24
months from the time she got employed as provided in her employment
contract. Petitioner issued a Letter of Suspension to respondent stating
she was preventively suspended without pay from August 25, 2011 to
September 9, 2011 for violation of several company policies.
Respondent sued petitioner for illegal suspension, among others. The
Labor Arbiter ruled that respondent’s preventive suspension was
unjustified. On appeal, the NLRC affirmed the ruling with modification.
The NLRC ordered the deduction of the PhP 80,000.00 employment
bond claimed by petitioners from respondent’s total monetary award.
On certiorari, the Court of Appeals nullified the NLRC’s directive to
deduct the PhP 80,000.00 employment bond from the total monetary
award due to respondent. It ruled that petitioners’ claim for payment of
employment bond is within the exclusive jurisdiction of regular courts.

ISSUE:
(1) Whether the Court of Appeals err when it ruled that petitioner’s
claim for payment of employment bond fell within the jurisdiction of
regular courts.

RULING:
(1) Yes, the Supreme Court ruled that the Court of Appeals
erred when it ruled that petitioner’s claim for payment of
employment bond fell within the jurisdiction of regular courts.

128
Page 129

Under Art. 224, Labor Code, the Labor Arbiter shall have original
and exclusive jurisdiction for claims for actual, moral, exemplary
damages arising from the employer-employee relations. The
jurisdiction of labor tribunals is comprehensive enough to include
claims for all forms of damages arising from the employer-employee
relations. Labor tribunals have jurisdiction to award not only the reliefs
provide by labor laws, but also damages governed by the Civil Code.

Petitioner’s claimed is inseparably intertwined with the parties’


employer-employee relationship. For it was respondent’s act of
prematurely severing her employment with the company which gave
rise to the latter’s cause of action for payment of employment bond. As
found by the NLRC, petitioner’s claim was an offshoot of the
resignation of respondent and the complications arising therefrom and
which eventually led to the filing of the case before the Labor Arbiter.
Hence, petitioners’ claim falls within the original and exclusive
jurisdiction of the labor tribunals.

The Court sustain the NLRC’s finding that respondent is liable for
payment of employment bond pursuant to her undertaking in the
employment contract. She herself has not disputed this liability arising
as it did from her breach of the minimum employment clause.

While petitioners are liable to respondent for her illegal suspension


and unpaid money claims, respondent, too, is liable to petitioners for
payment of the employment bond. As such, the NLRC correctly
ordered the offsetting of their respective money claims against each
other.

129
Page 130

Case Digests by: CHAVEZ, Jordan N.

LUIS G. GEMUDIANO, JR. vs.


NAESS SHIPPING PHILIPPINES, INC. et. al.
G.R. No. 223825. January 20, 2020.
Reyes, Jr. J.

DOCTRINE:
Art. 224 – Jurisdiction of the Labor Arbiter;
It is clear that claims for actual, moral, exemplary and other forms
of damages arising from employer-employee relations are under the
original and exclusive jurisdiction of labor arbiters.

FACTS:
Respondent Naess Shipping, for and in behalf of its principal
Royal Dragon, executed a Contract of Employment for Marine Crew on
Board a Domestic Vessels engaging the services of petitioner
Gemudiano Jr. as Second Officer aboard the vessel M/V Melling 11. It
was stipulated that the contract shall take effect on March 12, 2013.
Subsequently, an addendum was executed stating that the
employment relationship between them shall commence once the
boarding confirmation was issued to the petitioner. On March 8, 2013,
petitioner was informed that Royal Dragon cancelled his embarkation.
Thus, petitioner Gemudiano Jr. filed a complaint for breach of contract
against respondents before the Arbitration Branch of the NLRC.
Petitioner alleged that respondents' unilateral and unreasonable failure
to deploy him despite the perfected contract of employment constitutes
breach and gives rise to a liability to pay actual damages. The Labor
Arbiter found respondents to have breached their contractual
obligation to petitioner and ordered them to pay him PhP 180,000.00
representing his salary for the duration of the contract. On appeal, the
NLRC affirmed with modification the LA ruling. The Court of Appeals
reversed the NLRC ruling and ruled that the LA did not acquire
jurisdiction over the petitioner’s complaint because of the non-
existence of an employer-employee relationship between the parties.
It emphasized that the perfected contract of employment did not
commence since petitioner's deployment to his vessel of assignment
did not materialize.

ISSUE:
Whether the Labor Arbiter had jurisdiction over petitioner’s claim
for damages arising from breach of contract.

130
Page 131

RULING:

Yes, the Supreme Court ruled that the Labor Arbiter had
jurisdiction over petitioner’s claim for damages arising from
breach of contract.

Under Art. 224 (217) of the Labor Code, the Labor Arbiter shall
have exclusive and original jurisdiction to hear and decide claims for
actual, moral, exemplary and other forms of damages arising from the
employer-employee relationship.

While there are cases which hold that the existence of an


employer-employee relationship does not negate the civil jurisdiction
of the trial courts, in this case, the Court found that jurisdiction properly
lies with the Labor Arbiter.

The determination of propriety of petitioner's non-deployment


necessarily involves the interpretation and application of labor laws,
which are within the expertise of labor tribunals. The question of
whether respondents are justified in cancelling the deployment of
petitioner requires determination of whether a subsequent advice from
the same medical provider as to the health of petitioner could validly
supersede its initial finding during the required PEME that petitioner is
fit to work.

If the Court were to make a distinction between the perfection of a


contract of employment and the commencement of an employment
relationship on its face, and so rule that a mere perfected contract
would make the jurisdiction of the case fall under regular courts, the
Court will arrive at a dangerous conclusion where domestic seafarers'
only recourse in law in case of breach of contract is to file a complaint
for damages before the Regional Trial Court.

131
Page 132

Case Digests by: CRUZ, ASHER GRACE M.

PASAY CITY ALLIANCE CHURCH v. BENITO


G.R. No. 226908. November 28, 2019.
Reyes, Jr., J.

DOCTRINE:
JURISDICTION
In Pastor Austria v. NLRC, as reiterated in United Church of
Christ in the Philippines, Inc. v. Bradford United Church of Christ,
Inc., we already defined which matters are outside the jurisdiction of
civil courts and tribunals. Thus:

An ecclesiastical affair is "one that concerns doctrine, creed, or


form [of] worship of the church, or the adoption and enforcement
within a religious association of needful laws and regulations for the
government of the membership, and the power of excluding from
such associations those deemed unworthy of membership. Based on
this definition, an ecclesiastical affair involves the relationship
between the church and its members and relate to matters of faith,
religious doctrines, worship and governance of the congregation. To
be concrete, examples of this so-called ecclesiastical affairs to which
the State cannot meddle are proceedings for excommunication,
ordinations of religious ministers, administration of sacraments and
other activities x x x attached [with] religious significance.

FACTS:
Petitioner Pasay City Alliance Church (PCAC) is one of the
local churches of its co-petitioner, Christian and Missionary Alliance
Churches of the Philippines (CAMACOP), a religious society
registered with the Securities and Exchange Commission (SEC).

Respondent Benito, on the other hand, is a licensed Christian


Minister of CAMACOP. Benito first served as PCAC’s Head of
Fellowship and Discipleship. In 2005, Benito was appointed Head of
PCAC’s Membership and Evangelism Ministry, which was renamed
Pastoral Care and Membership and Evangelism Ministry, which was
renamed Pastoral Care and Membership in 2009. Benito served
without written contract. Pastoral Care and Membership is under the
supervision of the Church Ministry Team (CMT) and co-petitioner
Reverend William Cargo (Rev. Cargo).

132
Page 133

The present controversy stemmed from CAMACOP and


PCAC’s policy requiring pastors or ministers without written contracts
to tender a courtesy resignation every year.

Pastors who are not reappointed to their previous posts may


reapply, in which case, they are assigned to another position, local
church or specialized ministry. Notwithstanding the adoption and
ratification of this policy by CAMACOP’s member congregations in
2005, the practice of requiring courtesy resignations in PCAC began
implementation only in 2009, after Rev. Cargo assumed as Senior
Pastor or during his leadership.

In compliance, Benito tendered her courtesy resignation as


Head of Pastoral Care and Membership on January 30, 2011. The
CMT reappointed Benito to the same position for another year.

When the CMT convened the following year, or on February 12,


2012, it then decided not to reappoint Benito and recommended that
she reapply to a more suitable position.

The decision not to extend Benito’s term was not immediately


pursued by the CMT, and Benito held the post for another year.

On February 17, 2013, Benito complied anew and submitted a


courtesy resignation, without prejudice to the CMT's evaluation. Upon
instructions, Benito also submitted her credentials to help the CMT in
determining whether or not her term should be extended. Meanwhile,
on May 29, 2013, Benito was instructed to endorse her workload and
turn over the prayer ministry to another pastor.

Finally, Benito was informed, through a letter dated December


15, 2013, of the CMT's decision to uphold its February 12, 2012
recommendation to the District Ministry Supervisor regarding the non-
extension of her engagement as PCAC's Head of Pastoral Care and
Membership.

Aggrieved, Benito filed a complaint for illegal dismissal,


damages and attorney’s fees before the Labor Arbiter, anchored on
the claim that she had already attained regular status by operation of
law and entitled to security of tenure in view of her long years of
service with PCAC.

ISSUE:
Whether the matter at hand is an ecclesiastical matter over which
our labor tribunals are deprived of jurisdiction.

133
Page 134

RULING:
Yes. There is no question among the parties in this case that our
constitutionally protected policy is non-interference by the State in
matters that are purely ecclesiastical. It is also settled that religious
associations can be employers for whom religious ministers often
perform dual roles. They not only minister to the spiritual needs of
their members in most instances, but also take on administrative
functions in their organizations.

In Pastor Austria v. NLRC, as reiterated in United Church of


Christ in the Philippines, Inc. v. Bradford United Church of Christ,
Inc., we already defined which matters are outside the jurisdiction of
civil courts and tribunals. Thus:

An ecclesiastical affair is "one that concerns doctrine,


creed, or form [of] worship of the church, or the adoption and
enforcement within a religious association of needful laws and
regulations for the government of the membership, and the
power of excluding from such associations those deemed
unworthy of membership. Based on this definition, an
ecclesiastical affair involves the relationship between the
church and its members and relate to matters of faith, religious
doctrines, worship and governance of the congregation. To be
concrete, examples of this so-called ecclesiastical affairs to
which the State cannot meddle are proceedings for
excommunication, ordinations of religious ministers,
administration of sacraments and other activities x x x attached
[with] religious significance.

At the center of the present controversy is the enforcement of a


religious denomination's internal rules in the governance of its
member churches. Petitioners' contention that there was no dismissal
to speak of and the matter concerns their right to transfer or reassign
one of their licensed ministers is well taken. We find the claimed right
to be infused with religious color because it bears down on the
relationship of a church and its members in faith-based matters. If a
church or religious association has the sole prerogative to exclude
members perceived to be unworthy in light of its doctrinal standards,
all the more does it have sole prerogative in determining who are best
fit to minister to its members in activities attached with religious
significance.

134
Page 135

Case Digests by: CRUZ, ASHER GRACE M.

TUMAODOS v. SAN MIGUEL YAMAMURA PACKAGING


CORPORATION
G.R. No. 241865. February 19, 2020.
Inting, J.

DOCTRINE:
JURISDICTION: REASONABLE CAUSAL CONNECTION RULE
If there is a reasonable causal connection between the claim
asserted and the employer-employee relations, then the case is
within the jurisdiction of the labor courts, and in the absence thereof,
it is the regular courts that have jurisdiction.

FACTS:
As an employee of respondent, petitioner became a member of
SMC Employees & Its Subsidiaries Multi-Purpose Cooperative
(Cooperative).

Due to its plant reorganization, respondent implemented an


Involuntary Separation Program effective November 15, 2014.
Petitioner was one of the employees who availed himself of the
program. His separation package was computed at P3,080,244.66,
but respondent withheld the amount of P1,400,000.00 on behalf of
the Cooperative, to which petitioner allegedly had an outstanding
indebtedness.

On October 13, 2014, respondent paid out petitioner’s


separation benefits, less the amount withheld. Petitioner signed a
Receipt and Release in favor of respondent, but he made a notation
that the amount of P1,400,000.00 was still subject to verification.

On November 28, 2014, respondent received a letter from


petitioner wherein he claimed that he no longer had any outstanding
obligation to the Cooperative. Thus, petitioner demanded respondent
to release to him the withheld amount. On February 13, 2015,
respondent also received a letter from the Cooperative, disputing
petitioner’s assertions and also claiming entitlement to the withheld
amount.

Due to petitioner’s and Cooperative’s conflicting claims,


respondent, on March 17, 2015 filed a Complaint for interpleader with
Consignation before Branch 55, Regional Trial Court (RTC),
Mandaue City.

135
Page 136

Meanwhile, on April 22, 2015, petitioner filed a complaint before


the NLRC Regional Arbitration Branch No. VII for non-payment of
separation pay and damages. The case was docketed as NLRC RAB
VII 04-1000-15. Considering that settlements failed, the Labor Arbiter
(LA) directed the parties to simultaneously file their respective
position papers.

Petitioner claimed that respondent made deductions totaling


P529,464.00, which was more than double the sum that he owed to
the Cooperative. He averred that he had not only paid his loan in full
but had made excess payment in the amount of P279,464.00, which
respondent must return.

ISSUE:
Whether the regular courts have jurisdiction over petitioner’s
claims

RULING:
Yes, it is the regular courts that have jurisdiction over petitioner’s
claims.

Not all controversies or money claims by an employee against


the employer or vice versa fall within the exclusive jurisdiction of the
LA. With regard to money claims and damages, Article 224 (formerly
Article 217) of the Labor Code, as amended, bestows upon the LA
original and exclusive jurisdiction over cases filed by workers
involving wages, among others, if accompanied by a claim for
reinstatement; all claims, except those for Employees Compensation,
Social Security, Medicare and maternity benefits, arising from
employer-employee relations involving an amount exceeding
P5,000.00 regardless of whether accompanied with a claim for
reinstatement; and claims for actual, moral, exemplary and other
forms of damages arising from employer-employee relations.

As can be gleaned above, the jurisdiction of the LA over money


claims and damages is confined to those cases which are either
accompanied by a claim for reinstatement or arising from employer-
employee relations. Here, the Court finds that petitioner's claims do
not fall under any of these cases.

To the Court, petitioner's claims have no "reasonable causal


connection" with his employment relationship with respondent. It
bears to point out that the case that petitioner filed was neither a
complaint for illegal dismissal nor a claim for reinstatement. His
complaint was for alleged nonpayment of separation benefits and
damages. It is notable, however, that respondent never denied

136
Page 137

petitioner's entitlement to his separation pay. In fact, on October 13,


2014, respondent paid out petitioner's separation package, except
that it withheld the amount of P1,400,000.00, which, purportedly, was
his outstanding indebtedness to the Cooperative. Petitioner, in turn,
signed a Receipt and Release in favor of respondent but made a
notation that the amount of P1,400,000.00 was still subject to
verification. Thus, by signing the Receipt and Release, petitioner had
in fact acknowledged that he had been paid all amounts due him
comprising his separation benefits, except that he questioned the
withholding of the P1,400,000.00 as he claimed that he no longer had
existing loan obligations to the Cooperative. It appears, thus, that the
principal relief sought by petitioner in his complaint was not the
payment of his separation package but the release to him of the
withheld amount of P1,400,000.00, to which both he and the
Cooperative claimed entitlement. In addition, he also sought the
return of the alleged excess deductions made for his 2007 loan in the
amount of P279,464.00.

The determination of petitioner's case is beyond the competence


of the labor tribunals for the following reasons: 1) petitioner's claims
have no reasonable causal connection with his employment
relationship with respondent; 2) the Cooperative is not a party to the
labor complaint and would therefore be deprived of the opportunity to
plead its claims; and 3) the Interpleader with Consignation case
before the RTC, which was filed by respondent prior to petitioner's
labor complaint, was the proper forum to ventilate the claimants'
respective claims over the disputed amount of P1,400,000.00.

137
Page 138

Case Digests by: CRUZ, ASHER GRACE M.

ZONIO v. 88 ACES MARITIME SERVICES, INC.


G.R. No. 239052. October 16, 2019
Inting, J.

DOCTRINE:
PRESCRIPTION OF ACTION
SENA being a pre-requisite to the filing of a Complaint before
the Labor Arbiter, the date when Apolinario should be deemed to
have instituted his claim was when he instituted his Request for
SENA on March 25, 2015. Considering that the expiration of
Apolinario's cause of action was on April 11, 2015, his claim was filed
well within the 3-year prescriptive period.

FACTS:
On February 4, 2010, Apolinario was hired as an "ordinary
seaman" by 88 Aces to board the vessel MV Algosaibi 42. His
contract was for a duration of six months with a basic monthly salary
of US$506.15.

As an ordinary seaman, Apolinario's job on board the vessel


included the following: 1) give assistance to the able seaman; 2)
assist in the handling and operation of all deck gear such as topping,
cradling and housing of booms; 3) aid the carpenter in the repair work
when requested; and 4) to scale and chip paint, handle lines in the
mooring of the ship, assist in the actual tying up and letting go of the
vessel and stand as a lookout in the vessel.

After completing his six-month contract with 88 Aces in August


2010, Apolinario however was not repatriated as he directly entered
into a new contract with 88 Aces' foreign principal, Khalifa Algosaibi.
His new contract with Khalifa Algosaibi lasted until April 2012. In April
2012, Apolinario was repatriated in Manila. On May 8, 2015, he filed
a Complaint before the Labor Arbiter against 88 Aces, Jocson and
Khalifa Algosaibi (collectively referred to as respondents) for the
payment of disability benefits, attorney's fees, medical fees, sickness
allowance and moral, exemplary and compensatory damages.

In his Position Paper, Apolinario alleged that while on board MV


Algosaibi 42 in December 2010, he suddenly experienced dizziness.
As his condition did not improve, he was sent to as Salama Hospital
in Al-Khobar, Saudi Arabia where he was found to have high glucose
and cholesterol. Apolinario posited that he was given medicine by the

138
Page 139

doctor and was advised to observe proper diet and avoid stress. After
taking the doctor's advice, his medical condition improved and he
was able to perform his work well.

However, after two years, particularly in January 2012,


Apolinario alleged that his dizziness recurred, accompanied by the
blurring of his vision. On April 2, 2012, he stated that he returned to
As Salama Hospital where he was diagnosed to have diabetes
mellitus and dislipedemia.

After his repatriation to the Philippines on April 11, 2012,


Apolinario posited that he immediately reported to the office of 88
Aces to get his unpaid wages and for him to be referred to the
company physician. However, since his repatriation was due to the
completion of his six-month Philippine Overseas Employment
Administration (POEA)-approved employment contract, he was
allegedly told by President Janet Jocson that 88 Aces could not
shoulder his medical expenses. Apolinario did not insist anymore and
just continued taking the medicine given by the doctor in Saudi
Arabia.

Subsequently, Apolinario felt well and thought that his illness


was already cured. However, it recurred on August 2, 2013.
Apolinario consulted Dr. Joseph Glenn Dimatatac, an internal
medicine physician, and was informed that his illness was indeed
diabetes mellitus.

On March 17, 2015, Apolinario consulted Dr. Rufo Luna, the


Municipal Health Officer of the Municipality of San Jose, who
declared him to be physically unfit to continue work due to his
hyperglycemia. Consequently, Apolinario demanded from
respondents the payment of his disability benefits, but to no avail.

Apolinario argued that his illness is presumed as work-related.


According to him, his stress was a factor in the development of his
diabetes mellitus since he was exposed to frequent overtime, lack of
sleep, and emotional/psychological stress for being away from his
family. Moreover, Apolinario contended that his disability is
permanent and total because he was already incapacitated to resume
his sea duties for more than 240 days. Apolinario maintained that his
cause of action to file a claim against respondents did not prescribe
yet since his action was instituted within three years from his
disembarkation from the vessel.

To counter Apolinario's claim, respondents, on the other hand,


argued that Apolinario finished his six-month POEA-approved

139
Page 140

employment contract in August 2010 without any medical issue


whatsoever. They contended that since the filing of his Complaint
was made five years after the completion of his contract in August
2010, his cause of action had already prescribed for not having been
filed within the three-year prescriptive period. Moreover, respondents
claimed that contrary to Apolinario's allegation, he actually failed to
comply with the three-day post-employment medical examination
requirement. As such, he cannot be entitled to his money claims,
moral, compensatory and exemplary damages.

ISSUE:
Whether the cause of action had already prescribed

RULING:
No.

Section 30 of the 2000 POEA-SEC provides for the prescriptive


period for filing claims arising from the contract:

Sec. 30. PRESCRIPTION OF ACTION. —

All claims arising from this Contract shall be made within


three (3) years from the date the cause of action arises,
otherwise the same shall be barred.

It is well-settled that a seafarer's cause of action arises upon his


disembarkation from the vessel. As Apolinario's disembarkation from
Algosaibi 42 was on April 11, 2012, he had three years from the date,
or until April 11, 2015, to make a claim for disability benefits. Records
show that Apolinario had requested for a SENA before the NLRC as
early as March 25, 2015. To elucidate, SENA is an administrative
approach to provide an accessible, speedy, and inexpensive
settlement of complaints arising from employer-employee relationship
to prevent cases from ripening into full blown disputes. All labor and
employment disputes undergo this 30-day mandatory conciliation-
mediation process.

Notwithstanding, that Apolinario filed his Complaint before the


Labor Arbiter only on May 8, 2015 is of no moment. SENA being a
pre-requisite to the filing of a Complaint before the Labor Arbiter, the
date when Apolinario should be deemed to have instituted his claim
was when he instituted his Request for SENA on March 25, 2015.
Considering that the expiration of Apolinario's cause of action was on
April 11, 2015, his claim was filed well within the 3-year prescriptive
period.

140
Page 141

Case Digests by: CRUZ, ASHER GRACE M.

PHILIPPINE AIRLINES, INC. v. NATIONAL LABOR


RELATIONS COMMISSION
G.R. No. 120567. March 20, 1998
Martinez, J.

DOCTRINE:
POWER TO ISSUE AN INJUNCTIVE WRIT ORIGINATES FROM A
LABOR DISPUTE
In labor cases, under Article 218 of the Labor Code and Sec. 1,
Rule XI of the New Rules of Procedure of the NLRC, the power of the
NLRC to issue an injunctive writ originates from “any labor dispute”
upon application by a party thereof, which application if not granted
“may cause grave or irreparable damage to any party or render
ineffectual any decision in favor of such party.”

FACTS:
Private respondents are flight stewards of the petitioner. Both
were dismissed from the service for their alleged involvement in the
currency smuggling in Hong Kong. Aggrieved by said dismissal,
private respondents filed with the NLRC a petition for injunction. The
NLRC issued a temporary mandatory injunction enjoining petitioner to
cease and desist from enforcing its Memorandum of Dismissal.
Hence, this petition for certiorari.

ISSUE:
Can the NLRC, even without a complaint for illegal dismissal filed
before the labor arbiter, entertain an action for injunction and issue
such writ enjoining petitioner Philippine Airlines, Inc. from enforcing
its Orders of dismissal against private respondents, and ordering
petitioner to reinstate the private respondents to their previous
positions?||

RULING:
No. The power of the NLRC to issue an injunctive writ originates
from "any labor dispute" upon application by a party thereof, which
application if not granted "may cause grave or irreparable damage to
any party or render ineffectual any decision in favor of such party."

The term "labor dispute" is defined as "any controversy or matter


concerning terms and conditions of employment or the association or
representation of persons in negotiating, fixing, maintaining, changing,
or arranging the terms and conditions of employment regardless of

141
Page 142

whether or not the disputants stand in the proximate relation of


employers and employees."
The term "controversy" is likewise defined as "a litigated
question; adversary proceeding in a court of law; a civil action or suit,
either at law or in equity; a justiciable dispute."
A "justiciable controversy" is "one involving an active
antagonistic assertion of a legal right on one side and a denial thereof
on the other concerning a real, and not a mere theoretical question or
issue."
Taking into account the foregoing definitions, it is an essential
requirement that there must first be a labor dispute between the
contending parties before the labor arbiter. In the present case, there
is no labor dispute between the petitioner and private respondents as
there has yet been no complaint for illegal dismissal filed with the labor
arbiter by the private respondents against the petitioner.
||

142
Page 143

Case Digests by: CRUZ, ASHER GRACE M.

LAND BANK OF THE PHILIPPINES v. LISTANA


G.R. No. 152611. August 5, 2003
Ynares-Santiago, J.

DOCTRINE:
POWER TO CITE PERSONS FOR INDIRECT CONTEMPT
Quasi-judicial agencies that have the power to cite persons for
indirect contempt pursuant to Rule 71 of the Rules of Court can only
do so by initiating them in the proper Regional Trial Court. It is not
within their jurisdiction and competence to decide the indirect
contempt cases. These matters are still within the province of the
Regional Trial Courts.

FACTS:
Respondent herein voluntarily offered to sell his land to the
government through the Department of Agrarian Reform. After
summary administrative proceedings, the Department of Agrarian
Reform Adjudication Board (DARAB) set the just compensation of the
land and ordered the petitioner bank to pay the said amount to the
herein respondent. Respondent filed a Motion for Contempt with the
PARAD when the petitioner failed to comply with the writ of
execution. PARAD granted the motion for contempt, and later on
directed the issuance of an arrest order against petitioner's manager.
Petitioner filed a petition for injunction of the arrest order, which was
approved by the trial court. After the respondent's motion for
reconsideration was denied by the court, it filed a special civil action
for certiorari with the Court of Appeals (CA). The appellate court
nullified the order of the trial court.

ISSUE:
Whether the PARAD acquired competent jurisdiction over the
contempt proceedings inasmuch as it was initiated by mere motion
for contempt and not by verified petition, in violation of Section 2,
Rule XI of the New DARAB Rules of Procedure and of Rule 71 of the
Revised Rules of Court

RULING:
No. The requirement of a verified petition is mandatory. Justice
Florenz D. Regalado, Vice-Chairman of the Revision of the Rules of
Court Committee that drafted the 1997 Rules of Civil Procedure
explains this requirement: 1. This new provision clarifies with a
regulatory norm the proper procedure for commencing contempt
proceedings. While such proceeding has been classified as a special

143
Page 144

civil action under the former Rules, the heterogeneous practice,


tolerated by the courts, has been for any party to file a mere motion
without paying any docket or lawful fees therefor and without
complying with the requirements for initiatory pleadings, which is now
required in the second paragraph of this amended section. xxx xxx xxx
Henceforth, except for indirect contempt proceedings initiated motu
proprio by order of or a formal charge by the offended court, all charges
shall be commenced by a verified petition with full compliance with the
requirements therefor and shall be disposed of in accordance with the
second paragraph of this section.
||

Therefore, there are only two ways a person can be charged


with indirect contempt, namely, (1) through a verified petition; and (2)
by order or formal charge initiated by the court motu proprio. In the
case at bar, neither of these modes was adopted in charging Mr.
Lorayes with indirect contempt. More specifically, Rule 71, Section 12
of the 1997 Rules of Civil Procedure, referring to indirect contempt
against quasi-judicial entities, provides: Sec. 12. Contempt against
quasi-judicial entities. — Unless otherwise provided by law, this Rule
shall apply to contempt committed against persons, entities, bodies or
agencies exercising quasi-judicial functions, or shall have suppletory
effect to such rules as they may have adopted pursuant to authority
granted to them by law to punish for contempt. The Regional Trial
Court of the place wherein the contempt has been committed shall
have jurisdiction over such charges as may be filed therefore.

The foregoing amended provision puts to rest once and for all
the questions regarding the applicability of these rules to quasi-
judicial bodies, to wit: 1. This new section was necessitated by the
holdings that the former Rule 71 applied only to superior and inferior
courts and did not comprehend contempt committed against
administrative or quasi judicial officials or bodies, unless said
contempt is clearly considered and expressly defined as contempt of
court, as is done in the second paragraph of Sec. 580, Revised
Administrative Code. The provision referred to contemplates the
situation where a person, without lawful excuse, fails to appear, make
oath, give testimony or produce documents when required to do so
by the official or body exercising such powers. For such violation,
said person shall be subject to discipline, as in the case of contempt
of court, upon application of the official or body with the Regional Trial
Court for the corresponding sanctions.

Evidently, quasi-judicial agencies that have the power to cite


persons for indirect contempt pursuant to Rule 71 of the Rules of
Court can only do so by initiating them in the proper Regional Trial
Court. It is not within their jurisdiction and competence to decide the

144
Page 145

indirect contempt cases. These matters are still within the province of
the Regional Trial Courts. In the present case, the indirect contempt
charge was filed, not with the Regional Trial Court, but with the
PARAD, and it was the PARAD that cited Mr. Lorayes with indirect
contempt. Hence, the contempt proceedings initiated through an
unverified "Motion for Contempt" filed by the respondent with the
PARAD were invalid for the following reasons: First, the Rules of
Court clearly require the filing of a verified petition with the Regional
Trial Court, which was not complied with in this case. The charge was
not initiated by the PARAD motu proprio; rather, it was by a motion
filed by respondent. Second, neither the PARAD nor the DARAB
have jurisdiction to decide the contempt charge filed by the
respondent. The issuance of a warrant of arrest was beyond the
power of the PARAD and the DARAB. Consequently, all the
proceedings that stemmed from respondent's "Motion for Contempt,"
specifically the Orders of the PARAD dated August 20, 2000 and
January 3, 2001 for the arrest of Alex A. Lorayes, are null and void.

145
Page 146

Case Digests by: CRUZ, ASHER GRACE M.

ROBOSA v. NATIONAL LABOR RELATIONS


COMMISSION
G.R. No. 176085. February 8, 2012
Brion, J.

DOCTRINE:
CONTEMPT POWERS
Under Article 218 of the Labor Code,the NLRC (and the labor
arbiters) may hold any offending party in contempt, directly or
indirectly, and impose appropriate penalties in accordance with law.
The penalty for direct contempt consists of either imprisonment or
fine, the degree or amount depends on whether the contempt is
against the Commission or the labor arbiter. The Labor Code,
however, requires the labor arbiter or the Commission to deal with
indirect contempt in the manner prescribed under Rule 71 of the
Rules of Court.|||

FACTS:
Federico S. Robosa, Rolando E. Pandy, Noel D. Roxas,
Alexander Angeles, Veronica Gutierrez, Fernando Embat and
Nanette H. Pinto (petitioners) were rank-and-file employees of
respondent Chemo-Technische Manufacturing, Inc. (CTMI), the
manufacturer and distributor of Wella products. They were officers
and members of the CTMI Employees Union-DFA (union).
Respondent Procter and Gamble Philippines, Inc. (P & GPI) acquired
all the interests, franchises and goodwill of CTMI during the pendency
of the dispute.

On July 15, 1991, CTMI, through its President and General


Manager Franklin R. de Luzuriaga, issued a memorandum
announcing that territories were demobilized. On the same day, CTMI
issued another memorandum informing the company’s sales
representatives and sales drivers of the new system in the Salon
Business Groups selling operations. The union asked for the
withdrawal and deferment of CTMIs directives, branding them as
union busting acts constituting unfair labor practice. CTMI ignored the
request. Instead, it issued on July 23, 1991 a notice of termination of
employment to the sales drivers, due to the abolition of the sales
driver positions. On August 1, 1991, the union and its affected
members filed a complaint for illegal dismissal and unfair labor
practice, with a claim for damages, against CTMI, De Luzuriaga and
other CTMI officers.

146
Page 147

The Compulsory Arbitration Proceedings: The labor arbiter


handling the case denied the unions motion for a stay order on the
ground that the issues raised by the petitioners can best be ventilated
during the trial on the merits of the case.

NLRC: It directed CTMI, De Luzuriaga and other company


executives to cease and desist from dismissing any member of the
union and from implementing the subject memorandums and to
return to status quo.

Allegedly, the respondents did not comply with the NLRCs


August 23, 1991 resolution. They instead moved to dissolve the TRO
and opposed the unions petition for preliminary injunction. The union
opposed the motion and urgently moved to cite the responsible CTMI
officers in contempt of court. Meanwhile, the NLRC heard the
contempt charge. On October 31, 2000, it issued a resolution
dismissing the charge. It ordered the labor arbiter to proceed hearing
the main case on the merits.

The Court of Appeals held that the NLRC's dismissal of the


contempt charges against the respondents amounts to an acquittal in
a criminal case and is not subject to appeal.|||

ISSUES:
Whether the NLRC has contempt powers

RULING:
Yes. On the first issue, we stress that under Article 218 of the
Labor Code, the NLRC (and the labor arbiters) may hold any
offending party in contempt, directly or indirectly, and impose
appropriate penalties in accordance with law. The penalty for direct
contempt consists of either imprisonment or fine, the degree or
amount depends on whether the contempt is against the Commission
or the labor arbiter. The Labor Code, however, requires the labor
arbiter or the Commission to deal with indirect contempt in the
manner prescribed under Rule 71 of the Rules of Court.

Rule 71 of the Rules of Court does not require the labor arbiter
or the NLRC to initiate indirect contempt proceedings before the trial
court. This mode is to be observed only when there is no law granting
them contempt powers. As is clear under Article 218 (d) of the Labor
Code, the labor arbiter or the Commission is empowered or has
jurisdiction to hold the offending party or parties in direct or indirect
contempt. The petitioners, therefore, have not improperly brought the
indirect contempt charges against the respondents before the NLRC.

147
Page 148

Case Digests by: CRUZ, ASHER GRACE M.

JOLO’S KIDDIE CARTS v. CABALLA


G.R. No. 230682. November 29, 2017
Perlas-Bernabe, J.

DOCTRINE:
MOTION FOR RECONSIDERATION: CONDITION SINE QUA NON
TO THE FILING OF A PETITION FOR CERTIORARI
As a rule, the filing of a motion for reconsideration is a condition
sine qua non to the filing of a petition for certiorari. The rationale for
this requirement is that "the law intends to afford the tribunal, board
or office an opportunity to rectify the errors and mistakes it may have
lapsed into before resort to the courts of justice can be had." Notably,
however, there are several recognized exceptions to the rule, one of
which is when the order is a patent nullity.

FACTS:
The instant case stemmed from a complaint for illegal
dismissal, underpayment of salaries/wages and 13th month pay,
non-payment of overtime pay, holiday pay, and separation pay,
damages, and attorney's fees filed by Evelyn A. Caballa (Caballa),
Anthony M. Bautista (Bautista; collectively, respondents), and one
Jocelyn S. Colisao (Colisao) against petitioners before the National
Labor Relations Commission (NLRC).

The Labor Arbiter's (LA) Ruling: In a Decision dated


November 27, 2015, the LA dismissed the case insofar as Colisao
is concerned for failure to prosecute. However, the LA ruled in
favor of respondents, and accordingly, ordered petitioners to
solidarily pay them the following, plus attorney's fees equivalent to
ten percent (10%) of the total monetary awards.

The LA found that respondents' adequate substantiation of


their claim that they were no longer given any work assignment
and were not allowed to go anywhere near their respective
workstations, coupled with petitioners' failure to prove
abandonment, justifies the finding that respondents were indeed
dismissed without just cause nor due process.

Aggrieved, petitioners appealed to the NLRC.

148
Page 149

The NLRC Ruling: In a Decision dated April 28, 2016, the


NLRC modified the LA ruling, finding no illegal dismissal nor
abandonment of work. Accordingly, the NLRC ordered petitioners to
reinstate respondents to their former or substantially equivalent
positions without loss of seniority rights and privileges; deleted the
awards for payment of backwages, separation pay, and moral and
exemplary damages; and affirmed the rest of the awards. For this
purpose, the NLRC attached a Computation of Monetary Award
detailing the monetary awards due to respondents, as follows: (a)
for Caballa, P15,623.00 as holiday pay, P109,870.80 as wage
differential, and P75,156.12 as 13th month pay; (b) for Bautista,
P15,623.00 as holiday pay, P112,294.00 as wage differential, and
P74,480.12 as 13th month pay; and (c) attorney's fees amounting to
ten percent (10%) of the total monetary value awarded.

Anent the procedural matters raised by petitioners, the NLRC


ruled that: (a) petitioners waived the issue of improper venue when
they failed to raise the same before the filing of position papers; and
(b) respondents substantially complied with the requirement of
verifying their position papers, and thus, the same is not fatal to their
complaint. As to the merits, while the NLRC agreed with the LA's
finding that there was no abandonment on the part of respondents,
the latter were unable to adduce any proof that petitioners indeed
committed any overt or positive act operative of their dismissal. In
view of the finding that there was neither dismissal on the part of
petitioners nor abandonment on the part of respondents, the NLRC
ordered the latter's reinstatement but without backwages. Finally,
the NLRC held that respondents should be entitled to their holiday
pay as it is a statutory benefit which payment petitioners failed to
prove.
Dissatisfied, petitioners directly filed a petition for certiorari
before the CA, without moving for reconsideration before the NLRC.

The CA Ruling: In a Resolution dated July 28, 2016, the CA


denied the petition due to petitioners' failure to file a motion for
reconsideration before the NLRC prior to the filing of a petition for
certiorari before the CA. It held that the prior filing of such motion
before the lower tribunal is an indispensable requisite in elevating
the case to the CA via certiorari, and that petitioners' failure to do
so resulted in the NLRC ruling attaining finality.

Petitioners moved for reconsideration, but the same was


denied in a Resolution dated February 22, 2017; hence, this
petition.

ISSUES:

149
Page 150

Whether the CA erred in dismissing the petition for certiorari


before it due to petitioners’ non-filing of a prior motion for
reconsideration before the NLRC

RULING:
Yes. Upon petitioners' appeal to the NLRC, the LA ruling was
modified, deleting the awards for separation pay, backwages, moral
damages, and exemplary damages, while affirming the awards for
wage differential and 13th month pay. In the Computation of
Monetary Award attached to the NLRC ruling — which according to
the NLRC itself, shall form part of its decision— it was indicated that
Caballa's awards for wage differential and 13th month pay are in the
amounts of P109,870.80 and P75,156.12, respectively; while the
awards in Bautista's favor were pegged at P112,294.00 and
P74,480.12, respectively. However, a simple counter checking of the
NLRC's computation with the LA ruling readily reveals that: (a) the
amounts of P109,870.80 and P112,294.00 clearly pertain to the
awards of backwages, which were already deleted in the NLRC
ruling; (b) the amounts of P75,156.12 and P74,480.12 pertain to the
awards of wage differential; and (c) the amount of P10,608.00 which
pertain to the awards of 13th month pay for both respondents, were
no longer reflected in the NLRC computation. While this is obviously
just an oversight on the part of the NLRC, it is not without any
implications as such oversight resulted in an unwarranted increase in
the monetary awards due to respondents. Clearly, such an increase
is a patent nullity as it is bereft of any factual and/or legal basis.
Verily, the CA erred in dismissing the petition for certiorari filed
before it based on the aforesaid technical ground, as petitioners
were justified in pursuing a direct recourse to the CA even without
first moving for reconsideration before the NLRC. In such instance,
court procedure dictates that the case be remanded to the CA for a
resolution on the merits. However, when there is already enough
basis on which a proper evaluation of the merits may be had, as in
this case, the Court may dispense with the time-consuming
procedure of remand in order to prevent further delays in the
disposition of the case and to better serve the ends of justice. In
view of the foregoing — as well as the fact that petitioners pray for
a resolution on the merits— the Court finds it appropriate to
exhaustively resolve the instant case.

150
Page 151

Case Digests by: CRUZ, ASHER GRACE M.

FRONDOZO v. MANILA ELECTRIC COMPANY


G.R. No. 178379. August 22, 2017
Carpio, J.

DOCTRINE:
SUSPENSION OF EXECUTION OF JUDGMENT
There are instances when writs of execution may be assailed.
They are:
(1) the writ of execution varies the judgment;
(2) there has been a change in the situation of the parties
making execution inequitable or unjust;
(3) execution is sought to be enforced against property exempt
from execution;
(4) it appears that the controversy has been submitted to the
judgment of the court;
(5) the terms of the judgment are not clear enough and there
remains room for interpretation thereof; or
(6) it appears that the writ of execution has been improvidently
issued, or that it is defective in substance, or issued against the wrong
party, or that the judgment debt has been paid or otherwise satisfied,
or the writ was issued without authority.

FACTS:
On 26 July 1991, MERALCO terminated the services of Crispin
S. Frondozo (Frondozo), Danilo M. Perez (Perez), Jose A. Zafra
(Zafra), Arturo B. Vito (Vito), Cesar S. Cruz (Cruz), Nazario C. dela
Cruz (N. dela Cruz), Luisito R. Diloy (Diloy), and Danilo D. Dizon
(Dizon) for having committed unlawful acts and violence during a
strike.
On 25 July 1991, MEWA filed a second Notice of Strike (second
strike) on the ground of discrimination and union busting that resulted
to the dismissal from employment of 25 union officers and workers.
Then DOLE Secretary Ruben D. Torres issued an Order dated 8
August 1991 that certified the issues raised in the second strike to the
NLRC for consolidation with the first strike and strictly enjoined any
strike or lockout pending resolution of the labor dispute. The Order
also directed MERALCO to suspend the effects of termination of the
employees and re-admit the employees under the same terms and
conditions without loss of seniority rights.

151
Page 152

The labor dispute resulted to the filing of two complaints for


illegal dismissal.The NLRC consolidated the two illegal dismissal cases
with NLRC NCR CC No. 000021-91 (In the Matter of the Labor Dispute at
the Manila Electric Company) and NLRC NCR Case No. 00-05-03381-93
(MEWA v. MERALCO). NLRC modified its final decision ordering
MERALCO to reinstate to their former or equivalent positions Dizon and
Diloy, without loss of seniority rights and payment of backwages computed
from the time of their dismissal.
Two petitions for certiorari were filed before the Court of
Appeals. Pending the case in CA, the NLRC issued an entry of
judgment. MERALCO informed the NLRC of the payroll
reinstatement of the respondents. Meanwhile, the Court of Appeals
Special Second Division found that the strike of 6-8 June 1991 was
illegal because it
occurred despite an assumption order by the DOLE Secretary and
because of the commission of illegal acts marred with violence and
coercion. In view thereof, MERALCO stopped their payroll
reinstatement. Labor Arbiter Guerrero ruled that the Court of Appeals’
30 May 2003 Decision had not attained finality and as such,
respondents should be reinstated from the time they were removed
from the payroll until their actual/payroll reinstatement based on their
latest salary prior to their dismissal. Court of Appeals Fourteenth
Division rendered its decision that respondent MERALCO is ordered
to pay the petitioners full backwages computed from July 26, 1991,
when they were illegally dismissed, up to the date of their actual
reinstatement in the service. The respondents moved for the
issuance of an Alias Writ of Execution for the satisfaction of their
accrued wages arising from the recall of their payroll reinstatement.
On 10 June 2004, Labor Arbiter Guerrero granted the motion.

ISSUES:
Whether the suspension of execution of judgment is proper |||

RULING:
Yes. The Court of Appeals cited the 2005 Revised Rules of
Procedure of the NLRC which provides that "[u]pon issuance of the
entry of judgment, the Commission, motu proprio or upon motion by
the proper party, may cause the execution of the judgment in the
certified case." According to the Court of Appeals, the 2005 Revised
Rules of Procedure of the NLRC did not make a distinction between
decisions or resolutions decided by the Labor Arbiter and those
decided by the Commission in certified cases when an order of
reinstatement is involved. Thus, even when the employer had
perfected an appeal, the Labor Arbiter must issue a writ of execution
for actual or payroll reinstatement of the employees illegally
dismissed from the service. The Court of Appeals also cited Article

152
Page 153

223 of the Labor Code which provides that the reinstatement aspect
of the Labor Arbiter's Decision is immediately executory.
In this case, the applicable rule is Article 263 of the Labor Code
and the NLRC Manual on Execution of Judgment, as amended by
Resolution No. 02-02, series of 2002.
A judicial review of the decisions of the NLRC may be filed
before the Court of Appeals via a petition for certiorari under Rule 65
of the Rules of Court but the petition shall not stay the execution of
the assailed decision unless a restraining order is issued by the Court
of Appeals.
In this case, the NLRC issued an Entry of Judgment stating that
the 29 May 2002 NLRC Order became final and executory on 19 June
2002; a Writ of Execution was issued; and MERALCO complied with
the payroll reinstatement of petitioners. However, with the
promulgation of the 30 May 2003 Decision of the Court of Appeals'
Special Second Division, finding that the 6-8 June 1991 strike was
illegal, illegal acts marred with violence and coercion were
committed, and dismissing petitioners from the service, MERALCO
stopped the payroll reinstatement. This prompted petitioners to move
for the issuance of an Alias Writ of Execution for the satisfaction of
their accrued wages arising from the recall of their payroll
reinstatement which Labor Arbiter Guerrero granted on 10 June
2004. Later, a second Alias Writ of Execution was issued.
As both the NLRC and the Court of Appeals stated, they were
confronted with two contradictory Decisions of two different Divisions
of the Court of Appeals. The petitions questioning these two
Decisions of the Court of Appeals were both denied by this Court and
the denial attained finality. The Court of Appeals sustained the NLRC
that the 30 May 2003 Decision of the Court of Appeals' Special
Second Division is a subsequent development that justified the
suspension of the Alias Writs of Execution.
There are instances when writs of execution may be assailed.
They are:
(1) the writ of execution varies the judgment;
(2) there has been a change in the situation of the parties
making execution inequitable or unjust;
(3) execution is sought to be enforced against property exempt
from execution;
(4) it appears that the controversy has been submitted to the
judgment of the court;
(5) the terms of the judgment are not clear enough and there
remains room for interpretation thereof; or
(6) it appears that the writ of execution has been improvidently
issued, or that it is defective in substance, or issued against the wrong

153
Page 154

party, or that the judgment debt has been paid or otherwise satisfied,
or the writ was issued without authority.
The situation in this case is analogous to a change in the
situation of the parties making execution unjust or inequitable.
MERALCO's refusal to reinstate petitioners and to pay their
backwages is justified by the 30 May 2003 Decision in CA-G.R. SP
No. 72480. On the other hand, petitioners' insistence on the
execution of judgment is anchored on the 27 January 2004 Decision
of the Court of Appeals' Fourteenth Division in CA-G.R. SP No.
72509. Given this situation, we see no reversible error on the part of
the Court of Appeals in holding that the NLRC did not commit grave
abuse of discretion in suspending the proceedings. Grave abuse of
discretion implies that the respondent court or tribunal acted in a
capricious, whimsical, arbitrary or despotic manner in the exercise of
its jurisdiction as to be equivalent to lack of jurisdiction. Thus, this
Court declared:
The term "grave abuse of discretion" has a specific
meaning. An act of a court or tribunal can only be considered
as with grave abuse of discretion when such act is done in a
"capricious or whimsical exercise of judgment as is equivalent
to lack of jurisdiction." The abuse of discretion must be so
patent and gross as to amount to an "evasion of a positive duty
or to a virtual refusal to perform a duty enjoined by law, or to
act at all in contemplation of law, as where the power is
exercised in an arbitrary and despotic manner by reason of
passion and hostility." Furthermore, the use of a petition for
certiorari is restricted only to "truly extraordinary cases wherein
the act of the lower court or quasi-judicial body is wholly void."
From the foregoing definition, it is clear that the special civil
action of certiorari under Rule 65 can only strike an act down
for having been done with grave abuse of discretion if the
petitioner could manifestly show that such act was patent and
gross. x x x.
Clearly, the NLRC did not act in a capricious, whimsical,
arbitrary, or despotic manner. It suspended the proceedings because
it cannot revise or modify the conflicting Decisions of the Court of
Appeals.
However, we need to resolve the issue on the conflicting
Decisions in order to put an end to this litigation.
The Court of Appeals stated that "the finality of the CA Decision
in SP No. 72480 on May 24, 2004, is a supervening event which
transpired after the CA Decision in SP No. 72509 (which was in favor
of petitioners) had become final and executory." 45 This is not
accurate. The Decision in CA-G.R. SP No. 72480 was promulgated
on 30 May 2003. The Decision in CA-G.R. SP No. 72509 was
promulgated on 27 January 2004. Even when the cases were

154
Page 155

elevated to this Court, G.R. No. 161159 and G.R. No. 161311 were
resolved first before G.R. No. 164998. The Court's 23 February 2004
Resolution and the 24 May 2004 Resolution, both favoring
MERALCO, became final and executory on 15 July 2004 and 2
September 2004, respectively, while theResolution of 15 June 2005
which denied MERALCO's petition for review became final and
executory on 4 October 2005, over a year after the final resolutions
in G.R. Nos. 161159 and 161311.

155
Page 156

Case Digests by: CRUZ, ASHER GRACE M.

BISIG MANGGAGAWA SA CONCRETE AGGREGATES v.


NATIONAL LABOR RELATIONS COMMISSION
G.R. No. 105090. September 16, 1993
Puno, J.

DOCTRINE:
EX PARTE TEMPORARY RESTRAINING ORDER IN A LABOR
DISPUTE
To be sure, the issuance of an ex parte temporary restraining
order in a labor dispute is not per se prohibited. Its issuance,
however, should be characterized by care and caution for the law
requires that it be clearly justified by considerations of extreme
necessity, i.e., when the commission of unlawful acts is causing
substantial and irreparable injury to company properties and the
company is, for the moment, bereft of an adequate remedy at law.

FACTS:
The labor conflict between the parties broke out in the open
when the petitioner union struck on April 6, 1992 protesting issues
ranging from unfair labor practices and union busting allegedly
committed by the private respondent. The union picketed the
premises of the private respondent at Bagumbayan and Longos in
Quezon City; Angono and Antipolo in Rizal; San Fernando,
Pampanga and San Pedro, Laguna.
The strike hurt the private respondent. On April 8, 1992, it filed
with the NLRC a petition for injunction 3 to stop the strike which it
denounced as illegal.
The petition was set for hearing on April 13, 1992 at 3 p.m. The
union, however, claimed that it was not furnished a copy of the
petition. Allegedly, the company misrepresented its address to be at
Rm. 205-6 Herald Bldg., Muralla St., Intramuros, Manila. On April 13,
1992, the NLRC heard the evidence of the company alone. The ex
parte hearing started at 2:30 p.m., where testimonial and
documentary evidence were presented.
Before the day was over, the respondent NLRC (First Division)
issued a temporary restraining order against the union.
No copy of this Order was furnished the union. The union
learned of the Order only when it was posted on April 15, 1992 at the
premises of the company. On April 21, 1992, it filed its
Opposition/Answer to the petition for Injunction.

156
Page 157

On April 24, 1992, the union also filed its own Petition for
Injunction to enjoin the company "from asking the aid of the police
and the military officer in escorting scabs to enter the struck
establishment."
The records show that the case was heard on April 24 and 30,
May 4 and 5, 1992 by respondent Labor Arbiter Enrilo Peñalosa. On
April 30, 1992, the company filed a Motion for the Immediate
Issuance of Preliminary Injunction.
The union got wind of the motion only on May 4, 1992. The next
day, May 5, 1992, it opposed the motion.
The same day, however, the respondent NLRC issued its
disputed Order granting the company's motion for preliminary
injunction.
The union then filed the instant petition for certiorari and
mandamus.

ISSUES:
Whether the NLRC can issue a preliminary injunction after the
lapse of a twenty day temporary restraining order without regard to
the specific provision of Article 218 (e) of the Labor Code

RULING:
In the case at bar, the records will show that the NLRC failed to
comply with the letter and spirit of Article 218 (e), (4) and (5) of the
Labor Code in issuing its Order of May 5, 1992. Article 218 (e) of the
Labor Code provides both the procedural and substantive
requirements which must strictly be complied with before a temporary
or permanent injunction can issue in a labor dispute.
The records reveal the continuing misuse of unfair strategies
to secure ex parte temporary restraining orders against striking
employees. Petitioner union did not receive any copy of private
respondent's petition for injunction in Case No. 000249-92 filed on
April 8, 1992. Its address as alleged by the private respondent turned
out to be "erroneous". Consequently, the petitioner was denied the
right to attend the hearing held on April 13, 1992 while the private
respondent enjoyed a field day presenting its evidence ex parte. On
the basis of uncontested evidence, the public respondent, on the
same day April 13, 1992, temporarily enjoined the petitioner from
committing certain alleged illegal acts. Again, a copy of the Order was
sent to the wrong address of the petitioner. Knowledge of the Order
came to the petitioner only when its striking members read it after it
was posted at the struck areas of the private respondent.
To be sure, the issuance of an ex parte temporary restraining
order in a labor dispute is not per se prohibited. Its issuance,

157
Page 158

however, should be characterized by care and caution for the law


requires that it be clearly justified by considerations of extreme
necessity, i.e., when the commission of unlawful acts is causing
substantial and irreparable injury to company properties and the
company is, for the moment, bereft of an adequate remedy at law.
This is as it ought to be, for imprudently issued temporary restraining
orders can break the back of employees engaged in a legal strike.
Often times, they unduly tilt the balance of a labor warfare in favor of
capital. When that happens, the deleterious effects of a wrongfully
issued, ex parte temporary restraining order on the rights of striking
employees can no longer be repaired for they defy simple
monetization. Moreover, experience shows that ex parte applications
for restraining orders are often based on fabricated facts and
concealed truths. A more becoming sense of fairness, therefore,
demands that such ex parte applications should be more minutely
examined by hearing officers, lest, our constitutional policy of
protecting labor becomes nothing but a synthetic shibboleth. The
immediate need to hear and resolve these ex parte applications does
not provide any excuse to lower our vigilance in protecting labor
against the issuance of indiscriminate injunctions. Stated otherwise,
it behooves hearing officers receiving evidence in support of ex parte
injunctions against employees in strike to take a more active stance
in seeing to it that their right to social justice is in no way violated
despite their absence. This equalizing stance was not taken in the
case at bar by the public respondents.

158
Page 159

Case Digests by: CRUZ, ASHER GRACE M.

MANILA ELECTRIC COMPANY v. GALA


G.R. No. 1911288 & 191304. February 29, 2012
Brion, J.

DOCTRINE:
ARTICLE 227
It it is the spirit and intention of labor legislation that the NLRC
and the labor arbiters shall use every reasonable means to ascertain
the facts in each case speedily and objectively, without regard to
technicalities of law or procedure, provided due process is duly
observed.

FACTS:
On March 2, 2006, Gala commenced employment with the
Meralco Electric Company (Meralco) as a probationary lineman. He
was assigned at Meralco's Valenzuela Sector. He initially served as
member of the crew of Meralco's Truck No. 1823 supervised by
Foreman Narciso Matis. After one month, he joined the crew of Truck
No. 1837 under the supervision of Foreman Raymundo Zuñiga, Sr.
On July 27, 2006, barely four months on the job, Gala was
dismissed for alleged complicity in pilferages of Meralco's electrical
supplies, particularly, for the incident which took place on May 25,
2006. On that day, Gala and other Meralco workers were instructed
to replace a worn-out electrical pole at the Pacheco Subdivision in
Valenzuela City. Gala and the other linemen were directed to join
Truck No. 1891, under the supervision of Foreman Nemecio Hipolito.
When they arrived at the worksite, Gala and the other workers
saw that Truck No. 1837, supervised by Zuñiga, was already there.
The linemen of Truck No. 1837 were already at work. Gala and the
other members of the crew of Truck No. 1891 were instructed to help
in the digging of a hole for the pole to be installed. While the Meralco
crew was at work, one Llanes, a non-Meralco employee, arrived. He
appeared to be known to the Meralco foremen as they were seen
conversing with him. Llanes boarded the trucks, without being
stopped, and took out what were later found as electrical supplies.
Aside from Gala, the foremen and the other linemen who were at the
worksite when the pilferage happened were later charged with
misconduct and dishonesty for their involvement in the incident.
Unknown to Gala and the rest of the crew, a Meralco
surveillance task force was monitoring their activities and recording
everything with a Sony video camera.

159
Page 160

Despite Gala's explanation, Meralco proceeded with the


investigation and eventually terminated his employment on July 27,
2006. Gala responded by filing an illegal dismissal complaint against
Meralco.
In a decision dated September 7, 2007, Labor Arbiter Teresita
D. Castillon-Lora dismissed the complaint for lack of merit. She held
that Gala's participation in the pilferage of Meralco's property
rendered him unqualified to become a regular employee.
Gala appealed to the National Labor Relations Commission
(NLRC). In its decision of May 2, 2008, the NLRC reversed the labor
arbiter's ruling. It found that Gala had been illegally dismissed, since
there was "no concrete showing of complicity with the alleged
misconduct/dishonesty[.]" The NLRC, however, ruled out Gala's
reinstatement, stating that his tenure lasted only up to the end of his
probationary period. It awarded him backwages and attorney's fees.
Both parties moved for partial reconsideration; Gala, on the
ground that he should have been reinstated with full backwages,
damages and interests; and Meralco, on the ground that the NLRC
erred in finding that Gala had been illegally dismissed. The NLRC
denied the motions. Relying on the same grounds, Gala and Meralco
elevated the case to the CA through a petition for certiorari under
Rule 65 of the Rules of Court.
In its decision of August 25, 2009, the CA denied Meralco's
petition for lack of merit and partially granted Gala's petition. It
concurred with the NLRC that Gala had been illegally dismissed, a
ruling that was supported by the evidence. It opined that nothing in
the records show Gala's knowledge of or complicity in the pilferage.
It found insufficient the joint affidavit of the members of Meralco's task
force testifying that Gala and two other linemen knew Llanes.
The CA modified the NLRC decision of May 2, 200811 and
ordered Gala's reinstatement with full backwages and other benefits.
The CA also denied Meralco's motion for reconsideration. Hence, the
present petition for review on certiorari.

ISSUES:
Whether there is a clear lack of basis to support the termination
of Gala’s employment

RULING:
Yes. Gala would want the petition to be dismissed outright on
procedural grounds, claiming that the "Verification and Certification,"
"Secretary's Certificate" and "Affidavit of Service" accompanying the
petition do not contain the details of the Community Tax Certificates
of the affiants, and that the lawyers who signed the petition failed to

160
Page 161

indicate their updated MCLE certificate numbers, in violation of


existing rules.
We stress at this point that it is the spirit and intention of labor
legislation that the NLRC and the labor arbiters shall use every
reasonable means to ascertain the facts in each case speedily and
objectively, without regard to technicalities of law or procedure,
provided due process is duly observed. In keeping with this policy
and in the interest of substantial justice, we deem it proper to give
due course to the petition, especially in view of the conflict between
the findings of the labor arbiter, on the one hand, and the NLRC and
the CA, on the other. As we said in S.S. Ventures International, Inc.
v. S.S. Ventures Labor Union, "the application of technical rules of
procedure in labor cases may be relaxed to serve the demands of
substantial justice.”

161
Page 162

Case Digests by: CRUZ, ASHER GRACE M.

NATIONWIDE SECURITY AND ALLIED SERVICES v.


COURT OF APPEALS
G.R. No. 155844. July 14, 2008
Quisumbing, J.

DOCTRINE:
ARTICLE 227
The rules, particularly the requirements for perfecting an appeal
within the reglementary period specified in the law, must be strictly
followed as they are considered indispensable interdictions against
needless delays and for the orderly discharge of judicial business. It
is only in highly meritorious cases that this Court will opt not to strictly
apply the rules and thus prevent a grave injustice from being done.
FACTS:
Labor Arbiter Manuel M. Manansala found petitioner
Nationwide Security and Allied Services, Inc., a security agency, not
liable for illegal dismissal in NLRC NCR 00-01-00833-96 and 00-02-
01129-96 involving eight security guards who were employees of the
petitioner. However, the Labor Arbiter directed the petitioner to pay
the aforementioned security guards P81,750.00 in separation pay,
P8,700.00 in unpaid salaries, P93,795.68 for underpayment and 10%
attorney's fees based on the total monetary award.
Dissatisfied with the decision, petitioner appealed to the NLRC
which dismissed its appeal for two reasons — first, for having been
filed beyond the reglementary period within which to perfect the
appeal and second, for filing an insufficient appeal bond.
Its motion for reconsideration having been denied, petitioner
then appealed to the Court of Appeals to have the appeal resolved
on the merits rather than on pure technicalities in the interest of due
process. The Court of Appeals dismissed the case, holding that in a
special action for certiorari, the burden is on petitioner to prove not
merely reversible error, but grave abuse of discretion amounting to
lack of or excess of jurisdiction on the part of public respondent
NLRC.
The Court of Appeals (CA) likewise denied the petitioner’s
motion for reconsideration.

ISSUES:
Whether the CA erred in dismissing the case based on
technicalities

162
Page 163

RULING:
No. After considering all the circumstances in this case and the
submission by the parties, we are in agreement that the petition lacks
merit.
In the instant case, both the NLRC and the CA found that
petitioner received the decision of the Labor Arbiter on July 16, 1999.
This factual finding is supported by sufficient evidence, and we take
it as binding on us. Petitioner then simultaneously filed its "Appeal
Memorandum", "Notice of Appeal" and "Motion to Reduce Bond", by
registered mail on July 29, 1999, under Registry Receipt No. 003098.
These were received by the NLRC on July 30, 1999. The appeal to
the NLRC should have been perfected, as provided by its Rules,
within a period of 10 days from receipt by petitioner of the decision
on July 16, 1999. Clearly, the filing of the appeal — three days after
July 26, 1999 — was already beyond the reglementary period and in
violation of the NLRC Rules and the pertinent Article on Appeal in the
Labor Code.
Failure to perfect an appeal renders the decision final and
executory. The right to appeal is a statutory right and one who seeks
to avail of the right must comply with the statute or the rules. The
rules, particularly the requirements for perfecting an appeal within the
reglementary period specified in the law, must be strictly followed as
they are considered indispensable interdictions against needless
delays and for the orderly discharge of judicial business. It is only in
highly meritorious cases that this Court will opt not to strictly apply the
rules and thus prevent a grave injustice from being done. The
exception does not obtain here. Thus, we are in agreement that the
decision of the Labor Arbiter already became final and executory
because petitioner failed to file the appeal within 10 calendar days
from receipt of the decision.
Clearly, the NLRC committed no grave abuse of discretion in
dismissing the appeal before it. It follows that the Court of Appeals,
too, did not err, nor gravely abuse its discretion, in sustaining the
NLRC Order, by dismissing the petition for certiorari before it. Hence,
with the primordial issue resolved, we find no need to tarry on the
other issues raised by the petitioner.

163
Page 164

Case Digests by: CRUZ, ASHER GRACE M.

DIAMOND TAXI v. LLAMAS


G.R. No. 190724. March 12, 2014
Brion, J.

DOCTRINE:
ARTICLE 227
The dismissal of an employee's appeal on purely technical
ground is inconsistent with the constitutional mandate on protection to
labor." Under the Constitution and the Labor Code, the State is bound
to protect labor and assure the rights of workers to security of tenure
— tenurial security being a preferred constitutional right that, under
these fundamental guidelines, technical infirmities in labor pleadings
cannot defeat.

FACTS:
Llamas worked as a taxi driver for petitioner Diamond Taxi,
owned and operated by petitioner Bryan Ong. On July 18, 2005,
Llamas filed before the Labor Arbiter (LA) a complaint for illegal
dismissal against the petitioners.
In their position paper, the petitioners denied dismissing
Llamas. They claimed that Llamas had been absent without official
leave for several days, beginning July 14, 2005 until August 1, 2005.
The petitioners submitted a copy of the attendance logbook to prove
that Llamas had been absent on these cited dates. They also pointed
out that Llamas committed several traffic violations in the years 2000-
2005 and that they had issued him several memoranda for acts of
insubordination and refusal to heed management instructions. They
argued that these acts — traffic violations, insubordination and
refusal to heed management instructions — constitute grounds for
the termination of Llamas' employment.
Llamas failed to seasonably file his position paper.
On November 29, 2005, the LA rendered a decision dismissing
Llamas' complaint for lack of merit. The LA held that Llamas was not
dismissed, legally or illegally. Rather, the LA declared that Llamas left
his job and had been absent for several days without leave. Llamas
received a copy of this LA decision on January 5, 2006. Meanwhile,
he filed his position paper on December 20, 2005. In his position
paper, Llamas claimed that he failed to seasonably file his position
paper because his previous counsel, despite his repeated pleas, had
continuously deferred compliance with the LA's orders for its
submission. Hence, he was forced to secure the services of another

164
Page 165

counsel on December 19, 2005 in order to comply with the LA's


directive.
On the merits of his complaint, Llamas alleged that he had a
misunderstanding with Aljuver Ong, Bryan's brother and operations
manager of Diamond Taxi, on July 13, 2005 (July 13, 2005 incident).
When he reported for work on July 14, 2005, Bryan refused to give
him the key to his assigned taxi cab unless he would sign a prepared
resignation letter. He did not sign the resignation letter. He reported
for work again on July 15 and 16, 2005, but Bryan insisted that he
sign the resignation letter prior to the release of the key to his
assigned taxi cab. Thus, he filed the illegal dismissal complaint.
On January 16, 2006, Llamas filed before the LA a motion for
reconsideration of its November 29, 2005 decision. The LA treated
Llamas' motion as an appeal per Section 15, Rule V of the 2005
Revised Rules of Procedure of the NLRC (2005 NLRC Rules) (the
governing NLRC Rules of Procedure at the time Llamas filed his
complaint before the LA).
In its May 30, 2006 resolution, the NLRC dismissed for non-
perfection Llamas' motion for reconsideration treated as an appeal.
The NLRC pointed out that Llamas failed to attach the required
certification of non-forum shopping per Section 4, Rule VI of the 2005
NLRC Rules. Llamas moved to reconsider the May 30, 2006 NLRC
resolution; he attached the required certification of non-forum
shopping. When the NLRC denied his motion for reconsideration in
its August 31, 2006 resolution, Llamas filed before the CA a petition
for certiorari.

ISSUES:
Whether the NLRC committed grave abuse of discretion in
dismissing Llamas’ appeal on mere technicality

RULING:
Yes. Under Article 221 (now Article 227) of the Labor Code,
"the Commission and its members and the Labor Arbiters shall use
every and all reasonable means to ascertain the facts in each case
speedily and objectively and without regard to technicalities of law or
procedure, all in the interest of due process." Consistently, we have
emphasized that "rules of procedure are mere tools designed to
facilitate the attainment of justice. A strict and rigid application which
would result in technicalities that tend to frustrate rather than promote
substantial justice should not be allowed . . . . No procedural rule is
sacrosanct if such shall result in subverting justice." Ultimately, what
should guide judicial action is that a party is given the fullest
opportunity to establish the merits of his action or defense rather than
for him to lose life, honor, or property on mere technicalities.

165
Page 166

Then, too, we should remember that "the dismissal of an


employee's appeal on purely technical ground is inconsistent with the
constitutional mandate on protection to labor." Under the Constitution
and the Labor Code, the State is bound to protect labor and assure
the rights of workers to security of tenure — tenurial security being a
preferred constitutional right that, under these fundamental
guidelines, technical infirmities in labor pleadings cannot defeat.
In this case, Llamas' action against the petitioners concerned
his job, his security of tenure. This is a property right of which he could
not and should not be deprived of without due process. But, more
importantly, it is a right that assumes a preferred position in our legal
hierarchy.
Under these considerations, we agree that the NLRC
committed grave abuse of discretion when, in dismissing Llamas'
appeal, it allowed purely technical infirmities to defeat Llamas'
tenurial security without full opportunity to establish his case's merits.

166
Page 167

Case Digests by: CRUZ, ASHER GRACE M.

SARA LEE PHILIPPINES, INC. v. MACATLANG


G.R. No. 180147. January 14, 2015
Perez, J.

DOCTRINE:
COMPROMISE AGREEMENTS
Article 273 of the Labor Code of the Philippines authorizes
compromise agreements voluntarily agreed upon by the parties, in
conformity with the basic policy of the State "to promote and
emphasize the primacy of free collective bargaining and negotiations,
including voluntary arbitration, mediation and conciliation, as modes
of settling labor or industrial disputes." "The provision reads:"

ART. 227. Compromise Agreements. — Any compromise


settlement, including those involving labor standard laws,
voluntarily agreed upon by the parties with the assistance of the
Bureau or the regional office of the Department of Labor, shall
be final and binding upon the parties. The National Labor
Relations Commission or any court shall not assume
jurisdiction over issues involved therein except in case of
noncompliance thereof or if there is prima facie evidence that
the settlement was obtained through fraud, misrepresentation,
or coercion.

A compromise agreement is valid as long as the consideration


is reasonable and the employee signed the waiver voluntarily, with a
full understanding of what he was entering into.

FACTS:

Aris permanently ceased operations on 9 October 1995


displacing 5,984 rank-and-file employees. On 26 October 1995, FAPI
was incorporated prompting former Aris employees to file a case for
illegal dismissal on the allegations that FAPI was a continuing business
of Aris. SLC, SLP and Cesar Cruz were impleaded as defendants
being major stockholders of FAPI and officers of Aris, respectively. On
30 October 2004, the Labor Arbiter found the dismissal of 5,984 Aris
employees illegal and awarded them monetary benefits amounting to
P3,453,664,710.86. The judgment award is composed of separation
pay of one month for every year of service, backwages, moral and
exemplary damages and attorney's fees.

167
Page 168

The Corporations filed a Notice of Appeal with Motion to Reduce


Appeal Bond. They posted a P4.5 Million bond. The NLRC granted the
reduction of the appeal bond and ordered the Corporations to post an
additional P4.5 Million bond.

The 5,984 former Aris employees, represented by Emilinda


Macatlang (Macatlang petition), filed a petition for review before the
Court of Appeals insisting that the appeal was not perfected due to
failure of the Corporations to post the correct amount of the bond which
is equivalent to the judgment award.

While the case was pending before the appellate court, the
NLRC prematurely issued an order setting aside the decision of the
Labor Arbiter for being procedurally infirmed.

The Court of Appeals, on 26 March 2007, ordered the


Corporations to post an additional appeal bond of P1 Billion.

ISSUES:
Whether the compromise agreement is valid

RULING:
No. A review of the compromise agreement shows a gross
disparity between the amount offered by the Corporations compared
to the judgment award. The judgment award is P3,453,664,710.86 or
each employee is slated to receive P577,149.85. On the other hand,
the P342,284,800.00 compromise is to be distributed among 5,984
employees which would translate to only P57,200.00 per employee.
From this amount, P8,580.00 as attorney's fees will be deducted,
leaving each employee with a measly P48,620.00. In fact, the
compromised amount roughly comprises only 10% of the judgment
award.
In our Decision, the appeal bond was set at P725 Million after
taking into consideration the interests of all parties. To reiterate, the
underlying purpose of the appeal bond is to ensure that the employer
has properties on which he or she can execute upon in the event of
a final, providential award. Thus, nonpayment or woefully insufficient
payment of the appeal bond by the employer frustrates these ends.
As a matter of fact, the appeal bond is valid and effective from the
date of posting until the case is terminated or the award is satisfied.
Our Decision highlights the importance of an appeal bond such that
said amount should be the base amount for negotiation between the
parties. As it is, the P342,284,800.00 compromise is still measly
compared to the P725 Million bond we set in this case, as it only
accounts to approximately 50% of the reduced appeal bond.

168
Page 169

In our Decision, we have already directed the NLRC to act with


dispatch in resolving the merits of the case upon receipt of the cash
or surety bond in the amount of P725 Million within 10 days from
receipt of the Decision. If indeed the parties want an immediate and
expeditious resolution of the case, then the NLRC should be
unhindered with technicalities to dispose of the case. Accepting an
outrageously low amount of consideration as compromise defeats the
complainants' legitimate claim.

169
Page 170

Case Digest by: CUNANAN, CYRUS S.

DELA ROSA LINER VS. BORELA


G.R. No. 207286. July 29, 2015
Brion, J.

DOCTRINE:
ARTICLE 227
While rights may be waived, the waiver must not be contrary to
law, public policy, morals, or good customs; or prejudicial to a third
person with a right recognized by law. In labor law, claims for 13th-
month pay, overtime pay, and statutory wages, among others, cannot
simply be generally waived as they are granted for workers' protection
and welfare; it takes more than a general waiver to give up workers'
rights to these legal entitlements.

FACTS:

Respondents Calixto Borela, bus driver, and Estelo Amarille,


conductor, filed separate complaints against petitioners Dela Rosa
Liner, Inc., a public transport company, Rosauro Dela Rosa, Sr., and
Nora Dela Rosa, for underpayment/non-payment of salaries, holiday
pay, overtime pay, service incentive leave pay, 13th month pay, sick
leave and vacation leave, night shift differential, illegal deductions, and
violation of Wage Order Nos. 13, 14, 15 and 16.

The petitioners asked the labor arbiter to dismiss the case for
forum shopping. They alleged that CA 13th Division disposed of a
similar case between the parties after they entered into a compromise
agreement which covered all claims and causes of action they had
against each other in relation to the respondents' employment.

LA & NLRC: The LA upheld the petitioners' position and dismissed


the complaint on grounds of forum shopping. Respondents appealed
to the NLRC, which granted the same, reversing the LA’s decision. The
NLRC held that the respondents could not have committed forum
shopping as there was no identity of causes of action between the two
cases. Respondent’s then filed a petition for certiorari to the CA.

CA: The CA denied the petition. It found no grave abuse of


discretion in the NLRC ruling that the respondents did not commit
forum shopping when they filed their second complaint.

170
Page 171

ISSUE:
Whether or not the CA erred in upholding the NLRC ruling that
there was neither forum shopping nor res judicata that would bar the
second complaint. – NO.

RULING:
The Supreme Court held that contrary to the petitioners'
submission, respondents' second complaint, a money claim, was not a
"similar case" to the first complaint. The elements of forum shopping
are: (1) identity of parties; (2) identity of rights asserted and relief
prayed for, the relief being founded on the same facts; and (3) identity
of the two preceding particulars such that any judgment rendered in
the other action will, regardless of which party is successful, amount to
res judicata in the action under consideration. Forum shopping and res
judicata were not applicable in the present case. There was no identity
of rights asserted and reliefs prayed for, and the judgment rendered in
the previous action would not amount to res judicata in the action now
under consideration.

There was also no identity of causes of action in the first complaint


and in the second complaint. The first complaint involved illegal
dismissal/suspension, unfair labor practice with prayer for damages
and attorney's fees; while the second complaint (the subject of the
present appeal) involves claims for labor standards benefits — the
petitioners' alleged violation of Wage Orders Nos. 13, 14, 15 and 16;
non-payment of respondents' sick and vacation leave pays, 13th-
month pay, service incentive leave benefit, overtime pay, and night
shift differential.

The Court was also not persuaded that "The Compromise


Agreement covered all claims and causes of action that the parties
may have against each other in relation to the private respondents'
employment." The compromise agreement had been concluded to
terminate the illegal dismissal and unfair labor case then pending
before the CA. While the parties agreed that no further action shall be
brought by the parties against each other, they pointedly stated that
they referred to actions on the same grounds. The phrase same
grounds can only refer to the grounds raised in the first complaint and
not to any other grounds.

Lastly, the provision "to all claims and damages or losses either
party may have against each other whether those damages or losses
are known or unknown, foreseen or unforeseen" in the compromise
agreement was held as too sweeping effectively excludes any claims
by the respondents against the petitioners, including those that by law
and jurisprudence cannot be waived without appropriate consideration
such as non-payment or underpayment of overtime pay and wages.

171
Page 172

While rights may be waived, the waiver must not be contrary to law,
public policy, morals, or good customs; or prejudicial to a third person
with a right recognized by law. In labor law, claims for 13th-month pay,
overtime pay, and statutory wages, among others, cannot simply be
generally waived as they are granted for workers' protection and
welfare; it takes more than a general waiver to give up workers' rights
to these legal entitlements.

172
Page 173

Case Digest by: CUNANAN, CYRUS S.

MAGSAYSAY MARITIME VS. DE JESUS


G.R. No. 203943 August 30, 2017
Leonen, J.

DOCTRINE:
ARTICLE 227
A conditional settlement of a judgment award may be treated as a
compromise agreement and a judgment on the merits of the case if it
turns out to be highly prejudicial to one of the parties.

FACTS:

Petitioner Magsaysay Maritime Corporation, the local manning


agent of Princess Cruise Lines, Limited, hired Bernardine De Jesus as
an Accommodation Supervisor for the cruise ship Regal Princess.
Bernardine boarded Regal Princess and he eventually disembarked 10
months later after his contract of employment ended. Bernardine was
soon diagnosed with Aortic Aneurysm and he had a coronary
angiography. He also underwent a Left Axillofemoral Bypass but later
died. Respondent Cynthia De Jesus, Bernardine's widow, filed a
complaint against Magsaysay for payment of death benefits, medical
expenses, sickness allowance, damages, and attorney's fees. Cynthia
and Magsaysay were unable to amicably settle the case; hence, they
were directed to submit their respective position papers.

LA/RTC/NLRC RULING:LA: granted Cynthia's complaint and


directed Magsaysay to pay her claims for death benefits, additional
benefits, burial expenses, and attorney's fees.

NLRC: denied Magsaysay's appeal and upheld the Labor Arbiter's


finding that Bernardine's cardio-vascular disease was work-related. It
also noted that while the general rule in compensability of death is that
a seafarer's death must have occurred during the term of the
employment contract, an exception to this rule is when a seafarer
contracted an illness while under the contract and this illness caused
his death Magsaysay paid Cynthia an amount as conditional
satisfaction of the judgment award against it and without prejudice to
its Petition for Certiorari pending before the Court of Appeals. In light
of the conditional settlement between the parties, the Labor Arbiter
considered the case closed and terminated but without prejudice to
Magsaysay's pending petition before the Court of Appeals.

173
Page 174

CA: dismissed the petition for being moot and academic.

APPEAL TO THE SC: Petitioners filed their Petition for Review on


Certiorari where they continue to assert that the Court of Appeals erred
in dismissing their Petition for Certiorari for being moot and academic.
Petitioners emphasize that Leonis Navigation v. Villamater stated that
if the Court of Appeals grants a petition for certiorari, the assailed
decision of the National Labor Relations Commission will become void
ab initio and will never attain finality. Petitioners maintain that Leonis
ruled that even if the employer voluntarily pays the judgment award,
the seafarer's beneficiary is estopped from claiming that the
controversy has ended with the Labor Arbiter's Order closing and
terminating the case. This is because the beneficiary acknowledged
that the payment received was without prejudice to the final outcome
of the petition for certiorari pending before the Court of Appeals.

ISSUE:
1. Whether or not the payment of money judgment has rendered
the Petition for Certiorari before the Court of Appeals moot and
academic.

2. Whether or not the award of death benefits was proper.

RULING:
1. Yes. In the instant case, the parties entered into a compromise
agreement when they executed a Conditional Satisfaction of
Judgment Award. However, the prohibition on the part of
respondent to pursue any of the available legal remedies
should the Court of Appeals or Supreme Court reverse the
judgment award of the labor tribunals or prosecute any other
suit or action in another country puts the seafarer's
beneficiaries at a grave disadvantage. Thus, Career Philippines
is applicable and the Court of Appeals did not err in treating the
conditional settlement as an amicable settlement, effectively
rendering the Petition for Certiorari moot and academic. Article
2028 of the Civil Code defines a compromise agreement as "a
contract whereby the parties, by making reciprocal
concessions, avoid a litigation or put an end to one already
commenced." Parties freely enter into a compromise
agreement, making it a judgment on the merits of the case with
the effect of res judicata upon them. While the general rule is
that a valid compromise agreement has the power to render a
pending case moot and academic, being a contract, the parties
may opt to modify the legal effects of their compromise
agreement to prevent the pending case from becoming moot.

174
Page 175

In the Conditional Satisfaction of Judgment Award, respondent


acknowledged receiving a conditional payment of the judgment
award from the petitioners. Both parties agreed that the
payment of the judgment award was without prejudice to the
pending certiorari proceedings before the Court of Appeals and
was only made to prevent the imminent execution being
undertaken by respondent and the National Labor Relations
Commission. Finally, in the event the judgment award of the
labor tribunals is reversed by the Court of Appeals or by this
Court, respondent agreed to return whatever she would have
received back to petitioners and in the same vein, if the Court
of Appeals or this Court affirms the decisions of the labor
tribunals, petitioners shall pay respondent the balance of the
judgment award without need of demand. Respondent then
signed a Receipt of Payment where she reiterated the
undertakings, she took in the Conditional Satisfaction of
Judgement Award. However, in the Affidavit of Heirship,
respondent was prohibited from seeking further redress against
petitioners, making the compromise agreement ultimately
prejudicial to respondent.

2. Yes. Section 20 (A) of the POEA-SEC requires that for a


seafarer to be entitled to death benefits, he must have suffered a work-
related death during the term of his contract. Against the self-serving
denials of the respondents, complainant has shown that her husband,
prior to his death, suffered chest pains while on board and reported his
condition but he was not allowed to seek medical attention. When he
was repatriated, he asked the respondents anew for medical check up
but his request was again denied. Having substantially established that
the causative circumstances leading to her husband's death had
transpired during his employment, the complainant is entitled to the
death compensation and other benefits under the POEA Standard
Contract. Probability and not the ultimate degree of certainty is the test
of proof in compensation proceedings.

175
Page 176

Case Digest by: CUNANAN, CYRUS S.

GBMLT MANPOWER VS. MALINAO


G.R. No. 189262 July 6, 2015
Sereno, CJ.

DOCTRINE:
ARTICLE 229
The appeal bond requirement for judgments involving monetary
awards may be relaxed in meritorious cases, as in instances when a
liberal interpretation would serve the desired objective of resolving
controversies on the merits.

FACTS:

Sometime in May 2005, respondent applied to petitioner for a job


as teacher for deployment abroad. She was endorsed for the post of
accounting lecturer. Petitioner issued her a wage response slip, with a
monthly salary of USD900. Respondent was assigned to teach at the
Alemaya University in Ethiopia. Later on, a memorandum was issued
where respondent's designation was lowered from lecturer to assistant
lecturer with a monthly salary of USD600. Respondent later on
requested Vice President Alamirew to issue a notice of termination to
her. Another memorandum was issued addressing which indicated that
the qualification of respondent was disputed as the faculty never
recruited expatriate staff who were bachelor's degree holders.
Respondent took great offense at being referred to as a bachelor's
degree holder, insisting that she was a certified public accountant and
a law graduate. Thus, VP Almirew issued the notice of termination to
respondent stating that respondent is insulting students, staff and
management and that it is difficult to expect any positive contribution
by keeping her. Thus, the notice was also deemed as a three-month
advance notice to her as regards contract termination, pursuant to the
provisions in their contract.

While waiting for the three-month period to expire, the respondent


was offered a post at the Internal Audit Department in the University,
which she initially accepted, only to later reject the offered post. She
was later repatriated and signed a Quitclaim and Release in favor of
petitioner for US$900.

176
Page 177

LA: The LA found respondent to be unduly repatriated in breach


of the employment contract. The labor arbiter also declared that the
Quitclaim and Release was not valid.

NLRC: In her Motion to Dismiss Appeal, respondent indicated that


petitioner had received a copy of the Decision of the labor arbiter on
13 April 2007, giving it a period until 23 April 2007 within which to
perfect its appeal. When petitioner filed its memorandum of appeal on
20 April 2007, it issued a check as payment for the appeal bond. The
check was presented for payment only on 23 April 2007. Considering
that it takes three days for checks to clear — and that checks only
produce the effect of payment when they have been cashed — the
appeal bond was posted beyond the 10-day reglementary period.
Hence, according to respondent, petitioner's appeal was not perfected,
and the labor arbiter's ruling had attained finality.

The NLRC issued a Decision dismissing respondent's complaint,


because her claims had been the subject of a valid release, waiver and
quitclaim. It held that respondent was a CPA Lawyer who could hardly
be "duped into signing any document…”

CA: The CA reinstated the Decision of the labor arbiter with


modifications. The CA ruled that the amount of USD900 given to
respondent by virtue of the Quitclaim and Release was
unconscionable. As regards the appeal bond before the NLRC, the CA
ruled that since petitioner's check payment was encashed only after
the reglementary period within which to appeal, the appeal was
considered to have been filed out of time. According to the CA, the
rules provide that only a cash or surety bond may be considered as
appeal bond, and noncompliance with the rule was fatal to petitioner's
cause.

ISSUE:
1. Whether respondent was illegally dismissed

2. Whether the Quitclaim and Release was valid

3. Whether petitioner's appeal was perfected on time

RULING:
1. NO. The Contract of Employment signed by respondent is first
and foremost a contract, which has the force of law between the parties
as long as its stipulations are not contrary to law, morals, public order,
or public policy. We had occasion to rule that stipulations providing that
either party may terminate a contract even without cause are legitimate
if exercised in good faith.

177
Page 178

Respondent also never denied the grounds cited in the notice of


termination. Despite the grounds cited in the notice of termination.
When respondent accepted the post offered at the Internal Audit
Department, the parties decided to revert to the status quo ante of
harmonious employment relationship and to do away with the previous
termination of her employment. It was she who terminated the Contract
of Employment, when she sent a letter to President Kassa declaring
that the advance notice of termination is still in force and effect and she
cannot now claim that she was illegally dismissed.

2. YES. In this case, respondent admits that she had a full


understanding of the terms and conditions of the Quitclaim and
Release and voluntarily signed it. The bone of contention is the
reasonableness of the amount of USD900 as consideration for the
waiver of all other purported claims against petitioner. Respondent's
contentions that she "was in dire need of cash" and that "[s]he was
forced by circumstances of need to sign the document" do not qualify
as coercion or undue influence that give rise to a vice of consent. "Dire
necessity" is an acceptable ground to nullify quitclaims only if the
consideration is unconscionably low and the employee was tricked into
accepting it. Respondent is a learned professional and a teacher no
less. Anyone would be hard put to trick her into agreeing to something
like signing a waiver

3. YES. The intention of the requirement [of employers posting a


cash or surety bond to perfect appeal is fulfilled when the employer is
able to deposit with the NLRC an amount that is equivalent to the
monetary award adjudged by the labor arbiter in the employee's favor,
and that shall subsist until the final resolution of the appeal In this case,
there is no question that the NLRC accepted the appeal bond posted
by petitioner through a current-dated check, as evidenced by Official
Receipt No. 0701550 dated 20 April 2007. That check was deposited
to the bank account of the NLRC on 23 April 2007 without incident.

To our mind, the appeal of petitioner has been perfected on time


by virtue of its compliance with the appeal bond requirement. We note
that its payment of the appeal bond through the issuance of a check
was not even an issue before the NLRC. Nevertheless, we have had
occasion to rule that the appeal bond requirement for judgments
involving monetary awards may be relaxed in meritorious cases, as in
instances when a liberal interpretation would serve the desired
objective of resolving controversies on the merits. We do not see why
the same liberality — if at all needed — cannot be applied to this case
in particular, in which it is clear that respondent's allegations of illegal
dismissal and money claims are unfounded.

178
Page 179

Case Digest by: CUNANAN, CYRUS S.

OROZCO VS. CA
G.R. No. 155207 April 29, 2005
Nachura, J.

DOCTRINE:
ARTICLE 229
While the posting of a cash or surety bond is jurisdictional and is
a condition sine qua non to the perfection of an appeal, there is a
plethora of jurisprudence recognizing exceptional instances wherein
the Court relaxed the bond requirement as a condition for posting the
appeal. Technicality should not be allowed to stand in the way of
equitably and completely resolving the rights and obligations of the
parties.

FACTS:
Philippine Daily Inquirer engaged the services of petitioner
Wilhelmina Orozco to write a weekly column for its Lifestyle section.
Petitioner religiously submitted her articles every week, except for a
six-month stint in New York City when she, nonetheless, sent several
articles through mail. She received compensation of P250.00 – later
increased to P300.00 – for every column published.

Later on, however, petitioner claims that her then editor, Logarta,
told her that respondent Magsanoc, the PDI Editor in Chief, wanted to
stop publishing her column for no reason at all and advised petitioner
to talk to Magsanoc herself. Petitioner narrates that when she talked to
Magsanoc, the latter informed her that it was PDI Chairperson Apostol
who had asked to stop publication of her column, but that in a
telephone conversation with Apostol, the latter said that Magsanoc
informed Apostol that the Lifestyle section already had many
columnists. On the other hand, PDI claims that Magsanoc met with the
Lifestyle section editor to discuss how to improve said section. They
agreed to cut down the number of columnists by keeping only those
whose columns were well-written, with regular feedback and following.
In their judgment, petitioner’s column failed to improve, continued to be
superficially and poorly written, and failed to meet the high standards
of the newspaper. Hence, they decided to terminate the petitioner's
column. Aggrieved by the newspaper’s action, the petitioner filed a
complaint for illegal dismissal, backwages, moral and exemplary
damages, and other money claims before the NLRC.

LA: Ruled in favor of petitioner, stating that there was an employer-


employee relationship between the parties.

179
Page 180

NLRC: Affirmed LA's decision. It initially noted that PDI failed to


perfect its appeal, under Article 223 of the Labor Code, due to non-
filing of a cash or surety bond. The NLRC said that the reason proffered
by PDI for not filing the bond – that it was difficult or impossible to
determine the amount of the bond since the Labor Arbiter did not
specify the amount of the judgment award – was not persuasive.

CA: Reversed NLRC's decision on the ground that the facts of the
case failed to measure up to the control test necessary for an
employer-employee relationship to exist. Hence, the petitioner filed a
petition for review with the Supreme Court on the ground that the CA
erred in granting the petition of PDI by reason of failure to pay the cash
bond.

ISSUE:
Whether or not the CA erred in not dismissing PDI's petition for
review on the ground of failure to post a cash or surety bond - NO

RULING:
NO. In a Resolution dated April 29, 2005, the Court, without giving
due course to the petition, ordered the Labor Arbiter to clarify the
amount of the award due petitioner and, thereafter, ordered PDI to post
the requisite bond. Upon compliance therewith, the petition would be
given due course. Labor Arbiter Amansec clarified that the award
under the Decision amounted to P15,350.00. Thus, PDI posted the
requisite bond.

While the posting of a cash or surety bond is jurisdictional and is


a condition sine qua non to the perfection of an appeal, there is a
plethora of jurisprudence recognizing exceptional instances wherein
the Court relaxed the bond requirement as a condition for posting the
appeal. As a rule, compliance with the requirements for the perfection
of an appeal within the reglementary period is mandatory and
jurisdictional. However, in National Federation of Labor Unions v.
Ladrido as well as in several other cases, this Court relaxed the
requirement of the posting of an appeal bond within the reglementary
period as a condition for perfecting the appeal. This is in line with the
principle that substantial justice is better served by allowing the appeal
to be resolved on the merits rather than dismissing it based on a
technicality. In the case of NFLU v. Ladrido III, this Court postulated
that "private respondents cannot be expected to post such appeal bond
equivalent to the amount of the monetary award when the amount
thereof was not included in the decision of the labor arbiter." The
computation of the amount awarded to petitioner not having been
clearly stated in the decision of the labor arbiter, private respondents
had no basis for determining the amount of the bond to be posted.

180
Page 181

Thus, while the requirements for perfecting an appeal must be


strictly followed as they are considered indispensable interdictions
against needless delays and for orderly discharge of judicial business,
the law does admit of exceptions when warranted by the
circumstances. Technicality should not be allowed to stand in the way
of equitably and completely resolving the rights and obligations of the
parties. But while this Court may relax the observance of reglementary
periods and technical rules to achieve substantial justice, it is not
prepared to give due course to this petition and make a
pronouncement on the weighty issue obtaining in this case until the law
has been duly complied with and the requisite appeal bond duly paid
by private respondents. Nevertheless, Records show that PDI has
complied with the Court’s directive for the posting of the bond; thus,
that issue has been laid to rest.

181
Page 182

Case Digest by: CUNANAN, CYRUS S.

LEPANTO CONSOLIDATED MINING VS. ICAO


G.R. No. 196047 January 15, 2014
Sereno, CJ.

DOCTRINE:
ARTICLE 229
When the law does not clearly provide a rule or norm for the
tribunal to follow in deciding a question submitted, but leaves to the
tribunal the discretion to determine the case in one way or another, the
judge must decide the question in conformity with justice, reason, and
equity, in view of the circumstances of the case.

FACTS:

Private respondent filed a complaint for illegal dismissal against


petitioner, he alleged that he was a lead miner in petitioners
underground mine in Benguet, that he did some barring down and
installed blast holes, that after the blasting he decided to take a bath
where he was frisked by a Security Guard who forcibly pulled his
skullgard from his head causing it to fall down, thereafter he saw
another Security Guard pick up a wrapped object and invited him to
the investigation office. Private respondent was then charged with
highgrading, or the act of concealing, possessing or unauthorized
extraction of highgrade material/ore without proper authority.
Consequently, he was dismissed from work.

LA Ruling: The LA held that petitioner is liable for illegal dismissal


and ordered them to pay respondent his backwages and separation
pay for Php345,879. Petitioner filed an Appearance with Memorandum
of Appeal before the NLRC. Instead of posting the required cash or
surety bond, they filed a Consolidated Motion for Release of Cash
Bond and to Apply Bond Subject for Release as Payment for Appeal
Bond concerning the bond that they had posted in a separate case
which had already been decided, with finality, in their favor.

NLRC Ruling: The NLRC dismissed the appeal of petitioner for


non-perfection of appeal. It held that petitioner had failed to post the
required appeal bond, and that the consolidated motion cannot be
considered as compliance with the requirement.

CA Ruling: The CA affirmed the NLRC’s decision.

182
Page 183

ISSUE:

Whether or not the Consolidated Motion to release the cash bond


it had posted in another case, with a view to applying the same to the
present case is compliance with the rules for posting a cash or security
bond. -YES

RULING:

YES. Petitioner has substantially complied with the appeal bond


requirement based on the doctrine that, when the law does not clearly
provide a rule or norm for the tribunal to follow in deciding a question
submitted, but leaves to the tribunal the discretion to determine the
case in one way or another, the judge must decide the question in
conformity with justice, reason, and equity, in view of the
circumstances of the case. In this case, it is found that, first, there is
no question that the appeal was filed within the 10- day reglementary
period, second, it is also undisputed that petitioner has an
unencumbered amount of money in the form of cash in the custody of
the NLRC, which have ought to have been released to petitioner.

A bond is encumbered and bound to a case only for as long as i)


the case has not been finally decided, resolved or terminated, or ii) the
award has not been satisfied. In this case, since the money is now
unencumbered, he employer who posted it should now have
unrestricted access to the cash which he may as he pleases. Third, the
cash bond is more than enough to cover the appeal bond, fourth, this
ruling remains faithful to the spirit behind the appeal bond requirement
which is to ensure that workers will receive the money awarded in their
favor when the employers appeal eventually fails, finally, the Court has
liberally applied the NLRC rules and the Labor Code on posting an
appeal bond only in very highly exceptional cases.

183
Page 184

Case Digest by: CUNANAN, CYRUS S.

FOREVER SECURITY VS. FLORES


G.R. No. 147961 September 7, 2007
Azcuna, J.

DOCTRINE:
ARTICLE 229
The requirement of a cash or surety bond for the perfection of an
appeal from the Labor Arbiter's monetary award is not only mandatory
but jurisdictional as well, and non-compliance therewith is fatal and has
the effect of rendering the award final and executory. This is consistent
with the State's constitutional mandate to afford full protection to labor
in order to forcefully and meaningfully underscore labor as a primary
social and economic force.

FACTS:
Romeo Flores and Lope Rallama were employed as security
officers of Forever Security and General Services (Forever Security) in
1990 and 1988, respectively. As security officers, they worked for 12
hours every day including Sundays and holidays. On February 15,
1993, Forever Security dismissed Flores and Rallama on the ground
that they abandoned their posts, duties and responsibilities as security
guards. Hence, they filed Complaints for Illegal Dismissal with the
NLRC, against Forever Security and/or its Executive Vice President
Antonio Garin. In his complaint, Flores alleged that he did not receive
his salary from January 18 to February 15, 1993. The reason given
was that he was allegedly absent without official leave (AWOL) since
December 26, 1992. He vehemently denied this and averred that his
absence from such date until January 15, 1993 was with the company's
consent and that he resumed work since then until he was terminated
from service. Rallama, on the other hand, averred that he failed to go
to work on January 3 to 31, 1993 because he was hospitalized. When
he returned for work, he was told that he was considered AWOL. Flores
and Rallama further claimed that during their employment with Forever
Security, they were not paid the proper overtime pay, premium pay,
rest day and holiday pay, and night shift differential, service incentive
leave pay and 13th month pay. They prayed for reinstatement with
payment of backwages and other monetary claims plus attorney's fees.
For its part, Forever Security, thru its Vice President Garin, averred
that Flores and Rallama went on vacation and sick leave, respectively,
but failed to report for work thereafter, thus, they were considered to
have abandoned their posts, duties and responsibilities which is a
ground for their dismissal from service. It likewise asserted that it had
fully paid the complainants' salaries and wages, overtime pay,

184
Page 185

premium pay for holiday and rest day, night shift differential, service
incentive leave pay and 13th month pay. It prayed that the case be
dismissed for lack of merit. In the scheduled hearing, Forever Security
and Vice President Garin failed to appear.

LA: in favor of complainants-employees. Dismissal was illegal and


unjustified. Ordered Forever Security to reinstate them to their former
positions without loss of seniority rights and other privileges.

NLRC: On appeal, instead of posting a cash or surety bond,


Forever Security and Garin filed a Motion for Extension of Time to
File/Submit Appeal/Surety Bond, alleging among others, that
appellants are finalizing appropriate arrangements with an insurance
bonding company; that due to lack of material time, they are not able
to file surety/appeal bond.

The appeal was dismissed for appellants' failure to perfect the


same in accordance with the requirements of the Labor Code,
specifically by posting the required cash or surety bond. Appellants
filed a Motion for Reconsideration but it was denied. Flores and
Rallama moved for the issuance of a Writ of Execution which was
favorably acted upon by the Labor Arbiter. Pursuant to the Writ of
Execution and Alias Writs of Execution, a Notice of Garnishment was
issued. Thereafter, Forever Security and Garin filed an urgent ex-parte
motion to quash the writ of execution and alias writs of execution since
it was not yet final and executory considering that respondents' counsel
failed to receive a copy of the same. However, the same was denied.

CA: ruled against the petitioner. The court further held that the
petitioner failed to observe the procedural rules provided for by the
Labor Code. As to the allegation that petitioner's counsel did not
receive a copy of the NLRC resolution denying its motion for
reconsideration, the court applied the rule on presumption of receipt in
the ordinary course of mail. Consequently, there is nothing left to be
done except to implement the orders of execution and garnishment.

ISSUE:
Whether or not CA erred for holding final and executory the
decision of the Labor Arbiter.

RULING:
NO. The petition is without merit on both the procedural and
substantive issues. At the outset, this Court would like to point out that
the present case at bench had long become final and executory for
failure of petitioner to comply with procedural rules on perfection of
appeals to the NLRC. Article 223 of the Labor Code sets forth the rules

185
Page 186

on appeal from the Labor Arbiter's monetary award, thus: Article 223.
Appeal. - Decisions, awards, or orders of the Labor Arbiter are final
and executory unless appealed to the Commission by any or both
parties within 10 calendar days from receipt of such decisions, awards,
or orders.

In case of a judgment involving a monetary award, an appeal by


the employer may be perfected only upon the posting of a cash or
surety bond issued by a reputable bonding company duly accredited
by the Commission in the amount equivalent to the monetary award in
the judgment appealed from. Moreover, Sections 1 and 6, Rule VI of
the New Rules of Procedure of the NLRC39 provide:

SEC 1. Periods of Appeal. - Decisions, awards or orders of the


Labor Arbiter and the POEA Administrator shall be final and executory
unless appealed to the Commission by any or both parties within ten
(10) calendar days from receipt of such decisions, awards or orders of
the Labor Arbiter x x x.

SEC. 6. Bond. - In case the decision of a Labor Arbiter, POEA


Administrator and Regional Director or his duly authorized hearing
officer involves a monetary award, an appeal by the employer shall be
perfected only upon the posting of a cash or surety bond issued by a
reputable bonding company duly accredited by the Commission or the
Supreme Court in an amount equivalent to the monetary award.

The requirement of a cash or surety bond for the perfection of an


appeal from the Labor Arbiter's monetary award is not only mandatory
but jurisdictional as well, and noncompliance therewith is fatal and has
the effect of rendering the award final and executory. The logical
purpose of an appeal bond is to insure, during the period of appeal,
against any occurrence that would defeat or diminish recovery under
the judgment if subsequently affirmed; it also validates and justifies, at
least prima facie, an interpretation that would limit the amount of the
bond to the aggregate of the sums awarded other than in the concept
of moral and exemplary damages. This is consistent with the State's
constitutional mandate to afford full protection to labor in order to
forcefully and meaningfully underscore labor as a primary social and
economic force As clearly explained by the NLRC, petitioner filed its
appeal and paid the appeal fee on April 27, 1995. Instead of posting a
cash or surety bond, it filed a motion for extension of time to file appeal
or surety bond. Petitioner asked that it be given until May 27, 1995
within which to post the bond. Apparently, petitioner did not make good
its promise on the specified date. At the time of the issuance of the
NLRC Resolution dated July 31, 1995 dismissing the appeal, no bond
was posted. Records reveal that it was only in 1999 when the petitioner
posted the bond, long after the finality of the Labor Arbiter's decision.

186
Page 187

Case Digest by: CUNANAN, CYRUS S.

UERM-MEMORIAL MEDICAL CENTER V. NLRC


G.R. No. 110419 March 3, 1997
Puno, J.

DOCTRINE:
ARTICLE 229
Requiring a cash or surety bond in the amount equivalent to the
monetary award in the judgment appealed from for the appeal to be
perfected, may be considered a jurisdictional requirement,
nevertheless, adhering to the principle that substantial justice is better
served by allowing the appeal on the merits threshed out by the NLRC,
the Court finds and so holds that the foregoing requirements of the law
should be given a liberal interpretation.

FACTS:

The facts show that on 14 December 1987 Republic Act No. 6640
took effect which mandated a P10.00 peso increase on the prevailing
daily minimum wage of P54.00. There was a difference of P95.00 in
the salaries of the two classes of employees. Private respondents who
are rank and file employees demanded payment of the difference.
Before the parties could settle their dispute, Republic Act No. 6727
took effect on 1 July 1989 which again increased the daily minimum
wage in the private sector (whether agricultural or non-agricultural) by
P25.00. Again, there was a difference of P237.42 per month between
the salaries of union members and non-union members. In September
1987, petitioners increased the hiring rate of the new employees to
P188.00 per month. Private respondents once more demanded from
the petitioner’s payment of the salary differential mandated by RA No.
6727 and correction of the wage distortion brought about by the
increase in the hiring rate of new employees. Petitioners challenged
the validity of said Policy Instruction and refused to pay the salaries of
the private respondents for Saturdays and Sundays.

Consequently, a complaint was filed by the private respondents,


represented by the Federation of Free Workers (FFW), claiming salary
differentials under Republic Act Nos. 6640 and 6727, correction of the
wage distortion and the payment of salaries for Saturdays and
Sundays under Policy Instruction No. 54. Labor Arbiter Nieves de
Castro sustained the private respondents except for their claim of wage
distortion.

187
Page 188

Within the reglementary period for appeal, the petitioners filed their
Notice and Memorandum of Appeal with a Real Estate Bond consisting
of land and various improvements therein worth P102,345,650.2 The
private respondents moved to dismiss the appeal on the ground that
Article 223 of the Labor Code, as amended, requires the posting of a
cash or surety bond. The NLRC directed petitioners to post a cash or
surety bond of P17,082,448.56 with a warning that failure to do so
would cause the dismissal of the appeal. The petitioners filed a Motion
for Reconsideration alleging it is not in a viable financial condition to
post a cash bond nor to pay the annual premium of P700,000.00 for a
surety bond. On 6 October 1992, the NLRC dismissed petitioners’
appeal. Petitioners’ Motion for Reconsideration was also denied by the
NLRC in a resolution dated 7 June 1993.

ISSUE:
Whether or not in perfecting an appeal to the NLRC a property
bond is excluded by the two forms of appeal bond—cash or surety—
as enumerated in Article 223 of the Labor Code.

RULING:

NO. We have given a liberal interpretation to this provision. IN YBL


(Your Bus Line) v. NLRC we ruled: “x x x that while Article 223 of the
Labor Code, as amended by Republic Act No. 6715, requiring a cash
or surety bond in the amount equivalent to the monetary award in the
judgment appealed from for the appeal to be perfected, may be
considered a jurisdictional requirement, nevertheless, adhering to the
principle that substantial justice is better served by allowing the appeal
on the merits threshed out by the NLRC, the Court finds and so holds
that the foregoing requirements of the law should be given a liberal
interpretation.”

The Court reiterates the policy which stresses the importance of


deciding cases on the basis of their substantive merit and not on strict
technical rules. In the case at bar, the judgment involved is more than
P17 million and its precipitate execution can adversely affect the
existence of petitioner medical center. Likewise, the issues involved
are not insignificant and they deserve a full discourse by our quasi-
judicial and judicial authorities. We are also confident that the real
property bond posted by the petitioners sufficiently protects the
interests of private respondents should they finally prevail. It is not
disputed that the real property offered by petitioners is worth
P102,345,650. The judgment in favor of private respondent is only a
little more than P17 million.

188
Page 189

Case Digest by: CUNANAN, CYRUS S.

MANILA MINING vs. LOWITO AMOR, ET. AL.


G.R. No. 182800 April 20, 2015
Perez, J.

DOCTRINE:
ARTICLE 229
The posting of a bond is indispensable to the perfection of an
appeal in cases involving monetary awards from the decision of the
Labor Arbiter. Since it is the posting of a cash or surety bond which
confers jurisdiction upon the NLRC, the rule is settled that non-
compliance is fatal and has the effect of rendering the award final and
executory.

FACTS:
Respondents were regular employees of petitioner Manila Mining
Corporation, a domestic corporation which operated a mining claim in
Placer, Surigao del Norte, in large-scale open-pit mining for gold and
copper ore. Petitioner maintained TP No. 7, a tailings containment
facility required for the storage of waste materials generated by its
mining operations. When the mine tailings being pumped reached the
maximum level, petitioner temporarily shut down its mining operations
pending approval of its application to increase said facility’s capacity
by the DENR-Environment Management Bureau.

Although the DENR-EMB issued a temporary authority for it to be


able to continue operating TP No. 7 for another (6) months and to
increase its capacity, petitioner failed to secure an extension permit
when said temporary authority eventually lapsed. Petitioner served a
notice, informing its employees and DOLE Regional Office No. XII of
the temporary suspension of its operations for six months and the
temporary lay-off of two-thirds of its employees. It was extended for
another 6 months. Respondents filed the complaint for constructive
dismissal and monetary claims.

Ruling of lower courts: The LA held the petitioners liable for


constructive dismissal. Aggrieved, petitioner filed an appeal and
moved for the reduction of the appeal bond to ₱100,000.00, on the
ground that its financial losses in the preceding years had rendered it
unable to put up one in cash and/or surety equivalent to the monetary
award. Respondents argued that the appeal bond tendered by
petitioner was so grossly disproportionate to monetary award for the
same to be considered substantial compliance with the requirements
for the perfection of an appeal from a Labor Arbiter’s decision. NLRC

189
Page 190

ruled that, under Article 283 of the Labor Code, respondents were not
even entitled to separation pay considering the eventual closure of
their employer’s business due to serious business losses or financial
reverses. Respondents filed under Rule 65 petition for certiorari in the
CA insisting that petitioner’s memorandum of appeal was filed 65 days
after the lapse of reglementary period for appeal and the check in the
sum of ₱100,000.00 deposited by petitioner by way of appeal bond
was dishonored upon presentment for payment. The CA decreed that
the Labor Arbiter’s Decision had already attained finality and, for said
reason, had been placed beyond the NLRC’s power of review.

ISSUE:
Whether or not the appeal was perfected.

RULING:

NO. While it is true that reduction of the appeal bond has been
allowed in meritorious cases on the principle that substantial justice is
better served by allowing appeals on the merits, it has been ruled that
the employer should comply with the following conditions: (1) the
motion to reduce the bond shall be based on meritorious grounds; and
(2) a reasonable amount in relation to the monetary award is posted
by the appellant, otherwise the filing of the motion to reduce bond shall
not stop the running of the period to perfect an appeal.

In this case, we see that with no proof to substantiate its claim,


petitioner moved for a reduction of the appeal bond on the preferred
basis of serious losses and reverses it supposedly sustained in the
years prior to the rendition of the Labor Arbiter's decision.

Respondent correctly called attention to the fact that the check


submitted by petitioner was dishonored upon presentment for
payment, thereby rendering the tender thereof ineffectual. Having filed
its motion and memorandum on the very last day of the reglementary
period for appeal, petitioner had no one but itself to blame for failing to
post the full amount pending the NLRC’s action on its motion for
reduction of the appeal bond. The posting of a bond is indispensable
to the perfection of an appeal in cases involving monetary awards from
the decision of the Labor Arbiter. Since it is the posting of a cash or
surety bond which confers jurisdiction upon the NLRC, the rule is
settled that non-compliance is fatal and has the effect of rendering the
award final and executory.

190
Page 191

Case Digest by: CUNANAN, CYRUS S.

BANAHAW BROADCASTING vs. PACANA


G.R. No. 171673 May 30, 2011
Leonardo-De Castro, J.

DOCTRINE:
ARTICLE 229
BBC’s function is purely commercial or proprietary and not
governmental. As such, BBC cannot be deemed entitled to an
exemption from the posting of an appeal bond.

FACTS:
Respondents DXWG personnel are supervisory and rank and file
employees of the DXWG-Iligan City radio station which is owned by
petitioner Banahaw Broadcasting Corporation (BBC), a corporation
managed by Intercontinental Broadcasting Corporation (IBC). On
August 29, 1995, the DXWG personnel filed with the Sub-regional
Arbitration Branch No. XI, Iligan City a complaint for illegal dismissal,
unfair labor practice, reimbursement of unpaid CBA benefits, and
attorney’s fees against IBC and BBC. On June 21, 1996, LA Alug
rendered his Decision awarding the DXWG personnel a total of
₱12,002,157.28 as unpaid CBA benefits consisting of unpaid wages
and increases, 13th month pay, longevity pay, sick leave cash
conversion, rice and sugar subsidy, retirement pay, loyalty reward and
separation pay. The LA denied the other claims of the DXWG
personnel for Christmas bonus, educational assistance, medical
check-up and optical expenses. Both sets of parties appealed to the
NLRC. On May 15, 1997, a Motion to Dismiss, Release, Waiver and
Quitclaim, was jointly filed by IBC and the DXWG personnel based on
the latter’s admission that IBC is not their employer as it does not own
DXWG-Iligan City.

On April 21, 1997, the NLRC granted the Motion and dismissed
the case with respect to IBC. BBC filed a Motion for Reconsideration
alleging that (1) neither BBC nor its duly authorized representatives or
officers were served with summons and/or a copy of the complaint
when the case was pending before the Labor Arbiter or a copy of the
Decision therein; (2) since the liability of IBC and BBC is solidary, the
release and quitclaim issued by the DXWG personnel in favor of IBC
totally extinguished BBC’s liability; (3) it was IBC that effected the
termination of the DXWG personnel’s employment; (4) the DXWG
personnel are members of the IBC union and are not employees of
BBC; and (5) the sequestered properties of BBC cannot be levied
upon. On December 12, 1997, the NLRC issued a Resolution vacating

191
Page 192

the Decision of LA Alug and remanding the case to the arbitration


branch of origin on the ground that while the complaint was filed
against both IBC and BBC, only IBC was served with summons,
ordered to submit a position paper, and furnished a copy of the
assailed decision.

LA: LA Palangan rendered a Decision adjudging BBC to be liable


for the same amount (P12,002,157.28) discussed in the vacated
Decision of LA Alug. Both BBC and respondents appealed to the NLRC
anew. In their appeal, the DXWG personnel reasserted their claim for
the remaining CBA benefits not awarded to them, and alleged error in
the reckoning date of the computation of the monetary award. BBC, in
its own Memorandum of Appeal, challenged the monetary award itself,
claiming that such benefits were only due to IBC, not BBC, employees.
In the same Memorandum of Appeal, BBC incorporated a Motion for
the Recomputation of the Monetary Award in order that the appeal
bond may be reduced.

NLRC: The NLRC issued an Order denying the Motion for the
Recomputation of the Monetary Award. According to the NLRC, such
recomputation would result in the premature resolution of the issue
raised on appeal. The NLRC ordered BBC to post the required bond
within 10 days from receipt of said Order, with a warning that
noncompliance will cause the dismissal of the appeal for non-
perfection. Instead of complying with the Order to post the required
bond, BBC filed a Motion for Reconsideration, alleging this time that
since it is wholly owned by the Republic of the Philippines, it need not
post an appeal bond. NLRC denied the Motion for Reconsideration of
BBC on its September 16, 1999, Order and accordingly dismissed the
appeal of BBC for non-perfection. The NLRC likewise dismissed the
appeal of the DXWG personnel for lack of merit in the same Decision.
BBC filed with the Court of Appeals a Petition for Certiorari under Rule
65 of the Rules of Court.

CA: The CA denied BBC’s Petition for Certiorari. It held that BBC,
though owned by the government, is a corporation with a personality
distinct from the Republic or any of its agencies or instrumentalities,
and therefore do not partake in the latter’s exemption from the posting
of appeal bonds.

ISSUE:

Whether or not BBC is exempt from posting an appeal bond – NO

192
Page 193

RULING:

NO. As a general rule, the government and all the attached


agencies with no legal personality distinct from the former are exempt
from posting appeal bonds, whereas government-owned and
controlled corporations (GOCCs) are not similarly exempted. This
distinction is brought about by the very reason of the appeal bond itself:
to protect the presumptive judgment creditor against the insolvency of
the presumptive judgment debtor. When the State litigates, it is not
required to put up an appeal bond because it is presumed to be always
solvent. This exemption, however, does not, as a general rule, apply
to GOCCs for the reason that the latter has a personality distinct from
its shareholders. Thus, while a GOCCs majority stockholder, the State,
will always be presumed solvent, the presumption does not necessarily
extend to the GOCC itself. However, when ad GOCC becomes a
government machinery to carry out a declared government policy, it
becomes similarly situated as its majority stockholder as there is the
assurance that the government will necessarily fund its primary
functions. Thus, a GOCC that is sued in relation to its governmental
functions may be, under appropriate circumstances, exempted from
the payment of appeal fees.

In the case at bar, BBC was organized as a private corporation,


sequestered in the 1980ds and the ownership of which was
subsequently transferred to the government in a compromise
agreement. Further, it is stated in its Amended Articles of Incorporation
that BBC has the primary function to engage in commercial radio and
television broadcasting, and for this purpose, to establish, operate and
maintain such stations, both terrestrial and satellite or interplanetary,
as may be necessary for broadcasting on a network wide or
international basis. It is therefore crystal clear that BBC’s function is
purely commercial or proprietary and not governmental. As such, BBC
cannot be deemed entitled to an exemption from the posting of an
appeal bond.

Consequently, the NLRC did not commit an error, and much less
grave abuse of discretion, in dismissing the appeal of BBC on account
of non-perfection of the same. In doing so, the NLRC was merely
applying Article 223 of the Labor Code, which provides that in case of
a judgment involving a monetary award, an appeal by the employer
may be perfected only upon the posting of a cash or surety bond issued
by a reputable bonding company duly accredited by the Commission
in the amount equivalent to the monetary award in the judgment
appealed from.

193
Page 194

The posting of the appeal bond within the period provided by law
is not merely mandatory but jurisdictional. The failure on the part of
BBC to perfect the appeal thus had the effect of rendering the judgment
final and executory. Neither was there an interruption of the period to
perfect the appeal when BBC filed (1) its Motion for the recomputation
of the Monetary Award in order to reduce the appeal bond, and (2) its
Motion for Reconsideration of the denial of the same. In the case at
bar, BBC already took a risk when it filed its Motion for the
Recomputation of the Monetary Award without posting the bond itself.

The Motion for the Recomputation of the Monetary Award filed by


BBC was itself tantamount to a motion for extension to perfect the
appeal, which is prohibited by the rules. The NLRC already exhibited
leniency when, instead of dismissing the appeal outright, it merely
ordered BBC to post the required bond within 10 days from receipt of
said Order, with a warning that noncompliance will cause the dismissal
of the appeal for non-perfection. When BBC further demonstrated its
unwillingness by completely ignoring this warning and by filing a Motion
for Reconsideration on an entirely new ground, the NLRC cannot be
said to have committed grave abuse of discretion by making good its
warning to dismiss the appeal.

194
Page 195

Case Digest by: CUNANAN, CYRUS S.

MCBURNIE V. GANZON
G.R. No. 178034 October 17, 2013
Reyes, J.

DOCTRINE:
ARTICLE 229
The bond may be reduced upon motion by the employer, this is
subject to the conditions that (1) the motion to reduce the bond shall
be based on meritorious grounds; and (2) a reasonable amount in
relation to the monetary award is posted by the appellant, otherwise
the filing of the motion to reduce bond shall not stop the running of the
period to perfect an appeal.

FACTS:
McBurnie, an Australian national, instituted a complaint for illegal
dismissal and other monetary claims against the respondents.
McBurnie claimed that on May 11, 1999, he signed a five-year
employment agreement with the company EGI as an Executive Vice-
President who shall oversee the management of the company's hotels
and resorts within the Philippines. He performed work for the company
until sometime in November 1999, when he figured in an accident that
compelled him to go back to Australia while recuperating from his
injuries. While in Australia, he was informed by respondent Ganzon
that his services were no longer needed because their intended project
would no longer push through. The respondents opposed the
complaint, contending that their agreement with McBurnie was to
jointly invest in and establish a company for the management of hotels.
They did not intend to create an employer-employee relationship, and
the execution of the employment contract that was being invoked by
McBurnie was solely for the purpose of allowing McBurnie to obtain an
alien work permit in the Philippines. At the time McBurnie left for
Australia for his medical treatment, he had not yet obtained a work
permit. Moreover, McBurnie never acquired a work permit which is
required in order for aliens to work in the Philippines.

LA: Ruled in favor of McBurnie. Responded appealed said


decision to the NLRC and filed a Motion to Reduce Bond and posted
an appeal bond in the amount of P100,000. They stated that the award
of “more than P60 million pesos to a single foreigner who had no work
permit and who left the country for goof one month after the purported
commencement of his employment” was a patent nullity. They also
cited business losses as the reason why they lack the capacity to pay
the bond of almost P60 million.

195
Page 196

NLRC: Denied the Motion to Reduce Bond "in cases involving


monetary award, an employer seeking to appeal the [LA's] decision to
the Commission is unconditionally required by Art. 223, Labor Code to
post bond in the amount equivalent to the monetary award . . . ."Thus,
the NLRC required from the respondents the posting of an additional
bond in the amount of P54,083,910.00. Due to the failure of respondent
to post the required additional bond, the NLRC dismissed their appeal.
Respondent filed with a CA a TRO and Writ of Preliminary Injunction

CA: Granted the writ of preliminary injunction. CA also granted the


motion to reduce appeal bond (reduced the appeal bond to P10 million.
the CA ruled that the NLRC committed grave abuse of discretion in
immediately denying the motion without fixing an appeal bond in an
amount that was reasonable, as it denied the respondents of their right
to appeal from the decision of the LA. The CA explained that "(w)hile
Art. 223 of the Labor Code requiring bond equivalent to the monetary
award is explicit, Section 6, Rule VI of the NLRC Rules of Procedure,
as amended, recognized as exception a motion to reduce bond upon
meritorious grounds and upon posting of a bond in a reasonable
amount in relation to the monetary award.

ISSUE:
Whether or not the court may relax the requirement in labor cases
from a decision involving a monetary award may be perfected only
upon posting of a cash or surety bond.

RULING:

YES. In accordance with the foregoing, although the general rule


provides that an appeal in labor cases from a decision involving a
monetary award may be perfected only upon the posting of a cash or
surety bond, the Court has relaxed this requirement under certain
exceptional circumstances in order to resolve controversies on their
merits.

These circumstances include: (1) the fundamental consideration


of substantial justice; (2) the prevention of miscarriage of justice or of
unjust enrichment; and (3) special circumstances of the case combined
with its legal merits, and the amount and the issue involved. Guidelines
that are applicable in the reduction of appeal bonds were also
explained in Nicol v. Footjoy Industrial Corporation. The bond
requirement in appeals involving monetary awards has been and may
be relaxed in meritorious cases, including instances in which (1) there
was substantial compliance with the Rules, (2) surrounding facts and
circumstances constitute meritorious grounds to reduce the bond, (3)

196
Page 197

a liberal interpretation of the requirement of an appeal bond would


serve the desired objective of resolving controversies on the merits, or
(4) the appellants, at the very least, exhibited their willingness and/or
good faith by posting a partial bond during the reglementary period. A
serious error of the NLRC was its outright denial of the motion to
reduce the bond, without even considering the respondents'
arguments and totally unmindful of the rules.

197
Page 198

Case Digest by: CUNANAN, CYRUS S.

SARA LEE PHILIPPINES vs. MACATLANG


G.R. No. 180147 January 14, 2015
Perez, J.

DOCTRINE:
ARTICLE 229
The NLRC retains its authority and duty to resolve the motion and
determine the final amount of bond that shall be posted by the
appellant, still in accordance with the standards of "meritorious
grounds" and "reasonable amount." Should the NLRC, after
considering the motion’s merit, determine that a greater amount or the
full amount of the bond needs to be posted by the appellant, then the
party shall comply accordingly. The appellant shall be given a period
of 10 days from notice of the NLRC order within which to perfect the
appeal by posting the required appeal bond.

FACTS:
This resolution treats of the following cases:

1.) Motion for Reconsideration with Urgent Petition for the Court's
Approval of the Pending "Motion for Leave of Court to File and Admit
Herein Statement and Confession of Judgment — to Buy Peace and/or
Secure against any Possible Contingent Liability by Sara Lee
Corporation" filed by Sara Lee Philippines, Inc. (SLPI), Aris Philippines,
Inc. (Aris), Sara Lee Corporation (SLC) and Cesar C. Cruz, 2.) Motion
for Reconsideration filed by Fashion Accessories Phils., Inc. (FAPI),
and 3) Manifestation of Conformity to the Motion for Leave of Court to
File and Admit Confession of Judgment to Buy Peace and/or to Secure
against any Possible Contingent Liability by Petitioner SLC.

Aris permanently ceased operations on 9 October 1995 displacing


5,984 rank-and-file employees. Thereafter, FAPI was incorporated
prompting former employees of Aris to file a case for illegal dismissal
on the allegations that FAPI was a continuing business of Aris. SLC,
SLP and Cesar Cruz were impleaded as defendants being major
stockholders of FAPI and officers of Aris, respectively.

LA: The Labor Arbiter found the dismissal of 5,984 Aris employees
illegal and awarded them monetary beneAts amounting to
P3,453,664,710.86. The judgment award is composed of separation
pay of one month for every year of service, backwages, moral and
exemplary damages and attorney's fees. The Corporations filed a

198
Page 199

Notice of Appeal with Motion to Reduce Appeal Bond. They posted a


P4.5 Million bond.

NLRC: The NLRC granted the reduction of the appeal bond and
ordered the Corporations to post an additional P4.5 Million bond. The
5,984 former Aris employees, represented by Emilinda Macatlang
(Macatlang petition), filed a petition for review before the Court of
Appeals insisting that the appeal was not perfected due to failure of the
Corporations to post the correct amount of the bond which is equivalent
to the judgment award. While the case was pending before the
appellate court, the NLRC prematurely issued an order setting aside
the decision of the Labor Arbiter for being procedurally infirmed.

CA: The CA ordered the Corporations to post an additional appeal


bond of P1 Billion. The decision of CA was modified by the Supreme
Court in their 4 June 2014 decision which directed the corporations to
post P725 Million, in cash or surety bond, within 10 days from the
receipt of the decision.

Hence, this MR where the Corporations, relying on McBurnie v.


Ganzon, argued that only 10% of the monetary award is required to be
posted as bond. Furthermore, the Corporations filed a Motion to Admit
Confession of Judgment claiming that the Corporations entered into a
compromise with some of the former Aris employees.

ISSUE:
1.) Whether the 10% bond requirement is applicable in this case;

2.) Whether the ruling of the NLRC is premature thus, the


applicability of the judicial courtesy rule.

RULING:

1.No. The 10% requirement pertains to the reasonable amount


which the NLRC would accept as the minimum of the bond that should
accompany the motion to reduce bond in order to suspend the period
to perfect an appeal under the NLRC rules. The 10% is based on the
judgment award and should in no case be construed as the minimum
amount of bond to be posted in order to perfect appeal. There is no
room for a different interpretation when in the case of McBurnie it was
made clear that the percentage of bond set is provisional. As explained
in the case of McBurnie v. Ganzon, The NLRC retains its authority and
duty to resolve the motion and determine the Anal amount of bond that
shall be posted by the appellant, still in accordance with the standards
of "meritorious grounds" and "reasonable amount." Should the NLRC,

199
Page 200

after considering the motion's merit, determine that a greater amount


or the full amount of the bond needs to be posted by the appellant, then
the party shall comply accordingly. The appellant shall be given a
period of 10 days from notice of the NLRC order within which to perfect
the appeal by posting the required appeal bond.

The appeal bond was set at P 725 Million after taking into
consideration the interests of all parties. To reiterate, the underlying
purpose of the appeal bond is to ensure that the employer has
properties on which he or she can execute upon in the event of a final,
providential award. Thus, non-payment or woefully insufficient
payment of the appeal bond by the employer frustrates these ends. As
a matter of fact, the appeal bond is valid and effective from the date of
posting until the case is terminated or the award is satisfied.

2.) Yes. There was a legal impediment for NLRC to issue the
resolution vacating the LA’s decision. Judicial courtesy indeed applies
if there is a strong probability that the issues before the higher court
would be rendered moot as a result of the continuation of the
proceedings in the lower court. The 19 December 2006 ruling of the
NLRC would moot the appeal filed before the higher courts because
the issue involves the appeal bond which is an indispensable
requirement to the perfection of the appeal before the NLRC. Unless
this issue is resolved, the NLRC should be precluded from ruling on
the merits of the case. This is the essence of judicial courtesy.

Note: The compromise agreement herein was declared void being


contrary to law, morals, good customs and public policy. A review of
the compromise agreement shows a gross disparity between the
amount offered by the Corporations compared to the judgment award.
The judgment award is P3,453,664,710.86 or each employee is slated
to receive P577,149.85. On the other hand, the P342,284,800.00
compromise is to be distributed among 5,984 employees which would
translate to only P57,200.00 per employee. From this amount,
P8,580.00 as attorney's fees will be deducted, leaving each employee
with a measly P48,620.00. In fact, the compromised amount roughly
comprises only 10% of the judgment award. Accepting an outrageously
low amount of consideration as compromise defeats the complainants'
legitimate claim. The Court DENIED petitioners' Motion for
Reconsideration and Motion for Leave of Court to File and Admit
Herein Statement and Confession of Judgment; and the respondents'
Partial Motion for Reconsideration for their lack of merit.

200
Page 201

Case Digest by: CUNANAN, CYRUS S.

AFP GENERAL INSURANCE vs. MOLINA


G.R. No. 151133 June 30, 2008
Quisumbing, J.

DOCTRINE:
ARTICLE 229
1. The perfection of an appeal by an employer "only" upon the
posting of a cash or surety bond clearly shows the intent of the
lawmakers to make the posting of a cash or surety bond by the
employer to be the exclusive means by which an employer's appeal
may be perfected.

2. The surety bond shall remain valid and in force until finality and
execution of judgment, with the resultant discharge of the surety
company only thereafter.

FACTS:
Respondents filed a complaint for illegal dismissal against Radon
Security & Allied Services Agency and/or Raquel Aquias and Ever
Emporium, Inc. The Labor Arbiter ruled in their favor and ordered
Radon Security to pay them separation pay, backwages, and other
monetary claims. Radon Security appealed the Labor Arbiter's
decision to public respondent NLRC and posted a supersedeas bond,
issued by petitioner AFPGIC as surety. The NLRC affirmed the
decision. The NLRC Research and Information Unit submitted a
Computation of the Monetary Awards in accordance with the NLRC
decision. Radon Security opposed said computation.

Labor Arbiter issued a Writ of Execution incorporating the


computation of the NLRC Research and Information Unit and
dismissed the Motion for Recomputation filed by Radon Security. The
NLRC Sheriff issued a Notice of Garnishment against the supersedeas
bond. Ever Emporium, Inc. and Radon Security moved to quash the
writ of execution but was denied. Radon Security appealed to the
NLRC. AFPGIC filed an Omnibus Motion to Quash Notice/Writ of
Garnishment and to Discharge AFPGIC's Appeal Bond before the LA
on the ground that said bond has been cancelled and thus non-existent
in view of the failure of Radon Security to pay the yearly premiums.

LA: The LA denied pointing out that the question of non-payment


of premiums is a dispute between the party who posted the bond and
the insurer; to allow the bond to be cancelled because of the non-

201
Page 202

payment of premiums would result in a factual and legal absurdity


wherein a surety will be rendered nugatory by the simple expedient of
non-payment of premiums.

NLRC: Dismissed AFPGIC’s appeal. FPGIC's theory that the bond


cannot anymore be proceeded against for failure of Radon Security to
pay the premium is untenable, considering that the bond is effective
until the finality of the decision. A contrary ruling would allow
respondents to simply stop paying the premium to frustrate satisfaction
of the money judgment AFPGIC’s motion for reconsideration was also
denied.

CA: Denied the special civil action for certiorari and the motion for
reconsideration.

ISSUE:
Whether or not AFPGIC can cancel the surety bond on the ground
that Radon Security failed to pay premiums.

RULING:

NO. Rule VI, Section 6 states that the cash or surety bond posted
in appeals involving monetary awards in labor disputes "shall be in
effect until final disposition of the case." This could only be construed
to mean that the surety bond shall remain valid and in force until finality
and execution of judgment, with the resultant discharge of the surety
company only thereafter, if we are to give teeth to the labor protection
clause of the Constitution. To construe the provision any other way
would open the floodgates to unscrupulous and heartless employers
who would simply forego paying premiums on their surety bond in
order to evade payment of the monetary judgment.

AFPGIC contends that under Section 64 of the Insurance Code an


insurer may cancel a policy upon non-payment of the premium. Said
cancellation is binding upon the beneficiary as the right of a beneficiary
is subordinate to that of the insured. Further, in South Sea Surety &
Insurance Co., Inc. v. CA, the SC held that payment of premium is a
condition precedent to and essential for the efficaciousness of a
contract of insurance. Respondents, on the other hand, adopted that
ruling of the CA that as a supersedeas bond was posted for the benefit
of a third person to guarantee that the money judgment will be satisfied
in case it is affirmed on appeal, the third person who stands to benefit
from said bond is entitled to notice of its cancellation for any reason.
The NLRC should have been notified to enable it to take the proper
action under the circumstances. Respondents argue that from its very

202
Page 203

nature, a supersedeas bond remains effective and the surety liable


thereon until formally discharged from said liability. To hold otherwise
would enable a losing party to frustrate a money judgment by the
simple expedient of ceasing to pay premiums. The Supreme Court
agrees. Moreover, the Insurance Code supports the respondents'
arguments. AFGIC's reliance on Sections 64 and 77 of the Insurance
Code is misplaced as said provisions refer to insurance contracts in
general. The instant case pertains to a surety bond thus the applicable
provision is Section 177 which specifically governs suretyship.

In the present case, when AFPGIC cancelled the surety bond


because Radon Security failed to pay the premiums, it gave due notice
to the latter but not to the NLRC. By its failure to give notice to the
NLRC, AFPGIC failed to acknowledge that the NLRC had jurisdiction
not only over the appealed case, but also over the appeal bond. This
oversight amounts to disrespect and contempt for a quasi-judicial
agency tasked by law with resolving labor disputes. Until the surety is
formally discharged, it remains subject to the jurisdiction of the NLRC.
When the LA directed the NLRC Sheriff to garnish the surety bond
issued by AFPGIC, the latter, as surety, is mandated to comply with
the writ of garnishment, for the bond remains enforceable and under
the jurisdiction of the NLRC until it is discharged. In turn, AFPGIC may
proceed to collect the amount it paid on the bond, plus the premiums
due and demandable, plus any interest owing from Radon Security.
This is pursuant to the principle of subrogation enunciated in Article
2067 of the Civil Code which we apply to the suretyship agreement
between AFPGIC and Radon Security, in accordance with Section 178
of the Insurance Code.

203
Page 204

Case Digest by: CUNANAN, CYRUS S.

ISLRIZ TRADING vs. CAPADA


G.R. No. 168501 January 31, 2011
Del Castillo, J.

DOCTRINE:
ARTICLE 229
Employees are entitled to their accrued salaries during the period
between the Labor Arbiter's order of reinstatement pending appeal and
the resolution of the NLRC overturning that of the Labor Arbiter.
Otherwise stated, even if the order of reinstatement of the Labor Arbiter
is reversed on appeal, the employer is still obliged to reinstate and pay
the wages of the employee during the period of appeal until reversal
by a higher court or tribunal.

FACTS:

Respondents were drivers and helpers of petitioner Islriz Trading,


a gravel sand business owned and operated by Victor Hugo Lu.
Claiming that they were illegally dismissed, respondents filed a
Complaint for illegal dismissal and nonpayment of overtime pay,
holiday pay, rest day pay, allowances and separation pay against
petitioner before the Labor Arbiter. On his part, petitioner imputed
abandonment of work against respondents.

LA Waldo Gan: Declared Islriz guilty of illegal dismissal and


ordered it to reinstate complainants to their former positions without
loss of seniority rights and the payment of full backwages from date of
dismissal to actual reinstatement.

NLRC: Granted the appeal of Islriz and set aside the Decision of
LA Gan. Respondents then filed with the LA an Ex-Parte Motion to Set
Case for Conference with Motion. They averred therein that since the
Decision of Labor Arbiter Gan ordered their reinstatement, a Writ of
Execution was already issued for the enforcement of its reinstatement
aspect as same is immediately executory even pending appeal. Thus,
respondents prayed that in view of the orders of reinstatement, a
computation of the award of backwages be made and that an Alias Writ
of Execution for its enforcement be issued.

LA through Fiscal Examiner: Issued a Computation of the


backwages. Total: P1.1M.

204
Page 205

LA Danna Castillon: Issued a Writ of Execution to enforce the


monetary award in accordance with the above computation. The
Sheriff issued a Notice of Sale/Levy by virtue of which petitioner’s
properties were levied and set for auction sale. Petitioner brought the
matter to CA arguing that since LA Gan’s Decisions has already been
reversed and set aside by the NLRC, the Writ of Execution issued by
LA Castillon should have confined itself to the said NLRC Resolution
which ordered respondents’ reinstatement without backwages. Hence,
petitioner imputed grave abuse of discretion amounting to lack or
excess of jurisdiction on LA Castillon.

CA: Agreed with LA Castillon that what is sought to be enforced


by the subject Writ of Execution is the accrued salaries owing to
respondents by reason of the reinstatement order of LA Gan.

ISSUE:
Whether or not the respondents may collect their wages during the
period between the LA's order of reinstatement pending appeal and
the NLRC Resolution overturning that of the LA.

RULING:
The Court ruled that respondents are entitled to their accrued
salaries from the time petitioner received a copy of the Decision of the
Labor Arbiter declaring respondents' termination illegal and ordering
their reinstatement up to the date of the NLRC resolution overturning
that of the Labor Arbiter.

The prevailing doctrine on reinstatement pending appeal vis-à-vis


the effect of a reversal on appeal is that, even if the order of
reinstatement of the LA is reversed on appeal, it is obligatory on the
part of the employer to reinstate and pay the wages of the dismissed
employee during the period of appeal until reversal by the higher court
or tribunal. It likewise settled the view that the LA's order of
reinstatement is immediately executory and the employer has to either
re-admit them to work under the same terms and conditions prevailing
prior to their dismissal, or to reinstate them in the payroll, and that
failing to exercise the options in the alternative, employer must pay the
employee's salaries.

However, after the LA's decision is reversed by a higher tribunal,


the employee may be barred from collecting the accrued wages, if it is
shown that the delay in enforcing the reinstatement pending appeal
was without fault on the part of the employer.

The two-fold test in determining whether an employee is barred


from recovering his accrued wages, to wit:

205
Page 206

(1) there must be actual delay or that the order of reinstatement


pending appeal was not executed prior to its reversal; and

(2) the delay must not be due to the employer's unjustified act or
omission.

If the delay is due to the employer's unjustified refusal, the


employer may still be required to pay the salaries notwithstanding the
reversal of the LA's Decision.

In the instant case, Islriz Trading did not undergo any situation
which would justify petitioner's non-exercise of the options provided
under Article 223 of the Labor Code and merely gave as reason in not
immediately effecting reinstatement after he was served with the Writ
of Execution dated April 22, 2002 was that he would first refer the
matter to his counsel as he could not effectively act on the order of
execution without the latter's advice. Petitioner, however, without any
satisfactory reason, failed to fulfill this promise and respondents
remained to be not reinstated until the NLRC resolved petitioner's
appeal.

Evidently, the delay in the execution of respondents' reinstatement


was due to petitioner's unjustified refusal to effect the same. Hence,
respondents have the right to collect their accrued salaries during the
period between the LA's Decision ordering their reinstatement pending
appeal and the NLRC Resolution overturning the same because
petitioner's failure to reinstate them either actually or through payroll
was due to petitioner's unjustified refusal to effect reinstatement.

206
Page 207

Case Digests by: Alecx Frances S. Daquioag

FSFI (FILIPINAS SYSTEMS INC.) V. NLRC


G.R. No. 1234561. December 11, 2003.
Puno, C. J.

DOCTRINE:
ARTICLE 229 (FORMERLY ARTICLE 223)
Art. 223 of the Labor Code and Section 1 of the NLRC Rules of
Procedure provide a ten (10) day period from receipt of the decision of
the Arbiter to file an appeal together with an appeal bond if the decision
involves a monetary award. The late filing of the bond divested the
NLRC of its jurisdiction to entertain petitioner’s appeal.

FACTS:
A complaint for illegal dismissal and monetary claims were filed by
private respondents against their employe. Filipinas Systems, Inc.
(Filsystems for brevity). Filsystems failed to file its position paper in
spite of the order of the Labor Arbiter prompting the Labor Arbiter to
decide in favor of respondents in the illegal dismissal complaints anD
awarded their monetary claims.

Filsystems appealed to the NLRC submitting for the first time


evidence showing that respondents were project employees whose
dismissal was due to the discontinuation of the project they were
assigned. Respondents Questioned the jurisdiction of the NLRC over
the appeal as petitioner belatedly filed the appeal bond however, the

NLRC assumed jurisdiction and remanded the case to the Labor


Arbiter for further proceedings. Respondents’ motion for
reconsideration was denied so they appealed to the CA via a petition
for Certiorari. The CA Held that the NLRC lacks jurisdiction over the
appeal for late filing of the appeal bond and reinstated the Labor
Arbiter’s decision. Peitioner’s motion for reconsideration was denied.

ISSUE:
WON the NLRC acquired jurisdiction over the appeal

RULING:
No. Art.223 of the Labor Code and Section 1 of the NLRC Rules
of Procedure provide a ten(10)day period from receipt of the decision
of the Arbiter to file an appeal together with an appeal bond if the
decision involves a monetary award. Records showed that petitioners
received a copy of the Arbiters decision on October 31. Their
memorandum of appeal was dated November 9, but their appeal bond

207
Page 208

was effected only on November 17. No partial payment of the bond


was made within the reglementary period nor did they submit an
explanation for its late filing. The late filing of the bond divested the
NLRC of its jurisdiction to entertain petitioner’s appeal.

Further, petitioners failed to submit their evidence to the Labor


Arbiter despite opportunities given them and only submitted the
evidence to the NLRC when the decision was adverse to their interest.
The court dismissed the petition and the decision of the Labor Arbiter
was reinstated with the modification that if reinstatement of
respondents is not feasible, petitioner should pay their separation pay
in accordance with law.

208
Page 209

Case Digests by: Alecx Frances S. Daquioag

BUENAOBRA V. LIM KING GUAN


G.R. No. 150147. January 20, 2004.
Corona, C. J.

DOCTRINE:
ARTICLE 229 (FORMERLY ARTICLE 223)
The provision of Article 223 (now 229) of the Labor Code requiring
the posting of bond on appeals involving monetary awards must be
given liberal interpretation in line with the desired objective of resolving
controversies on the merits. If only to achieve substantial justice, strict
observance of the reglementary periods may be relaxed if warranted.

FACTS:
Petitioners (Buenaobra et al) were employees of private
respondent Unix International Export Corporation (UNIX), a
corporation engaged in the business of manufacturing bags, wallets,
and the like. Petitioners filed several cases against UNIX and its
incorporators and officers for unfair labor practice, illegal
lockout/dismissal, underpayment of wages, holiday pay, proportionate
13th month pay, unpaid wages, interest, moral and exemplary
damages, and attorney’s fees.

Labor Arbiter Patis rendered a decision ordering respondent Unix


Export Corporation to pay complainants back wages, separation pay,
wage differentials, regular holiday pay differentials; and proportionate
13th month pay for 1990.

There being no appeal by respondents or petitioners, the decision


of labor arbiter eventually became final and executory. However,
petitioners complained that the decision could not be executed
because UNIX allegedly diverted, invested, and transferred all its
money, assets, and properties to respondent Fuji Zipper Manufacturing
Corporation (FUJI) whose stockholders and officers were also those of
UNIX.

Petitioners filed another complaint against respondents UNIX, its


corporate officers and stockholders of record, and FUJI. Petitioners
mainly prayed that respondents UNIX and FUJI be held jointly and
severally held liable for the payment of the monetary awards ordered
by labor arbiter de Vera.

ISSUE:
WON the posting of bond beyond the reglementary period may
be allowed

209
Page 210

RULING:
Yes. The provision of Article 223 (now 229) of the Labor Code
requiring the posting of bond on appeals involving monetary awards
must be given liberal interpretation in line with the desired objective of
resolving controversies on the merits. If only to achieve substantial
justice, strict observance of the reglementary periods may be relaxed
if warranted. The NLRC, Third Division could not be said to have
abused its discretion in requiring the posting of bond after it denied
private respondents’ motion to be exempted therefrom.

It is true that the perfection of an appeal in the manner and within


the period prescribed by law is not only mandatory but jurisdictional,
and failure to perfect an appeal has the effect of making the judgment
final and executory. However, technicality should not be allowed to
stand in the way of equitably and completely resolving the rights and
obligations of the parties. We have allowed appeals from the decisions
of the labor arbiter to the NLRC, even if filed beyond the reglementary
period, in the interest of justice. The facts and circumstances of the
instant case warrant liberality considering the amount involved and the
fact that petitioners already obtained a favorable judgment on February
23, 1993 against their employer UNIX.

It is only fair and just that respondent FUJI be afforded the


opportunity to be heard on appeal before the NLRC, especially in the
light of labor arbiter Patis later decision holding FUJI jointly and
severally liable with UNIX in the payment of the monetary awards
adjudged by labor arbiter de Vera against UNIX.

210
Page 211

Case Digests by: Alecx Frances S. Daquioag

BERGONIO V. SOUTH EAST ASIAN AIRLINES


G.R. No. 195227. April 21, 2014.
Brion, C. J.

DOCTRINE:
ARTICLE 229 (FORMERLY ARTICLE 223)
A dismissed employee whose case was favorably decided by the
LA is entitled to receive wages pending appeal upon reinstatement,
which reinstatement is immediately executory. Unless the appellate
tribunal issues a restraining order, the LA is duty bound to implement
the order of reinstatement and the employer has no option but to
comply with it. Moreover, an order of reinstatement by the LA is self-
executory; dismissed employee need not apply for and the LA need
not even issue a writ of execution.

FACTS:
Petitioners Bergonio, et. al. filed before the LA a complaint for
illegal dismissal and illegal suspension with prayer of reinstatement
against respondents South East Asian Airlines (SEAIR) and Irene
Dornier as SEAIR President. The LA found that petitioners were
illegally dismissed. The petitioners filed before the LA a motion for
issuance of writ of execution for their immediate reinstatement. The
respondents opposed the motion for execution claiming that the
relationship between them and the petitioners had already been
strained because of the petitioners' threatening text messages. The LA
granted the motion of petitioners. Respondents moved to quash the
writ of execution, maintaining their ground as that of their previous
opposition.

Meanwhile, on February 21, 2006, respondents issued a


memorandum directing the petitioners to report for work on February
24, 2006, which the petitioners failed to do. As the writ of execution
remained unsatisfied, the petitioners filed a motion for recomputation
of accrued wages, which the LA granted.

ISSUE:
Whether the petitioners may recover the accrued wages prior to
the CA's reversal of the LA's May 31, 2005 decision.

RULING:
Yes, under Article 229, in any event, the decision of the Labor
Arbiter reinstating a dismissed or separated employee, insofar as the
reinstatement aspect is concerned, shall immediately be executory,
pending appeal. The employee shall either be admitted back to work

211
Page 212

under the same terms and conditions prevailing prior to his dismissal
or separation or, at the option of the employer, merely reinstated in the
payroll. The posting of a bond by the employer shall not stay the
execution for reinstatement provided herein.

The LA's order for the reinstatement of an employee found


illegally dismissed is immediately executory even during pendency of
the employer's appeal from the decision. Under this provision, the
employer must reinstate the employee — either by physically admitting
him under the conditions prevailing prior to his dismissal, and paying
his wages; or, at the employer's option, merely reinstating the
employee in the payroll until the decision is reversed by the higher
court. Failure of the employer to comply with the reinstatement order,
by exercising the options rendered in the alternative, renders him liable
to pay the employee’s salaries.

Hence, as a general rule, an employee may still recover the


accrued wages up to and despite the reversal by the higher tribunal.
As an exception, an employee may be barred from collecting the
accrued wages if shown that the delay in enforcing the reinstatement
pending appeal was without fault on the part of the employer. To
determine whether an employee is thus barred, two tests must be
satisfied: (1) actual delay; and (2) the delay must not be due to the
employer’s unjustified act or omission.

In the case at bar, there was actual delay in the execution of


reinstatement embodied in May 31, 2005 decision. From said date until
CA reversed the decision on December 17, 2008, the respondents had
not reinstated the petitioners, either actually or in the payroll. In the
second aspect. the delay was due to respondents’ unjustified acts,
considering that respondents filed several pleadings to suspend the
LA’s execution of its reinstatement order, neither they were given an
ample opportunity to comply with the return-to-work directive, because
the February 21, 2006 Memorandum was only delivered in the
afternoon of February 23, 2006. Lastly, petitioners pursued the
execution of the reinstatement aspect of the LA’s decision.

As per the 2005 Revised Rules of Procedure of the NLRC (2005


NLRC Rules) employers are required to submit a report of compliance
within ten (10) calendar days from receipt of the LA's decision,
noncompliance with which signifies a clear refusal to reinstate.

212
Page 213

Case Digests by: Alecx Frances S. Daquioag

LOON ET. AL V. POWERMASTER, INC.


G.R. No. 189404. December 11, 2 013
Brion, C. J.

DOCTRINE:
ARTICLE 229 (FORMERLY ARTICLE 223)
The respondents perfected their appeal with the NLRC because
the revocation of the bonding company's authority has a prospective
application. Paragraph 2, Article 223 of the Labor Code provides that
"[i]n case of a judgment involving a monetary award, an appeal by the
employer may be perfected only upon the posting of a cash or surety
bond issued by a reputable bonding company duly accredited by the
Commission in the amount equivalent to the monetary award in the
judgment appealed from."

FACTS:
Respondents Power Master, Inc. and Tri-C General Services
employed and assigned the petitioners as janitors and leadsmen in
various Philippine Long Distance Telephone Company (PLDT) offices
in Metro Manila area. Subsequently, the petitioners filed a complaint
for money claims against Power Master, Inc., Tri-C General Services
and their officers, the spouses Homer and Carina Alumisin
(collectively, the respondents). The petitioners alleged in their
complaint that they were not paid minimum wages, overtime, holiday,
premium, service incentive leave, and thirteenth month pays. They
further averred that the respondents made them sign blank payroll
sheets. On June 11, 2001, the petitioners amended their complaint and
included illegal dismissal as their cause of action. They claimed that
the respondents relieved them from service in retaliation for the filing
of their original complaint.

Notably, the respondents did not participate in the proceedings


before the Labor Arbiter except on April 19, 2001 and May 21, 2001
when Mr. Romulo Pacia, Jr. appeared on the respondents’ behalf. The
respondents’ counsel also appeared in a preliminary mandatory
conference on July 5, 2001. However, the respondents neither filed
any position paper nor proffered pieces of evidence in their defense
despite their knowledge of the pendency of the case.

ISSUE:
WON the appeal was perfected despite revocation of bonding
company’s authority

213
Page 214

RULING:
Yes. The respondents perfected their appeal with the NLRC
because the revocation of the bonding company's authority has a
prospective application. Paragraph 2, Article 223 of the Labor Code
provides that "[i]n case of a judgment involving a monetary award, an
appeal by the employer may be perfected only upon the posting of a
cash or surety bond issued by a reputable bonding company duly
accredited by the Commission in the amount equivalent to the
monetary award in the judgment appealed from."

Contrary to the respondents’ claim, the issue of the appeal


bond’s validity may be raised for the first time on appeal since its proper
filing is a jurisdictional requirement. The requirement that the appeal
bond should be issued by an accredited bonding company is
mandatory and jurisdictional. The rationale of requiring an appeal bond
is to discourage the employers from using an appeal to delay or evade
the employees' just and lawful claims. It is intended to assure the
workers that they will receive the money judgment in their favor upon
the dismissal of the employer’s appeal.

In the present case, the respondents filed a surety bond issued by


Security Pacific Assurance Corporation (Security Pacific) on June 28,
2002. At that time, Security Pacific was still an accredited bonding
company. However, the NLRC revoked its accreditation on February
16, 2003. Nonetheless, this subsequent revocation should not
prejudice the respondents who relied on its then subsisting
accreditation in good faith. In Del Rosario v. Philippine Journalists, Inc.,
we ruled that a bonding company’s revocation of authority is
prospective in application.

However, the respondents should post a new bond issued by an


accredited bonding company in compliance with paragraph 4, Section
6, Rule 6 of the NLRC Rules of Procedure. This provision states that
"[a] cash or surety bond shall be valid and effective from the date of
deposit or posting, until the case is finally decided, resolved or
terminated or the award satisfied."

214
Page 215

Case Digests by: Alecx Frances S. Daquioag

WATERFRONT CEBU CITY CASINO HOTEL, INC. V.


LEDESMA
G.R. No. 123456. January 01, 2022.
Peralta, C. J.

DOCTRINE:
ARTICLE 229 (FORMERLY ARTICLE 223); RULE 65
When a party to a suit appears by counsel, service of every
judgment and all orders of the court must be sent to the counsel. This
is so because notice to counsel is an effective notice to the client, while
notice to the client and not his counsel is not notice in law. Receipt of
notice by the counsel of record is the reckoning point of the
reglementary period.

FACTS:
Respondent was employed as a House Detective at Waterfront
located at Salinas Drive, Cebu City. Based on the complaints filed
before Waterfront by Christe Mandal, a supplier of a concessionaire of
Waterfront, and Rosanna Lofranco, who was seeking a job at the same
hotel, Ledesma was dismissed from employment. From the affidavits
and testimonies of Christe Mandal and Rosanna Lofranco during the
administrative hearings conducted by Waterfront, the latter found,
among others, that Ledesma kissed and mashed the breasts of Christe
Mandal inside the hotel’s elevator and exhibited his penis and asked
Rosanna Lofranco to masturbate him at the conference room of the
hotel. On August 12, 2008, Ledesma filed a complaint for illegal
dismissal.

ISSUE:
WON the petition for certiorari (Rule 65) was filed on time

RULING:
The unjustified failure of Ledesma to file his petition for certiorari
before the CA within the 60-day period is a ground for the outright
dismissal of said petition. Ledesma erroneously asserted in his petition
for certiorari filed before the CA, that the 60th day is May 15, 2010,
counted from March 15, 2010. In computing a period, the first day shall
be excluded, and the last included hence the last day to file his petition
for certiorari is on May 14, 2010, a Friday. Ledesma therefore belatedly
filed his petition on May 17, 2010.

215
Page 216

When a party to a suit appears by counsel, service of every


judgment and all orders of the court must be sent to the counsel. This
is so because notice to counsel is an effective notice to the client, while
notice to the client and not his counsel is not notice in law. Receipt of
notice by the counsel of record is the reckoning point of the
reglementary period.

216
Page 217

Case Digests by: Alecx Frances S. Daquioag

BALITE V. SS VENTURES
G.R. No. 195109. February 4, 2015.
Peralta, C. J.

DOCTRINE:
ARTICLE 229 (FORMERLY ARTICLE 223)
Section 6. Bond. In case the decision of the Labor Arbiter, or the
Regional Director involves a monetary award, an appeal by the
employer shall be perfected only upon the posting of a bond, which
shall either be in the form of cash deposit or surety bond equivalent in
amount to the monetary award, exclusive of damages and attorney’s
fees.

FACTS:
Respondent SS Ventures International, Inc. is a domestic
corporation duly engaged in the business of manufacturing footwear
products for local sales and export abroad. Petitioners Andy Balite
(Balite), Monaliza Bihasa (Bihasa) and Delfin Anzaldo (Anzaldo) were
regular employees of the respondent company until their employments
were severed for violation of various company policies.

Balite was issued a Show Cause Memorandum by the


respondent company charging him with making false reports, malicious
and fraudulent statements and rumor-mongering against the company;
threatening and intimidating co-workers; refusing to cooperate in the
conduct of investigation; and gross negligence in the care and use of
the company property resulting in the damage of the finished products.

After respondent found Balite’s explanation insufficient, he was


dismissed from employment, through a Notice of Termination. Bihasa,
on the other hand, was charged with absence without leave on two
occasions and with improper behavior, stubbornness, arrogance and
uncooperative attitude towards superiors and employees. Bihasa was
likewise terminated from the service. Anzaldo was also dismissed from
employment after purportedly giving him due process. The records of
the infractions he committed as well as the date of his termination,
however, are not borne by the records.

The three employees charged respondents with illegal dismissal


and recovery of backwages, 13th month pay and attorney’s fees before
the Labor Arbiter. On appeal to the NLRC, the complaint was
dismissed for non-perfection.

217
Page 218

ISSUE:
WON the appeal bond in full is not only mandatory but a
jurisdictional requirement that must be complied with in order to perfect
the appeal

RULING:
Section 4. Requisites for Perfection of Appeal. – (a) The appeal shall
be:
(1) filed within the reglementary period as provided in Section 1
of this Rule;
(2) verified by the appellant himself/herself in accordance with
Section 4, Rule 7 of the Rules of Court, as amended;
(3) in the form a of a memorandum of appeal which shall state
the grounds relied upon and the arguments in support thereof;
the relief prayed for; and with a statement of the date when the
appellant received the appealed decision, award or order;
(4) in three (3) legibly typewritten or printed copies; and
(5) accompanied by:
i) proof of payment of the required appeal fee and legal
research fee;
ii) posting of cash or surety bond as provided in Section 6
of this Rule; and
iii) proof of service upon the other parties.
xxxx

(b) A mere notice of appeal without complying with the other


requisites aforestated shall not stop the running of the period for
perfecting an appeal.
xxxx

Section 6. Bond. - In case the decision of the Labor Arbiter, or the


Regional Director involves a monetary award, an appeal by the
employer shall be perfected only upon the posting of a bond, which
shall either be in the form of cash deposit or surety bond equivalent in
amount to the monetary award, exclusive of damages and attorney’s
fees.

xxxx
In a judgment involving a monetary award, the appeal shall be
perfected only upon (1) proof of payment of the required appeal fee;
(2) posting of a cash or surety bond issued by a reputable bonding
company; and (3) filing of a memorandum of appeal.
In McBurnie v. Ganzon, we harmonized the provision on appeal
that its procedures are fairly applied to both the petitioner and the
respondent, assuring by such application that neither one or the other
party is unfairly favored. We pronounced that the posting of a cash or
surety bond in an amount EQUIVALENT TO 10% of the monetary

218
Page 219

award pending resolution of the motion to reduce appeal bond shall be


deemed sufficient to perfect an appeal

The rule We set in McBurnie was clarified by the Court in Sara


Lee Philippines v. Ermilinda Macatlang. Considering the peculiar
circumstances in Sara Lee, we determined what is the reasonable
amount of appeal bond. We underscored the fact that the amount of
10% of the award is not a permissible bond but is only such amount
that shall be deemed reasonable in the meantime that the appellant’s
motion is pending resolution by the Commission. The actual
reasonable amount yet to be determined is necessarily a bigger
amount.

We sustain the Court of Appeals in so far as it increases the


amount of the required appeal bond. But we deem it reasonable to
reduce the amount of the appeal bond to ₱725 Million. This directive
already considers that the award if not illegal, is extraordinarily huge
and that no insurance company would be willing to issue a bond for
such big money. The amount of ₱725 Million is approximately 25% of
the basis above calculated. It is a balancing of the constitutional
obligation of the state to afford protection to labor which, specific to this
case, is assurance that in case of affirmance of the award, recovery is
not negated; and on the other end of the spectrum, the opportunity of
the employer to appeal.

By reducing the amount of the appeal bond in this case, the


employees would still be assured of at least substantial compensation
in case a judgment award is affirmed. On the other hand, management
will not be effectively denied of its statutory privilege of appeal.

In line with Sara Lee and the objective that the appeal on the
merits to be threshed out soonest by the NLRC, the Court holds that
the appeal bond posted by the respondent in the amount of
₱100,000.00 which is equivalent to around 20% of the total amount of
monetary bond is sufficient to perfect an appeal.

With the employer's demonstrated good faith in filing the motion


to reduce the bond on demonstrable grounds coupled with the posting
of the appeal bond in the requested amount, as well as the filing of the
memorandum of appeal, the right of the employer to appeal must be
upheld. This is in recognition of the importance of the remedy of
appeal, which is an essential part of our judicial system and the need
to ensure that every party litigant is given the amplest opportunity for
the proper and just disposition of his cause freed from the constraints
of technicalities.

219
Page 220

Case Digests by: Alecx Frances S. Daquioag

TURKS SHAWARMA COMPANY AND ZENAROS V.


PAJARON
G.R. No. 207156. January 16, 2017.
Del Castillo, C. J.

DOCTRINE:
ARTICLE 229 (FORMERLY ARTICLE 223)
The reduction of the appeal bond is allowed, subject to the
following conditions: (1) the motion to reduce the bond shall be based
on meritorious grounds; and (2) a reasonable amount in relation to the
monetary award is posted by the appellant. Compliance with these two
conditions will stop the running of the period to perfect an appeal.

FACTS:
Petitioners hired Feliciano Pajaron and Larry Carbonilla in 2007
as service crew and head crew, respectively. In April 2015, both would
file complaints their respective complaints against their employers for
constructive and actual illegal dismissal, non-payment of overtime pay,
holiday pay, holiday premium, rest day premium, service incentive
leave pay and 13th month pay against petitioners. Both Complaints
were consolidated.

Pajaron alleged that Zeñarosa asked him to sign a piece of paper


stating that he was receiving the correct amount of wages and that he
had no claim against petitioners. He refused to sign the same and that
that caused his dismissal. As for Carbonilla he had an altercation with
his supervisor, causing his immediate termination. He was also asked
to sign a piece of paper acknowledging a debt of 7000php.

Both Pajaron and Carbonilla claim that there was no just or


authorized cause for their dismissal. Petitioners on the other hand
claim that both abandoned their work, with Pajaron being a habitual
absentee for extended periods and that he was simply rehired on the
spot; while Carbonilla habitually misbehaved and left employment
without settling an unpaid obligation of 78,900php, prompting the filing
of the charge of estafa against him.

ISSUE:
WON the motion to reduce bond should be granted

RULING:
No. The Supreme Court cited Secs. 4 and 6 of the NLRC Rules
of Procedure, with the latter providing:

220
Page 221

No motion to reduce bond shall be entertained except on


meritorious grounds, and upon the posting of a bond in a reasonable
amount. The mere filing of a motion to reduce bond without complying
with the requisites in the preceding paragraphs shall not stop the
running of the period to perfect an appeal.

"It is clear from both the Labor Code and the NLRC Rules of
Procedure that there is legislative and administrative intent to strictly
apply the appeal bond requirement, and the Court should give utmost
regard to this intention."

The posting of cash or surety bond is therefore mandatory and


jurisdictional; failure to comply with this requirement renders the
decision of the Labor Arbiter final and executory. The reduction of the
appeal bond is allowed, subject to the following conditions: (1) the
motion to reduce the bond shall be based on meritorious grounds; and
(2) a reasonable amount in relation to the monetary award is posted
by the appellant. Compliance with these two conditions will stop the
running of the period to perfect an appeal.

The NLRC correctly held that the supposed ground cited in the
motion is not well-taken for there was no evidence to prove Zen arosa's
claim that the payment of the full amount of the award would greatly
affect his business due to financial setbacks.

221
Page 222

Case Digests by: Alecx Frances S. Daquioag

PIONEER TEXTURIZING CORP V. NLRC


G.R. No. 118651. October 16, 1997.
Francisco, C. J.

DOCTRINE:
REINSTATEMENT ASPECT OF LA’S DECISION
To require the application for and issuance of a writ of execution
as prerequisites for the execution of a reinstatement award would
certainly betray and run counter to the very object and intent of Article
223, i.e., the immediate execution of a reinstatement order.

FACTS:
Private respondent Lourdes A. de Jesus is petitioners'
reviser/trimmer since 1980. As reviser/trimmer, de Jesus based her
assigned work on a paper note posted by petitioners. The posted paper
which contains the corresponding price for the work to be
accomplished by a worker is identified by its P.O. Number.

On August 15, 1992, de Jesus worked on P.O. No. 3853 by


trimming the cloths' ribs. She thereafter submitted tickets
corresponding to the work done to her supervisor. Three days later, de
Jesus received from petitioners' personnel manager a memorandum
requiring her to explain why no disciplinary action should be taken
against her for dishonesty and tampering of official records and
documents with the intention of cheating as P.O. No. 3853 allegedly
required no trimming. The memorandum also placed her under
preventive suspension for thirty days starting from August 19, 1992. In
her handwritten explanation, de Jesus maintained that she merely
committed a mistake in trimming P.O. No. 3853 as it has the same
style and design as P.O. No. 3824 which has an attached price list for
trimming the ribs and admitted that she may have been negligent in
presuming that the same work was to be done with P.O. No. 3853, but
not for dishonesty or tampering. Petitioners' personnel department,
nonetheless, terminated her from employment and sent her a notice of
termination dated September 18, 1992.

On September 22, 1992, de Jesus filed a complaint for illegal


dismissal against petitioners.

ISSUE:
WON de Jesus was illegally dismissed
WON an order for reinstatement needs a writ of execution

222
Page 223

RULING:
1. Yes. Petitioners simply failed, both before the Labor Arbiter and
the NLRC, to discharge the burden of proof and to validly justify
De Jesus' dismissal from service. Lack of a just cause in the
dismissal from service of an employee, as in this case, renders
the dismissal illegal, despite the employer's observance of
procedural due process. The Court also found the imposition of
the extreme penalty of dismissal against de Jesus as harsh and
grossly disproportionate to the negligence committed.

2. No. We note that prior to the enactment of R.A. No. 6715, Article
223 of the Labor Code contains no provision dealing with the
reinstatement of an illegally dismissed employee. A closer
examination, however, shows that the necessity for a writ of
execution under Article 224 applies only to nal and executory
decisions which are not within the coverage of Article 223. For
comparison, we quote the material portions of the subject
articles:

ART. 223. In any event, the decision of the Labor Arbiter


reinstating a dismissed or separated employee, insofar as the
reinstatement aspect is concerned, shall immediately be
executory, even pending appeal. The employee shall either be
admitted back to work under the same terms and conditions
prevailing prior to his dismissal or separation or, at the option of
the employer, merely reinstated in the payroll. The posting of a
bond by the employer shall not stay the execution for
reinstatement provided herein.

Article 224 states that the need for a writ of execution applies
only within five (5) years from the date a decision, an order or
award becomes final and executory. It can not relate to an award
or order of reinstatement still to be appealed or pending appeal
which Article 223 contemplates. The provision of Article 223 is
clear that an award for reinstatement shall be immediately
executory even pending appeal and the posting of a bond by the
employer shall not stay the execution for reinstatement. To
require the application for and issuance of a writ of execution as
prerequisites for the execution of a reinstatement award would
certainly betray and run counter to the very object and intent of
Article 223, i.e., the immediate execution of a reinstatement
order. The reason is simple. An application for a writ of execution
and its issuance could be delayed for numerous reasons. A mere
continuance or postponement of a scheduled hearing, for
instance, or an inaction on the part of the Labor Arbiter or the
NLRC could easily delay the issuance of the writ thereby setting
at naught the strict mandate and noble purpose envisioned by
Article 223.

223
Page 224

Case Digests by: Alecx Frances S. Daquioag

ROQUERO V. PHILIPPINES AIRLINES


G.R. No. 152329. April 22, 2003.
Puno, C. J.

DOCTRINE:
REINSTATEMENT ASPECT OF LA’S DECISION
The order of reinstatement is immediately executory. The
unjustified refusal of the employer to reinstate a dismissed employee
entitles him to payment of his salaries effective from the time the
employer failed to reinstate him despite the issuance of a writ of
execution. Unless there is a restraining order issued, it is ministerial
upon the Labor Arbiter to implement the order of reinstatement.

FACTS:
Alejandro Roquero, along with Rene Pabayo, were ground
equipment mechanics of respondent Philippine Airlines, Inc. (PAL for
brevity). From the evidence on record, it appears that Roquero and
Pabayo were caught red-handed possessing and using
Methampethamine Hydrochloride or shabu in a raid conducted by PAL
security officers and NARCOM personnel.

The two alleged that they did not voluntarily indulge in the said
act but were instigated by a certain Jojie Alipato who was then
introduced to them by Joseph Ocul, Manager of the Airport
Maintenance Division of PAL.

Pabayo alleged that Alipato often bragged about the drugs he


could smuggle inside the company premises and invited other
employees to take the prohibited drugs. Alipato was unsuccessful, until
one day, he was able to persuade Pabayo to join him in taking the
drugs. They met Roquero along the way and he agreed to join them.
Inside the company premises, they locked the door and Alipato lost no
time in preparing the drugs to be used. When they started the
procedure of taking the drugs, armed men entered the room, arrested
Roquero and Pabayo and seized the drugs and the paraphernalia
used. Roquero and Pabayo were subjected to a physical examination
where the results showed that they were positive of drugs. They were
also brought to the security office of PAL where they executed written
confessions without the benefit of counsel.

Roquero and Pabayo received a “notice of administrative


charge” for violating the PAL Code of Discipline. They were required to
answer the charges and were placed under preventive suspension.

224
Page 225

Roquero and Pabayo, in their reply, assailed their arrest and


asserted that they were instigated by PAL to take the drugs. They
argued that Alipato was not really a trainee of PAL but was placed in
the premises to instigate the commission of the crime. They based their
argument on the fact that Alipato was not arrested. Moreover, Alipato
has no record of employment with PAL.

Both were eventually dismissed by PAL. Thus, they filed a case


for illegal dismissal.

ISSUE:
WON the decision may be stayed by an appeal

RULING:
No. Article 223 (3rd paragraph) of the Labor Code, as amended,
provide that an order of reinstatement by the Labor Arbiter is
immediately executory even pending appeal.

The unjustified refusal of the employer to reinstate a dismissed


employee entitles him to payment of his salaries effective from the time
the employer failed to reinstate him despite the issuance of a writ of
execution. Unless there is a restraining order issued, it is ministerial
upon the Labor Arbiter to implement the order of reinstatement. In the
case at bar, no restraining order was granted. Thus, it was mandatory
on PAL to actually reinstate Roquero or reinstate him in the payroll.
Having failed to do so, PAL must pay Roquero the salary he is entitled
to, as if he was reinstated, from the time of the decision of the NLRC
until the finality of the decision of this Court.

The rule that technicalities have no room in labor cases where


the Rules of Court are applied only in a suppletory manner and only to
effectuate the objectives of the Labor Code and not to defeat them.
Hence, even if the order of reinstatement of the Labor Arbiter is
reversed on appeal, it is obligatory on the part of the employer to
reinstate and pay the wages of the dismissed employee during the
period of appeal until reversal by the higher court. On the other hand,
if the employee has been reinstated during the appeal period and such
reinstatement order is reversed with finality, the employee is not
required to reimburse whatever salary he received for he is entitled to
such, more so if he actually rendered services during the period.

The SC affirmed the dismissal of Roquero, but respondent PAL


is ordered to pay the wages to which Roquero is entitled from the time
the reinstatement order was issued until the finality of this decision.

225
Page 226

Case Digests by: Alecx Frances S. Daquioag

AIR PHILIPPINES CORP V. ENRICO ZAMORA


G.R. No. 148247. August 7, 2006.
Austria-Martinez, J.

DOCTRINE:
REINSTATEMENT ASPECT OF LA’S DECISION
Hence, even if the order of reinstatement of the Labor Arbiter is
reversed on appeal, it is obligatory on the part of the employer to
reinstate and pay the wages of the dismissed employee during the
period of appeal until reversal by the higher court. On the other hand,
if the employee has been reinstated during the appeal period and such
reinstatement order is reversed with finality, the employee is not
required to reimburse whatever salary he received for he is entitled to
such, more so if he actually rendered services during the period.

FACTS:
Enrico Zamora (Zamora) was employed with Air Philippines
Corporation (APC) as a Flight Deck Crew. He applied for promotion to
the position of airplane captain and underwent the requisite training
program. After completing training, he inquired about his promotion but
APC did not act on it; instead, it continued to give him assignments as
flight deck crew. Thus, Zamora filed a Complaint with the Labor Arbiter.
He argued that the act of APC of withholding his promotion rendered
his continued employment with it oppressive and unjust. He therefore
asked that APC be held liable for constructive dismissal.

APC denied that it dismissed complainant. It pointed out that,


when the complaint was filed on May 14, 1997, complainant was still
employed with it. It was only on May 22, 1997 that complainant stopped
reporting for work, not because he was forced to resign, but because
he had joined a rival airline, Grand Air.

ISSUE:
WON the order of reinstatement is immediately executory

RULING:
Yes. The NLRC did not commit grave abuse of discretion in
holding petitioner liable to respondent for P198,502.30. The premise of
the award of unpaid salary to respondent is that prior to the reversal by
the NLRC of the decision of the Labor Arbiter, the order of
reinstatement embodied therein was already the subject of an alias writ
of execution even pending appeal. Hence, even if the order of
reinstatement of the Labor Arbiter is reversed on appeal, it is obligatory
on the part of the employer to reinstate and pay the wages of the

226
Page 227

dismissed employee during the period of appeal until reversal by the


higher court. On the other hand, if the employee has been reinstated
during the appeal period and such reinstatement order is reversed with
finality, the employee is not required to reimburse whatever salary he
received for he is entitled to such, more so if he rendered services
during the period.

227
Page 228

Case Digests by: Alecx Frances S. Daquioag

LANSANGAN V. AMKOR TECHNOLOGY PHILIPPINES


G.R. No. 177026. January 30, 2009.
Carpio-Morales, J.

DOCTRINE:
REINSTATEMENT ASPECT OF LA’S DECISION
Roquero vs. PAL and Article 223 of LC do not apply where there
is no finding of illegal dismissal, as in the present case. Article 223
concerns itself with an interim relief, granted to a dismissed or
separated employee while the case for illegal dismissal is pending
appeal, as what happened in Roquero.

FACTS:
Petitioners Lansangan and Cendaña are supervisory employees
of respondent Amkor Technology Philippines. An anonymous e-mail
was sent to the respondent’s General Manager detailing allegations of
malfeasance on the part of petitioners for stealing company time. Upon
investigation, petitioners admitted in handwritten letters their
wrongdoing that they were swiping another employees ID card or
requesting another employee to swipe ones ID card to gain personal
advantage and/or in the interest of cheating. Respondent terminated
petitioners for extremely serious offenses as defined in its Code of
Discipline. Hence, the petitioners to file a complaint for illegal dismissal
with the LA.

ISSUE:
WON the interim relief under Article 223 applies despite the
absence of illegal dismissal

RULING:
No. Roquero, as well as Article 223 of the Labor Code which the
CA also relied, finds no application in the present case. Article 223
concerns itself with an interim relief, granted to a dismissed or
separated employee while the case for illegal dismissal is pending
appeal, as what happened in Roquero. It does not apply where there
is no finding of illegal dismissal, as in the present case.

Article 223 provides: “In any event, the decision of the Labor
Arbiter reinstating a dismissed or separated employee, insofar as the
reinstatement aspect is concerned, shall immediately be executory,
pending appeal. The employee shall either be admitted back to work
under the same terms and conditions prevailing prior to his dismissal

228
Page 229

or separation or, at the option of the employer, merely reinstated in the


payroll. The posting of a bond by the employer shall not stay the
execution for reinstatement provided herein.”

LA decision finding that the petitioners’ dismissal was valid


because they committed dishonesty as a form of serious misconduct
and fraud, or breach of trust had become final. Petitioners not having
appealed the same before the NLRC as in fact they even moved for
the execution of the reinstatement aspect of the decision. It bears
recalling that it was only respondent which assailed the LA decision to
the NLRC to solely question the propriety of the order for
reinstatement, and it succeeded.

Article 279 provides: “In cases of regular employment, the


employer shall not terminate the services of an employee except for a
just cause or when authorized by this Title. An employee who is
unjustly dismissed from work shall be entitled to reinstatement without
loss of seniority rights and other privileges and to his full backwages,
inclusive of allowances, and to his other benefits or their monetary
equivalent computed from the time his compensation was withheld
from him up to the time of his actual reinstatement.”

229
Page 230

Case Digests by: Alecx Frances S. Daquioag

MARILOU GENUINO V. NATIONAL LABOR RELATIONS


COMMISSION
G.R. No. 142732-33. December 04, 2007
Velasco, C. J.

DOCTRINE:
REINSTATEMENT ASPECT OF LA’S DECISION
If the decision of the labor arbiter is later reversed on appeal upon
the finding that the ground for dismissal is valid, then the employer has
the right to require the dismissed employee on payroll reinstatement to
refund the salaries s/he received while the case was pending appeal,
or it can be deducted from the accrued benefits that the dismissed
employee was entitled to receive from his/her employer under existing
laws, collective bargaining agreement provisions, and company
practices. However, if the employee was reinstated to work during the
pendency of the appeal, then the employee is entitled to the
compensation received for actual services rendered without need of
refund.

FACTS:
Citibank is an American banking corporation duly licensed to do
business in the Philippines. William Ferguson was the Manila Country
Corporate Officer and Business Head of the Global Finance Bank of
Citibank while Aziz Rajkotwala was the International Business
Manager for the Global Consumer Bank of Citibank.

Genuino was employed by Citibank as Treasury Sales Division


Head with the rank of Assistant Vice-President. Citibank sent Genuino
a letter charging her with knowledge and/or involvement in transactions
which were irregular or even fraudulent. In the same letter, Genuino
was informed she was under preventive suspension.

Genuino did not appear in the administrative investigation. Her


lawyers wrote a letter to Citibanks counsel asking what bank clients
funds were diverted from the bank and invested in other companies,
the specific amounts involved, the manner by which and the date when
such diversions were purportedly affected. In reply, Citibank gave
Genuino up to September 23, 1993 to submit her written explanation.
Genuino did not submit her written explanation.

On September 27, 1993, Citibank informed Genuino of the result


of their investigation. It found that Genuino with Santos used facilities
of Genuinos family corporation, namely, Global Pacific, personally and
actively participated in the diversion of bank clients funds to products

230
Page 231

of other companies that yielded interests higher than what Citibank


products offered, and that Genuino and Santos realized substantial
financial gains, all in violation of existing company policy and the
Corporation Code which carries a penal sanction.

Therefore, Genuinos employment was terminated by Citibank on


grounds of (1) serious misconduct, (2) willful breach of the trust
reposed upon her by the bank, and (3) commission of a crime against
the bank.

Genuino filed before the Labor Arbiter a Complaint illegal


suspension and illegal dismissal.

ISSUE:
WON the NLRC was correct in ruling ordering Citibank to pay the
salaries from the date of reinstatement up until the date of decision

RULING:
No, the Court hereby cancels the said award in view of its finding
that the dismissal of Genuino is for a legal and valid ground.

Ordinarily, the employer is required to reinstate the employee


during the pendency of the appeal pursuant to Art. 223, paragraph 3 of
the Labor Code, which states:
In any event, the decision of the Labor Arbiter reinstating a dismissed
or separated employee, insofar as the reinstatement aspect is
concerned, shall immediately be executory, even pending appeal. The
employee shall either be admitted back to work under the same terms
and conditions prevailing prior to his dismissal or separation or, at the
option of the employer, merely reinstated in the payroll. The posting of
a bond by the employer shall not stay the execution for reinstatement
provided herein.

If the decision of the labor arbiter is later reversed on appeal


upon the finding that the ground for dismissal is valid, then the
employer has the right to require the dismissed employee on payroll
reinstatement to refund the salaries s/he received while the case was
pending appeal, or it can be deducted from the accrued benefits that
the dismissed employee was entitled to receive from his/her employer
under existing laws, collective bargaining agreement provisions, and
company practices. However, if the employee was reinstated to work
during the pendency of the appeal, then the employee is entitled to the
compensation received for actual services rendered without need of
refund. Considering that Genuino was not reinstated to work or placed
on payroll reinstatement, and her dismissal is based on a just cause,
then she is not entitled to be paid the salaries stated in item no. 3 of
the fallo of the September 3, 1994 NLRC Decision.

231
Page 232

Case Digests by: Alecx Frances S. Daquioag

GARCIA V. PHILIPPINE AIRLINES


G.R. No. 164856. January 20, 2009.
Carpio Morales, C. J.

DOCTRINE:
REINSTATEMENT ASPECT OF LA’S DECISION
The obligation to pay the employee's salaries upon the employer's
failure to exercise the alternative options under Article 223 of the Labor
Code is not a hard and fast rule, considering the inherent constraints
of corporate rehabilitation.

FACTS:
The case stemmed from the administrative charge filed by PAL
against its employees-herein petitioners after they were allegedly
caught in the act of sniffing shabu when a team of company security
personnel and law enforcers raided the PAL Technical Center's
Toolroom Section.

After due notice, PAL dismissed petitioners for transgressing the


PAL Code of Discipline, prompting them to file a complaint for illegal
dismissal and damages. On Jan. 11, 1999, the Labor Arbiter ruled in
favor of petitioners, thus ordering PAL to, inter alia, immediately
comply with the reinstatement aspect of the decision.

Prior to the promulgation of the Labor Arbiter's decision, the SEC


placed PAL which was suffering from severe financial losses, under an
Interim Rehabilitation Receiver, who was subsequently replaced by a
Permanent Rehabilitation Receiver.

Subsequently or on October 5, 2000, the Labor Arbiter issued a


Writ of Execution (Writ) respecting the reinstatement aspect of his
January 11, 1999 Decision. PAL filed a Petition for Injunction before
the NLRC.

ISSUE:
WON the rule that employers are obligated to reinstate employees
under Art 223 is absolute

RULING:
No. Even if the order of reinstatement of the Labor Arbiter is
reversed on appeal, it is obligatory on the part of the employer to
reinstate and pay the wages of the dismissed employee during the
period of appeal until reversal by the higher court. It settles the view

232
Page 233

that the Labor Arbiter's order of reinstatement is immediately


executory, and the employer has to either re-admit them to work under
the same terms and conditions prevailing prior to their dismissal, or to
reinstate them in the payroll, and that failing to exercise the options in
the alternative, employer must pay the employee's salaries.

After the Labor Arbiter's decision is reversed by a higher tribunal,


the employee may be barred from collecting the accrued wages, if it is
shown that the delay in enforcing the reinstatement pending appeal
was without fault on the part of the employer.

The test is two-fold:


(1) there must be actual delay or the fact that the order of
reinstatement pending appeal was not executed prior to its
reversal; and
(2) the delay must not be due to the employer's unjustified act or
omission. If the delay is due to the employer's unjustified refusal,
the employer may still be required to pay the salaries
notwithstanding the reversal of the Labor Arbiter's decision.

While reinstatement pending appeal aims to avert the continuing


threat or danger to the survival or even the life of the dismissed
employee and his family, it does not contemplate the period when the
employer-corporation itself is similarly in a judicially monitored state of
being resuscitated in order to survive.

More importantly, there are legal effects arising from a judicial


order placing a corporation under rehabilitation. Respondent was,
during the period material to the case, effectively deprived of the
alternative choices under Article 223 of the Labor Code, not only by
virtue of the statutory injunction but also in view of the interim
relinquishment of management control to give way to the full exercise
of the powers of the rehabilitation receiver. Had there been no need to
rehabilitate, respondent may have opted for actual physical
reinstatement pending appeal to optimize the utilization of resources.
Then again, though the management may think this wise, the
rehabilitation receiver may decide otherwise, not to mention the
subsistence of the injunction on claims.

In sum, the obligation to pay the employee's salaries upon the


employer's failure to exercise the alternative options under Article 223
of the Labor Code is not a hard and fast rule, considering the inherent
constraints of corporate.

233
Page 234

Case Digests by: DE LOS REYES, JANA MARIE F.

MT. CARMEL COLLEGE v. RESUENA


G.R. No. 173076. October 10, 2007.
CHICO-NAZARIO, J.

DOCTRINE:
REINSTATEMENT ASPECT OF LA’S DECISION.
Reinstatement is immediately executory even pending appeal
only when the Labor Arbiter himself ordered the reinstatement.

FACTS:
Herein respondents participated in a protest action against
petitioner. A memorandum was issued directing respondents to explain
in writing why they should not be dismissed for loss of trust and
confidence for joining the protest action against the school
administration. After hearing, petitioner issued written notices of
termination to respondents.

Separate complaints were filed against petitioner for illegal


dismissal. The LA found for the petitioner ruling that the respondents
were legally dismissed. On appeal, the NLRC reversed the decision of
the LA finding respondents illegally dismissed. The CA affirmed the
NLRC decision. No MR was filed, thus, the decision became final and
executory.

ISSUE:
WON reinstatement is self-executory and does not need a writ of
execution for its enforcement.

RULING:
NO. Verily, Article 223 of the Labor Code is not applicable in the
instant case. The said provision stipulates that the decision of the
Labor Arbiter reinstating a dismissed or separated employee, insofar
as the reinstatement aspect is concerned, shall immediately be
executory, even pending appeal.

Art. 223 of the Labor Code provides that reinstatement is


immediately executory even pending appeal only when the Labor
Arbiter himself ordered the reinstatement. In this case, the original
Decision of Labor Arbiter Drilon did not order reinstatement.
Reinstatement in this case was actually ordered by the NLRC, affirmed
by the Court of Appeals.

234
Page 235

Considering that the order for reinstatement was first decided


upon appeal to the NLRC and affirmed with finality by the Court of
Appeals, petitioner rightly invoked Art. 224 of the Labor Code. As
contemplated by Article 224 of the Labor Code, the Secretary of Labor
and Employment or any Regional Director, the Commission or any
Labor Arbiter, or med-arbiter or voluntary arbitrator may, motu proprio
or on motion of any interested party, issue a writ of execution on a
judgment within five (5) years from the date it becomes final and
executory. Consequently, under Rule III of the NLRC Manual on the
Execution of Judgment, it is provided that if the execution be for the
reinstatement of any person to a position, an office or an employment,
such writ shall be served by the sheriff upon the losing party or upon
any other person required by law to obey the same, and such party or
person may be punished for contempt if he disobeys such decision or
order for reinstatement.

235
Page 236

Case Digests by: DE LOS REYES, JANA MARIE F.

BUENVIAJE v. CA
G.R. No. 147806. November 12, 2002.
PUNO, J.

DOCTRINE:
REINSTATEMENT ASPECT OF LA’S DECISION.
In any event, the decision of the Labor Arbiter reinstating a
dismissed or separated employee, insofar as the reinstatement aspect
is concerned, shall immediately be executory, even pending appeal.

FACTS:
Petitioners were former promo girls for garment products of
Cottonway. Allegedly, the company was suffering from business losses
which led to the termination of the petitioners’ employment. This led to
the filing of complaint for illegal dismissal, inter alia.

The LA found for Cottonway and ruled the petitioners’


retrenchment valid. The NLRC reversed the decision of the LA and
ordered the reinstatement of petitioners. The LA issued an order for
backwages but was set aside by the NLRC. The CA ruled that
reinstatement is no longer possible for petitioners’ deliberate refusal to
return to work

ISSUE:
WON the computation of petitioners’ backwages should be
computed from the time of their illegal dismissal until their actual
reinstatement.

RULING:
YES. In any event, the decision of the Labor Arbiter reinstating a
dismissed or separated employee, insofar as the reinstatement aspect
is concerned, shall immediately be executory, even pending appeal.
The employee shall either be admitted back to work under the same
terms and conditions prevailing prior to his dismissal or separation or,
at the option of the employer, merely reinstated in the payroll. The
posting of a bond by the employer shall not stay the execution for
reinstatement provided herein.

The foregoing provision is intended for the benefit of the employee


and cannot be used to defeat their own interest. The law mandates the
employer to either admit the dismissed employee back to work under
the same terms and conditions prevailing prior to his dismissal or to

236
Page 237

reinstate him in the payroll to abate further loss of income on the part
of the employee during the pendency of the appeal.

Petitioners are entitled to receive full backwages computed from


the time their compensation was actually withheld until their actual
reinstatement, or if reinstatement is no longer possible, until the finality
of the decision, in accordance with the Decision of the NLRC which
has attained finality.

237
Page 238

Case Digests by: DE LOS REYES, JANA MARIE F.

PFIZER, INC. v. VELASCO


G.R. No. 177467. March 9, 2011.
LEONARDO-DE CASTRO, J.

DOCTRINE:
REINSTATEMENT ASPECT OF LA’S DECISION.
Reinstatement pending appeal necessitates that it must be
immediately self-executory without need for a writ of execution during
the pendency of the appeal, if the law is to serve its noble purpose, and
any attempt on the part of the employer to evade or delay its execution
should not be allowed.

FACTS:
Velasco was a former professional health care representative of
Pfizer, Inc. Due to medical reasons, Velasco had to absent herself
frequently. While on leave, Pfizer dismissed her from employment.
This prompted Velasco to file a complaint for illegal dismissal.

The LA and NLRC rendered the dismissal illegal and ordered for
reinstatement with backwages. The CA upheld the validity of
respondents dismissal from employment but ordered Pfizer to pay
Velasco wages from the date of the Labor Arbiters decision ordering
her reinstatement until the CA rendered its decision declaring
Velasco’s dismissal valid

ISSUE:
WON the CA committed serious but reversible error when it
ordered Pfizer to pay Velasco wages from the decision of the LA
ordering reinstatement until the CA’s decision rendering dismissal valid

RULING:
NO. In the case at bar, Pfizer did not immediately admit
respondent back to work which, according to the law, should have
been done as soon as an order or award of reinstatement is handed
down by the Labor Arbiter without need for the issuance of a writ of
execution. Thus, respondent was entitled to the wages paid to her
under the aforementioned writ of execution. At most, Pfizer’s payment
of the same can only be deemed partial compliance/execution of the
Court of Appeals Resolution.

To reiterate, under Article 223 of the Labor Code, an employee


entitled to reinstatement "shall either be admitted back to work under

238
Page 239

the same terms and conditions prevailing prior to his dismissal or


separation or, at the option of the employer, merely reinstated in the
payroll."

It is established in jurisprudence that reinstatement means


restoration to a state or condition from which one had been removed
or separated. The person reinstated assumes the position he had
occupied prior to his dismissal. Reinstatement presupposes that the
previous position from which one had been removed still exists, or that
there is an unfilled position which is substantially equivalent or of
similar nature as the one previously occupied by the employee.

In sum, the Court reiterates the principle that reinstatement


pending appeal necessitates that it must be immediately self-executory
without need for a writ of execution during the pendency of the appeal,
if the law is to serve its noble purpose, and any attempt on the part of
the employer to evade or delay its execution should not be allowed.
Furthermore, we likewise restate our ruling that an order for
reinstatement entitles an employee to receive his accrued backwages
from the moment the reinstatement order was issued up to the date
when the same was reversed by a higher court without fear of
refunding what he had received.

239
Page 240

Case Digests by: DE LOS REYES, JANA MARIE F.

WENPHIL. v. VELASCO
G.R. No. 177467. March 9, 2011.
LEONARDO-DE CASTRO, J.

DOCTRINE:
REINSTATEMENT ASPECT OF LA’S DECISION.
It is established in jurisprudence that reinstatement means
restoration to a state or condition from which one had been removed
or separated. The person reinstated assumes the position he had
occupied prior to his dismissal. Reinstatement presupposes that the
previous position from which one had been removed still exists, or that
there is an unfilled position which is substantially equivalent or of
similar nature as the one previously occupied by the employee.

FACTS:
Velasco was a former professional health care representative of
Pfizer, Inc. Due to medical reasons, Velasco had to absent herself
frequently. While on leave, Pfizer dismissed her from employment.
This prompted Velasco to file a complaint for illegal dismissal.

The LA and NLRC rendered the dismissal illegal and ordered for
reinstatement with backwages. The CA upheld the validity of
respondents dismissal from employment but ordered Pfizer to pay
Velasco wages from the date of the Labor Arbiters decision ordering
her reinstatement until the CA rendered its decision declaring
Velasco’s dismissal valid

ISSUE:
WON the CA committed serious but reversible error when it
ordered Pfizer to pay Velasco wages from the decision of the LA
ordering reinstatement until the CA’s decision rendering dismissal valid

RULING:
NO. In the case at bar, Pfizer did not immediately admit
respondent back to work which, according to the law, should have
been done as soon as an order or award of reinstatement is handed
down by the Labor Arbiter without need for the issuance of a writ of
execution. Thus, respondent was entitled to the wages paid to her
under the aforementioned writ of execution. At most, Pfizer’s payment
of the same can only be deemed partial compliance/execution of the
Court of Appeals Resolution.

240
Page 241

To reiterate, under Article 223 of the Labor Code, an employee


entitled to reinstatement "shall either be admitted back to work under
the same terms and conditions prevailing prior to his dismissal or
separation or, at the option of the employer, merely reinstated in the
payroll."

It is established in jurisprudence that reinstatement means


restoration to a state or condition from which one had been removed
or separated. The person reinstated assumes the position he had
occupied prior to his dismissal. Reinstatement presupposes that the
previous position from which one had been removed still exists, or that
there is an unfilled position which is substantially equivalent or of
similar nature as the one previously occupied by the employee.

In sum, the Court reiterates the principle that reinstatement


pending appeal necessitates that it must be immediately self-executory
without need for a writ of execution during the pendency of the appeal,
if the law is to serve its noble purpose, and any attempt on the part of
the employer to evade or delay its execution should not be allowed.
Furthermore, we likewise restate our ruling that an order for
reinstatement entitles an employee to receive his accrued backwages
from the moment the reinstatement order was issued up to the date
when the same was reversed by a higher court without fear of
refunding what he had received.

241
Page 242

Case Digests by: DE LOS REYES, JANA MARIE F.

SMART COMMUNICATIONS v. SOLIDUM


G.R. No. 204646. April 15, 2015.
CARPIO, J.

DOCTRINE:
REINSTATEMENT ASPECT OF LA’S DECISION.
Employees are entitled to their accrued salaries, allowances,
benefits, incentives and bonuses until the NLRC’s reversal of the labor
arbiter’s order of reinstatement becomes final and executory, as shown
on the entry of judgment.

FACTS:
Jose Solidum was hired by SMART to head the Smart Buddy
Department. Some years thereafter, however, Solidum was placed
under preventive suspension. He was charged with serious
dishonesty. Solidum then filed a case for illegal dismissal against
Smart.

The LA ruled in favor of Solidum. On appeal, the same was


reversed. Solidum moved for the issuance of writ of execution and
aliases. It appears that Smart has yet to reinstate him. Meanwhile, the
NLRC rendered a decision reversing the LA. On the strength of this
reversal, the LA refused the issuance of the alias. Aggrieved, Solidum
filed an appeal before the NLRC. NLRC granted the same. Petitioner
then 65 to CA. Noting that Solidum used the wrong remedy to
challenge the denial of the writ, CA reversed the NLRC. On MR, CA
partially reversed itself and ordered the payment to Solidum.

ISSUE:
WON the respondent is entitled to the payment

RULING:
YES. The NLRC decision became final and executory as shown
on the entry of judgment. Thus, Solidum is entitled to P2,881,335.86,
representing his accrued salaries, allowances, benefits, incentives and
bonuses.

In Bago v. NLRC, the Court held that employees are entitled to


their accrued salaries, allowances, benefits, incentives and bonuses
until the NLRC’s reversal of the labor arbiter’s order of reinstatement
becomes final and executory, as shown on the entry of judgment.

242
Page 243

Case Digests by: DE LOS REYES, JANA MARIE F.

MANILA DOCTORS COLLEGE v. OLORES


G.R. No. 225044. October 3, 2016.
PERLAS-BERNARBE, J.

DOCTRINE:
REINSTATEMENT ASPECT OF LA’S DECISION.
Notwithstanding the reversal of the finding of illegal dismissal, an
employer, who, despite the LA's order of reinstatement, did not
reinstate the employee during the pendency of the appeal up to the
reversal by a higher tribunal may still be held liable for the accrued
wages of the employee, i.e., the unpaid salary accruing up to the time
of the reversal.

FACTS:
Olores was a faculty member of Manila Doctors College who was
dismissed for grave misconduct, gross inefficiency and incompetence,
after due investigation. He, thus, filed a case for illegal dismissal
among others.

LA ruled in favor of respondent and ordered petitioners to reinstate


Olores under the same terms and conditions of his employment.
However, instead of reinstating, Olores was given the option to receive
separation pay.

Petitioners appealed. The NLRC found Olores guilty of serious


misconduct. While the case is pending, Olores filed a motion for writ of
execution. The LA granted the motion.

ISSUE:
WON the Olores is entitled to the award of reinstatement
backwages.

RULING:
YES. Under Article 223 (now Article 229) of the Labor Code, "the
decision of the [LA] reinstating a dismissed or separated employee,
insofar as the reinstatement aspect is concerned, shall immediately be
executory, even pending appeal. The employee shall either be
admitted back to work under the same terms and conditions prevailing
prior to his dismissal or separation or, at the option of the employer,
merely reinstated in the payroll. The posting of a bond by the employer
shall not stay the execution for reinstatement.” Verily, the employer is

243
Page 244

duty-bound to reinstate the employee, failing which, the employer is


liable instead to pay the dismissed employee's salary.

However, in the event that the LA's decision is reversed by a


higher tribunal, the employer's duty to reinstate the dismissed
employee is effectively terminated. This means that an employer is no
longer obliged to keep the employee in the actual service or in the
payroll. The employee, in tum, is not required to return the wages that
he had received prior to the reversal of the LA's
decision. Notwithstanding the reversal of the finding of illegal
dismissal, an employer, who, despite the LA's order of reinstatement,
did not reinstate the employee during the pendency of the appeal up
to the reversal by a higher tribunal may still be held liable for the
accrued wages of the employee, i.e., the unpaid salary accruing up to
the time of the reversal. By way of exception, an employee may be
barred from collecting the accrued wages if shown that the delay in
enforcing the reinstatement pending appeal was without fault on the
part of the employer.

In this case, while respondent may have been given an alternative


option to instead receive separation pay, there is no denying that
based on the provisions of the Labor Code, it is his employer who
should have first discharged its duty to reinstate him.

244
Page 245

Case Digests by: DE LOS REYES, JANA MARIE F.

SY et. al., v. FAIRLAND KNITCRAFT CO.


G.R. No. 182915. December 12, 2011.
DEL CASTILLO, J.

DOCTRINE:
EXECUTION OF DECISIONS, ORDERS OR AWARDS.
For the purpose of computing the period for filing an appeal from
the NLRC to the CA, same shall be counted from receipt of the
decision, order or award by the counsel of record pursuant to the
established rule that notice to counsel is notice to party. And since the
period for filing of an appeal is reckoned from the counsel's receipt of
the decision, order or award, it necessarily follows that the reckoning
period for their finality is likewise the counsel's date of receipt thereof,
if a party is represented by counsel.

FACTS:
The case stemmed from filing of complaint by the petitioners for
underpayment and/or non-payment of wages among others against
Weesan. Later, Weesan filed a report on its temporary closure for a
period of not less than six months. Thus, petitioners filed an amended
complaint to include the charge of illegal dismissal.

The LA dismissed the complaint. However, upon appeal, the


NLRC ruled for the petitioners.

ISSUE:
WON the controverted decision may be enforced against Fairland
despite the fact the it was not furnished with a separate copy of the
NLRC Resolution denying the motion for reconsideration.

RULING:
YES.Article 224 contemplates the furnishing of copies of final
decisions, orders or awards both to the parties and their counsel in
connection with the execution of such final decisions, orders or awards.

However, for the purpose of computing the period for filing an


appeal from the NLRC to the CA, same shall be counted from receipt
of the decision, order or award by the counsel of record pursuant to the
established rule that notice to counsel is notice to party. And since the
period for filing of an appeal is reckoned from the counsel's receipt of
the decision, order or award, it necessarily follows that the reckoning
period for their finality is likewise the counsel's date of receipt thereof,

245
Page 246

if a party is represented by counsel. Hence, the date of receipt referred


to in Sec. 14, Rule VII of the then in force New Rules of Procedure of
the NLRC which provides that decisions, resolutions or orders of the
NLRC shall become executory after 10 calendar days from receipt of
the same, refers to the date of receipt by counsel.

Thus contrary to the CA's conclusion, the said NLRC Decision


became final, as to Fairland, 10 calendar days after Atty. Tecson's
receipt thereof. In sum, we hold that the Labor Arbiter had validly
acquired jurisdiction over Fairland and its manager, Debbie, through
the appearance of Atty. Geronimo as their counsel and likewise,
through the latter's filing of pleadings on their behalf.

246
Page 247

Case Digests by: DE LOS REYES, JANA MARIE F.

IBM NESTLE PHILIPPINES, INC. v. NESTLE PHILIPPINES


G.R. No. 198675. September 23, 2015.
PERALTA, J.

DOCTRINE:
EXECUTION OF DECISIONS, ORDERS OR AWARDS.
It is clear from the above law and rules that a judgment may be
executed on motion within five years from the date of its entry or from
the date it becomes final and executory. After the lapse of such time,
and before it is barred by the statute of limitations, a judgment may be
enforced by action. If the prevailing party fails to have the decision
enforced by a mere motion after the lapse of five years from the date
of its entry (or from the date it becomes final and executory), the said
judgment is reduced to a mere right of action in favor of the person
whom it favors and must be enforced, as are all ordinary actions, by
the institution of a complaint in a regular form.

FACTS:
Petitioner union staged a strike against respondent for the alleged
violation the CBA, dismissal of union officers and members and other
ULP. Respondent filed a petition to declare strike illegal. SOLE certified
the strike as illegal. After a series of conciliation meetings, a
compromise agreement was reached.

After more than 11 years from the time of execution of the MOA,
petitioners filed a motion of writ of execution contending that they have
not been paid the amounts they are entitled to in accordance with the
MOA. On the other hands, respondents claim that the petitioner’s
remedy is already barred by prescription.

ISSUE:
WON the petitioners’ demand to be paid has prescribed.

RULING:
YES. It is settled that when a compromise agreement is given
judicial approval, it becomes more than a contract binding upon the
parties. Having been sanctioned by the court, it is entered as a
determination of a controversy and has the force and effect of a
judgment. It is immediately executory and not appealable, except for
vices of consent or forgery. The non-fulfillment of its terms and
conditions justifies the issuance of a writ of execution; in such an
instance, execution becomes a ministerial duty of the court. Stated

247
Page 248

differently, a decision on a compromise agreement is final and


executory. Such agreement has the force of law and is conclusive
between the parties. It transcends its identity as a mere contract
binding only upon the parties thereto, as it becomes a judgment that is
subject to execution in accordance with the Rules.

It is clear from the above law and rules that a judgment may be
executed on motion within five years from the date of its entry or from
the date it becomes final and executory. After the lapse of such time,
and before it is barred by the statute of limitations, a judgment may be
enforced by action. If the prevailing party fails to have the decision
enforced by a mere motion after the lapse of five years from the date
of its entry (or from the date it becomes final and executory), the said
judgment is reduced to a mere right of action in favor of the person
whom it favors and must be enforced, as are all ordinary actions, by
the institution of a complaint in a regular form.

In the present case, the five-and ten-year periods provided by law


and the rules are more than sufficient to enable petitioners to enforce
their right under the subject MOA. In this case, it is clear that the
judgment of the NLRC, having been based on a compromise embodied
in a written contract, was immediately executory upon its issuance on
October 12, 1998. Thus, it could have been executed by motion within
five (5) years. It was not. Nonetheless, it could have been enforced by
an independent action within the next five (5) years, or within ten (10)
years from the time the NLRC Decision was promulgated. It was not.
Therefore, petitioners' right to have the NLRC judgment executed by
mere motion as well as their right of action to enforce the same
judgment had prescribed by the time they filed their Motion for Writ of
Execution on January 25, 2010.

248
Page 249

Case Digests by: DE LOS REYES, JANA MARIE F.

YUPANGCO COTTONMILLS v. CA.


G.R. No. 126322. January 16, 2002.
PARDO, J.

DOCTRINE:
THIRD-PARTY CLAIMS.
The third party may avail of alternative remedies cumulatively.
Thus, a party may avail the following: (1) file a third-party claim with
the sheriff of the LA, and (2) if the third-party claim is denied, the third
party may appeal the denial to the NLRC. Even if a third party was
denied, a third party still file a proper action with a competent court to
recover ownership of the property illegally seized by the sheriff.

FACTS:
Petitioner alleged that a sheriff of the NLRC erroneously and
unlawfully levied upon certain properties which it claims as its own. It
filed a notice of third-party claim with the LA. It also filed an affidavit of
adverse claim with the NLRC, which was dismissed by the LA. Its
appeal to the NLRC was dismissed. Because of this, it filed a complaint
for accion reinvidicatoria in the RTC, but it was also dismissed.

The CA dismissed the petition on the ground of forum shopping


and that petitioner’s remedy was to seek relief from the Supreme Court

ISSUE:
WON there was forum shopping; and WON the accion
reinvidicatoria should be dismissed

RULING:
NO. Anent the first issue, there is no forum shopping where two
different orders were questioned, two different causes of action and
issues were raised, and two objectives were sought.

In this case, there was no identity of parties, rights, causes of


action, and reliefs sought. The case before the NLRC where the LA
issued the writ of execution was a labor dispute between Artex and
Samar-Anglo. Petitioner was not a party.

On the other hand, accion reinvindicatoria filed by the petitioner in


the RTC was to recover the property illegally levied upon and sold at
auction. Hence, the causes of action in these cases were different.

249
Page 250

Anent the second issue, it should not be dismissed. A third party


whose property has been levied upon by a sheriff to enforce a decision
against judgment debtor is afforded with several alternative remedies
to protects its interests.

The third party may avail of alternative remedies cumulatively.


Thus, a party may avail the following: (1) file a third-party claim with
the sheriff of the LA, and (2) if the third-party claim is denied, the third
party may appeal the denial to the NLRC. Even if a third party was
denied, a third party still file a proper action with a competent court to
recover ownership of the property illegally seized by the sheriff. This
finds support in Section 16, Rule 39 of the ROC.

250
Page 251

Case Digests by: DE LOS REYES, JANA MARIE F.

ANDO v. CAMPO
G.R. No. 126322. January 16, 2002.
PARDO, J.

DOCTRINE:
EXECUTION OF DECISIONS, ORDERS OR AWARDS.
The power of the NLRC, or the courts, to execute its judgment
extends only to properties unquestionably belonging to the judgment
debtor alone. A sheriff, therefore, has no authority to attach the
property of any person except that of the judgment debtor. Likewise,
there is no showing that the sheriff ever tried to execute on the
properties of the corporation.

FACTS:
Petitioner was the president of Premier Allied and Contracting
Services, Inc. (PACSI), an independent labor contractor. Respondents
were dismissed from employment. Thus, they filed a case for illegal
dismissal and money claims with the NLRC.

The LA ruled in favor of respondents. The NLRC ruled that


petitioners failed to perfect his appeal for failing to pay the supersedeas
bond. It also affirmed the LA’s decision with modification. Upon finality,
respondents moved for its execution. To answer for the monetary
award, the NLRC acting sheriff issued a notice of sale on execution of
personal property over the property in the name of Paquito Ando
married to Erlinda Ando.

This prompted petitioner to file an action for prohibition and


damages with prayer for the issuance of a TRO before the RTC.

ISSUE:
WON petitioner’s wife can file a third-party claim

RULING:
YES. The RTC was correct. The proper remedy is a third-party
claim before the NLRC. The power of the NLRC, or the courts, to
execute its judgment extends only to properties unquestionably
belonging to the judgment debtor alone. A sheriff, therefore, has no
authority to attach the property of any person except that of the
judgment debtor. Likewise, there is no showing that the sheriff ever
tried to execute on the properties of the corporation.

251
Page 252

There is no doubt that petitioner’s complaint is a third-party claim


within the cognizance of the NLRC. Petitioner may indeed be
considered a “third party” in relation to the property subject of the
execution vis-à-vis the LA’s decision. The property belongs to the
conjugal partnership of the petitioner and his wife, and not the
corporation.

A property which belongs to the conjugal partnership and not the


corporation belongs to a third party. At the very least, the Court can
consider Paquito’s wife as a third party. Hence, the correct remedy is
a third-party claim. Further, Article 266 (254) prohibits the issuance of
a permanent or temporary injunction in any case growing out of labor
dispute.

252
Page 253

Case Digests by: DE LOS REYES, JANA MARIE F.

PAL v. BICHARA
G.R. No. 213729. September 2, 2015.
PERLAS-BERNARBE, J.

DOCTRINE:
EXECUTION OF DECISIONS, ORDERS OR AWARDS.
The principle of immutability of judgments, from which the above-
stated rule on writ of executions proceed, allow courts, as an
exception, to recognize circumstances that transpire after the finality of
the decision which would render its execution unjust and inequitable
and act accordingly. Thus, in view of the supervening events above-
mentioned, this Court deems the award of salary differential to be the
just and equitable award under the circumstances herein prevailing.

FACTS:
PAL hired Bichara as a flight attendant. After 3 years, PAL
implemented a retrenchment program wherein Bichara voluntarily
resigned. After 4 years, he was rehired.

Eighteen years after, Bichara was included in PAL’s purser


program. As a flight purser, he was required to comply with its
qualifications but failed. Consequently, he was demoted to the position
of flight steward.

Bichara appealed his demotion to PAL, but to no avail. Hence, he


filed a complaint for illegal demotion against PAL. The LA declared the
demotion as illegal and ordered PAL to reinstate Bichara to his position
as flight purser. PAL appealed to the NLRC wherein it affirmed the LA’s
finding. The decision became final and executory.

ISSUE:
WON Bichara is entitled to the awards of monetary benefits.

RULING:
YES. Bichara is entitled to the salary differential of a flight purser
from a flight attendant from the time of his illegal demotion up until the
time he was reinstated.

PAL's supervening retrenchment of its employees, which included


Bichara, in July 1998, and his compulsory retirement in July 2005,
however, prevent the enforcement of the reinstatement of Bichara to
the position of flight purser under the June 16, 1997 Decision.

253
Page 254

Nonetheless, since this Decision had already settled the illegality of


Bichara's demotion with finality, this Court finds that Bichara should,
instead, be awarded the salary differential of a flight purser from a
flight steward from the time of his illegal demotion on March 21, 1994
up until the time he was retrenched in July 1998. Notably, unlike LA
Macam's award of separation pay in lieu of reinstatement, the award
of salary differential is not dependent on the validity of his termination,
as it is, in fact, intrinsically linked to the illegality of Bichara's demotion.
Hence, with this direct relation, there should be no obstacle in
rendering this award.

Further, it should be pointed out that the principle of immutability


of judgments, from which the above-stated rule on writ of executions
proceed, allow courts, as an exception, to recognize circumstances
that transpire after the finality of the decision which would render its
execution unjust and inequitable and act accordingly. Thus, in view of
the supervening events above-mentioned, this Court deems the award
of salary differential to be the just and equitable award under the
circumstances herein prevailing. Jurisprudence holds that courts may
modify or alter the judgment to harmonize the same with justice and
the facts when after judgment has been rendered and the latter has
become final, facts and circumstances transpire which render its
execution impossible or unjust, as in this case.

A judgment should be implemented according to the terms of its


dispositive portion is a long and established rule. As such, where the
writ of execution is not in harmony with and exceeds the judgment
which gives it life, the writ has pro tanto no validity.

A companion to this rule is the principle of immutability of final


judgment, subject to exceptions: (1) correction of clerical errors; (2) so-
called nunc pro tunc entries which cause prejudice to any party; (3)
void judgment; and (4) whenever circumstances transpire after the
finality of the decision rendering its execution unjust and inequitable.

254
Page 255

Case Digests by: DE LOS REYES, JANA MARIE F.

GUILLERMO v. USON
G.R. No. 213729. March 7, 2016.
PERLAS-BERNARBE, J.

DOCTRINE:
PIERCING THE VEIL OF CORPORATE FICTION.
The veil of corporate fiction can be pierced, and responsible
corporate directors and officers or even a separate but related
corporation may be impleaded and held answerable solidarily in a labor
case, even after final judgment and on execution, so long as it is
established that such persons have deliberately used the corporate
vehicle to unjustly evade the judgment obligation, or have resorted to
fraud, bad faith or malice in doing so.

FACTS:
Uson began his employment with Royal Class Venure Phils., Inc.
(Royal Class Venture) ac an accounting clerk. Eventually. he was
promoted as an accounting supervisor until he was allegedly dismissed
from employment. He foled a complaint with the LA. Royal Class
Venture did not make an appearance in the case despite its receipt of
summons.

The LA rendered a decision in favor of Uson ordering Royal class


venture for restatement and payment of the former’s backwages. No
appeal was made. Two writs of execution were filed by Uson but
remained unsatisfied. He filed another writ of execution and motion to
hold directors and officers liable for satisfaction of decision. The LA
held the officers jointly and severally liable for obligations.

Guillermo, owner, filed, by way of special appearance, an MR


contending that the decision sought to be enforced is already final and
executory. Hence, it is immutable and unalterable.

ISSUE:
WON the LA was correct in piercing the corporate veil even if the
decision has already become final and executory

RULING:
YES. The veil of corporate fiction can be pierced, and responsible
corporate directors and officers or even a separate but related
corporation may be impleaded and held answerable solidarily in a labor
case, even after final judgment and on execution, so long as it is

255
Page 256

established that such persons have deliberately used the corporate


vehicle to unjustly evade the judgment obligation, or have resorted to
fraud, bad faith or malice in doing so. The key element is the presence
of fraud, malice or bad faith.

The records bear that Guillermo dissolved Royal Class Venture


and helped incorporate a new firm, located in the same aches as the
former, where he is again a stockholder. In the case at hand,
respondent Uson’s sworn allegations stating that Guillermo was the
responsible officer in charge of running the company as well as the one
who maliciously and illegally dismissed Uson from employment was
uncontroverted. Furthermore, it was Guillermo himself, as President
and General Manager of the company, who received the summons to
the case, and who also subsequently and without justifiable cause
refused to receive all notices and orders of the Labor Arbiter that
followed. He, likewise, was shown to have a role in dissolving the
original obligor company in an obvious "scheme to avoid liability".

256
Page 257

Case Digests by: DE LOS REYES, JANA MARIE F.

DUTCH MOVERS INC. v. LEQUIN


G.R. No. 210032. April 25, 2017.
DEL CASTILLO, J.

DOCTRINE:
PIERCING THE VEIL OF CORPORATE FICTION
The act of hiding behind the cloak of corporate fiction will not be
allowed in such situation where it is used to evade one's obligations,
which "equitable.”

FACTS:
The case stemmed from the filing of a complaint by respondents
against petitioners for illegal dismissal. DMI informed respondents that
it would cease its hauling operation for no reason. Records reveal that
DMI did not file any notice of business closure.

The LA dismissed the case for lack of cause of action. However,


the NLRC reversed the earlier decision and ruled that respondents
were illegally dismissed.

ISSUE:
WON there is legal basis to pierce the veil of corporate fiction of
DMI

RULING:
YES. In considering the foregoing events, the Court is not
unmindful of the basic tenet that a corporation has a separate and
distinct personality from its stockholders, and from other corporations
it may be connected with. However, such personality may be
disregarded, or the veil of corporate fiction may be pierced attaching
personal liability against responsible person if the corporation's
personality "is used to defeat public convenience, justify wrong, protect
fraud or defend crime, or is used as a device to defeat the labor laws."

Here, the veil of corporate fiction must be pierced and accordingly,


petitioners should be held personally liable for judgment awards
because the peculiarity of the situation shows that they controlled DMI;
they actively participated in its operation such that DMI existed not as
a separate entity but only as business conduit of petitioners. As will be
shown be shown below, petitioners controlled DMI by making it appear
to have no mind of its own, and used DMI as shield in evading legal

257
Page 258

liabilities, including payment of the judgment awards in favor of


respondents.

While it is true that one's control does not by itself result in the
disregard of corporate fiction; however, considering the irregularity in
the incorporation of DMI, then there is sufficient basis to hold that such
corporation was used for an illegal purpose, including evasion of legal
duties to its employees, and as such, the piercing of the corporate veil
is warranted. The act of hiding behind the cloak of corporate fiction will
not be allowed in such situation where it is used to evade one's
obligations, which "equitable piercing doctrine was formulated to
address and prevent.”

258
Page 259

Case Digests by: Duterte, Camille Anne E.

GENUINO AGRO v. ROMANO


G.R. No. 204782, September 18, 2019
Reyes, J. Jr., J.

DOCTRINE:
PIERCING THE VEIL OF CORPORATE FICTION
A corporation is an artificial being invested by law with a
personality separate and distinct from its stockholders and from other
corporations to which it may be connected. However, the corporate
mask may be lifted and the corporate veil may be pierced when a
corporation is just but the alter ego of a person or of another
corporation. Moreover, piercing the corporate veil may also be resorted
to by the courts or quasi-judicial bodies when the separate personality
of a corporation is used as a means to perpetrate fraud or an illegal
act, or as a vehicle for the evasion of an existing obligation, the
circumvention of statutes, or to confuse legitimate issues.

FACTS:
Armando G. Romano, et. al (respondents) claimed that they
worked as brine men at Genuino Ice Company Inc.'s (petitioner herein)
ice plant. Respondents averred that sometime in September 2004, the
workers were given a work schedule where each worker does not work
for 15 days for a period of 90 days. When respondents reported back
to work after their 15 days forced leave, they were told that their
employment was already terminated. Thus, respondents filed a
complaint for illegal dismissal with prayer for separation pay against
Genuino Ice and Vicar before the DOLE. Genuino Ice, for its part,
claimed that respondents charged the wrong party as they were never
its employees but of petitioner, Genuino Agro, its affiliate company. By
reason of Genuino Ice's contention that respondents charged the
wrong party, they amended their complaint by impleading the
petitioner.
Respondents filed a complaint for illegal dismissal. LA held that
respondents were regular employees illegally dismissed. The NLRC
affirmed the decision. CA dismissed the petition for certiorari filed by
petitioner. Respondents pray that Genuino Ice be declared solidarily
liable with the petitioner to pay respondents the monetary awards
granted to them by the LA.

ISSUE:
WON Genuino Ice, as an affiliate company of Genuino Agro, shall
be held solidarily liable with the latter for its liabilities to its employees

RULING:

259
Page 260

Yes. It is an elementary and fundamental principle of corporation


law that a corporation is an artificial being invested by law with a
personality separate and distinct from its stockholders and from other
corporations to which it may be connected. However, the corporate
mask may be lifted and the corporate veil may be pierced when a
corporation is just but the alter ego of a person or of another
corporation. Moreover, piercing the corporate veil may also be
resorted to by the courts or quasi-judicial bodies when "[the separate
personality of a corporation] is used as a means to perpetrate
fraud or an illegal act, or as a vehicle for the evasion of an existing
obligation, the circumvention of statutes, or to confuse legitimate
issues." Furthermore, the veil of corporate fiction may also be pierced
as when the same is made as a shield to confuse legitimate issue.
Once the veil of corporate fiction is pierced, the separate but
related corporation becomes solidarily liable in labor cases.

A deep scrutiny of the circumstances necessitates the


application of the doctrine of piercing the veil of corporate fiction. The
circumstances indubitably establish that both Genuino Ice and the
petitioner are using their respective distinct corporate
personalities in bad faith and to confuse legitimate issues in the
hope of evading its obligation to the respondents. x x x The
circumstances show that both Genuino Ice and the petitioner have
taken turns in representing each other's common cause and in
pursuing remedies to protect its common interest in repelling the
respondents' monetary claims. Whenever a claim is directed against
one of them, the other admits the monetary liability so that the former
may be shielded and vice versa. X x x

As observed, when an "affiliate company" takes the cudgels


for another, it means that both have a common interest. If indeed
there was no commonality or intertwining of an interest in frustrating
the respondents' monetary claims, the petitioner and not Genuino Ice
would have posted a bond for its own appeal. X x x for purposes of this
litigation and for the satisfaction of the respondents' monetary claims,
both Genuino Ice and the petitioner shall be treated as one and the
same entity, and held liable solidarily for the same.

260
Page 261

Case Digests by: Duterte, Camille Anne E.

CARAG v. NLRC
G.R. No. 147590, April 2, 2007
Carpio, J.:

DOCTRINE:
ARTICLE 230
A director is not personally liable for the debts of a corporation,
which has a separate legal personality of its own. To hold a director
personally liable for debts of the corporation, and thus pierce the veil
of corporate fiction, the bad faith or wrongdoing of the director must be
established clearly and convincingly.
For a wrongdoing to make a director personally liable for debts
of the corporation, the wrongdoing approved or assented to by the
director must be a patently unlawful act. Mere failure to comply with
the notice requirement of labor laws on company closure or dismissal
of employees does not amount to a patently unlawful act.
FACTS:
National Federation of Labor Unions (NAFLU) and Mariveles
Apparel Corporation Labor Union (MACLU) (respondent
complainants), on behalf of all of MAC's rank and file employees, filed
a complaint against Mariveles Apparel Corporation (MAC) for illegal
dismissal brought about by its illegal closure of business. They sought
to implead Antonio C. Carag (petitioner) in his official capacity as
Chairman of the Board as party respondents in order to guarantee
satisfaction of any judgment award in favour of the illegally dismissed
employees. Carag claims that he was not afforded due process
because he was not summoned to the mandatory conference as
required by the NLRC rules. LA ruled in favour of the Union; Carag was
ordered to pay more than P50 million to the complainants. the CA also
ruled that Carag was solidarily liable with MAC.

ISSUE:
WON Carag, in his official capacity as Chairman of the Board, can
be held solidarily liable with the corporation despite failure to be
summoned during the mandatory conference.

RULING:
No. Carag’s right to due process was violated. It is clear from the
narration in Arbiter Ortiguerra's Decision that she only summoned
complainants and MAC, and not Carag, to a conference for possible
settlement. X X X at the time of the conference, Carag was not yet
a party to the case. Thus, Arbiter Ortiguerra could not have
possibly summoned Carag to the conference.
Carag may not be held solidarily liable with the Corporation for
the illegal dismissal of MAC’s employees. The rule is that a director is

261
Page 262

not personally liable for the debts of a corporation, which has a


separate legal personality of its own. To hold a director personally
liable for debts of the corporation, and thus pierce the veil of
corporate fiction, the bad faith or wrongdoing of the director must
be established clearly and convincingly. Bad faith is never
presumed. Bad faith does not connote bad judgment or negligence.
Bad faith imports a dishonest purpose. Bad faith means breach of a
known duty through some ill motive or interest. Bad faith partakes of
the nature of fraud. Neither does bad faith arise automatically just
because a corporation fails to comply with the notice requirement
of labor laws on company closure or dismissal of employees. The
failure to give notice is not an unlawful act because the law does not
define such failure as unlawful. Such failure to give notice is a
violation of procedural due process but does not amount to an
unlawful or criminal act. Such procedural defect is called illegal
dismissal because it fails to comply with mandatory procedural
requirements, but it is not illegal in the sense that it constitutes an
unlawful or criminal act.
For a wrongdoing to make a director personally liable for
debts of the corporation, the wrongdoing approved or assented
to by the director must be a patently unlawful act. Mere failure to
comply with the notice requirement of labor laws on company
closure or dismissal of employees does not amount to a patently
unlawful act. Patently unlawful acts are those declared unlawful by
law which imposes penalties for commission of such unlawful acts.
There must be a law declaring the act unlawful and penalizing the act.
In this case, Article 283 of the Labor Code, requiring a one-
month prior notice to employees and the DOLE before any
permanent closure of a company, does not state that
noncompliance with the notice is an unlawful act punishable
under the Code. There is no provision in any other Article of the Labor
Code declaring failure to give such notice an unlawful act and providing
for its penalty. Complainants did not allege or prove, and Arbiter
Ortiguerra did not make any finding, that Carag approved or assented
to any patently unlawful act to which the law attaches a penalty for its
commission. On this score alone, Carag cannot be held personally
liable for the separation pay of complainants.

262
Page 263

Case Digests by: Duterte, Camille Anne E.

ROCA v. DABUYAN
G.R. No. 215281, March 15, 2018
Del Castillo, J.

DOCTRINE:
ARTICLE 230
The contract of employment between respondents, on the one
hand, and Oceanic and Ewayan on the other, is effective only between
them; it does not extend to petitioner, who is not a party thereto. His
only role is as lessor of the premises which Oceanic leased to operate
as a hotel; he cannot be deemed as respondent's employer - not even
under the pretext that he took over as the "new management" of the
hotel operated by Oceanic.
FACTS:
Dabuyan et al. (respondents) filed a complaint for illegal
dismissal against RAF Mansion Hotel Old Management and New
Management and Victoriano Ewayan, and later amended their
complaint and included Rolando De Roca (petitioner) as co-
respondent.
LA directed De Roca to pay backwages and other monetary
award to respondents. De Roca filed a Motion to Dismiss before the
LA on the ground of lack of jurisdiction but was denied for being filed
beyond the period allowed, as was the Motion to implead Oceanic
Travel and Tours Agency as additional respondent. He instituted the
petition for annulment of LA judgment, which the NLRC dismissed for
being filed beyond the 10-day reglementary period prescribed. He filed
a Petition for Certiorari before the CA, which was dismissed since he
filed it 31 days after such receipt.
He argues that LA’s decision is null and void as there was no
determination of facts and evidence relative to his supposed liability to
respondents since he was not at any time their employer, but merely
the owner-lessor of the premises and it was Ewayan and his Oceanic
Travel and Tours Agency who operated the RAF Mansion Hotel where
respondents were employed as hotel staff; that the NLRC did not
acquire jurisdiction over him since the element of employer-employee
relationship was lacking, and that he was impleaded in the case only
because respondents could no longer trace the whereabouts of their
true employer, Ewayan, who appears to have absconded.

ISSUE:
WON De Roca, as mere owner-lessor of the premises, where an
employer found liable for illegal dismissal operates, can be held liable
for money claims of the latter’s employees

RULING:

263
Page 264

No. Petitioner's building was an existing hotel called the "RAF


Mansion Hotel", which Oceanic agreed to continue to operate under
the same name. There is no connection between petitioner and
Oceanic oilier than through the lease agreement executed by
them; they are not partners in the operation of RAF Mansion
Hotel. It just so happens that Oceanic decided to continue operating
the hotel using the original name - "RAF Mansion Hotel".
It must be noted as well that when they originally filed the labor
case, respondents did not include petitioner as respondent therein. It
was only later on that they moved to amend their complaint, impleading
petitioner x x x.
The contract of employment between respondents, on the
one hand, and Oceanic and Ewayan on the other, is effective only
between them; it does not extend to petitioner, who is not a party
thereto. His only role is as lessor of the premises which Oceanic
leased to operate as a hotel; he cannot be deemed as
respondent's employer - not even under the pretext that he took
over as the "new management" of the hotel operated by Oceanic.
X x x To allow respondents to recover their monetary claims from
petitioner would necessarily result in their unjust enrichment.
The decision of the Labor Arbiter must be set aside for being
grossly erroneous and unjust. At worst, it is null and void, and, as
petitioner correctly put it, it is a "lawless thing, which can be treated as
an outlaw and slain at sight, or ignored wherever it exhibits its head."
Being of such nature, it could not have acquired finality, contrary to
what respondents believe - as it "creates no rights and imposes no
duties. Any act performed pursuant to it and any claim emanating from
it have no legal effect."

264
Page 265

Case Digests by: Duterte, Camille Anne E.

FERNANDEZ ET. AL. v. NEWFIELD STAFF SOLUTIONS, INC.


G.R. No. 201979, July 10, 2013
Villarama, Jr., J.

DOCTRINE:
ARTICLE 230
There is solidary liability when the obligation expressly so states,
when the law so provides, or when the nature of the obligation so
requires
A corporation, being a juridical entity, may act only through its
directors, officers and employees. Obligations incurred by them, acting
as such corporate agents, are not theirs but the direct accountabilities
of the corporation they represent. True, solidary liability may at times
be incurred but only when exceptional circumstances warrant such as,
generally, in the following cases:
1. When directors and trustees or, in appropriate cases, the
officers of a corporation -
(a) vote for or assent to patently unlawful acts of the corporation;
(b) act in bad faith or with gross negligence in directing the
corporate affairs;
Corporate directors and officers are solidarily liable with the
corporation for the termination of employment of employees done with
malice or in bad faith.
FACTS:
Newfield Staff Solutions, Inc. hired Gilda A. Fernandez
(petitioner) as Recruitment Manager starting September 30, 2008, as
well are Bernadette A. Beltran as probationary Recruitment Specialist.
On October 17, 2008, Arnold "Jay" Lopez, Jr., Newfield’s General
Manager (respondent), terminated their employment on the ground
that they failed to perform satisfactorily. Lopez, Jr. ordered them to
immediately turn over the records in their possession to their
successors. A week later, petitioners received Lopez, Jr.’s return-to-
work letters which stated that they did not report since October 20,
2008 without resigning, in violation of their employment agreements.
Fernandez and Beltran countered with demand letters.
They filed a complaint for illegal dismissal, non-payment of salary
and overtime pay, reimbursement of cell phone billing, moral and
exemplary damages and attorney’s fees against respondents. LA ruled
that petitioners’ dismissal was illegal and held Lopez, Jr. solidarily
liable with Newfield. NLRC affirmed the Labor Arbiter’s decision. CA
reversed the NLRC and dismissed petitioners’ complaint for illegal
dismissal.

ISSUE:

265
Page 266

WON Lopez, Jr., as General Manager of Newfield, can be held


solidarily liable with it for illegal dismissal of the latter’s employees

RULING:
No. Lopez,Jr., as General Manager, is not solidarily liable with
Newfield. The Supreme Court agreed with the NLRC and the LA that
petitioners were illegally dismissed. There is solidary liability when
the obligation expressly so states, when the law so provides, or
when the nature of the obligation so requires. In MAM Realty
Development Corporation v. NLRC, the Court discussed solidary
liability of corporate officers in labor disputes was discussed in this
wise: "A corporation, being a juridical entity, may act only through its
directors, officers and employees. Obligations incurred by them, acting
as such corporate agents, are not theirs but the direct accountabilities
of the corporation they represent. True, solidary liability may at times
be incurred but only when exceptional circumstances warrant
such as, generally, in the following cases:

1. When directors and trustees or, in appropriate cases, the


officers of a corporation -
(a) vote for or assent to patently unlawful acts of the corporation;
(b) act in bad faith or with gross negligence in directing the
corporate affairs;
xxxx

In labor cases, for instance, the Court has held corporate


directors and officers solidarily liable with the corporation for the
termination of employment of employees done with malice or in
bad faith."

Bad faith does not connote bad judgment or negligence; It Imports


dishonest purpose or some moral obliquity and conscious doing of
wrong; it means breach of a known duty through some motive or
interest or ill will; it partakes of the nature of fraud. To sustain such a
finding, there should be evidence on record that an officer or director
acted maliciously or in bad faith in terminating the employee. But here,
the LA and NLRC have not found Lopez, Jr. guilty of malice or bad
faith. Thus, there is no basis to hold Lopez, Jr. solidarily liable with
Newfield. Payment of the judgment award is the direct accountability
of Newfield.

266
Page 267

Case Digests by: Duterte, Camille Anne E.

ST. MARTIN FUNERAL HOMES v. NLRC


G.R. No. 130866, September 16, 1998
Regalado, J.

DOCTRINE:
RELIEFS AGAINST JUDGMENTS/DECISIONS RENDERED BY THE
COMMISSION
All references in the amended Section 9 of B.P. No. 129 to
supposed appeals from the NLRC to the Supreme Court are
interpreted and hereby declared to mean and refer to petitions for
certiorari under Rule 65. Consequently, all such petitions should hence
forth be initially filed in the Court of Appeals in strict observance of the
doctrine on the hierarchy of courts.
FACTS:
Bienvenido Arcayos (respondent) claimed before the NLRC that
he was illegally dismissed by St. Martin Funeral Homes for allegedly
misappropriating funds worth P38,000 which was supposed to be
taxes paid to the BIR. St. Martin (petitioner) contended that respondent
is not an employee due to the lack of an employer-employee contract;
that he was not listed on St. Martin’s monthly payroll.
LA ruled in favor of petitioner, finding that there was no employer-
employee relationship between the two as he was a mere volunteer,
hence, there could be no illegal dismissal. The respondent appealed
to the NLRC which set aside the decision and remanded the case to
the LA. Petitioner filed a motion for reconsideration, but was denied by
the NLRC. Petitioner appealed to the SC through a petition for
certiorari under Rule 65 of the Rules of Court, alleging that the NLRC
committed grave abuse of discretion.

IS8SUE:
WON the petition for certiorari under Rule 65 of the Rules of Court
was properly filed before the Supreme Court

RULING:
** note that the majority of the discussion in the ruling is on the
interpretation of conflicting laws on the proper judicial review of NLRC
decisions. In sum, it was held that the proper review of the NLRC
decision is through a Petition for Certiorari under Rule 65, but following
the doctrine on hierarchy of courts, it must be brought first before the
CA instead of immediately before the SC.
NO. All references in the amended Section 9 of B.P. No. 129
to supposed appeals from the NLRC to the Supreme Court are
interpreted and hereby declared to mean and refer to petitions for

267
Page 268

certiorari under Rule 65. Consequently, all such petitions should


hence forth be initially filed in the Court of Appeals in strict
observance of the doctrine on the hierarchy of courts as the
appropriate forum for the relief desired.
Apropos to this directive that resort to the higher courts should
be made in accordance with their hierarchical order, this
pronouncement in Santiago vs. Vasquez, et al. should be taken into
account: this Court will not entertain direct resort to it unless the
redress desired cannot be obtained in the appropriate courts or
where exceptional and compelling circumstances justify
availment of a remedy within and calling for the exercise of our
primary jurisdiction.

DISCUSSION ON THE CONFLICTING LAWS GOVERNING THE


PROPER JUDICIAL REVIEW OF NLRC DECISIONS:
Our mode of judicial review over decisions of the NLRC has for
some time now been understood to be by a petition for certiorari under
Rule 65 of the Rules of Court. This is, of course, a special original
action limited to the resolution of jurisdictional issues, that is, lack or
excess of jurisdiction and, in almost all cases that have been brought
to us, grave abuse of discretion amounting to lack of jurisdiction.
It will, however, be noted that paragraph (3), Section 9 of B.P.
No. 129 now grants exclusive appellate jurisdiction to the Court of
Appeals over all final adjudications of the Regional Trial Courts and the
quasi-judicial agencies generally or specifically referred to therein
except, among others, "those falling within the appellate jurisdiction of
the Supreme Court in accordance with . . . the Labor Code of the
Philippines under P.D. No. 442, as amended, . . . ." This would
necessarily contradict what has been ruled and said all along that
appeal does not lie from decisions of the NLRC. Yet, under such
excepting clause literally construed, the appeal from the NLRC cannot
be brought to the Court of Appeals, but to this Court by necessary
implication.
The same exceptive clause further confuses the situation by
declaring that the Court of Appeals has no appellate jurisdiction over
decisions falling within the appellate jurisdiction of the Supreme Court
in accordance with the Constitution, the provisions of B.P. No. 129, and
those specified cases in Section 17 of the Judiciary Act of 1948. These
cases can, of course, be properly excluded from the exclusive
appellate jurisdiction of the Court of Appeals. However, because of the
aforementioned amendment by transposition, also supposedly
excluded are cases falling within the appellate jurisdiction of the

268
Page 269

Supreme Court in accordance with the Labor Code. This is illogical and
impracticable, and Congress could not have intended that procedural
gaffe, since there are no cases in the Labor Code the decisions,
resolutions, orders or awards wherein are within the appellate
jurisdiction of the Supreme Court or of any other court for that matter.
A review of the legislative records on the antecedents of R.A. No.
7902 persuades us that there may have been an oversight in the
course of the deliberations on the said Act or an imprecision in the
terminology used therein. In fine, Congress did intend to provide for
judicial review of the adjudications of the NLRC in labor cases by the
Supreme Court, but there was an inaccuracy in the term used for the
intended mode of review.
The Court is, therefore, of the considered opinion that ever since
appeals from the NLRC to the Supreme Court were eliminated, the
legislative intendment was that the special civil action of certiorari was
and still is the proper vehicle for judicial review of decisions of the
NLRC. The use of the word "appeal" in relation thereto and in the
instances we have noted could have been a lapsus plumae because
appeals by certiorari and the original action for certiorari are both
modes of judicial review addressed to the appellate courts. The
important distinction between them, however, and with which the Court
is particularly concerned here is that the special civil action of certiorari
is within the concurrent original jurisdiction of this Court and the Court
of Appeals; whereas to indulge in the assumption that appeals by
certiorari to the Supreme Court are allowed would not subserve, but
would subvert, the intention of Congress as expressed in the
sponsorship speech on Senate Bill No. 1495.
While we do not wish to intrude into the Congressional sphere
on the matter of the wisdom of a law, on this score we add the further
observations that there is a growing number of labor cases being
elevated to this Court which, not being a trier of fact, has at times been
constrained to remand the case to the NLRC for resolution of unclear
or ambiguous factual findings; that the Court of Appeals is procedurally
equipped for that purpose, aside from the increased number of its
component divisions; and that there is undeniably an imperative need
for expeditious action on labor cases as a major aspect of
constitutional protection to labor.

269
Page 270

Case Digests by: Duterte, Camille Anne E.

VELOSO v. CHINA AIRLINES


G.R. No. 104302, July 14, 1999
Quisumbing, J.

DOCTRINE:
RELIEFS AGAINST JUDGMENTS/DECISIONS RENDERED BY THE
COMMISSION
A motion for reconsideration is indispensable, for it affords the
NLRC an opportunity to rectify errors or mistakes it might have
committed before resort to the courts can be had. It is settled that
certiorari will lie only if there is no appeal or any other plain, speedy
and adequate remedy in the ordinary course of law against acts of
public respondent.
Without a motion for reconsideration seasonably filed within the
ten-day reglementary period, an order, decision or resolution of the
NLRC, becomes final and executory after 10 calendar days from
receipt thereof.

FACTS:
Rebecca R. Veloso (petitioner) was employed as supervisor of
the ticketing section at the Manila branch office of China Airlines Ltd.
or “CAL” (respondent). On October 29, 1986, K.Y. Chang (private
respondent), then district manager of the Manila branch office of CAL,
informed petitioner and her co-employees that management had
decided to temporarily close its ticketing section in order to prevent
further losses. On November 5, 1986, petitioner and her staff members
were informed that their recent lay off from employment will be
considered permanent, effective one month from receipt of such
notice. A notice of said retrenchment was filed with the labor
department on November 11, 1986. Later, petitioner was advised to
claim her retirement pay and other benefits.
Petitioner eventually filed with before the NLRC a complaint for
ULP and illegal dismissal with prayer for reinstatement, payment of
backwages, damages and attorney's fees. The LA ruled found
respondents guilty of ULP; that Veloso was illegally dismissed. The
NLRC set aside the decision of the LA. Instead of filing the required
MR, petitioner filed the instant petition for certiorari before the SC.

IS8SUE:
WON the immediate filing of petition for certiorari under Rule 65
before the SC, instead of motion for reconsideration before the NLRC,
was the proper recourse for the petitioner

RULING:

270
Page 271

No. This precipitate filing of petition for certiorari under Rule 65


without first moving for reconsideration of the assailed resolution
warrants the outright dismissal of this case. A motion for
reconsideration is indispensable, for it affords the NLRC an
opportunity to rectify errors or mistakes it might have committed
before resort to the courts can be had. It is settled that certiorari
will lie only if there is no appeal or any other plain, speedy and
adequate remedy in the ordinary course of law against acts of
public respondent. In this case, the plain and adequate remedy
expressly provided by law is a motion for reconsideration of the
impugned resolution, to be made under oath and filed within ten (10)
days from receipt of the questioned resolution of the NLRC, a
procedure which is jurisdictional.
Hence, the filing of the petition for certiorari in this case is
patently violative of prevailing jurisprudence and will not prosper
without undue damage to the fundamental doctrine that undergirds the
grant of this prerogative writ. Further, it should be stressed that
without a motion for reconsideration seasonably filed within the
ten-day reglementary period, an order, decision or resolution of
the NLRC, becomes final and executory after ten (10) calendar
days from receipt thereof.
Hence, the resolution of the NLRC had become final and
executory on January 17, 1992, insofar as petitioner is concerned,
because she admits under oath having received notice thereof on
January 7, 1992. The merits of her case may no longer be reviewed to
determine if the public respondent might be faulted for grave abuse of
discretion, as alleged in her petition dated March 14, 1992. Thus, the
Court has no recourse but to sustain the respondent's position on
jurisdictional and other grounds, that the petition ought not be given
due course and the case should be dismissed for lack of merit.

271
Page 272

Case Digests by: Duterte, Camille Anne E.

STANFILCO v. TEQUILLO
G.R. No. 209735, July 17, 2019
A. Reyes, JR., J.

DOCTRINE:
RELIEFS AGAINST JUDGMENTS/DECISIONS RENDERED BY THE
COMMISSION
The remedy from an adverse decision or final order of the NLRC
is to file a petition for certiorari before the CA on the ground that the
former tribunal acted with grave abuse of discretion in arriving at its
determination of the case.
Rule 45 petitions in labor cases ultimately concern whether the
NLRC's decision is tainted with grave abuse of discretion, and not
whether said decision is correct on the merits

FACTS:
Stanfilco (petitioner herein) is a corporation that operates a
banana plantation in Bukidnon and which holds employee gathering
called “Kaibigan Fellowship” every week. Tequillo (respondent) was a
Farm Associate who worked on petitioner's plantation since 2004.
Instead of attending the said fellowship on 12 Sep 2009, Tequillo went
on a drinking spree and while intoxicated, he mauled his co-worker,
Gayon, when he suggested to air his grievance against Stanfilco with
the high-ranking employees. Tequillo, after administrative hearings,
was terminated on 24 May 2010 on the ground of serious misconduct.
He filed a complaint for illegal dismissal against the Stanfilco. LA
ruled in favor of Stanfilco, finding that the drinking and fighting incident
had been duly proved and Tequillo's acts constituted serious
misconduct and willful disobedience to company rules justifying the
dismissal. NLRC reversed the LA's decision, finding that Tequillo was
illegally dismissed since he was not performing official work at the time
he mauled Gayon; hence, his act could not be work-related. CA
affirmed NLRC's resolution holding that Tequillo's acts at most
amounted only to simple misconduct.

IS8SUE:
WON the CA erred in ruling that no grave abuse of discretion
attended the NLRC's decision declaring Tequillo's dismissal illegal

RULING:
YES. The Court's power to decide Rule 45 petitions in labor
cases is not unlimited. Under our labor laws, a decision or final order
of the NLRC cannot be appealed. This, however, does not mean
that parties are absolutely prohibited from seeking relief from
adverse NLRC decisions. Appellate courts are still vested with the

272
Page 273

power to review such decisions even if the law is silent as to an


explicit right to appeal.
The remedy from an adverse decision or final order of the
NLRC is to file a petition for certiorari before the CA on the ground
that the former tribunal acted with grave abuse of discretion in
arriving at its determination of the case. It follows then that, in labor
cases, the Court enquires into the legal correctness of the CA's
determination of the presence or absence of grave abuse of discretion
in the NLRC decision. As such, the Court is limited to:
(1) Ascertaining the correctness of the CA's decision in finding
the presence or absence of grave abuse of discretion. This is done
by examining, on the basis of the parties' presentations, whether the
CA correctly determined that at the NLRC level, all the adduced pieces
of evidence were considered; no evidence which should not have been
considered was considered; and the evidence presented supports the
NLRC's findings; and
(2) Deciding other jurisdictional error that attended the CA's
interpretation or application of the law.
This entails that Rule 45 petitions in labor cases ultimately
concern whether the NLRC's decision is tainted with grave abuse
of discretion, and not whether said decision is correct on the
merits. Both petitioner and the CA erred in equating work-relatedness
to the time when and place where the offense was committed.
Jurisprudence requires that the confrontation be "rooted on workplace
dynamics" or connected with the performance of the employees'
duties. Clearly, then, the fact that the act complained of in this case,
particularly the mauling of Gayon, took place at the plantation and
while the "Kaibigan Fellowship" was being held is of no moment. Based
on Technol, the enquiry should be into the proximate cause of or
the motive behind the attack. Therefore, while it may be true that
Tequillo acted out of resentment towards petitioner, the same
resentment was essentially attributable to his own work-related
neglect. It follows, then, that the attack was connected to the sub-
standard performance of Tequillo's duties, and that it was
fundamentally rooted in his confounded notion of workplace dynamics.
Verily, to allow him to remain in petitioner's employ would put his fellow
farm workers at risk of physical harm every time he feels wronged.

273
Page 274

Case Digests by: Duterte, Camille Anne E.

HANJIN ENGINEERING v. CA
G.R. NO. 165910, April 10, 2006
Callejo, Sr., J.

DOCTRINE:
RELIEFS AGAINST JUDGMENTS/DECISIONS RENDERED BY THE
COMMISSION
The judicial review of the decisions or final orders of the NLRC
should be filed with the CA under Section 5 of Rule 65, on the ground
that the NLRC committed grave abuse of discretion amounting to
excess or lack of jurisdiction. The remedy of the aggrieved party from
the CA decision, in turn, shall be by petition for review on certiorari with
this Court under Rule 45.
The proper recourse of the aggrieved party from a decision of the
CA is a petition for review on certiorari under Rule 45 of the Revised
Rules of Court. On the other hand, if the error subject of the recourse
is one of jurisdiction, or the act complained of was perpetrated by a
quasi-judicial officer or agency with grave abuse of discretion
amounting to lack or excess of jurisdiction, the proper remedy available
to the aggrieved party is a petition for certiorari under Rule 65 of the
said Rules
FACTS:
Hanjin Development Co., Ltd., (petitioner) had been contracted
by the Philippine Government for the construction of various foreign-
financed projects. Through the National Irrigation Administration (NIA),
they executed contracts for the construction of the Malinao Dam at
Pilar, Bohol. Hanjin contracted the services of 712 carpenters, masons,
truck drivers, helpers, laborers, heavy equipment operators, leadmen,
engineers, steelmen, mechanics, electricians, and others. The 712
employees (respondents) filed complaints for illegal dismissal and for
payment of benefits against petitioners before the NLRC. Petitioners
alleged that the complainants were mere project employees in its
Bohol Irrigation Project.
The LA ruled in favor of the respondents, granting separation pay
and attorney's fees to each of them. Petitioners appealed before the
NLRC, which affirmed with modification the LA's ruling. Petitioners filed
a Petition for Certiorari under Rule 65 of the Revised Rules of Court in
the CA. CA dismissed the petition. Petitioners filed an MR, which the
CA denied. Hence, the instant Petition for Certiorari under Rule 65 of
the Revised Rules of Court.
Respondents aver that petitioners' recourse to Rule 65 of the
Revised Rules of Court, as amended, is mal apropos, citing St. Martin
Funeral Home v. NLRC, where the Court ruled that petitions for
certiorari seeking to review NLRC decisions should initially be filed in
the CA, conformably with the principle of hierarchy of courts. Thus,

274
Page 275

petitioners should have filed a Petition for Review on Certiorari under


Rule 45 of the said Rules; that the only issues raised by petitioners are
factual in nature, not proper in a petition for certiorari under Rule 65.

IS8SUE:
WON petitioners correctly filed before the SC a Petition for
Certiorari under Rule 65 in reviewing CA’s decision in a Petition for
Certiorari under Rule 65 assailing grave abuse of discretion on the part
of NLRC

RULING:
NO. Petitioners’ recourse to this Court via Rule 65 of the Revised
Rules of Court was inappropriate. The SC has original jurisdiction over
petitions for certiorari, prohibition and mandamus, and may review on
appeal or certiorari as the law on the Rules of Court may provide final
judgment and orders of lower courts, and cases in which only
questions of law is involved. However, if a petition for certiorari
involves the acts or omissions of a quasi-judicial agency and
unless otherwise provided by law or the Rules of Court, the
petition for certiorari shall be final and is cognizable only by the
Court of Appeals. One such quasi-judicial agency is the NLRC.
Inasmuch as the appellate court has exclusive appellate jurisdiction
over quasi-judicial agencies under Rule 43, petitions for review on
certiorari should be filed only with the CA, unless otherwise provided
by law or the Rules.
Thus, under the Constitution and the Revised Rules of Court,
judicial review of the decisions or final orders of the NLRC should
be filed with the CA under Section 5 of Rule 65, on the ground that
the NLRC committed grave abuse of discretion amounting to
excess or lack of jurisdiction. The remedy of the aggrieved party
from the CA decision, in turn, shall be by petition for review on
certiorari with this Court under Rule 45.
The aggrieved party is proscribed from assailing a decision
or final order of the CA via Rule 65 because such recourse is
proper only if the party has no plain, speedy and adequate remedy
in the course of law. In this case, petitioners have an adequate
remedy, namely, a petition for review on certiorari under Rule 45 of the
Rules of Court. It must be stressed that the remedies of appeal under
Rule 45 and an original action for certiorari under Rule 65 are mutually
exclusive.
The proper recourse of the aggrieved party from a decision
of the CA is a petition for review on certiorari under Rule 45 of the
Revised Rules of Court. On the other hand, if the error subject of
the recourse is one of jurisdiction, or the act complained of was
perpetrated by a quasi-judicial officer or agency with grave abuse
of discretion amounting to lack or excess of jurisdiction, the

275
Page 276

proper remedy available to the aggrieved party is a petition for


certiorari under Rule 65 of the said Rules.
Since the Court of Appeals had jurisdiction over the petition
under Rule 65, any alleged errors committed by it in the exercise of its
jurisdiction would be errors of judgment which are reviewable by timely
appeal and not by a special civil action of certiorari. If the aggrieved
party fails to do so within the reglementary period, and the
decision accordingly becomes final and executory, he cannot
avail himself of the writ of certiorari, his predicament being the
effect of his deliberate inaction.

276
Page 277

Case Digests by: Duterte, Camille Anne E.

EMPLOYEES UNION OF BAYER PHILS. v. BAYER PHILS


G.R. No. 162943, December 6, 2010
Villarama, Jr., J.

DOCTRINE:
ARTICLE 232
The act of the management of Bayer in dealing and negotiating
with Remigio's splinter group despite its validly existing CBA with
EUBP can be considered unfair labor practice.
In Silva vs NLRC, the Court explained the correlations if Article
248 (1) and 261 of the Labor Code to mean that for a ULP case to be
cognizable by the LA, and for the NLRC to exercise appellate
jurisdiction thereon, the allegations in the complaint must show prima
facie the concurrence of two things: 1) gross violation of the CBA and
2) the violation pertains to the economic provisions of the CBA.
This pronouncement in Silva, however, should not be construed
to apply to violations of the CBA which can be considered as gross
violations per se, such as the utter disregard of the very existence of
the CBA itself, similar to what happened in this case. When an
employer proceeds to negotiate with a splinter union despite the
existence of its valid CBA with the duly certified and exclusive
bargaining agent, the former indubitably abandons its recognition of
the latter and terminates the entire CBA.
FACTS:
Employees Union of Bayer Philippines (petitioner) is SEBA of all
rank-and-file employees of Bayer Philippines (respondent), and is an
affiliate of the Federation of Free Workers. During the negotiations
between EUBP President Juanito S. Facundo and Bayer, EUBP
rejected Bayer's 9.9% wage-increase proposal resulting in a
bargaining deadlock. EUBP staged a strike, prompting the SOLE to
assume jurisdiction over the dispute. A new CBA was thereafter
registered pursuant to the arbitral award of DOLE to execute a CBA
retroactive January 1, 1997.
Barely six months from the signing of the new CBA, Avelina
Remigio solicited signatures from union members in support of a
resolution containing the decision of the signatories to: (1) disaffiliate
from FFW, (2) rename the union as Reformed Employees Union of
Bayer Philippines (REUBP), (3) adopt a new constitution and by-laws
for the union, (4) abolish all existing officer positions in the union and
elect a new set of interim officers, and (5) authorize REUBP to
administer the CBA between EUBP and Bayer. The said resolution
was signed by 147 of the 257 local union members. Remigio asked
Bayer to desist from further transacting with EUBP. Facundo,
meanwhile, sent similar requests to Bayer requesting for the
remittance of union dues in favor of EUBP and accusing the company

277
Page 278

of interfering with purely union matters. Bayer responded by deciding


not to deal with either of the two groups, and by placing the union dues
collected in a trust account until the conflict between the two groups is
resolved.
EUBP filed a complaint for ULP (1st ULP complaint) against
Bayer for non-remittance of union dues. While the first ULP case was
still pending and despite EUBP's repeated request for a grievance
conference, Bayer decided to turn over the collected union dues to
Anastacia Villareal, Treasurer of REUBP. Petitioners filed a 2nd ULP
complaint against amending the complaint charging the respondents
with ULP committed by organizing a company union, gross violation of
the CBA and violation of their duty to bargain, complaining the refusal
to remit the collected union dues to EUBP despite several demands
sent to the management, and opting to negotiate instead with
Remigio's group despite demands to negotiate. LA dismissed the 1st
ULP complaint and the 2nd ULP complaint for lack of jurisdiction.
NLRC dismissed the appeal on the 2ND ULP complaint. Upon petition
for certiorari, CA sustained both the LA and NLRC’s rulings.

IS8SUE:
WON Bayer Phil. committed ULP when it dealt and negotiated with
a splinter group, and consequently WON LA has jurisdiction over the
same

RULING:
Yes. The act of the management of Bayer in dealing and
negotiating with Remigio's splinter group despite its validly
existing CBA with EUBP can be considered unfair labor practice.
A CBA is entered into in order to foster stability and mutual cooperation
between labor and capital. An employer should not be allowed to
rescind unilaterally its CBA with the duly certified bargaining
agent it had previously contracted with, and decide to bargain
anew with a different group if there is no legitimate reason for
doing so and without first following the proper procedure. If such
behavior would be tolerated, bargaining and negotiations between the
employer and the union will never be truthful and meaningful, and no
CBA forged after arduous negotiations will ever be honored or be relied
upon.
Thus, when a valid and binding CBA had been entered into by
the workers and the employer, the latter is behooved to observe the
terms and conditions thereof bearing on union dues and
representation. If the employer grossly violates its CBA with the duly
recognized union, the former may be held administratively and
criminally liable for unfair labor practice.
Respondents contend that their acts cannot constitute ULP as
the same did not involve gross violations in the economic provisions of
the CBA, citing the provisions of Art. 248 (1) and 261 of the Labor Code

278
Page 279

as amended. Their argument is misplaced. In Silva vs NLRC, the Court


explained the correlations if Article 248 (1) and 261 of the Labor Code
to mean that for a ULP case to be cognizable by the LA, and for the
NLRC to exercise appellate jurisdiction thereon, the allegations in
the complaint must show prima facie the concurrence of two
things: 1) gross violation of the CBA and 2) the violation pertains
to the economic provisions of the CBA.
This pronouncement in Silva, however, should not be
construed to apply to violations of the CBA which can be
considered as gross violations per se, such as the utter disregard
of the very existence of the CBA itself, similar to what happened in
this case. When an employer proceeds to negotiate with a splinter
union despite the existence of its valid CBA with the duly certified
and exclusive bargaining agent, the former indubitably abandons
its recognition of the latter and terminates the entire CBA.

279
Page 280

Case Digests by: Duterte, Camille Anne E.

MONTAÑO v. VERCELES
G.R. No. 168583, July 26, 2010
Del Castillo, J.

DOCTRINE:
ARTICLE 232
BLR and the Regional Directors of DOLE have concurrent
jurisdiction over inter-union and intra-union disputes.
Pursuant to its authority under Article 226, this Bureau exercises
original jurisdiction over intra-union disputes involving federations. It is
well-settled that FFW, having local unions all over the country,
operates in more than one administrative region. Therefore, this
Bureau maintains original and exclusive jurisdiction over disputes
arising from any violation of or disagreement over any provision of its
constitution and by-laws.
FACTS:
Atty. Allan S. Montaño (petitioner) worked as legal assistant of
FFW Legal Center on October 1, 1994. Subsequently, he joined the
union of rank-and-file employees, the FFW Staff Association, and
eventually became the employees' union president in July 1997. In
November 1998, he was likewise designated officer-in-charge of FFW
Legal Center.
During the 21st National Convention and Election of National
Officers of FFW, Atty. Montaño was nominated and elected for the
position of National Vice-President despite the finding of FFW
COMELEC that Atty. Montaño is not qualified to run for the position
because Section 76 of Article XIX of the FFW Constitution and By-
Laws prohibits federation employees from sitting in its Governing
Board and strong opposition and protest of Atty. Ernesto C. Verceles
(respondent), a delegate to the convention and president of University
of the East Employees' Association, which is an affiliate union of FFW.
Atty. Verceles, as President of UEEA-FFW and officer of the
Governing Board of FFW, filed before the BLR a petition for the
nullification of the election of Atty. Montaño as FFW National Vice-
President. Atty. Montaño filed his Comment with Motion to Dismiss on
the grounds that the Regional Director of the DOLE and not the BLR
has jurisdiction over the case. The BLR dismissed the petition; it
upheld its jurisdiction over the intra-union dispute case and affirmed
Atty. Verceles’ legal personality to institute the action as the president
of an affiliate union of FFW. Atty. Verceles filed a petition for certiorari
before the CA which set aside the decision of the BLR, affirming the
jurisdiction of BLR but nullifying the election of Atty. Montaño.

IS8SUE:

280
Page 281

WON BLR has jurisdiction over an intra-union dispute between


member-unions of a federation with respect to interpretation of the
federation’s constitution and by-laws in an election.

RULING:
YES. The BLR has jurisdiction over the case. Section 226 of the
Labor Code clearly provides that the BLR and the Regional Directors
of DOLE have concurrent jurisdiction over inter-union and intra-
union disputes. Such disputes include the conduct or nullification of
election of union and workers' association officers. There is, thus, no
doubt as to the BLR's jurisdiction over the instant dispute
involving member-unions of a federation arising from
disagreement over the provisions of the federation's constitution
and by-laws.
Rule XVI lays down the decentralized intra-union dispute
settlement mechanism. Section 1 states that any complaint in this
regard ‘shall be filed in the Regional Office where the union is
domiciled.' The concept of domicile in labor relations regulation is
equivalent to the place where the union seeks to operate or has
established a geographical presence for purposes of collective
bargaining or for dealing with employers concerning terms and
conditions of employment.
The matter of venue becomes problematic when the intra-union
dispute involves a federation, because the geographical presence of a
federation may encompass more than one administrative region.
Pursuant to its authority under Article 226, this Bureau exercises
original jurisdiction over intra-union disputes involving
federations. It is well-settled that FFW, having local unions all over
the country, operates in more than one administrative region.
Therefore, this Bureau maintains original and exclusive
jurisdiction over disputes arising from any violation of or
disagreement over any provision of its constitution and by-laws.”

281
Page 282

Case Digests by: Duterte, Camille Anne E.

DIOKNO, ET AL v. CACDAC
G.R. No. 168475, July 4, 2007
Chico-Nazario, J.

DOCTRINE:
ARTICLE 232
Even as the dispute involves allegations that Daya, et al., sought
the help of non- members of the union in their election campaign to the
detriment of FLAMES, the same does not detract from the real
character of the controversy. It remains as one which involves the
grievance over the constitution and bylaws of a union, and it is a
controversy involving members of the union.

FACTS:
FLAMES, an LLO, is the supervisory union of Meralco.
Petitioners and private respondents are members of FLAMES.
FLAMES Executive Board created the COMELEC (or simply,
COMELEC) for the conduct of its union elections. Jimmy S. Ong,
Nardito C. Alvarez, Alfredo J. Escall, and Jaime T. Valeriano (among
the respondents) filed their respective certificates of candidacy, but the
COMELEC rejected Ong's candidacy as he was not a member of
FLAMES, and the candidacies of Alvarez, Escall, and. Valeriano on
the basis of the exclusion of their department from the scope of the
existing CBA, as the employees therein are deemed disqualified from
membership in the union for being confidential employees.
Private respondents filed a petition before the Med-Arbiter Unit
of DOLE, praying for the nullification of the order of the COMELEC
(Petitioners) which disallowed their candidacy. Petitioners then filed a
petition with the COMELEC seeking disqualification of private
respondents, alleging that Daya, et al. (also among the respondents)
allowed themselves to be assisted by non-union members and having
committed acts of disloyalty inimical to the interests of FLAMES.
The COMELEC then declared Daya, et al. disqualified to run
and/or participate in the elections. The COMELEC stated that they
violated the FLAMES Constitution and By-Laws by allowing non-
members to aid them in their campaign, deemed to be inimical to the
interests of FLAMES. Another group also filed a petition with the Med-
Arbiter unit of DOLE against petitioners to nullify the elections as it was
not free, orderly, and peaceful. The Med-Arbiter ruled in favor of private
respondents and upheld its jurisdiction to rule over the case because
even as the election of union officers is an internal affair of the union,
his office has the right to inquire into the merits and conduct of elections
when its jurisdiction is sought. The BLR Director affirmed the Med-
Arbiter’s decision. The CA found petitioner’s appeal to be without merit.

282
Page 283

IS8SUE:
WON the petition seeking to disqualify Daya, et al for seeking the
help of non-union members is an intra-union dispute within the
jurisdiction of BLR

RULING:
YES. Article 226 of the Labor Code has declared that the BLR
shall have original and exclusive authority to act on all inter-union and
intra-union conflicts.:
(z) "Intra-Union Dispute" refers to any conflict between and among
union members, and includes all disputes or grievances arising from
any violation of or disagreement over any provision of the constitution
and by-laws of a union, including cases arising from chartering or
affiliation of labor organizations or from any violation of the rights and
conditions of union membership provided for in the Code.
An intra-union conflict would refer to a conflict within or
inside a labor union, while an inter-union controversy or dispute
is one occurring or carried on between or among unions.
This is an intra-union dispute. It involves a dispute within or inside
FLAMES, a labor union. At issue is the propriety of the disqualification
of Daya, et al., by the FLAMES COMELEC. It must also be stressed
that even as the dispute involves allegations that Daya, et al.,
sought the help of non- members of the union in their election
campaign to the detriment of FLAMES, the same does not detract
from the real character of the controversy. It remains as one which
involves the grievance over the constitution and bylaws of a
union, and it is a controversy involving members of the union.
Moreover, the non-members of the union who were alleged to have
aided Daya, et al., are not parties in the case. The Petition, which was
initiated by private respondents Daya, et al., before the BLR was
properly within its cognizance, it being an intra-union dispute.
Indubitably, when Daya, et al., brought the case to the BLR, it was an
invocation of the power and authority of the BLR to act on an intra-
union conflict.

283
Page 284

Case Digests by: Duterte, Camille Anne E.

LA TONDEÑA WORKERS UNION v. SECRETARY OF LABOR


G.R. No. 96821, December 9, 1994
Mendoza, J.

DOCTRINE:
ARTICLE 232
The "union accounts examiners of the Bureau" mentioned in
Rule 1, Sec. 1(ff) of the IRR as having the power to audit the books of
accounts of unions are actually officials of the BLR because the word
"Bureau" is defined in sec. 1(b) as the Bureau of Labor Relations
(BLR). The delegation of authority to union accounts examiners is not
exclusive. By indorsing the case to the BLR, SOLE must be presumed
to have authorized the BLR to act on his behalf.
Independently of any delegation, the BLR had power of its own
to conduct the examination of accounts in this case as provided by
Book IV, Title VII, Chapter 4, Sec. 16 of the Administrative Code of
1987.
FACTS:
La Toñeda Worker’s Union (Petitioner) was, for more than 30
years, the bargaining agent of the RnF workers of La Tondeña Inc. at
its Tondo Plant. On March 14, 1989, about 200, out of its 1,015
members, petitioned the DOLE-NCR for an audit or examination of the
funds and financial records of the union. The acting auditing examiner
of the DOLE-NCR found Ramon de Ia Cruz and Norma Marin
accountable for P367,553.00 for union dues remitted by La Tondeña
Inc. to LTWU. De Ia Cruz and Marin appealed to SOLE Drilon,
complaining that they had not been heard before the report was made.
It was indorsed to the respondent Director of BLR.
The BLR Director found that indeed De la Cruz and Marin had
not been heard before they were held liable for union funds; she set
aside the findings and recommendations of the DOLE-NCR and
ordered another audit/examination to be conducted. Petitioner filed an
MR, contending that under Art. 274 LC, as amended by R.A. No. 6715,
the power to order an examination of the books of accounts and
financial activities of a union is vested in the SOLE or his
representative, and the BLR cannot be considered the SOLE’s
representative, which was denied. Petitioner filed a petition for review
of the orders before the SOLE, but BLR proceeded with its
examination. The union officers refused to comply with its orders,
hence, the BLR based the audit/examination on the certification of the
company. BLR found the union officers personally accountable and
liable for the union dues. SOLE did not act on the petition for review of
the union. Instead, he referred the petition to the BLR which denied the
petition for having become moot and academic.
.

284
Page 285

IS8SUE:
WON the power to examine the books of accounts of a labor union
is also vested to the BLR

RULING:
YES. The "union accounts examiners of the Bureau"
mentioned in Rule 1, Sec. 1(ff) of the IRR as having the power to
audit the books of accounts of unions are actually officials of the
BLR because the word "Bureau" is defined in sec. 1(b) as the
Bureau of Labor Relations (BLR). The delegation of authority to
union accounts examiners is not exclusive. By indorsing the case
to the BLR, SOLE must be presumed to have authorized the BLR
to act on his behalf. SOLE made two indorsements: first, when he
referred to the BLR the letter of Ramon de la Cruz and Norma Marin
seeking the annulment of the audit report of the DOLE NCR, and
second, when, instead of acting on the Petition for Review of the union,
he indorsed it to the BLR.
Independently of any delegation, the BLR had power of its
own to conduct the examination of accounts in this case as
provided by Book IV, Title VII, Chapter 4, Sec. 16 of the
Administrative Code of 1987: It shall also set policies, standards, and
procedure relating to collective bargaining agreements, and the
examination of financial records of accounts of labor organizations to
determine compliance with relevant laws.
Art. 226 of the Labor Code, as amended by RA 6715, likewise
authorizes the BLR to decide intra- union disputes. This includes the
examinations of accounts. Conflicts affecting labor-management
relations are apart from intra-union conflicts, as is apparent from the
text of Art. 226.

285
Page 286

Case Digests by: Duterte, Camille Anne E.

ABBOT LABORATORIES v. ABBOT LABORATORIES


EMPLOYEE UNION
G.R. No. 131374, January 26, 2000
Davide, JR., CJ.

DOCTRINE:
ARTICLE 232
The appellate jurisdiction of the SOLE is limited only to a review
of cancellation proceedings decided by the BLR in the exercise of its
exclusive and original jurisdiction.
The decisions of the BLR on cases brought before it on appeal
from the Regional Director are final and executory. Hence, the remedy
of the aggrieved party is to seasonably avail of the special civil action
of certiorari under Rule 65.
FACTS:
The Abbott Laboratories Employees Union or ALEU
(respondent) represented by its president, Alvin B. Buerano, filed an
application for union registration in the DOLE. It alleged that it is a labor
organization with members consisting of 30 RnF employees in the
manufacturing unit of ABBOTT (petitioner), that there was no certified
bargaining agent in the manufacturing unit it sought to represent. The
application was approved by BLR.
ABBOTT filed a petition for cancellation of the Certificate of
Registration issued to ALEU by the Regional Office of the BLR.
ABBOTT assailed the certificate of registration since ALEU's
application was not signed by at least 20% of the total 286 RnF
employees of the entire employer unit; and that it omitted to submit
copies of its books of account. The Regional Director of BLR decreed
the cancellation of ALEU's registration certificate on the ground of
absence of common interest among the RnF employees in the
manufacturing unit to justify the formation of a separate bargaining unit.
Upon appeal to the SOLE, the latter referred the same back to
the Director of the BLR. The assailed decision was reversed by the
BLR. ABBOTT elevated the case to the SOLE who to act thereon on
the ground that it had no jurisdiction over decisions rendered on appeal
by the BLR in cancellation cases emanating from the Regional Offices.
.
IS8SUE:
WON SOLE has the authority to review the decision of the BLR on
appeals in cancellation cases emanating from the Regional Office

RULING:
No. The appellate jurisdiction of the SOLE is limited only to a
review of cancellation proceedings decided by the BLR in the exercise
of its exclusive and original jurisdiction. The SOLE has no jurisdiction

286
Page 287

over decisions of the BLR rendered in the exercise of its appellate


power to review the decision of the Regional Directors in a petition to
cancel the union's certificate of registration, said decisions being final
and unappealable.
The decisions of the BLR on cases brought before it on appeal
from the Regional Director are final and executory. Hence, the remedy
of the aggrieved party is to seasonably avail of the special civil action
of certiorari under Rule 65.

287
Page 288

Case Digest by: FAN, PAULON

TAKATA CORPORATION VS. BLR


G.R. No. 196276. June 4, 2014.
Peralta, J.

DOCTRINE:
ARTICLE 232.
For fraud and misrepresentation to be grounds for cancellation of
union registration under Article 239 of the Labor Code, the nature of
the fraud and misrepresentation must be grave and compelling enough
to vitiate the consent of a majority of union members.

FACTS:
Petitioner filed with the Department of Labor and Employment
(DOLE) Regional Office a Petition for Cancellation of the Certificate of
Union Registration of Respondent Samahang Lakas Manggagawa ng
Takata (SALAMA1) on the ground that the latter is guilty of
misrepresentation, false statement and fraud with respect to the
number of those who participated in the organizational meeting, the
adoption and ratification of its Constitution and By-Laws, and in the
election of its officers.

It contended that in the organizational meeting of respondent, only


68 attendees signed the attendance sheet, and which number
comprised only 17% of the total number of the 396 regular rank- and-
file employees which respondent sought to represent, and hence,
respondent failed to comply with the 20% minimum membership
requirement. Petitioner insisted that the document "Pangalan ng mga
Kasapi ng Unyon" bore no signatures of the alleged 119 union
members; and that employees were not given sufficient information on
the documents they signed; that the document "Sama-Samang
Pahayag ng Pagsapi" was not submitted at the time of the filing of
respondent's application for union registration; that the 119 union
members were actually only 117; and, that the total number of
petitioner's employees as of May 1, 2009 was 470, and not 396 as
respondent claimed. DOLE Regional Director, Atty. Ricardo S.
Martinez, Sr., issued a Decision granting the petition for cancellation of
respondent's certificate of registration.

ISSUE:
Whether or not respondent’s registration should be cancelled on
the grounds of fraud and misrepresentation bearing on the minimum
requirement of the law as to its membership, considering the big

288
Page 289

disparity in numbers, between the organizational meeting and the list


of members.

RULING:
No. It does not appear in Article 234 (b) of the Labor Code that the
attendees in the organizational meeting must comprise 20% of the
employees in the bargaining unit. In fact, even the Implementing Rules
and Regulations of the Labor Code does not so provide. It is only under
Article 234 (c) that requires the names of all its members comprising
at least twenty percent (20%) of all the employees in the bargaining
unit where it seeks to operate.

Clearly, the 20% minimum requirement pertains to the employees’


membership in the union and not to the list of workers who participated
in the organizational meeting. Indeed, Article 234 (b) and (c) provide
for separate requirements, which must be submitted for the union's
registration, and which respondent did submit. Here, the total number
of employees in the bargaining unit was 396, and 20% of which was
about 79. Respondent submitted a document entitled "Pangalan ng
Mga Kasapi ng Unyon" showing the names of 119 employees as union
members, thus respondent sufficiently complied even beyond the 20%
minimum membership requirement. Respondent also submitted the
attendance sheet of the organizational meeting which contained the
names and signatures of the 68 union members who attended the
meeting.

Considering that there are 119 union members which are more
than 20% of all the employees of the bargaining unit, and since the law
does not provide for the required number of members to attend the
organizational meeting, the 68 attendees which comprised at least the
majority of the 119 union members would already constitute a quorum
for the meeting to proceed and to validly ratify the Constitution and By-
laws of the union. There is, therefore, no basis for petitioner to contend
that grounds exist for the cancellation of respondent's union
registration.

For fraud and misrepresentation to be grounds for cancellation of


union registration under Article 239 of the Labor Code, the nature of
the fraud and misrepresentation must be grave and compelling enough
to vitiate the consent of a majority of union members.

289
Page 290

Case Digest by: FAN, PAULON

MAGBUANA VS. UY
G.R. No. 161003. May 6, 2005.
Panganiban, J.

DOCTRINE:
ARTICLE 233.
The presence or the absence of counsel when a waiver is
executed does not determine its validity. There is no law requiring the
presence of a counsel to validate a waiver. The test is whether it was
executed voluntarily, freely and intelligently; and whether the
consideration for it was credible and reasonable.

FACTS:
As a final consequence of the final and executory decision of the
Supreme Court in Rizalino P. Uy v. National Labor Relations
Commission, et. al. (GR No. 117983, September 6, 1996) which
affirmed with modification the decision of the NLRC in NLRC Case No.
V-0427-93, hearings were conducted [in the National Labor Relations
Commission Sub-Regional Arbitration Branch in Iloilo City] to
determine the amount of wage differentials due the eight (8)
complainants therein, now [petitioners]. As computed, the award
amounted to P1,487,312.69.

On February 3, 1997, [petitioners] filed a Motion for Issuance of


Writ of Execution. On May 19, 1997, [respondent] Rizalino Uy filed a
Manifestation requesting that the cases be terminated and closed,
stating that the judgment award as computed had been complied with
to the satisfaction of [petitioners]. Said Manifestation was also signed
by the eight (8) [petitioners]. Together with the Manifestation is a Joint
Affidavit dated May 5, 1997 of [petitioners], attesting to the receipt of
payment from [respondent] and waiving all other benefits due them in
connection with their complaint.

On June 3, 1997, [petitioners] filed an Urgent Motion for Issuance


of Writ of Execution wherein they confirmed that each of them
received P40,000 from [respondent] on May 2, 1997. On June 9, 1997,
[respondent] opposed the motion on the ground that the judgment
award had been fully satisfied. In their Reply, [petitioners] claimed that
they received only partial payments of the judgment award.

On October 20, 1997, six (6) of the eight (8) [petitioners] filed a
Manifestation requesting that the cases be considered closed and

290
Page 291

terminated as they are already satisfied of what they have received (a


total of P320,000) from [respondent]. Together with said Manifestation
is a Joint Affidavit in the local dialect, dated October 20, 1997, of the
six (6) [petitioners] attesting that they have no more collectible amount
from [respondent] and if there is any, they are abandoning and waiving
the same. On February 27, 1998, the Labor Arbiter issued an order
denying the motion for issuance of writ of execution and [considered]
the cases closed and terminated.

On appeal, the [National Labor Relations Commission (hereinafter


‘NLRC’)] reversed the Labor Arbiter and directed the immediate
issuance of a writ of execution, holding that a final and executory
judgment can no longer be altered and that quitclaims and releases
are normally frowned upon as contrary to public policy.

ISSUE:
1. Whether or not the final and executory judgment of the Supreme
Court could be subject to compromise settlement; and

2. Whether or not the petitioners’ affidavit waiving their awards in


[the] labor case executed without the assistance of their counsel and
labor arbiter is valid

RULING:
1. Yes. A compromise must not be contrary to law, morals, good
customs and public policy; and must have been freely and intelligently
executed by and between the parties. To have the force of law between
the parties, it must comply with the requisites and principles of
contracts. Upon the parties, it has the effect and the authority of res
judicata, once entered into. Jesalva v. Bautista upheld a compromise
agreement that covered cases pending trial, on appeal, and with final
judgment.

The Court noted that Article 2040 impliedly allowed such


agreements; there was no limitation as to when these should be
entered into. Palanca v. Court of Industrial Relations sustained a
compromise agreement, notwithstanding a final judgment in which only
the amount of back wages was left to be determined. The Court found
no evidence of fraud or of any showing that the agreement was
contrary to law, morals, good customs, public order, or public policy.

Gatchalian v. Arlegui upheld the right to compromise prior to the


execution of a final judgment. The Court ruled that the final judgment
had been novated and superseded by a compromise agreement. Also,
Northern Lines, Inc. v. Court of Tax Appeals recognized the right to

291
Page 292

compromise final and executory judgments, as long as such right was


exercised by the proper party litigants.

2. Yes. The presence or the absence of counsel when a waiver is


executed does not determine its validity. There is no law requiring the
presence of a counsel to validate a waiver. The test is whether it was
executed voluntarily, freely and intelligently; and whether the
consideration for it was credible and reasonable. Where there is clear
proof that a waiver was wangled from an unsuspecting or a gullible
person, the law must step in to annul such transaction. In the present
case, petitioners failed to present any evidence to show that their
consent had been vitiated.

292
Page 293

Case Digest by: FAN, PAULON

PHILIPPINE TRANSMARINE CARRIER VS. PELAGIO


G.R. No. 211302. August 12, 2015.
Perlas-Bernabe, J.

DOCTRINE:
ARTICLE 233.
Compromise agreements: Once entered into, it has the effect and
the authority of res judicata upon the parties. In other words, a valid
compromise agreement may render a pending case moot and
academic. However, the parties may opt to put therein clauses,
conditions, and the like that would prevent a pending case from
becoming moot and academic - such as when the execution of such
agreement is without prejudice to the final disposition of the said case.

FACTS:
PTCI, for and on behalf of his foreign principal, Norwegian Crew
Management A/S, hired Pelagio as a Motorman on board the vessel
MN Drive Mahone for a period of six (6) months, under a Philippine
Overseas Employment Administration (POEA)-approved employment
contract dated September 29, 2009, as well as the collective
bargaining agreement between Norwegian Crew Management A/S
and Associated Marine Officers' and Seamen's Union of the
Philippines (CBA). After being declared fit for employment, Pelagio
boarded M/V Drive Mahone on November 3, 2009.

Sometime in February 2010, Pelagio experienced difficulty in


breathing and pains on the nape, lower back, and joints while at work.
Pelagio was then referred to a port doctor in Said, Egypt, where he was
diagnosed with "Myositis" and declared unfit to work. On March 2,
2010, Pelagio was repatriated back to the Philippines for further
medical treatment, and thereafter, promptly sought the medical
attention of the company-designated physician, Dr. Robert D. Lim (Dr.
Lim), at the Metropolitan Medical Center.

Pelagio sought payment of permanent total disability benefits from


petitioners, but to no avail. Hence, he filed a complaint for disability
benefits, reimbursement of medical expenses, illness allowance,
damages, and attorney's fees against petitioners before the Arbitration
Branch of the National Labor Relations Commission
(NLRC). Essentially, Pelagio contended that his inability to work for
more than 120 days from repatriation entitles him to permanent total
disability benefits.

293
Page 294

ISSUE:
Whether or not the execution of the Satisfaction of Judgment
between the parties rendered the certiorari proceedings before the CA
moot and academic.

RULING:
No. A compromise agreement is a contract whereby the parties,
by making reciprocal concessions, avoid a litigation or put an end to
one already commenced. To be considered valid and binding between
the contracting parties, a compromise agreement must be: (a) not
contrary to law, morals, good customs, public order, and public policy;
(b) freely and intelligently executed by and between the parties; and
(c) compliant with the requisites and principles of contracts.

Once entered into, it has the effect and the authority of res
judicata upon the parties. In other words, a valid compromise
agreement may render a pending case moot and academic. However,
the parties may opt to put therein clauses, conditions, and the like that
would prevent a pending case from becoming moot and academic -
such as when the execution of such agreement is without prejudice to
the final disposition of the said case. After all, a compromise
agreement is still a contract by nature, and as such, the parties are free
to insert clauses to modify its legal effects, so long as such
modifications are not contrary to law, morals, good customs, public
order, or public policy.

In the instant case, it is undisputed that the parties had entered


into a Satisfaction of Judgment signifying that petitioners had already
given Pelagio the amount of P3,313,772.00 as full and complete
satisfaction of the NLRC ruling. While this document may be properly
deemed as a compromise agreement, it is conditional in nature,
considering that it is without prejudice to the certiorari proceedings
pending before the CA, i.e., it obliges Pelagio to return the aforesaid
proceeds to petitioners should the CA ultimately rule in the latter's
favor. In Leonis Navigation Co., Inc. v. Villamater (Leonis Navigation),
the Court held that such an agreement will not render a pending case
moot and academic as it does not preclude the employer from
recovering from the employee should the courts ultimately decide in
favor of the former.

A reading of the foregoing documents reveals that: (a) petitioners


paid Pelagio P3,313,772.00 as full and complete satisfaction of the
NLRC rulings; (b) such payment is made in order to prevent imminent
execution of such rulings being undertaken by the NLRC and Pelagio;
(c) such payment is without prejudice to the outcome of
the certiorari proceedings before the CA; and (d) in case of partial or

294
Page 295

complete reversal of the NLRC judgment by the CA, Pelagio is obliged


to reimburse petitioners accordingly.

More importantly, the foregoing documents do not have any


clause prohibiting either of the parties from seeking further redress
against each other. Thus, both petitioners and Pelagio may pursue any
of the available legal remedies should any eventuality arise in their
dispute, i.e., when the CA renders a ruling adverse to their respective
interests.

It can, therefore, be said that the agreement entered into by the


petitioners and Pelagio is fair and is not prejudicial to either party, and
thus, such agreement did not render the certiorari proceedings before
the CA moot and academic.

295
Page 296

Case Digest by: FAN, PAULON

MAGSAYSAY MARITIME VS. DE JESUS


G.R. No. 203943. August 30, 2017.
Leonen, J.

DOCTRINE:
ARTICLE 233.
A conditional settlement of a judgment award may be treated as a
compromise agreement and a judgment on the merits of the case if it
turns out to be highly prejudicial to one of the parties.

FACTS:
Magsaysay Maritime Corporation (Magsaysay), the local manning
agent of Princess Cruise Lines, Limited, hired Bernardine De Jesus
(Bernardine) as an Accommodation Supervisor for the cruise ship
Regal Princess. Based on the contract of employment that he signed,
Bernardine was to receive a basic monthly wage of US$388.00 for a
period of 10 months. On March 9, 2006, Bernardine boarded Regal
Princess and he eventually disembarked 10 months later, or on
January 16, 2007, after his contract of employment ended.

Bernardine was soon diagnosed with Aortic Aneurysm and on


March 15, 2007, he had a coronary angiography. On March 21, 2007,
he underwent a Left Axillofemoral Bypass. He died on March 26, 2007.
On September 24, 2008, respondent Cynthia De Jesus (Cynthia),
Bernardine's widow, filed a complaint against Magsaysay for "payment
of death benefits, medical expenses, sickness allowance, damages,
and attorney's fees." Cynthia and Magsaysay were unable to amicably
settle the case; hence, they were directed to submit their respective
position papers.

Magsaysay paid Cynthia P3,370,514.40 as conditional


satisfaction of the judgment award against it and without prejudice to
its Petition for Certiorari pending before the Court of Appeals. In light
of the conditional settlement between the parties, the Labor Arbiter
considered the case closed and terminated but without prejudice to
Magsaysay's pending petition before the Court of Appeals.

ISSUE:
Whether or not the payment of money judgment has rendered the
Petition for Certorari before the Court of Appeals moot and academic

RULING:

296
Page 297

The Court of Appeals did not err in treating the conditional


settlement as an amicable settlement, effectively rendering the Petition
for Certiorari moot and academic. In the instant case, the parties
entered into a compromise agreement when they executed a
Conditional Satisfaction of Judgment Award.

Article 2028 of the Civil Code defines a compromise agreement as


"a contract whereby the parties, by making reciprocal concessions,
avoid a litigation or put an end to one already commenced." Parties
freely enter into a compromise agreement, making it a judgment on the
merits of the case with the effect of res judicata upon them.

While the general rule is that a valid compromise agreement has


the power to render a pending case moot and academic, being a
contract, the parties may opt to modify the legal effects of their
compromise agreement to prevent the pending case from becoming
moot.

In the Conditional Satisfaction of Judgment Award, respondent


acknowledged receiving the sum of P3,370,514.40 from petitioners as
conditional payment of the judgment award. Both parties agreed that
the payment of the judgment award was without prejudice to the
pending certiorari proceedings before the Court of Appeals and was
only made to prevent the imminent execution being undertaken by
respondent and the National Labor Relations Commission. Finally, in
the event the judgment award of the labor tribunals is reversed by the
Court of Appeals or by this Court, respondent agreed to return
whatever she would have received back to petitioners and in the same
vein, if the Court of Appeals or this Court affirms the decisions of the
labor tribunals, petitioners shall pay respondent the balance of the
judgment award without need of demand.

297
Page 298

Case Digest by: FAN, PAULON

SOLOMON ET. AL. VS. POWERTECH CORP.


G.R. No. 150861. January 22, 2008.
Reyes, R.T. J.

DOCTRINE:
ARTICLE 233.
Article 227 of the Labor Code empowers the NLRC to void a
compromise agreement for fraud.

Any compromise settlement, including those involving labor


standard laws, voluntarily agreed upon by the parties with the
assistance of the Bureau or the regional office of the Department of
Labor, shall be final and binding upon the parties. The National Labor
Relations Commission or any court shall not assume jurisdiction over
issues involved therein except in case of non-compliance thereof or if
there is prima facie evidence that the settlement was obtained
through fraud, misrepresentation, or coercion.

FACTS:
The case stems from a complaint for illegal dismissal and other
money claims filed by the Nagkakaisang Manggagawa Ng Powertech
Corporation on behalf of its 52 individual members and non-union
members against their employer, Powertech. The case was dismissed
as to twenty-seven (27) employees by virtue of duly executed affidavits
of repudiation and quitclaim. The case proceeded with respect to the
remaining twenty-five (25) employees, petitioners in this case.

Labor Arbiter Renell Joseph R. Dela Cruz rendered a


Decision declaring illegal the termination of twenty (20) of petitioners
and granting their monetary claims in the total amount
of P2,538,728.84.

Powertech appealed to the NLRC. During its pendency, Carlos


Gestiada, for himself and on behalf of other petitioners, executed a
quitclaim, release and waiver in favor of Powertech in consideration of
the amount of P150,000.00. Earlier, Gestiada was appointed by his co-
petitioners as their attorney-in-fact. The appointment was evidenced
by a special power of attorney. The compromise amount was paid to
Gestiada by check.

Relying on the quitclaim and release, Powertech filed a motion for


the withdrawal of the appeal and cash bond. The NLRC granted the

298
Page 299

motion, dismissed the appeal and ordered the release of the cash
bond. The P150,000.00 check, however, bounced due to a stop
payment order of Powertech.

Aggrieved, petitioners moved to nullify the release and quitclaim


for lack of consideration. In a Resolution, the NLRC declared the
quitclaim, release and waiver void for lack of consideration, reinstated
the appeal and ordered Powertech to post a cash or surety bond for
the monetary judgment less the amount it had previously posted.

A day later, Powertech paid P150,000.00 to Gestiada purportedly


as compromise amount for all of petitioners. That same day, Gestiada,
through Atty. Felipe, and Powertech filed a joint motion to dismiss with
the NLRC based on the compromise agreement. Atty. Evangelista
opposed the motion, alleging that the compromise agreement is
unconscionable, that he was illegally terminated as counsel for the
other petitioners without their consent, and that the P150,000.00 was
received by Gestiada as payment solely for his backwages and other
monetary claims.

ISSUE:
Whether or not the compromise agreement is void

RULING:
Yes. The P150,000 was paid to Gestiada solely as payment for
his backwages, not those of petitioners; there is evident collusion
between Powertech and Gestiada, hence, the compromise agreement
is void.

First, the P150,000 compromise is rather measly when taken in


light of the more than P2.5 million judgment on appeal to the NLRC.

Second, even granting for the mere sake of argument that


the P150,000 was a fair and reasonable compromise for all, petitioners
failed to receive a single centavo from the compromise. This
conclusively indicates that Gestiada received the P150,000 in payment
of his backwages and no other.

Third, We give credence to the admission of Gestiada that he


received the P150,000.00 as payment for his own backwages. In his
letter to Atty. Evangelista, Gestiada said that he was pressured by
Powertech to sign the waiver and quitclaim for petitioners in order to
receive his share in the P2.5 million judgment.

299
Page 300

Fourth, the events that led to the execution of the compromise


agreement show that Powertech was negotiating in bad faith. More
importantly, they show that Powertech colluded with Gestiada to
defraud petitioners of their share of the P2.5 million Labor Arbiter
judgment.

Collusion is a species of fraud. Article 227 of the Labor Code


empowers the NLRC to void a compromise agreement for fraud, thus:

Any compromise settlement, including those involving labor


standard laws, voluntarily agreed upon by the parties with the
assistance of the Bureau or the regional office of the Department of
Labor, shall be final and binding upon the parties. The National Labor
Relations Commission or any court shall not assume jurisdiction over
issues involved therein except in case of non-compliance thereof or if
there is prima facie evidence that the settlement was obtained
through fraud, misrepresentation, or coercion.

In fine, We find that the CA erred in upholding the compromise


agreement between Powertech and Gestiada. The NLRC justifiably
declared the compromise agreement as void.

300
Page 301

Case Digest by: FAN, PAULON

PHILIPPINE JOURNALISTS INC., VS. NLRC


G.R. No. 166421. September 05, 2006.
Callejo, SR., J.

DOCTRINE:
ARTICLE 233.
A compromise agreement cannot bind a party who did not
voluntarily take part in the settlement itself and gave specific individual
consent. It must be remembered that a compromise agreement is also
a contract; it requires the consent of the parties, and it is only then that
the agreement may be considered as voluntarily entered into.

FACTS:
The Philippine Journalists, Inc. (PJI) is a domestic corporation
engaged in the publication and sale of newspapers and magazines.
The exclusive bargaining agent of all the rank-and-file employees in
the company is the Journal Employees Union (Union for brevity).

The Union filed a notice of strike before the National Conciliation


and Mediation Board (NCMB), claiming that PJI was guilty of unfair
labor practice. PJI was then going to implement a retrenchment
program. The Secretary of the Department of Labor and Employment
(DOLE) certified the labor dispute to the National Labor Relations
Commission (NLRC) for compulsory arbitration pursuant to Article 263
(g) of the Labor Code.

In its Resolution dated May 31, 2001, the NLRC declared that the
31 complainants were illegally dismissed and that there was no basis
for the implementation of petitioner's retrenchment program.

Thereafter, the parties executed a Compromise Agreement,


where PJI undertook to reinstate the 31 complainant-employees
without loss of seniority rights and benefits; 17 of them who were
previously retrenched were agreed to be given full and complete
payment of their respective monetary claims, while 14 others would be
paid their monetary claims minus what they received by way of
separation pay. The compromise agreement was submitted to the
NLRC for approval.

The NLRC forthwith issued another Resolution, declaring that the


Clarificatory Motion of complainants Floro Andrin, Jr. and Jazen M.
Jilhani had been mooted by the compromise agreement as they

301
Page 302

appeared to be included in paragraph 2.c and paragraph 2.d,


respectively thereof.

In the meantime, the Union filed another notice of strike claiming


that 29 employees were illegally dismissed and that the salaries and
benefits of 50 others had been illegally reduced. The NLRC ruled that
the complainants were not illegally dismissed because the resolution
declaring the retrenchment program illegal did not attain finality as it
had been mooted by the compromise agreement entered into by the
parties. The appellate court however, held that the compromise
agreement referred only to the award given by the NLRC to the
complainants in the case.

ISSUE:
Whether or not the compromise agreement entered into by the
parties mooted the NLRC resolution finding that the union members
had been illegally dismissed

RULING:
No. The nature of a compromise is spelled out in Article 2028 of
the New Civil Code: it is "a contract whereby the parties, by making
reciprocal concessions, avoid litigation or put an end to one already
commenced." Parties to a compromise are motivated by "the hope of
gaining, balanced by the dangers of losing."

It contemplates mutual concessions and mutual gains to avoid the


expenses of litigation, or, when litigation has already begun, to end it
because of the uncertainty of the result. Article 227 of the Labor Code
of the Philippines authorizes compromise agreements voluntarily
agreed upon by the parties, in conformity with the basic policy of the
State "to promote and emphasize the primacy of free collective
bargaining and negotiations, including voluntary arbitration, mediation
and conciliation, as modes of settling labor or industrial disputes."

Thus, a judgment rendered in accordance with a compromise


agreement is not appealable, and is immediately executory unless a
motion is filed to set aside the agreement on the ground of fraud,
mistake, or duress, in which case an appeal may be taken against the
order denying the motion. Under Article 2037 of the Civil Code, "a
compromise has upon the parties the effect and authority of res
judicata," even when effected without judicial approval; and under the
principle of res judicata, an issue which had already been laid to rest
by the parties themselves can no longer be relitigated.

Thus, contrary to the allegation of petitioners, the execution and


subsequent approval by the NLRC of the agreement forged between it

302
Page 303

and the respondent Union did not render the NLRC resolution
ineffectual, nor rendered it "moot and academic." The agreement
becomes part of the judgment of the court or tribunal, and as a logical
consequence, there is an implicit waiver of the right to appeal.

In any event, the compromise agreement cannot bind a party who


did not voluntarily take part in the settlement itself and gave specific
individual consent. It must be remembered that a compromise
agreement is also a contract; it requires the consent of the parties, and
it is only then that the agreement may be considered as voluntarily
entered into.

A careful perusal of the wordings of the compromise agreement


will show that the parties agreed that the only issue to be resolved was
the question of the monetary claim of several employees. The
agreement was later approved by the NLRC. The case was considered
closed and terminated and the Resolution fully implemented insofar as
the employees “mentioned in paragraphs 2c and 2d of the compromise
agreement” were concerned. Hence, the CA was correct in holding that
the compromise agreement pertained only to the “monetary obligation”
of the employer to the dismissed employees, and in no way affected
the Resolution where the NLRC made the pronouncement that there
was no basis for the implementation of petitioners’ retrenchment
program.

303
Page 304

Case Digest by: FAN, PAULON

PERIQUET VS. NLRC


G.R. No. 91298. June 22, 1990.
Cruz, J.

DOCTRINE:
ARTICLE 233.
Not all waivers and quitclaims are invalid as against public policy.
If the agreement was voluntarily entered into and represents a
reasonable settlement, it is binding on the parties and may not later be
disowned simply because of a change of mind.

It is only where there is clear proof that the waiver was wangled
from an unsuspecting or gullible person, or the terms of settlement are
unconscionable on its face, that the law will step in to annul the
questionable transaction.

FACTS:
The petitioner was dismissed as toll collector by the Construction
Development Corporation of the Philippines, private respondent
herein, for willful breach of trust and unauthorized possession of
accountable toll tickets allegedly found in her purse during a surprise
inspection. Claiming she had been "framed," she filed a complaint for
illegal dismissal and was sustained by the labor arbiter, who ordered
her reinstatement within ten days "without loss of seniority rights and
other privileges and with fun back wages to be computed from the date
of her actual dismissal up to date of her actual reinstatement." On
appeal, this order was affirmed in toto by public respondent NLRC.

Almost nine years later, the petitioner filed a motion for the
issuance of a writ of execution of the decision. The motion was granted
by the executive labor arbiter in an order, which required payment to
the petitioner of the sum of P205,207.42 "by way of implementing the
balance of the judgment amount" due from the private
respondent. Pursuant thereto, the said amount was garnished by the
NLRC sheriff. However, the NLRC sustained the appeal of the CDCP
and set aside the order, the corresponding writ of execution, and the
notice of garnishment.

In its decision, the public respondent held that the motion for
execution was time-barred, having been filed beyond the five-year
period prescribed by both the Rules of Court and the Labor Code. It
also rejected the petitioner's claim that she had not been reinstated on

304
Page 305

time and ruled as valid the two quitclaims she had signed waiving her
right to reinstatement and acknowledging settlement in full of her back
wages and other benefits. The petitioner contends that this decision is
tainted with grave abuse of discretion and asks for its reversal.

ISSUE:
Whether or not the validity of a compromise agreement can be
attacked several years later

RULING:
No. The original decision called for her reinstatement within ten
days from receipt thereof following its affirmance by the NLRC on
August 29, 1980, but there is no evidence that she demanded her
reinstatement or that she complained when her demand was rejected.
What appears is that she entered into a compromise agreement with
CDCP where she waived her right to reinstatement and received from
the CDCP the sum of P14,000.00 representing her back wages from
the date of her dismissal to the date of the agreement.

Dismissing the compromise agreement, the petitioner now claims


she was actually reinstated only on March 16, 1987, and so should be
granted back pay for the period beginning November 28, 1978, date of
her dismissal, until the date of her reinstatement. She conveniently
omits to mention several significant developments that transpired
during and after this period that seriously cast doubt on her candor
and bona fides.

Not all waivers and quitclaims are invalid as against public policy.
If the agreement was voluntarily entered into and represents a
reasonable settlement, it is binding on the parties and may not later be
disowned simply because of a change of mind. It is only where there
is clear proof that the waiver was wangled from an unsuspecting or
gullible person, or the terms of settlement are unconscionable on its
face, that the law will step in to annul the questionable transaction. But
where it is shown that the person making the waiver did so voluntarily,
with full understanding of what he was doing, and the consideration for
the quitclaim is credible and reasonable, the transaction must be
recognized as a valid and binding undertaking. As in this case. The
petitioner obviously decided that a bird in hand was worth two on the
wing and so opted for the compromise agreement. The amount she
was then waiving, it is worth noting, had not yet come up to the
exorbitant sum of P205,207.42 that she was later to demand after the
lapse of eight years.

305
Page 306

Case Digest by: FAN, PAULON

AUJERO VS. PHILCOMSAT


G.R. No. 193484. January 18, 2012.
Reyes, J.

DOCTRINE:
ARTICLE 233.
While the law looks with disfavor upon releases and quitclaims by
employees who are inveigled or pressured into signing them by
unscrupulous employers seeking to evade their legal responsibilities,
a legitimate waiver representing a voluntary settlement of a laborer's
claims should be respected by the courts as the law between the
parties.

FACTS:
It was in 1967 that the petitioner started working for respondent
Philippine Communications Satellite Corporation (Philcomsat) as an
accountant in the latter's Finance Department. After thirty-four (34)
years of service, the petitioner applied for early retirement. His
application for retirement was approved, entitling him to receive
retirement benefits at a rate equivalent to one and a half of his monthly
salary for every year of service. At that time, the petitioner was
Philcomsat's Senior Vice-President with a monthly salary of Two
Hundred Seventy-Four Thousand Eight Hundred Five Pesos
(₱274,805.00).

The petitioner executed a Deed of Release and Quitclaim in


Philcomsat’s favor, following his receipt from the latter of a check in the
amount of Nine Million Four Hundred Thirty-Nine Thousand Three
Hundred Twenty-Seven and 91/100 Pesos (₱9,439,327.91).

Almost three (3) years thereafter, the petitioner filed a complaint


for unpaid retirement benefits, claiming that the actual amount of his
retirement pay is Fourteen Million Fifteen Thousand and Fifty-Five
Pesos (₱14,015,055.00) and the ₱9,439,327.91 he received from
Philcomsat as supposed settlement for all his claims is
unconscionable, which is more than enough reason to declare his
quitclaim as null and void. According to the petitioner, he had no choice
but to accept a lesser amount as he was in dire need thereof and was
all set to return to his hometown and he signed the quitclaim despite
the considerable deficiency as no single centavo would be released to
him if he did not execute a release and waiver in Philcomsat's favor.

306
Page 307

The petitioner claims that Philcomsat has no right to withhold any


portion of his retirement benefits as the trust fund created pursuant to
the Retirement Plan is for the exclusive benefit of Philcomsat
employees and Philcomsat had expressly recognized that it has no
right or claim over the trust fund even on the portion pertaining to its
contributions.

ISSUE:
Whether or not the quitclaim executed by the petitioner in
Philcomsat’s favor is valid, thereby foreclosing his right to institute any
claim against Philcomsat

RULING:
Yes. Absent any evidence that any of the vices of consent is
present and considering the petitioner’s position and education, the
quitclaim executed by the petitioner constitutes a valid and binding
agreement. In Goodrich Manufacturing Corporation, v. Ativo, this Court
reiterated the standards that must be observed in determining whether
a waiver and quitclaim has been validly executed:

Not all waivers and quitclaims are invalid as against public policy.
If the agreement was voluntarily entered into and represents a
reasonable settlement, it is binding on the parties and may not later be
disowned simply because of a change of mind. It is only where there
is clear proof that the waiver was wangled from an unsuspecting
or gullible person, or the terms of settlement are unconscionable
on its face, that the law will step in to annul the questionable
transaction. But where it is shown that the person making the waiver
did so voluntarily, with full understanding of what he was doing, and
the consideration for the quitclaim is credible and reasonable, the
transaction must be recognized as a valid and binding undertaking.

While the law looks with disfavor upon releases and quitclaims by
employees who are inveigled or pressured into signing them by
unscrupulous employers seeking to evade their legal responsibilities,
a legitimate waiver representing a voluntary settlement of a laborer's
claims should be respected by the courts as the law between the
parties. Considering the petitioner's claim of fraud and bad faith against
Philcomsat to be unsubstantiated, this Court finds the quitclaim in
dispute to be legitimate waiver.

While the petitioner bewailed as having been coerced or


pressured into signing the release and waiver, his failure to present
evidence renders his allegation self-serving and inutile to invalidate the
same. That no portion of his retirement pay will be released to him or

307
Page 308

his urgent need for funds does not constitute the pressure or coercion
contemplated by law.

308
Page 309

Case Digest by: FAN, PAULON

CAROLINA’S LACE SHOPPE VS. MAQUILAN


G.R. No. 219419. April 10, 2019.
Reyes, J. JR., J.

DOCTRINE:
ARTICLE 233.
In illegal dismissal cases, the fundamental rule is that when an
employer interposes the defense of resignation, the burden to prove
that the employee indeed voluntarily resigned necessarily rests upon
the employer.

The act of the employee before and after the alleged resignation
must be considered to determine whether in fact, he or she intended
to relinquish such employment.

FACTS:
In 2005 and 2007, Gloria Maquilan (Gloria) and Joy Maquilan (Joy)
were employed by Carolina's Lace Shoppe (CLS) as sales clerk and
header, respectively.

In 2008, the Department of Labor and Employment (DOLE)


inspected CLS. Upon inspection, one of the latter's employees,
Santiago A. Espultero (Espultero) told the labor inspector that he was
receiving a daily wage of ₱250.00. Thereafter, Espultero was
terminated from his employment by CLS' manager Claudine
Mangasing (Mangasing). In order to receive a "separation pay"
amounting to ₱60,000.00 despite his 17 years in service, Espultero
was allegedly made to sign a quitclaim.

One month thereafter, Gloria was dismissed from the service for
no reason given. Like Espultero, she was allegedly made to sign a
quitclaim in order to claim her "separation pay" amounting to
₱15,000.00 despite her three years in service. The same fate
happened to Joy, daughter of Gloria, who was dismissed from the
service and was forced to sign a quitclaim as she received ₱4,000.00
as "separation pay."

Gloria, Joy, Espultero, and Eminda B. Tagalo (Tagalo) were


constrained to file a case for illegal dismissal with money claims and
damages against CLS, Mangasing and sole proprietor Lourdes Ragas
(Ragas) (collectively as respondents). However, only Gloria and Joy
filed their position papers.

309
Page 310

Aside from their claim that CLS caused their illegal dismissal,
Gloria and Joy averred that: (a) they worked on holidays and special
holidays without holiday and premium pay; (b) they worked for more
than one year but were not given five days service incentive leave; (c)
they were given 13th month pay, but its computation was not in
accordance with the minimum wage rates; and (d) they worked for 10
hours a day with no overtime pay.

For their part, respondents claimed that Gloria, Joy and Espultero
were not illegally dismissed as they voluntarily resigned, evidenced by
their resignation letters.

ISSUE:
Whether or not Gloria and Joy were illegally dismissed from
employment

RULING:
Yes. "In illegal dismissal cases, the fundamental rule is that when
an employer interposes the defense of resignation, the burden to prove
that the employee indeed voluntarily resigned necessarily rests upon
the employer." Putting forth their claim that Gloria indeed voluntarily
resigned, respondents insist that the former offered no evidence which
depicted that force or fraud was employed when the resignation letter
with quitclaim was executed. Hence, the same was accomplished
voluntarily.

Citing Fortuny Garments/Johnny Co v. Castro, the case of Torreda


v. Investment and Capital Corporation of the Philippines discusses how
an employee's act of severing from employment may be measured, to
wit: The act of the employee before and after the alleged resignation
must be considered to determine whether in fact, he or she intended
to relinquish such employment. If the employer introduces evidence
purportedly executed by an employee as proof of voluntary resignation
and the employee specifically denies the authenticity and due
execution of said document, the employer is burdened to prove the due
execution and genuineness of such document.

Here, prior to her resignation, there was no indication that Gloria


intended to relinquish her employment. Such alleged resignation
actually took place after the DOLE conducted an inspection, which
yielded to an information that CLS was not giving its employees their
due wages. A month after such inspection, like the employee who
reported such labor standards violation, Gloria was separated from
employment by virtue of a resignation letter. In this regard, there was
no clear intention on the part of Gloria to relinquish her employment.

310
Page 311

As to her acts after her resignation, Gloria filed a complaint for


illegal dismissal and money claims 12 days thereafter. On this note,
this Court reiterates that such act of filing said complaint is difficult to
reconcile with voluntary resignation.

Admittedly, the quitclaim does not indicate that Gloria received the
amount of ₱15,000.00 as full and final settlement. Similarly, there was
nothing which indicates that said amount constitutes said full and final
settlement. The quitclaim was also couched in general terms and the
tenor of the same does not show that Gloria understood the importance
of the same considering that on the same day that she resigned, she
immediately relieved respondents from their liabilities. There was also
no indication that Gloria intends to give up her claimed benefits in
consideration of a fixed compromise amount. It must be emphasized
that Gloria was constrained to receive the amount of ₱15,000.00 as
she was eight months pregnant at that time and lives with no other
means aside from her employment with CLS’.

As to Joy, there was no indication that she intended to voluntarily


resign. There was no execution of a resignation letter, but merely a
quitclaim,30 which likewise does not contain the above-mentioned
stipulations as the same was a standard clearance and quitclaim form
which Joy merely filled out.

311
Page 312

Case Digest by: FAN, PAULON

COLEGIO DE SAN JUAN DE LETRAN VS. ASSOCIATION


OF EMPLOYEES AND FACULTY OF LETRAN
G.R. No. 141471. September 18, 2000.
Kapunan, J.

DOCTRINE:
ARTICLE 238.
In order to allow the employer to validly suspend the bargaining
process there must be a valid petition for certification election raising a
legitimate representation issue. Hence, the mere filing of a petition for
certification election does not ipso facto justify the suspension of
negotiation by the employer. The petition must first comply with the
provisions of the Labor Code and its Implementing Rules. Foremost is
that a petition for certification election must be filed during the sixty-
day freedom period.

The "Contract Bar Rule": If a collective bargaining agreement has


been duly registered in accordance with Article 231 of the Code, a
petition for certification election or a motion for intervention can only be
entertained within sixty (60) days prior to the expiry date of such
agreement.

FACTS:
Salvador Abtria, then President of respondent union, Association
of Employees and Faculty of Letran, initiated the renegotiation of its
Collective Bargaining Agreement with petitioner Colegio de San Juan
de Letran for the last two (2) years of the CBA's five (5) year lifetime
from 1989-1994. On the same year, the union elected a new set of
officers wherein private respondent Eleanor Ambas emerged as the
newly elected President. Ambas wanted to continue the renegotiation
of the CBA but petitioner, through Fr. Edwin Lao, claimed that the CBA
was already prepared for signing by the parties. Petitioner accused the
union officers of bargaining in bad faith before the National Labor
Relations Commission (NLRC). Labor Arbiter Edgardo M. Madriaga
decided in favor of petitioner. However, the Labor Arbiter's decision
was reversed on appeal before the NLRC.

The union notified the National Conciliation and Mediation Board


(NCMB) of its intention to strike on the grounds (sic) of petitioner's:
non-compliance with the NLRC (1) order to delete the name of Atty.
Federico Leynes as the union's legal counsel; and (2) refusal to
bargain. The parties agreed to disregard the unsigned CBA and to start

312
Page 313

negotiation on a new five-year CBA starting 1994-1999. Due to


petitioner's inaction, the union filed a notice of strike.

However, petitioner stopped the negotiations after it purportedly


received information that a new group of employees had filed a petition
for certification election. The union finally struck. Public respondent the
Secretary of Labor and Employment assumed jurisdiction and ordered
all striking employees including the union president to return to work
and for petitioner to accept them back under the same terms and
conditions before the actual strike. Public respondent issued an order
declaring petitioner guilty of unfair labor practice on two counts and
directing the reinstatement of private respondent Ambas with
backwages.

ISSUE:
1. Whether or not the petitioner is guilty of unfair labor practice by
refusing to bargain with the union when it unilaterally suspended the
ongoing negotiations for a new CBA; and

2. Whether or not the termination of the union president amounts


to an interference of the employees’ right to self-organization

RULING:
1. Yes. As regards the first issue, Article 252 of the Labor Code
defines the meaning of the phrase "duty to bargain collectively.
Petitioner's utter lack of interest in bargaining with the union is obvious
in its failure to make a timely reply to the proposals presented by the
latter. This is a clear violation of Article 250 of the Labor Code
governing the procedure in collective bargaining. As we have held in
the case of Kiok Loy vs. NLRC, the company's refusal to make
counterproposal to the union's proposed CBA is an indication of its bad
faith. Where the employer did not even bother to submit an answer to
the bargaining proposals of the union, there is a clear evasion of the
duty to bargain collectively.

Petitioner, likewise, claims that the suspension of negotiation was


proper since by the filing of the petition for certification election the
issue on majority representation of the employees has arisen.

We disagree. In order to allow the employer to validly suspend the


bargaining process there must be a valid petition for certification
election raising a legitimate representation issue. Hence, the mere
filing of a petition for certification election does not ipso facto justify the
suspension of negotiation by the employer. The petition must first
comply with the provisions of the Labor Code and its Implementing
Rules. Foremost is that a petition for certification election must be filed

313
Page 314

during the sixty-day freedom period. The "Contract Bar Rule" under
Section 3, Rule XI, Book V, of the Omnibus Rules Implementing the
Labor Code, provides that: " .… If a collective bargaining agreement
has been duly registered in accordance with Article 231 of the Code, a
petition for certification election or a motion for intervention can only be
entertained within sixty (60) days prior to the expiry date of such
agreement."

The rule is based on Article 232, in relation to Articles 253, 253-A


and 256 of the Labor Code. No petition for certification election for any
representation issue may be filed after the lapse of the sixty-day
freedom period. The old CBA is extended until a new one is signed.
The rule is that despite the lapse of the formal effectivity of the CBA
the law still considers the same as continuing in force and effect until
a new CBA shall have been validly executed. Hence, the contract bar
rule still applies. The purpose is to ensure stability in the relationship
of the workers and the company by preventing frequent modifications
of any CBA earlier entered into by them in good faith and for the
stipulated original period.

2. Yes. Concerning the issue on the validity of the termination of


the union president, we hold that the dismissal was effected in violation
of the employees' right to self-organization.

While we recognize the right of the employer to terminate the


services of an employee for a just or authorized cause, nevertheless,
the dismissal of employees must be made within the parameters of law
and pursuant to the tenets of equity and fair play. The employer's right
to terminate the services of an employee for just or authorized cause
must be exercised in good faith. More importantly, it must not amount
to interfering with, restraining or coercing employees in the exercise of
their right to self-organization because it would amount to, as in this
case, unlawful labor practice under Article 248 of the Labor Code.

314
Page 315

Case Digest by: FAN, PAULON

MARIWASA SIAM CERAMICS, INC. VS. SEC OF LABOR


G.R. No. 183317. December 21, 2009.
Nachura, J.

DOCTRINE:
ARTICLE 240.
Article 234 of the Labor Code merely requires a 20% minimum
membership during the application for union registration. It does not
mandate that a union must maintain the 20% minimum membership
requirement all throughout its existence.

FACTS:
Respondent Samahan Ng Mga Manggagawa Sa Mariwasa Siam
Ceramics, Inc. (SMMSC-Independent) was issued a Certificate of
Registration as a legitimate labor organization by the Department of
Labor and Employment (DOLE), Region IV-A.

Petitioner Mariwasa Siam Ceramics, Inc. filed a Petition for


Cancellation of Union Registration against respondent, claiming that
the latter violated Article 234 of the Labor Code for not complying with
the 20% requirement, and that it committed massive fraud and
misrepresentation in violation of Article 239 of the same code.

The Regional Director of DOLE IV-A issued an Order granting the


petition, revoking the registration of respondent, and delisting it from
the roster of active labor unions. Aggrieved, respondent appealed to
the Bureau of Labor Relations (BLR). In a Decision, the BLR granted
respondent’s appeal. Petitioner filed a Motion for Reconsideration but
the BLR denied it in a Resolution.

ISSUE:
1. Whether or not there was failure to comply with the 20% union
membership requirement; and

2. Whether or not the withdrawal of 31 union members affected


the petition for certification election insofar as the 30% requirement is
concerned

RULING:
1. No. While it is true that the withdrawal of support may be
considered as a resignation from the union, the fact remains that at the
time of the union’s application for registration, the affiants were

315
Page 316

members of respondent and they comprised more than the required


20% membership for purposes of registration as a labor union. Article
234 of the Labor Code merely requires a 20% minimum membership
during the application for union registration. It does not mandate that a
union must maintain the 20% minimum membership requirement all
throughout its existence.

2. On the second issuwhether or not the withdrawal of 31 union


members from NATU affected the petition for certification election
insofar as the 30% requirement is concerned, We reserve the Order of
the respondent Director of the Bureau of Labor Relations, it appearing
undisputably that the 31 union members had withdrawn their support
to the petition before the filing of said petition. It would be otherwise if
the withdrawal was made after the filing of the petition for it would then
be presumed that the withdrawal was not free and voluntary. The
presumption would arise that the withdrawal was procured through
duress, coercion or for valuable consideration. In other words, the
distinction must be that withdrawals made before the filing of the
petition are presumed voluntary unless there is convincing proof to the
contrary, whereas withdrawals made after the filing of the petition are
deemed involuntary.

The reason for such distinction is that if the withdrawal or retraction


is made before the filing of the petition, the names of employees
supporting the petition are supposed to be held secret to the opposite
party. Logically, any such withdrawal or retraction shows voluntariness
in the absence of proof to the contrary. Moreover, it becomes apparent
that such employees had not given consent to the filing of the petition,
hence the subscription requirement has not been met.

When the withdrawal or retraction is made after the petition is filed,


the employees who are supporting the petition become known to the
opposite party since their names are attached to the petition at the time
of filing. Therefore, it would not be unexpected that the opposite party
would use foul means for the subject employees to withdraw their
support.

The bare fact that two signatures appeared twice on the list of
those who participated in the organizational meeting would not, to our
mind, provide a valid reason to cancel respondent’s certificate of
registration. The cancellation of a union’s registration doubtless has an
impairing dimension on the right of labor to self-organization. For fraud
and misrepresentation to be grounds for cancellation of union
registration under the Labor Code, the nature of the fraud and
misrepresentation must be grave and compelling enough to vitiate the
consent of a majority of union members.

316
Page 317

Case Digest by: FAN, PAULON

ELECTROMAT MANUFACTURING AND RECORDING


CORPORATION VS. LAGUNZAD
G.R. No. 172699. July 27, 2011.
Brion, J.

DOCTRINE:
ARTICLE 240.
The intent of the law in imposing lesser requirements in the case
of a branch or local of a registered federation or national union is to
encourage the affiliation of a local union with a federation or national
union in order to increase the local union’s bargaining powers
respecting terms and conditions of labor.

FACTS:
The private respondent Nagkakaisang Samahan ng Manggagawa
ng Electromat-Wasto (union), a charter affiliate of the Workers
Advocates for Struggle, Transformation and Organization (WASTO),
applied for registration with the Bureau of Labor Relations (BLR). The
BLR thereafter issued the union a Certification of Creation of Local
Chapter (equivalent to the certificate of registration of an independent
union), pursuant to Department Order No. (D.O.) 40-03.

The petitioner Electromat Manufacturing and Recording


Corporation (company) filed a petition for cancellation of the union’s
registration certificate, for the union’s failure to comply with Article 234
of the Labor Code. It argued that D.O. 40-03 is an unconstitutional
diminution of the Labor Code’s union registration requirements under
Article 234. Acting Director Ciriaco A. Lagunzad of the Department of
Labor and Employment (DOLE)-National Capital Region dismissed the
petition.

In the appeal by the company, BLR Director Hans Leo J. Cacdac


affirmed the dismissal. The company thereafter sought relief from the
CA through a petition for certiorari, contending that the BLR committed
grave abuse of discretion in affirming the union’s registration despite
its non-compliance with the requirements for registration under Article
234 of the Labor Code. It assailed the validity of D.O. 40-03 which
amended the rules of Book V (Labor Relations) of the Labor Code. It
posited that the BLR should have strictly adhered to the union
registration requirements under the Labor Code, instead of relying on
D.O. 40-03 which it considered as an invalid amendment of the law
since it reduced the requirements under Article 234 of the Labor Code.
It maintained that the BLR should not have granted the union’s

317
Page 318

registration through the issuance of a Certification of Creation of Local


Chapter since the union submitted only the Charter Certificate issued
to it by WASTO.

ISSUE:
Whether or not D.O. 40-03 is a valid exercise of the rule-making
power of the DOLE

RULING:
Yes. Earlier in Progressive Development Corporation v. Secretary,
Department of Labor and Employment, the Court encountered a
similar question on the validity of the old Section 3, Rule II, Book V of
the Rules Implementing the Labor Code which stated:

Union affiliation; direct membership with a national union. - The


affiliate of a labor federation or national union may be a local or chapter
thereof or an independently registered union.

a) The labor federation or national union concerned shall issue a


charter certificate indicating the creation or establishment of a local or
chapter, copy of which shall be submitted to the Bureau of Labor
Relations within thirty (30) days from issuance of such charter
certificate.

e) The local or chapter of a labor federation or national union shall


have and maintain a constitution and by-laws, set of officers and books
of accounts. For reporting purposes, the procedure governing the
reporting of independently registered unions, federations or national
unions shall be observed.

Interpreting these provisions of the old rules, the Court said that
by force of law, the local or chapter of a labor federation or national
union becomes a legitimate labor organization upon compliance with
Section 3, Rule II, Book V of the Rules Implementing the Labor Code,
the only requirement being the submission of the charter certificate to
the BLR. Further, the Court noted that Section 3 omitted several
requirements which are otherwise required for union registration, as
follows:

1) The requirement that the application for registration must be


signed by at least 20% of the employees in the appropriate bargaining
unit;

318
Page 319

2) The submission of officers’ addresses, principal address of the


labor organization, the minutes of organization meetings and the list of
the workers who participated in such meetings;

3) The submission of the minutes of the adoption or ratification of


the constitution and by-laws and the list of the members who
participated in it.

Notwithstanding these omissions, the Court upheld the


government’s implementing policy expressed in the old rules when it
declared in Progressive Development –

Undoubtedly, the intent of the law in imposing lesser requirements


in the case of a branch or local of a registered federation or national
union is to encourage the affiliation of a local union with a federation or
national union in order to increase the local union’s bargaining powers
respecting terms and conditions of labor.

It was this same Section 3 of the old rules that D.O. 40-03 fine-
tuned when the DOLE amended the rules on Book V of the Labor
Code, thereby modifying the government’s implementing policy on the
registration of locals or chapters of labor federations or national
unions.

Pagpalain cannot also allege that Department Order No. 9 is


violative of public policy. x x x [T]he sole function of our courts is to
apply or interpret the laws. It does not formulate public policy, which is
the province of the legislative and executive branches of government.

319
Page 320

Case Digest by: FAN, PAULON

EAGLE RIDGE GOLF AND COUNTRY CLUB VS. CA


G.R. No. 178989. March 18, 2010.
Velasco, JR., J.

DOCTRINE:
ARTICLE 240.
Before their amendment by Republic Act No. 9481 on June 15,
2007, the then governing Art. 234 (on the requirements of registration
of a labor union) and Art. 239 (on the grounds for cancellation of union
registration) of the Labor Code.

FACTS:
At least 20% of Eagle Ridge’s rank-and-file employees—the
percentage threshold required under Article 234(c) of the Labor Code
for union registration—had a meeting where they organized
themselves into an independent labor union, named "Eagle Ridge
Employees Union" (EREU or Union), elected a set of officers, and
ratified their constitution and by-laws.

EREU formally applied for registration and filed before the


Department of Labor and Employment (DOLE) Regional Office IV (RO
IV). In time, DOLE RO IV granted the application and issued EREU
Registration Certificate. The EREU then filed a petition for certification
election in Eagle Ridge Golf & Country Club. Going into specifics,
Eagle Ridge alleged that the EREU declared in its application for
registration having 30 members, when the minutes of its organizational
meeting showed it only had 26 members. The misrepresentation was
exacerbated by the discrepancy between the certification issued by the
Union secretary and president that 25 members actually ratified the
constitution and by-laws and the fact that 26 members affixed their
signatures on the documents, making one signature a forgery.

Finally, Eagle Ridge contended that five employees who attended


the organizational meeting had manifested the desire to withdraw from
the union. The five executed individual affidavits or Sinumpaang
Salaysay, attesting that they arrived late at said meeting which they
claimed to be drinking spree; that they did not know that the documents
they signed on that occasion pertained to the organization of a union;
and that they now wanted to be excluded from the Union. The
withdrawal of the five, Eagle Ridge maintained, effectively reduced the
union membership to 20 or 21, either of which is below the mandatory
minimum 20% membership requirement under Art. 234(c) of the Labor

320
Page 321

Code. Reckoned from 112 rank-and-file employees of Eagle Ridge, the


required number would be 22 or 23 employees.

DOLE Regional Director granted Eagle Ridge’s petition and


delisted EREU from the roster of legitimate labor organizations. EREU
appealed to the BLR, which initially affirmed the order of the Regional
Director, but upon filing of the EREU of a motion for reconsideration it
was reinstated in the roster of legitimate labor organizations. Eagle
Ridge filed a motion for reconsideration but was denied, thus a petition
for certiorari to the CA.

ISSUE:
Whether or not Eagle Ridge’s petition to cancel EREU’s
registration should be denied

RULING:
Yes. To us, Eagle Ridge has not satisfactorily explained its failure
to comply. There was also no fraud in the application.

Eagle Ridge cites the grounds provided under Art. 239(a) and (c)
of the Labor Code for its petition for cancellation of the EREU’s
registration. On the other hand, the Union asserts bona
fide compliance with the registration requirements under Art. 234 of the
Code, explaining the seeming discrepancies between the number of
employees who participated in the organizational meeting and the total
number of union members at the time it filed its registration, as well as
the typographical error in its certification which understated by one the
number of union members who ratified the union’s constitution and by-
laws.

Before their amendment by Republic Act No. 9481 on June 15,


2007, the then governing Art. 234 (on the requirements of registration
of a labor union) and Art. 239 (on the grounds for cancellation of union
registration) of the Labor Code.

A scrutiny of the records fails to show any misrepresentation, false


statement, or fraud committed by EREU to merit cancellation of its
registration. The Union submitted the required documents attesting to
the facts of the organizational meeting on December 6, 2005, the
election of its officers, and the adoption of the Union’s constitution and
by-laws. EREU complied with the mandatory minimum 20%
membership requirement under Art. 234(c). when it had 30 employees
as member when it registered. Any seeming infirmity in the application
and admission of union membership, most especially in cases of
independent labor unions, must be viewed in favor of valid
membership.

321
Page 322

In the issue of the affidavits of retraction executed by six union


members, the probative value of these affidavits cannot overcome
those of the supporting affidavits of 12 union members and their
counsel as to the proceedings and the conduct of the organizational
meeting on December 6, 2005. The DOLE Regional Director and the
BLR OIC Director obviously erred in giving credence to the affidavits
of retraction, but not according the same treatment to the supporting
affidavits. It is settled that affidavits partake the nature of hearsay
evidence, since they are not generally prepared by the affiant but by
another who uses his own language in writing the affiant’s statement,
which may thus be either omitted or misunderstood by the one writing
them. It is required for affiants to re-affirm the contents of their affidavits
during the hearing of the instant case for them to be examined by the
opposing party, i.e., the Union. For their non-presentation, the six
affidavits of retraction are inadmissible as evidence against the Union
in the instant case. Twenty percent (20%) of 112 rank-and-file
employees in Eagle Ridge would require a union membership of at
least 22 employees. When the EREU filed its application for
registration on December 19, 2005, there were clearly 30 union
members.

Thus, when the certificate of registration was granted, there is no


dispute that the Union complied with the mandatory 20% membership
requirement. Prior to their withdrawal, the six employees who retracted
were bona fide union members. With the withdrawal of six union
members, there is still compliance with the mandatory membership
requirement under Art. 234(c), for the remaining 24 union members
constitute more than the 20% membership requirement of 22
employees.

322
Page 323

Case Digests by: GENEVIEVE E. FLORALDE

TAGAYTAY HIGHLAND'S INTERNATIONAL GOLF


CLUB INCORPORATED
vs. TAGAYTAY HIGHLANDS EMPLOYEES UNION-
PGTWO
G.R. No. 142000. January 22, 2003.
Carpio-Morales, J.
DOCTRINE:

GROUNDS OF CANCELLATION OF UNION REGISTRATION


The grounds for cancellation of union registration are provided
for under Article 239 of the Labor Code. The inclusion in a union of
disqualified employees is not among the grounds for cancellation,
unless such inclusion is due to misrepresentation, false statement or
fraud under the circumstances enumerated in Sections (a) and (c) of
Article 239 Article 239 of the Labor Code.

FACTS:
The Tagaytay Highlands Employees Union (THEU) — Philippine
Transport and General Workers Organization (PTGWO), Local
Chapter No. 776, a legitimate labor organization said to represent
majority of the rank-and-file employees of THIGCI, filed a petition for
certification election before the DOLE Mediation-Arbitration Unit,
Regional Branch No. IV. THIGCI opposed THEU's petition for
certification election on the ground that the list of union members
submitted by it was defective and fatally flawed as it included the
names and signatures of supervisors, resigned, terminated and absent
without leave (AWOL) employees, as well as employees of The
Country Club, Inc., a corporation distinct and separate from THIGCI;
and that out of the 192 signatories to the petition, only 71 were actual
rank-and-file employees of THIGCI.THIGCI thus submitted a list of the
names of its 71 actual rank-and-file employees and incorporated a
tabulation showing the number of signatories to said petition whose
membership in the union was being questioned as disqualified. THIGCI
also alleged that some of the signatures in the list of union members
were secured through fraudulent and deceitful means, and submitted
copies of the handwritten denial and withdrawal of some of its
employees from participating in the petition. THEU asserted that it had
complied with all the requirements for valid affiliation and inclusion in
the roster of legitimate labor on account of which it was duly granted a
Certification of Affiliation by DOLE and that Section 5, Rule V of
said Department Order provides that the legitimacy of its registration
cannot be subject to collateral attack, and for as long as there is no
final order of cancellation, it continues to enjoy the rights accorded to
a legitimate organization. THEU thus concluded that under the

323
Page 324

circumstances, the Med-Arbiter should, pursuant to Article 257 of


the Labor Code and Section 11, Rule XI of DOLE Department Order
No. 09, automatically order the conduct of a certification election.
DOLE Med-Arbiter Anastacio Bactin ordered the holding of a
certification election among the rank-and-file employees of THIGCI.
THIGCI appealed to the Office of the DOLE Secretary which set aside
the said Med-Arbiter's Order and dismissed the petition for certification.
Upon Motion for Reconsideration by THEU, DOLE Undersecretary
Rosalinda Dimalipis-Baldoz set aside the resolution dismissing the
petition for certification election. The records of the case were thus
ordered remanded to the Office of the Med-Arbiter for the conduct of
certification election. The Court of Appeals denied THIGCI's Petition
for Certiorari. Hence, the present petition for certiorari.

ISSUE:
Whether or not the inclusion in a union of disqualified employees is
among the grounds for cancellation.

RULING:
No. After a certificate of registration is issued to a union, its legal
personality cannot be subject to collateral attack. It may be questioned
only in an independent petition for cancellation. The grounds for
cancellation of union registration are provided for under Article 239 of
the Labor Code. The inclusion in a union of disqualified employees is
not among the grounds for cancellation, unless such inclusion is due
to misrepresentation, false statement or fraud under the circumstances
enumerated in Sections (a) and (c) of Article 239 Article 239 of
the Labor Code. THEU, having been validly issued a certificate of
registration, should be considered to have already acquired juridical
personality which may not be assailed collaterally. As for petitioner's
allegation that some of the signatures in the petition for certification
election were obtained through fraud, false statement and
misrepresentation, the proper procedure is, for it to file a petition for
cancellation of the certificate of registration, and not to intervene in a
petition for certification election. Regarding the alleged withdrawal of
union members from participating in the certification election, this
Court's following ruling is instructive: [T]he best forum for determining
whether there were indeed retractions from some of the laborers is in
the certification election itself wherein the workers can freely express
their choice in a secret ballot. Suffice it to say that the will of the rank-
and-file employees should in every possible instance be determined
by secret ballot rather than by administrative or quasi-judicial inquiry.
Such representation and certification election cases are not to be taken
as contentious litigations for suits but as mere investigations of a non-
adversary, fact-finding character as to which of the competing unions
represents the genuine choice of the workers to be their sole and
exclusive collective bargaining representative with their employer."

324
Page 325

Case Digests by: GENEVIEVE E. FLORALDE

S.S. VENTURES INTERNATIONAL, INC., vs. S.S.


VENTURES LABOR UNION (SSVLU) and DIR. HANS
LEO CACDAC, in His capacity as Director of the
Bureau of Labor Relations (BLR)

G.R. No. 161690. July 23, 2008.


Velasco, Jr. J.
DOCTRINE:

CANCELLATION OF UNION REGISTRATION

After a labor organization has filed the necessary registration


documents, it becomes mandatory for the BLR to check if the
requirements under Art. 234 of the Labor Code have been sedulously
complied with. The bare fact that three signatures twice appeared on
the list of those who participated in the organizational meeting would
not provide a valid reason to cancel Certificate of Registration No.
RO300-00-02-UR-0003. As the Union tenably explained without
rebuttal from Ventures, the double entries are no more than "normal
human error", effected without malice. Even the labor arbiter who found
for Ventures sided with the Union in its explanation on the absence of
malice. Further, for fraud and misrepresentation [to be grounds for]
cancellation of union registration under Article 239 [of the Labor Code],
the nature of the fraud and misrepresentation must be grave and
compelling enough to vitiate the consent of a majority of union
members.

FACTS:
Petitioner S.S. Ventures International, Inc. (Ventures), a PEZA-
registered export firm is in the business of manufacturing sports
shoes. Respondent S.S. Ventures Labor Union (Union), on the
other hand, is a labor organization registered with the Department
of Labor and Employment (DOLE). The Union filed with DOLE-
Region III a petition for certification election in behalf of the rank-
and-file employees of Ventures. Five hundred forty two (542)
signatures, 82 of which belong to terminated Ventures employees,
appeared on the basic documents supporting the petition. Ventures
filed a Petition to cancel the Union's certificate of registration
invoking the grounds set forth in Article 239 (a) of the Labor Code.
It alleged that the Union deliberately and maliciously included the
names of more or less 82 former employees no longer connected
with Ventures in its list of members who attended the organizational
meeting and in the adoption/ratification of its constitution and by-

325
Page 326

laws and the Union forged the signatures of these 82 former


employees to make it appear they took part in the organizational
meeting and adoption and ratification of the constitution; That the
Union maliciously twice entered the signatures of three persons
namely: Mara Santos, Raymond Balangbang, and Karen Agunos;
That no organizational meeting and ratification actually took place;
and that the Union's application for registration was not supported
by at least 20% of the rank-and-file employees of Ventures, or 418
of the total 2,197-employee complement. Since more or less 82 of
the 500 signatures were forged or invalid, then the remaining valid
signatures would only be 418, which is very much short of the 439
minimum (2197 total employees x 20% = 439.4) required by the
Labor Code. In its Answer with Motion to Dismiss, the Union denied
committing the imputed acts of fraud or misrepresentation. Regional
Director Ana C. Dione of DOLE-Region III found for Ventures. BLR
Director reseversed the decision of Dione. The CA rendered a
Decision, dismissing Ventures' petition. Ventures' motion for
reconsideration met a similar fate. Hence, this petition for review
under Rule 45.

ISSUE:
Whether or not there was fraud or misrepresentation on the part of
the Union sufficient to justify cancellation of its registration.
RULING:
No, the mostly undated written statements submitted by
Ventures on March 20, 2001, or seven months after it filed its
petition for cancellation of registration, partake of the nature of
withdrawal of union membership executed after the Union's filing of
a petition for certification election on March 21, 2000. The
employees' withdrawal from a labor union made before the filing of
the petition for certification election is presumed voluntary, while
withdrawal after the filing of such petition is considered to be
involuntary and does not affect the same. Now then, if a withdrawal
from union membership done after a petition for certification election
has been filed does not vitiate such petition, is it not but logical to
assume that such withdrawal cannot work to nullify the registration
of the union? Thus, the affidavits of retraction of the 82 members
had no evidentiary weight.
After a labor organization has filed the necessary registration
documents, it becomes mandatory for the BLR to check if the
requirements under Art. 234 of the Labor Code have been
sedulously complied with. Thus, the issuance to the Union of
Certificate of Registration No. RO300-00-02-UR-0003 necessarily
implies that its application for registration and the supporting
documents thereof are prima facie free from any vitiating
irregularities. Second, the assailed inclusion of the said 82
individuals to the meeting and proceedings adverted to is not really

326
Page 327

fatal to the Union's cause for, as determined by the BLR, the


allegations of falsification of signatures or misrepresentation with
respect to these individuals are without basis. The procedure for
acquiring or losing union membership and the determination of who
are qualified or disqualified to be members are matters internal to
the union and flow from its right to self-organization. The relevancy
of the 82 individuals' active participation in the Union's
organizational meeting and the signing ceremonies thereafter
comes in only for purposes of determining whether or not the Union,
even without the 82, would still meet what Art. 234 (c) of the Labor
Code requires to be submitted. The BLR, based on its official
records, answered the poser in the affirmative. The bare fact that
three signatures twice appeared on the list of those who participated
in the organizational meeting would not provide a valid reason to
cancel Certificate of Registration No. RO300-00-02-UR-0003. As
the Union tenably explained without rebuttal from Ventures, the
double entries are no more than "normal human error", effected
without malice. Even the labor arbiter who found for Ventures sided
with the Union in its explanation on the absence of malice. Further,
for fraud and misrepresentation [to be grounds for] cancellation of
union registration under Article 239 [of the Labor Code], the nature
of the fraud and misrepresentation must be grave and compelling
enough to vitiate the consent of a majority of union members.
Ventures should not interfere in the certification election by actively
and persistently opposing the certification election of the Union. A
certification election is exclusively the concern of employees and
the employer lacks the legal personality to challenge it. In fact,
jurisprudence frowns on the employer's interference in a
certification election for such interference unduly creates the
impression that it intends to establish a company union.

327
Page 328

Case Digests by: GENEVIEVE E. FLORALDE

THE HERITAGE HOTEL MANILA, acting through its owner,


GRAND PLAZA HOTEL CORPORATION vs. NATIONAL
UNION OF WORKERS IN THE HOTEL, RESTAURANT AND
ALLIED INDUSTRIES-
HERITAGE HOTEL MANILA SUPERVISORS CHAPTER
(NUWHRAIN-HHMSC)

G.R. No. 178296. January 12, 2011.


Nachura, J.
DOCTRINE:

CANCELLATION OF UNION REGISTRATION


Articles 238 and 239 of the Labor Code give the Regional
Director ample discretion in dealing with a petition for cancellation of
a union's registration, particularly, determining whether the union still
meets the requirements prescribed by law. It is sufficient to give the
Regional Director license to treat the late filing of required documents
as sufficient compliance with the requirements of the law.

FACTS:
Respondent filed with the Department of Labor and
Employment-National Capital Region (DOLE-NCR) a petition for
certification election. The Med-Arbiter granted the petition and
ordered the holding of a certification election. On appeal, the DOLE
Secretary affirmed the Med-Arbiter's order and remanded the case
to the Med-Arbiter for the holding of a pre-election conference but it
was not held as initially scheduled so petitioner moved to archive or
to dismiss the petition due to alleged repeated non-appearance of
respondent. The latter agreed to suspend proceedings until further
notice. The pre-election conference resumed but subsequently,
petitioner discovered that respondent had failed to submit to the
Bureau of Labor Relations (BLR) its annual financial report for
several years and the list of its members since it filed its registration
papers in 1995. Consequently, petitioner filed a Petition for
Cancellation of Registration of respondent, on the ground of the
non-submission of the said documents. Petitioner prayed that
respondent's Certificate of Creation of Local/Chapter be cancelled
and its name be deleted from the list of legitimate labor
organizations. It further requested the suspension of the certification
election proceedings. Petitioner reiterated its request by filing a
Motion to Dismiss or Suspend the [Certification Election]
Proceedings. Nevertheless, the certification election pushed
through and respondent emerged as the winner. Petitioner filed a

328
Page 329

Protest with Motion to Defer Certification of Election Results and


Winner and then prayed that the certification of the election results
and winner be deferred until the petition for cancellation shall have
been resolved, and that respondent's members who held
confidential or managerial positions be excluded from the
supervisors' bargaining unit. Respondent prayed for the dismissal
of the petition. The Med-Arbiter dismissed petitioner's protest, and
certified respondent as the sole and exclusive bargaining agent of
all supervisory employees. Petitioner subsequently appealed the
said Order to the DOLE Secretary but was later dismissed.
Petitioner moved for reconsideration, but the motion was also
denied. In the meantime, Regional Director Alex E. Maraan
(Regional Director Maraan) of DOLE-NCR finally resolved the
petition for cancellation of registration. While finding that respondent
had indeed failed to file financial reports and the list of its members
for several years, he, nonetheless, denied the petition, ratiocinating
that freedom of association and the employees' right to self-
organization are more substantive considerations. Aggrieved,
petitioner appealed the decision but the same was dismissed as
well as the subsequent petition for certiorari.
In this petition, petitioner argues that respondent's registration
as a legitimate labor union should be cancelled. Petitioner posits
that once it is determined that a ground enumerated in Article 239
of the Labor Code is present, cancellation of registration should
follow; it becomes the ministerial duty of the Regional Director to
cancel the registration of the labor organization, hence, the use of
the word "shall." Petitioner points out that the Regional Director has
admitted in its decision that respondent failed to submit the required
documents for a number of years; therefore, cancellation of its
registration should have followed as a matter of course.
ISSUE:
Whether or not it becomes the ministerial duty of the Regional
Director to cancel the registration of the labor organization it is
determined that a ground enumerated in Article 239 of the Labor
Code is present.
RULING:
No. Articles 238 and 239 of the Labor Code give the Regional Director
ample discretion in dealing with a petition for cancellation of a union's
registration, particularly, determining whether the union still meets the
requirements prescribed by law. It is sufficient to give the Regional
Director license to treat the late filing of required documents as
sufficient compliance with the requirements of the law. With the
submission of the required documents by respondent, the purpose of
the law has been achieved, though belatedly. The union members
and, in fact, all the employees belonging to the appropriate
bargaining unit should not be deprived of a bargaining agent, merely

329
Page 330

because of the negligence of the union officers who were responsible


for the submission of the documents to the BLR.
The Labor Code's provisions on cancellation of union
registration and on reportorial requirements have been recently
amended by Republic Act (R.A.) No. 9481, An Act Strengthening
the Workers' Constitutional Right to Self-Organization, Amending
for the Purpose Presidential Decree No. 442, As Amended,
Otherwise Known as the Labor Code of the Philippines, which
lapsed into law on May 25, 2007 and became effective on June 14,
2007. Thus, R.A. No. 9481 amended Article 239. The court quoted
with approval the DOLE Secretary's rationale for denying the
petition: It is undisputed that appellee failed to submit its annual
financial reports and list of individual members in accordance with
Article 239 of the Labor Code. However, the existence of this
ground should not necessarily lead to the cancellation of union
registration. Article 239 recognizes the regulatory authority of the
State to exact compliance with reporting requirements. Yet there is
more at stake in this case than merely monitoring union activities
and requiring periodic documentation thereof. The more substantive
considerations involve the constitutionally guaranteed freedom of
association and right of workers to self-organization. Moreover,
submission of the required documents is the duty of the officers of
the union. It would be unreasonable for this Office to order the
cancellation of the union and penalize the entire union membership
on the basis of the negligence of its officers.

330
Page 331

Case Digests by: GENEVIEVE E. FLORALDE

REPUBLIC OF THE PHILIPPINES, represented by


Department of Labor and Employment (DOLE)
vs. KAWASHIMA TEXTILE MFG., PHILIPPINES, INC

G.R. No. 160352. July 23, 2008.


Austria-Martinez, J.
DOCTRINE:

GROUNDS OF CANCELLATION OF UNION REGISTRATION


The inclusion in a union of disqualified employees is not among the
grounds for cancellation, unless such inclusion is due to
misrepresentation, false statement or fraud under the circumstances
enumerated in Sections (a) and (c) of Article 239 of the Labor Code.
FACTS:
KFWU filed with DOLE Regional Office No. IV, a Petition for
Certification Election to be conducted in the bargaining unit
composed of 145 rank-and-file employees of respondent. Attached
to its petition are a Certificate of Creation of Local/Chapter, stating
that it [KFWU] submitted to said office a Charter Certificate issued
to it by the national federation Phil. Transport & General Workers
Organization (PTGWO), and a Report of Creation of Local/Chapter.
Respondent filed a Motion to Dismiss the petition on the ground that
KFWU did not acquire any legal personality because its
membership of mixed rank-and-file and supervisory employees
violated Article 245 of the Labor Code, and because of its failure to
submit its books of account. Med-Arbiter Bactin found KFWU's legal
personality defective and dismissed its petition for certification
election. KFWU appealed to the DOLE which granted the same.
Respondent filed a Motion for Reconsideration but the DOLE
denied the same. The CA reversed and MR was denied.
ISSUE:
Whether or not a mixed membership of rank-and-file and
supervisory employees in a union is a ground for the dismissal of a
petition for certification election in view of the amendment brought
about by D.O. 9, series of 1997, which deleted the phraseology in
the old rule that "the appropriate bargaining unit of the rank-and-file
employee shall not include the supervisory employees and/or
security guards; Whether or not the legitimacy of a duly registered
labor organization can be collaterally attacked in a petition for a
certification election through a motion to dismiss filed by an
employer such as Kawashima Textile Manufacturing Phils., Inc.
RULING:

331
Page 332

The law and rules in force at the time of the filing by KFWU of
the petition for certification election on January 24, 2000 are R.A.
No. 6715, amending Book V of Presidential Decree (P.D.) No.
442 (Labor Code), as amended, and the Rules and Regulations
Implementing R.A. No. 6715, as amended by Department Order No.
9, series of 1997.
It is within the parameters of R.A. No. 6715 and the
Implementing Rules that the Court resolved the two issues raised
by petitioner. If there is one constant precept in our labor laws — be
it Commonwealth Act No. 213 (1936), R.A. No. 875 (1953), P.D. No.
442 (1974), Executive Order (E.O.) No. 111 (1986) or R.A. No. 6715
(1989) — it is that only a legitimate labor organization may exercise
the right to be certified as the exclusive representative of all the
employees in an appropriate collective bargaining unit for purposes
of collective bargaining. What has varied over the years has been
the degree of enforcement of this precept, as reflected in the shifting
scope of administrative and judicial scrutiny of the composition of a
labor organization before it is allowed to exercise the right of
representation. One area of contention has been the composition of
the membership of a labor organization, specifically whether there
is a mingling of supervisory and rank-and-file employees and how
such questioned mingling affects its legitimacy. In Tagaytay
Highlands Int’l. Golf Club, Inc. v. Tagaytay Highlands Employees
Union-PGTWO, the Court abandoned the view in Toyota and
Dunlop and reverted to its pronouncement in Lopez that while there
is a prohibition against the mingling of supervisory and rank-and-file
employees in one labor organization, the Labor Code does not
provide for the effects thereof. Thus, the Court held that after a labor
organization has been registered, it may exercise all the rights and
privileges of a legitimate labor organization. Any mingling between
supervisory and rank-and-file employees in its membership cannot
affect its legitimacy for that is not among the grounds for
cancellation of its registration, unless such mingling was brought
about by misrepresentation, false statement or fraud under Article
239 of the Labor Code. In San Miguel Corp. (Mandaue Packaging
Products Plants) v. Mandaue Packing Products Plants-San Miguel
Packaging Products-San Miguel Corp. Monthlies Rank-and-File
Union-FFW, the Court explained that since the 1997 Amended
Omnibus Rules does not require a local or chapter to provide a list
of its members, it would be improper for the DOLE to deny
recognition to said local or chapter on account of any question
pertaining to its individual members. More to the point is Air
Philippines Corporation v. Bureau of Labor Relations, Court therein
reiterated its ruling in Tagaytay Highlands that the inclusion in a
union of disqualified employees is not among the grounds for
cancellation, unless such inclusion is due to misrepresentation,
false statement or fraud under the circumstances enumerated in

332
Page 333

Sections (a) and (c) of Article 239 of the Labor Code. All said, while
the latest issuance is R.A. No. 9481, the 1997 Amended Omnibus
Rules, as interpreted by the Court in Tagaytay Highlands, San
Miguel and Air Philippines, had already set the tone for
it. Toyota and Dunlop no longer hold sway in the present altered
state of the law and the rules. Consequently, the Court reversed the
ruling of the CA and reinstates that of the DOLE granting the petition
for certification election of KFWU.

333
Page 334

Case Digests by: GENEVIEVE E. FLORALDE

DEL PILAR ACADEMY, EDUARDO ESPEJO and


ELISEO OCAMPO, JR. vs. DEL PILAR ACADEMY
EMPLOYEES UNION

G.R. No. 170112 April 30, 2008.


Nachura, J.
DOCTRINE:
RIGHTS AND CONDITIONS OF MEMBERSHIP IN A LABOR
ORGANIZATION; COLLECTION OF AGENCY FEES AND VALID
CHECK-OFF
The collection of agency fees in an amount equivalent to union
dues and fees, from employees who are not union members, is
recognized by Article 248 (e) of the Labor Code.
When so stipulated in a collective bargaining agreement or
authorized in writing by the employees concerned, the Labor Code
and its Implementing Rules recognize it to be the duty of the
employer to deduct the sum equivalent to the amount of union dues,
as agency fees, from the employees' wages for direct remittance to
the union. The system is referred to as check off. No requirement of
written authorization from the non-union employees is necessary if
the non-union employees accept the benefits resulting from the
CBA.
FACTS:
Respondent Del Pilar Academy Employees Union (the
UNION) is the certified collective bargaining representative of
teaching and non-teaching personnel of petitioner Del Pilar
Academy (DEL PILAR), an educational institution operating in Imus,
Cavite. On September 15, 1994, the UNION and DEL PILAR
entered into a Collective Bargaining Agreement (CBA) granting
salary increase and other benefits to the teaching and non-teaching
staff. The UNION then assessed agency fees from non-union
employees, and requested DEL PILAR to deduct said assessment
from the employees' salaries and wages. DEL PILAR, however,
refused to effect deductions claiming that the non-union employees
were not amenable to it.
The UNION negotiated for the renewal of the CBA. DEL
PILAR, however, refused to renew the same unless the provision
regarding entitlement to two (2) months summer vacation leave with
pay will be amended by limiting the same to teachers, who have
rendered at least three (3) consecutive academic years of
satisfactory service. The UNION objected to the proposal claiming

334
Page 335

diminution of benefits. DEL PILAR refused to sign the CBA,


resulting in a deadlock. The UNION requested DEL PILAR to submit
the case for voluntary arbitration, but the latter allegedly refused,
prompting the UNION to file a case for unfair labor practice with the
Labor Arbiter. Labor Arbiter Nieves V. De Castro dismissed the
charge of unfair labor. The National Labor Relations Commission
(NLRC) affirmed the Arbiter's ruling. CA affirmed with modification
the resolutions of the NLRC.
DEL PILAR admitted its failure to deduct the agency fees from
the salaries of non-union employees, but justifies the non-deduction
by the absence of individual written authorization. It posits that
Article 248 (e) is inapplicable considering that its employees derived
no benefits from the CBA. The annual salary increase of its
employee is a benefit mandated by law, and not derived from the
CBA. According to DEL PILAR, the Department of Education,
Culture and Sports (DECS) required all educational institutions to
allocate at least 70% of tuition fee increases for the salaries and
other benefits of teaching and non-teaching personnel; that even
prior to the execution of the CBA in September 1994, DEL PILAR
have already granting annual salary increases to its employees.
Besides, the non-union employees objected to the deduction;
hence, a written authorization is indispensable to effect a valid
check off. DEL PILAR urges this Court to reverse the CA ruling
insofar as it orders the deduction of agency fees from the salaries
of non-union employees, arguing that such conclusion proceeds
from a misplaced premise that the salary increase has risen from
the CBA.
ISSUE:
Whether or not the UNION is entitled to collect agency fees from
non-union members, and if so, whether an individual written
authorization is necessary for a valid check off.

RULING:
The collection of agency fees in an amount equivalent to union
dues and fees, from employees who are not union members, is
recognized by Article 248 (e) of the Labor Code.
When so stipulated in a collective bargaining agreement or
authorized in writing by the employees concerned, the Labor Code
and its Implementing Rules recognize it to be the duty of the
employer to deduct the sum equivalent to the amount of union dues,
as agency fees, from the employees' wages for direct remittance to
the union. The system is referred to as check off. No requirement of
written authorization from the non-union employees is necessary if
the non-union employees accept the benefits resulting from the
CBA.

335
Page 336

Contrary to what DEL PILAR wants to portray, the grant of


annual salary increase is not the only provision in the CBA that
benefited the non-union employees. The UNION has negotiated for
other benefits, namely, limitations on teaching assignments to 23
hours per week, additional compensation for overload units or
teaching assignments in excess of the 23 hour per week limit, and
payment of longevity pay. It has also negotiated for entitlement to
summer vacation leave with pay for two (2) months for teaching staff
who have rendered six (6) consecutive semesters of service. For
the non-teaching personnel, the UNION worked for their entitlement
to fifteen (15) days leave with pay. These provisions in the CBA
surely benefits the non-union employees, justifying the collection of,
and the UNION's entitlement to, agency fees.
Accordingly, no requirement of written authorization from the
non-union employees is needed to effect a valid check off. Article
248 (e) makes it explicit that Article 241, paragraph (o), requiring
written authorization is inapplicable to non-union members,
especially in this case where the non-union employees receive
several benefits under the CBA. As explained by this Court in Holy
Cross of Davao College, Inc. v. Hon. Joaquin: “The employee's
acceptance of benefits resulting from a collective bargaining
agreement justifies the deduction of agency fees from his pay and
the union's entitlement thereto. In this aspect, the legal basis of the
union's right to agency fees is neither contractual nor statutory, but
quasi-contractual, deriving from the established principle that non-
union employees may not unjustly enrich themselves by benefiting
from employment conditions negotiated by the bargaining union” By
this jurisprudential yardstick, this Court held that the CA did not err
in upholding the UNION's right to collect agency fees.

336
Page 337

Case Digests by: GENEVIEVE E. FLORALDE

EDUARDO J. MARIÑO, JR., MA. MELVYN P. ALAMIS,


NORMA P. COLLANTES, and FERNANDO PEDROSA
vs. GIL Y. GAMILLA, RENE LUIS TADLE, NORMA S.
CALAGUAS, MA. LOURDES C. MEDINA, EDNA B.
SANCHEZ, REMEDIOS GARCIA, MAFEL YSRAEL,
ZAIDA GAMILLA, and AURORA DOMINGO

G.R. No. 149763. July 07, 2009.


Chico-Nazario, J.
DOCTRINE:
RIGHTS AND CONDITIONS OF MEMBERSHIP IN A LABOR
ORGANIZATION; CHECK-OFF
Article 222 (b) of the Labor Code, as amended, prohibits the
payment of attorney's fees only when it is effected through forced
contributions from the employees from their own funds as
distinguished from union funds. Hence, the general rule is that
attorney's fees, negotiation fees, and other similar charges may only
be collected from union funds, not from the amounts that pertain to
individual union members. As an exception to the general rule,
special assessments or other extraordinary fees may be levied upon
or checked off from any amount due an employee for as long as
there is proper authorization by the employee. A check-off is a
process or device whereby the employer, on agreement with the
Union, recognized as the proper bargaining representative, or on
prior authorization from the employees, deducts union dues or
agency fees from the latter's wages and remits them directly to the
Union.
FACTS:
(1) Case No. NCR-OD-M-9412-022
At the time when the numerous controversies in the instant
case first came about, petitioners were among the executive officers
and directors (collectively called the Mariño Group) of the University
of Sto. Tomas Faculty Union (USTFU), a labor union duly organized
and registered and the bargaining representative of the faculty
members of the University of Santo Tomas (UST). Respondents
were UST professors and USTFU members.
The 1986 Collective Bargaining Agreement (CBA) between
UST and USTFU expired on 31 May 1988. Thereafter, bargaining
negotiations ensued between UST and the Mariño Group, which
represented USTFU. As the parties were not able to reach an
agreement despite their earnest efforts, a bargaining deadlock was
declared and USTFU filed a notice of strike. Subsequently, then

337
Page 338

Secretary of the Department of Labor and Employment (DOLE)


Franklin Drilon assumed jurisdiction over the dispute. The DOLE
Secretary issued an Order, laying the terms and conditions for a
new CBA between the UST and USTFU. In accordance with said
Order, the UST and USTFU entered into a CBA in 1991, which was
to be effective for the period of 1 June 1988 to 31 May 1993
(hereinafter 1988-1993 CBA). In keeping with Article 253-A of
the Labor Code, as amended, the economic provisions of the 1988-
1993 CBA were subject to renegotiation for the fourth and fifth
years. USTFU, through its President, petitioner Atty. Mariño, wrote
a to the UST Treasurer requesting the release to the union of the
sum of P4.2 million, which was 10% of the P42 million economic
benefits package granted by the MOA to faculty members belonging
to the collective bargaining unit. The P4.2 million was sought by
USTFU in consideration of its efforts in obtaining the said P42
million economic benefits package. UST remitted the sum of P4.2
million to USTFU on 9 October 1992.
After deducting from the P42 million economic benefits
package the P4.2 million check-off to USTFU, the amounts owed to
UST, and the salary increases and bonuses of the covered faculty
members, a net amount of P6,389,145.04 remained. The remaining
amount was distributed to the faculty members on 18 November
1994. Thereafter, respondents filed with the Med-Arbiter, DOLE-
National Capital Region (NCR), a Complaint for the expulsion of the
Mariño Group as USTFU officers and directors. Respondents
alleged in their Complaint that the Mariño Group violated the rights
and conditions of membership in USTFU, particularly by: 1)
investing the unspent balance of the P42 million economic benefits
package given by UST without prior approval of the general
membership; 2) simultaneously holding elections viva voce; 3)
ratifying the CBA involving the P42 million economic benefits
package; and 4) approving the attorney's/agency fees worth P4.2
million in the form of check-off. Respondents prayed that the Mariño
Group be declared jointly and severally liable for refunding all
collected attorney's/agency fees from individual members of
USTFU and the collective bargaining unit; and that, after due
hearing, the Mariño group be expelled as USTFU officers and
directors.
(2) Case No. NCR-OD-M-9510-028
On 16 December 1994, UST and USTFU, represented by the
Mariño Group, entered into a new CBA, effective 1 June 1993 to 31
May 1998 (1993-1998 CBA). This new CBA was registered with the
DOLE on 20 February 1995. filed with the Med-Arbiter, DOLE-NCR,
on 18 October 1995, another Complaint against the Mariño Group
for violation of the rights and conditions of union membership, which
primarily sought to invalidate certain provisions of the 1993-1998

338
Page 339

CBA negotiated by the Mariño Group for USTFU and the


registration of said CBA with the DOLE.
(3) Case No. NCR-OD-M-9610-001
On 24 September 1996, petitioner Norma Collantes, as
USTFU Secretary-General, posted notices in some faculty rooms at
UST, informing the union members of a general for the election of
new USTFU officers. The following day, respondents wrote a
letter to the USTFU Committee on Elections, urging the latter to re-
schedule the elections to ensure a free, clean, honest, and orderly
election and to afford the union members the time to prepare
themselves for the same. The USTFU Committee on Elections
failed to act positively on respondents' letter, and neither did they
adopt and promulgate the rules and regulations for the conduct of
the scheduled election. Thus, respondents filed with the Med-
Arbiter, DOLE-NCR, an Urgent Ex-Parte Petition/Complaint.
(4) Case No. NCR-OD-M-9610-016
Also on 4 October 1996, the UST Secretary General headed
a general faculty assembly attended by USTFU members, as well
as USTFU non-members, but who were members of the collective
bargaining unit. During said assembly, respondents were among
the elected officers of USTFU (collectively referred to as the Gamilla
Group). Petitioners filed with the Med-Arbiter, DOLE-NCR, a
Petition seeking injunctive reliefs and the nullification of the results
of the 4 October 1994 election. The Court promulgated its Decision
in G.R. No. 131235, affirming the BLR resolution which ruled that
the purported election of USTFU officers held on 4 October 1996
was void for violating the Constitution and By-Laws of the union.
(5) Case No. NCR-OD-M-9611-009
On 15 November 1996, respondents filed before the Med-
Arbiter, DOLE-NCR, a fourth Complaint/Petition against the Mariño
Group, as well as the Philippine Foundation for the Advancement of
the Teaching Profession, Inc., Security Bank Corporation, and Bank
of the Philippine Islands. They claimed in their latest
Complaint/Petition that they were the legitimate USTFU officers,
having been elected on 4 October 1996. In the meantime, the
election of USTFU officers was held as scheduled on 14 January
2000, in which the Gamilla Group claimed victory. The Gamilla
group, as the new USTFU officers, entered into a Memorandum of
Agreement with the UST, which provided for the economic benefits
to be granted to the faculty members of the UST for the years 1999-
2001. Said Agreement was ratified by the USTFU members on 9
March 2000.
Petitioners' Motion for Partial Reconsideration of the foregoing
Decision was denied by the BLR. CA ruled in favor of respondents.
Petitioners elevated the case to Supreme Court. Petitioners

339
Page 340

contend that the P4.2 million check-off, from the P42 million
economic benefits package, was lawfully made since the
requirements of Article 222 (b) of the Labor Code, as amended,
were complied with by the Mariño Group. The individual paychecks
of the covered faculty employees were not reduced and the P4.2
million deducted from the P42 million economic benefits package
became union funds, which were then used to pay attorney's fees,
negotiation fees, and similar charges arising from the CBA. In
addition, the P4.2 million constituted a special assessment upon the
USTFU members, the requirements for which were properly
observed. The special assessment was authorized in writing by the
general membership of USTFU during a meeting in which it was
included as an item in the agenda. Petitioners fault the Court of
Appeals for disregarding the authorization of the special
assessment by USTFU members. There is no law that prohibits the
insertion of a written authorization for the special assessment in the
same instrument for the ratification of the 10 September 1992 MOA.
Neither is there a law prescribing a particular form that needs to be
accomplished for the authorization of the special assessment. The
faculty members who signed the ratification of the MOA, which
included the authorization for the special assessment, have high
educational attainment, and there is ample reason to believe that
they affixed their signatures thereto with full comprehension of what
they were doing.
ISSUE:
Whether or not the 10% check-off collected by the Mariño Group from
the P42 million economic benefits package is legal.

RULING:
No. Article 222 (b) of the Labor Code, as amended, prohibits the
payment of attorney's fees only when it is effected through forced
contributions from the employees from their own funds as
distinguished from union funds. Hence, the general rule is that
attorney's fees, negotiation fees, and other similar charges may only
be collected from union funds, not from the amounts that pertain to
individual union members. As an exception to the general rule,
special assessments or other extraordinary fees may be levied upon
or checked off from any amount due an employee for as long as
there is proper authorization by the employee.A check-off is a
process or device whereby the employer, on agreement with the
Union, recognized as the proper bargaining representative, or on
prior authorization from the employees, deducts union dues or
agency fees from the latter's wages and remits them directly to the
Union.

340
Page 341

In this case the. Court held that the P42 million economic benefits
package granted by UST did not constitute union funds from
whence the P4.2 million could have been validly deducted as
attorney's fees. The P42 million economic benefits package was not
intended for the USTFU coffers, but for all the members of the
bargaining unit USTFU represented, whether members or non-
members of the union. A close reading of the terms of the MOA
reveals that after the satisfaction of the outstanding obligations of
UST under the 1986 CBA, the balance of the P42 million was to be
distributed to the covered faculty members of the collective
bargaining unit in the form of salary increases, returns on paycheck
deductions; and increases in hospitalization, educational, and
retirement benefits, and other economic benefits. The deduction of
the P4.2 million, as alleged attorney's/agency fees, from the P42
million economic benefits package effectively decreased the share
from said package accruing to each member of the collective
bargaining unit.
Petitioners' line of argument — that the amount of P4.2 million
became union funds after its deduction from the P42 million
economic benefits package and, thus, could already be used to pay
attorney's fees, negotiation fees, or similar charges from the CBA
— is absurd. Petitioners' reasoning is evidently flawed since the
attorney's fees may only be paid from union funds; yet the amount
to be used in paying for the same does not become union funds until
it is actually deducted as attorney's fees from the benefits awarded
to the employees. It is just a roundabout argument. What the law
requires is that the funds be already deemed union funds even
before the attorney's fees are deducted or paid therefrom; it does
not become union funds after the deduction or payment. To rule
otherwise will also render the general prohibition stated in Article
222 (b) nugatory, because all that the union needs to do is to deduct
from the total benefits awarded to the employees the amount
intended for attorney's fees and, thus, "convert" the latter to union
funds, which could then be used to pay for the said attorney's fees.
The Court further determines that the requisites for a valid levy
and check-off of special assessments, laid down by Article 241 (n)
and (o), respectively, of the Labor Code, as amended, have not
been complied with in the case at bar. To recall, these requisites
are: (1) an authorization by a written resolution of the majority of all
the union members at the general membership meeting duly called
for the purpose; (2) secretary's record of the minutes of the meeting;
and (3) individual written authorization for check-off duly signed by
the employee concerned. Additionally, Section 5, Rule X of the
USTFU Constitution and By-Laws mandates that: Section
5. Special assessments or other extraordinary fees such as for
payment of attorney's fees shall be made only upon a resolution
duly ratified by the general membership by secret balloting. In an

341
Page 342

attempt to comply with the foregoing requirements, the Mariño


Group caused the majority of the general membership of USTFU to
individually sign a document, which embodied the ratification of the
MOA between UST and USTFU, dated 10 September 1992, as well
as the authorization for the check-off of P4.2 million, from the P42
million economic benefits package, as payment for attorney's fees.
As held by the Court of Appeals, however, the said documents
constitute unsatisfactory compliance with the requisites set forth in
the Labor Code, as amended, and in the USTFU Constitution and
By-Laws, even though individually signed by a majority of USTFU
members. The inclusion of the authorization for a check-off of union
dues and special assessments for the Labor Education Fund and
attorney's fees, in the same document for the ratification of the 10
September 1992 MOA granting the P42 million economic benefits
package, necessarily vitiated the consent of USTFU members. For
sure, it is fairly reasonable to assume that no individual member of
USTFU would casually turn down the substantial and lucrative
award of P42 million in economic benefits under the MOA. However,
there was no way for any individual union member to separate his
or her consent to the ratification of the MOA from his or her
authorization of the check-off of union dues and special
assessments. As it were, the ratification of the MOA carried with it
the automatic authorization of the check-off of union dues and
special assessments in favor of the union. Such a situation militated
against the legitimacy of the authorization for the P4.2 million check-
off by a majority of USTFU membership. Although the law does not
prescribe a particular form for the written authorization for the levy
or check-off of special assessments, the authorization must, at the
very least, embody the genuine consent of the union member. The
failure of the Mariño Group to strictly comply with the requirements
set forth by the Labor Code, as amended, and the USTFU
Constitution and By-Laws, invalidates the questioned special
assessment. Substantial compliance is not enough in view of the
fact that the special assessment will diminish the compensation of
the union members. Their express consent is required, and this
consent must be obtained in accordance with the steps outlined by
law, which must be followed to the letter. No shortcuts are allowed.

342
Page 343

Case Digests by: GENEVIEVE E. FLORALDE

ERGONOMICS SYSTEMS PHIL INC. vs. ENASE

G.R. No. 195163. December 13, 2017.


Martires, J.
DOCTRINE:
AFFILIATION AND DISAFFILIATION: A local union does not
owe its existence to the federation with which it is affiliated. It is a
separate and distinct voluntary association owing its creation to the
will of its members. Mere affiliation does not divest the local union
of its own personality, neither does it give the mother federation the
license to act independently of the local union. It only gives rise to a
contract of agency, where the former acts in representation of the
latter. Hence, local unions are considered principals while the
federation is deemed to be merely their agent.
FACTS:

Respondents were union officers and members of Ergonomic System


Employees Union-Workers Alliance Trade Unions (local union). The
local union entered into a Collective Bargaining
Agreement (CBA) with petitioner Ergonomic Systems Philippines,
Inc. (ESPI), which was valid for five (5) years or until October 2004.
The local union, which was affiliated with Workers Alliance Trade
Unions-Trade Union Congress of the Philippines (Federation), was
not independently registered. Thus, on 15 November 2001, before
the CBA expired, the union officers secured the independent
registration of the local union with the Regional Office of the
Department of Labor and Employment (DOLE). Later on, the union
officers were charged before the Federation and investigated for
attending and participating in other union's seminars and activities
using union leaves without the knowledge and consent of the
Federation and ESPI as well as in initiating and conspiring in the
disaffiliation before the freedom period. The the Federation rendered
a decision finding respondents-union guilty of disloyalty so they were
penalized with immediate expulsion from the Federation. The local
union then filed a notice of strike with the National Conciliation and
Mediation Board (NCMB). The local union staged a series of noise
barrage and "slow down" activities. Meanwhile, 40 union members
refused to submit their Daily Production Reports (DPRs). 28 union
members abandoned their work and held a picket line outside the
premises of ESPI. Certain union members did not report for work
without official leave. Subsequently, for refusal to submit DPRs and

343
Page 344

for abandonment, respondents-union members were issued letters of


termination. The respondents then filed a complaint for illegal
dismissal and unfair labor practice against ESPI, Phillip C. Ng, and
Ma. Lourminda O. Ng (petitioners). The LA ordered all the
respondents to return to work but without back wages. Unconvinced,
petitioners and respondents appealed before the NLRC but the
NLRC affirmed the ruling of the LA. The petition was partially granted
by the CA. Aggrieved, petitioners and respondents moved for
reconsideration but the same was denied by the CA in a resolution
hence this petition. Petitioners aver that the respondents violated the
union security clause under the CBA; that their termination was
effected in response to the Federation's demand to dismiss them; that
they did not comply with the requisites of a valid strike; that they
refused to submit their DPRs and abandoned their work; and that the
award of separation pay had no basis because the respondents had
been legally dismissed from their employment. The respondents
counter that they were not legally terminated because the grounds
relied upon by the petitioners were non-existent; that as ruled by the
NLRC, they merely disaffiliated from the Federation but they were not
disloyal to the local union; that reinstatement is not physically
impossible because it was the industrial building owned by Ergo
Contracts Philippines, Inc. that was gutted down by fire, not that of
ESPI; that even if the manufacturing plant of ESPI was indeed
destroyed by fire, the petitioners have other offices around the
country where the respondents may be reinstated; and that having
failed to comply with the order to reinstate them and having ceased
operations, the petitioners must be ordered to pay their separation
pay.

ISSUE:
Whether or not the respondents’ dismissal from employment was
valid.
RULING:
In the present case, respondents-union officers stand to be
dismissed as they conducted a strike despite knowledge that a
strike vote had not yet been approved by majority of the union and
the corresponding strike vote report had not been submitted to the
NCMB. With respect to respondents-union members, the petitioners
merely alleged that they committed illegal acts during the strike such
as obstruction of ingress to and egress from the premises of ESPI
and execution of acts of violence and intimidation. There is,
however, a dearth of evidence to prove such claims. Hence, there
is no basis to dismiss respondents-union members from
employment on the ground that they committed illegal acts during
the strike.

344
Page 345

While it is true that the award of back wages is a legal


consequence of a finding of illegal dismissal, in G & S Transport
Corporation v. Infante, the Court pronounced that the dismissed
workers are entitled only to reinstatement considering that they did
not render work for the employer during the strike, viz.: With respect
to back wages, the principle of a "fair day's wage for a fair day's
labor" remains as the basic factor in determining the award
thereof. If there is no work performed by the employee there can be
no wage or pay unless, of course, the laborer was able, willing and
ready to work but was illegally locked out, suspended or dismissed
or otherwise illegally prevented from working. While it was found
that respondents expressed their intention to report back to work,
the latter exception cannot apply in this case. In Philippine Marine
Officers' Guild v. Compañia Maritima, as affirmed in Philippine
Diamond Hotel and Resort v. Manila Diamond Hotel Employees
Union, the Court stressed that for this exception to apply, it is
required that the strike be legal, a situation that does not obtain in
the case at bar.
Thus, in the case at bar, respondents-union members'
reinstatement without back wages suffices for the appropriate relief.
Fairness and justice dictate that back wages be denied the
employees who participated in the illegal concerted activities to the
great detriment of the employer. Nevertheless, separation pay is
made an alternative relief in lieu of reinstatement in certain
circumstances, like: (a) when reinstatement can no longer be
effected in view of the passage of a long period of time or because
of the realities of the situation; (b) reinstatement is inimical to the
employer's interest; (c) reinstatement is no longer feasible; (d)
reinstatement does not serve the best interests of the parties
involved; (e) the employer is prejudiced by the workers' continued
employment; (f) facts that make execution unjust or inequitable
have supervened; or (g) strained relations between the employer
and employee. Given the lapse of considerable time from the
occurrence of the strike, the Court ruled that the award of separation
pay of one (1) month salary for each year of service, in lieu of
reinstatement, is in order. This relief strikes a balance between the
respondents-union members who may not have known that they
were participating in an illegal strike but who, nevertheless, have
rendered service to the company for years prior to the illegal strike
which caused a rift in their relations, and the employer who definitely
suffered losses on account of respondents-union members' failure
to report to work during the illegal strike.

345
Page 346

Case Digests by: GENEVIEVE E. FLORALDE

Heritage Hotel Manila v. NUWHRAIN-HHMSC

G.R. No. 178296. January 12, 2011.


Nachura, J.
.
Martires, J.
DOCTRINE:
RIGHTS OF LEGITIMATE LABOR ORGANIZATIONS
The union members and, in fact, all the employees belonging to
the appropriate bargaining unit should not be deprived of a bargaining
agent, merely because of the negligence of the union officers who
were responsible for the submission of the documents to the BLR.
Labor authorities should, indeed, act with circumspection in treating
petitions for cancellation of union registration, lest they be accused of
interfering with union activities

FACTS:
Respondent filed with the Department of Labor and
Employment-National Capital Region (DOLE-NCR) a petition for
certification election. The Med-Arbiter granted the petition and
ordered the holding of a certification election. On appeal, the DOLE
Secretary affirmed the Med-Arbiter's order and remanded the case
to the Med-Arbiter for the holding of a pre-election conference but it
was not held as initially scheduled so petitioner moved to archive or
to dismiss the petition due to alleged repeated non-appearance of
respondent. The latter agreed to suspend proceedings until further
notice. The pre-election conference resumed but subsequently,
petitioner discovered that respondent had failed to submit to the
Bureau of Labor Relations (BLR) its annual financial report for
several years and the list of its members since it filed its registration
papers in 1995. Consequently, petitioner filed a Petition for
Cancellation of Registration of respondent, on the ground of the
non-submission of the said documents. Petitioner prayed that
respondent's Certificate of Creation of Local/Chapter be cancelled
and its name be deleted from the list of legitimate labor
organizations. It further requested the suspension of the certification
election proceedings. Petitioner reiterated its request by filing a
Motion to Dismiss or Suspend the [Certification Election]
Proceedings. Nevertheless, the certification election pushed
through and respondent emerged as the winner. Petitioner filed a
Protest with Motion to Defer Certification of Election Results and
Winner and then prayed that the certification of the election results
and winner be deferred until the petition for cancellation shall have

346
Page 347

been resolved, and that respondent's members who held


confidential or managerial positions be excluded from the
supervisors' bargaining unit. Respondent prayed for the dismissal
of the petition. The Med-Arbiter dismissed petitioner's protest, and
certified respondent as the sole and exclusive bargaining agent of
all supervisory employees. Petitioner subsequently appealed the
said Order to the DOLE Secretary but was later dismissed.
Petitioner moved for reconsideration, but the motion was also
denied. In the meantime, Regional Director Alex E. Maraan
(Regional Director Maraan) of DOLE-NCR finally resolved the
petition for cancellation of registration. While finding that respondent
had indeed failed to file financial reports and the list of its members
for several years, he, nonetheless, denied the petition, ratiocinating
that freedom of association and the employees' right to self-
organization are more substantive considerations. Aggrieved,
petitioner appealed the decision but the same was dismissed as
well as the subsequent petition for certiorari.
In this petition, petitioner argues that respondent's registration
as a legitimate labor union should be cancelled. Petitioner posits
that once it is determined that a ground enumerated in Article 239
of the Labor Code is present, cancellation of registration should
follow; it becomes the ministerial duty of the Regional Director to
cancel the registration of the labor organization, hence, the use of
the word "shall." Petitioner points out that the Regional Director has
admitted in its decision that respondent failed to submit the required
documents for a number of years; therefore, cancellation of its
registration should have followed as a matter of course.
ISSUE:
Whether or not the Regional Director and the DOLE Secretary
were in grave abuse of discretion in denying the petition for
cancellation of respondent’s registration.

RULING:

The union members and, in fact, all the employees belonging to


the appropriate bargaining unit should not be deprived of a bargaining
agent, merely because of the negligence of the union officers who were
responsible for the submission of the documents to the BLR. Labor
authorities should, indeed, act with circumspection in treating petitions
for cancellation of union registration, lest they be accused of interfering
with union activities. In resolving the petition, consideration must be
taken of the fundamental rights guaranteed by Article XIII, Section 3 of
the Constitution, i.e., the rights of all workers to self-organization,
collective bargaining and negotiations, and peaceful concerted
activities. Labor authorities should bear in mind that registration
confers upon a union the status of legitimacy and the concomitant right
and privileges granted by law to a legitimate labor organization,

347
Page 348

particularly the right to participate in or ask for certification election in


a bargaining unit. Thus, the cancellation of a certificate of registration
is the equivalent of snuffing out the life of a labor organization. For
without such registration, it loses - as a rule - its rights under the Labor
Code. As aptly ruled by respondent Bureau of Labor Relations Director
Noriel: "The rights of workers to self-organization finds general and
specific constitutional guarantees. Such constitutional guarantees
should not be lightly taken much less nullified. A healthy respect for the
freedom of association demands that acts imputable to officers or
members be not easily visited with capital punishments against the
association itself." The court concluded that on 19 May 2000, appellee
had submitted its financial statement for the years 1996-1999. With this
submission, appellee has substantially complied with its duty to submit
its financial report for the said period. To rule differently would be to
preclude the union, after having failed to meet its periodic obligations
promptly, from taking appropriate measures to correct its omissions.

348
Page 349

Case Digests by: GENEVIEVE E. FLORALDE

Abaria vs. NLRC


G.R. No. 154113. December 07, 2011.
Peralta, C. J.
.
DOCTRINE:
RIGHTS OF LEGITIMATE LABOR ORGANIZATIONS
A local union which is not independently registered cannot, upon
disaffiliation from the federation, exercise the rights and privileges
granted by law to legitimate labor organizations; thus, it cannot file a
petition for certification election.

FACTS:
Metro Cebu Community Hospital, Inc. (MCCHI), presently
known as the Visayas Community Medical Center (VCMC), is a
non-stock, non-profit corporation. It operates the Metro Cebu
Community Hospital (MCCH), a tertiary medical institution owned
by the United Church of Christ in the Philippines (UCCP) and Rev.
Gregorio P. Iyoy is the Hospital Administrator.
The National Federation of Labor (NFL) is the exclusive bargaining
representative of the rank-and-file employees of MCCHI. On
December 6, 1995, Nava wrote Rev. Iyoy expressing the union's
desire to renew the CBA, attaching to her letter a statement of
proposals signed/endorsed by 153 union members. Nava
subsequently requested that certain employees be allowed to avail
of one-day union leave with pay on December 19, 1995. On
February 26, 1996, upon the request of Atty. Alforque, MCCHI
granted one-day union leave with pay for 12 union members. The
next day, several union members led by Nava and her group
launched a series of mass actions such as wearing black and red
armbands/headbands, marching around the hospital premises and
putting up placards, posters and streamers. Atty. Alforque
immediately disowned the concerted activities being carried out by
union members which are not sanctioned by NFL. Rev. Iyoy, having
been informed that Nava and her group have also been suspended
by NFL, directed said officers to appear before his office for
investigation in connection with the illegal strike wherein they
reportedly uttered slanderous and scurrilous words against the
officers of the hospital, threatening other workers and forcing them
to join the strike. Said union officers, however, invoked the
grievance procedure provided in the CBA to settle the dispute
between management and the union.
On March 13 and 19, 1996, the Department of Labor and
Employment (DOLE) Regional Office No. 7 issued certifications
stating that there is nothing in their records which shows that NAMA-

349
Page 350

MCCH-NFL is a registered labor organization, and that said union


submitted only a copy of its Charter Certificate on January 31, 1995.
It was explained that the picketing employees wore armbands to
protest MCCHI's refusal to bargain; it was also contended that
MCCHI cannot question the legal personality of the union which had
actively assisted in CBA negotiations and implementation.
On March 13, 1996, NAMA-MCCH-NFL filed a Notice of Strike
but the same was deemed not filed for want of legal personality on
the part of the filer. The National Conciliation and Mediation Board
(NCMB) Region 7 office likewise denied their motion for
reconsideration on March 25, 1996. Despite such rebuff, Nava and
her group still conducted a strike vote on April 2, 1996 during which
an overwhelming majority of union members approved the
strike. On March 30, 1996, MCCHI sent termination letters to union
leaders and other members who participated in the strike and
picketing activities. For their continued picketing activities despite
warnings, more than 100 striking employees were dismissed.
Unfazed, the striking union members held more mass actions. With
the volatile situation adversely affecting hospital operations and the
condition of confined patients, MCCHI filed a petition for injunction
in the NLRC and it was granted. Thereafter, several complaints for
illegal dismissal and unfair labor practice were filed by the
terminated employees against MCCHI, Rev. Iyoy, UCCP and
members of the Board of Trustees of MCCHI. The LA dismissed the
same and it was affirmed by the NLRC. The CA likewise affirmed.
Both petitioners and private respondents in CA-G.R. SP No. 66540
appealed to this Court. Private respondent MCCHI in CA-G.R. SP
No. 84998, under its new name Visayas Community Medical Center
(VCMC), filed a petition for certiorari in this Court.
ISSUE:
Whether or not MCCHI is guilty of unfair labor practice.
RULING:
No. NAMA-MCCH-NFL charged MCCHI with refusal to
bargain collectively when the latter refused to meet and convene for
purposes of collective bargaining, or at least give a counter-
proposal to the proposed CBA the union had submitted and which
was ratified by a majority of the union membership. MCCHI, on its
part, deferred any negotiations until the local union's dispute with
the national union federation (NFL) is resolved considering that the
latter is the exclusive bargaining agent which represented the rank-
and-file hospital employees in CBA negotiations since 1987.
The court ruled in favor of MCCHI. Records of the NCMB and
DOLE Region 7 confirmed that NAMA-MCCH-NFL had not
registered as a labor organization, having submitted only its charter
certificate as an affiliate or local chapter of NFL. Not being a
legitimate labor organization, NAMA-MCCH-NFL is not entitled to

350
Page 351

those rights granted to a legitimate labor organization. Aside from


the registration requirement, it is only the labor organization
designated or selected by the majority of the employees in an
appropriate collective bargaining unit which is the exclusive
representative of the employees in such unit for the purpose of
collective bargaining, as provided in Art. 255. NAMA-MCCH-NFL is
not the labor organization certified or designated by the majority of
the rank-and-file hospital employees to represent them in the CBA
negotiations but the NFL, as evidenced by CBAs concluded in 1987,
1991 and 1994. While it is true that a local union has the right to
disaffiliate from the national federation, NAMA-MCCH-NFL has not
done so as there was no any effort on its part to comply with the
legal requisites for a valid disaffiliation during the "freedom
period" or the last 60 days of the last year of the CBA, through a
majority vote in a secret balloting in accordance with Art. 241
(d). Nava and her group simply demanded that MCCHI directly
negotiate with the local union which has not even registered as one.
To prove majority support of the employees, NAMA-MCCH-NFL
presented the CBA proposal allegedly signed by 153 union
members. However, the petition signed by said members showed
that the signatories endorsed the proposed terms and conditions
without stating that they were likewise voting for or designating the
NAMA-MCCH-NFL as their exclusive bargaining representative. In
any case, NAMA-MCCH-NFL at the time of submission of said
proposals was not a duly registered labor organization, hence it
cannot legally represent MCCHI's rank-and-file employees for
purposes of collective bargaining. Hence, even assuming that
NAMA-MCCH-NFL had validly disaffiliated from its mother union,
NFL, it still did not possess the legal personality to enter into CBA
negotiations. A local union which is not independently registered
cannot, upon disaffiliation from the federation, exercise the rights
and privileges granted by law to legitimate labor organizations;
thus, it cannot file a petition for certification election. Besides, the
NFL as the mother union has the right to investigate members of its
local chapter under the federation's Constitution and By-Laws, and
if found guilty to expel such members. MCCHI therefore cannot be
faulted for deferring action on the CBA proposal submitted by
NAMA-MCCH-NFL in view of the union leadership's conflict with
the national federation. We have held that the issue of disaffiliation
is an intra-union dispute which must be resolved in a different forum
in an action at the instance of either or both the federation and the
local union or a rival labor organization, not the employer. Not being
a legitimate labor organization nor the certified exclusive bargaining
representative of MCCHI's rank-and-file employees, NAMA-MCCH-
NFL cannot demand from MCCHI the right to bargain collectively in
their behalf. Hence, MCCHI's refusal to bargain then with NAMA-
MCCH-NFL cannot be considered an unfair labor practice to justify
the staging of the strike.

351
Page 352

Case Digests by: GENEVIEVE E. FLORALDE

Peninsula Employees Union v. Esquivel

G.R. No. 218454. December 01, 2016.


Peralta, C. J.
.
DOCTRINE:
RIGHTS OF LEGITIMATE LABOR ORGANIZATIONS; VALID LEVY
OF INCREASED UNION DUES
Case law interpreting Article 250 (n) and (o) (formerly Article 241) of
the Labor Code, as amended, mandates the submission of three (3)
documentary requisites in order to justify a valid levy of increased
union dues. These are: (a) an authorization by a written resolution of
the majority of all the members at the general membership meeting
duly called for the purpose; (b) the secretary's record of the minutes
of the meeting, which shall include the list of all members present, the
votes cast, the purpose of the special assessment or fees and the
recipient of such assessment or fees; and (c) individual written
authorizations for check-off duly signed by the employees concerned.
FACTS:
PEU's Board of Directors passed Local Board Resolution No.
12, series of 2007 authorizing (a) the affiliation of PEU with
NUWHRAIN, and the direct membership of its individual members
thereto; (b) the compliance with all the requirements therefor;
and (c) the Local President to sign the affiliation agreement with
NUWHRAIN upon acceptance of such affiliation. On the same day,
the said act was submitted to the general membership, and was
duly ratified by 223 PEU members. PEU-NUWHRAIN sought to
increase the union dues/agency fees from one percent (1%) to two
percent (2%) of the rank and file employees' monthly salaries,
brought about by PEU's affiliation with NUWHRAIN, which
supposedly requires its affiliates to remit to it two percent (2%) of
their monthly salaries. Meanwhile, in a Decision, the OSEC resolved
the collective bargaining deadlock between PEU-NUWHRAIN and
The Peninsula Manila Hotel (Hotel), ordering the parties to execute
a collective bargaining agreement (CBA) incorporating the
dispositions therein (arbitral award). The parties have yet to
actually sign a CBA but have, for the most part, implemented the
arbitral award.
PEU-NUWHRAIN requested the OSEC for Administrative
Intervention for Dispute Avoidance (AIDA) pursuant to DOLE
Circular No. 1, series of 2006 in relation to the issue, among others,
of its entitlement to collect increased agency fees from the non-PEU
members. The non-PEU members objected to the assessment of

352
Page 353

increased agency fees arguing that: (a) the new CBA is


unenforceable since no written CBA has been formally signed and
executed by PEU-NUWHRAIN and the Hotel; (b) the 2% agency fee
is exorbitant and unreasonable; and (c) PEU-NUWHRAIN failed to
comply with the mandatory requirements for such increase.
The OSEC upheld PEU-NUWHRAIN's right to collect agency
fees from the non-PEU members in accordance with Article 4,
Section 2 of the expired CBA, which was declared to be in full force
and effect pursuant to the October 10, 2008 Decision, but only at
the rate of one percent (1%), and denied its bid to increase the
agency fees to two percent (2%) for failure to show that its general
membership approved the same. The OSEC issued an
Order partially granting PEU-NUWHRAIN's motion for
reconsideration but unperturbed, respondents filed a petition
for certiorari with the CA which set aside the OSEC's Order.
ISSUE:
Whether or not the CA committed reversible error in ruling that
PEU-NUWHRAIN had no right to collect the increased agency fees.

RULING:
The petition lacks merit. In the present case, PEU-
NUWHRAIN's right to collect agency fees is not disputed. However,
the rate of agency fees it seeks to collect from the non-PEU
members is contested, considering its failure to comply with the
requirements for a valid increase of union dues, rendering the
collection of increased agency fees unjustified. Case law
interpreting Article 250 (n) and (o) (formerly Article 241) of the Labor
Code, as amended, mandates the submission of three (3)
documentary requisites in order to justify a valid levy of increased
union dues. These are: (a) an authorization by a written resolution
of the majority of all the members at the general membership
meeting duly called for the purpose; (b) the secretary's record of the
minutes of the meeting, which shall include the list of all members
present, the votes cast, the purpose of the special assessment or
fees and the recipient of such assessment or fees; and (c) individual
written authorizations for check-off duly signed by the employees
concerned. In the present case, however, PEU-NUWHRAIN failed
to show compliance with the foregoing requirements. It is evident
that while the matter of implementing the two percent (2%) union
dues was taken up during the PEU-NUWHRAIN's 8th General
Membership Meeting on October 28, 2008, there was no sufficient
showing that the same had been duly deliberated and
approved. The minutes of the Assembly itself belie PEU-
NUWHRAIN's claim that the increase in union dues and the
corresponding check-off were duly approved since it merely stated
that "the [two percent (2%)] Union dues will have to be

353
Page 354

implemented," meaning, it would still require the submission of such


matter to the Assembly for deliberation and approval. Such
conclusion is bolstered by the silence of the October 28, 2008 GMR
on the matter of two percent (2%) union dues, in contrast to the
payment of 10% attorney's fees from the CBA backwages which
was clearly spelled out as having been "discussed and
approved." Thus, as aptly pointed out by the CA: "If indeed majority
of the members of [PEU-NUWHRAIN] approved the increase in
union dues, the same should have been mentioned in the [October
28, 2008 minutes], and reflected in the GMR of the same date." C
Having failed to establish due deliberation and approval of the
increase in union dues from one percent (1%) to two percent (2%),
as well as the deduction of the two percent (2%) union dues during
PEU-NUWHRAIN's 8th General Membership Meeting on October
28, 2008, there was nothing to confirm, affirm, or ratify through the
July 1, 2010 GMR. Contrary to the ruling of the OSEC in its March
6, 2012 Order, the July 1, 2010 GMR, by itself, cannot justify the
collection of two percent (2%) agency fees from the non-PEU
members beginning July 2010. The Assembly was not called for the
purpose of approving the proposed increase in union dues and the
corresponding check-off, but merely to "confirm and affirm" a
purported prior action which PEU-NUWHRAIN, however, failed to
establish. Corollarily, no individual check-off authorizations can
proceed therefrom, and the submission of the November 2008
check-off authorizations becomes inconsequential. Jurisprudence
states that the express consent of the employee to any deduction in
his compensation is required to be obtained in accordance with the
steps outlined by the law, which must be followed to the letter;
however, PEU-NUWHRAIN failed to comply. Thus, the CA correctly
ruled that there is no legal basis to impose union dues and agency
fees more than that allowed in the expired CBA, i.e., at one percent
(1%) of the employee's monthly basic salary.

354
Page 355

Case Digests by: GENEVIEVE E. FLORALDE

ERGONOMICS SYSTEMS PHIL INC. vs. ENASE

G.R. No. 195163. December 13, 2017.


Martires, J.
DOCTRINE:
AFFILIATION AND DISAFFILIATION: A local union does not
owe its existence to the federation with which it is affiliated. It is a
separate and distinct voluntary association owing its creation to the
will of its members. Mere affiliation does not divest the local union
of its own personality, neither does it give the mother federation the
license to act independently of the local union. It only gives rise to a
contract of agency, where the former acts in representation of the
latter. Hence, local unions are considered principals while the
federation is deemed to be merely their agent.
FACTS:

Respondents were union officers and members of Ergonomic System


Employees Union-Workers Alliance Trade Unions (local union). The
local union entered into a Collective Bargaining
Agreement (CBA) with petitioner Ergonomic Systems Philippines,
Inc. (ESPI), which was valid for five (5) years or until October 2004.
The local union, which was affiliated with Workers Alliance Trade
Unions-Trade Union Congress of the Philippines (Federation), was
not independently registered. Thus, on 15 November 2001, before
the CBA expired, the union officers secured the independent
registration of the local union with the Regional Office of the
Department of Labor and Employment (DOLE). Later on, the union
officers were charged before the Federation and investigated for
attending and participating in other union's seminars and activities
using union leaves without the knowledge and consent of the
Federation and ESPI as well as in initiating and conspiring in the
disaffiliation before the freedom period. The the Federation rendered
a decision finding respondents-union guilty of disloyalty so they were
penalized with immediate expulsion from the Federation. The local
union then filed a notice of strike with the National Conciliation and
Mediation Board (NCMB). The local union staged a series of noise
barrage and "slow down" activities. Meanwhile, 40 union members
refused to submit their Daily Production Reports (DPRs). 28 union
members abandoned their work and held a picket line outside the
premises of ESPI. Certain union members did not report for work
without official leave. Subsequently, for refusal to submit DPRs and
for abandonment, respondents-union members were issued letters of

355
Page 356

termination. The respondents then filed a complaint for illegal


dismissal and unfair labor practice against ESPI, Phillip C. Ng, and
Ma. Lourminda O. Ng (petitioners). The LA ordered all the
respondents to return to work but without back wages. Unconvinced,
petitioners and respondents appealed before the NLRC but the
NLRC affirmed the ruling of the LA. The petition was partially granted
by the CA. Aggrieved, petitioners and respondents moved for
reconsideration but the same was denied by the CA in a resolution
hence this petition. Petitioners aver that the respondents violated the
union security clause under the CBA; that their termination was
effected in response to the Federation's demand to dismiss them; that
they did not comply with the requisites of a valid strike; that they
refused to submit their DPRs and abandoned their work; and that the
award of separation pay had no basis because the respondents had
been legally dismissed from their employment. The respondents
counter that they were not legally terminated because the grounds
relied upon by the petitioners were non-existent; that as ruled by the
NLRC, they merely disaffiliated from the Federation but they were not
disloyal to the local union; that reinstatement is not physically
impossible because it was the industrial building owned by Ergo
Contracts Philippines, Inc. that was gutted down by fire, not that of
ESPI; that even if the manufacturing plant of ESPI was indeed
destroyed by fire, the petitioners have other offices around the
country where the respondents may be reinstated; and that having
failed to comply with the order to reinstate them and having ceased
operations, the petitioners must be ordered to pay their separation
pay.

ISSUE:
Whether or not the Federation may invoke the union security clause
in the CBA in demanding the respondents’ dismissal.
RULING:
"Union security is a generic term, which is applied to and
comprehends 'closed shop,' 'union shop,' 'maintenance of
membership,' or any other form of agreement which imposes upon
employees the obligation to acquire or retain union membership as
a condition affecting employment. Before an employer terminates
an employee pursuant to the union security clause, it needs to
determine and prove that: (1) the union security clause is applicable;
(2) the union is requesting the enforcement of the union security
provision in the CBA; and (3) there is sufficient evidence to support
the decision of the union to expel the employee from the union. In
this case, the primordial requisite, i.e., the union is requesting the
enforcement of the union security provision in the CBA, is clearly
lacking. Under the Labor Code, a chartered local union acquires
legal personality through the charter certificate issued by a duly
registered federation or national union and reported to the Regional

356
Page 357

Office. "A local union does not owe its existence to the federation
with which it is affiliated. It is a separate and distinct voluntary
association owing its creation to the will of its members. Mere
affiliation does not divest the local union of its own personality,
neither does it give the mother federation the license to act
independently of the local union. It only gives rise to a contract of
agency, where the former acts in representation of the latter. Hence,
local unions are considered principals while the federation is
deemed to be merely their agent."
A perusal of the CBA shows that the local union, not the
Federation, was recognized as the sole and exclusive collective
bargaining agent for all its workers and employees in all matters
concerning wages, hours of work, and other terms and conditions
of employment. Consequently, only the union may invoke the union
security clause in case any of its members commits a violation
thereof. Even assuming that the union officers were disloyal to the
Federation and committed acts inimical to its interest, such
circumstance did not give the Federation the prerogative to demand
the union officers' dismissal pursuant to the union security clause
which, in the first place, only the union may rightfully invoke.
Certainly, it does not give the Federation the privilege to act
independently of the local union. At most, what the Federation could
do is to refuse to recognize the local union as its affiliate and revoke
the charter certificate it issued to the latter. In fact, even if the local
union itself disaffiliated from the Federation, the latter still has no
right to demand the dismissal from employment of the union officers
and members because concomitant to the union's prerogative to
affiliate with a federation is its right to disaffiliate therefrom. In sum,
the Federation could not demand the dismissal from employment of
the union officers on the basis of the union security clause found in
the CBA between ESPI and the local union.

357
Page 358

Case Digests by: GENEVIEVE E. FLORALDE

Weslayan University Phils. v. Weslayan University Philippines


Faculty and Staff Association

G.R. No. 181806. March 12, 2014.


Del Castillo, J.
.
DOCTRINE:
NATURE OF CBA; When the provision of the CBA is clear, leaving
no doubt on the intention of the parties, the literal meaning of the
stipulation shall govern. However, if there is doubt in its interpretation,
it should be resolved in favor of labor, as this is mandated by no less
than the Constitution.

FACTS:
Petitioner Wesleyan University-Philippines is a non-stock, non-
profit educational institution duly organized and existing under the laws
of the Philippines. Respondent Wesleyan University-Philippines
Faculty and Staff Association, on the other hand, is a duly registered
labor organization acting as the sole and exclusive bargaining agent of
all rank-and-file faculty and staff employees of petitioner. In December
2003, the parties signed a 5-year CBA effective June 1, 2003 until May
31, 2008. On August 16, 2005, petitioner, through its President, Atty.
Guillermo T. Maglaya (Atty. Maglaya), issued a
Memorandum providing guidelines on the implementation of vacation
and sick leave credits as well as vacation leave commutation. On
August 25, 2005, respondent's President, Cynthia L. De Lara (De Lara)
wrote a letter to Atty. Maglaya informing him that respondent is not
amenable to the unilateral changes made by petitioner. De Lara
questioned the guidelines for being violative of existing practices and
the CBA, specifically Sections 1 and 2, Article XII of the CBA. On
February 8, 2006, a Labor Management Committee (LMC) Meeting
was held during which petitioner advised respondent to file a grievance
complaint on the implementation of the vacation and sick leave policy.
In the same meeting, petitioner announced its plan of implementing a
one-retirement policy, which was unacceptable to respondent. Unable
to settle their differences at the grievance level, the parties referred the
matter to a Voluntary Arbitrator which declared the one-retirement
policy and the Memorandum dated August 16, 2005 contrary to law.
The CA affirmed the same hence this petition.

ISSUE:

358
Page 359

Whether or not the CA committed grave and palpable error in


revoking the 16 August 2005 Memorandum of Petitioner WU-P
for being contrary to extant policy.
RULING:
The court did not find any reason to disturb the findings of the
CA that the Memorandum dated August 16, 2005 is contrary to the
existing CBA. Sections 1 and 2 of Article XII of the CBA provide that
all covered employees are entitled to 15 days sick leave and 15
days vacation leave with pay every year and that after the second
year of service, all unused vacation leave shall be converted to cash
and paid to the employee at the end of each school year, not later
than August 30 of each year. The Memorandum dated August 16,
2005, however, states that vacation and sick leave credits are not
automatic as leave credits would be earned on a month-to-month
basis. This, in effect, limits the available leave credits of an
employee at the start of the school year. For example, for the first
four months of the school year or from June to September, an
employee is only entitled to five days vacation leave and five days
sick leave. Considering that the Memorandum dated August 16,
2005 imposes a limitation not agreed upon by the parties nor stated
in the CBA, we agree with the CA that it must be struck down. When
the provision of the CBA is clear, leaving no doubt on the intention
of the parties, the literal meaning of the stipulation shall govern.
However, if there is doubt in its interpretation, it should be resolved
in favor of labor, as this is mandated by no less than the Constitution

359
Page 360

Case Digests by: GENEVIEVE E. FLORALDE

SAMAHAN NG MANGGAGAWA SA HANJIN


SHIPYARD rep. by its President, ALFIE ALIPIO vs.
BUREAU OF LABOR RELATIONS, HANJIN HEAVY
INDUSTRIES AND CONSTRUCTION CO., LTD. (HHIC-
PHIL.)

G.R. No. 211145. October 14, 2015.


Mendoza, J.
.
DOCTRINE:
COVERAGE AND EMPLOYEES’ RIGHT TO SELF-
ORGANIZATION.
Inherent in the right to self-organization is the right to choose
whether to form a union for purposes of collective bargaining or a
workers' association for purposes of providing mutual aid and
protection; There is no provision in the Labor Code that states that
employees with definite employers may form, join or assist unions
only; The right to form a workers' association is not exclusive to
ambulant, intermittent and itinerant workers. The option to form or
join a union or a workers' association lies with the workers
themselves, and whether they have definite employers or not.
FACTS:
On February 16, 2010, Samahan, through its authorized
representative, Alfie F. Alipio, filed an application for registration of
its name "Samahan ng mga Manggagawa sa Hanjin Shipyard" with
the DOLE. Attached to the application were the list of names of the
association's officers and members, signatures of the attendees of
the February 7, 2010 meeting, copies of their Constitution and By-
laws. The application stated that the association had a total of 120
members. The DOLE Regional Office No. 3, City of San Fernando,
Pampanga (DOLE-Pampanga), issued the corresponding
certificate of registration in favor of Samahan.
On March 15, 2010, respondent Hanjin Heavy Industries and
Construction Co., Ltd. Philippines (Hanjin), filed a petition with
DOLE-Pampanga praying for the cancellation of registration of
Samahan's association on the ground that its members did not fall
under any of the types of workers enumerated in the second
sentence of Article 243 (now 249). Hanjin opined that only
ambulant, intermittent, itinerant, rural workers, self-employed, and
those without definite employers may form a workers' association.
It further posited that one third (1/3) of the members of the
association had definite employers and the continued existence and
registration of the association would prejudice the company's

360
Page 361

goodwill. Hanjin filed a supplemental petition, adding the alternative


ground that Samahan committed a misrepresentation in connection
with the list of members and/or voters who took part in the
ratification of their constitution and by-laws in its application for
registration. Hanjin claimed that Samahan made it appear that its
members were all qualified to become members of the workers'
association. DOLE Regional Director Ernesto Bihis ruled in favor of
Hanjin. The BLR granted Samahan's appeal and reversed the ruling
of the Regional Director. The CA dismissed its petition and the
BLR's directive.
Samahan argues that the right to form a workers' association
is not exclusive to intermittent, ambulant and itinerant workers.
While the Labor Code allows the workers "to form, join or assist
labor organizations of their own choosing" for the purpose of
collective bargaining, it does not prohibit them from forming a labor
organization simply for purposes of mutual aid and protection.
ISSUE:
Whether or not the Court of Appeals seriously erred in finding
that SAMAHAN cannot form a workers’ association of
employees in Hanjin and instead should have formed a union,
hence their registration as a workers’ association should be
cancelled.
RULING:
In the case at bench, the Court cannot sanction the opinion of
the CA that Samahan should have formed a union for purposes of
collective bargaining instead of a workers' association because the
choice belonged to it. The right to form or join a labor organization
necessarily includes the right to refuse or refrain from exercising the
said right. It is self-evident that just as no one should be denied the
exercise of a right granted by law, so also, no one should be
compelled to exercise such a conferred right. Also inherent in the
right to self-organization is the right to choose whether to form a
union for purposes of collective bargaining or a workers' association
for purposes of providing mutual aid and protection. The right to self-
organization, however, is subject to certain limitations as provided
by law. For instance, the Labor Code specifically disallows
managerial employees from joining, assisting or forming any labor
union. Meanwhile, supervisory employees, while eligible for
membership in labor organizations, are proscribed from joining the
collective bargaining unit of the rank and file employees. Even
government employees have the right to self-organization. It is not,
however, regarded as existing or available for purposes of collective
bargaining, but simply for the furtherance and protection of their
interests.
Hanjin posits that the members of Samahan have definite
employers, hence, they should have formed a union instead of a

361
Page 362

workers' association. The Court disagrees. There is no provision in


the Labor Code that states that employees with definite employers
may form, join or assist unions only. The Court cannot subscribe
either to Hanjin's position that Samahan's members cannot form the
association because they are not covered by the second sentence
of Article 243 (now 249). Article 243 should be read together with
Rule 2 of Department Order (D.O.) No. 40-03, Series of 2003. There
is nothing in the implementing rules which provides that workers,
with definite employers, cannot form or join a workers' association
for mutual aid and protection. Section 2 thereof even broadens the
coverage of workers who can form or join a workers' association.
Thus, the Court agrees with Samahan's argument that the right to
form a workers' association is not exclusive to ambulant, intermittent
and itinerant workers. The option to form or join a union or a
workers' association lies with the workers themselves, and whether
they have definite employers or not.

362
Page 363

Case Digest by: GINGOYON, CORDELL JERICHO M.

UNITED PEPSI -COLA SUPERVISORY UNION (UPSU)


VS. HON. BIENVENIDO LAGUESMA AND PEPSI-COLA
PRODUCTS, PHILS., INC.
G.R. No. 122226. March 25, 1998.
Mendoza, J.

DOCTRINE:
ARTICLE 253.
The guarantee of organizational right in Art. III, Section 8 is
NOT infringed by a ban against managerial employees from forming
a union. The right guaranteed in Art. III, Section 8 is subject to the
condition that its exercise should be for purposes “not contrary to
law”.

FACTS:
United Pepsi-Cola Supervisory Union (UPSU) was composed
of route managers of Pepsi-Cola Products, Philippines, Incorporated.
It appeared that on March 20, 1995, UPSU filed a petition for
certification election on behalf of its membership. The petition was
denied by the med-arbiter and, on appeal, by the Secretary of Labor
and Employment. The decision was based on the ground that the
route managers were managerial employees ineligible for union
membership.

Official corporate documents showed that these route


managers had the powers to endorse the hiring or firing of rank-and-
file employees, and the fulfillment of designated tasks to complete
sales objectives.

Petitioner brought this suit to challenge the validity of the order


dated August 31, 1995 of the Secretary of Labor and Employment. Its
petition was dismissed by the Third Division for lack of showing that
respondent committed grave abuse of discretion. But the petitioner
filed a motion for reconsideration, pressing for resolution its
contention that the first sentence of Art. 245 of the Labor Code, so far
as it declared that managerial employees were ineligible to form,
assist or join unions, contravened Art. III, Section 8 of the 1987
Constitution.

ISSUES:
1. Whether or not Article 245 of the Labor Code (Now article 255)
contravened Article III, Section 8 of the 1987 Constitution.

363
Page 364

2. Whether or not the route managers of the corporation were


validly classified as managerial employees, which are
prohibited from forming unions.

RULING:
1. No. Article 245 was the result of the amendment of the Labor
Code in 1989 by R.A. No. 6715, otherwise known as the Herrera-
Veloso Law. Unlike the Industrial Peace Act or the provisions of the
Labor Code which it superseded, R.A. No. 6715 provided for
separate definitions of the terms “managerial” and “supervisory
employees”.

In the case of Art. 245, there was a rational basis for prohibiting
managerial employees from forming or joining labor organizations.
The rationale for this inhibition has been stated to be that if these
managerial employees would belong to or be affiliated with a Union,
the latter might not be assured of their loyalty to the Union in view of
the evident conflict of interests. The Union can also become
company-dominated with the presence of managerial employees in
Union membership.

2. Yes. Distinction is evident in the work of the route managers


which sets them apart from supervisors in general. Unlike supervisors
who basically merely direct operating employees in line with set tasks
assigned to them, route managers were responsible for the success
of the company’s main line of business through management of their
respective sales teams. Such management necessarily involves the
planning, direction, operation and evaluation of their individual teams
and areas which the work of supervisors does not entail.

A distinction exist between those who have the authority to


devise, implement and control strategic and operational policies (top
and middle managers) and those whose task is simply to ensure that
such polices are carried out by the rank-and-file employees of an
organization (first-level managers/supervisors). Managerial
employees may therefore be said to fall into two distinct categories:
the managers per se, who compose the former group described
above, and the supervisors who form the latter group.

364
Page 365

Case Digest by: GINGOYON, CORDELL JERICHO M.

METROLAB INDUSTRIES, INC. vs. HON. MA. ROLDAN-


CONFESOR AND METRO DRUG CORPORATION
EMPLOYEES - ASSOCIATION
G.R. No. 108855. February 28, 1996.
Kapunan, J.

DOCTRINE:
ARTICLE 253.
Although Article 245 of the Labor Code limits the ineligibility to
join, form and assist any labor organization to managerial employees,
jurisprudence has extended this prohibition to confidential employees
or those who by reason of their positions or nature of work are
required to assist or act in a fiduciary manner to managerial
employees and hence, are likewise privy to sensitive and highly
confidential records.

FACTS:

Private respondent Metro Drug Corporation Employees


Association-Federation of Free Workers was a labor organization
representing the rank and file employees of petitioner Metrolab
Industries, Inc. and also of Metro Drug, Inc. On 31 December 1990,
the Collective Bargaining Agreement between Metrolab and the
Union expired. The negotiations for a new CBA ended in a deadlock.

Consequently, on 23 August 1991, the Union filed a notice of


strike against Metrolab and Metro Drug Inc. The parties failed to
settle their dispute despite the conciliation efforts of the NCMB. To
contain the escalating dispute, the then Secretary of Labor and
Employment, Ruben D. Torres, issued an assumption order dated 20
September 1991.

The secretary issued an order resolving all the disputed items in


the CBA and ordered the parties involved to execute a new CBA. The
Union filed a Motion for Reconsideration. During the pendency thereof,
Metrolab laid off ninety-four of its rank and file employees. The Union
filed a motion for a cease and desist order to enjoin Metrolab from
implementing the mass layoff.

Thereafter, on various dates, Metrolab recalled some of the laid


off workers on a temporary basis due to availability of work in the
production lines. The Acting Secretary of Labor, Nieves Confesor,
issued a resolution that declared that the layoff of Metrolab’s ninety-

365
Page 366

four rank and file workers illegal and ordered their reinstatement with
full backwages. Executive secretaries were included as part of the
bargaining unit of the rank and file employees. The executive
secretaries were confidential employees of the corporation.

ISSUES:
3. Whether or not executive secretaries as confidential employees
may form, assist, or join unions with rank and file employees.
4. Whether or not the Secretary of Labor may exercise assumption
powers despite the potential adverse effects on the negotiations of
contracting parties.

RULING:
1. Yes. The rationale for this inhibition has been stated to be that if
these managerial employees would belong to or be affiliated with a
Union, the latter might not be assured of their loyalty to the Union in
view of evident conflict of interests. The union can also become
company-dominated with the presence of managerial employees in
Union membership.

This rationale holds true also for confidential employees such


as accounting personnel, radio and telegraph operators, who having
access to confidential information, may become the source of undue
advantage. Said employees may act as a spy or spies of either party
to a collective bargaining agreement. This is specially true in the case
where the petitioning Union is already the bargaining agent of the
rank-and-file employees in the establishment. To allow the
confidential employees to join the existing Union of the rank-and-file
would be in violation of the terms of the Collective Bargaining
Agreement wherein this kind of employees by the nature of their
functions or positions are expressly excluded.

2. Yes. The Secretary of Labor is expressly given the power under


the Labor Code to assume jurisdiction and resolve labor disputes
involving industries indispensable to national interest. The disputed
injunction is subsumed under this special grant of authority

When, in his opinion, there exists a labor dispute causing or likely to


cause a strike or lockout in an industry indispensable to the national
interest, the Secretary of Labor and Employment may assume
jurisdiction over the dispute and decide it or certify the same to the
Commission for compulsory arbitration. Such assumption or
certification shall have the effect of automatically enjoining the
intended or impending strike or lockout as specified in the assumption
or certification order. If one has already taken place at the time of
assumption or certification, all striking or locked out employees shall
immediately return to work and the employer shall immediately

366
Page 367

resume operations and readmit all workers under the same terms and
conditions prevailing before the strike or lockout. The Secretary of
Labor and Employment or the Commission may seek the assistance
of law enforcement agencies to ensure compliance with this
provision as well as with such orders as he may issue to enforce the
same.

367
Page 368

Case Digest by: GINGOYON, CORDELL JERICHO M.

SAN MIGUEL FOODS, INCORPORATED. vs. SAN


MIGUEL CORPORATION SUPERVISORS AND EXEMPT
UNION
G.R. No. 146206. August 1, 2011.
Peralta, J.

DOCTRINE:
ARTICLE 253.
Confidential employees are thus excluded from the rank-and-
file bargaining unit.

FACTS:
In G.R. No. 110399, entitled San Miguel Corporation Supervisors and
Exempt Union v. Laguesma, the Court held that even if they handle
confidential data regarding technical and internal business
operations, supervisory employees 3 and 4 and the exempt
employees of petitioner San Miguel Foods, Inc. were not to be
considered confidential employees, because the same do not pertain
to labor relations, particularly, negotiation and settlement of
grievances. Consequently, they were allowed to form an appropriate
bargaining unit for the purpose of collective bargaining. The Court
also declared that the employees belonging to the three different
plants of San Miguel Corporation Magnolia Poultry Products Plants in
Cabuyao, San Fernando, and Otis, having "community or mutuality of
interests," constituted a single bargaining unit.

Pursuant thereto, a certification election was conducted on


September 30, 1998. On the date of the election, petitioner
corporation filed the Omnibus Objections and Challenge to Voters,
questioning the eligibility to vote by some of its employees on the
grounds that certain employees should not be allowed to vote
because these were confidential employees.

368
Page 369

Based on the results of the votes, the Med-Arbiter issued the


Order stating that since the "Yes" vote received 97% of the valid
votes cast, respondent union was certified to be the exclusive
bargaining agent of the supervisors and exempt employees of
petitioner's Magnolia Poultry Products Plants in Cabuyao, San
Fernando, and Otis.

On appeal, the then Acting DOLE Undersecretary affirmed the


Order dated April 13, 1999, with modification that four voters be
excluded from the bargaining unit which respondent sought to
represent. She opined that the challenged voters should be excluded
from the bargaining unit, because two were members of Magnolia
Poultry Processing Plants Monthly Employees Union, while the other
two were employees of San Miguel Corporation, which was a
separate and distinct entity from petitioner.

The Court of Appeals affirmed with modification the Resolution


of the DOLE Undersecretary, stating that those holding the positions
of Human Resource Assistant and Personnel Assistant are excluded
from the bargaining unit. The petitioner averred that a payroll master
was a confidential employee and that their votes should not have
been counted.

ISSUES:
5. Whether or not confidential employees may form, assist, or join
unions with rank and file employees.

RULING:

1. Yes. A confidential employee is one entrusted with confidence


on delicate, or with the custody, handling or care and protection of the
employer’s property. Confidential employees, such as accounting
personnel, should be excluded from the bargaining unit, as their
access to confidential information may become the source of undue
advantage.

However, such fact does not apply to the position of Payroll


Master and the whole gamut of employees who, as perceived by
petitioner, has access to salary and compensation data. The CA
correctly held that the position of Payroll Master does not involve
dealing with confidential labor relations information in the course of
the performance of his functions. Since the nature of his work does
not pertain to company rules and regulations and confidential labor
relations, it follows that he cannot be excluded from the subject
bargaining unit.

In this regard, the CA correctly ruled that the positions of


Human Resource Assistant and Personnel Assistant belong to the

369
Page 370

category of confidential employees and, hence, are excluded from the


bargaining unit, considering their respective positions and job
descriptions. As Human Resource Assistant, the scope of one’s work
necessarily involves labor relations, recruitment and selection of
employees. Therefore, in the discharge of their functions, both gain
access to vital labor relations information which outrightly disqualifies
them from union membership.

370
Page 371

Case Digest by: GINGOYON, CORDELL JERICHO M.

SMCC vs. CHARTER CHEMICAL AND COATING CORP.


G.R. No. 169717. March 16, 2011.
Del Castillo, J.

DOCTRINE:
ARTICLE 255.
The inclusion of supervisory employees in a labor organization
seeking to represent the bargaining unit of rank-and-file employees
does not divest it of its status as a legitimate labor organization.

FACTS:
On February 19, 1999, petitioner union Samahang
Manggagawasa Charter Chemical Solidarity of Unions in the
Philippines for Empowerment and Reforms filed a petition for
certification election among the regular rank-and-file employees of
Charter Chemical and Coating Corporation with the Mediation
Arbitration Unit of the DOLE, National Capital Region. On April 14,
1999, respondent company filed an Answer with Motion to Dismiss on
the ground that petitioner union was not a legitimate labor
organization because of the failure to comply with the documentation
requirements set by law, and the inclusion of supervisory employees
within petitioner union.

The Med-Arbiter agreed with the respondent company. Though


the DOLE disagreed with the Med-Arbiter on its findings regarding the
documentation requirements and the inclusion of supervisory
employees in the union. It ruled that the petitioner union did not file its
petition on time. Another union, supposedly, had filed a petition for
certification election and its petition has been decided with finality.
The CA upheld the findings of the Med-Arbiter.

ISSUES:
1. Whether or not the mingling of supervisory employees with rank
and file employees automatically nullifies the legal personality
of the union

RULING:
1. No. In Tagaytay Highlands Int'l. Golf Club, Inc. v. Tagaytay
Highlands Employees Union-PGTWO in which the core issue was
whether mingling affects the legitimacy of a labor organization and
its right to file a petition for certification election, the Court, given
the altered milieu, abandoned the view in Toyota and Dunlop and
reverted to its pronouncement in Lopez that while there is a
prohibition against the mingling of supervisory and rank-and-file

371
Page 372

employees in one labor organization, the Labor Code does not


provide for the effects thereof.

Thus, the Court held that after a labor organization has been
registered, it may exercise all the rights and privileges of a
legitimate labor organization. Any mingling between supervisory
and rank-and-file employees in its membership cannot affect its
legitimacy for that is not among the grounds for cancellation of its
registration, unless such mingling was brought about by
misrepresentation, false statement or fraud under Article 239 of
the Labor Code. As a result, petitioner union was not divested of
its status as a legitimate labor organization even if some of its
members were supervisory employees; it had the right to file the
subject petition for certification election.

372
Page 373

Case Digest by: GINGOYON, CORDELL JERICHO M.

CATHAY PACIFIC STEEL CORP. vs. COURT OF


APPEALS, ENRIQUE TAMONDONG III, ET AL.
G.R. No. 164561. August 30, 2006.
Chico-Nazario, J.

DOCTRINE:
ARTICLE 255.
The disqualification upon managerial employees from joining,
assisting or forming a labor organization does not apply to a
supervisory employee.

FACTS:
Sometime in June 1996, the supervisory personnel of
CAPASCO launched a move to organize a union among their ranks,
later known as private respondent CAPASCO Union of Supervisory
Employees. Private respondent Enrique Tamondong III, personnel
superintendent, actively involved himself in the formation of the union
and was even elected as one of its officers after its creation.
Consequently, petitioner CAPASCO sent a memo to private
respondent Tamondong requiring him to explain and to discontinue
from his union activities, with a warning that a continuance thereof
shall adversely affect his employment in the company.

Private respondent Tamondong ignored said warning and made


a reply letter invoking his right as a supervisory employee to join and
organize a labor union. In view of thereof, on 6 February 1997,
petitioner CAPASCO through a memo terminated the employment of
private respondent Tamondong on the ground of loss of trust and
confidence, citing his union activities as acts constituting serious
disloyalty to the company.

Private respondent Tamondong challenged his dismissal for


being illegal and as an act involving unfair labor practice by filing a
Complaint for Illegal Dismissal and Unfair Labor Practice before the
NLRC. According to him, there was no just cause for his dismissal
and it was anchored solely on his involvement and active participation
in the organization of the union of supervisory personnel in
CAPASCO. The NLRC, later reversed by the CA, ruled in favor of the
petitioner.

ISSUES:
2. Whether or not a supervisory employee is prohibited from
forming, assisting, or joining a labor organization.

373
Page 374

RULING:

1. No. Enrique Tamondong III may have possessed enormous


powers and was performing important functions that goes with the
position of Personnel Superintendent but, nevertheless, there was no
clear showing that he was at liberty, by using his own discretion and
disposition, to lay down and execute major business and operational
policies for and in behalf of CAPASCO. Petitioner CAPASCO
miserably failed to establish that private respondent Tamondong was
authorized to act in the interest of the company using his independent
judgment.

Being a supervisory employee of CAPASCO, he cannot be


prohibited from joining or participating in the union activities of private
respondent CUSE, and in making such a conclusion, the Court of
Appeals did not act whimsically, capriciously or in a despotic manner,
rather, it was guided by the evidence submitted before it. Thus, given
the foregoing findings of the Court of Appeals that private respondent
was a supervisory employee, it was indeed an exercise of an unfair
labor practice on the part of petitioner CAPASCO to dismiss him on
account of his union activities, thereby curtailing his constitutionally
guaranteed right to self-organization.

374
Page 375

Case Digest by: GINGOYON, CORDELL JERICHO M.

AIM vs. AIM FACULTY ASSOCIATION


G.R. No. 207971. January 23, 2017.
Del Castillo, J.

DOCTRINE:
ARTICLE 255.
In case of alleged inclusion of disqualified employees in a
union, the proper procedure for an employer is to directly file a
petition for cancellation of the union's certificate of registration due to
misrepresentation, false statement or fraud under the circumstances
enumerated in Article 239 of the Labor Code, as amended.

FACTS:
On May 16, 2007, respondent Management Faculty Association
filed a petition for certification election seeking to represent a
bargaining unit in Asian Institute of Management (AIM), duly
registered educational institution, consisting of forty faculty members.
Petitioner AIM opposed the petition, claiming that respondent's
members were neither rank-and-file nor supervisory, but rather,
managerial employees.

On July 11, 2007, petitioner AIM filed a petition for cancellation


of respondent's certificate of registration on the grounds of
misrepresentation in registration and that respondent is composed of
managerial employees who are prohibited from organizing as a
union. The petition for certification election was denied by the med-
arbiter because of the aforementioned ground.

Meanwhile, in DOLE Case No. NCR-OD-0707-001-LRD, an


Order dated February 16, 2009 was issued by DOLE-NCR Regional
Director Raymundo G. Agravante granting AIM's petition for
cancellation of respondent's certificate of registration and ordering its
delisting from the roster of legitimate labor organizations. This Order
was appealed by respondent before the Bureau labor Relations
(BLR), which, in a December 29, 2009 Decision, reversed the same
and ordered respondent's retention in the roster of legitimate labor
organizations. The BLR held that the grounds relied upon in the
petition for cancellation are not among the grounds authorized under
Article 239 of the Labor Code, and that respondent's members were
not managerial employees. Petitioner moved to reconsider, but was
rebuffed in a March 18, 2010 Resolution. But the Court of Appeals
reversed the decision on October 22, 2010.

Respondent sought reconsideration, but was denied. It thus instituted


a Petition for Review on Certiorari before this Court on July 4, 2011.

375
Page 376

The Petition, docketed as G.R. No. 197089, remained pending to


date.

ISSUES:
3. Whether or not misrepresentation as to the membership of a
labor organization may be a ground for the cancellation of its
certificate of registration.

RULING:

1. Yes. Petitioner's sole ground for seeking cancellation of


respondent's certificate of registration - that its members were
managerial employees and for this reason - was, in a sense, an
accusation that respondent was guilty of misrepresentation for
registering under the claim that its members were not managerial
employees.

However, the issue of whether respondent's members are


managerial employees was still pending resolution by way of petition
for review on certiorari in G.R. No. 197089, which is the culmination
of all proceedings in DOLE Case No. NCR-OD-M-0705-007 -- where
the issue relative to the nature of respondent's membership was first
raised by petitioner itself and was there fiercely contested. The
resolution of this issue cannot be pre-empted; until it is determined
with finality in G.R. No. l 97089, the petition for cancellation of
respondent's certificate of registration on the grounds alleged by
petitioner cannot be resolved. As a matter of courtesy and in order to
avoid conflicting decisions, the Court deferred from making a decision
pending the resolution of the petition in G.R. No. 197089.

376
Page 377

Case Digest by: GINGOYON, CORDELL JERICHO M.

HOLY CHILD CATHOLIC SCHOOL vs. HON. STO.


TOMAS
G.R. No. 179146. July 23, 2013.
Peralta, J.

DOCTRINE:
ARTICLE 256.
A bargaining unit has been defined as a "group of employees of
a given employer, comprised of all or less than all of the entire body
of employees, which the collective interests of all the employees,
indicated to be best suited to serve reciprocal rights and duties of the
parties under the collective bargaining provisions of the law."

FACTS:
On May 31 2002,
a petition for certification election was filed by
private respondent labor union. Petitioner Holy Child Catholic School
had ninety-eight teaching personnel, twenty-five non-teaching
academic employees and thirty-three non-teaching and nonacademic
employees. These one hundred fifty-six employees supported the
petition for certification election filed by Union. The School assailed
the legitimacy of the Union and its right to file a petition for certificate
election due to the commingling of rank-and-file and supervisory
employees.

On August 10, 2002, Med-Arbiter Agatha Ann L. Daquigan


denied the petition for certification election on the ground that the unit
which private respondent sought to represent was inappropriately
provided for.

This was a petition for certiorari with regards to the decision of


the Secretary of Labor that there should be two separate bargaining
units for those teaching and non-teaching personnel. The Court held
that the
Secretary’s decision was based on facts, law and jurisprudence
pointing to the need for two separate bargaining units for those
teaching and non-teaching personnel due to substantial differences in
interests.

ISSUES:
4. Whether or not a bargaining unit is the same as the concept of
a legitimate labor organization.

RULING:

377
Page 378

1. No. In determining the proper collective bargaining unit and


what unit would be appropriate to be the collective bargaining
agency, the Court, in the seminal case of Democratic Labor
Association v. Cebu Stevedoring Company, Inc., mentioned several
factors that should be considered, to wit: will of employees (Globe
Doctrine); affinity and unity of employees' interest, such as
substantial similarity of work and duties, or similarity of compensation
and working conditions; prior collective bargaining history; and
employment status, such as temporary, seasonal and probationary
employees. The Secretary of Labor did not abuse its discretion.

378
Page 379

Case Digest by: GINGOYON, CORDELL JERICHO M.

T & H SHOPFITTERS CORPORATION/GIN QUEEN


CORPORATION, STINNES HUANG, BEN HUANG AND
ROGELIO MADRIAGA vs, T & H SHOPFITTERS
CORPORATION/GIN QUEEN WORKERS UNION, ET AL.
G.R. No. 191714. February 26, 2014.
Mendoza, J.

DOCTRINE:
ARTICLES 258-260.
In essence, an unfair labor practice relates to the commission
of acts that transgress the workers’ right to organize. The prohibited
acts must necessarily relate to the workers’ right to self–organization.

FACTS:
The petitioner corporation committed the following acts: 1)
sponsoring a field trip to Zambales for its employees, to the exclusion
of union members, before the scheduled certification election; 2) the
active campaign by the sales officer of petitioners against the union
prevailing as a bargaining agent during the field trip; 3) escorting its
employees after the field trip to the polling center; 4) the continuous
hiring of subcontractors performing respondents’ functions; 5)
assigning union members to the Cabangan site to work as grass
cutters; and 6) the enforcement of work on a rotational basis for union
members.

The respondent filed suit against the petitioners for commission


of unfair labor practices. In its Decision, dated December 21, 2005,
the Labor Arbiter dismissed respondents’ complaint and all their
money claims for lack of merit. The National Labor Relations
Commission reversed the decision of the Labor Arbiter and ruled in
favor of the respondents, by which the Court of Appeals upheld the
same.

ISSUES:
5. Whether or not seemingly innocuous but disruptive actions that
coincide with a certificate election may constitute acts of unfair
labor practices.

RULING:

1. Yes. Indubitably, the various acts of petitioners, taken together,


reasonably support an inference that, indeed, such were all
orchestrated to restrict respondents’ free exercise of their right to self–
organization. The Court was of the considered view that petitioners’

379
Page 380

undisputed actions prior and immediately before the scheduled


certification election, while seemingly innocuous, unduly meddled in
the affairs of its employees in selecting their exclusive bargaining
representative. Thus, petitioners had no business persuading and/or
assisting its employees in their legally protected independent process
of selecting their exclusive bargaining representative. The fact and
peculiar timing of the field trip sponsored by petitioners for its
employees not affiliated with THS–GQ Union, although a positive
enticement, was undoubtedly extraneous influence designed to
impede respondents in their quest to be certified. This cannot be
countenanced.

Not content with achieving a “no union” vote in the certification


election, petitioners launched a vindictive campaign against union
members by assigning work on a rotational basis while subcontractors
performed the latter’s functions regularly. Worse, some of the
respondents were made to work as grass cutters in an effort to
dissuade them from further collective action. Again, this cannot be
countenanced.

More importantly, petitioners’ bare denial of some of the


complained acts and unacceptable explanations, a mere afterthought
at best, cannot prevail over respondents’ detailed narration of the
events that transpired. At this juncture, it bears to emphasize that in
labor cases, the quantum of proof necessary is substantial evidence,
or that amount of relevant evidence as a reasonable mind might accept
as adequate to support a conclusion, even if other minds, equally
reasonable, might conceivably opine otherwise.

380
Page 381

Case Digest by: GINGOYON, CORDELL JERICHO M.

REN TRANSPORT vs. NLRC


G.R. No. 188020. June 27, 2016.
Sereno, C.J.

DOCTRINE:
ARTICLES 258-260.
Violation of the duty to bargain collectively is an unfair labor
practice under Article 258(g) of the Labor Code, as amended.

FACTS:
Private respondent Samahan ng Manggagawa sa Ren
Transport (SMART) was a registered union, which had a five-year
collective bargaining agreement (CBA) with Ren Transport Corp. set
to expire on 31 December 2004. The 60-day freedom period of the
CBA passed without a challenge to SMART'S majority status as
bargaining agent. SMART thereafter conveyed its willingness to
bargain with Ren Transport, to which it sent bargaining proposals.
Ren Transport failed to reply to the demand.

Subsequently, two members of SMART wrote to the Department of


Labor and Employment - National Capital Region (DOLE-NCR). The
office was informed that a majority of the members of SMART had
decided to disaffiliate from their mother federation to form another
union, Ren Transport Employees Association (RTEA). SMART
contested the alleged disaffiliation through a letter dated 4 April 2005.

During the pendency of the disaffiliation dispute at the DOLE-NCR,


Ren Transport stopped the remittance to SMART of the union dues
that had been checked off from the salaries of union workers as
provided under the CBA. Further, on 19 April 2005, Ren Transport
voluntarily recognized RTEA as the sole and exclusive bargaining
agent of the rank-and-file employees of their company.

On 6 July 2005, SMART filed with the labor arbiter a complaint for
unfair labor practice against Ren Transport. The Labor Arbiter, as
later affirmed by the NLRC and the Court of Appeals, ruled in favor of
the private respondent.

ISSUES:
6. Whether or not disaffiliation of the majority of the rank-and-file
employees may be a valid ground for refusing to collectively
bargain.

RULING:

381
Page 382

1. No. Under Article 263 in relation to Article 267 of the Labor


Code, it is during the freedom period — or the last 60 days before the
expiration of the CBA — when another union may challenge the
majority status of the bargaining agent through the filing of a petition
for a certification election. If there is no such petition filed during the
freedom period, then the employer "shall continue to recognize the
majority status of the incumbent bargaining agent where no petition
for certification election is filed.”

In the present case, no petition for certification election


challenging the majority status of SMART was filed during the
freedom period, which was from November 1 to December 31, 2004
— the 60-day period prior to the expiration of the five-year CBA.
SMART therefore remained the exclusive bargaining agent of the
rank-and-file employees.

Given that SMART continued to be the workers' exclusive


bargaining agent, Ren Transport had the corresponding duty to
bargain collectively with the former. Ren Transport's refusal to do so
constitutes an unfair labor practice. Consequently, Ren Transport
cannot avail itself of the defense that SMART no longer represents
the majority of the workers. The fact that no petition for certification
election was filed within the freedom period prevented Ren Transport
from challenging SMART'S existence and membership.

Furthermore, the labor arbiter found that the failure to remit the
union dues to SMART and the voluntary recognition of RTEA were
clear indications of interference with the employees' right to self-
organization.

382
Page 383

Case Digest by: GINGOYON, CORDELL JERICHO M.

DIGITAL TELECOMMUNICATIONS PHILIPPINES, INC. vs.


DIGITEL EMPLOYEES UNION
G.R. No. 184903. October 10, 2012.
Perez, J.

DOCTRINE:
ARTICLES 258-260.
Contracting out services for functions normally performed by
union members constitutes unfair labor practice if coupled with bad
faith.

FACTS:
On 4 November 2004, the President and then members of
Digiserv Union filed a case for preventive mediation before the
National Conciliation Mediation Board with the petitioner. Digitel
Telecommunications refused to negotiate with Digiserv supposedly
due to a long lapse of time of no communication. This was despite
the orders of the Secretary of Labor.

During the pendency of the suit, Interactive Solutions, Inc. (I-


Tech) was incorporated. It performed the services normally provided
by Digitel Service, Inc. I-techs primary purpose was to provide call
center/customer contact service, the same service provided by
Digiserv. It conducts its business inside the Digitel office at 110 E.
Rodriguez Jr. Avenue, Bagumbayan, Quezon City. The former head
of Digiserv, Ms. Teresa Taniega, was also an officer of I-tech.
Partially because of this instance, Digiserv filed a Termination Report
stating that it will cease its business operation. The closure affected
at least one hundred employees, forty-two of whom are members of
the herein respondent Union.

A complaint for damages was filed against the petitioner by the


respondents, which were ultimately ruled in favor of by the Court of
Appeals.

ISSUES:
7. Whether or not the contracting out of services of functions
normally provided for by the union members to a newly created
corporation may constitute an act of an unfair labor practice.

RULING:

1. Yes. The timing of the creation of I-tech was dubious. It was


incorporated on 18 January 2005 while the labor dispute within Digitel

383
Page 384

was pending. Thus, when Digiserv was closed down, some of the
employees presumably non-union members were rehired by I-tech.

The closure of Digiserv pending the existence of an assumption


order coupled with the creation of a new corporation, performing
similar functions as Digiserv, leaves no iota of doubt that the target of
the closure were the union member-employees. These factual
circumstances proved that Digitel terminated the services of the
affected employees to defeat their security of tenure. The termination
of service was not a valid retrenchment; it was an illegal dismissal of
employees.

It was mentioned too that the dismissal constitutes an unfair


labor practice under Article 248(c) of the Labor Code which refers to
contracting out services or functions being performed by union
members when such will interfere with, restrain or coerce employees
in the exercise of their rights to self-organization. At the height of the
labor dispute, occasioned by Digitels reluctance to negotiate with the
Union, I-tech was formed to provide, as it did provide, the same
services performed by Digiserv, the Union members nominal
employer.

384
Page 385

Case Digest by: GINGOYON, CORDELL JERICHO M.

GENERAL SANTOS COCA-COLA PLANT FREE


WORKERS UNION TUPAS vs. COCA-COLA BOTTLERS
INC.
G.R. No. 178647. February 13, 2009.
Nachura, J.

DOCTRINE:
ARTICLES 258-260.
Contracting out services does not amount to an exercise of an
unfair labor practice if it does not amount to a violation of the right of
the workers to self-organization.

FACTS:
Respondent Coca-Cola Bottlers Phil., Inc. experienced a
significant decline in profitability due to the Asian economic crisis.
Thus to curb the negative effects on the company, it implemented
three waves of an Early Retirement Program. An inter-office
memorandum was also issued mandating to put on hold all requests
for hiring to fill in vacancies in both regular and temporary positions in
the Head Office and in the Plants.

Faced with the “freeze hiring” directive, CCBPI Gen San


engaged the services of JLBP Services Corporation, a manning
agency. Petitioner Union then filed with the National Conciliation and
Mediation Board a Notice of Strike on the ground of alleged unfair
labor practice committed by CCBPI Gen San for contracting-out
services regularly performed by union members.

In a Resolution, the NLRC ruled that CCBPI was not guilty of


unfair labor practice for contracting out jobs to JLBP. The NLRC held
that petitioner failed to prove by substantial evidence that the system
was meant to curtail the right to self-organization of petitioner’s
members. Later, the CA upheld the NLRC’s finding that CCBPI was
not guilty of unfair labor practice. It held that the contract between
CCBPI and JLBP did not amount to labor-only contracting. It found
that JLBP was an independent contractor and that the decision to
contract out jobs was a valid exercise of management prerogative to
meet exigent circumstances.

ISSUES:
8. Whether or not contracting out services normally performed by
union members automatically constitutes an exercise of an
unfair labor practice.

385
Page 386

RULING:

1. No. Unfair labor practice refers to "acts that violate the workers'
right to organize." The prohibited acts are related to the workers' right
to self-organization and to the observance of a CBA. Without that
element, the acts, even if unfair, are not unfair labor practices. Both
the NLRC and the CA found that petitioner was unable to prove its
charge of unfair labor practice. It was the Union that had the burden
of adducing substantial evidence to support its allegations of unfair
labor practice,17 which burden it failed to discharge.

386
Page 387

Case Digest by: GINGOYON, CORDELL JERICHO M.

UST FACULTY UNION vs. UNIVERSITY OF SANTO


TOMAS
G.R. No. 180892. April 7, 2009.
Velasco, JR., J.

DOCTRINE:
ARTICLES 258-260.
In order to show that the employer committed ULP under the
Labor Code, substantial evidence is required to support the claim.
Substantial evidence has been defined as such relevant evidence as
a reasonable mind might accept as adequate to support a conclusion.
In other words, whether the employee or employer alleges that the
other party committed ULP, it is the burden of the alleging party to
prove such allegation with substantial evidence. Such principle finds
justification in the fact that ULP is punishable with both civil and/or
criminal sanctions.

FACTS:

On September 21, 1996, the University of Santo Tomas Faculty


Union (USTFU) wrote a letter to all its members informing them of a
General Assembly that was to be held on October 5, 1996. The letter
contained an agenda for the GA which included an election of
officers. The then incumbent president of the USTFU was Atty.
Eduardo J. Mariño, Jr.

On October 2, 1996, Fr. Rodel Aligan, O.P., Secretary General


of the UST, issued a Memorandum allowing the request of the
Faculty Clubs of the university to hold a convocation on October 4,
1996.

A election was conducted among those present, and Gil Gamilla and
other faculty members (Gamilla Group) were elected as the president
and officers, respectively, of the union. Such election was
communicated to the UST administration in a letter dated October 4,
1996. Thus, there were two groups claiming to be the USTFU: the
Gamilla Group and the group led by Atty. Mariño, Jr.

On October 8, 1996, the Mariño Group filed a complaint for


ULP against the UST with the Arbitration Branch of the NLRC. It also
filed on October 11, 1996 a complaint with the Office of the Med-
Arbiter of the Department of Labor and Employment (DOLE), praying
for the nullification of the election of the Gamilla Group as officers of
the USTFU. On December 3, 1996, a Collective Bargaining
Agreement was entered into by the Gamilla Group and the UST. The
CBA superseded an existing CBA entered into by the UST and

387
Page 388

USTFU which was intended for the period of June 1, 1993 to May 31,
1998.

On February 11, 1997, the med-arbiter issued a Resolution, declaring


the election of the Gamilla group as null and void and ordering that
this group cease and desist from performing the duties and
responsibilities of USTFU officers. This Resolution was appealed to
the Director of the Bureau of Labor Relations (BLR), which was
denied.

Thus, on January 21, 2000, USTFU filed a Manifestation with the


Arbitration Branch of the NLRC, informing it of the Decision of the
Court. Thereafter, on August 15, 2003, the Arbitration Branch of the
NLRC issued a Decision dismissing the complaint for lack of merit.

The complaint was dismissed on the ground that USTFU failed to


establish with clear and convincing evidence that indeed UST was
guilty of ULP. In his Decision, the labor arbiter explained that the
alleged Memorandum dated October 2, 1996 merely granted the
request of faculty members to hold such convocation. Moreover, by
USTFU’s own admission, no member of the UST administration
attended or participated in the convocation.

As to the CBA, the labor arbiter ruled that when the new CBA was
entered into, the Gamilla Group presented more than sufficient
evidence to establish that they had been duly elected as officers of
the USTFU; and the ruling of the med-arbiter that the election of the
Gamilla Group was null and void was not yet final and executory.
Thus, UST was justified in dealing with and entering into a CBA with
the Gamilla Group, including helping the Gamilla Group in securing
the USTFU office.

ISSUES:
9. Whether or not the employer has the utmost burden to inquire
as to which among the conflicting unions is the exclusive
bargaining agent for certain employees.

RULING:

1. No. The records are bereft of any evidence to show that the
Mariño Group informed the UST of their objections to the election of
the Gamilla Group. In fact, there is even no evidence to show that the
scheduled elections on October 5, 1996 that was supposed to be
presided over by the Mariño Group ever pushed through. Instead,
petitioner filed a complaint with the med-arbiter on October 11, 1996
praying for the nullification of the election of the Gamilla Group.

As such, there was no reason NOT to recognize the Gamilla


Group as the new officers and directors of USTFU. And as stated in

388
Page 389

the above-quoted provisions of the Labor Code, the UST was


obligated to deal with the USTFU, as the recognized representative of
the bargaining unit, through the Gamilla Group. UST’s failure to
negotiate with the USTFU would have constituted ULP.

It is not the duty or obligation of respondents to inquire into the


validity of the election of the Gamilla Group. Such issue is properly an
intra-union controversy subject to the jurisdiction of the med-arbiter of
the DOLE. Respondents could not have been expected to stop
dealing with the Gamilla Group on the mere accusation of the Mariño
Group that the former was not validly elected into office.

389
Page 390

Case Digest by: GINGOYON, CORDELL JERICHO M.

PHIL SKYLANDERS INC. vs. NLRC, ET AL.


G.R. No. 127374. January 31, 2002.
Bellosillo, JR., J.

DOCTRINE:
ARTICLES 258-260.
The local unions remain the basic units of association, free to
serve their own interests subject to the restraints imposed by the
constitution and by-laws of the national federation, and free also to
renounce the affiliation upon the terms laid down in the agreement
which brought such affiliation into existence. An agent of a local union
cannot file suit for an exercise of an unfair labor practice if not
provided for by the principal.

FACTS:

In November 1993 the Philippine Skylanders Employees


Association (PSEA), a local labor union affiliated with the Philippine
Association of Free Labor Unions (PAFLU) September (PAFLU), won
in the certification election conducted among the rank and file
employees of Philippine Skylanders, Inc. (PSI). Its rival union,
Philippine Skylanders Employees Association-WATU (PSEA-WATU)
immediately protested the result of the election before the Secretary
of Labor.

Several months later, pending settlement of the controversy,


PSEA sent PAFLU a notice of disaffiliation citing as reason PAFLU's
supposed deliberate and habitual dereliction of duty toward its
members. Attached to the notice was a copy of the resolution
adopted and signed by the officers and members of PSEA
authorizing their local union to disaffiliate from its mother federation.

PSEA subsequently affiliated itself with the National Congress


of Workers (NCW), changed its name to Philippine Skylanders
Employees Association - National Congress of Workers (PSEA-
NCW), and to maintain continuity within the organization, allowed the
former officers of PSEA-PAFLU to continue occupying their positions
as elected officers in the newly-forged PSEA-NCW.

On 17 March 1994 PSEA-NCW entered into a collective


bargaining agreement with PSI which was immediately registered
with the Department of Labor and Employment.

Later Agitated by PSI's recognition of PSEA-NCW, PAFLU


through Serafin Ayroso filed two consolidated complaints for unfair

390
Page 391

labor practice against PSI, its president Mariles Romulo and


personnel manager Francisco Dakila.

In a Decision rendered on 30 June 1995 the Labor Arbiter declared


PSEA's disaffiliation from PAFLU invalid and held PSI, PSEA-PAFLU
and their respective officers guilty of unfair labor practice. The
Decision explained that despite PSEA-PAFLU's status as the sole
and exclusive bargaining agent of PSI's rank and file employees, the
company knowingly sanctioned and confederated with Dakila in
actively assisting a rival union.

PSI, PSEA and their respective officers appealed to the National


Labor Relations Commission (NLRC). But the NLRC upheld the
Decision of the Labor Arbiter and conjectured that since an election
protest questioning PSEA-PAFLU's certification as the sole and
exclusive bargaining agent was pending resolution before the
Secretary of Labor, PSEA could not validly separate from PAFLU,
join another national federation and subsequently enter into a
collective bargaining agreement with its employer-company.

Petitioners separately moved for reconsideration but both motions


were denied.

ISSUES:
10. Whether or not an independent and separate local union
may validly disaffiliate from a federation pending the settlement
of an election protest questioning the status of another union as
the sole and exclusive bargaining agent of rank and file
employees.
11. Whether or not a local union may always file suit for unfair
labor practice despite disaffiliation.

RULING:

1. Yes. The right of a local union to disaffiliate from its mother


federation is not a novel thesis unillumined by case law. In the
landmark case of Liberty Cotton Mills Workers Union vs. Liberty
Cotton Mills, Inc. the Court upheld the right of local unions to separate
from their mother federation on the ground that as separate and
voluntary associations, local unions do not owe their creation and
existence to the national federation to which they are affiliated but,
instead, to the will of their members. The sole essence of affiliation is
to increase, by collective action, the common bargaining power of
local unions for the effective enhancement and protection of their

391
Page 392

interests. Admittedly, there are times when without succor and


support local unions may find it hard, unaided by other support
groups, to secure justice for themselves.

Upon an application of the aforecited principle to the issue at


hand, the impropriety of the questioned Decisions becomes clearly
apparent. There is nothing shown in the records nor is it claimed by
PAFLU that the local union was expressly forbidden to disaffiliate
from the federation nor were there any conditions imposed for a valid
breakaway. As such, the pendency of an election protest involving
both the mother federation and the local union did not constitute a bar
to a valid disaffiliation.

2. No. It stands unchallenged that PAFLU instituted the complaint


for unfair labor practice against the wishes of workers whose interests
it was supposedly protecting. The mere act of disaffiliation did not
divest PSEA of its own personality; neither did it give PAFLU the
license to act independently of the local union. Recreant to its
mission, PAFLU cannot simply ignore the demands of the local
chapter and decide for its welfare. PAFLU might have forgotten that
as an agent it could only act in representation of and in accordance
with the interests of the local union. The complaint then for unfair
labor practice lodged by PAFLU against PSI, PSEA and their
respective officers, having been filed by a party which has no legal
personality to institute the complaint, should have been dismissed at
the first instance for failure to state a cause of action.

392
Page 393

Case Digests by: GIRON, JOSE JUSTIN C.

TROPICAL HUT EMPLOYEE’S UNION v.


TROPICAL HUT FOOT MARKET INC.
G.R. No. L-43495-99. January 20, 1990.
Medialdea, J.

DOCTRINE:
DISAFFILIATION BY THE LEGITIMATE LABOR ORGANIZATION
FROM THE FEDERATION IS NOT VIOLATIVE OF THE UNION
SECURITY CLAUSE.
When the THEU disaffiliated from its mother federation, the former
did not lose its legal personality as the bargaining union under the CBA.
A local union owes its creation and continued existence to the will of
its members and not to the federation to which it belongs. When the
local union withdrew from the old federation to join a new federation, it
was merely exercising its primary right to labor organization for the
effective enhancement and protection of common interests. In the
absence of enforceable provisions in the federation's constitution
preventing disaffiliation of a local union, a local may sever its
relationship with its parent.

FACTS:
On January 2, 1968, the rank and file workers of the Tropical Hut
Food Market, Incorporated, referred to herein as respondent company,
organized a local union called the Tropical Hut Employees Union,
known for short as the THEU, elected their officers, adopted their
constitution and by-laws and immediately sought affiliation with the
National Association of Trade Unions (NATU). On January 3, 1968, the
NATU accepted the THEU application for affiliation. Following such
affiliation with NATU, Registration Certificate No 5544-IP was issued
by the Department of Labor in the name of the Tropical Hut Employees
Union — NATU. It appears, however, that NATU itself as a labor
federation, was not registered with the Department of Labor.

After several negotiations were conducted between THEU-NATO,


represented by its local president and the national officers of the
NATU, its President, Executive Vice- President, Vice President, and
respondent Tropical Hut Food Market, Incorporated, thru its President
and General Manager, a Collective Bargaining Agreement.

The said agreement clearly expressed that the Union is a


legitimate labor organization and that the Company duly recognizes
the said Union to be the sole and exclusive bargaining agent. Said
agreement also contains the following Union Security Clause:

393
Page 394

Article III, Section 3 — Any employee who is expelled from the


UNION for joining another federation or forming another union, or who
fails or refuses to maintain his membership therein as required, xxx
shall, upon written request of the UNION, be discharged by the
COMPANY."

Thereafter, NATU received a letter, jointly signed by the incumbent


officers of the local union informing NATU that THEU was disaffiliating
from the NATU federation and its affiliation with the Confederation of
General Workers (CGW). This letter was passed around among the
members of the THEU-NATU, to which around 137 signatures
appeared as having given their consent to and acknowledgment of the
decision to disaffiliate the THEU from the NATU.

NATU federation wrote a letter to respondent company directing


the latter to dismiss Encinas, the president of the THEU-NATU, in view
of his violation of the Union Security Clause. The respondent company
suspended Encinas pending the application for clearance with the
DOLE to dismiss him. It was the suspension of Encinas that caused
the filing of NLRC Case against private respondents here, charging
them of unfair labor practice.

On February 24, 1974, the secretary of THEU-NATU, notified the


entire rank and file employees of the company that they will be given
48 hours upon receipt of the notice within which to answer and affirm
their membership with THEU-NATU. When the petitioner employees
failed to reply, THEU-NATU advised them thru letters in 3 different
dates that the THEU-NATU shall enforce the union security clause set
forth in the CBA, and that they had requested respondent company to
dismiss them. No comment or reply was received from petitioners,
hence 63 workers were suspended pending approval by the SOLE for
their dismissal

In view of said actions by the company, a letter signed by 146


members of the THEU-CGW was sent to the President of the
Philippines informing him of the unfair labor practice committed by
private respondents against THEU-CGW members.

ISSUE:
Whether or not the respondent company, in dismissing the
president of the THEU-NATU, committed an unfair labor practice.

RULING:
Yes.

394
Page 395

In the celebrated case of Liberty Cotton Mills Workers Union v.


Liberty Cotton Mills, L-33187, September 4, 1975, 66 SCRA 512, the
Supreme Court held that the validity of the dismissals pursuant to the
union security clause in the collective bargaining agreement hinges on
the validity of the disaffiliation of the local union from the federation.

The right of a local union to disaffiliate from its mother federation


is well-settled. A local union, being a separate and voluntary
association, is free to serve the interest of all its members including the
freedom to disaffiliate when circumstances warrant. This right is
consistent with the constitutional guarantee of freedom of association.

The inclusion of the word NATU after the name of the local union
THEU in the registration with the Department of Labor is merely to
stress that the THEU is NATU's affiliate at the time of the registration.
It does not mean that the said local union cannot stand on its own.
Neither can it be interpreted to mean that it cannot pursue its own
interests independently of the federation. A local union owes its
creation and continued existence to the will of its members and not to
the federation to which it belongs. When the local union withdrew from
the old federation to join a new federation, it was merely exercising its
primary right to labor organization for the effective enhancement and
protection of common interests. In the absence of enforceable
provisions in the federation's constitution preventing disaffiliation of a
local union, a local may sever its relationship with its parent.

There is nothing in the constitution of the NATU or in the


constitution of the THEU-NATU that the THEU was expressly
forbidden to disaffiliate from the federation.

Further, there is no merit in the contention of the respondents that


the act of disaffiliation violated the union security clause of the CBA
and that their dismissal as a consequence thereof is valid. A perusal of
the collective bargaining agreements shows that the THEU-NATU, and
not the NATU federation, was recognized as the sole and exclusive
collective bargaining agent for all its workers and employees in all
matters concerning wages, hours of work and other terms and
conditions of employment.

Although NATU was designated as the sole bargaining agent in


the check-off authorization form attached to the CBA, this simply
means it was acting only for and in behalf of its affiliate. The NATU
possessed the status of an agent while the local union remained the
basic principal union which entered into contract with the respondent
company. When the THEU disaffiliated from its mother federation, the
former did not lose its legal personality as the bargaining union under

395
Page 396

the CBA. Moreover, the union security clause embodied in the


agreements cannot be used to justify the dismissals meted to
petitioners since it is not applicable to the circumstances obtaining in
this case.

The CBA imposes dismissal only in case an employee is expelled


from the union for joining another federation or for forming another
union or who fails or refuses to maintain membership therein. The case
at bar does not involve the withdrawal of merely some employees from
the union but of the whole THEU itself from its federation.

Clearly, since there is no violation of the union security provision


in the CBA, there was no sufficient ground to terminate the
employment of petitioners.

396
Page 397

Case Digests by: GRION, JOSE JUSTIN C.

PUREFOODS CORP. v. NAGKAKAISANG SAMAHANG


MANGAGAWA NG PUREFOODS RANK AND FILE
G.R. No. 150896. August 28, 2008.
NACHURA, J:

DOCTRINE:
UNFAIR LABOR PRACTICE BY THE EMPLOYER; BADGES OF
BAD FAITH; SUDDEN TRANSFER OF INVENTORY AND
TERMINATION OF EMPLOYEES.
The Supreme Court held that the sudden termination of the
STFWU members is tainted with ULP because it was done to because
it was done to interfere with, restrain or coerce employees in the
exercise of their right to self-organization.

FACTS:
Three labor organizations and a federation are respondents in this
case — Nagkakaisang Samahang Manggagawa Ng Purefoods Rank-
And-File (NAGSAMA-Purefoods), the exclusive bargaining agent of
the rank-and-file workers of Purefoods' meat division throughout
Luzon; St. Thomas Free Workers Union (STFWU), of those in the farm
in Sto. Tomas, Batangas; and Purefoods Grandparent Farm Workers
Union (PGFWU), of those in the poultry farm in Sta. Rosa, Laguna.
These organizations were affiliates of the respondent federation,
Purefoods Unified Labor Organization (PULO).

On February 8, 1995, NAGSAMA-Purefoods manifested to


petitioner corporation its desire to re-negotiate the collective bargaining
agreement (CBA) then due to expire on the 28th of the said month.
Together with its demands and proposal, the organization submitted to
the company its January 28, 1995 General Membership Resolution
approving and supporting the union's affiliation with PULO, adopting
the draft CBA proposals of the federation, and authorizing a negotiating
panel which included among others a PULO representative. While
Purefoods formally acknowledged receipt of the union's proposals, it
refused to recognize PULO and its participation, even as a mere
observer, in the negotiation. Consequently, notwithstanding the PULO
representative's non-involvement, the negotiation of the terms of the
CBA still resulted in a deadlock. A notice of strike was then filed by
NAGSAMA-Purefoods on May 15, 1995. In the subsequent conciliation
conference, the deadlock issues were settled except the matter of the
company's recognition of the union's affiliation with PULO.

397
Page 398

On July 24, 1995, however, the petitioner company concluded a


new CBA with another union in its farm in Malvar, Batangas. Five days
thereafter, or on July 29, 1995, at around 8:00 in the evening, four
company employees facilitated the transfer of around 23,000 chickens
from the poultry farm in Sto. Tomas, Batangas (where STFWU was the
exclusive bargaining agent) to that in Malvar. The following day, the
regular rank-and-file workers in the Sto. Tomas farm were refused
entry in the company premises; and on July 31, 1995, 22 STFWU
members were terminated from employment. The farm manager,
supervisors and electrical workers of the Sto. Tomas farm, who were
members of another union, were nevertheless retained by the
company in its employ.

Aggrieved by these developments, the four respondent labor


organizations jointly instituted a complaint for unfair labor practice
(ULP), illegal lockout/dismissal and damages, docketed as NLRC
Case No. NLRCNCR- 00-07-05159-95, with the Labor Arbitration
Branch of the National Labor Relations Commission (NLRC).

The LA rendered a Decision dismissing the complaint, and


declaring that the company neither committed ULP nor illegally
dismissed the employees.

On appeal, the NLRC reversed the ruling of the LA, ordered the
payment of P500,000.00 as moral and exemplary damages and the
reinstatement with full backwages of the STFWU members. The NLRC
stated that the labor commission ruled that the petitioner company's
refusal to recognize the labor organizations' affiliation with PULO was
unjustified considering that the latter had been granted the status of a
federation by the Bureau of Labor Relations; and that this refusal
constituted undue interference in, and restraint on the exercise of the
employees' right to self-organization and free collective bargaining.
The NLRC said that the real motive of the company in the sudden
closure of the Sto. Tomas farm and the mass dismissal of the STFWU
members was union busting, as only the union members were locked
out, and the company subsequently resumed operations of the closed
farm under a new contract with the landowner.

CA dismissed outright the company’s petition for certiorari on the


ground that the verification and certification of non-forum shopping was
defective.

Hence this petition for review before the SC under Rule 45.

ISSUE:
Whether or not the company committed an unfair labor practice.

398
Page 399

RULING:
Yes.

It is crystal clear that the closure of the Sto. Tomas farm was made
in bad faith. Badges of bad faith are evident from the following acts of
the petitioner: it unjustifiably refused to recognize the STFWU's and
the other unions' affiliation with PULO; it concluded a new CBA with
another union in another farm during the agreed indefinite suspension
of the collective bargaining negotiations; it surreptitiously transferred
and continued its business in a less hostile environment; and it
suddenly terminated the STFWU members, but retained and brought
the non-members to the Malvar farm. Petitioner presented no evidence
to support the contention that it was incurring losses or that the subject
farm's lease agreement was preterminated. Ineluctably, the closure of
the Sto. Tomas farm circumvented the labor organization's right to
collective bargaining and violated the members' right to security of
tenure.

The Supreme Court held that the sudden termination of the


STFWU members is tainted with ULP because it was done to because
it was done to interfere with, restrain or coerce employees in the
exercise of their right to self-organization. Thus, the petitioner company
is liable for the payment of the aforesaid damages. As to the order of
reinstatement, the Court modifies the same in that if it is no longer
feasible considering the length of time that the employees have been
out of petitioner's employ, the company is ordered to pay the illegally
dismissed STFWU members separation pay equivalent to one (1)
month pay, or one-half (1/2) month pay for every year of service,
whichever is higher.

399
Page 400

Case Digests by: GIRON, JOSE JUSTIN C.

DE LA SALLE UNIVERSITY v. DLSUE-NAFTEAU


G.R. No. 1777283. April 7, 2009.
Morales, J.

DOCTRINE:
UNFAIR LABOR PRACTICE; INTERFERENCE BY THE
EMPLOYER.
The employer cannot withhold union and agency dues and
suspend normal relations with the union’s incumbent set of officers
pending the intra-union dispute because it constitutes “interference”.
Pending the final resolution of the intra-union dispute, respondent's
officers remained duly authorized to conduct union affairs.

FACTS:
In 2001, a splinter group of respondent led by one Belen Aliazas
(Aliazas group) filed a petition for conduct of elections with the
Department of Labor and Employment (DOLE), alleging that the then
incumbent officers of respondent had failed to call for a regular election
since 1985. Disputing the Aliazas group's allegation, respondent
claimed that an election was conducted in 1987 but by virtue of the
enactment of Republic Act 6715, which amended the Labor Code, the
term of office of its officers was extended to five years or until 1992
during which a general assembly was held affirming their hold-over
tenure until the termination of collective bargaining negotiations; and
that a collective bargaining agreement (CBA) was executed only on
March 30, 2000.

Acting on the petition for the conduct of election, the DOLE-NCR


held that the hold over authority of respondent’s incumbent set of
officers had been extinguished by virtue of the execution of the CBA.
It ordered the conduction of election to be placed under the control and
supervision of its Labor Relations Division.

Aliazas group requested the University “to please put on escrow


all union dues/agency fees and whatever money considerations
deducted from salaries of concerned co-academic personnel until such
time that an election of union officials had been scheduled and
subsequent elections has been held”

Responding to the Aliazas group's request, petitioners, advised


respondent that “By virtue of the 19 March 2001 Decision and the 06
July 2001 Order of the Department of Labor and Employment (DOLE),

400
Page 401

the holdover authority of your incumbent set of officers has been


considered extinguished and an election of new union officers, to be
conducted and supervised by the DOLE has been directed to be held.
Until the result of this election comes out and a declaration by the
DOLE of the validly elected officers is made, a void in the Union
leadership exists.

In the light of these circumstances, the University has no other


alternative but to temporarily do the following:

1. Establish a savings account for the Union where all collected


union dues and agency fees will be deposited and held in trust; and

2. Discontinue normal relations with any group within the Union


including the incumbent set of officers.”

Petitioners' above-quoted move drew respondent to file a


complaint against petitioners for Unfair Labor Practice (ULP
complaint), claiming that petitioners unduly interfered with its internal
affairs and discriminated against its members.

ISSUE:
Whether or not the acts of withholding union and agency dues and
suspension of normal relations with respondent’s incumbent set of
officers pending the intra-union dispute constitutes “interference” as to
amount to an unfair labor practice.

RULING:
Yes.

Pending the final resolution of the intra-union dispute,


respondent's officers remained duly authorized to conduct union
affairs. The 19 March 2001 Decision of DOLE-NCR Regional Director
should not be construed as an automatic termination of the incumbent
officers' tenure of office. As duly elected officers of the DLSUEA, their
leadership is not deemed terminated by the expiration of their terms of
office, for they shall continue their functions and enjoy the rights and
privileges pertaining to their respective positions in a hold-over
capacity, until their successors shall have been elected and qualified.

It bears noting that at the time petitioners' questioned moves were


adopted, a valid and existing CBA had been entered between the
parties. It thus behooved petitioners to observe the terms and
conditions thereof bearing on union dues and representation. It is
axiomatic in labor relations that a CBA entered into by a legitimate

401
Page 402

labor organization and an employer becomes the law between the


parties, compliance with which is mandated by express policy of the
law.

402
Page 403

Case Digests by: GIRON, JOSE JUSTIN C.

MSMG-UWP v. Hon. Ramos


G.R. No. 113907. February 28, 2000.
Purisima, J.

DOCTRINE:
UNFAIR LABOR PRACTICE, DISMISSAL OF UNION OFFICERS
WITHOUT GRANTING THEM DUE PROCESS.
The enforcement of union security clauses is authorized by law
provided such enforcement is not characterized by arbitrariness, and
always with due process. Even on the assumption that the federation
had valid grounds to expel the union officers, due process requires that
these union officers be accorded a separate hearing by respondent
company.

FACTS:
The petitioner, Malayang Samahan ng mga Manggagawa sa M.
Greenfield, Inc., (B) (MSMG), hereinafter referred to as the "local
union," is an affiliate of the private respondent, United Lumber and
General Workers of the Philippines (ULGWP), referred to as the
"federation."

The collective bargaining agreement of MSMG and M. Greenfield,


Inc. contains a union security clause and a program fund.

A local union election was held under the auspices of the ULGWP
wherein the herein petitioner, Beda Magdalena Villanueva, and the
other union officers were proclaimed as winners. Thereafter, a Petition
for Impeachment was filed with the national federation ULGWP by the
defeated candidates in the aforementioned election.

The local union held a general membership meeting at the


Caruncho Complex in Pasig. Several union members failed to attend
the meeting, prompting the Executive Board to create a committee
tasked to investigate the non-attendance of several union members in
the said assembly. This prompted the local union to write to respondent
a letter requesting it to deduct the union fines from the wages/salaries
of those union members who failed to attend the general membership
meeting.

The Federation wrote respondent company a letter advising the


latter not to deduct the fifty-peso fine from the salaries of the union
members. The following day, respondent company sent a reply to

403
Page 404

petitioner union's request in a letter, stating that it cannot deduct fines


from the employees' salary without going against certain laws.

The imposition of P50.00 fine became the subject of bitter


disagreement between the Federation and the local union culminating
in the latter's declaration of general autonomy from the former through
Resolution No. 10 passed by the local executive board and ratified by
the general membership on July 16, 1988.

In retaliation, the national federation asked respondent company


to stop the remittance of the local union's share in the education funds
effective August 1988. This was objected to by the local union which
demanded that the education fund be remitted to it in full.

Meanwhile, on September 2, 1988, several local unions (Top


Form, M. Greenfield, Grosby, Triumph International, General Milling,
and Vander Hons chapters) filed a Petition for Audit and Examination
of the federation and education funds of ULGWP which was granted
by Med-Arbiter on December 25, 1988 in an Order which directed the
audit and examination of the books of account of ULGWP.

Thereafter officials of ULGWP called a Special National Executive


Board Meeting at Nasipit, Agusan del Norte where a Resolution was
passed placing the MSMG under trusteeship and appointing
respondent Cesar Clarete as administrator. Said administrator wrote
the respondent company informing the latter of its designation of a
certain Alfredo Kalingking as local union president and "disauthorizing"
the incumbent union officers from representing the employees.

Then petitioner union officers received identical letters from the


administrator requiring them to explain within 72 hours why they should
not be removed from their office and expelled from union membership.

However, as early as November 21, 1988, the officers were


expelled from the ULGWP. The termination letter states that "You
failed and/or refused to offer an explanation inspite of the time granted
to you. Since you are no longer a member of good standing, ULGWP
is constrained to recommend for your termination from your
employment, and provided in Article II, Section 4, known as UNION
SECURITY, in the Collective Bargaining Agreement."

On the same day, the federation advised respondent company of


the expulsion of the 30 union officers and demanded their separation
from employment pursuant to the Union Security Clause in their
collective bargaining agreement. The Federation filed a Notice of Strike
with the National Conciliation and Mediation Board to compel the

404
Page 405

company to effect the immediate termination of the expelled union


officers.

On March 7, 1989, under the pressure of a threatened strike,


respondent company terminated the 30 union officers from
employment.

The petitioners filed a verified complaint with the Labor Arbiter


charging private respondents of unfair labor practice which consists of
union busting, illegal dismissal, illegal suspension, interference in
union activities, discrimination, threats, intimidation, coercion,
violence, and oppression.

The Labor Arbiter held that the termination is valid in compliance


with the union security clause of the collective bargaining agreement.

The NLRC affirmed the decision of the LA.

Hence this appeal.

Petitioners contend that their dismissal from work was effected in


an arbitrary, hasty, capricious and illegal manner because it was
undertaken by the respondent company without any prior
administrative investigation; that, had respondent company conducted
prior independent investigation it would have found that their expulsion
from the union was unlawful similarly for lack of prior administrative
investigation.

Private respondents, on the other hand, maintain that the thirty


dismissed employees who were former officers of the federation have
no cause of action against the company, the termination of their
employment having been made upon the demand of the federation
pursuant to the union security clause of the CBA.

ISSUE:
Whether or not respondent company was justified in dismissing
the employees.

RULING:
NO.

The ruling of NLRC is erroneous. The Supreme Court stated that


although this Court has ruled that union security clauses embodied in
the collective bargaining agreement may be validly enforced and that
dismissals pursuant thereto may likewise be valid, this does not erode

405
Page 406

the fundamental requirement of due process. The reason behind the


enforcement of union security clauses which is the sanctity and
inviolability of contracts cannot override one's right to due process.

In the case of Cariño vs. National Labor Relations Commission,


this Court pronounced that while the company, under a maintenance
of membership provision of the collective bargaining agreement, is
bound to dismiss any employee expelled by the union for disloyalty
upon its written request, this undertaking should not be done hastily
and summarily. The company acts in bad faith in dismissing a worker
without giving him the benefit of a hearing.

In the case under scrutiny, petitioner union officers were expelled


by the federation for allegedly committing acts of disloyalty and/or
inimical to the interest of ULGWP and in violation of its Constitution
and By-laws. Upon demand of the federation, the company terminated
the petitioners without conducting a separate and independent
investigation. Respondent company did not inquire into the cause of
the expulsion and whether or not the federation had sufficient grounds
to effect the same. Relying merely upon the federation's allegations,
respondent company terminated petitioners from employment when a
separate inquiry could have revealed if the federation had acted
arbitrarily and capriciously in expelling the union officers. Respondent
company's allegation that petitioners were accorded due process is
belied by the termination letters received by the petitioners which state
that the dismissal shall be immediately effective.

While respondent company may validly dismiss the employees


expelled by the union for disloyalty under the union security clause of
the collective bargaining agreement upon the recommendation by the
union, this dismissal should not be done hastily and summarily thereby
eroding the employees' right to due process, self-organization and
security of tenure. The enforcement of union security clauses is
authorized by law provided such enforcement is not characterized by
arbitrariness, and always with due process. Even on the assumption
that the federation had valid grounds to expel the union officers, due
process requires that these union officers be accorded a separate
hearing by respondent company.

With respect to the dismissal of individual petitioners, the Labor


Arbiter declared that their refusal to heed respondent's recall to work
notice is a clear indication that they were no longer interested in
continuing their employment and is deemed abandonment. It is
admitted that three return-to-work notices were sent by respondent
company to the striking employees on March 27, April 11, and April 21,
1989 and that 261 employees who responded to the notice were
admitted back to work.

406
Page 407

However, jurisprudence holds that for abandonment of work to


exist, it is essential (1) that the employee must have failed to report for
work or must have been absent without valid or justifiable reason; and
(2) that there must have been a clear intention to sever the employer-
employee relationship manifested by some overt acts. Deliberate and
unjustified refusal on the part of the employee to go back to his work
post and resume his employment must be established. Absence must
be accompanied by overt acts unerringly pointing to the fact that the
employee simply does not want to work anymore. And the burden of
proof to show that there was unjustified refusal to go back to work rests
on the employer.

In the present case, respondents failed to prove that there was a


clear intention on the part of the striking employees to sever their
employer- employee relationship. Although admittedly the company
sent three return to work notices to them, it has not been substantially
proven that these notices were actually sent and received by the
employees. As a matter of fact, some employees deny that they ever
received such notices. Others alleged that they were refused entry to
the company premises by the security guards and were advised to
secure a clearance from ULGWP and to sign a waiver. Some
employees who responded to the notice were allegedly told to wait for
further notice from respondent company as there was lack of work.

407
Page 408

Case Digests by: GIRON, JOSE JUSTIN C.

ALABANG COUNTRY CLUB v. NLRC


G.R. No. 170287. February 14, 2008.
VELASCO, JR., J.

DOCTRINE:
UNFAIR LABOR PRACTICE; DISMISSAL OF UNION OFFICERS;
SUBSTANTIAL COMPLIANCE WITH DUE PROCESS
In MSMG v. Hon. Ramos the court held that "Although this Court
has ruled that union security clauses embodied in the collective
bargaining agreement may be validly enforced and that dismissals
pursuant thereto may likewise be valid, this does not erode the
fundamental requirements of due process."

In this case, substantial compliance with due process is sufficient


to negate charges of illegal dismissal

FACTS:
Petitioner Alabang Country Club, Inc. (Club) is a domestic non-
profit corporation with principal office at Country Club Drive, Ayala
Alabang, Muntinlupa City. Respondent Alabang Country Club
Independent Employees Union (Union) is the exclusive bargaining
agent of the Club's rank-and-file employees. In April 1996,
respondents Christopher Pizarro, Michael Braza, and Nolasco
Castueras were elected Union President, Vice-President, and
Treasurer, respectively.

The Club and the Union entered into a Collective Bargaining


Agreement (CBA), which provided for a Union shop and maintenance
of membership shop.

Under Section 4, it states that - TERMINATION UPON UNION


DEMAND. Upon written demand of the UNION and after observing due
process, the Club shall dismiss a regular rank-and-file employee on
any of the following grounds: xxx (f) malversation of union funds, xxx

Subsequently, in July 2001, an election was held and a new set of


officers was elected. Soon thereafter, the new officers conducted an
audit of the Union funds. They discovered some irregularly recorded
entries, unaccounted expenses and disbursements, and uncollected
loans from the Union funds. The Union notified respondents Pizarro,
Braza, and Castueras of the audit results and asked them to explain
the discrepancies in writing. They filed their respective defenses.

408
Page 409

Despite their explanations, respondents Pizarro, Braza, and


Castueras were expelled from the Union and were furnished individual
letters of expulsion for malversation of Union funds.

The Union, invoking the Security Clause of the CBA, demanded


that the Club dismiss respondents Pizarro, Braza, and Castueras in
view of their expulsion from the Union. The Club required the three
respondents to show cause in writing within 48 hours from notice why
they should not be dismissed. Pizarro, Braza, and Castueras submitted
their respective written explanations.

Nonetheless, after weighing the verbal and written explanations of


the three respondents, the Club concluded that said respondents failed
to refute the validity of their expulsion from the Union. Thus, it was
constrained to terminate the employment of said respondents.

Respondents Pizarro, Braza, and Castueras challenged their


dismissal from the Club in an illegal dismissal complaint filed with the
NLRC.

The Labor Arbiter ruled in favor of the Club, and found that there
was justifiable cause in terminating said respondents.

The NLRC reversed the decision and ruled that there was no
justifiable cause for the termination of respondents Pizarro, Braza, and
Castueras. According to the NLRC, the respondents' expulsions from
the Union was illegal since the DOLE had not yet made any definitive
ruling on their liability regarding the administration of the Union's funds.

The CA upheld the NLRC ruling that the three respondents were
deprived of Due Process. The CA ruled that the dismissal was contrary
to the doctrine laid down in MSMG-UWP v. Ramos, where the Court
held that even on the assumption that the union has valid grounds to
expel the local union officers, due process requires that the union
officers be accorded a separate hearing by the employer company.

Hence the parties come before the Supreme Court.

ISSUE:
Whether or not the three respondents were illegally dismissed.

RULING:
NO. There was a valid dismissal.

409
Page 410

Under the Labor Code, an employee may be validly terminated on


the following grounds: (1) just causes under Art. 282; (2) authorized
causes under Art. 283; (3) termination due to disease under Art. 284;
and (4) termination by the employee or resignation under Art. 285.

Another cause for termination is dismissal from employment due


to the enforcement of the union security clause in the CBA. Here, Art.
II of the CBA on Union security contains the provisions on the Union
shop and maintenance of membership shop.

Termination of employment by virtue of a union security clause


embodied in a CBA is recognized and accepted in our jurisdiction. This
practice strengthens the union and prevents disunity in the bargaining
unit within the duration of the CBA. By preventing member disaffiliation
with the threat of expulsion from the union and the consequent
termination of employment, the authorized bargaining representative
gains more numbers and strengthens its position as against other
unions which may want to claim majority representation.

In terminating the employment of an employee by enforcing the


union security clause, the employer needs only to determine and prove
that: (1) the union security clause is applicable; (2) the union is
requesting for the enforcement of the union security provision in the
CBA; and (3) there is sufficient evidence to support the union's decision
to expel the employee from the union. These requisites constitute just
cause for terminating an employee based on the CBA's union security
provision.

In the case at Bar, the language of Art. II of the CBA that the Union
members must maintain their membership in good standing as a
condition sine qua non for their continued employment with the Club is
unequivocal. It is also clear that upon demand by the Union and after
due process, the Club shall terminate the employment of a regular
rank-and-file employee who may be found liable for a number of
offenses, one of which is malversation of Union funds.

Based from the facts of this case, the three respondents were
expelled from and by the Union after due investigation for acts of
dishonesty and malversation of Union funds. In accordance with the
CBA, the Union properly requested the Club. Then, in compliance with
the Union's request, the Club reviewed the documents submitted by
the Union, requested said respondents to submit written explanations,
and thereafter afforded them reasonable opportunity to present their
side. After it had determined that there was sufficient evidence that
said respondents’ malversed Union funds, the Club dismissed them
from their employment conformably with Sec. 4 (f) of the CBA.

410
Page 411

Considering the foregoing circumstances, we are constrained to


rule that there is sufficient cause for the three respondents' termination
from employment.

Further, there was no violation of due process. The Club


substantially complied with the due process requirements before it
dismissed the three respondents. The three respondents aver that the
Club violated their rights to due process as enunciated in case of
Malayang Samahan, when it failed to conduct an independent and
separate hearing before they were dismissed from service.

The CA and the three respondents err in relying on Malayang


Samahan, as its ruling has no application to this case. In Malayang
Samahan, the union members were expelled from the union and were
immediately dismissed from the company without any semblance of
due process. Both the union and the company did not conduct
administrative hearings to give the employees a chance to explain
themselves. In the present case, the Club has substantially complied
with due process. The three respondents were notified that their
dismissal was being requested by the Union, and their explanations
were heard. Then, the Club, through its President, conferred with said
respondents during the last week of October 2001. The three
respondents were dismissed only after the Club reviewed and
considered the documents submitted by the Union vis-à-vis the written
explanations submitted by said respondents. Under these
circumstances, we find that the Club had afforded the three
respondents a reasonable opportunity to be heard and defend
themselves.

411
Page 412

Case Digests by: GIRON, JOSE JUSTIN C.

STANDARD CHARTERED BANK EMPLOYEES UNION v.


HON. CONFESOR
G.R. No. 170287. February 14, 2008.
CALLEJO, SR., J.

DOCTRINE:
UNFAIR LABOR PRACTICE; INTERFERENCE; HOW COMMITTED
Parenthetically, if an employer interferes in the selection of its
negotiators or coerces the Union to exclude from its panel of
negotiators a representative of the Union, AND if it can be inferred that
the employer adopted the said act to yield adverse effects on the free
exercise to right to self-organization or on the right to collective
bargaining of the employees.

UNFAIR LABOR PRACTICE; SURFACE BARGAINING


Surface bargaining is defined as “going through the motions of
negotiating” without any legal intent to reach an agreement. It involves
the question of whether an employer’s conduct demonstrates an
unwillingness to bargain in good faith or is merely hard bargaining.

FACTS:
Standard Chartered Bank (the Bank, for brevity) is a foreign
banking corporation doing business in the Philippines. The exclusive
bargaining agent of the rank and file employees of the Bank is the
Standard Chartered Bank Employees Union (the Union, for brevity).

Thereafter the Bank and the Union signed a five-year collective


bargaining agreement (CBA) with a provision to renegotiate the terms
thereof on the third year. Prior to the expiration of the three-year period
2 but within the sixty-day freedom period, the Union initiated the
negotiations.

After 3 years the Union, through its President, Eddie L.


Divinagracia, sent a letter containing its proposals covering political
provisions and thirty-four (34) economic provisions. Included therein
was a list of the names of the members of the Union’s negotiating
panel.

In a Letter, the Bank, through its Country Manager Peter H. Harris,


took note of the Union’s proposals. The Bank attached its counter-
proposal to the non-economic provisions proposed by the Union.

412
Page 413

The parties agreed to set meetings to settle their differences on


the proposed CBA.

Before the commencement of the negotiation, the Union, through


Divinagracia, suggested to the Bank’s Human Resource Manager and
head of the negotiating panel, Cielito Diokno, that the bank lawyers
should be excluded from the negotiating team. The Bank acceded.

After a few days the parties met and set the ground rules for the
negotiation. Diokno suggested that the negotiation be kept a “family
affair." Towards the end of the meeting, the Union manifested that the
same should be changed to “DEADLOCKED” to indicate that such
items remained unresolved. Both parties agreed to place the notation
“DEFERRED/DEADLOCKED.”

Thereafter, the negotiation for economic provisions commenced.


A presentation of the basis of the Union’s economic proposals was
made. The next meeting, the Bank made a similar presentation.
Towards the end of the Bank’s presentation, Umali requested the Bank
to validate the Union’s “guestimates,” especially the figures for the rank
and file staff. In the succeeding meetings, Umali chided the Bank for
the insufficiency of its counter- proposal on the provisions on salary
increase, group hospitalization, death assistance and dental benefits.

The Bank suggested that the Union prioritize its economic


proposals, considering that many of such economic provisions
remained unresolved. The Union, however, demanded that the Bank
make a revised itemized proposal.

After 1 month, the Union suggested that if the Bank would not
make the necessary revisions on its counter-proposal, it would be best
to seek a third party assistance.

Diokno requested the Union panel to refrain from involving


personalities and to instead focus on the negotiations. He suggested
that in order to break the impasse, the Union should prioritize the items
it wanted to iron out. Divinagracia stated that the Bank should make
the first move and make a list of items it wanted to be included in the
economic package.

Except for the provisions on signing bonus and uniforms, the


Union and the Bank failed to agree on the remaining economic
provisions of the CBA. The Union declared a deadlock and filed a
Notice of Strike before the National Conciliation and Mediation Board
(NCMB).

413
Page 414

On the other hand, the Bank filed a complaint for Unfair Labor
Practice (ULP) and Damages before the Arbitration Branch of the
National Labor Relations Commission (NLRC) against the Union

The Bank alleged that the Union violated its duty to bargain, as it
did not bargain in good faith. It contended that the Union demanded
“sky high economic demands,” indicative of blue-sky bargaining.
Further, the Union violated its no strike- no lockout clause by filing a
notice of strike before the NCMB. Considering that the filing of notice
of strike was an illegal act, the Union officers should be dismissed.

The petitioner asserts that the private respondent committed ULP,


i.e., interference in the selection of the Union’s negotiating panel, when
Cielito Diokno, the Bank’s Human Resource Manager, suggested to
the Union’s President Eddie L. Divinagracia that Jose P. Umali, Jr.,
President of the NUBE, be excluded from the Union’s negotiating
panel.

Further the Union alleges that the Bank violated its duty to bargain;
hence, committed ULP under Article 248(g) when it engaged in surface
bargaining. It alleged that the Bank just went through the motions of
bargaining without any intent of reaching an agreement, as evident in
the Bank’s counter-proposals. It explained that of the 34 economic
provisions it made, the Bank only made 6 economic counterproposals.
Further, as borne by the minutes of the meetings, the Bank, after
indicating the economic provisions it had rejected, accepted, retained
or were open for discussion, refused to make a list of items it agreed
to include in the economic package.

ISSUE:
1. Whether or not the Bank committed an "Interference" as a
ground for unfair labor practice.

2. Whether or not the Bank committed "surface bargaining" as a


ground for unfair labor practice.

RULING:
1. No.

Article 248(a) of the Labor Code, considers it an unfair labor


practice when an employer interferes, restrains or coerces employees
in the exercise of their right to self-organization or the right to form
association. The right to self- organization necessarily includes the
right to collective bargaining.

414
Page 415

Parenthetically, if an employer interferes in the selection of its


negotiators or coerces the Union to exclude from its panel of
negotiators a representative of the Union, and if it can be inferred that
the employer adopted the said act to yield adverse effects on the free
exercise to right to self-organization or on the right to collective
bargaining of the employees, ULP under Article 248(a) in connection
with Article 243 of the Labor Code is committed.

The circumstances that occurred during the negotiation do not


show that the suggestion made by Diokno to Divinagracia is an anti-
union conduct from which it can be inferred that the Bank consciously
adopted such act to yield adverse effects on the free exercise of the
right to self-organization and collective bargaining of the employees,
especially considering that such was undertaken previous to the
commencement of the negotiation and simultaneously with
Divinagracia’s suggestion that the bank lawyers be excluded from its
negotiating panel.

The records show that after the initiation of the collective


bargaining process, with the inclusion of Umali in the Union’s
negotiating panel, the negotiations pushed through. The complaint was
made only on August 16, 1993 after a deadlock was declared by the
Union on June 15, 1993.

It is clear that such ULP charge was merely an afterthought. The


accusation occurred after the arguments and differences over the
economic provisions became heated and the parties had become
frustrated. It happened after the parties started to involve personalities.
As the public respondent noted, passions may rise, and as a result,
suggestions given under less adversarial situations may be colored
with unintended meanings. 49 Such is what appears to have happened
in this case.

2. No.

Surface bargaining is defined as “going through the motions of


negotiating” without any legal intent to reach an agreement. The
resolution of surface bargaining allegations never presents an easy
issue. The determination of whether a party has engaged in unlawful
surface bargaining is usually a difficult one because it involves, at
bottom, a question of the intent of the party in question, and usually
such intent can only be inferred from the totality of the challenged
party’s conduct both at and away from the bargaining table. It involves

415
Page 416

the question of whether an employer’s conduct demonstrates an


unwillingness to bargain in good faith or is merely hard bargaining.

The minutes of meetings from March 12, 1993 to June 15, 1993
do not show that the Bank had any intention of violating its duty to
bargain with the Union. Records show that after the Union sent its
proposal to the Bank on February 17, 1993, the latter replied with a list
of its counter-proposals on February 24, 1993. Thereafter, meetings
were set for the settlement of their differences. The minutes of the
meetings show that both the Bank and the Union exchanged economic
and non-economic proposals and counter- proposals.

The Union has not been able to show that the Bank had done acts,
both at and away from the bargaining table, which tend to show that it
did not want to reach an agreement with the Union or to settle the
differences between it and the Union.

416
Page 417

Case Digests by: GIRON, JOSE JUSTIN C.

GMC v. CA
G.R. No. 123456. January 01, 2022.
QUISUMBING. J.

DOCTRINE:
UNFAIR LABOR PRACTICE; DUTY TO BARGAIN COLLECTIVELY
There is no per se test of good faith in bargaining. Good faith or
bad faith is an inference to be drawn from the facts. The effect of an
employer's or a union's actions individually is not the test of good-faith
bargaining, but the impact of all such occasions or actions, considered
as a whole.

Failing to comply with the mandatory obligation to submit a reply


to the union's proposals, GMC violated its duty to bargain collectively,
making it liable for unfair labor practice.

FACTS:
In its two plants located at Cebu City and Lapu-Lapu City,
petitioner General Milling Corporation (GMC) employed 190 workers.
They were all members of private respondent General Milling
Corporation Independent Labor Union (union, for brevity), a duly
certified bargaining agent.

On April 28, 1989, GMC and the union concluded a collective


bargaining agreement (CBA) which included the issue of
representation effective for a term of three years. The CBA was
effective for three years retroactive to December 1, 1988. Hence, it
would expire on November 30, 1991.

On November 29, 1991, a day before the expiration of the CBA,


the union sent GMC a proposed CBA, with a request that a counter-
proposal be submitted within ten (10) days. However, as early as
October 1991, GMC had received collective and individual letters from
workers who stated that they had withdrawn from their union
membership, on grounds of religious affiliation and personal
differences. Believing that the union no longer had standing to
negotiate a CBA, GMC did not send any counter-proposal.

On December 16, 1991, GMC wrote a letter to the union's officers


stating that GMC felt there that was no basis to negotiate with a union
which no longer existed, but that management was nonetheless
always willing to dialogue with them on matters of common concern

417
Page 418

and was open to suggestions on how the company may improve its
operations.

Thus the union filed a complaint against GMC with the NLRC
arbitration division. The union alleged unfair labor practice on the part
of GMC for the latter's refusal to bargain collectively.

The Labor Arbiter dismissed the case and recommended that a


petition for certification election be held to determine if the union still
enjoyed the support of the workers. The union appealed to the NLRC.

The NLRC ruled, in a resolution by reason of a motion for


reconsideration, that GMC's doubts as to the status of the union is
justified and the allegations of coercion exerted by GMC on the union
members to resign is unfounded. Hence the union filed a petition for
certiorari before the Court of Appeals.

The Court of Appeals set aside the NLRC Resolution.

Hence this petition for certiorari with the Court.

ISSUE:
Whether or not GMC violated its duty to bargain collectively
resulting to unfair labor practice.

RULING:
YES.

Article 253-A of the Labor Code, as amended by Rep. Act No.


6715 mandates that the representation provision of a CBA should last
for five-years. The relation between labor and management should be
undisturbed until the last 60 days of the fifth year. Hence, it is
indisputable that when the union requested for a renegotiation of the
economic terms of the CBA on November 29, 1991, it was still the
certified collective bargaining agent of the workers, because it was
seeking said renegotiation within five (5) years from the date of
effectivity of the CBA on December 1, 1988. The union's proposal was
also submitted within the prescribed 3-year period from the date of
effectivity of the CBA, albeit just before the last day of said period. It
was obvious that GMC had no valid reason to refuse to negotiate in
good faith with the union.

418
Page 419

For refusing to send a counter-proposal to the union and to


bargain anew on the economic terms of the CBA, the company
committed an unfair labor practice under Article 248 (g) of the Labor
Code.

Article 252 of the Labor Code elucidates the meaning of the


phrase "duty to bargain collectively," thus:

ART. 252. Meaning of duty to bargain collectively. — The duty to


bargain collectively means the performance of a mutual obligation
to meet and convene promptly and expeditiously in good faith for
the purpose of negotiating an agreement. . . .

There is no per se test of good faith in bargaining. Good faith or


bad faith is an inference to be drawn from the facts. The effect of an
employer's or a union's actions individually is not the test of good-faith
bargaining, but the impact of all such occasions or actions, considered
as a whole.

Under Article 252 above cited, both parties are required to perform
their mutual obligation to meet and convene promptly and
expeditiously in good faith for the purpose of negotiating an agreement.
The union lived up to this obligation when it presented proposals for a
new CBA to GMC within three (3) years from the effectivity of the
original CBA. But GMC failed in its duty under Article 252. What it did
was to devise a flimsy excuse, by questioning the existence of the
union and the status of its membership to prevent any negotiation.

GMC's failure to make a timely reply to the proposals presented


by the union is indicative of its utter lack of interest in bargaining with
the union. Its excuse that it felt the union no longer represented the
workers, was mainly dilatory as it turned out to be utterly baseless.

We hold that GMC's refusal to make a counter-proposal to the


union's proposal for CBA negotiation is an indication of its bad faith.
Where the employer did not even bother to submit an answer to the
bargaining proposals of the union, there is a clear evasion of the duty
to bargain collectively.

Failing to comply with the mandatory obligation to submit a reply


to the union's proposals, GMC violated its duty to bargain collectively,
making it liable for unfair labor practice. Perforce, the Court of Appeals
did not commit grave abuse of discretion amounting to lack or excess
of jurisdiction in finding that GMC is, under the circumstances, guilty of
unfair labor practice.

419
Page 420

Case Digests by: GIRON, JOSE JUSTIN C.

HACIENDA FATIMA v.
NFSW FOOD AND GENERAL TRADE
G.R. No. 149440. January 28, 2003.
Panganiban, J.

DOCTRINE:
UNFAIR LABOR PRACTICE; INTERFERENCE
The petitioners herein refused to bargain collectively, their acts of
economic inducements resulting in the promotion of those who
withdrew from the said union, the use of armed guards to prevent the
organizers to come in, and the dismissal of union officials and
members is a clear indication that herein petitioners did not want a
union. This is a clear case of “interference” in the right of the workers
to self-organization which is being proscribed by the Labor Code.

FACTS:
It appears that herein respondents did not look with factor workers’
having organized themselves into a union. When respondent union
was certified as the collective bargaining representative in the
certification elections, the employer under the pretext that the result
was on appeal, refused to sit down with the union for the purpose of
entering into a collective bargaining agreement. Moreover, the workers
including complainants herein were not given work for more than one
month. In protest, herein respondents staged a strike which however
was settled upon the signing of a Memorandum of Agreement which
stipulated that the parties will meet and convene for Collective
Bargaining Agreement negotiations and will endeavor to conclude the
same within 30 days; the management will give priority to the women
workers who are members of the union; that the management will
provide 15 wagons for the workers; and other ground rules.

However, alleging that complainants failed to load the 15 wagons,


respondents reneged on its commitment to sit down and bargain
collectively. Instead, the management employed all means including
the use of private armed guards to prevent the organizers from
entering the premises.

Moreover, the employer did not any more give work assignments
to the complainants forcing the union to stage a strike. But due to the
conciliation efforts by the DOLE another Memorandum of Agreement
was signed by the herein respondents. Pursuant to this Agreement,
the parties subsequently met. The parties agreed that 4 named
employees are deemed not considered employees and another 12

420
Page 421

named employees shall be reinstated immediately upon availability of


work.

However, the employer again reneged on its commitment which


resulted to the filing of this present complaint by the Union.

The Labor Arbiter ruled that there was no illegal dismissal. The
NLRC set aside and vacated the decision of the LA. The NLRC
declared that complainants have been illegally dismissed. This ruling
was affirmed by the Court of Appeals.

The CA ruled that while work of respondents was seasonal in


nature, they were considered to be merely on leave during the off-
season and were therefore still employed by petitioner employers.
Moreover, the workers enjoyed security of tenure. Any infringement
upon this right was deemed by the CA to be tantamount to illegal
dismissal.

The CA likewise concurred with the NLRC’s finding that petitioner


were guilty of unfair labor practice. That “indeed, from respondents’
refusal to bargain, to their acts of economic inducements resulting in
the promotion of those who withdrew from the unions, the use of armed
guards to prevent the organizers to come in, and the dismissal of union
officials and members, one cannot but conclude that respondents did
not want a union in their hacienda - a clear interference in the right of
the workers to self-organization.

Hence this petition with the Supreme Court.

ISSUE:
Whether or not petitioner is guilty of unfair labor practice.

RULING:
YES.

There is no reason to disturb the CA’s dismissal of what petitioners


claim was their valid exercise of a management prerogative. The
student changes in work assignments reeked of bad faith. These
changes were implemented immediately after respondents had
organized themselves into a union and stated demanding collective
bargaining. Those who were union members were effectively deprived
of their jobs. Petitioners’ move actually amounted to unjustified
dismissal of respondents, in violation of the Labor Code.

421
Page 422

Where there is no showing of clear, valid and legal cause for the
termination of employment, the law considers the matter a case of
illegal dismissal and the burden is on the employer to prove that the
termination was for a valid and authorized cause. In the case at bar,
petitioner failed to prove any such cause for the dismissal of
respondents who are regular employees.

The Supreme Court also upholds the CA’s affirmation that herein
petitioners are guilty of unfair labor practice because the petitioners
herein refused to bargain collectively, their acts of economic
inducements resulting in the promotion of those who withdrew from the
said union, the use of armed guards to prevent the organizers to come
in, and the dismissal of union officials and members is a clear indication
that herein petitioners did not want a union. This is a clear case of
“interference” in the right of the workers to self-organization which is
being proscribed by the Labor Code.

422
Page 423

Case Digests by: GIRON, JOSE JUSTIN C.

ST. JOHN COLLEGES INC. v.


JOHN ACADEMY FACULTY AND EMPLOYEES UNION
G.R. No. 167892. October 27, 2006.
Ynares-Santiago, J.

DOCTRINE:
UNFAIR LABOR PRACTICE; CLOSURE IN BAD FAITH;
AMOUNTS TO RESTRAINT
The Labor Code does not authorize the employer to close down
the establishment on the ground of illegal or excessive demands of the
Union. Instead, aside from the remedy of submitting the dispute for
voluntary or compulsory arbitration, the employer may file a complaint
for ULP against the Union for bargaining in bad faith. If found guilty,
this gives rise to civil and criminal liabilities and allows the employer to
implement a lock out, but not the closure of the establishment resulting
to the permanent loss of employment of the whole workforce.

FACTS:
Petitioner St. John Colleges, Inc. (SJCI) is a domestic corporation
which owns and operates the St. John's Academy (later renamed St.
John Colleges) in Calamba, Laguna. The high school then employed
about 80 teaching and non-teaching personnel who were members of
the St. John Academy Faculty & Employees Union (Union).

The Collective Bargaining Agreement (CBA) between SJCI and


the Union was set to expire on May 31, 1997. During the ensuing
collective bargaining negotiations, SJCI rejected all the proposals of
the Union for an increase in worker's benefits. This resulted to a
bargaining deadlock which led to the holding of a valid strike by the
Union. In order to end the strike, SJCI and the Union, through the
efforts of the National Conciliation and Mediation Board (NCMB),
agreed to refer the labor dispute to the Secretary of Labor and
Employment (SOLE) for assumption of jurisdiction.

After which, the strike ended and classes resumed. Subsequently,


the SOLE issued an Order assuming jurisdiction over the labor dispute
pursuant to Article 263 of the Labor Code. The parties were required
to submit their respective position papers within ten (10) days from
receipt of said Order.

Pending resolution of the labor dispute before the SOLE, the


Board of Directors of SJCI approved a resolution recommending the

423
Page 424

closure of the high school which was approved by the stockholders


because of the irreconcilable differences between the school
management and the Academy's Union particularly the safety of our
students and the financial aspect of the ongoing CBA negotiations.

Thereafter, SJCI informed the Department of Labor and


Employment (DOLE), Department of Education, Culture and Sports
(DECS), parents, students and the Union of the impending closure of
the high school.

Subsequently, some teaching and non-teaching personnel of the


high school agreed to the closure. SJCI informed the DOLE that 51
employees had received their separation compensation package while
25 employees refused to accept the same.

The aforementioned 25 employees conducted a protest action


within the perimeter of the high school. The Union filed a notice of strike
with the NCMB. SJCI filed a petition to declare the strike illegal before
the NLRC.

The 25 employees filed a complaint for unfair labor practice (ULP),


illegal dismissal. The Union members alleged that the closure of the
high school was done in bad faith in order to get rid of the Union and
render useless any decision of the SOLE on the CBA deadlocked
issues.

The Labor Arbiter rendered a Decision dismissing the Union's


complaint for ULP and illegal dismissal while granting SJCI's petition
to declare the strike illegal.

The NLRC rendered judgment reversing the decision of the Labor


Arbiter. It found SJCI guilty of ULP and illegal dismissal and ordered it
to reinstate the 25 employees to their former positions without loss of
seniority rights and other benefits, and with full backwages.

The Court of Appeals affirmed the decision of the NLRC.

Hence this petition.

ISSUE:
Whether or not SJCI is liable for unfair labor practice when it
closed down the high school.

RULING:
YES.

424
Page 425

Under Article 283 of the Labor Code, the following requisites must
concur for a valid closure of the business: (1) serving a written notice
on the workers at least one (1) month before the intended date thereof;
(2) serving a notice with the DOLE one month before the taking effect
of the closure; (3) payment of separation pay equivalent to one (1)
month or at least one half (1/2) month pay for every year of service,
whichever is higher, with a fraction of at least six (6) months to be
considered as a whole year; and (4) cessation of the operation must
be bona fide.

The instant case revolves around the 4th requisite.

The determination of whether SJCI acted in bad faith depends on


the particular facts as established by the evidence on record. Bad faith
is, after all, an inference which must be drawn from the peculiar
circumstances of a case. The two decisive factors in determining
whether SJCI acted in bad faith are (1) the timing of, and reasons for
the closure of the high school, and (2) the timing of, and the reasons
for the subsequent opening of a college and elementary department,
and, ultimately, the reopening of the high school department by SJCI
after only one year from its closure.

Prior to the closure of the high school by SJCI, the parties agreed
to refer the 1997 CBA deadlock to the SOLE for assumption of
jurisdiction under Article 263 of the Labor Code. As a result, the strike
ended and classes resumed. After the SOLE assumed jurisdiction, it
required the parties to submit their respective position papers.
However, instead of filing its position paper, SJCI closed its high
school, allegedly because of the "irreconcilable differences between
the school management and the Academy's Union particularly the
safety of our students and the financial aspect of the ongoing CBA
negotiations." Thereafter, SJCI moved to dismiss the pending labor
dispute with the SOLE contending that it had become moot because of
the closure. Nevertheless, a year after said closure, SJCI reopened its
high school and did not rehire the previously terminated employees.

Under these circumstances, it is not difficult to discern that the


closure was done to defeat the parties' agreement to refer the labor
dispute to the SOLE; to unilaterally end the bargaining deadlock; to
render nugatory any decision of the SOLE; and to circumvent the
Union's right to collective bargaining and its members' right to security
of tenure. By admitting that the closure was due to irreconcilable
differences between the Union and school management, specifically,
the financial aspect of the ongoing CBA negotiations, SJCI in effect
admitted that it wanted to end the bargaining deadlock and eliminate
the problem of dealing with the demands of the Union. This is precisely
what the Labor Code abhors and punishes as unfair labor practice

425
Page 426

since the net effect is to defeat the Union's right to collective


bargaining.

Further, the alleged difficult labor problems merely show that SJCI
and the Union had disagreements regarding workers' benefits which is
normal in any business establishment. That SJCI agreed to
appropriate 100% of the tuition fee increase to the workers' benefits
sometime in 1995 does not mean that it was helpless in the face of the
Union's demands because neither party is obligated to precipitately
give in to the proposal of the other party during collective bargaining.
13 If SJCI found the Union's demands excessive, its remedy under the
law is to refer the matter for voluntary or compulsory dispute resolution.
Besides, this incident which occurred in 1995, could hardly establish
the good faith of SJCI or justify the high school's closure in 1998.

At any rate, even assuming that the Union's demands were illegal
or excessive, the important and crucial point is that these alleged illegal
or excessive demands did not justify the closure of the high school and
do not, in any way, establish SJCI's good faith. The employer cannot
unilaterally close its establishment on the pretext that the demands of
its employees are excessive. As already discussed, neither party is
obliged to give-in to the other's excessive or unreasonable demands
during collective bargaining, and the remedy in such case is to refer
the dispute to the proper tribunal for resolution.

426
Page 427

Case Digests by: GIRON, JOSE JUSTIN C.

CENTRAL AZUCARERA DE BAIS EMPLOYEES UNION


NFL v. CENTRAL AZUCARERA DE BAIS INC.
G.R. No. 186605. November 17, 2010.
Mendoza, J.

DOCTRINE:
UNFAIR LABOR PRACTICE; DUTY TO BARGAIN
COLLECTIVELY; ELEMENT OF BAD FAITH.
For a charge of unfair labor practice to prosper, it must be shown
that the employer was motivated by ill will, "bad faith, or fraud, or was
oppressive to labor, or done in a manner contrary to morals, good
customs, or public policy, and, of course, that social humiliation,
wounded feelings or grave anxiety resulted.

FACTS:
Respondent Central Azucarera De Bais, Inc. (CAB) is a
corporation duly organized and existing under the laws of the
Philippines. CABEU-NFL is a duly registered labor union and a certified
bargaining agent of the CAB rank-and-file employees.

CABEU-NFL sent CAB a proposedCollective Bargaining


Agreement (CBA) seeking increases in the daily wage and vacation
and sick leave benefits of the monthly employees and the grant of
leave benefits and 13th month pay to seasonal workers.

After 2 months, CAB responded with a counter-proposal to the


effect that the production bonus incentive and special production
bonus and incentives be maintained. In addition, respondent CAB
agreed to execute a pro-rated increase of wages every time the
government would mandate an increase in the minimum wage. CAB,
however, did not agree to grant additional and separate Christmas
bonuses.

CAB received an Amended Union Proposal 8 sent by CABEU-NFL


reducing its previous demand regarding wages and bonuses. CAB,
however, maintained its position on the matter. Thus, the collective
bargaining negotiations resulted in a deadlock.

On account of the impasse, "CABEU-NFL filed a Notice of Strike


with the National Conciliation and Mediation Board (NCMB).

427
Page 428

NCMB failed to act on the letter-response of CAB. Neither did it


convene CAB and CABEU-NFL to continue the negotiations between
them.

Reacting from the letter-response of CAB, CABEU-NFL filed a


Complaint for Unfair Labor Practice for the former's refusal to bargain
with it.

The Labor Arbiter dismissed the complaint.

On appeal, the NLRC reversed the LA's decision and found CAB
guilty of unfair labor practice stating that respondent violated its duty to
bargain with complainant when during the pendency of the conciliation
proceedings before the NCMB it concluded a CBA with another union
as a consequence, it refused to resume negotiation with complainant
upon the latter's demand.

Unsatisfied, CAB elevated the matter to the CA by way of a petition


for certiorari under Rule 65 alleging grave abuse of discretion on the
part of the NLRC in reversing the LA decision and issuing the
questioned resolution.

The CA reversed the NLRC decision and stated that private


respondent CABEU-NFL failed in its burden of proof to present
substantial evidence to support the allegation of unfair labor practice.

Hence this petition with the Supreme Court.

Ultimately, CABEU-NFL aggressively asserts that CAB is guilty of


unfair labor practice on the ground of its refusal to bargain collectively.
CABEU-NFL claims to be the duly certified bargaining agent of the
CAB rank-and-file employees such that it requested to bargain through
a letter-request which was subsequently turned down by CAB in its
letter-response.

On the other hand, CAB counters that in view of the disassociation


of more than 90% of rank-and-file workers from CABEU-NFL, it was
constrained to negotiate and conclude in good faith a new CBA with
CABELA, the newly established union by workers who disassociated
from CABEU-NFL. CAB emphasizes that it declined further
negotiations with CABEU-NFL in good faith because to continue with
it would serve no practical purpose.

428
Page 429

ISSUE:
Whether or not CABEU-NFL is guilty of unfair labor practice by
refusing to bargain collectively.

RULING:
NO.

Article 248. Unfair Labor Practices of Employers. — It shall be


unlawful for an employer to commit any of the following unfair labor
practice:

xxx xxx xxx

(g) To violate the duty to bargain collectively as prescribed by this


Code.

For a charge of unfair labor practice to prosper, it must be shown


that CAB was motivated by ill will, "bad faith, or fraud, or was
oppressive to labor, or done in a manner contrary to morals, good
customs, or public policy, and, of course, that social humiliation,
wounded feelings or grave anxiety resulted . . ." in suspending
negotiations with CABEU-NFL.

Notably, CAB believed that CABEU-NFL was no longer the


representative of the workers. It just wanted to foster industrial peace
by bowing to the wishes of the overwhelming majority of its rank and
file workers and by negotiating and concluding in good faith a CBA with
CABELA."

Furthermore, basic is the principle that good faith is presumed and


he who alleges bad faith has the duty to prove the same. By imputing
bad faith to the actuations of CAB, CABEU-NFL has the burden of
proof to present substantial evidence to support the allegation of unfair
labor practice. Apparently, CABEU-NFL refers only to the
circumstances mentioned in the letter-response, namely, the execution
of the supposed CBA between CAB and CABELA and the request to
suspend the negotiations, to conclude that bad faith attended CAB's
actions. The Court is of the view that CABEU-NFL, in simply relying on
the said letter-response, failed to substantiate its claim of unfair labor
practice to rebut the presumption of good faith.

429
Page 430

Case Digests by: GIRON, JOSE JUSTIN C.

UFE-DFA-KMU v. NESTLE PHILS. INC.


G.R. No. 158930-31 and 158944-45. March 3, 2008.
Chico-Nazario, J.

DOCTRINE:
UNFAIR LABOR PRACTICE; DUTY TO BARGAIN COLLECTIVELY
While the law makes it an obligation for the employer and the
employees to bargain collectively with each other, such compulsion
does not include the commitment to precipitately accept or agree to the
proposals of the other. All it contemplates is that both parties should
approach the negotiation with an open mind and make reasonable
effort to reach a common ground of agreement.

FACTS:
Add the facts here. UFE-DFA-KMU was the sole and exclusive
bargaining agent of the rank- and-file employees of Nestlé belonging
to the latter's Alabang and Cabuyao plants. As the existing collective
bargaining agreement (CBA) between Nestlé and UFE-DFA-KMU was
about to end in a month, the Presidents of the Alabang and Cabuyao
Divisions of UFE-DFA-KMU informed Nestlé of their intent to "open
[our] new Collective Bargaining Negotiation. In response thereto,
Nestlé informed them that it was also preparing its own counter-
proposal and proposed ground rules to govern the impending conduct
of the CBA negotiations.

In another letter to the UFE-DFA-KMU (Cabuyao Division only),


Nestlé reiterated its stance that "unilateral grants, one-time company
grants, company-initiated policies and programs, which include, but
are not limited to the Retirement Plan, Incidental Straight Duty Pay and
Calling Pay Premium, are by their very nature not proper subjects of
CBA negotiations and therefore shall be excluded therefrom."

Dialogue between the company and the union thereafter ensued,


however, Nestlé requested the National Conciliation and Mediation
Board (NCMB) to conduct preventive mediation proceedings between
it and UFE- DFA-KMU owing to an alleged impasse in said dialogue;
i.e., that despite fifteen (15) meetings between them, the parties failed
to reach any agreement on the proposed CBA.

Conciliation proceedings proved ineffective, though, and the UFE-


DFA- KMU filed a Notice of Strike with the NCMB, complaining, in

430
Page 431

essence, of a bargaining deadlock pertaining to economic issues, i.e.,


"retirement (plan), panel composition, costs and attendance, and CBA"

Another Notice of Strike was filed by the union, this time


predicated on Nestlé's alleged unfair labor practices, that is, bargaining
in bad faith by setting pre-conditions in the ground rules and/or refusing
to include the issue of the Retirement Plan in the CBA negotiations.
The result of a strike vote conducted by the members of UFE-DFA-
KMU yielded an overwhelming approval of the decision to hold a strike.

G.R. No. 158930-31 was filed by UFE-DFA-KMU against Nestlé


seeking to reverse the Court of Appeals Decision insofar as the
appellate court's failure to find Nestlé guilty of unfair labor practice was
concerned; while G.R. No. 158944-45 was instituted by Nestlé against
UFE-DFA-KMU likewise looking to annul and set aside the part of the
Court of Appeals Decision declaring that: 1) the Retirement Plan was
a valid collective bargaining issue; and 2) the scope of the power of the
Secretary of the Department of Labor and Employment (DOLE) to
assume jurisdiction over the labor dispute between UFE-DFA-KMU
and Nestlé was limited to the resolution of questions and matters
pertaining merely to the ground rules of the collective bargaining
negotiations to be conducted between the parties.

With respect to the charge of unfair labor practice, UFE-DFA-KMU


argues therein that Nestlé's "refusal to bargain on a very important
CBA economic provision constitutes unfair labor practice." It explains
that Nestlé set as a precondition for the holding of collective bargaining
negotiations the non-inclusion of the issue of Retirement Plan. In its
words, "respondent Nestlé Phils., Inc. insisted that the Union should
first agree that the retirement plan is not a bargaining issue before
respondent Nestlé would agree to discuss other issues in the CBA."

ISSUE:
Whether or not Nestlé is guilty of unfair labor practice.

RULING:
NO.

The duty to bargain collectively is mandated by Articles 252 and


253 of the Labor Code, as amended. Obviously, the purpose of
collective bargaining is the reaching of an agreement resulting in a
contract binding on the parties; but the failure to reach an agreement
after negotiations have continued for a reasonable period does not
establish a lack of good faith. The statutes invite and contemplate a
collective bargaining contract, but they do not compel one. The duty to
bargain does not include the obligation to reach an agreement.

431
Page 432

The crucial question, therefore, of whether or not a party has met


his statutory duty to bargain in good faith typically turns on the facts of
the individual case. The effect of an employer's or a union's individual
actions is not the test of good-faith bargaining, but the impact of all
such occasions or actions, considered as a whole, and the inferences
fairly drawn therefrom collectively may offer a basis for the finding of
the NLRC.

For a charge of unfair labor practice to prosper, it must be shown


that Nestlé was motivated by ill will, "bad faith, or fraud, or was
oppressive to labor, or done in a manner contrary to morals, good
customs, or public policy, and, of course, that social humiliation,
wounded feelings, or grave anxiety resulted. While the law makes it an
obligation for the employer and the employees to bargain collectively
with each other, such compulsion does not include the commitment to
precipitately accept or agree to the proposals of the other. All it
contemplates is that both parties should approach the negotiation with
an open mind and make reasonable effort to reach a common ground
of agreement.

Herein, the union merely bases its claim of refusal to bargain on a


letter dated 29 May 2001 written by Nestlé where the latter laid down
its position that "unilateral grants, one-time company grants, company-
initiated policies and programs, which include, but are not limited to the
Retirement Plan, Incidental Straight Duty Pay and Calling Pay
Premium, are by their very nature not proper subjects of CBA
negotiations and therefore shall be excluded therefrom." But as we
have stated in this Court's Decision, said letter is not tantamount to
refusal to bargain. In thinking to exclude the issue of Retirement Plan
from the CBA negotiations, Nestlé, cannot be faulted for considering
the same benefit as unilaterally granted, considering that eight out of
nine bargaining units have allegedly agreed to treat the Retirement
Plan as a unilaterally granted benefit. This is not a case where the
employer exhibited an indifferent attitude towards collective
bargaining, because the negotiations were not the unilateral activity of
the bargaining representative. Nestlé's desire to settle the dispute and
proceed with the negotiation being evident in its cry for compulsory
arbitration is proof enough of its exertion of reasonable effort at good-
faith bargaining.

In the case at bar, Nestle never refused to bargain collectively with


UFE-DFA-KMU. The corporation simply wanted to exclude the
Retirement Plan from the issues to be taken up during CBA
negotiations, on the postulation that such was in the nature of a
unilaterally granted benefit.

432
Page 433

Case Digests by: GIRON, JOSE JUSTIN C.

MALAYANG MANGAGAWA NG STAYFAST v. NLRC


G.R. No. 155306. August 28, 2013.
Leonardo-De Castro, J.

DOCTRINE:
UNFAIR LABOR PRACTICE; ALLEGATIONS MUST BE PROVEN
WITH SUFFICIENT EVIDENCE.
It is settled that questions of fact cannot be raised in an original
action for certiorari. Only established or admitted facts can be
considered.

Both the Labor Arbiter and the NLRC held that there was no
sufficient proof of respondent company's alleged discriminatory acts.
Thus, petitioner's unfair labor practice, union-busting and unlawful
lockout claims do not hold water.

FACTS:
Petitioner and Nagkakaisang Lakas ng Manggagawa sa Stayfast
(NLMS-Olalia) sought to be the exclusive bargaining agent of the
employees of respondent company, Stayfast Philippines, Inc. A
certification election was conducted. Out of the 223 valid votes cast,
petitioner garnered 109 votes while NLMS-Olalia received 112 votes
and 2 votes were for "No Union."

Thus, the Med-Arbiter certified NLMS-Olalia as the sole and


exclusive bargaining agent of all rank and file employees of respondent
company.

Petitioner appealed the Order of the Med-Arbiter to the Secretary


of Labor and Employment. The Secretary of Labor and Employment
affirmed the decision of the Med-Arbiter.

Petitioner elevated the matter via petition for certiorari to this


Court. The petition however was dismissed.

Meanwhile, NLMS-Olalia demanded to collectively bargain with


respondent company. The latter rejected petitioner's demand, insisting
that it would negotiate a collective bargaining agreement only with
whichever union is finally certified as the sole and exclusive bargaining
agent of the workers. Nevertheless, NLMS-Olalia went on strike on
April 1, 1997 until it was temporarily restrained eight days later.

433
Page 434

Subsequently, petitioner filed its own notice of strike in the


National Conciliation and Mediation Board (NCMB). Respondent
company opposed petitioner's move and filed a motion to dismiss on
the ground that petitioner was not the certified bargaining agent and
therefore lacked personality to file a notice of strike. Thereafter, the
parties were able to make concessions during the conciliation-
mediation stage in the NCMB which led petitioner to withdraw its notice
of strike.

However, petitioner's members staged a "sit-down strike" to


dramatize their demand for a fair and equal treatment as respondent
company allegedly continued to discriminate against them.
Respondent company issued a memorandum requiring the alleged
participants in the "sit- down strike" to explain within 24 hours why they
should not be terminated or suspended from work for infraction of
company rules and regulations pertaining to unauthorized work
stoppage, acts inimical to company interest, and disregard of
instruction of immediate supervisor to perform assigned task.

In support of its complaint, petitioner alleged that respondents had


repeatedly committed acts of discrimination, such as the denial of the
use of the company canteen for purposes of conducting a strike vote,
the constant denial of applications of petitioner's members for leave to
attend hearings in relation to certain labor cases while similar
applications of members of the other union were approved, and the
suspension of petitioner's president for being absent due to attendance
in hearings of labor cases involving petitioner's members. Petitioner
further claimed that the termination of about 127 of its officers and
members constituted union busting and unlawful lockout.

On the other hand, respondent company claimed that petitioner


lacked legal authority to go on strike since it is a minority union.
petitioner withdrew its notice of strike during the proceedings in the
NCMB, the strike conducted by petitioner was illegal as it constituted a
wildcat strike and later became a full-blown strike.

ISSUE:
Whether or not respondent company is guilty of unfair labor
practice.

RULING:
NO.

Petitioner reiterated its position that respondent company and its


General Manager committed discriminatory acts against petitioner's
members which constituted unfair labor practice; that the termination

434
Page 435

of employment of petitioner's officers and members was a case of


union-busting and unlawful lockout; and, that the said officers and
members were unlawfully dismissed from employment and are
therefore entitled to reinstatement with full backwages, plus damages
and attorney's fees. For petitioner to question the identical findings of
the Labor Arbiter and the NLRC is to raise a question of fact.

However, it is settled that questions of fact cannot be raised in an


original action for certiorari. Only established or admitted facts can be
considered.

Both the Labor Arbiter and the NLRC held that there was no
sufficient proof of respondent company's alleged discriminatory
acts.Thus, petitioner's unfair labor practice, union-busting and unlawful
lockout claims do not hold water.

435
Page 436

Case Digests by: GIRON, JOSE JUSTIN C.

HOLY CHILD CATHOLIC SCHOOL v.


SEC OF LABOR AND EMPLOYMENT
G.R. No. 179146. July 23, 2013.
Peralta, J.

DOCTRINE:
BARGAINING UNIT vis-a-vis LABOR UNION
A bargaining unit is a group of employees sought to be
represented by a petitioning union. Such employees need not be
members of a union seeking the conduct of a certification election. A
union certified as an exclusive bargaining agent represents not only its
members but also other employees who are not union members.

FACTS:
A petition for certification election was filed by private respondent
Pinag-Isang Tinig at Lakas ng Anakpawis — Holy Child Catholic
School Teachers and Employees Labor Union (HCCS-TELU-
PIGLAS) , alleging that: PIGLAS is a legitimate labor organization duly
registered with the Department of Labor and Employment (DOLE)
representing HCCS-TELU-PIGLAS; HCCS is a private educational
institution duly registered and operating under Philippine laws; there
are approximately 120 teachers and employees comprising the
proposed appropriate bargaining unit; and HCCS is unorganized, there
is no collective bargaining agreement or a duly certified bargaining
agent or a labor organization certified as the sole and exclusive
bargaining agent of the proposed bargaining unit within one year prior
to the filing of the petition.

Petitioner HCCS consistently noted that it is a parochial school


with a total of 156 employees as of June 28, 2002, broken down as
follows: ninety-eight (98) teaching personnel, twenty-five (25) non-
teaching academic employees, and thirty-three (33) non-teaching non-
academic workers. It averred that of the employees who signed to
support the petition, fourteen (14) already resigned and six (6) signed
twice.

Petitioner raised that members of private respondent do not


belong to the same class; it is not only a mixture of managerial,
supervisory, and rank-and-file employees — as three (3) are vice-
principals, one (1) is a department head/supervisor, and eleven (11)
are coordinators — but also a combination of teaching and non-
teaching personnel — as twenty-seven (27) are non-teaching
personnel. It insisted that, for not being in accord with Article 245 of the

436
Page 437

Labor Code, private respondent is an illegitimate labor organization


lacking in personality to file a petition for certification election.

Private respondent, however, countered that petitioner failed to


substantiate its claim that some of the employees included in the
petition for certification election holds managerial and supervisory
positions.

The Med-Arbiter denied the petition for certification election on the


ground that the unit which private respondent sought to represent is
inappropriate. She further stated that Section 1 (q), Rule I, Book V of
the Omnibus Rules defines a "bargaining unit" as a group of employees
sharing mutual interests within a given employer unit comprised of all
or less than all of the entire body of employees in the employer unit or
any specific occupational or geographical grouping within such
employer unit. This definition has provided the "community or mutuality
of interest" test as the standard in determining the constituency of a
collective bargaining unit. In the case at bar, the employees of
[petitioner], may, as already suggested, quite easily be categorized into
(2) general classes[:] one, the teaching staff; andtwo, the non-
teaching-staff. Not much reflection is needed to perceive that the
community or mutuality of interest is wanting between the teaching and
the non-teaching staff. These are plain and patent realities which
cannot be ignored. These dictate the separation of these two
categories of employees for purposes of collective bargaining.

Private respondent appealed before the SOLE who ruled against


the dismissal of the petition and directed the conduct of two separate
certification elections for the teaching and the non-teaching personnel.

Hence petitioner went before the Court of Appeals but the latter
dismissed the petition. Anent the alleged mixture of teaching and non-
teaching personnel, the CA agreed with petitioner that the nature of the
former's work does not coincide with that of the latter.

Hence this petition with the Supreme Court alleging the ruling in
Toyota v. Toyota Labor Union stating that supervisory employees are
not eligible to join any labor organizations under Art. 245 of the Labor
Code as amended.

petitioner argues that, in view of the improper mixture of teaching


and non-teaching personnel in private respondent due to the absence
of mutuality of interest among its members, the petition for certification
election should have been dismissed on the ground that private
respondent is not qualified to file such petition for its failure to qualify

437
Page 438

as a legitimate labor organization, the basic qualification of which is the


representation of an appropriate bargaining unit.

ISSUE:
Whether or not there is a need for a mutuality of interest among
the union members.

RULING:
NO.

All said, while the latest issuance is R.A. No. 9481, the 1997
Amended Omnibus Rules, as interpreted by the Court in Tagaytay
Highlands, San Miguel and Air Philippines, had already set the tone for
it. Toyota and Dunlop no longer hold sway in the present altered state
of the law and the rules.

In unequivocal terms, We reiterated that the alleged inclusion of


supervisory employees in a labor organization seeking to represent the
bargaining unit of rank-and-file employees does not divest it of its
status as a legitimate labor organization.

Indeed, Toyota and Dunlop no longer hold true under the law and
rules governing the instant case. The petitions for certification election
involved in Toyota and Dunlop were filed on November 26, 1992 and
September 15, 1995, respectively; hence, the 1989 Rules and
Regulations Implementing R.A. No. 6715 (1989 Amended Omnibus
Rules) was applied. In contrast, D.O. No. 9 is applicable in the petition
for certification election of private respondent as it was filed on May 31,
2002.

Following the doctrine laid down in Kawashima and SMCC-Super,


it must be stressed that petitioner cannot collaterally attack the
legitimacy of private respondent by praying for the dismissal of the
petition for certification election.

The concepts of a union and of a legitimate labor organization are


different from, but related to, the concept of a bargaining unit.

In case of alleged inclusion of disqualified employees in a union,


the proper procedure for an employer like petitioner is to directly file a
petition for cancellation of the union's certificate of registration due to
misrepresentation, false statement or fraud under the circumstances
enumerated in Article 239 of the Labor Code, as amended. To
reiterate, private respondent, having been validly issued a certificate of

438
Page 439

registration, should be considered as having acquired juridical


personality which may not be attacked collaterally.

On the other hand, a bargaining unit has been defined as a "group


of employees of a given employer, comprised of all or less than all of
the entire body of employees, which the collective interests of all the
employees, consistent with equity to the employer, indicated to be best
suited to serve reciprocal rights and duties of the parties under the
collective bargaining provisions of the law." In determining the proper
collective bargaining unit and what unit would be appropriate to be the
collective bargaining agency, the Court, in the seminal case of
Democratic Labor Association v. Cebu Stevedoring Company, Inc., 56
mentioned several factors that should be considered, to wit: (1) will of
employees (Globe Doctrine); (2) affinity and unity of employees'
interest, such as substantial similarity of work and duties, or similarity
of compensation and working conditions; (3) prior collective bargaining
history; and (4) employment status, such as temporary, seasonal and
probationary employees. We stressed, however, that the test of the
grouping is community or mutuality of interest, because "the basic test
of an asserted bargaining unit's acceptability is whether or not it is
fundamentally the combination which will best assure to all employees
the exercise of their collective bargaining rights."

Petitioner appears to have confused the concepts of membership


in a bargaining unit and membership in a union. In emphasizing the
phrase "to the exclusion of academic employees" stated in U.P. v.
Ferrer-Calleja, [petitioner] believed that the petitioning union could not
admit academic employees of the university to its membership. But
such was not the intention of the Supreme Court.

A bargaining unit is a group of employees sought to be


represented by a petitioning union. Such employees need not be
members of a union seeking the conduct of a certification election. A
union certified as an exclusive bargaining agent represents not only its
members but also other employees who are not union members. As
pointed out in our assailed Decision, there were two contending unions
in the U.P. case, namely[,] the Organization of Non-Academic
Personnel of U.P. (ONAPUP) and the All U.P. Worker's Union
composed of both U.P. academic and non-academic personnel.
ONAPUP sought the conduct of a certification election among the rank-
and-file non- academic personnel only, while the All U.P. Workers
Union intended to cover all U.P. rank-and-file employees, involving
both academic and non-academic personnel.

439
Page 440

Case Digests by: GOLANGCO, CLAUDINE GAYLE S.

EMPLOYEES UNION OF BAYER v. BAYER


G.R. No. 162943 December 6, 2010
Villarama, Jr., J.

DOCTRINE:
An intra-union dispute refers to any conflict between and among union members,
including grievances arising from any violation of the rights and conditions of
membership, violation of or disagreement over any provision of the union’s constitution
and by-laws, or disputes arising from chartering or disaffiliation of the union. It must be
remembered that a CBA is entered into in order to foster stability and mutual
cooperation between labor and capital. An employer should not be allowed to rescind
unilaterally its CBA with the duly certified bargaining agent it had previously contracted
with, and decide to bargain anew with a different group if there is no legitimate reason
for doing so and without first following the proper procedure. If such behavior would be
tolerated, bargaining and negotiations between the employer and the union will never
be truthful and meaningful, and no CBA forged after arduous negotiations will ever be
honored or be relied upon.

FACTS:
Petitioner Employees Union of Bayer Philippines 3 (EUBP) is the exclusive bargaining
agent of all rank-and file employees of Bayer Philippines (Bayer), and is an affiliate of the
Federation of Free Workers (FFW). During the negotiations between EUBP President
Facundo and Bayer, EUBP rejected Bayer's 9.9% wage increase proposal resulting in a
bargaining deadlock. Subsequently, EUBP staged a strike, prompting the Secretary of the
Department of Labor and Employment (DOLE) to assume jurisdiction over the dispute. A
new CBA was thereafter registered pursuant to the arbitral award of DOLE to execute a
CBA retroactive January 1, 1997. Barely six months from the signing of the new CBA,
Remigio solicited signatures from union members in support of a resolution containing
the decision of the signatories to: (1) disaffiliate from FFW, (2) rename the union as
Reformed Employees Union of Bayer Philippines (REUBP), (3) adopt a new constitution
and bylaws for the union, (4) abolish all existing officer positions in the union and elect a
new set of interim officers, and (5) authorize REUBP to administer the CBA between EUBP
and Bayer. The said resolution was signed by 147 of the 257 local union members. Remigio
asked Bayer to desist from further transacting with EUBP. Facundo, meanwhile, sent
similar requests to Bayer requesting for the remittance of union dues in favor of EUBP
and accusing the company of interfering with purely union matters. Bayer responded by
deciding not to deal with either of the two groups, and by placing the union dues collected
in a trust account until the conflict between the two groups is resolved. Thereafter, EUBP
filed a complaint for unfair labor practice (first ULP complaint) against Bayer for non
remittance of union dues. While the first ULP case was still pending and despite EUBP's
repeated request for a grievance conference, Bayer decided to turn over the collected
union dues amounting to P254,857.15 to respondent Anastacia Villareal, Treasurer of
REUBP. Petitioners filed a second ULP complaint against herein respondents, amending
the complaint charging the respondents with unfair labor practice committed by
organizing a company union, gross violation of the CBA and violation of their duty to
bargain, complaining the refusal remit the collected union dues to EUBP despite several
demands sent to the management, and opting to negotiate instead with Remigio's group
despite demands to negotiate.

ISSUE:

440
Page 441

Whether the act of the management of Bayer in dealing and negotiating with
Remigio's splinter group despite its validly existing CBA with EUBP can be considered
unfair labor practice and, if so, whether EUBP is entitled to any relief.

RULING:
Yes. CBA is entered into in order to foster stability and mutual cooperation between
labor and capital. An employer should not be allowed to rescind unilaterally its CBA with
the duly certified bargaining agent it had previously contracted with, and decide to
bargain anew with a different group if there is no legitimate reason for doing so and
without first following the proper procedure. If such behavior would be tolerated,
bargaining and negotiations between the employer and the union will never be truthful
and meaningful, and no CBA forged after arduous negotiations will ever be honored or be
relied upon. Art. 253 provides: ART. 253. Duty to bargain collectively when there exists a
collective bargaining agreement. — Where there is a collective bargaining agreement, the
duty to bargain collectively shall also mean that neither party shall terminate or modify
such agreement during its lifetime. However, either party can serve a written notice to
terminate or modify the agreement at least sixty (60) days prior to its expiration date. It
shall be the duty of both parties to keep the status quo and to continue in full force and
effect the terms and conditions of the existing agreement during the 60-day period
and/or until a new agreement is reached by the parties. This is the reason why it is
axiomatic in labor relations that a CBA entered into by a legitimate labor organization that
has been duly certified as the exclusive bargaining representative and the employer
becomes the law between them. Compliance with the terms and conditions of the CBA is
mandated by express policy of the law primarily to afford protection to labor and to
promote industrial peace. Thus, when a valid and binding CBA had been entered into by
the workers and the employer, the latter is behooved to observe the terms and conditions
thereof bearing on union dues and representation. If the employer grossly violates its CBA
with the duly recognized union, the former may be held administratively and criminally
liable for unfair labor practice.

441
Page 442

Case Digests by: GOLANGCO, CLAUDINE GAYLE S.

SACORU v. CCBPI
G.R. No. 200499, October 04, 2017
Caguioa, J.

DOCTRINE:
To prove the existence of unfair labor practice, substantial evidence has got to be
presented.To prove the existence of unfair labor practice, substantial evidence has got
to be presented.

FACTS:
On May 29, 2009, the private respondent company, Coca-Cola Bottlers Philippines.,
Inc. ("CCBPI") issued notices of termination to twenty seven (27) rank-and-file, regular
employees and members of the San Fernando Rank-and-File Union ("SACORU"),
collectively stated as "union members", on the bottom of redundancy because of the
ceding out of two selling and distribution systems, the standard Route System ("CRS") and
Mini Bodega System ("MB") to the Market Execution Partners ("MEPS"), better referred
to as "Dealership System". The union members were also granted individual separation
packages, which twenty-two (22) of them accepted, but under protest. To SACORU, the
new, reorganized selling and distribution systems adopted and implemented by CCBPI
would end in the diminution of the union membership amounting to union busting and
to a violation of the talks Agreement (CBA) provision against contracting out of services
or outsourcing of standard positions; hence, they filed a Notice of Strike with the National
Conciliation and Mediation Board (NCMB) on the bottom of ground of unfair labor
practice, among others. CCBPI, for its part, argued that the new business scheme is
essentially a management prerogative designed to boost the system of selling and
distributing products so as to achieve more consumers at a lesser cost with fewer
manpower complement, but leading to greater returns to investment. SACORU
maintained that that the termination will seriously affect the union membership because
out of 250 members, only 120 members are going to be left upon plan implementation;
that there's no redundancy because the sales force still exists except that job positions
are going to be contracted dead set a sales contractor using company equipment for the
aim of minimizing labor costs because contractual employees don't enjoy CBA benefits;
that the contractualization program of the corporate is against the law because it'll render
the union inutile in protecting the rights of its members as there'll be more contractual
employees than regular employees; which the redundancy program will lead to the
displacement of standard employees which could be a clear case of union busting.

ISSUE:
Whether or not CCBPI committed unfair labor practices.

RULING:
To prove the existence of unfair labor practice, substantial evidence has got to be
presented. SACORU didn't provide any proof that CCBPI acted in an exceedingly malicious
or arbitrarily manner in implementing the redundancy program which resulted within the
dismissal of the 27 employees, which CCBPI engaged instead the services of independent
contractors. As no credible, countervailing evidence had been put forth by SACORU with
which to challenge the validity of the redundancy program implemented by CCBPI, the
alleged unfair labor practice acts allegedly perpetrated against union members might not
be simply swallowed. SACORU was unable to prove its charge of unfair labor practice and

442
Page 443

support its allegations that the termination of the union members was through with the
end-in-view of weakening union leadership and representation. There was no showing
that the redundancy program was motivated by ill will, bad faith or malice, or that it had
been conceived for the aim of interfering with the employees' right to self-organize. The
Court accordingly affirms these findings of the NLRC and also the CA that SACORU did not
present any evidence to prove that the redundancy program interfered with their right to
self-organize.

443
Page 444

Case Digests by: GOLANGCO, CLAUDINE GAYLE S.

SWOFLU V. UNIVERSAL ROBINA CORP.


G.R. NO. 220383. JULY 5, 2017
LEONEN, J.

DOCTRINE:
The CBA controls the relationship between the parties. Any benefit not included in
it is not demandable. The respondent company granted this benefit to its employees to
induce them to waive their collective bargaining rights. But by granting this increase to
petitioners, this Court is eliminating the discrimination against them, which was a result
of respondent's unfair labor practice.

FACTS:
SONEDCO Workers Free Labor Union members are asking that the wage increase
given to their fellow employees be awarded to them as well. Their co-workers of the same
rank are allegedly earning P32.00/day more than they are receiving. In 2007, while there
was no Collective Bargaining Agreement in effect, URCSONEDCO offered, among other
benefits, a P16/day wage increase to their employees. To receive the benefits, employees
had to sign a waiver that said: "In the event that a subsequent CBA is negotiated, the new
CBA shall only be effective on January 1, 2008." Realizing that the waiver was an unfair
labor practice, some members of SONEDCO Workers Free Labor Union refused to sign. In
2008, the same arrangement was made for an additional P16/day wage increase. Both
NLRC and CA found URC-SONEDCO not guilty of unfair practice but it was ordered to pay
petitioners the same wage increase benefit but it was still denied in 2009. In that same
year, CBA came into effect but there was no stipulation to the P16/day increase hence, it
can no longer be imposed upon URC-SONEDCO.

ISSUE:
Whether or not a P32/day continuing wage increase beginning January 1, 2009 to
present should be awarded to petitioners and if respondent is guilty of unfair labor
practice.

RULING:
YES. Generally, the Collective Bargaining Agreement controls the relationship
between the parties. Any benefit not included in it is not demandable. However, in light
of the peculiar circumstances in this case, the requested wage increase should be granted.
The wage increase was integrated in the salary of those who signed the waivers. When
the affiants waived their rights, respondent rewarded them with a P32.00/day wage
increase that continues to this day. The respondent company granted this benefit to its
employees to induce them to waive their collective bargaining rights. This Court has
declared this an unfair labor practice. Accordingly, it is illegal to continue denying the
petitioners the wage increase that was granted to employees who signed the waivers. To
rule otherwise will perpetuate the discrimination against petitioners. All the
consequences of the unfair labor practice must be addressed. By granting this increase to
petitioners, this Court is eliminating the discrimination against them, which was a result
of respondent's unfair labor practice.

444
Page 445

Case Digests by: GOLANGCO, CLAUDINE GAYLE S.

Peninsula Employees Union (PEU) vs. Michael B. Esquivel, et al.


G.R. No. 218454 December 1, 2016
Perlas-Bernabe, J.

DOCTRINE:
Article 250 (n) and ( o ) of the Labor Code mandates the submission of three (3)
documentary requisites in order to justify a valid levy of increased union dues. These
are: (a) an authorization by a written resolution of the majority of all the members at
the general membership meeting duly called for the purpose; (b) the secretary’s record
of the minutes of the meeting, which shall include the list of all members present, the
votes cast, the purpose of the special assessment or fees and the recipient of such
assessment or fees; and (c) individual written authorizations for check-off duly signed by
the employees concerned.

FACTS:
On December 13, 2007, Peninsula Employees Union’ (PEU) Board of Directors
passed Local Board Resolution No. 12, series of 20078 authorizing, among others, the
affiliation of PEU with NUWHRAIN, and the direct membership of its individual members
thereto. On the same day, the said act was submitted to the general membership, and
was duly ratified by 223 PEU members. Beginning January 1, 2009, PEU-NUWHRAIN
sought to increase the union dues/agency fees from one percent (1 % ) to two percent
(2%) of the rank and file employees’ monthly salaries, brought about by PEU’s affiliation
with NUWHRAIN, which supposedly requires its affiliates to remit to it two percent (2%)
of their monthly salaries. The non-PEU members objected to the assessment of increased
agency fees arguing that: (a) the new CBA is unenforceable since no written CBA has been
formally signed and executed by PEU-NUWHRAIN and the Hotel; (b) the 2% agency fee is
exorbitant and unreasonable; and (c) PEU-NUWHRAIN failed to comply with the
mandatory requirements for such increase.

ISSUE:
Whether PEU-NUWHRAIN has the right to collect the increased agency fees.

RULING:
Yes. The recognized collective bargaining union which successfully negotiated the
CBA with the employer is given the right to collect a reasonable fee called ―agency fee‖
from non-union members who are employees of the appropriate bargaining unit, in an
amount equivalent to the dues and other fees paid by union members, in case they accept
the benefits under the CBA. While the collection of agency fees is recognized by Article
259 (formerly Article 248) of the Labor Code, as amended, the legal basis of the union’s
right to agency fees is neither contractual nor statutory, but quasi contractual, deriving
from the established principle that non-union employees may not unjustly enrich
themselves by benefiting from employment conditions negotiated by the bargaining
union. In the present case, PEU-NUWHRAIN’s right to collect agency fees is not disputed.

445
Page 446

Case Digests by: GOLANGCO, CLAUDINE GAYLE S.

UNION OF FILIPRO EMPLOYEES - DRUG, FOOD AND ALLIED INDUSTRIES UNIONS -


KILUSANG MAYO UNO (UFE-DFA-KMU) v. NESTLÉ PHILIPPINES, INCORPORATED
G.R. No. 158930-31 March 3, 2008
Chico-Nazario, J.

DOCTRINE:
For a charge of unfair labor practice to prosper, it must be shown that Nestlé was
motivated by ill will, "bad faith, or fraud, or was oppressive to labor, or done in a
manner contrary to morals, good customs, or public policy, and, of course, that social
humiliation, wounded feelings, or grave anxiety resulted x x x" in disclaiming unilateral
grants as proper subjects in their collective bargaining negotiations. While the law
makes it an obligation for the employer and the employees to bargain collectively with
each other, such compulsion does not include the commitment to precipitately accept
or agree to the proposals of the other. All it contemplates is that both parties should
approach the negotiation with an open mind and make reasonable effort to reach a
common ground of agreement.

FACTS:
The petition sought that Nestle be declared to have committed unfair labor practice
or setting a precondition to bargaining, but was denied. In another case, the petition is
partly granted where it reversed the rule of CA in G.R. SP. No. 69805 that the Secretary of
DOOLE gravely abused its discretion in failing to confine her assumption of jurisdiction
power over the ground rules of the CBA negotiation. Nestle filed a Motion for
Clarification, while Union of Filipino Employees filed a Motion for Partial Reconsideration.
Union of Filipro Employees – Drug, Food and Allied Industries Union – Kilusang Mayo Uno
(UFE-DFA-KMU) is the sole and exclusive bargaining agent of the rank-and-file employees
of Nestlé. The CBA of was about to end on June 5, 2001, and the President of Alabang and
Cabuyao Divisions of UFE-DFA-KMU informed Nestlé of their intent to "open [our] new
Collective Bargaining Negotiation for the year 2001-2004 x x x as early as June 2001.
Nestlé informed them that it was also preparing its own counter-proposal and proposed
ground rules to govern the impending conduct of the CBA negotiations. Nestle requested
NCMB to conduct preventive mediation proceedings between it and UFE-DFA-KMU owing
to an alleged impasse in said dialogue; i.e., that despite fifteen (15) meetings between
them, the parties failed to reach any agreement on the proposed CBA. Conciliation
proceeding proved bargaining deadlock. Another Notice of Strike was filed by the union
die to alleged Nestle’s unfair labor practice, such that it’s bargaining in bad faith by setting
pre-conditions in the rules and/or refusing to include the issue of the Retirement Plan in
the CBA negotiation. prior to holding the strike, Nestlé filed with the DOLE a Petition for
Assumption of Jurisdiction, 14 praying for the Secretary of the DOLE Hon. Sto Tomas, to
assume jurisdiction over the current labor dispute in order to effectively enjoin any
impending strike by the members. Sto Tomas assumed jurisdiction and ordered the strike
or lockout to be enjoined. UFE-DFA-KMU sought reconsideration but was denied. Despite
the order enjoining strike, the employee members went on a strike. An order directed by
Sec. Sto. Tomas provides: 1.) the members to return to work within 24 hrs from receipt;
2.) Nestle to accept back all returning workers; 3.) both parties to cease and desist from
committing act inimical to the on-going conciliation proceedings; and 4.) the submission
of their respective papers. Nestle submitted its position paper, but the Union filed several
pleading instead of a supplemental position paper. One of the pleadings was
Manifestation with Motion for Reconsideration of the Order assailing that the order was

446
Page 447

contrary to law and that Sec. Sto. Tomas could only assume jurisdiction over the issues
mentioned in the notice of strike subject of the current dispute," and that the Amended
Notice of Strike it filed did not cite, as one of the grounds, the CBA deadlock. The Union
filed a petition for certiorari before CA due to alleged grave abuse of discretion amounting
to lack or excess of jurisdiction. Then Acting Secretary of the DOLE, Hon. Arturo D. Brion,
came out with an Order. He dismissed the charge for unfair labor practice, and ruled
among others that the Retirement Plan is not a mandatory subject for bargaining. The
union filed an MR, but was denied. The Union went to CA and file a Petition for Certiorari
for the grave abuse of discretion amounting to lack or excess of jurisdiction

ISSUE:
Whether or not there’s unfair labor practice on the part of Nestle.

RULING:
For a charge of unfair labor practice to prosper, it must be shown that Nestlé was
motivated by ill will, "bad faith, or fraud, or was oppressive to labor, or done in a manner
contrary to morals, good customs, or public policy, and, of course, that social humiliation,
wounded feelings, or grave anxiety resulted x x x" in disclaiming unilateral grants as
proper subjects in their collective bargaining negotiations. While the law makes it an
obligation for the employer and the employees to bargain collectively with each other,
such compulsion does not include the commitment to precipitately accept or agree to the
proposals of the other. All it contemplates is that both parties should approach the
negotiation with an open mind and make reasonable effort to reach a common ground of
agreement. Nestle never refused to bargain collectively with UFE-DFA-KMU. The
corporation simply wanted to exclude the Retirement Plan from the issues to be taken up
during CBA negotiations, on the postulation that such was a unilaterally granted benefit.
An employer’s steadfast insistence to exclude a particular substantive provision is no
different from a bargaining representative’s perseverance to include one that they deem
of absolute necessity Also, Herein, Nestlé is accused of violating its duty to bargain
collectively when it purportedly imposed a precondition to its agreement to discuss and
engage in collective bargaining negotiations with UFE-DFA-KMU. However, a meticulous
review of the record and pleadings of the cases at bar shows that, of the two notices of
strike filed by UFE-DFA-KMU before the NCMB, it was only on the second that the ground
of unfair labor practice was alleged.

447
Page 448

Case Digests by: GOLANGCO, CLAUDINE GAYLE S.

UNION OF FILIPRO EMPLOYEES - DRUG, FOOD AND ALLIED INDUSTRIES UNIONS -


KILUSANG MAYO UNO (UFE-DFA-KMU) v. NESTLÉ PHILIPPINES, INCORPORATED
G.R. No. 158930-31 March 3, 2008
Chico-Nazario, J.

DOCTRINE:
In a CERTIFICATION ELECTION, all employees belonging to the appropriate
bargaining unit can vote. Therefore, a union member who likewise belongs to the
appropriate bargaining unit is entitled to vote in said election. However, the reverse is not
always true; an employee belonging to the appropriate bargaining unit but who is not a
member of the union cannot vote in the union election, unless otherwise authorized by
the constitution and bylaws of the union.

FACTS:
On October 4, 1996, the UST General Faculty Assembly was held. The general
assembly was attended by the University of Santo Tomas Faculty Union (USTFU) and also
non USTFU members. On this occasion, petitioners were elected as USTFU's new set of
officers by acclamation and clapping of hands. The election of the petitioners came about
upon a motion of one Atty. Lopez, not a member of USTFU, that the USTFU Constitution
and By-Laws (CBL) be suspended and that the election be held that day. On December 03,
1996, petitioners and UST entered into a Collective Bargaining Agreement (CBA)
prompting herein respondents to move for the issuance of a restraining order to prevent
petitioners from making further representations. Over petitioner's insistence, the med-
arbiter issued a temporary restraining order directing the respondents to cease and desist
from performing any and all acts pertaining to the duties and functions of the officers and
directors of the USTFU. On appeal, the Bureau of Labor Relations upheld the med-arbiter's
ruling that the USTFU officers' purported "election" was void for having been conducted
in violation of the union's Constitution and By-laws. Hence, the present petition. The main
issue is whether the public respondent Dir. Benedicto Ernesto R. Bitonio Jr. of the Bureau
of Labor Relations committed grave abuse of discretion in refusing to recognize the
officers "elected" during the October 4, 1996 General Assembly.

ISSUE:
Whether the public respondent committed grave abuse of discretion in refusing to
recognize the officers "Elected" During the october 4, 1996 general assembly

RULING:
In dismissing this Petition, we are not passing upon the merits of the
mismanagement allegations imputed by the petitioners to the private respondents; these
are not at issue in the present case SELF-ORGANIZATION is a fundamental right
guaranteed by the Philippine Constitution and the Labor Code. Employees have the right
to form, join or assist labor organizations for the purpose of collective bargaining or for
their mutual aid and protection. Whether employed for a de nite period or not, any
employee shall be considered as such, beginning on his first day of service, for purposes
of membership in a labor union. Corollary to this right is the prerogative not to join,
affiliate with or assist a labor union. Therefore, to become a union member, an employee
must, as a rule, not only signify the intent to become one, but also take some positive
steps to realize that intent. The procedure for union membership is usually embodied in
the union's constitution and bylaws. An employee who becomes a union member

448
Page 449

acquires the rights and the concomitant obligations that go with this new status and
becomes bound by the union's rules and regulations. "When a man joins a labor union (or
almost any other democratically controlled group), necessarily a portion of his individual
freedom is surrendered for the benefit of all members. He accepts the will of the majority
of the members in order that he may derive the advantages to be gained from the
concerted action of all. Just as the enactments of the legislature bind all of us, to the
constitution and by-laws of the union (unless contrary to good morals or public policy, or
otherwise illegal), which are duly enacted through democratic processes, bind all of the
members. If a member of a union dislikes the provisions of the by-laws, he may seek to
have them amended or may withdraw from the union; otherwise, he must abide by them.
It is not the function of courts to decide the wisdom or propriety of legitimate by-laws of
a trade union. "On joining a labor union, the constitution and by-laws become a part of
the member's contract of membership under which he agrees to become bound by the
constitution and governing rules of the union so far as it is not inconsistent with
controlling principles of law. The constitution and by-laws of an unincorporated trade
union express the terms of a contract, which define the privileges and rights secured to,
and duties assumed by, those who have become members. The agreement of a member
on joining a union to abide by its laws and comply with the will of the lawfully constituted
majority does not require a member to submit to the determination of the union any
question involving his personal rights." A UNION ELECTION is held pursuant to the union's
constitution and bylaws, and the right to vote in it is enjoyed only by union members. A
union election should be distinguished from a certification election, which is the process
of determining, through secret ballot, the sole and exclusive bargaining agent of the
employees in the appropriate bargaining unit, for purposes of collective
bargaining.Specifically, the purpose of a certification election is to ascertain whether or
not a majority of the employees wish to be represented by a labor organization and, in
the affirmative case, by which particular labor organization. In a CERTIFICATION
ELECTION, all employees belonging to the appropriate bargaining unit can vote.
Therefore, a union member who likewise belongs to the appropriate bargaining unit is
entitled to vote in said election. However, the reverse is not always true; an employee
belonging to the appropriate bargaining unit but who is not a member of the union cannot
vote in the union election, unless otherwise authorized by the constitution and bylaws of
the union. Verily, union affairs and elections cannot be decided in a non-union activity. In
both elections, there are procedures to be followed. Thus, the October 4, 1996 election
cannot properly be called a union election, because the procedure laid down in the
USTFU's CBL for the election of officers was not followed. It could not have been a
certification election either, because representation was not the issue, and the proper
procedure for such election was not followed. The participation of non-union members
in the election aggravated its irregularity.

449
Page 450

Case Digests by: GOLANGCO, CLAUDINE GAYLE S.

GENERAL MILLING CORPORATION v. COURT OF APPEALS


G.R. No. 146728, 11 February 2004
Quisumbing, J

DOCTRINE:
ART. 250. Procedure in collective bargaining. – The following procedures shall be
observed in collective bargaining: (a) When a party desires to negotiate an agreement, it
shall serve a written notice upon the other party with a statement of its proposals.

FACTS:
In its two plants located at Cebu City and Lapu-Lapu City, petitioner General Milling
Corporation (GMC) employed 190 workers. They were all members of private respondent
General Milling Corporation Independent Labor Union. On April 28, 1989, GMC and the
union concluded a collective bargaining agreement (CBA) which included the issue of
representation effective for a term of three years. The day before the expiration of the
CBA, the union sent GMC a proposed CBA, with a request that a counter-proposal be
submitted within ten (10) days. However, GMC had received collective and individual
letters from workers who stated that they had withdrawn from their union membership,
on grounds of religious affiliation and personal differences. Believing that the union no
longer had standing to negotiate a CBA, GMC did not send any counter-proposal. On
December 16, 1991, GMC wrote a letter to the union’s officers, Rito Mangubat and Victor
Lastimoso. The letter stated that it felt there was no basis to negotiate with a union which
no longer existed, but that management was nonetheless always willing to dialogue with
them on matters of common concern and was open to suggestions on how the company
may improve its operations. In answer, the union officers wrote a letter dated December
19, 1991 disclaiming any massive disaffiliation or resignation from the union and
submitted a manifesto, signed by its members, stating that they had not withdrawn from
the union. NLRC held that the action of GMC in not negotiating was ULP.

ISSUE:
WON the company (GMC) should have entered into collective bargaining with the
union.

RULING:
The law mandates that the representation provision of a CBA should last for five
years. The relation between labor and management should be undisturbed until the last
60 days of the fifth year. Hence, it is indisputable that when the union requested for a
renegotiation of the economic terms of the CBA on November 29, 1991, it was still the
certified collective bargaining agent of the workers, because it was seeking said
renegotiation within five (5) years from the date of effectivity of the CBA on December 1,
1988. The union’s proposal was also submitted within the prescribed 3-year period from
the date of effectivity of the CBA, albeit just before the last day of said period. It was
obvious that GMC had no valid reason to refuse to negotiate in good faith with the union.
For refusing to send a counter-proposal to the union and to bargain anew on the
economic terms of the CBA, the company committed an unfair labor practice under
Article 248 of the Labor Code. ART. 253-A. Terms of a collective bargaining agreement. –
Any Collective Bargaining Agreement that the parties may enter into shall, insofar as the
representation aspect is concerned, be for a term of five (5) years. No petition questioning

450
Page 451

the majority status of the incumbent bargaining agent shall be entertained and no
certification election shall be conducted by the Department of Labor and Employment
outside of the sixty-day period immediately before the date of expiry of such five year
term of the Collective Bargaining Agreement. All other provisions of the Collective
Bargaining Agreement shall be renegotiated not later than three (3) years after its
execution…. ART. 248. Unfair labor practices of employers. – It shall be unlawful for an
employer to commit any of the following unfair labor practice: (g) To violate the duty to
bargain collectively as prescribed by this Code; Under Article 252 above cited, both parties
are required to perform their mutual obligation to meet and convene promptly and
expeditiously in good faith for the purpose of negotiating an agreement. The union lived
up to this obligation when it presented proposals for a new CBA to GMC within three (3)
years from the effectiveness of the original CBA. But GMC failed in its duty under Article
252. What it did was to devise a flimsy excuse, by questioning the existence of the union
and the status of its membership to prevent any negotiation. ART. 250. Procedure in
collective bargaining. – The following procedures shall be observed in collective
bargaining: (a) When a party desires to negotiate an agreement, it shall serve a written
notice upon the other party with a statement of its proposals. The other party shall make
a reply thereto not later than ten (10) calendar days from receipt of such notice. GMC’s
failure to make a timely reply to the proposals presented by the union is indicative of its
utter lack of interest in bargaining with the union. Its excuse that it felt the union no longer
represented the workers, was mainly dilatory as it turned out to be utterly baseless.
Failing to comply with the mandatory obligation to submit a reply to the union’s
proposals, GMC violated its duty to bargain collectively, making it liable for unfairness.

451
Page 452

Case Digests by: GOLANGCO, CLAUDINE GAYLE S.

KIOK LOY v. NLRC


L-54334, 22 January 1986
Cuevas, J.

DOCTRINE:
Collective bargaining, which is defined as negotiations towards a collective
agreement, is one of the democratic frameworks under the New Labor Code, designed to
stabilize the relation between labor and management and to create a climate of sound
and stable industrial peace. It is a mutual responsibility of the employer and the Union
and is characterized as a legal obligation.

FACTS:
The Pambansang Kilusan ng Paggawa (Union for short), a legitimate labor
federation, was elected as the sole and exclusive bargaining agent of the rank-and-file
employees of Sweden Ice Cream Plant (Company for short). The Company’s motion for
reconsideration of the said resolution was denied.Thereafter, the Union furnished 4 the
Company with two copies of its proposed collective bargaining agreement. At the same
time, it requested the Company for its counter proposals, Eliciting no response to the
aforesaid request, the Union again wrote the Company reiterating its request for
collective bargaining negotiations and for the Company to furnish them with its counter
proposals. Both requests were ignored and remained unacted upon by the Company. Left
with no other alternative in its attempt to bring the Company to the bargaining table, the
Union filed a "Notice of Strike", with the Bureau of Labor Relations (BLR) on ground of
unresolved economic issues in collective bargaining. Attempts towards an amicable
settlement failed, prompting the Bureau of Labor Relations to certify the case to the NLRC
for compulsory arbitration pursuant to Presidential Decree No. 823, as amended. For
failure, however, of the parties to submit their respective position papers as required, the
hearing was canceled and reset to another date. Meanwhile, the Union submitted its
position paper. The Company did not, and instead requested for a resetting which was
granted. The Company was directed to submit its financial statements for the years 1976,
1977, and 1978. The case was reset many times upon the request of the defendant's
counsel. When the company’s representative failed to appear in a scheduled hearing and
the defendant's counsel again asked for a postponement, the labor arbiter denied the
request. He also ruled that the Company has waived its right to present further evidence
and, therefore, considered the case submitted for resolution. The LA submitted its report
to the NLRC, which ruled in favor of the union. Petitioner went to the SC assailing the
NLRC order. Petitioner Company now maintains that its right to procedural due process
has been violated when it was precluded from presenting further evidence in support of
its stand and when its request for further postponement was denied. Petitioner further
contends that the National Labor Relations Commission’s finding of unfair labor practice
for refusal to bargain is not supported by law and the evidence considering that it was
only on May 24. 1979 when the Union furnished them with a copy of the proposed
Collective Bargaining Agreement and it was only then that they came to know of the
Union’s demands; and finally, that the Collective Bargaining Agreement approved and
adopted by the National Labor Relations Commission is unreasonable.

ISSUE:
Whether or not Sweden is guilty of ULP for unjustified refusal to bargain.

452
Page 453

RULING:
Yes. Collective bargaining, which is defined as negotiations towards a collective
agreement, is one of the democratic frameworks under the New Labor Code, designed to
stabilize the relation between labor and management and to create a climate of sound
and stable industrial peace. It is a mutual responsibility of the employer and the Union
and is characterized as a legal obligation. So much so that Article 249, par. (g) of the Labor
Code makes it an unfair labor practice for an employer to refuse "to meet and convene
promptly and expeditiously in good faith for the purpose of negotiating an agreement
with respect to wages, hours of work, and all other terms and conditions of employment
including proposals for adjusting any grievance or question arising under such an
agreement and executing a contract incorporating such agreement, if requested by either
party." From the overall conduct of petitioner company in relation to the task of
negotiation, there can be no doubt that the Union has a valid cause to complain against
its (Company’s) attitude, the totality of which is indicative of the latter’s disregard of, and
failure to live up to, what is enjoined by the Labor Code — to bargain in good faith. The
Court is in total conformity with respondent NLRC’s pronouncement that petitioner
Company is guilty of unfair labor practice.

453
Page 454

Case Digests by: GOLANGCO, CLAUDINE GAYLE S.

COLEGIO DE SAN JUAN DE LETRAN v. ASSOC. EMPLOYEES ANS=D FACULTIES OF


LETRAN
G.R. 141471, September 18, 2001
KAPUNAN, J.

DOCTRINE:
The duty to bargain collectively includes the mutual obligation to meet and convene
promptly and expeditiously in good faith for the purpose of negotiating an agreement.

FACTS:
Salvador Abtria, President of the respondent union initiated renegotiations of its
CBA with petitioners for the last two years of CBA’s 5 years lifetime from 1989-1994. In
the same year, the union elected a new set of officers with private respondents Eleanor
Ambas as the newly elected President. Ambas wanted to continue renegotiation, but
petitioner claimed that the CBA was already prepared for signing. The CBA was submitted
to a referendum which was rejected by the union members. Later, the union notified the
NCMB of its intention to strike due to petitioners' refusal to bargain. Thereafter, the
parties agreed to disregard the unsigned CBA and to start negotiation on a new five-year
CBA. The union submitted its proposals to the petitioner, which notified the union that
the same was submitted to its Board of Trustees. Meanwhile, Ambas work schedule was
changed, which she protested and requested to be submitted to grievance machinery
under the old CBA. Due to petitioners’ inaction, the union filed a notice of strike. Later,
the Ambas were dismissed for alleged insubordination. Both parties again discussed the
ground rules for the CBA renegotiations; however, the petitioner stopped negotiations
after allegedly receiving information that a new group of employees had filed a PCE. The
union struck and the Secretary assumed jurisdiction ordering all striking workers to return
to work. All were readmitted except Ambas. Public respondent declared petitioner quilt
of ULP and directed reinstatement of Ambas with back wages.

ISSUE:
Whether petitioner is guilty of unfair labor practice by refusing to bargain within the
union when it unilaterally suspended the ongoing negotiations for a new Collective
Bargaining Agreement (CBA) upon mere information that a petition for certification has
been filed by another legitimate labor organization.

RULING:
No. The duty to bargain collectively includes the mutual obligation to meet and
convene promptly and expeditiously in good faith for the purpose of negotiating an
agreement. Petitioner failed to make a timely reply to the unions proposals, thereby
violating the proper procedure in collective bargaining as provided in Article 250. In order
to allow the employer to validly suspend the bargaining process, there must be a valid
PCE raising a legitimate representation issue, in this case, the petition was filed outside
the 60-days freedom period; therefore there was no legitimate representation issue and
the filing of the PCE did not constitute to the ongoing negotiation.

454
Page 455

Case Digests by: GOLANGCO, CLAUDINE GAYLE S.

PHILIPPINE AIRLINES vs. PHILIPPINE AIRLINES EMPLOYEES ASSOCIATION


G.R. No. 142399, 12 March 2008
Chico-Nazario, J.

DOCTRINE:
The benefits of a CBA extend to the laborers and employees in the collective
bargaining unit, including those who do not belong to the chosen bargaining labor
organization. Otherwise, it would be a clear case of discrimination.

FACTS:
In 1987, Philippine Airlines (PAL for brevity) and Philippine Airlines Employees
Association (PALEA for brevity), the collective bargaining agent of the rank and file
employees of PAL, entered into a Collective Bargaining Agreement covering the period of
1986-1989. Part of the said agreement required PAL to pay its rank and file employees
13th Month Pay (Mid-year Bonus) and Christmas Bonus. Prior to the payment of the 13th
Month Pay, PAL released an implementing guideline on 22 April 1988 which stated that
only the ground employees in the general payroll who are regular as of April 30, 1988 and
other ground employees in the general payroll, not falling within category previously
mentioned shall receive their 13th Month Pay on or before December 24, 1988. PALEA
assailed the implementation of the foregoing guideline. It contented that all employees
of PAL, whether regular or non-regular, should be paid their 13th month pay. In response,
PAL informed PALEA that rank and file employees who were regularized after 30 April
1988 were not entitled to the 13th month pay as they were already given the Christmas
bonus in December of 1988, per the Implementing Rules of Presidential Decree No. 851.
PALEA, disagreeing with PAL, filed a complaint for unfair labor practice before the NLRC
contending that "the cut-off period for regularization should not be used as the
parameter for granting the 13th month pay considering that the law does not distinguish
the status of employment but (sic) the law covers all employees." In refusing payment of
the mid-year bonus, PAL argued that 1) the CBA does not apply to non-regular employees
such that any benefits arising from said agreement cannot be made to apply to them,
including the mid-year bonus; and 2) it has always been the company practice not to
extend the mid-year bonus to those employees who have not attained regular status prior
to the month of May, when payment of the particular bonus accrues.

ISSUE:
Whether or not the benefits of the CBA between PAL and PALEA extend to non-
regular employees of the former.

RULING:
The Court ruled in the affirmative.It is a well-settled doctrine that the benefits of a
CBA extend to the laborers and employees in the collective bargaining unit, including
those who do not belong to the chosen bargaining labor organization. Otherwise, it would
be a clear case of discrimination. Hence, to be entitled to the benefits under the CBA, the
employees must be members of the bargaining unit, but not necessarily of the labor
organization designated as the bargaining agent. The allegation of petitioner PAL that the
non-regular employees do not belong to the collective bargaining unit and are thus not
covered by the CBA is unjustified and unsubstantiated. PAL excludes certain employees

455
Page 456

from the benefits of the CBA only because they have not yet achieved regular status by
the cut-off date, 30 April 1988. There is no showing that the non-regular status of the
concerned employees by said cut-off date sufficiently distinguishes their interests from
those of the regular employees so as to exclude them from the collective bargaining unit
and the benefits of the CBA. Hence, the benefits provided by the subject CBA are
applicable even to non-regular employees who belong to the bargaining.

456
Page 457

Case Digests by: GOLANGCO, CLAUDINE GAYLE S.

FVC LABOR UNION v. SANAMA-FVC-SIGLO


G.R. No. 176249, 27 November 2009
Brion, J

DOCTRINE:
Article 253-A of the Labor Code provides that a term of 5 years in so far as the
representation aspect is concerned. All other provisions of the CBA shall be renegotiated
not later than 3 years after its execution.

FACTS:
On December 22, 1997, the petitioner FVCLU-PTGWO, the recognized bargaining
agent of the rank-and file employees of the FVC Philippines, Inc., signed a five -year
collective bargaining agreement with the company to cover the period from February 1,
1998 to January 30, 2003. At the end of the 3rd year of the five-year term and pursuant
to the CBA, FVCLU-PTGWO and the company entered into the renegotiation of the CBA
and modified, among other provisions, the CBAs duration. The renegotiated CB provides
that the renegotiation agreement shall take effect beginning February 1, 2001 and until
May 31, 2003 thus extending the original five-year period of the CBA by four months.
However, on January 21, 2003, nine days before the January 30, 2003 expiration of the
original five-year CBA, the respondent SANAMA-FVC-SIGLO filed before the DOLE a
petition for certification election covered by the FVCLU-PTGWO CBA. Hence, FVCLU -
PTGWO moved to dismiss the petition on the ground that the certification election
petition was filed outside the freedom period or before the expiration of the CBA on May
31, 2003. MED-ARBITER RULING: dismissed the petition on the ground that it was filed
outside the 60-day period counted from the May 31, 2003 expiry date of the amended
CBA. SANAMA-SIGLO appealed the Med-Arbiters Order to the DOLE Secretary,
contending that the filing of the petition on January 21, 2003 was within 60-days from the
January 30, 2003 expiration of the original CBA term. DOLE RULING: sustained SANAMA-
SIGLOs position, thereby setting aside the decision of the Med-Arbiter. However, the
DOLE Acting Secretary later reversed the decision and ruled that the amended CBA had
been ratified by members of the bargaining unit some of whom later organized
themselves as SANAMA-SIGLO, the certification election applicant. CA RULING: set aside
the challenged DOLE Secretary decisions and reinstated her earlier ruling calling for a
certification election.

ISSUE:
Whether or not petitioner-union should remain to be the SEBA by virtue of the
extended term of the CBA.

RULING:
No. Article 253-A of the Labor Code provides that a term of 5 years in so far as the
representation aspect is concerned. All other provisions of the CBA shall be renegotiated
not later than 3 years after its execution. Any agreement on such other provisions of the
CBA entered into within six (6) months from the date of expiry of the term of such other
provisions as fixed in such Collective Bargaining Agreement, shall retroact to the day
immediately following such date. If any such agreement is entered into beyond six
months, the parties shall agree on the duration of retroactivity thereof. In case of a
deadlock in the renegotiation of the collective bargaining agreement, the parties may

457
Page 458

exercise their rights under this Code. The root of the controversy can be traced to a
misunderstanding of the interaction between a union’s exclusive bargaining
representation status in a CBA and the term or effective period of the CBA. FVCLU-PTGWO
has taken the view that its exclusive representation status should fully be in step with the
term of the CBA and that this status can be challenged only within 60 days before the
expiration of this term. Thus, when the term of the CBA was extended, its exclusive
bargaining status was similarly extended so that the freedom period for the filing of a
petition for certification election should be counted back from the expiration of the
amended CBA term. We hold this FVCLU-PTGWO position to be correct, but only with
respect to the original five-year term of the CBA which, by law, is also the effective period
of the union’s exclusive bargaining representation status. While the parties may agree to
extend the CBA’s original five-year term together with all other CBA provisions, any such
amendment or term in excess of five years will not carry with it a change in the union’s
exclusive collective bargaining status. By express provision of the above-quoted Article
253-A, the exclusive bargaining status cannot go beyond five years and the representation
status is a legal matter not for the workplace parties to agree upon. In other words,
despite an agreement for a CBA with a life of more than five years, either as an original
provision or by amendment, the bargaining union’s exclusive bargaining status is effective
only for five years and can be challenged within sixty (60) days prior to the expiration of
the CBA’s first five years. In the event however, that the parties, by mutual agreement,
enter into a renegotiated contract with a term of three (3) years or one which does not
coincide with the said five-year term and said agreement is ratified by majority of the
members in the bargaining unit, the subject contract is valid and legal and therefore,
binds the contracting parties. The same will however not adversely affect the right of
another union to challenge the majority status of the incumbent bargaining agent within
sixty (60) days before the lapse of the original five (5) year term of the CBA. In the present
case, the CBA was originally signed for a period of five years, i.e., from February 1, 1998
to January 30, 2003, with a provision for the renegotiation of the CBA’s other provisions
at the end of the 3rd year of the five-year CBA term. Thus, prior to January 30, 2001 the
workplace parties sat down for renegotiation but instead of confining themselves to the
economic and non-economic CBA provisions, also extended the life of the CBA for another
four months, i.e., from the original expiry date on January 30, 2003 to May 30, 2003. As
discussed above, this negotiated extension of the CBA term has no legal effect on the
FVCLU-PTGWO’s exclusive bargaining representation status which remained effective
only for five years ending on the original expiry date of January 30, 2003. Thus, sixty days
prior to this date, or starting December 2, 2002, SANAMA-SIGLO could properly file a
petition for certification election. Its petition, filed on January 21, 2003 or nine (9) days
before the expiration of the CBA and of FVCLU-PTGWO’s exclusive bargaining status, was
seasonably filed.

458
Page 459

Case Digests by: JAYMEE SAMANTHA O. GONZALEZ

IHONGKONG BANK INDEPENDENT LABOR UNION


(HBILU) vs. HONGKONG AND SHANGHAI BANKING
CORPORATION LIMITED
G.R. No. 218390. February 28, 2018.
Velasco, Jr., J.

DOCTRINE:
PROHIBITION AGAINST THE UNILATERAL MODIFICATION OF A
CBA DURING ITS SUBSISTENCE AND EVEN THEREAFTER UNTIL
A NEW AGREEMENT IS REACHED
The provisions of the CBA must be respected since its terms and
conditions constitute the law between the parties. And until a new CBA
is executed by and between the parties, they are duty-bound to keep
the status quo and to continue in full force and effect the terms and
conditions of the existing agreement.

FACTS:
During the negotiations for a new Collective Bargaining Agreement
with petitioner Hongkong Bank Independent Labor Union (HBILU),
respondent Hongkong and Shanghai Banking Corporaton Limited
(HSBC) proposed amendments to ‘Article XI - Salary Loans’
incorporating a checking proviso stating that "repayment defaults on
existing loans and adverse information on outside loans will be
considered in the evaluation of loan applications.” HBILU objected to
the proposals arguing that their insertions would curtail its members’
availment of salary loans and would in turn, violate the existing
exceptions set forth in BSP Circular 423, Series of 2004 and Section
X338.3 of the Manual of Regulations for Banks (MoRB). Due to the
objection, HSBC withdrew its proposed amendments, and Article XI
remained unchanged.

On April 12, 2012, HSBC sent an email to all its employees regarding
the enforcement of the Financial Assistance Plan and included therein
the Credit Checking Provisions despite the withdrawal of the proposal.
Afterwards, HBILU member Vince Mananghaya applied for a loan
under Article XI of the CBA. His first loan application in March 2012
was approved, but adverse findings from the external checks on his
credit background resulted in the denial of his September application.
As a result of such denial, HBILU filed the issue with the National
Conciliation Mediation Board (NCMB) arguing that under the

459
Page 460

terms of Article XI, there is no such requirement and that it cannot,


therefore, be unilaterally imposed by HSBC. On the other hand, HSBC
claimed the adoption of the Plan is a condition sine qua non for any
loan grant under Section X338 of the MoRB. Moreover, the Credit
Check policy has been in place since 2003, and is a sound practice in
the banking industry to protect the interests of the public and preserve
confidence in banks.

The NCMB-PVA rendered a decision in favor of HSBC. It ruled that


respondent, as an employer, has the right to issue and implement
guidelines for the availment of loan accommodations under the CBA
as part of its management prerogative. It further stated that the bank
was constrained to comply with Section X338 of the MoRB. As a
banking institution, HSBC cannot divorce itself from the regulatory
powers of the BSP. The CA sustained the ruling of NCMB-PVA.

ISSUE:
Whether or not HSBC could validly enforce the credit-checking
requirement under its BSP-approved Plan in processing the salary loan
applications of covered employees even when the said requirement is
not recognized under the CBA.

RULING:
No. Respondent HSBC’s Financial Assistance Plan, insofar as it
unilaterally imposed a credit checking proviso on the availment of
Salary Loans by its employees under Article XI of the 2010-2012 CBA,
is invalid.

The provisions of the CBA must be respected since its terms and
conditions constitute the law between the parties. And until a new CBA
is executed by and between the parties, they are duty-bound to keep
the status quo and to continue in full force and effect the terms and
conditions of the existing agreement. This finds basis under Art. 253 of
the Labor Code. The Plan was never made part of the CBA. As a
matter of fact, HBILU vehemently rejected the Plan's incorporation into
the agreement. Due to this lack of consensus, the bank withdrew its
proposal and agreed to the retention of the original provisions of the
CBA. The subsequent implementation of the Plan's external credit
check provisions in relation to employee loan applications under Article
XI of the CBA was then an imposition solely by HSBC.

The Court ruled that tolerating HSBC's conduct would be tantamount


to allowing a blatant circumvention of Article 253 of the Labor Code. It

460
Page 461

would contravene the express prohibition against the unilateral


modification of a CBA during its subsistence and even thereafter until
a new agreement is reached. It would unduly license HSBC to add,
modify, and ultimately further restrict the grant of Salary Loans beyond
the terms of the CBA by simply adding stringent requirements in its
Plan, and having the said Plan approved by BSP in the guise of
compliance with the MoRB. The express provisions of said CBA inked
by the parties clearly make no reference to the Plan. And even in the
enforcement thereof, credit checking was not included as one of its
requirements. This leads to the conclusion that HSBC originally never
intended the credit checking requirement under the Plan to apply to
salary loans under the CBA.

461
Page 462

Case Digests by: JAYMEE SAMANTHA O. GONZALEZ

MANILA ELECTRIC COMPANY vs. Hon. SECRETARY of LABOR


LEONARDO QUISUMBING and MERALCO EMPLOYEES and
WORKERS ASSOCIATION (MEWA)
G.R. No. 127598. February 22, 2000 and August 1, 2000.
Ynares-Santiago, J.

DOCTRINE:
RETROACTIVITY OF THE EFFECTIVITY OF ARBITRAL AWARDS
The Court rules that CBA arbitral awards granted after six months from
the expiration of the last CBA shall retroact to such time agreed upon
by both employer and the employees or their union. Absent such an
agreement as to retroactivity, the award shall retroact to the first day
after the six-month period following the expiration of the last day of the
CBA should there be one. In the absence of a CBA, the Secretary's
determination of the date of retroactivity as part of his discretionary
powers over arbitral awards shall control.

FACTS:
The Court promulgated a decision on January 27, 1999 directing the
parties to execute a Collective Bargaining Agreement incorporating the
terms and conditions contained in the portions of the Secretary of
Labor's orders of August 19, 1996 and December 28, 1996, and the
modifications stated. Dissatisfied with the decision, some members of
the respondent union Meralco Employees and Workers Association
(MEWA) filed a motion for intervention and a motion for
reconsideration. A separate intervention was likewise made by the
supervisor’s union (FLAMES) alleging that it has a bona fide legal
interest in the outcome of the case.

The issues raised involves the amount of wages and the retroactivity
of the Collective Bargaining Agreement (CBA) arbitral awards. On the
issue on the retroactivity of the CBA arbitral award, petitioner Manila
Electric Company claims that the award should retroact only from such
time that the Secretary of Labor rendered the award, invoking the 1995
decision in Pier 8 case, where the Court citing Union of Filipino
Employees v. NLRC, said: “Based on the provision of Section 253-A,
its retroactivity should be agreed upon by the parties. But since no
agreement to that effect was made, public respondent did not abuse
its discretion in giving the said CBA a prospective effect. The action of
the public respondent is within the ambit of its authority vested by
existing law."

462
Page 463

On the other hand, respondent union argues the award should retroact
to such time granted by the Secretary, citing the 1993 decision of St.
Luke's. “In the absence of a specific provision of law prohibiting
retroactivity of the effectivity of arbitral awards issued by the Secretary
of Labor pursuant to Article 263(g) of the Labor Code, such as herein
involved, public respondent is deemed vested with plenary and
discretionary powers to determine the effectivity thereof."

ISSUE:
When shall the arbitral award in a labor dispute where the Secretary
had assumed jurisdiction retroact?

RULING:
FEBRUARY 22, 2000 (RESOLUTION)
The subject CBA awards shall retroact from December 1, 1995 to
November 30,1997.

Labor laws are silent as to when an arbitral award in a labor dispute


where the Secretary had assumed jurisdiction by virtue of Article 263
(g) of the Labor Code shall retroact. Despite the silence of the law, the
Court rules herein that CBA arbitral awards granted after six months
from the expiration of the last CBA shall retroact to such time agreed
upon by both employer and the employees or their union. Absent such
an agreement as to retroactivity, the award shall retroact to the first day
after the six-month period following the expiration of the last day of the
CBA should there be one. In the absence of a CBA, the Secretary's
determination of the date of retroactivity as part of his discretionary
powers over arbitral awards shall control.

It is true that an arbitral award cannot per se be categorized as an


agreement voluntarily entered into by the parties because it requires
the interference and imposing power of the State thru the Secretary of
Labor when he assumes jurisdiction. However, the arbitral award can
be considered as an approximation of a collective bargaining
agreement which would otherwise have been entered into by the
parties. The terms or periods set forth in Article 253-A pertains
explicitly to a CBA. But there is nothing that would prevent its
application by analogy to an arbitral award by the Secretary
considering the absence of an applicable law.

Under Article 253-A: "If any such agreement is entered into beyond six
months, the parties shall agree on the duration of retroactivity thereof."
In other words, the law contemplates retroactivity whether the
agreement be entered into before or after the said six-month

463
Page 464

period. The agreement of the parties need not be categorically stated


for their acts may be considered in determining the duration of
retroactivity.

In this connection, the Court considers the letter of petitioner's


Chairman of the Board and its President addressed to their
stockholders, which states that the CBA "for the rank-and-file
employees covering the period December 1, 1995 to November 30,
1997 is still with the Supreme Court," as indicative of petitioner's
recognition that the CBA award covers the said period. Furthermore,
petitioner does not dispute the allegation that in the past CBA arbitral
awards, the Secretary granted retroactivity commencing from the
period immediately following the last day of the expired CBA. Thus, by
petitioner's own actions, the Court sees no reason to retroact the
subject CBA awards to a different date. The period is herein set at two
(2) years from December 1, 1995 to November 30, 1997.

AUGUST 1, 2000 (RESOLUTION)


The arbitral award shall retroact to the two-year period from June 1,
1996 to May 31, 1998.

The Court took into account the fact that petitioner belongs to an
industry imbued with public interest. As such, this Court can not ignore
the enormous cost that petitioner will have to bear as a consequence
of the full retroaction of the arbitral award to the date of expiry of the
CBA, and the inevitable effect that it would have on the national
economy. On the other hand, under the policy of social justice, the law
bends over backward to accommodate the interests of the working
class on the humane justification that those with less privilege in life
should have more in law.

Balancing these two contrasting interests, this Court turned to the


dictates of fairness and equitable justice and thus arrived at a formula
that would address the concerns of both sides. Hence, this Court held
that the arbitral award in this case be made to retroact to the first day
after the six-month period following the expiration of the last day of the
CBA, i.e., from June 1, 1996 to May 31, 1998. This Court, therefore,
maintains the foregoing rule in the assailed Resolution pro hac vice.

464
Page 465

Case Digests by: JAYMEE SAMANTHA O. GONZALEZ

INTERNATIONAL SCHOOL ALLIANCE OF EDUCATORS (ISAE)


vs. HON. LEONARDO QUISIMBING
G.R. No. 128845. June 1, 2000.
Kapunan, J.

DOCTRINE:
FOREIGN-HIRES SHOULD NOT BELONG TO THE SAME
BARGAINING UNIT AS LOCAL-HIRES
Foreign-hires do not belong to the same bargaining unit as the local-
hires.The factors in determining the appropriate collective bargaining
unit are (1) the will of the employees (Globe Doctrine); (2) affinity and
unity of the employees' interest, such as substantial similarity of work
and duties, or similarity of compensation and working conditions
(Substantial Mutual Interests Rule); (3) prior collective bargaining
history; and (4) similarity of employment status.The basic test of an
asserted bargaining unit's acceptability is whether or not it is
fundamentally the combination which will best assure to all employees
the exercise of their collective bargaining rights.

FACTS:
Private respondent International School, Inc. is a domestic educational
institution established primarily for dependents of foreign diplomatic
personnel and other temporary residents. It hires both foreign and local
teachers as members of its faculty, classifying the same into two: (1)
foreign-hires and (2) local hires.The School grants foreign-hires certain
benefits not accorded local-hires; these include housing,
transportation, shipping costs, taxes, and home leave travel allowance.
Foreign-hires are also paid a salary rate twenty-five percent (25%)
more than local-hires. The School justifies the difference on two
"significant economic disadvantages" foreign-hires have to endure,
namely: (a) the "dislocation factor" and (b) limited tenure.

During the negotiations for a new collective bargaining agreement,


petitioner International School Alliance of Educators, the collective
bargaining representative of all faculty members of the School,
objected to the difference in salary rates between foreign and local-
hires. Petitioner claims that the point-of-hire classification employed by
the School is discriminatory to Filipinos and that the grant of higher

salaries to foreign-hires constitutes racial discrimination. The issue


caused a deadlock between the parties. As a result thereof, the DOLE
Acting Secretary Trajano assumed jurisdiction over the dispute. Sec.

465
Page 466

Trajano ruled in favor of the School, stating: “there is a substantial


distinction between foreign hires and local hires, the former enjoying
only a limited tenure, having no amenities of their own in the Philippines
and have to be given a good compensation package in order to attract
them to join the teaching faculty of the School.”

ISSUE:
1) Whether or not the disparity between the salaries of foreign-hires
and local-hires are valid.
2) Whether or not foreign hires should be included in the appropriate
bargaining unit.

RULING:
1) No. Persons who work with substantially equal qualifications, skill,
effort and responsibility, under similar conditions, should be paid
similar salaries. This rule applies to the School, its "international
character" notwithstanding. In this case, there is no evidence here
that foreign-hires perform 25% more efficiently or effective than the
local-hires. Both groups have similar functions and responsibilities,
which they perform under similar working conditions.

The point-of-hire classification employed by respondent School to


justify the distinction in the salary rates of foreign-hires and local
hires to be an invalid classification. There is no reasonable
distinction between the services rendered by foreign-hires and
local-hires. The practice of the School of according higher salaries
to foreign-hires contravenes public policy and, certainly, does not
deserve the sympathy of this Court.

2) No. Foreign-hires do not belong to the same bargaining unit as the


local-hires. The factors in determining the appropriate collective
bargaining unit are:

a. the will of the employees (Globe Doctrine);


b. affinity and unity of the employees' interest, such as substantial
similarity of work and duties, or similarity of compensation and
working conditions (Substantial Mutual Interests Rule);
c. prior collective bargaining history; and
d. similarity of employment status.

The basic test of an asserted bargaining unit's acceptability is whether


or not it is fundamentally the combination which will best assure to all
employees the exercise of their collective bargaining rights

466
Page 467

In this case, it does not appear that foreign-hires have indicated their
intention to be grouped together with local-hires for purposes of
collective bargaining. The collective bargaining history in the School
also shows that these groups were always treated separately. Foreign-
hires have limited tenure; local-hires enjoy security of tenure. Although
foreign-hires perform similar functions under the same working
conditions as the local-hires, foreign-hires are accorded certain
benefits not granted to local-hires. These benefits, such as housing,
transportation, shipping costs, taxes, and home leave travel allowance,
are reasonably related to their status as foreign-hires, and justify the
exclusion of the former from the latter. To include foreign-hires in a
bargaining unit with local-hires would not assure either group the
exercise of their respective collective bargaining rights

467
Page 468

Case Digests by: JAYMEE SAMANTHA O. GONZALEZ

NATIONAL ASSOCIATION OF FREE TRADE UNIONS (NAFTU)


vs. MAINIT LUMBER DEVELOPMENT COMPANY WORKERS
UNION-UNITED LUMBER AND GENERAL WORKERS
OF THE PHILIPPINES (MALDECOWU-ULGWP)
G.R. No. 79526. December 21, 1990.
Paras, J.

DOCTRINE:
THE TEST OF GROUPING BARGAINING UNITS IS COMMUNITY
OR MUTUALITY OF INTERESTS
While the existence of a bargaining history is a factor that may be
reckoned with in determining the appropriate bargaining unit, the same
is not decisive or conclusive. Other factors must be considered. The
test of grouping is community or mutuality of interests. This is so
because "the basic test of an asserted bargaining unit's acceptability
is whether or not it is fundamentally the combination which will best
assure to all employees the exercise of their collective bargaining
rights."

FACTS:
Private respondent Mainit Lumber Development Company Workers
Union-United Lumber and General Workers of the Philippines
(ULGWP) filed a petition for certification election to determine the sole
and exclusive collective bargaining representative among the rank and
file workers/employees of Mainit Lumber Development Company Inc.
(MALDECO). The Med-Arbiter granted the petition but National
Association of Free Trade Union (NAFTU) appealed claiming that
MALDECO was composed of two bargainig units, the Sawmill Division
and the Logging Division, but both the petition and decision treated
these separate and distinct units only as one.

A certification election was held on separate dates at the employer's


sawmill division and logging area, respectively. In said election
MALDECOWU-ULGWP garnered a total vote of 146 while NAFTU
garnered a total of 2 votes.

ISSUE:
Whether or not it was appropriate for the Med-Arbiter to change the
employer from two separate bargaining units to only one.

468
Page 469

RULING:
Yes. While the existence of a bargaining history is a factor that may be
reckoned with in determining the appropriate bargaining unit, the same
is not decisive or conclusive. Other factors must be considered. The
test of grouping is community or mutuality of interests. This is so
because "the basic test of an asserted bargaining unit's acceptability
is whether or not it is fundamentally the combination which will best
assure to all employees the exercise of their collective bargaining
rights." Significantly, out of two hundred and one (201) employees of
MALDECO, one hundred seventy five (175) consented and supported
the petition for certification election, thereby confirming their desire for
one bargaining representative.

There is a mutuality of interest among the employees of the Sawmill


Division and the Logging Division. Their functions mesh with one
another. One group needs the other in the same way that the company
needs them both. There may be difference as to the nature of their
individual assignments but the distinctions are not enough to warrant
the formation of a separate bargaining unit.

469
Page 470

Case Digests by: JAYMEE SAMANTHA O. GONZALEZ

PICOP RESOURCES, INCORPORATED (PRI) vs.


RICARDO DEQUILLA ET AL.
G.R. No. 172666. December 7, 2011.
Mendoza, J.

DOCTRINE:
RECOGNITION OF THE MAJORITY STATUS OF THE INCUMBENT
BARGAINING AGENT DOES NOT HOLD TRUE WHEN PETITIONS
FOR CERTIFICATION ELECTION ARE FILED DURING THE
FREEDOM PERIOD
While it is incumbent for the employer to continue to recognize the
majority status of the incumbent bargaining agent even after the
expiration of the freedom period, they could only do so when no petition
for certification election was filed. The reason is, with a pending petition
for certification, any such agreement entered into by management with
a labor organization is fraught with the risk that such a labor union may
not be chosen thereafter as the collective bargaining representative.

FACTS:
Private respondents Ricardo Dequilla, Cesar Atienza and Ancieto
Orbeta filed a complaint before the NLRC Regional Arbitration for
Unfair Labor Practice and Illegal Dismissal with money claims,
damages and attorney’s fees against PICOP Resources, Incorporated.
Their employment was terminated due to acts of disloyalty, specifically,
for allegedly campaigning, supporting and signing a petition for the
certification of a rival union, the Federation of Free Workers Union
(FFW) before the 60-day "freedom period" and during the effectivity of
the CBA. According to Atty. Fuentes, then National President of the
Southern Philippines Federation of Labor (SPFL), such acts were
construed to be a valid cause for termination under the terms and
conditions of the CBA.

The LA declared as illegal the termination of the private respondents


whereas the NLRC reversed and set aside its former decision
dismissing the appeal of PICOP.

On appeal, the CA reinstated the decision of the LA. It ruled that the
act of signing an authorization for the filing of the petition for
certification election of a rival union is not a sufficient ground to
terminate the employment of the petitioners since the petition itself

was filed during the freedom period. There must be proof of


contemporaneous acts of resignation or withdrawal of their

470
Page 471

membership from the Respondent NAMAPRI-SPFL to which they are


members. Furthermore, Art. 256 of the Labor Code is applicable in this
case. Art. 256 provides that at the expiration of the 60-day period
reckoned from the expiration date of the CBA, the employer shall
continue to recognize the majority status of the incumbent bargaining
agent only where no petition for certification election is filed. However,
the petition was already filed by the Petitioners during the 60-day
freedom period. Clearly, from the imports of the provision, it will render
nugatory the purpose of the law providing for a freedom period for the
filing of a petition for certification election should the act of signing/filing
the said petition be interpreted as an act of disloyalty.

ISSUE:
1) Whether or not the of signing an authorization for the filing of the
petition for certification election of a rival union is an act of
disloyalty.
2) Whether or not Art. 256 (now Art. 268) is applicable in this case.

RULING:
1) No. Their mere act of signing an authorization for a petition for
certification election before the freedom period does not
necessarily demonstrate union disloyalty. It is far from being within
the definition of "acts of disloyalty" as PICOP would want the Court
to believe. The act of "signing an authorization for a petition for
certification election" is not disloyalty to the union per se
considering that the petition for certification election itself was filed
during the freedom period which started on March 22, 2000.

As ruled by the CA, the records are bereft of proof of any


contemporaneous acts of resignation or withdrawal of union
membership or non-payment of union dues. Neither is there proof
that private respondents joined FFW. The fact is, private
respondents remained in good standing with their union,
NAMAPRI-SPFL.

2) Yes. Art. 256 (now Art. 268) should be applied. Applying the
provision, it can be said that while it is incumbent for the employer
to continue to recognize the majority status of the incumbent
bargaining agent even after the expiration of the freedom period,
they could only do so when no petition for certification election was
filed. The reason is, with a pending petition for certification, any
such agreement entered into by management with a labor
organization is fraught with the risk that such a labor union may

471
Page 472

not be chosen thereafter as the collective bargaining


representative. The provision for status quo is conditioned on the
fact that no certification election was filed during the freedom
period. Therefore, following Art. 256, at the expiration of the
freedom period, PRI's obligation to recognize NAMAPRI-SPFL as
the incumbent bargaining agent does not hold true when petitions
for certification election were filed.

472
Page 473

Case Digests by: JAYMEE SAMANTHA O. GONZALEZ

NATIONAL UNION OF WORKERS IN HOTELS,


RESTAURANTS AND ALLIED INDUSTRIES-MANILA
PAVILION HOTEL CHAPTER vs. SECRETARY OF
LABOR AND EMPLOYMENT
G.R. No. 181531. July 31, 2009.
Carpio Morales, J.

DOCTRINE:
ARTICLE 268 - SOLE AND EXCLUSIVE BARAGAINING AGENT
Under Art. 256 (now Art. 268) of the Labor Code, the union obtaining
the majority of the valid votes cast by the eligible voters shall be
certified as the sole and exclusive bargaining agent of all the workers
in the appropriate bargaining unit.

FACTS:
A certification election was conducted among the rank-and-file
employees of respondent Holiday Inn Manila Pavilion Hotel with
NUWHRAIN-MPHC garnering 151 votes and HIMPHLU garnering 169
votes. However, in view of the 22 segregated votes (out of 346 total
votes cast), contending unions, petitioner, NUHWHRAIN-MPHC, and
respondent HIMPHLU, referred the case back to Med-Arbiter
Calabocal to decide which among those votes would be opened and
tallied. The Med-Arbiter ruled for the opening of 17 out of the 22
segregated votes, specially those cast by the 11 dismissed employees
and those cast by the 6 supposedly supervisory employees.

In its appeal to the Secretary of Labor and Employment (SOLE),


petitioner argued that the votes of the probationary employees should
have been opened considering that probationary employee
Gatbonton's vote was tallied and that respondent HIMPHLU, which
garnered 169 votes, should not be immediately certified as the
bargaining agent, as the opening of the 17 segregated ballots would
push the number of valid votes cast to 338, hence, the 169 votes which
HIMPHLU garnered would be one vote short of the majority.

The SOLE held that pursuant to the Omnibus Rules Implementing the
Labor Code on exclusion and inclusion of voters in a certification
election, the probationary employees cannot vote, as at the time the
Med-Arbiter issued on August 9, 2005 the Order granting the petition

for the conduct of the certification election, the 6 probationary


employees were not yet hired. With respect to the votes cast by the 11
dismissed employees, they could be considered since their dismissal

473
Page 474

was still pending appeal. As to the votes case by the 6 alleged


supervisory employees, the SOLE ruled that their votes should be
counted since their promotion took effect months after the issuance of
the August 9, 2005 Order of the Med-Arbiter, hence, they were still
considered as rank-and-file. Finally, respecting Gatbonton's vote, the
SOLE stated that the same could be the basis to include the votes of
the other probationary employees, as the records show that during the
pre-election conferences, there was no disagreement as to his
inclusion in the voters' list, and neither was it timely challenged when
he voted on election day. In conclusion, the SOLE deemed proper the
certification of HIMPHLU as the exclusive bargaining agent.

ISSUE:
1) Whether or not employees on probationary status at the time of the
certification election should be allowed to vote.
2) Whether or not HIMPHLU can be considered as the exclusive
bargaining agent.

RULING:
1) Yes. The inclusion of Gatbonton's vote was proper not because it
was not questioned but because probationary employees have the
right to vote in a certification election. The votes of the six other
probationary employees should thus also have been counted.

As stated in Airtime Specialists, Inc. V. Ferrer-Calleja:


In a certification election, all rank and file employees in the appropriate
bargaining unit, whether probationary or permanent are entitled to vote. This
principle is clearly stated in Art. 255 of the Labor Code which states that the
"labor organization designated or selected by the majority of the employees
in an appropriate bargaining unit shall be the exclusive representative of the
employees in such unit for purposes of collective bargaining". Collective
bargaining covers all aspects of the employment relation and he resultant
CBA negotiated by the certified union binds all employees in the bargaining
unit. Hence, all rank and file employees, probationary or permanent, have
a substantial interest in the selection of the bargaining representative. The
Code makes no distinction as to their employment status as basis for
eligibility in supporting the petition for certification election. The law refers
to "all" the employees in the bargaining unit.

Prescinding from the principle that all employees are, from the first
day of their employment, eligible for membership in a labor
organization, it is evident that the period of reckoning in
determining who shall be included in the list of eligible voters is, in
cases where a timely appeal has been filed from the Order of the

474
Page 475

Med-Arbiter, the date when the Order of the Secretary of Labor and
Employment, whether affirming or denying the appeal, becomes
final and executory.

In the case, records show that the probationary employees,


including Gatbonton, were included in the list of employees in the
bargaining unit submitted by the Hotel on May 25, 2006 in
compliance with the directive of the Med-Arbiter after the appeal
and subsequent motion for reconsideration have been denied by
the SOLE, rendering the Med-Arbiter's August 22, 2005 Order final
and executory 10 days after the March 22, 2007 Resolution
(denying the motion for reconsideration of the January 22 Order
denying the appeal), and rightly so. Because, for purposes of self-
organization, those employees are deemed eligible to vote.

2) No. Prescinding from the Court's ruling that all the probationary
employees' votes should be deemed valid votes while that of the
supervisory employees should be excluded, it follows that the
number of valid votes cast would increase — from 321 to 337.
Under Art. 256 of the Labor Code, the union obtaining the majority
of the valid votes cast by the eligible voters shall be certified as the
sole and exclusive bargaining agent of all the workers in the
appropriate bargaining unit. This majority is 50% + 1. Hence, 50%
of 337 is 168.5 + 1 or at least 170. HIMPHLU obtained 169 while
petitioner received 151 votes. Clearly, HIMPHLU was not able to
obtain a majority vote.

475
Page 476

Case Digests by: JAYMEE SAMANTHA O. GONZALEZ

COCA-COLA BOTTLERS PHILIPPINES, INC.


vs. ILOCOS PROFESSIONAL AND TECHNICAL
EMPLOYEES UNION (IPTEU)
G.R. No. 193798. September 9, 2015.
Peralta, J.

DOCTRINE:
EXCLUSION OF CONFIDENTIAL EMPLOYEES FROM THE
BARGAINING UNIT
Access to vital labor information is the imperative consideration in
determining whether an employee is considered as a confidential
employee. An employee must assist or act in a confidential capacity
and obtain confidential information relating to labor relations policies.
Exposure to internal business operations of the company is not per se
a ground for the exclusion in the bargaining unit.

FACTS:
Respondent Ilocos Professional and Technical Employees Union
(IPTEU) filed a verified Petition for certification election seeking to
represent a bargaining unit consisting of approximately 22 rank-and-
file professional and technical employees of petitioner Coca-Cola
Bottlers Philippines, Inc. (CCBPI) Ilocos Norte Plant. However, CCBPI
prayed for the denial and dismissal of the petition, arguing that the said
22 rank-and-file employees are supervisory and confidential
employees, and hence, ineligible for inclusion as members of IPTEU.
It also sought to cancel and revoke the registration of IPTEU for failure
to comply with the 20% membership requirement based on all the
supposed employees in the bargaining unit it seeks to operate.

Mediator-Arbiter Gacad-Ulep granted IPTEU'S petition. In the pre-


election conference, CCBPI and IPTEU agreed to conduct a
certification election. On election day, only 16 of the 22 employees in
the IPTEU list voted. However, no votes were canvassed. CCBPI filed
and registered a Protest questioning the conduct and mechanics of the
election and a Challenge to Votes on the ground that the voters are
supervisory and confidential employees.

The Mediator-Arbiter, in denying CCBPI's challenge to the 16 votes,


ruled that the voters are rank-and-file employees holding positions
that are not confidential in nature, and who are not, or used to be,

476
Page 477

members of Ilocos Monthlies Union (IMU) due to the reclassification of


their positions by CCBPI and have been excluded from the CBA
entered into by IMU and CCBPI from 1997 to 2005. Consequently, the
challenged votes were opened and canvassed. After garnering 14 out
of the 16 votes cast, IPTEU was proclaimed as the sole and exclusive
bargaining agent of the rank-and file exempt workers in CCBPI Ilocos
Norte Plant.

The SOLE, ruling on the appeal of CCBPI, held that the 22 employees
sought to be represented by IPTEU are not part of IMU and are
excluded from its CBA coverage; that even if the 16 challenged voters
may have access to information which are confidential from the
business standpoint, the exercise of their right to self-organization
could not be defeated because their common functions do not show
that there exist a confidential relationship within the realm of labor
relations.

ISSUE:
Whether or not the 16 voters are confidential employees and are
therefore excluded from the bargaining unit.

RULING:
No. In this case, organizational charts, detailed job descriptions, and
training programs were presented by CCBPI before the Mediator-
Arbiter, the SOLE, and the CA. Despite these, the Mediator-Arbiter
ruled that employees who encounter or handle trade secrets and
financial information are not automatically classified as confidential
employees. It was admitted that the subject employees encounter and
handle financial as well as physical production data and other
information which are considered vital and important from the business
operations' standpoint. Nevertheless, it was opined that such
information is not the kind of information that is relevant to collective
bargaining negotiations and settlement of grievances as would classify
them as confidential employees. The SOLE, which the CA affirmed,
likewise held that the questioned voters do not have access to
confidential labor relations information.

The Court defer to the findings of fact of the Mediator-Arbiter, the


SOLE, and the CA. Certainly, access to vital labor information is the
imperative consideration. An employee must assist or act in a
confidential capacity and obtain confidential information relating to
labor relations policies. Exposure to internal business operations of the
company is not per se a ground for the exclusion in the bargaining unit.

477
Page 478

Case Digests by: JAYMEE SAMANTHA O. GONZALEZ

MIGUELA SANTUYO ET. AL. vs. REMERCO GARMENTS


MANUFACTURING, INC. and/or VICTORIA REYES
G.R. No. 174420. March 22, 2010.
Corona, J.

DOCTRINE:
JURISDICTION OVER CASES INVOLVING THE
IMPLEMENTATION OF COLLECTIVE BARGAINING
AGREEMENTS
Pursuant to Articles 217 in relation to Articles 260 and 261 of the Labor
Code, the labor arbiter should have referred the matter involving the
implementation of the CBAs to the grievance machinery provided in
the CBA. Because the labor arbiter clearly did not have jurisdiction over
the subject matter, his decision was void.

FACTS:
Petitioners, members of Kaisahan ng Manggagawa sa Remerco
Garments Manufacturing Inc. (KMM Kilusan), were employed as
sewers by Remerco Garments Manufacturing, Inc. (RGMI). They
staged an illegal strike and as a consequence thereof, employees who
wanted to sever their employment were paid separation pay while
those who wanted to resume work were recalled on the condition that
they would no longer be paid a daily rate but on a piece-rate basis.

Thereafter, the union filed a notice of strike with the NCMB arguing that
RGMI committed an unfair labor practice when it changed the salary
scheme from a daily rate to piece-rate basis without consulting it.
Consequently, RGMI filed a notice of lockout in the NCMB. During the
conciliation proceedings, the corporation transferred its factory site and
then the union went on strike and blocked the entry to RGMI’s new
premises.

The SOLE assumed jurisdiction pursuant to Art. 263(g) of the Labor


Code and ordered the striking workers to return to work immediately.
The Secretary, after evaluating the arguments of the parties, held that
RGMI did not lock out its employees since it informed them of the
transfer of the worksite. Furthermore, the Secretary ordered all
employees to return to work and RGMI to pay its employees their
unpaid salaries (from September 25, 1995 to October 14, 1995) on the
piece rate basis. No appeal was made from such order.

While the conciliation proceedings between the union and respondent


were pending, petitioners filed a complaint for illegal dismissal against

478
Page 479

RGMI and respondent Victoria Reyes, accusing the latter of


harassment. On the other hand, respondents moved to dismiss the
complaint in view of the pending conciliation proceedings in the NCMB.
The respondents also claimed that the Labor Arbiter had no jurisdiction
over the complaint considering alleged violations of the CBA should be
resolved according to the grievance procedure laid out therein.

ISSUE:
Whether or not the Labor Arbiter has jurisdiction over the dispute.

RULING:
No. Petitioners clearly and consistently questioned the legality of
RGMI's adoption of the new salary scheme (i.e., piece-rate basis),
asserting that such action, among others, violated the existing CBA.
Indeed, the controversy was not a simple case of illegal dismissal but
a labor dispute involving the manner of ascertaining employees'
salaries, a matter which was governed by the existing CBA.

Pursuant to Articles 217 in relation to Articles 260 and 261 of the Labor
Code, the labor arbiter should have referred the matter to the grievance
machinery provided in the CBA. Because the labor arbiter clearly did
not have jurisdiction over the subject matter, his decision was void.

Nonetheless, the Secretary of the Labor assumed jurisdiction over the


labor dispute between the union and RGMI and resolved the same in
his September 18, 1996 order. Article 263 (g) of the Labor Code gives
the Secretary of Labor discretion to assume jurisdiction over a labor
dispute likely to cause a strike or a lockout in an industry indispensable
to the national interest and to decide the controversy or to refer the
same to the NLRC for compulsory arbitration. In doing so, the
Secretary of Labor shall resolve all questions and controversies in
order to settle the dispute.

The Secretary assumed jurisdiction over the controversy because


RGMI had a substantial number of employees and was a major
exporter of garments to the United States and Canada. Since neither
the union nor RGMI appealed the said order, it became final and
executory.

479
Page 480

Case Digests by: JAYMEE SAMANTHA O. GONZALEZ

ALBERT TENG vs. ALFREDO S. PAHAGAC ET. AL.


G.R. No. 169704. November 17, 2010.
Brion, J.

DOCTRINE:
FILING A MOTION FOR RECONSIDERATION IN RELATION TO
THE VA’S DECISION
The Voluntary Arbitrator's decision may still be reconsidered on the
basis of a motion for reconsideration seasonably filed within 10 days
from receipt thereof. The seasonable filing of a motion for
reconsideration is a mandatory requirement to forestall the finality of
such decision.

FACTS:
Respondent workers Pahagac et. al. filed a complaint for illegal
dismissal against Albert Teng Fish Trading, Teng and Chua before the
NCMB. They claimed that in December 2002, Teng informed them that
their services had been terminated after the latter expressed his doubts
on the correct volume of fish caught in every fishing voyage.
Respondents claimed that Teng hired them, without any written
employment contract, to serve as his "eyes and ears" aboard the
fishing boats; to classify the fish caught by bañera; to report to Teng
via radio communication the classes and volume of each catch; to
receive instructions from him as to where and when to unload the
catch; to prepare the list of the provisions requested by the maestro
and the mechanic for his approval; and, to procure the items as
approved by him.

For his defense, Teng stated that as owner of the business he


customarily enters into joint venture agreements with master fishermen
(maestros) who are skilled and are experts in deep sea fishing; they
take charge of the management of each fishing venture, including the
hiring of the members of its complement. Furthermore, the maestros
hired the respondent workers as checkers to determine the volume of
the fish caught in every fishing voyage. Therefore, no employer-
employee relationship existed between him and the respondent
workers.

The VA rendered a decision Teng's favor and declared that no


employer-employee relationship existed between Teng and the

480
Page 481

respondent workers. The respondent workers received the VA's


decision on June 12, 2003. Consequently, they filed a motion for
reconsideration, which was denied in an order dated June 27, 2003
and which they received on July 8, 2003. The VA reasoned out that
Section 6, Rule VII of the 1989 Procedural Guidelines in the Conduct
of Voluntary Arbitration Proceedings (1989 Procedural Guidelines)
does not provide the remedy of a motion for reconsideration to the
party adversely affected by the VA's order or decision.

The case was then elevated to the CA by the respondent workers. The
CA reversed the VA's decision after finding sufficient evidence showing
the existence of employer-employee relationship. In his motion for
reconsideration, Teng argued the VA's decision is not subject to a
motion for reconsideration in the absence of any specific provision
allowing this recourse under Article 262-A of the Labor Code. He
further claimed that when the respondent workers received the VA's
decision on June 12, 2003, they had 10 days, or until June 22, 2003,
to file an appeal. As the respondent workers opted instead to move for
reconsideration, the 10-day period to appeal continued to run; thus, the
VA's decision had already become final and executory.

ISSUE:
1) Whether or not an employer-employee relationship existed
between Teng and the respondent workers.
2) Whether or not Art. 262-A precludes the filing of a motion for
reconsideration of the VA’s decision within the 10-period to appeal.

RULING:
1) Yes. There exists an employer-employee relationship between
Teng and the respondent workers.

While Teng alleged that it was the maestros who hired the
respondent workers, it was his company that issued to the
respondent workers identification cards (IDs) bearing their names
as employees and Teng's signature as the employer. Generally, in
a business establishment, IDs are issued to identify the holder as
a bona fide employee of the issuing entity.

For the 13 years that the respondent workers worked for Teng,
they received wages on a regular basis, in addition to their shares
in the fish caught. The worksheet showed that the respondent
workers received uniform amounts within a given year, which
amounts annually increased until the termination of their

481
Page 482

employment in 2002. Teng's claim that the amounts received by


the respondent workers are mere commissions is incredulous, as
it would mean that the fish caught throughout the year is uniform
and increases in number each year.

More importantly, the element of control — which we have ruled in


a number of cases to be a strong indicator of the existence of an
employer-employee relationship — is present in this case. Teng
not only owned the tools and equipment, he directed how the
respondent workers were to perform their job as checkers; they, in
fact, acted as Teng's eyes and ears in every fishing expedition.

Teng cannot hide behind his argument that the respondent


workerswere hired by the maestros. To consider the respondent
workers as employees of the maestros would mean that Teng
committed impermissible labor-only contracting.

2) No. Article 262-A of the Labor Code does not prohibit the filing of
motion for reconsideration.

As stated in Imperial Textile Mills Inc. v. Sampang, it is true that


the present rule [Art. 262-A] makes the voluntary arbitration award
final and executory after ten calendar days from receipt of the copy
of the award or decision by the parties. Presumably, the decision
may still be reconsidered by the Voluntary Arbitrator on the basis
of a motion for reconsideration duly filed during that period.

It was also likewise ruled in Coca-Cola Bottlers Phil., Inc., Sales


Force Union-PTGWO-Balais v. Coca-Cola Bottlers
Philippines, Inc., VA's decision may still be reconsidered on the
basis of a motion for reconsideration seasonably filed within 10
days from receipt thereof. The seasonable filing of a motion for
reconsideration is a mandatory requirement to forestall the finality
of such decision.

These rulings fully establish that the absence of a categorical


language in Article 262-A does not preclude the filing of a motion
for reconsideration of the VA's decision within the 10-day period.
Teng's allegation that the VA's decision had become final and
executory by the time the respondent workers filed an appeal with
the CA thus fails. We consequently rule that the respondent
workers seasonably filed a motion for reconsideration of the VA's
judgment, and the VA erred in denying the motion because no
motion for reconsideration is allowed.

482
Page 483

Case Digests by: JAYMEE SAMANTHA O. GONZALEZ

ISAMAHAN NG MGA MANGGAGAWA SA HYATT


(SAMASAHNUWHRAIN) vs. HON. VOLUNTARY
ARBITRATOR BUENAVENTURA C. MAGSALIN and
HOTEL ENTERPRISES OF THE PHILIPPINES, INC.
G.R. No. 164939. June 6, 2011.
Villarama, Jr., J.

DOCTRINE:
PETITION FOR REVEIW UNDER RULE 43 AS THE PROPER
REMEDY IN APPEALING THE DECISION OF THE VOLUNTARY
ARBITRATOR
A decision or award of a voluntary arbitrator is appealable to the CA
via petition for review under Rule 43.

FACTS:
Petitioner Angelito Caragdag, a waiter at Hyatt Regency Manila’s Cafe
Al Fresco restaurant and a director of the union Samahan ng mga
Manggagawa sa Hyatt-NUWHRAIN-APL was dismissed from his
employment because of the succession of infractions he committed,
namely, refusing to be frisked by the security personnel; threatening,
intimidating, coercing, and provoking to a fight your superior for
reasons directly connected with his discharge of official; and leaving
his work assignment during official hours without prior permission. He
questioned his dismissal and the dispute was referred to voluntary
arbitration.

The VA held that Caragdag was validly dismissed pursuant to Hyatt's


Code of Discipline, which states that an employee who commits three
different acts of misconduct within a twelve (12)-month period commits
serious misconduct. Petitioner sought reconsideration of the decision
which was denied. Afterwards, petitioner assailed the decision of the
VA before the CA in a petition for certiorari. The CA dismissed the
petition outright for being the wrong remedy. Said court explained;
“Rule 43, Section 5 of the 1997 Rules of Civil Procedure explicitly
provides that the proper mode of appeal from judgments, final orders
or resolution of voluntary arbitrators is through a Petition for Review
which should be filed within fifteen (15) days from the receipt of notice
of judgment, order or resolution of the voluntary arbitrator.”

Petitioner duly filed a motion for reconsideration of the dismissal, but


the motion was denied by the CA. Thus, petitioner filed before the SC
a petition for review on certiorari

483
Page 484

ISSUE:
Whether or not the petition for certiorari is an improper mode of appeal.

RULING:
Yes. The well-settled rule is that a decision or award of a voluntary
arbitrator is appealable to the CA via petition for review under Rule 43.

In the case at bar, upon receipt on May 26, 2003 of the Voluntary
Arbitrator's Resolution denying petitioner's motion for reconsideration,
petitioner should have filed with the CA, within the fifteen (15)-day
reglementary period, a petition for review, not a petition for certiorari.

Petitioner insists on a liberal interpretation of the rules but we find no


cogent reason in this case to deviate from the general rule. Verily, rules
of procedure exist for a noble purpose, and to disregard such rules in
the guise of liberal construction would be to defeat such purpose.
Procedural rules are not to be disdained as mere technicalities. They
may not be ignored to suit the convenience of a party. Adjective law
ensures the effective enforcement of substantive rights through the
orderly and speedy administration of justice. Rules are not intended to
hamper litigants or complicate litigation. But they help provide for a vital
system of justice where suitors may be heard following judicial
procedure and in the correct forum. Public order and our system of
justice are well served by a conscientious observance by the parties of
the procedural rules.

484
Page 485

Case Digests by: JAYMEE SAMANTHA O. GONZALEZ

ROGELIO BARONDA vs. HON. COURT OF APPEALS,


and HIDECO SUGAR MILLING CO., INC.
G.R. No. 161006. October 14, 2015.
Bersamin, J.

DOCTRINE:
THE PROPER REMEDY FROM THE ORDER OF THE VOLUNTARY
ARBITRATOR IS TO APPEAL TO THE COURT OF APPEALS BY
PETITION FOR REVIEW UNDER RULE 43
Although it is true that certiorari cannot be a substitute for a lost appeal,
and that either remedy was not an alternative of the other, we have at
times permitted the resort to certiorari despite the availability of appeal,
or of any plain speedy and adequate remedy in the ordinary course of
law in exceptional situations, such as: (1) when the remedy of certiorari
is necessary to prevent irreparable damages and injury to a party; (2)
where the trial judge capriciously and whimsically exercised his
judgment; (3) where there may be danger of a failure of justice; (4)
where appeal would be slow, inadequate and insufficient; (5) where
the issue raised is one purely of law; (6) where public interest is
involved; and (7) in case of urgency.

FACTS:
Petitioner Rogelio Baronda was employed as a mud press truck driver
by respondent Hideco Sugar Milling Co., Inc. On May 1, 1998, he hit
HIDECO's transmission lines while operating a dump truck, causing a
total factory blackout from 9:00 pm until 2:00 am of the next day. Due
to this incident, petitioner’s employment was terminated by the
corporation. Petitioner then filed in the Office of the Voluntary Arbitrator
of the National Conciliation and Mediation Board a complaint for illegal
dismissal against HIDECO.

The Voluntary Arbitrator rendered a decision finding the petitioner’s


dismissal illegal and ordering his reinstatement. Thereafter, petitioner
filed his manifestation with motion for the issuance of the writ of
execution of the decision and insisting on being entitled to backwages
and other benefits corresponding to the period from January 16, 1999
up to September 28, 2000 based on Article 279 of the Labor Code. The
VA denied the motion on the ground that the decision did not award
any backwages.

485
Page 486

The petitioner filed another motion for execution praying that a writ of
execution requiring HIDECO to pay to him unpaid wages, 13th month
pay and bonuses from January 16, 2001, the date when his
reinstatement was effected, until his actual reinstatement. The VA
granted such motion and cited as basis Art. 223 of the Labor Code.
HIDECO then instituted a special civil action for certiorari in the CA.

Opposing HIDECO’s petition, petitioner countered that the petition for


certiorari should be dismissed considering that HIDECO should have
appealed the decision of the Voluntary Arbitrator under Rule 43 of the
Rules of Court because certiorari was not a substitute for a lost appeal
and that respondent did not file a motion for reconsideration of the
questioned order, which would have been an adequate remedy at law.
The CA treated HIDECO's petition for certiorari as a petition for review
brought under Rule 43, and brushed aside the matters raised by the
petitioner. It observed that the petition for certiorari included the
contents required by Section 6, Rule 43 for the petition for review.

ISSUE:
Whether or not a petition for certiorari is the proper remedy to appeal
the order of the Voluntary Arbitrator.

RULING:
No. HIDECO's proper recourse was to appeal by petition for review;
hence, the CA erred in granting HIDECO's petition for certiorari.

The proper remedy from the order of the Voluntary Arbitrator was to
appeal to the CA by petition for review under Rule 43 of the Rules of
Court, whose Section 1 specifically provides: “Section 1. Scope . —
This Rule shall apply to appeals from judgments or final orders …and
voluntary arbitrators authorized by law.”

The period of appeal was 10 days from receipt of the copy of the order
of July 25, 2001 by the parties. It is true that Section 4 of Rule 43
stipulates that the appeal shall be taken within 15 days from notice of
the award, judgment, final order or resolution, or from the date of its
last publication, if publication is required by law for its effectivity, or of
the denial of the petitioner's motion for new trial or reconsideration duly
filed in accordance with the governing law of the court or agency a quo.
However, Article 262-A of the Labor Code, the relevant portion of which
follows, expressly states that the award or decision of the Voluntary
Arbitrator shall be final and executory after 10 calendar days from
receipt of the copy of the award or decision by the parties.

486
Page 487

In the case at bar, HIDECO filed the petition for certiorari, not a petition
for review under Rule 43, and the CA liberally treated the petition for
certiorari as a petition for review under Rule 43. HIDECO did not
establish that its case came within any of the exceptional situations in
order for the court to permit the resort to certiorari despite the
availability of appeal or of any plain speedy and adequate remedy in
the ordinary course of law.

Secondly, HIDECO filed the petition for certiorari on October 2, 2001.


Even assuming, as the CA held, that the petition for certiorari contained
the matters required by Rule 43, such filing was not timely because 56
days had already lapsed from HIDECO's receipt of the denial by the
Voluntary Arbitrator of the motion for reconsideration. In short,
HIDECO had thereby forfeited its right to appeal.

487
Page 488

Case Digests by: JAYMEE SAMANTHA O. GONZALEZ

NYK-FIL SHIP MANAGEMENT, INCORPORATED vs.


GENER G. DABU
G.R. No. 225142. September 13, 2017.
Peralta, J.

DOCTRINE:
VOLUNTARY ARBITRATOR’S DECISION SHALL BE APPEALED
BEFORE THE COURT OF APPEALS WITHIN 10 DAYS FROM
RECEIPT OF THE AWARD OR DECISION
Despite Rule 43 providing for a 15-day period to appeal, we rule that
the Voluntary Arbitrator's decision must be appealed before the Court
of Appeals within 10 calendar days from receipt of the decision as
provided in Article 262-A of the Labor Code.

FACTS:
Respondent Gener G. Dabu was hired as an oiler for nine months on
board the vessel M/V Hojin by petitioner NYK-Fil Ship Management,
Inc. Before respondent embarked on the vessel, he underwent a pre-
employment medicial examination where he disclosed that he has
diabetes mellitus.

On April 10, 2013, respondent consulted a doctor in Sri Lanka after


experiencing palpitations, pains all over his body, numbness of hands
and legs, lack of sleep and nervousness. The doctor declared him unfit
for sea duty. As a result thereof, respondent was repatriated to Manila
on April 12, 2013; and upon his arrival, he was immediately referred to
the company-designated physician who examined him. Respondent
had undergone many follow up examinations with corresponding
laboratory tests, and was prescribed medicines and eventually placed
on insulin treatment.

The company-designated physician declared that respondent's


diabetes mellitus is not work-related. However, respondent continued
his follow-up consultations as he still complained of body pains and
weakness and was prescribed medicines.The physician reiterated her
findings that respondent’s illness is not work-related. Respondent then
consulted Dr. Vicaldo and Dr. Miguel who both found him permanently
unfit to resume work as a seaman in any capacity and that his illness
is considered work-related.

488
Page 489

Respondent filed a notice to arbitrate with the NCMB after failing to


reach a settlement agreement with the petitioner. The NCMB-PVA
rendered a decision awarding disability compensation to respondent.
Petitioner received a copy of the PVA decision on February 9, 2015
and filed with the CA a petition for review under Rule 43 of the Rules
of Court on February 24, 2015 alleging that the PVA committed serious
errors in rendering its decision and sought to enjoin the PVA from
enforcing its decision.

On April 27, 2015, the NCMB-PVA issued a Writ of Execution directing


the satisfaction of the judgment award of the PVA, which petitioner had
complied without prejudice to the outcome of their petition for review.
The CA then issued a decision which reversed and set aside the ruling
of the NCMB-PVA. However, upon the motion for reconsideration of
respondent wherein he argued that the petition should be dismissed
for being filed out of time, the CA amended its decision.

ISSUE:
Whether or not the petition was filed out of time.

RULING:
Yes. The decision of the voluntary arbitrator becomes final and
executory after 10 days from receipt thereof. The proper remedy to
reverse or modify a voluntary arbitrators' or panel of voluntary
arbitrators' decision is to appeal the award or decision via a petition
under Rule 43 of the 1997 Rules of Civil Procedure. And under Section
4 of Rule 43, the period to appeal to the CA is 15 days from receipt of
the decision. Notwithstanding, since Article 262-A of the Labor Code
expressly provides that the award or decision of the voluntary arbitrator
shall be final and executory after ten (10) calendar days from receipt
of the decision by the parties, the appeal of the VA decision to the CA
must be filed within 10 days.

As provided in Philippine Electric Corporation (PHILEC) v. Court


of Appeals:

Despite Rule 43 providing for a 15-day period to appeal,we rule


that the Voluntary Arbitrator's decision must be appealed before
the Court of Appeals within 10 calendar days from receipt of the
decision as provided in the Labor Code.

Appeal is a "statutory privilege," which may be exercised "only in


the manner and in accordance with the provisions of the law."

489
Page 490

"Perfection of an appeal within the reglementary period is not


only mandatory but also jurisdictional so that failure to do so
rendered the decision final and executory, and deprives the
appellate court of jurisdiction to alter the final judgment much less
to entertain the appeal."

We ruled that Article 262-A of the Labor Code allows the appeal
of decisions rendered by Voluntary Arbitrators. Statute provides
that the Voluntary Arbitrator's decision "shall be final and
executory after ten (10) calendar days from receipt of the copy
of the award or decision by the parties." Being provided in the
statute, this 10-day period must be complied with; otherwise, no
appellate court will have jurisdiction over the appeal.

Furthermore, under Article VIII, Section 5(5) of the Constitution,


this court "shall not diminish, increase, or modify substantive
rights" in promulgating rules of procedure in courts. The 10-day
period to appeal under the Labor Code being a substantive right,
this period cannot be diminished, increased, or modified through
the Rules of Court.

In this case, petitioner received the PVA decision on February 9, 2015,


and filed the petition for review 15 days after receipt thereof, i.e., on
February 24, 2015. The CA, upon respondent's motion for
reconsideration, rendered its Amended Decision dated March 3, 2016
dismissing the petition and vacating the earlier decision it made
granting the petition. The CA dismissed the petition for being filed out
of time, citing the PHILEC case. We find no error committed by the CA
in dismissing the petition for being filed out of time as the petition was
not filed within the 10 day period. Since the timely perfection of an
appeal is jurisdictional, the CA has no more authority to act on the
appeal filed by petitioner. The CA correctly held that inasmuch as the
PVA decision had lapsed into finality, the same may no longer be
modified in any respect.

490
Page 491

Case Digests by: JAYMEE SAMANTHA O. GONZALEZ

GUAGUA NATIONAL COLLEGES vs. COURT OF


APPEALS
G. R. No. 188492. August 28, 2018.
Bersamin, J.

DOCTRINE:
THE PETITION FOR REVIEW SHALL BE FILED WITHIN 15 DAYS
PURSUANT TO SECTION 4, RULE 43 OF THE RULES OF COURT
The 10-day period stated in Article 276 should be understood as the
period within which the party adversely affected by the ruling of the
Voluntary Arbitrators or Panel of Arbitrators may file a motion for
reconsideration. Only after the resolution of the motion for
reconsideration may the aggrieved party appeal to the CA by filing the
petition for review under Rule 43 of the Rules of Court within 15 days
from notice pursuant to Section 4 of Rule 43.

FACTS:
Under R.A. No. 6728, 70% of the increase in tuition fees shall go to the
payment of salaries, wages, allowances and other benefits of the
teaching and non-teaching personnel. Pursuant to this provision, the
petitioner Guagua National Colleges imposed a 7% increase of its
tuition fees for school year 2006-2007. In order to save the depleting
funds of the petitioner's Retirement Plan, its Board of Trustees
approved the funding of the retirement program out of the 70% net
incremental proceeds arising from the tuition fee increases.
Respondents GNC-Faculty Labor Union and GNC Non-Teaching
Maintenance Labor Union challenged the petitioner's unilateral
decision by claiming that the increase violated Section 5(2) of R.A. No.
6728.

Voluntary Arbitrator Bacungan rendered his decision dated June 16,


2008 in favor of GNC, holding that retirement benefits fell within the
category of "other benefits" that could be charged against the 70% net
incremental proceeds. The respondents filed an Urgent Motion for
Extension praying that the CA grant them an extension of 15 days from
July 1, 2008, or until July 16, 2008, within which to file their petition for
review.

On July 2, 2008, the CA issued a resolution granting the Urgent Motion


for Extension. The respondents filed the petition for review on

491
Page 492

July 16, 2008. The petitioner filed its Motion to Dismiss asserting that
the decision of the Voluntary Arbitrator had already become final and
executory pursuant to Article 276 of the Labor Code and in accordance
with the ruling in Coca-Cola Bottlers Philippines, Inc. Sales Force
Union-PTGWO-Balais v. Coca-Cola Bottlers Philippines, Inc. The CA
denied the said motion.

ISSUE:
Whether or not the CA went beyond its jurisdiction when it denied the
Motion to Dismiss despite the finality of the decision of the Arbitrator
pursuant to Art. 276 of the Labor Code.

RULING:
No. The 10-day period stated in Article 276 should be understood as
the period within which the party adversely affected by the ruling of the
Voluntary Arbitrators or Panel of Arbitrators may file a motion for
reconsideration. Only after the resolution of the motion for
reconsideration may the aggrieved party appeal to the CA by filing the
petition for review under Rule 43 of the Rules of Court within 15 days
from notice pursuant to Section 4 of Rule 43.

In the 2010 ruling in Teng v. Pahagac the Court clarified that the 10-
day period set in Article 276 of the Labor Code gave the aggrieved
parties the opportunity to file their motion for reconsideration, which
was more in keeping with the principle of exhaustion of administrative
remedies, holding:

By allowing a 10-day period, the obvious intent of Congress in


amending Article 263 to Article 262-A is to provide an opportunity
for the party adversely affected by the VA's decision to seek
recourse via a motion for reconsideration or a petition for review
under Rule 43 of the Rules of Court filed with the CA. Indeed, a
motion for reconsideration is the more appropriate remedy in line
with the doctrine of exhaustion of administrative remedies. For
this reason, an appeal from administrative agencies to the CA
via Rule 43 of the Rules of Court requires exhaustion of available
remedies as a condition precedent to a petition under that Rule.

The requirement that administrative remedies be exhausted is


based on the doctrine that in providing for a remedy before an
administrative agency, every opportunity must be given to the
agency to resolve the matter and to exhaust all opportunities for
a resolution under the given remedy before bringing an action

in, or resorting to, the courts of justice. Where Congress has not
clearly required exhaustion, sound judicial discretion governs,
guided by congressional intent.

492
Page 493

The Court notes that despite the clarification made in Teng v. Pahagac,
DOLE and the NCMB have not revised or amended the Revised
Procedural Guidelines in the Conduct of Voluntary Arbitration
Proceedings insofar as its Section 7 of Rule VII is concerned. This
inaction has obviously sown confusion, particularly in regard to the
filing of the motion for reconsideration as a condition precedent to the
filing of the petition for review in the CA. Consequently, we need to
direct the DOLE and the NCMB to cause the revision or amendment of
Section 7 of Rule VII of the Revised Procedural Guidelines in the
Conduct of Voluntary Arbitration Proceedings in order to allow the filing
of motions for reconsideration in line with Article 276 of the Labor Code.

Moreover, the CA did not act arbitrarily in denying the petitioner's


Motion to Dismiss. It correctly noted that Coca-Cola Bottlers
Philippines, Inc., Sale Force Union-PTGWO-Balais v. Coca-Cola
Bottlers Philippines, Inc. did not make a definitive ruling on the correct
reglementary period for the filing of the petition for review. Given the
varying applications of the periods define in Article 276 and Section 4
of Rule 43, the CA could not be objectively held to be guilty of grave
abuse of discretion in applying the equitable rule on construction in
favor of labor.

493
Page 494

Case Digests by: GUEVARRA, JOY L.

Octavio v. PLDT
G.R. No. 175492 February 27, 2013
,||

Del Castillo, J.

Every CBA shall provide a grievance machinery to which all


disputes arising from its implementation or interpretation will be
subjected to compulsory negotiations. This essential feature of a CBA
provides the parties with a simple, inexpensive and expedient system
of finding reasonable and acceptable solutions to disputes and helps
in the attainment of a sound and stable industrial peace.

FACTS:
On May 28, 1999, PLDT and Gabay ng Unyon sa
Telekomunikasyon ng mga Superbisor (GUTS) entered into a CBA
covering the period January 1, 1999 to December 31, 2001 (CBA of
1999-2001). Article VI, Section I thereof provides:

Section 1.The COMPANY agrees to grant the following across-


the-board salary increase during the three years covered by this
Agreement to all employees covered by the bargaining unit as of the
given dates:

Effective Jan 1, 1999 —10% of basic or P2,000.00 whichever is higher;


Effective January 1, 2000 — 11% of basic wage or P2,250.00 whichever is higher;
Effective January 1, 2001 — 12% of basic wage or P2,500.00 whichever is higher.

On October 1, 2000, PLDT hired Octavio as Sales System Analyst


I on a probationary status. He became a member of GUTS. When
Octavio was regularized on January 1, 2001, he was receiving a
monthly basic salary of P10,000.00. On February 1, 2002, he was
promoted to the position of Sales System Analyst 2 and his salary was
increased to P13,730.00.

On May 31, 2002, PLDT and GUTS entered into another CBA
covering the period January 1, 2002 to December 31, 2004 (CBA of
2002-2004) which provided for the following salary increases: 8% of
basic wage or P2,000.00 whichever is higher for the first year (2002);
10% of basic wage or P2,700.00 whichever is higher for the second
year (2003); and, 10% of basic wage or P2,400.00 whichever is higher
for the third year (2004).

Claiming that he was not given the salary increases of P2,500.00


effective January 1, 2001 and P2,000.00 effective January 1, 2002,

494
Page 495

Octavio wrote the President of GUTS, Adolfo Fajardo (Fajardo). Acting


thereon and on similar grievances from other GUTS members, Fajardo
wrote the PLDT Human Resource Head to inform management of the
GUTS members' claim for entitlement to the across-the-board salary
increases.

Accordingly, the Grievance Committee convened on October 7,


2002 consisting of representatives from PLDT and GUTS. The
Grievance Committee, however, failed to reach an agreement. In
effect, it denied Octavio's demand for salary increases.

ISSUE:
a. Whether or not the employer and bargaining representative may
amend the provisions of the collective bargaining agreement without
the consent and approval of the employees;
b. If so, whether the said agreement is binding on the employees;

RULING:
Octavio cannot claim that the Committee Resolution is not valid,
binding and conclusive as to him for being a modification of the CBA
in violation of Article 253 of the Labor Code. It bears to stress that the
said resolution is a product of the grievance procedure outlined in the
CBA itself. It was arrived at after the management and the union
through their respective representatives conducted negotiations in
accordance with the CBA. On the other hand, Octavio never assailed
the competence of the grievance committee to take cognizance of his
case. Neither did he question the authority or credibility of the union
representatives; hence, the latter are deemed to have properly
bargained on his behalf since "unions are the agent of its members for
the purpose of securing just and fair wages and good working
conditions." In fine, it cannot be gainsaid that the Committee
Resolution is a modification of the CBA. Rather, it only provides for the
proper implementation of the CBA provision respecting salary
increases.

495
Page 496

Case Digests by: GUEVARRA, JOY L.

EDEN GLADYS ABARIA, et al. v. NLRC


G.R. No. 154113, December 7, 2011
VILLARAMA, JR., J

FACTS:
The consolidated petitions before us involve the legality of mass
termination of hospital employees who participated in strike and
picketing activities.

In a letter addressed to Nava, Ernesto Canen, Jr., Jesusa Gerona,


Hannah Bongcaras, Emma Remocaldo, Catalina Alsado and Albina
Baz, Atty. Alforque suspended their union membership for serious
violation of the Constitution and By-Laws.

Upon the request of Atty. Alforque, MCCHI granted one-day union


leave with pay for 12 union members.The next day, several union
members led by Nava and her group launched a series of mass actions
such as wearing black and red armbands/headbands, marching
around the hospital premises and putting up placards, posters and
streamers.For their continued picketing activities despite the said
warning, more than 100 striking employees were dismissed.Unfazed,
the striking union members held more mass actions. With the volatile
situation adversely affecting hospital operations and the condition of
confined patients, MCCHI filed a petition for injunction. A temporary
restraining order (TRO) was issued.

Thereafter, several complaints for illegal dismissal and unfair labor


practice were filed by the terminated employees against MCCHI, Rev.
Iyoy, UCCP and members of the Board of Trustees of MCCHI.

Executive Labor Arbiter Reynoso A. Belarmino rendered his


decision dismissing the complaints for unfair labor practice.
Complainants appealed to the Commission, which affirmed the Labor
Arbiter.

Complainants elevated the case to the Court of Appeals (CA)


(Cebu Station) via a petition for certiorari. CAs Eighth Division
dismissed the petition on the ground that out of 88 petitioners only 47
have signed the certification against forum shopping.

In G.R. No. 196156, MCCHI/VCMC prayed for the annulment of


Resolution of the CA, for this Court to declare the dismissal of

496
Page 497

respondents Yballe, et al. as valid and legal and to reinstate the


Resolution of the NLRC.

G.R. No. 187861 was consolidated with G.R. Nos. 154113 and
187778 as they involve similar factual circumstances and identical or
related issues. G.R. No. 196156 was later also consolidated with the
aforesaid cases.

ISSUE:
Whether or not respondents are illegally dismissed?

RULING:
Records of the NCMB and DOLE Region 7 confirmed that NAMA-
MCCH-NFL had not registered as a labor organization, having
submitted only its charter certificate as an affiliate or local chapter of
NFL.Not being a legitimate labor organization, NAMA-MCCH-NFL is
not entitled to those rights granted to a legitimate labor organization
under Art. 242.

Aside from the registration requirement, it is only the labor


organization designated or selected by the majority of the employees
in an appropriate collective bargaining unit which is the exclusive
representative of the employees in such unit for the purpose of
collective bargaining, as provided in Art. 255.NAMA-MCCH-NFL is not
the labor organization certified or designated by the majority of the
rank-and-file hospital employees to represent them in the CBA
negotiations but the NFL, as evidenced by CBAs concluded in 1987,
1991 and 1994.

Even assuming that NAMA-MCCH-NFL had validly disaffiliated


from its mother union, NFL, it still did not possess the legal personality
to enter into CBA negotiations. A local union which is not independently
registered cannot, upon disaffiliation from the federation, exercise the
rights and privileges granted by law to legitimate labor organizations;
thus, it cannot file a petition for certification election.Besides, the NFL
as the mother union has the right to investigate members of its local
chapter under the federations Constitution and By-Laws, and if found
guilty to expel such members.MCCHI therefore cannot be faulted for
deferring action on the CBA proposal submitted by NAMA-MCCH-NFL
in view of the union leaderships conflict with the national federation.
We have held that the issue of disaffiliation is an intra-union
disputewhich must be resolved in a different forum in an action at the
instance of either or both the federation and the local union or a rival
labor organization, not the employer.

497
Page 498

The above provision makes a distinction between workers and


union officers who participate in an illegal strike: an ordinary striking
worker cannot be terminated for mere participation in an illegal strike.
There must be proof that he or she committed illegal acts during a
strike. A union officer, on the other hand, may be terminated from work
when he knowingly participates in an illegal strike, and like other
workers, when he commits an illegal act during a strike.

Considering their persistence in holding picketing activities despite


the declaration by the NCMB that their union was not duly registered
as a legitimate labor organization and the letter from NFLs legal
counsel informing that their acts constitute disloyalty to the national
federation, and their filing of the notice of strike and conducting a strike
vote notwithstanding that their union has no legal personality to
negotiate with MCCHI for collective bargaining purposes, there is no
question that NAMA-MCCH-NFL officers knowingly participated in the
illegal strike. The CA therefore did not err in ruling that the termination
of union officers Perla Nava, Catalina Alsado, Albina Baz, Hannah
Bongcaras, Ernesto Canen, Jesusa Gerona and Guillerma Remocaldo
was valid and justified.

Separation pay is made an alternative relief in lieu of reinstatement


in certain circumstances, like: (a) when reinstatement can no longer be
effected in view of the passage of a long period of time or because of
the realities of the situation; (b) reinstatement is inimical to the
employer's interest; (c) reinstatement is no longer feasible; (d)
reinstatement does not serve the best interests of the parties involved;
(e) the employer is prejudiced by the workers continued employment;
(f) facts that make execution unjust or inequitable have supervened; or
(g) strained relations between the employer and employee.

498
Page 499

Case Digests by: GUEVARRA, JOY L.

YSS Employees Union-PTGWO v. YSS Laboratories, Inc.


G.R. No. 155125, December 4, 2009
CHICO-NAZARIO, J

FACTS:
YSS Laboratories is a domestic corporation engaged in the
pharmaceutical business. YSSEU is a duly registered labor
organization and the sole and exclusive bargaining representative of
the rank and file employees of YSS Laboratories.

In order to arrest escalating business losses, YSS Laboratories


implemented a retrenchment program which affected 11 employees
purportedly chosen in accordance with the reasonable standards
established by the company. Of the 11 employees sought to be
retrenched, nine were officers and members of YSSEU. Initially, these
employees were given the option to avail themselves of the early
retirement program of the company. When no one opted to retire early,
YSS Laboratories exercised its option to terminate the services of its
employees as allegedly authorized under Article 283 of the Labor
Code.Thus, copies of the Notices of Termination were filed with DOLE
on 19 March 2001 and were served to concerned employees on 20
March 2001.

Claiming that YSS Laboratories was guilty of discrimination and


union-busting in carrying out the said retrenchment program, YSSEU
decided to hold a strike. After the necessary strike vote was taken
under the supervision of the National Conciliation Mediation Board-
National Capital Region (NCMB-NCR), YSSEU staged a strike on 20
April 2001.

In order to forge a compromise, a number of conciliation


proceedings were conducted by the NCMB-NCR, but these efforts
proved futile since the parties' stance was unbending.

This prompted the Secretary of Labor to finally intervene in order


to put an end to a prolonged labor dispute. Underscoring the
government's policy of preserving economic gains and employment
levels, the Secretary of Labor deemed that the continuation of the labor
dispute was inimical to national interest. Thus, in an Order dated 11
May 2001, the Secretary of Labor certified the labor dispute to the
National Labor Relations Commission (NLRC) for compulsory
arbitration. Accordingly, all striking workers were thereby directed to
return to work within 24 hours from their receipt of the said Order, and

499
Page 500

YSS Laboratories to accept them under the terms and conditions


prevailing before the strike.

YSS Laboratories, however, refused to fully comply with the


directive of the Secretary of Labor. In its Urgent Motion for
Reconsideration, YSS Laboratories argued that nine union officers and
members who were previously terminated from service pursuant to a
valid retrenchment should be excluded from the operation of the return-
to-work order. It also asserted that the union officers who participated
in the purported illegal strike should likewise not be allowed to be back
to their employment for they were deemed to have already lost their
employment status.

YSSEU, for its part, moved that YSS Laboratories be cited for
contempt for refusing to admit the 18 workers back to work. In addition,
YSSEU prayed for the award of backwages in favor of these
employees who were not permitted by YSS Laboratories to return to
their respective stations despite the Secretary of Labor's directive.

ISSUE:
Whether or not the retrenched employees should be excluded
from the operation of the return to work order.

RULING:
The assumption or certification order shall have the effect of
automatically enjoining the intended or impending strike or lockout.
Moreover, if one has already taken place, all striking workers shall
immediately return to work, and the employer shall immediately
resume operations and readmit all workers under the same terms and
conditions prevailing before the strike or lockout.

YSS Laboratories' vigorous insistence on the exclusion of the


retrenched employees from the coverage of the return-to-work order
seriously impairs the authority of the Secretary of Labor to forestall a
labor dispute that he deems inimical to the national economy. The
Secretary of Labor is afforded plenary and broad powers, and is
granted great breadth of discretion to adopt the most reasonable and
expeditious way of writing finis to the labor dispute.

Accordingly, when the Secretary of Labor directed YSS


Laboratories to accept all the striking workers back to work, the
Secretary did not exceed his jurisdiction, or gravely abuse the same. It
is significant at this point to point out that grave abuse of discretion
implies a capricious and whimsical exercise of judgment. Thus, an act
may be considered as committed in grave abuse of discretion when
the same is performed in a capricious or whimsical exercise of

500
Page 501

judgment, which is equivalent to lack of jurisdiction. The abuse of


discretion must be so patent and gross as to amount to an evasion of
a positive duty or to a virtual refusal to perform a duty enjoined by law,
or to act at all in contemplation of law, as where the power is exercised
in an arbitrary and despotic manner by reason of passion or personal
hostility. In the case at bar, there is no showing that the assailed orders
were issued in an arbitrary or despotic manner. The Orders dated 11
May 2001 and 9 June 2001 were issued by the Secretary of Labor, with
the end in view of preserving the status quo ante while the main issues
of the validity of the retrenchment and legality of the strike were being
threshed out in the proper forum. This was done for the promotion of
the common good, considering that a lingering strike could be inimical
to the interest of both employer and employee. The Secretary of Labor
acts to maintain industrial peace. Thus, his certification for compulsory
arbitration is not intended to interfere with the management's rights but
to obtain a speedy settlement of the dispute.

Certainly, the determination of who among the strikers could be


admitted back to work cannot be made to depend upon the discretion
of employer, lest we strip the certification or assumption-of-jurisdiction
orders of the coercive power that is necessary for attaining their
laudable objective. The return-to-work order does not interfere with the
management's prerogative, but merely regulates it when, in the
exercise of such right, national interests will be affected. The rights
granted by the Constitution are not absolute. They are still subject to
control and limitation to ensure that they are not exercised arbitrarily.
The interests of both the employers and employees are intended to be
protected and not one of them is given undue preferrence.

501
Page 502

Case Digests by: GUEVARRA, JOY L.

NUWHRAIN-APL-IUF DUSIT HOTEL NIKKO CHAPTER v. CA


G.R. No. 163942/ G.R. No. 166295
November 11, 2008
VELASCO JR, J.

FACTS:
The Union is the certified bargaining agent of the regular rank-and-
file employees of Dusit Hotel Nikko (Hotel).

On October 24, 2000, the Union submitted its Collective


Bargaining Agreement (CBA) negotiation proposals to the Hotel. As
negotiations ensued, the parties failed to arrive at mutually acceptable
terms and conditions. Due to the bargaining deadlock, the Union, on
December 20, 2001, filed a Notice of Strike on the ground of the
bargaining deadlock with the National Conciliation and Mediation
Board (NCMB)

Thereafter, conciliation hearings were conducted which proved


unsuccessful.

Soon thereafter, in the afternoon of January 17, 2002, the Union


held a general assembly at its office located in the Hotels basement,
where some members sported closely cropped hair or cleanly shaven
heads.

The next day, or on January 18, 2002, more male Union members
came to work sporting the same hair style. The Hotel prevented these
workers from entering the premises claiming that they violated the
Hotels Grooming Standards.

In view of the Hotels action, the Union staged a picket outside the
Hotel premises. Later, other workers were also prevented from
entering the Hotel causing them to join the picket. For this reason the
Hotel experienced a severe lack of manpower which forced them to
temporarily cease operations in three restaurants.

Consequently, the Hotel issued notices to Union members,


preventively suspending them and charging them with the following
offenses: (1) violation of the duty to bargain in good faith; (2) illegal
picket; (3) unfair labor practice; (4) violation of the Hotels Grooming
Standards; (5) illegal strike; and (6) commission of illegal acts during
the illegal strike.

502
Page 503

The next day, the Union filed with the NCMB a second Notice of
Strike on the ground of unfair labor practice and violation of Article
248(a) of the Labor Code on illegal lockout

The Hotel terminated the services of 29 Union officers and 61


members; and suspended 81 employees for 30 days, forty-eight 48
employees for 15 days, 4) employees for 10 days, and 3 employees
for five days. On the same day, the Union declared a strike. Starting
that day, the Union engaged in picketing the premises of the Hotel.
During the picket, the Union officials and members unlawfully blocked
the ingress and egress of the Hotel premises.

The Union filed its third Notice of Strike with the NCMB, this time
on the ground of unfair labor practice and union-busting.

NLRC Decision: It ordered the Hotel and the Union to execute a


CBA within 30 days from the receipt of the decision. NLRC ruled that
the strike conducted was illegal.

CA affirmed NLRC’s decision

ISSUE:
Whether or not Union is guilty of illegal strike

RULING:
YES. First, the Unions violation of the Hotels Grooming Standards
was clearly a deliberate and concerted action to undermine the
authority of and to embarrass the Hotel and was, therefore, not a
protected action. The appearances of the Hotel employees directly
reflect the character and well-being of the Hotel, being a five-star hotel
that provides service to top-notch clients. It can be gleaned from the
records before us that the Union officers and members deliberately and
in apparent concert shaved their heads or cropped their hair. This was
shown by the fact that after coming to work on January 18, 2002, some
Union members even had their heads shaved or their hair cropped at
the Union office in the Hotels basement. Clearly, the decision to violate
the company rule on grooming was designed and calculated to place
the Hotel management on its heels and to force it to agree to the
Unions proposals. This Court is of the opinion, therefore, that the act
of the Union was not merely an expression of their grievance or
displeasure but, indeed, a calibrated and calculated act designed to
inflict serious damage to the Hotels finances or its reputation. Thus, we
hold that the Unions concerted violation of the Hotels Grooming
Standards which resulted in the temporary cessation and disruption of
the Hotels operations is an unprotected act and should be considered
as an illegal strike.

503
Page 504

Second, the Unions concerted action which disrupted the Hotels


operations clearly violated the CBAs No Strike, No Lockout provision.
The facts are clear that the strike arose out of a bargaining deadlock
in the CBA negotiations with the Hotel. The concerted action is an
economic strike upon which the afore-quoted no strike/work stoppage
and lockout prohibition is squarely applicable and legally binding.

Third, the Union officers and members concerted action to shave


their heads and crop their hair not only violated the Hotels Grooming
Standards but also violated the Unions duty and responsibility to
bargain in good faith. By shaving their heads and cropping their hair,
the Union officers and members violated then Section 6, Rule XIII of
the Implementing Rules of Book V of the Labor Code. This rule
prohibits the commission of any act which will disrupt or impede the
early settlement of the labor disputes that are under conciliation. Since
the bargaining deadlock is being conciliated by the NCMB, the Unions
action to have their officers and members heads shaved was
manifestly calculated to antagonize and embarrass the Hotel
management and in doing so effectively disrupted the operations of the
Hotel and violated their duty to bargain collectively in good faith.

Fourth, the Union failed to observe the mandatory 30-day cooling-


off period and the seven-day strike ban before it conducted the strike
on January 18, 2002. The NLRC correctly held that the Union failed to
observe the mandatory periods before conducting or holding a strike.
Records reveal that the Union filed its Notice of Strike on the ground of
bargaining deadlock on December 20, 2001. The 30-day cooling-off
period should have been until January 19, 2002. On top of that, the
strike vote was held on January 14, 2002 and was submitted to the
NCMB only on January 18, 2002; therefore, the 7-day strike ban should
have prevented them from holding a strike until January 25, 2002. The
concerted action committed by the Union on January 18, 2002 which
resulted in the disruption of the Hotels operations clearly violated the
above-stated mandatory periods.

Last, the Union committed illegal acts in the conduct of its strike.
The NLRC ruled that the strike was illegal since, as shown by the
pictures presented by the Hotel, the Union officers and members
formed human barricades and obstructed the driveway of the Hotel.
There is no merit in the Unions argument that it was not its members
but the Hotels security guards and the police officers who blocked the
driveway, as it can be seen that the guards and/or police officers were
just trying to secure the entrance to the Hotel. The pictures clearly
demonstrate the tense and highly explosive situation brought about by
the strikers presence in the Hotels driveway.

504
Page 505

Case Digests by: GUEVARRA, JOY L.

Jackbilt Industries Inc., v. Jackbilt Employees Workers Union-NAFLU-KMU


GR No. 171618-19, March 20, 2009
Corona, J.

FACTS:
Due to the adverse effects of the Asian economic crisis on the
construction industry beginning 1997, petitioner Jackbilt Industries,
Inc. decided to temporarily stop its business of producing concrete
hollow blocks, compelling most of its employees to go on leave for six
months.

Respondent Jackbilt Employees Workers Union-NAFLU-KMU


immediately protested the temporary shutdown. Because its collective
bargaining agreement with petitioner was expiring during the period of
the shutdown, respondent claimed that petitioner halted production to
avoid its duty to bargain collectively. The shutdown was allegedly
motivated by anti-union sentiments.

Accordingly, on March 9, 1998, respondent went on strike. Its


officers and members picketed petitioner's main gates and deliberately
prevented persons and vehicles from going into and out of the
compound.

On March 19, 1998, petitioner filed a petition for injunction with a


prayer for the issuance of a temporary restraining order (TRO) in the
National Labor Relations Commission (NLRC). It sought to enjoin
respondent from obstructing free entry to and exit from its production
facility

On April 14, 1998, the NLRC issued a TRO directing the


respondents to refrain from preventing access to petitioner's property.

The reports of both the implementing officer and the investigating


labor arbiter revealed, however, that respondent union violated the
April 14, 1998 order. Union members, on various occasions, stopped
and inspected private vehicles entering and exiting petitioner's
production facility. Thus, in a decision dated July 17, 1998, the NLRC
ordered the issuance of a writ of preliminary injunction

Meanwhile, petitioner sent individual memoranda to the officers


and members of respondent who participated in the strike... ordering
them to explain why they should not be dismissed for committing illegal
acts in the course of a strike

505
Page 506

However, respondent repeatedly ignored petitioner's memoranda


despite the extensions granted

Thus, on May 30, 1998, petitioner dismissed the concerned


officers and members and barred them from entering its premises
effective June 1, 1998.

Aggrieved, respondent filed complaints for illegal lockout, runaway


shop and damages, unfair labor practice, illegal dismissal and
attorney's fees, and refusal to bargain on behalf of its officers and
members against petitioner and its corporate officers. It argued that
there was no basis for the temporary partial shutdown as it was
undertaken by petitioner to avoid its duty to bargain collectively.

Petitioner, on the other hand, asserted that because respondent


conducted a strike without observing the procedural requirements
provided in Article 263 of the Labor Code, the March 9, 1998 strike was
illegal. Furthermore, in view of the July 17,1998 decision of the NLRC
(which found that respondent obstructed the free ingress to and egress
from petitioner's premises), petitioner validly dismissed respondent's
officers and employees for committing illegal acts in the course of a
strike.

In a decision dated October 15, 1999, the labor arbiter dismissed


the complaints for illegal lockout and unfair labor practice for lack of
merit. However, because petitioner did not file a petition to declare the
strike illegal before terminating respondent's officers and employees,
it was found guilty of illegal dismissal.

Petitioner asserts that the filing of a petition to declare the strike


illegal was unnecessary since the NLRC, in its July 17, 1998 decision,
had already found that respondent committed illegal acts in the course
of the strike.

ISSUE:
Whether or not the filing of a petition with the labor arbiter to
declare a strike illegal is a condition sine qua non for the valid
termination of employees who commit an illegal act in the course of
such strike.

RULING:
We grant the petition. The principle of conclusiveness of judgment,
embodied in Section 47(c), Rule 39 of the Rules of Court, holds that
the parties to a case are bound by the findings in a previous judgment
with respect to matters actually raised and adjudged therein.

506
Page 507

Article 264(e) of the Labor Code prohibits any person engaged in


picketing from obstructing the free ingress to and egress from the
employer's premises. Since respondent was found in the July 17, 1998
decision of the NLRC to have prevented the free entry into and exit of
vehicles from petitioner's compound, respondent's officers and
employees clearly committed illegal acts in the course of the March 9,
1998 strike.

The use of unlawful means in the course of a strike renders such


strike illegal. Therefore, pursuant to the principle of conclusiveness of
judgment, the March 9, 1998 strike was ipso facto illegal. The filing of
a petition to declare the strike illegal was thus unnecessary.

Consequently, we uphold the legality of the dismissal of


respondent's officers and employees. Article 264 of the Labor Code
further provides that an employer may terminate employees found to
have committed illegal acts in the course of a strike. Petitioner clearly
had the legal right to terminate respondent's officers and employees.

507
Page 508

Case Digests by: GUEVARRA, JOY L.

AIRLINE PILOTS ASSOCIATION OF THE PHILIPPINES, Petitioner, v.


PHILIPPINE AIRLINES, INC
G.R. No. 168382, June 6, 2011
Del Castillo, J.

FACTS:
The present controversy stemmed from a labor dispute between
respondent Philippine Airlines, Inc. (PAL) and ALPAP, the legitimate
labor organization and exclusive bargaining agent of all commercial
pilots of PAL. Claiming that PAL committed unfair labor practice,
ALPAP filed on December 9, 1997, a notice of strike against
respondent PAL with the DOLE, docketed as NCMB NCR NS 12-514-
97. Upon PALs petition and considering that its continued operation is
impressed with public interest, the DOLE Secretary assumed
jurisdiction over the labor dispute.

Accordingly, all strikes and lockouts at the Philippine Airlines, Inc.,


whether actual or impending, are hereby strictly prohibited. The parties
are also enjoined from committing any act that may exacerbate the
situation.

In a subsequent Order dated May 25, 1998, the DOLE Secretary


reiterated the prohibition contained in the December 23, 1997 Order.
Despite such reminder to the parties, however, ALPAP went on strike
on June 5, 1998. This constrained the DOLE, through then Secretary
Cresenciano B. Trajano, to issue a return-to-work order on June 7,
1998. However, it was only on June 26, 1998 when ALPAP officers
and members reported back to work as shown in a logbook signed by
each of them. As a consequence, PAL refused to accept the returning
pilots for their failure to comply immediately with the return-to-work
order.

On June 29, 1998, ALPAP filed with the Labor Arbiter a complaint
for illegal lockout against PAL, docketed as NLRC NCR Case No. 00-
06-05253-98. ALPAP contended that its counsel received a copy of the
return-to-work order only on June 25, 1998, which justified their non-
compliance therewith until June 26, 1998. It thus prayed that PAL be
ordered to accept unconditionally all officers and members of ALPAP
without any loss of pay and seniority and to pay whatever salaries and
benefits due them pursuant to existing contracts of employment.

On PALs motion, the Labor Arbiter consolidated the illegal lockout


case with NCMB NCR NS 12-514-97 (strike case) pending before the
DOLE Secretary since the controversy presented in the lockout case

508
Page 509

is an offshoot of the labor dispute over which the DOLE Secretary has
assumed jurisdiction and because the factual allegations in both cases
are interrelated. In a Resolution dated January 18, 1999, the NLRC
sustained the consolidation of the illegal lockout case with the strike
case, opining that the DOLE Secretary has the authority to resolve all
incidents attendant to his return-to-work order.

Through then DOLE Secretary Bienvenido E. Laguesma, a


Resolution dated June 1, 1999 was rendered in NCMB NCR NS 12-
514-97, declaring the strike conducted by ALPAP on June 5, 1998
illegal and pronouncing the loss of employment status of its officers
and members who participated in the strike in defiance of the June 7,
1998 return-to-work order and that DECLARES the strike conducted
by ALPAP on June 5, 1998 and thereafter as illegal for being
procedurally infirm and in open defiance of the return-to-work order of
June 7, 1998 and, consequently, the strikers are deemed to have lost
their employment status; and DISMISSES the complaint for illegal
lockout for lack of merit.

In a Resolution dated July 23, 1999, ALPAPs motion for


reconsideration was denied. Thus, ALPAP filed a Petition for Certiorari
with the CA assailing both the June 1, 1999 and July 23, 1999 DOLE
Resolutions. The case was docketed as CA-G.R. SP No. 54880.

Meanwhile, several ALPAP members filed separate individual


complaints for illegal dismissal and non-payment of monetary benefits
against PAL with the Labor Arbiters of the NLRC, questioning their
termination as a result of the strike staged by other ALPAP members
on June 5, 1998.

On January 13, 2003, ALPAP filed before the Office of the DOLE
Secretary a Motion in NCMB NCR NS 12-514-97, requesting the said
office to conduct an appropriate legal proceeding to determine who
among its officers and members should be reinstated or deemed to
have lost their employment with PAL for their actual participation in the
strike conducted in June 1998.

By reason of the final ruling of the Honorable Supreme Court, the


erring pilots have lost their employment status and second, because
these pilots have filed cases to contest such loss before another forum,
the Motion and Supplemental Motion of ALPAP as well as the
arguments raised therein are merely noted by this Office.

ALPAP filed its motion for reconsideration arguing that the issues
raised in its motions have remained unresolved hence, it is the duty of

509
Page 510

DOLE to resolve the same it having assumed jurisdiction over the labor
dispute.

The CA, in its Decision dated December 22, 2004, dismissed the
petition. It found no grave abuse of discretion on the part of Sto. Tomas
and Imson in refusing to conduct the necessary proceedings to
determine issues relating to ALPAP members employment status and
entitlement to employment benefits.

ALPAP moved for reconsideration which was denied for lack of


merit in CA Resolution dated May 30, 2005. Hence, this petition.

ISSUE:
Whether the termination of ALPAP is valid.

RULING:
The records reveals that in NCMB NCR NS 12-514-97, the DOLE
Secretary declared the ALPAP officers and members to have lost their
employment status based on either of two grounds, viz: their
participation in the illegal strike on June 5, 1998 or their defiance of the
return-to-work order of the DOLE Secretary. The records of the case
unveil the names of each of these returning pilots. The logbook with
the heading "Return To Work Compliance/ Returnees" bears their
individual signature signifying their conformity that they were among
those workers who returned to work only on June 26, 1998 or after the
deadline imposed by DOLE. From this crucial and vital piece of
evidence, it is apparent that each of these pilots is bound by the
judgment. Besides, the complaint for illegal lockout was filed on behalf
of all these returnees. Thus, a finding that there was no illegal lockout
would be enforceable against them. In fine, only those returning pilots,
irrespective of whether they comprise the entire membership of
ALPAP, are bound by the June 1, 1999 DOLE Resolution.

510
Page 511

Case Digests by: GUEVARRA, JOY L.

Olisa et. al. v. Escario Et. al., VCMC v. Yballe


GR No. 196156, 2014-01-15
VILLARAMA JR., J.

FACTS:
Respondents were hired as staff nurses (Ong and Angel) and
midwives (Yballe and Cortez) by petitioner Visayas Community
Medical Center (VCMC), formerly the Metro Cebu Community
Hospital, Inc. (MCCHI). MCCHI is a non-stock, non-profit corporation
hich operates the Metro Cebu Community Hospital (MCCH), a tertiary
medical institution owned by the United Church of Christ in the
Philippines (UCCP).

The National Federation of Labor (NFL) is the exclusive bargaining


representative of the rank-and-file employees of MCCHI. Under the
1987 and 1991 Collective Bargaining Agreements (CBAs).

On December 6, 1995, Nava wrote Rev. Iyoy expressing the union


desire to renew the CBA, attaching to her letter a statement of
proposals signed/endorsed by 153 union members. Nava
subsequently requested that the following employees be allowed to
avail of one-day union leave with pay on December 19, 1995.
However, MCCHI returned the CBA proposal for Nava to secure first
the endorsement of the legal counsel of NFL as the official bargaining
representative of MCCHI employees.

Meanwhile, Atty. Alforque informed MCCHI that the proposed CBA


submitted by Nava was never referred to NFL and that NFL has not
authorized any other legal counsel or any person for collective
bargaining negotiations.

On February 26, 1996, upon the request of Atty. Alforque, MCCHI


granted one-day union leave with pay for 12 union members. The next
day, several union members led by Nava and her group launched a
series of mass actions such as wearing black and red
armbands/headbands, marching around the hospital premises and
putting up placards, posters and streamers. Atty. Alforque immediately
disowned the concerted activities being carried out by union members
which are not sanctioned by NFL.MCCHI directed the union officers
led by Nava to submit within 48 hours a written explanation why they
should not be terminated for having engaged in illegal concerted
activities amounting to strike, and place them under immediate
preventive suspension.

511
Page 512

Responding to this directive, Nava and her group denied there was
a temporary stoppage of work, explaining that employees wore their
armbands only as a sign of protest and reiterating their demand for
MCCHI to comply with its duty to bargain collectively.

On March 13 and 19, 1996, the Department of Labor and


Employment (DOLE) Regional Office No. 7 issued certifications stating
that there is nothing in their records which shows that NAMA-MCCH-
NFL is a registered labor organization, and that said union submitted
only a copy of its Charter Certificate on January 31, 1995.MCCHI then
sent individual notices to all union members asking them to submit
within 72 hours a written explanation why they should not be
terminated for having supported the illegal concerted activities of
NAMA-MCCH-NFL which has no legal personality as per DOLE
records

On March 13, 1996, NAMA-MCCH-NFL filed a Notice of Strike but


the same was deemed not filed for want of legal personality on the part
of the filer.

Meanwhile, the scheduled investigations did not push through


because the striking union members insisted on attending the same
only as a group. MCCHI again sent notices informing them that their
refusal to submit to investigation is deemed a waiver of their right to
explain their side and management shall proceed to impose proper
disciplinary action under the circumstances.

Unfazed, the striking union members held more mass actions. The
means of ingress to and egress from the hospital were blocked so that
vehicles carrying patients and employees were barred from entering
the premises. Placards were placed at the hospital entrance gate
stating: lease proceed to another hospital and e are on protest.
Employees and patients reported acts of intimidation and harassment
perpetrated by union leaders and members. With the intensified
atmosphere of violence and animosity within the hospital premises as
a result of continued protest activities by union members, MCCHI
suffered heavy losses due to low patient admission rates.

The hospital suppliers also refused to make further deliveries on


credit. With the volatile situation adversely affecting hospital operations
and the condition of confined patients, MCCHI filed a petition for
injunction in the NLRC on July 9, 1996.A temporary restraining order
(TRO) was issued on July 16, 1996. MCCHI presented 12 witnesses
(hospital employees and patients), including a security guard who was
stabbed by an identified sympathizer while in the company of Nava
group. MCCHI petition was granted and a permanent injunction was

512
Page 513

issued on September 18, 1996 enjoining the Nava group from


committing illegal acts mentioned in Art. 264 of the Labor Code.

Thereafter, several complaints for illegal dismissal and unfair labor


practice were filed by the terminated employees against MCCHI. Rev.
Iyoy, UCCP and members of the Board of Trustees of MCCHI.

Executive Labor Arbiter Reynoso A. Belarmino rendered in his


decision dismissing the claim of unfair labor practice and illegal
dismissal and declaring the termination of the following as an offshoot
of the illegal strike.

NLRC dismissed the motion for reconsideration filed by the


respondents. CA reversed the rulings of the Labor Artbiter and NLRC,
ordered the reinstatement of respondents and the payment of their full
back wages.

ISSUE:
Whether or not respondents did not commit illegal acts during
strike?

RULING:
The strike held by respondents were illegal. Paragraph 3, Article
264(a) of the Labor Code provides that any union officer who knowingly
participates in an illegal strike and any worker or union officer who
knowingly participates in the commission of illegal acts during a strike
may be declared to have lost his employment status.

We stress that the law makes a distinction between union


members and union officers. A worker merely participating in an illegal
strike may not be terminated from employment. It is only when he
commits illegal acts during a strike that he may be declared to have
lost employment status. In contrast, a union officer may be terminated
from employment for knowingly participating in an illegal strike or
participates in the commission of illegal acts during a strike. The law
grants the employer the option of declaring a union officer who
participated in an illegal strike as having lost his employment. It
possesses the right and prerogative to terminate the union officers
from service.

In this case, the NLRC affirmed the finding of the Labor Arbiter that
respondents supported and took part in the illegal strike and further
declared that they were guilty of insubordination. It noted that the
striking employees were determined to force management to negotiate
with their union and proceeded with the strike despite knowledge that
NAMA-MCCH-NFL is not a legitimate labor organization and without

513
Page 514

regard to the consequences of their acts consisting of displaying


placards and marching noisily inside the hospital premises, and
blocking the entry of vehicles and persons

Are respondents then entitled to back wages? In G & S Transport


Corporation v. Infante, ruled in the negative: with respect to
backwages, the principle of a air day wage for a fair day labor remains
as the basic factor in determining the award thereof. If there is no work
performed by the employee there can be no wage or pay unless, of
course, the laborer was able, willing and ready to work but was illegally
locked out, suspended or dismissed or otherwise illegally prevented
from working. In Philippine Marine Officers Guild v. Compaia Aritima,
as affirmed in Philippine Diamond Hotel and Resort v. Manila Diamond
Hotel Employees Union, the Court stressed that for this exception to
apply, it is required that the strike be legal, a situation that does not
obtain in the case at bar.

In fine, we sustain the CA in ruling that respondents who are mere


union members were illegally dismissed for participating in the illegal
strike conducted by the Nava group. However, we set aside the order
for their reinstatement and payment of full back wages.

514
Page 515

Case Digests by: GUEVARRA, JOY L.

Tabangao Shell Refinery Employees Association v. Pilipinas Shell


G.R. No. 170007, April 7, 2014
Leonardo-De Castro, J.

FACTS:
As the current CBA between the union and corporation is
coming to an end, the parties started negotiations for a new CBA. The
union proposed a 20% annual across-the-board salary increase for the
next 3 years however, the company proposed a lump sum of
P80,000 yearly for the 3-year period to all covered employees. In
reply to the union’s request to provide in full details the basis for its
counter-proposal, the company explained that it is based on the
affordability of the corporation and the current salary levels in the
industry but the union rejected it.

After another negotiation, the company increased its offer to


P88,000 but again the union requested for justification but the
company refused to give in insisting that the financial measures were
available in the refinery scorecard in the website and shared
network drives. The union charged that the company was bargaining
in bad faith while the company expressed its disagreement with the
union’s manifestation and proposed the declaration of a deadlock and
recommended a third party’s assistance.

The union filed a Notice of Strike in the NCMB alleging bad faith
bargaining on the part of the company. In the mandatory conciliation-
mediation proceedings, the parties failed to reach an amicable
settlement. When the company learned of the union’s unanimous vote
to hold a strike, it filed a Petition for Assumption of Jurisdiction with the
Secretary of Labor & Employment under Art. 263(g) of the Labor Code.
The DOLE, finding that the strike would have a negative impact on
national interest assumed jurisdiction over the dispute and directed
the parties to submit their respective position paper and maintain the
status quo existing at the time of the Order or if the strike had
commenced, the workers were directed to return to work and the
employer to readmit all workers under the same terms and conditions
prevailing before the strike.

The union filed a petition before the CA questioning the


jurisdiction of the DOLE as the issue raised was unfair labor practice
of the company in the form of bad faith bargaining and not the CBA
deadlock citing item 8 of the CBA rules that deadlock can only
be declared upon mutual consent of both parties but where the union
did not consent. The CA dismissed the petition for lack of merit

515
Page 516

declaring that the DOLE acquired jurisdiction over the dispute


vested to it by Art. 263(g)of the Labor Code.

Meanwhile, the union filed a complaint against the company in the


NLRC on allegations of the company’s refusal to bargain. The NLRC
finding that the case arose from the same CBA-related labor dispute,
transmitted the case to the DOLE. The DOLE, holding that there was
already deadlock and there was no showing that the company
engaged in unfair labor practice by bargaining in bad faith, decided
the matter of the wage increase and other economic issues of the
new CBA, i.e. to give a lump sum package of P95,000 per year for
three years per covered employee & retention of benefits covered
by the preceding CBA. Both parties did not appeal the DOLE
decision thus, it attained finality. The union then went to the Court via
a petition for review under Rule 45 of the Rules of Court.

ISSUE:
Whether or not respondent corporation is guilty of bargaining in
bad faith?

RULING:
NO. The Court held that duty to bargain does not compel any
party to accept a proposal or to make any concession (Art. 252, Labor
Code). While the purpose of collective bargaining is the reaching of an
agreement between the employer and the employee’s union resulting
in a binding contract between the parties, the failure to reach an
agreement after negotiations continued for a reasonable period does
not mean lack of good faith. A CBA, like any contract is a product of
mutual consent and not of compulsion and the duty to bargain does
not include the obligation to reach an agreement.

Respondent’s unswerving position on the matter of annual lump


sum payment in lieu of wage increase did not, by itself, constitute bad
faith even if such position caused a stalemate in the negotiations. As
there was no bad faith on the part of the company in its bargaining with
the union, deadlock was possible and did occur. Fact is, that the
negotiations between the union and the company were stalled by the
opposing offers of yearly wage increase by the union, on the one hand,
and annual lump sum payment by the company, on the other hand.
Each party found the other’s offer unacceptable and neither party was
willing to yield. The company suggested seeking the assistance of a
third party to settle the issue but the union preferred the remedy of filing
a notice of strike. The absence of the parties’ mutual declaration of
deadlock does not mean that there was no deadlock. This is the
essence of Article263(g) of the Labor Code which gives the Secretary
of Labor & Employment jurisdiction over a labor dispute causing or
likely to cause a strike or lockout in an industry indispensable to the

516
Page 517

national interest. The Court denied the petition declaring that the
Secretary of Labor and Employment committed no abuse of discretion
when she assumed jurisdiction over the labor dispute of the union and
the company.

517
Page 518

Case Digests by: GUEVARRA, JOY L.

ASIA BREWERY, INC., VS. TPMA


G.R. Nos. 171594-96, September 18, 2013
Del Castillo, J.

FACTS:
After 18 sessions of negotiations, it still resulted to a deadlock.
After the union filed a notice of strike with the NCMB, still they did not
come to terms.

When union conducted a strike vote with a 768/840 voted in favor


of holding a strike, the corporation then petitioned the Secretary of
DOLE to assume jurisdiction over the parties’ labor dispute, invoking
Article 263 (g) of the Labor Code. Union opposed the assumption of
jurisdiction, reasoning therein that the business of petitioner
corporation is not in dispensable to the national interest. The public
respondent, through Undersecretary/Acting Secretary issued an order
assuming jurisdiction over the labor dispute between the Respondent
union and petitioner corporation. It then resolved the deadlock and
granted arbitral awards.

Upon appeal to the CA, it ordered that the assailed Decision of the
respondent Secretary with respect to the issue on salary increases is
remanded to her office for a definite resolution using as basis the
externally audited financial statements to be submitted by corporation.

ISSUE:
Whether or not the order of the CA remanding the issue on
salary increase to the Secretary of Labor is proper.

RULING:
Yes. The remand of this case to the Secretary of Labor as to the
issue of wage increase was proper. The Supreme Court has
recognized the Secretary of Labor’s distinct expertise in the study and
settlement of labor disputes falling under his power of compulsory
arbitration. It is also well-settled that factual findings of labor
administrative officials, if supported by substantial evidence, are
entitled not only to great respect but even to finality. But at the same
time, the Court also recognize the possibility that abuse of discretion
may attend the exercise of the Secretary’s arbitral functions; his
findings in an arbitration case are usually based on position papers and
their supporting documents (as they are in the present case), and not
on the thorough examination of the parties’ contending claims that may
be present in a court trial and in the face-to-face adversarial process

518
Page 519

that better insures the proper presentation and appreciation of


evidence, etc.

The SC held that the Secretary of Labor gravely abused her


discretion when she relied on the unaudited financial statements of the
corporation in determining the wage award because such evidence is
self-serving and inadmissible. Not only did this violate the Order of the
Secretary of Labor herself to petitioner corporation to submit its
complete audited financial statements, but this may have resulted to a
wage award that is based on an inaccurate and biased picture of
petitioner corporation’s capacity to pay — one of the more significant
factors in making a wage award. The appellate court, thus, correctly
remanded this case to the Secretary of Labor for the proper
determination of the wage award which should utilize, among others,
the audited financial statements of petitioner corporation.

519
Page 520

Case Digests by: GUEVARRA, JOY L.

Escario Et. al. v NLRC


GR No. 160302, 2010-09-27
Del Castillo, J.

FACTS:
The petitioners were among the regular employees of respondent
Pinakamasarap Corporation (PINA), a corporation engaged in
manufacturing and selling food seasoning. They were members of
petitioner Union.

At 8:30 in the morning of March 13, 1993, all the officers and some
200 members of the Union walked out of PINA’s premises and
proceeded to the barangay office to show support for Juanito Cañete,
an officer of the Union charged with oral defamation by Aurora Manor,
PINA’s personnel manager, and Yolanda Fabella, Manor’s secretary.
It appears that the proceedings in the barangay resulted in a
settlement, and the officers and members of the Union all returned to
work thereafter.

As a result of the walkout, PINA preventively suspended all


officers of the Union because of the March 13, 1993 incident. PINA
terminated the officers of the Union after a month.

PINA filed a complaint for ULP and damages. Labor Arbiter Raul
Aquino ruled that the March 13, 1993 incident was an illegal walkout
constituting ULP; and that all the Union’s officers, except Cañete, had
thereby lost their employment.

The Union filed a notice of strike, claiming that PINA was guilty of
union busting through the constructive dismissal of its officers. The
Union held a strike vote, at which a majority of 190 members of the
Union voted to strike. The strike was held in the afternoon of June 15,
1993.

PINA retaliated by charging the petitioners with ULP and


abandonment of work, stating that they had violated provisions on
strike of the CBA.

The Third Division of the NLRC issued a TRO, enjoining the


Union’s officers and members to cease and desist from barricading
and obstructing the entrance to and exit from PINA’s premises, to
refrain from committing any and all forms of violence, and to remove
all forms of obstructions such as streamers, placards, or human
barricade.

520
Page 521

On appeal, the NLRC sustained the finding that the strike was
illegal, but reversed the LA’s ruling that there was abandonment, viz:

Under Article 264 of the Labor Code, as amended, the union


officers who knowingly participate in the illegal strike may be declared
to have lost their employment status. However, mere participation of a
union member in the illegal strike does not mean loss of employment
status unless he participates in the commission of illegal acts during
the strike. While it is true that complainant thru individual memorandum
directed the respondents to return to work (pp. 1031-1112, Records)
there is no showing that respondents deliberately refused to return to
work. A worker who joins a strike does so precisely to assert or improve
the terms and conditions of his work. If his purpose is to abandon his
work, he would not go to the trouble of joining a strike (BLTB v. NLRC,
212 SCRA 794).

On appeal, the CA affirmed the NLRC. In denying the petitioners’


claim for full backwages, the CA applied the third paragraph of Article
264(a) instead of Article 279 of the Labor Code, explaining that the
only instance under Article 264 when a dismissed employee would be
reinstated with full backwages was when he was dismissed by reason
of an illegal lockout; that Article 264 was silent on the award of
backwages to employees participating in a lawful strike; and that a
reinstatement with full backwages would be granted only when the
dismissal of the petitioners was not done in accordance with Article
282 (dismissals with just causes) and Article 283 (dismissals with
authorized causes) of the Labor Code.

ISSUE:
Are petitioners entitled to full backwages from the date of
dismissal until the date of actual reinstatement due to their not being
found to have abandoned their jobs?

RULING:
We sustain the CA, but modify the decision on the amount of the
backwages in order to accord with equity and jurisprudence.

Contemplating two causes for the dismissal of an employee, that


is: (a) unlawful lockout; and (b) participation in an illegal strike, the third
paragraph of Article 264(a) authorizes the award of full backwages only
when the termination of employment is a consequence of an unlawful
lockout.

On the consequences of an illegal strike, the provision


distinguishes between a union officer and a union member
participating in an illegal strike. A union officer who knowingly

521
Page 522

participates in an illegal strike is deemed to have lost his employment


status, but a union member who is merely instigated or induced to
participate in the illegal strike is more benignly treated. Part of the
explanation for the benign consideration for the union member is the
policy of reinstating rank-and-file workers who are misled into
supporting illegal strikes, absent any finding that such workers
committed illegal acts during the period of the illegal strikes.

The petitioners were terminated for joining a strike that was later
declared to be illegal. The NLRC ordered their reinstatement or, in lieu
of reinstatement, the payment of their separation pay, because they
were mere rank-and-file workers whom the Union’s officers had misled
into joining the illegal strike. They were not unjustly dismissed from
work. Based on the text and intent of the two aforequoted provisions of
the Labor Code, therefore, it is plain that Article 264(a) is the applicable
one.

Conformably with the long honored principle of a fair day’s wage


for a fair day’s labor, employees dismissed for joining an illegal strike
are not entitled to backwages for the period of the strike even if they
are reinstated by virtue of their being merely members of the striking
union who did not commit any illegal act during the strike.

As a general rule, backwages are granted to indemnify a


dismissed employee for his loss of earnings during the whole period
that he is out of his job. Considering that an illegally dismissed
employee is not deemed to have left his employment, he is entitled to
all the rights and privileges that accrue to him from the employment.
The grant of backwages to him is in furtherance and effectuation of the
public objectives of the Labor Code, and is in the nature of a command
to the employer to make a public reparation for his illegal dismissal of
the employee in violation of the Labor Code.

That backwages are not granted to employees participating in an


illegal strike simply accords with the reality that they do not render work
for the employer during the period of the illegal strike.

The petitioners herein do not deny their participation in the June


15, 1993 strike. As such, they did not suffer any loss of earnings during
their absence from work. Their reinstatement sans backwages is in
order, to conform to the policy of a fair day’s wage for a fair day’s labor.

522
Page 523

Case Digests by: GUEVARRA, JOY L.

University of San Augustin Employees Union v. CA


175492 February 27, 2013
,||

Garcia, J.

FACTS:
During the negotiations, the parties could not agree on the manner
of computing the TIP, thus the need to undergo preventive mediation
proceedings before the NCMB, Iloilo City. The computation of TIP was
not resolved. This development prompted the Union to declare a
bargaining deadlock grounded on the parties’ failure to arrive at a
mutually acceptable position on the manner of computing the seventy
percent (70%) of the net TIP to be allotted for salary. Thereafter, the
Union filed a Notice of Strike before the NCMB which was expectedly
opposed by the University in a Motion to Strike Out Notice of Strike and
to Refer the Dispute to Voluntary Arbitration, invoking the "No strike,
no lockout" clause of the parties’ CBA. The NCMB, however, failed to
resolve the University’s motion.

Petitioner Union is the duly recognized collective bargaining unit


for teaching and non-teaching rank-and-file personnel of the University
while the other individual petitioners are its officers. On July 27, 2000,
the parties entered into a 5-year CBA which, among other things,
provided that the economic provisions thereof shall have a period of
three (3) years or up to 2003. Complementary to said provisions is
Section 3 of Article VIII of the CBA providing for salary increases for
School Years (SY) 2000-2003, such increase to take the form of either
a lump sum or a percentage of the tuition incremental proceeds (TIP).

The Union staged a strike. At 6:45 a.m. of the same day, Sheriffs
Francisco L. Reyes and Rocky M. Francisco had arrived at San
Agustin University to serve the AJO on the Union. At the main entrance
of the University, the sheriffs saw some elements of the Union at the
early stages of the strike. There they met Merlyn Jara, the Union’s vice
president, upon whom the sheriffs tried to serve the AJO, but who, after
reading it, refused to receive the same, citing Union Board Resolution
No. 3 naming the union president as the only person authorized to do
so. The sheriffs explained to Ms. Jara that even if she refused to
acknowledge receipt of the AJO, the same would be considered
served. Sheriff Reyes further informed the Union that once the sheriffs
post the AJO, it would be considered received by the Union.

The CBA contained a "no strike, no lockout" clause and a


grievance machinery procedure to resolve management-labor

523
Page 524

disputes, including a voluntary arbitration mechanism should the


grievance committee fail to satisfactorily settle such disputes.

At approximately 8:45 a.m., the sheriffs posted copies of the


AJO at the main gate of San Agustin University, at the main entrance
of its buildings and at the Union’s office inside the campus. At 9:20
a.m., the sheriffs served the AJO on the University.

Notwithstanding the sheriffs’ advice as to the legal implication of


the Union’s refusal to be served with the AJO, the Union went ahead
with the strike. University filed a Petition to Declare Illegal Strike and
Loss of Employment Status at the NLRC Sub-regional Arbitration
Branch No. VI in Iloilo City.

Lacerna a former DOLE Regional Director. Atty. Lacerna however


refused to be officially served the Order again pointing to Board
Resolution No. 3 passed by the Union officers. Atty. Lacerna then
informed the undersigned Sheriffs that the Union president will accept
the Order at around 5:00 o’clock in the afternoon. Atty. Lacerna told
the undersigned Sheriff that only when the Union president receives
the Order at 5:00 p.m. shall the Union recognize the Secretary of Labor
as having assumed jurisdiction over the labor dispute.

ISSUE:
Whether or not the CBA clause on no strike no lockout was
violated?

RULING:
YES. When the SOLE assumes jurisdiction over a labor dispute in
an industry indispensable to national interest or certifies the same to
the NLRC for compulsory arbitration, such assumption or certification
shall have the effect of automatically enjoining the intended or
impending strike or lockout. Moreover, if one had already taken place,
all striking workers shall immediately return to work and the employer
shall immediately resume operations and readmit all workers under the
same terms and conditions prevailing before the strike or lockout.

In Trans-Asia Shipping Lines, Inc., et al. vs. CA, et al., the Court
declared that when the Secretary exercises these powers, he is
granted great breadth of discretion in order to find a solution to a labor
dispute. The most obvious of these powers is the automatic enjoining
of an impending strike or lockout or the lifting thereof if one has already
taken place. Assumption of jurisdiction over a labor dispute, or the
certification of the same to the NLRC for compulsory arbitration, always
co-exists with an order for workers to return to work immediately and

524
Page 525

for employers to readmit all workers under the same terms and
conditions prevailing before the strike or lockout.

In this case, the AJO was served at 8:45 a.m. of September 19,
2003. The strikers then should have returned to work immediately.
However, they persisted with their refusal to receive the AJO and
waited for their union president to receive the same at 5:25 p.m. The
Union’s defiance of the AJO was evident in the sheriff’s report:
“We went back to the main gate of the University and there NCMB
Director Dadivas introduced us to the Union lawyer, Atty. Mae”

Thus, we see no reversible error in the CA’s finding that the strike
of September 19, 2003 was illegal. Consequently, the Union officers
were deemed to have lost their employment status for having
knowingly participated in said illegal act. We likewise find logic in the
CA’s directive for the herein parties to proceed with voluntary
arbitration as provided in their CBA. As we see it, the issue as to the
economic benefits, which included the issue on the formula in
computing the TIP share of the employees, is one that arises from the
interpretation or implementation of the CBA. To be sure, the parties’
CBA provides for a grievance machinery to resolve any "complaint or
dissatisfaction arising from the interpretation or implementation of the
CBA and those arising from the interpretation or enforcement of
company personnel policies." Moreover, the same CBA provides that
should the grievance machinery fail to resolve the grievance or dispute,
the same shall be "referred to a Voluntary Arbitrator for arbitration and
final resolution." However, through no fault of the University these
processes were not exhausted. It must be recalled that while
undergoing preventive mediation proceedings before the NCMB, the
Union declared a bargaining deadlock, filed a notice of strike and
thereafter, went on strike. The University filed a Motion to Strike Out
Notice of Strike and to Refer the Dispute to Voluntary Arbitration but
the motion was not acted upon by the NCMB. As borne by the records,
the University has been consistent in its position that the Union must
exhaust the grievance machinery provisions of the CBA which ends in
voluntary arbitration.

The University’s stance is consistent with Articles 261 and 262 of


the Labor Code, as amended. The University filed a Motion to Strike
Out Notice of Strike and Refer the Dispute to Voluntary Arbitration
precisely to call the attention of the NCMB and the Union to the fact
that the CBA provides for a grievance machinery and the parties’
obligation to exhaust and honor said mechanism. Accordingly, the
NCMB should have directed the Union to honor its agreement with the
University to exhaust administrative grievance measures and bring the
alleged deadlock to voluntary arbitration. Unfortunately, the NCMB did
not resolve the University’s motion thus paving the way for the strike

525
Page 526

on September 19, 2003 and the deliberate circumvention of the CBA’s


grievance machinery and voluntary arbitration provisions. As we see
it, the failure or refusal of the NCMB and thereafter the SOLE to
recognize, honor and enforce the grievance machinery and voluntary
arbitration provisions of the parties’ CBA unwittingly rendered said
provisions, as well as, Articles 261 and 262 of the Labor Code, useless
and inoperative. As here, a union can easily circumvent the grievance
machinery and a previous agreement to resolve differences or conflicts
through voluntary arbitration through the simple expedient of filing a
notice of strike. On the other hand, management can avoid the
grievance machinery and voluntary arbitration provisions of its CBA by
simply filing a notice of lockout.

526
Page 527

Case Digests by: GUEVARRA, JOY L.

Philippine Diamond Hotel & Resort, Inc. v. Manila Diamond Hotel


Employees Union,
G.R. No. 158075, June 30, 2006
CARPIO MORALES, J

FACTS:
On November 11, 1996, the union, which was registered on
August 19, 1996 before the Department of Labor and Employment
(DOLE), filed a Petition for Certification Election before the DOLE-
National Capital Region (NCR) seeking certification as the exclusive
bargaining representative of its members.

The DOLE-NCR denied the union's petition as it failed to comply


with legal requirements, specifically Section 2, Rule V, Book V of the
Rules and Regulations Implementing the Labor Code, and was seen
to fragment the employees of petitioner.

Through its president Kimpo, the union later notified petitioner of


its intention to negotiate, by Notice to Bargain, a Collective Bargaining
Agreement (CBA) for its members.

Acting on the notice, the Hotel, through its Human Resource


Development Manager Mary Anne Mangalindan, advised the union
that since it was not certified by the DOLE as the exclusive bargaining
agent, it could not be recognized as such.

The union clarified that it sought to bargain "for its members only,"
and declared that "[the Hotel's] refusal to bargain [would prompt] the
union to engage in concerted activities to protect and assert its rights
under the Labor Code."

By Notice to its members dated September 18, 1997, the union


announced that its executive officers as well as its directors decided to
go on strike in view of the management's refusal to bargain collectively,
and thus called for the taking of strike vote.

Petitioner thereupon issued a Final Reminder and Warning to


respondent against continuing misinformation campaign and activities
which confused the Hotel employees and disturbed their work
performance.

The union went on to file a Notice of Strike on September 29, 1997


with the National Conciliation and Mediation Board (NCMB) due to

527
Page 528

unfair labor practice (ULP) in that the Hotel refused to bargain with it
and the rank-and-file employees were being harassed and prevented
from joining it.

ISSUE:
Whether or not there is an illegal strike.

RULING:
The respondent union is admittedly not the exclusive
representative of the majority of the employees of petitioner, hence, it
could not demand from petitioner the right to bargain collectively in
their behalf.

Respondent insists, however, that it could validly bargain in behalf


of "its members," relying on Article 242 of the Labor Code.39
Respondent's reliance on said article, a general provision on the rights
of legitimate labor organizations, is misplaced, for not every legitimate
labor organization possesses the rights mentioned therein. 40 Article
242 (a) must be read in relation to above-quoted Article 255.

On respondent's contention that it was bargaining in behalf only of


its members, the appellate court, affirming the NLRC's observation that
the same would only "fragment the employees" of petitioner, held that
"what [respondent] will be achieving is to divide the employees, more
particularly, the rank-and-file employees of [petitioner] . . . the other
workers who are not members are at a serious disadvantage, because
if the same shall be allowed, employees who are non-union members
will be economically impaired and will not be able to negotiate their
terms and conditions of work, thus defeating the very essence and
reason of collective bargaining, which is an effective safeguard against
the evil schemes of employers in terms and conditions of work." This
Court finds the observation well-taken.

It bears noting that the goal of the DOLE is geared towards "a
single employer wide unit which is more to the broader and greater
benefit of the employees working force." The philosophy is to avoid
fragmentation of the bargaining unit so as to strengthen the employees'
bargaining power with the management. To veer away from such goal
would be contrary, inimical and repugnant to the objectives of a strong
and dynamic unionism.

Petitioner's refusal to bargain then with respondent cannot be


considered a ULP to justify the staging of the strike.

An ordinary striking worker cannot, thus be dismissed for mere


participation in an illegal strike. There must be proof that he committed

528
Page 529

illegal acts during a strike, unlike a union officer who may be dismissed
by mere knowingly participating in an illegal strike and/or committing
an illegal act during a strike.

The appellate court found no convincing and substantial proof,


however, that the strikers-members of respondent who participated in
the illegal strike committed illegal acts.

529
Page 530

Case Digests by: JACABA, JOYCE B.

SOLIDBANK V. GAMIER
G.R. No. 159460. November 15, 2010.
VILLARAMA, JR., J.

DOCTRINE:
STRIKE; LIABILITY OF UNION MEMBERS AND OFFICERS
For knowingly participating in an illegal strike or participating in the
commission of illegal acts during a strike, the law provides that a union
officer may be terminated from employment. The law grants the
employer the option of declaring a union officer who participated in an
illegal strike as having lost his employment. It possesses the right and
prerogative to terminate the union officers from service. However, a
worker merely participating in an illegal strike may not be terminated
from employment. It is only when he commits illegal acts during a strike
that he may be declared to have lost employment status.

FACTS:
Solidbank and Solidbank Employees' Union (Union) were set to
renegotiate the economic provisions Collective Bargaining Agreement
(CBA). Seeing that an agreement was unlikely, the Union declared a
deadlock and filed a Notice of Strike. Some Union members staged a
series of mass actions. In view of the impending actual strike, then
Secretary of Labor and Employment assumed jurisdiction over the
labor dispute.The assumption order directed the parties "to cease and
desist from committing any and all acts that might exacerbate the
situation."

Dissatisfied with the Secretary's ruling, the Union officers and


members decided to protest the same by holding a rally infront of the
Office of the Secretary of Labor and Employment. Thus, an
overwhelming majority of employees, including the individual
respondents, joined the "mass leave" and "protest action" at the
Department of Labor and Employment (DOLE) office while the bank's
provincial branches in Cebu, Iloilo, Bacolod and Naga followed suit and
"boycotted regular work." The union members also picketed the bank's
Head Office in Binondo, and Paseo de Roxas branch. As a result of
the employees' concerted actions, Solidbank's business operations
were paralyzed.

The herein 129 individual respondents were among the 199


employees who were terminated for their participation in the three-day
work boycott and protest action. Respondents Gamier, Condevillamar,
Arriola and De Guzman filed separate complaints for illegal dismissal,

530
Page 531

moral and exemplary damages and attorney's fees. The cases were
then consolidated. Respondent Union joined by the 129 dismissed
employees filed a separate suit against petitioners for illegal dismissal,
unfair labor practice and damages.

ISSUES:
1. Whether the concerted mass actions staged by
respondents shall be considered a strike

2. Whether the respondents were validly terminated

RULING:
1. YES. Article 212 of the Labor Code, as amended, defines
strike as any temporary stoppage of work by the concerted action of
employees as a result of an industrial or labor dispute. A labor dispute
includes any controversy or matter concerning terms and conditions of
employment or the association or representation of persons in
negotiating, fixing, maintaining, changing or arranging the terms and
conditions of employment, regardless of whether or not the disputants
stand in the proximate relation of employers and employees. The term
"strike" shall comprise not only concerted work stoppages, but also
slowdowns, mass leaves, sitdowns, attempts to damage, destroy or
sabotage plant equipment and facilities and similar activities.

Considering that the mass actions stemmed from a bargaining


deadlock and an order of assumption of jurisdiction had already been
issued by the Secretary of Labor to avert an impending strike, there is
no doubt that the concerted work abandonment/boycott was the result
of a labor dispute. Moreover, it is explicit from the directive of the
Secretary in his Order that the Union and its members shall refrain from
committing "any and all acts that might exacerbate the situation," which
certainly includes concerted actions. For all intents and purposes,
therefore, the respondents staged a strike ultimately aimed at realizing
their economic demands. Whether such pressure was directed against
the petitioners or the Secretary of Labor, or both, is of no moment. All
the elements of strike are evident in the Union-instigated mass actions.

2. Article 264 (a) of the Labor Code, as amended, considers


it a prohibited activity to declare a strike "during the pendency of cases
involving the same grounds for the same strike." There is no dispute
that when respondents conducted their mass actions while the
proceedings before the Secretary of Labor were still pending as both
parties filed motions for reconsideration of the Order. Clearly,
respondents knowingly violated the aforesaid provision by holding a
strike in the guise of mass demonstration simultaneous with concerted
work abandonment/boycott.

531
Page 532

For knowingly participating in an illegal strike or participating in the


commission of illegal acts during a strike, the law provides that a union
officer may be terminated from employment. The law grants the
employer the option of declaring a union officer who participated in an
illegal strike as having lost his employment. It possesses the right and
prerogative to terminate the union officers from service. However, a
worker merely participating in an illegal strike may not be terminated
from employment. It is only when he commits illegal acts during a strike
that he may be declared to have lost employment status.

With respect to respondents who are union officers, the validity of


their termination by petitioners cannot be questioned. Being fully aware
that the proceedings before the Secretary of Labor were still pending
as in fact they filed a motion for reconsideration, they cannot invoke
good faith as a defense.

For the rest of the individual respondents who are union members,
there must be proof that he or she committed illegal acts during a strike.
In all cases, the striker must be identified. But proof beyond reasonable
doubt is not required. Substantial evidence available under the
attendant circumstances, which may justify the imposition of the
penalty of dismissal, may suffice. Liability for prohibited acts is to be
determined on an individual basis. Petitioners have not adduced
evidence on such illegal acts committed by each of the individual
respondents who are union members. The dismissal of herein
respondent-union members are therefore unjustified in the absence of
a clear showing that they committed specific illegal acts during the
mass actions and concerted work boycott.

532
Page 533

Case Digests by: JACABA, JOYCE B.

C. ALCANTARA & SONS, INC. V. COURT OF APPEALS


G.R. No. 155109. September 29, 2010.
ABAD, J.

DOCTRINE:
VALIDITY OF STRIKE WHEN THERE IS AN AGREEMENT
CONTAINING A NO STRIKE CLAUSE
A strike may be regarded as invalid although the labor union has
complied with the strict requirements for staging one as provided in
Article 263 of the Labor Code when the same is held contrary to an
existing agreement, such as a no strike clause or conclusive arbitration
clause.

FACTS:
C. Alcantara & Sons, Inc., (the Company) and Nagkahiusang
Mamumuo sa Alsons-SPFL (the Union) entered into a Collective
Bargaining Agreement (CBA) that bound them to hold no strike and no
lockout in the course of its life. The parties began negotiating the
economic provisions of their CBA but this ended in a deadlock,
prompting the Union to file a notice of strike. The Union conducted a
strike vote that resulted in an overwhelming majority of its members
favoring it. The Union reported the strike vote to the DOLE and, after
the observance of the mandatory cooling-off period, went on strike.

The Company filed a petition for the issuance of a writ of


preliminary injunction with prayer for the issuance of a temporary
restraining order (TRO) Ex Parte with the National Labor Relations
Commission (NLRC) to enjoin the strikers from intimidating,
threatening, molesting, and impeding by barricade the entry of non-
striking employees at the Company's premises. The NLRC first issued
a 20-day TRO and, after hearing, a writ of preliminary injunction.

The Company, on the other hand, filed a petition with the Regional
Arbitration Board to declare the Union's strike illegal, citing its violation
of the no strike, no lockout, provision of their CBA. Subsequently, the
Company amended its petition to implead the named Union members
who allegedly committed prohibited acts during the strike. For their
part, the Union, its officers, and its affected members filed against the
Company a counterclaim for unfair labor practices, illegal dismissal,
and damages.

533
Page 534

ISSUES:
1. Whether or not the Union staged an illegal strike

2. Assuming the strike to be illegal, whether or not the


impleaded Union members committed illegal acts during the strike,
justifying their termination from employment

3. Whether or not the terminated Union members are entitled


to accrued backwages and separation pay

RULING:
1. YES. A strike may be regarded as invalid although the
labor union has complied with the strict requirements for staging one
as provided in Article 263 of the Labor Code when the same is held
contrary to an existing agreement, such as a no strike clause or
conclusive arbitration clause. Here, the CBA between the parties
contained a "no strike, no lockout" provision that enjoined both the
Union and the Company from resorting to the use of economic
weapons available to them under the law and to instead take recourse
to voluntary arbitration in settling their disputes. No law or public policy
prohibits the Union and the Company from mutually waiving the strike
and lockout maces available to them to give way to voluntary
arbitration. Indeed, no less than the 1987 Constitution recognizes in
Section 3, Article XIII, preferential use of voluntary means to settle
disputes.

2. YES. As regards the rank and file Union members, Article


264 of the Labor Code provides that termination from employment is
not warranted by the mere fact that a union member has taken part in
an illegal strike. It must be shown that such a union member, clearly
identified, performed an illegal act or acts during the strike.

The Sheriff of the NLRC said in his Report that, in the course of
his implementation of the writ of injunction, he observed that the
striking employees blocked the exit lane of the Alson drive with their
tent. Tungapalan, a non-striking employee, identified the Union
members who threatened and coerced him. Indeed, he filed criminal
actions against them. Lastly, the photos taken of the strike show the
strikers, properly identified, committing the acts complained of. These
constitute substantial evidence in support of the termination of the
subject Union members.

3. YES.

AS TO BACKWAGES: Article 223, which provides that the


decision of the Labor Arbiter reinstating a dismissed employee shall
immediately be executory pending appeal, cannot but apply to all

534
Page 535

terminations irrespective of the grounds on which they are based.


Here, although the Labor Arbiter failed to act on the terminated Union
members' motion for reinstatement pending appeal, the Company had
the duty under Article 223 to immediately reinstate the affected
employees even if it intended to appeal from the decision ordaining
such reinstatement. The Company's failure to do so makes it liable for
accrued backwages until the eventual reversal of the order of
reinstatement by the NLRC, a period of four months and nine days.

AS TO SEPARATION PAY: While it is true that generally the grant


of separation pay is not available to employees who are validly
dismissed, there are, in furtherance of the law's policy of
compassionate justice, certain circumstances that warrant the grant of
some relief in favor of the terminated Union members based on equity.

535
Page 536

Case Digests by: JACABA, JOYCE B.

SUKHOTHAI CUISINE AND RESTAURANT V. COURT OF


APEALS
G.R. No. 150437. July 17, 2006.
AUSTRIA-MARTINEZ, J.

DOCTRINE:
STRIKES IN VIOLATION OF AGREEMENTS PROVIDING FOR
ARBITRATION
Strikes staged in violation of agreements providing for arbitration
are illegal, since these agreements must be strictly adhered to and
respected if their ends are to be achieved. The rationale of the
prohibition under Article 264 is that once jurisdiction over the labor
dispute has been properly acquired by competent authority, that
jurisdiction should not be interfered with by the application of the
coercive processes of a strike.

FACTS:
Private respondent PLAC Local 460 Sukhothai Restaurant
Chapter (Union) filed a Notice of Strike with the National Conciliation
and Mediation Board (NCMB) on the ground of unfair labor practice.
The representatives of the petitioner agreed and guaranteed that there
will be no termination of the services of private respondents during the
pendency of the case, with the reservation of the management
prerogative to issue memos to erring employees for the infraction, or
violation of company policies. The petitioner and the Union entered into
a Submission Agreement, thereby agreeing to submit the issue of
unfair labor practice for voluntary arbitration with a view to prevent the
strike.

During the pendency of the voluntary arbitration proceedings, the


petitioner dismissed Eugene Lucente, a union member, due to an
alleged petty quarrel with a coemployee. The Union filed with the
NLRC a complaint for illegal dismissal. Subsequently, private
respondent Jose Lanorias, a union member, was relieved from his
post, and his employment as cook, terminated. Respondent Billy
Bacus, the union vice-president, conferred with Ernesto Garcia and
protested Lanorias's dismissal. Thereafter, respondents staged a
"wildcat strike."

Notice of Strike was re-filed by the private respondents and the


protest, according to the respondents, was converted into a "sit-down
strike." The same was transformed into an "actual strike." The
petitioner filed a complaint for illegal strike with the NLRC against

536
Page 537

private respondents, seeking to declare the strike illegal, and to declare


respondents, who participated in the commission of illegal acts, to have
lost their employment status.

ISSUES:
1. Whether the strike staged by the respondents was illegal

2. Whether private respondents are deemed to have lost their


employment status by participating in the commission of illegal acts
during the strike

RULING:
1. YES. Strikes staged in violation of agreements providing
for arbitration are illegal, since these agreements must be strictly
adhered to and respected if their ends are to be achieved. The
rationale of the prohibition under Article 264 is that once jurisdiction
over the labor dispute has been properly acquired by competent
authority, that jurisdiction should not be interfered with by the
application of the coercive processes of a strike.

The alleged dismissals of the union members, which allegedly


triggered the wildcat strike, are not sufficient grounds to justify the
radical recourse on the part of the private respondents. Private
respondents should have availed of the appropriate remedies under
the Labor Code, such as the institution of cases of illegal dismissal or,
by agreement of the parties, the submission of the cases to the
grievance machinery of the CBA, if one is available, so that they may
be subjected to separate voluntary arbitration proceedings, or simply
seek to terminate the pending voluntary arbitration case and complete
the mandatory procedure for a lawful strike. Private respondents
should have availed themselves of any of these alternative remedies
instead of resorting to a drastic and unlawful measure, specifically, the
holding a wildcat strike. And because of the fact that the Union was
fully aware that the arbitration proceedings were pending, good faith
cannot be invoked as a defense.

For failing to exhaust all steps in the arbitration proceedings by


virtue of the Submission Agreement, in view of the proscription under
Article 264 of the Labor Code, and the prevailing state policy as well
as its underlying rationale, this Court declares that the strike staged by
the private respondents is illegal.

What is more, the strike had been attended by the widespread


commission of prohibited acts. Even if the strike were to be declared
valid because its objective or purpose is lawful, the strike may still be
declared invalid where the means employed are illegal. Among such
limits are the prohibited activities under Article 264 of the Labor Code,

537
Page 538

particularly paragraph (e), which states that no person engaged in


picketing shall: a) commit any act of violence, coercion, or intimidation
or b) obstruct the free ingress to or egress from the employer's
premises for lawful purposes, or c) obstruct public thoroughfares.

The following acts have been held to be prohibited activities:


where the strikers shouted slanderous and scurrilous words against
the owners of the vessels; where the strikers used unnecessary and
obscene language or epithets to prevent other laborers to go to work,
and circulated libelous statements against the employer which show
actual malice; where the protestors used abusive and threatening
language towards the patrons of a place of business or against co-
employees, going beyond the mere attempt to persuade customers to
withdraw their patronage; where the strikers formed a human cordon
and blocked all the ways and approaches to the launches and vessels
of the vicinity of the workplace and perpetrated acts of violence and
coercion to prevent work from being performed; and where the strikers
shook their fists and threatened non-striking employees with bodily
harm if they persisted to proceed to the workplace. Permissible
activities of the picketing workers do not include obstruction of access
of customers.

2. DEPENDS. In Samahang Manggagawa sa Sulpicio Lines,


Inc.-NAFLU v. Sulpicio Lines, Inc. this Court explained that the effects
of such illegal strikes, outlined in Article 264, make a distinction
between workers and union officers who participate therein: an
ordinary striking worker cannot be terminated for mere participation in
an illegal strike. There must be proof that he or she committed illegal
acts during a strike. A union officer, on the other hand, may be
terminated from work when he knowingly participates in an illegal
strike, and like other workers, when he commits an illegal act during a
strike. In all cases, the striker must be identified. But proof beyond
reasonable doubt is not required. Substantial evidence available under
the attendant circumstances, which may justify the imposition of the
penalty of dismissal, may suffice.

538
Page 539

Case Digests by: JACABA, JOYCE B.

BIFLEX PHILS. INC. LABOR UNIONS V. FILFLEX


INDUSTRIAL MANUFACTURING CORPORATION
G.R. No. 155679. December 19, 2006.
CARPIO MORALES, J.

DOCTRINES:
GENERAL STRIKE AS AN ILLEGAL STRIKE
Stoppage of work due to welga ng bayan is in the nature of a
general strike, an extended sympathy strike. It affects numerous
employers including those who do not have a dispute with their
employees regarding their terms and conditions of employment.
Employees who have no labor dispute with their employer but who, on
a day they are scheduled to work, refuse to work and instead join a
welga ng bayan commit an illegal work stoppage

MEANS OF CARRYING OUT THE STRIKE AFFECTS ITS


LEGALITY
The legality of a strike is determined not only by compliance with
its legal formalities but also by the means by which it is carried out.

FACTS:
Petitioners are officer of Biflex (Phils.) Inc. Labor Union and Filflex
Industrial and Manufacturing Labor Union. The labor sector staged a
welga ng bayan to protest the accelerating prices of oil. Petitioner-
unions, led by their officers, herein petitioners, staged a work stoppage
which lasted for several days, prompting respondents to file a petition
to declare the work stoppage illegal for failure to comply with
procedural requirements.

Petitioners, claiming that they were illegally locked out by


respondents, assert that aside from the fact that the welga ng bayan
rendered it difficult to get a ride and the apprehension that violence
would erupt between those participating in the welga and the
authorities, respondents' workers were prevented from reporting for
work. Respondents, on the other hand, maintain that the work
stoppage was illegal since the following requirements for the staging
of a valid strike were not complied with: (1) filing of notice of strike; (2)
securing a strike vote, and (3) submission of a report of the strike vote
to the Department of Labor and Employment.

ISSUE:
Whether the strike was legal

539
Page 540

RULING:
NO. Stoppage of work due to welga ng bayan is in the nature of a
general strike, an extended sympathy strike. It affects numerous
employers including those who do not have a dispute with their
employees regarding their terms and conditions of employment.
Employees who have no labor dispute with their employer but who, on
a day they are scheduled to work, refuse to work and instead join a
welga ng bayan commit an illegal work stoppage.

Even if petitioners' joining the welga ng bayan were considered


merely as an exercise of their freedom of expression, freedom of
assembly or freedom to petition the government for redress of
grievances, the exercise of such rights is not absolute. There being no
showing that petitioners notified respondents of their intention, or that
they were allowed by respondents, to join the welga ng bayan, their
work stoppage is beyond legal protection.

Even assuming arguendo that in staging the strike, petitioners had


complied with legal formalities, the strike would just the same be illegal,
for by blocking the free ingress to and egress from the company
premises, they violated Article 264(e) of the Labor Code which
provides that "[n]o person engaged in picketing shall . . . obstruct the
free ingress to or egress from the employer's premises for lawful
purposes, or obstruct public thoroughfares." In fine, the legality of a
strike is determined not only by compliance with its legal formalities but
also by the means by which it is carried out. Petitioners, being union
officers, should thus bear the consequences of their acts of knowingly
participating in an illegal strike, conformably with the third paragraph of
Article 264 (a) of the Labor Code.

540
Page 541

Case Digests by: JACABA, JOYCE B.

SANTA ROSA COCA-COLA PLANT EMPLOYEES UNION


V. COCA-COLA BOTTLERS PHILS. INC.
G.R. Nos. 164302-03. January 24, 2007.
CALLEJO, SR., J.

DOCTRINE:
PENALTY IN ILLEGAL STRIKES; DEFINITION OF OFFICERS
Officers normally mean those who hold defined offices. An officer
is any person occupying a position identified as an office. An office may
be provided in the constitution of a labor union or by the union itself in
its CBA with the employer. An office is a word of familiar usage and
should be construed according to the sense of the thing.

FACTS:
The Sta. Rosa Coca-Cola Plant Employees Union (Union) and the
Coca-Cola Bottlers Philippines, Inc. (Company) had entered into a
three-year Collective Bargaining Agreement (CBA). Upon the
expiration of the CBA, the Union informed the Company of its desire to
renegotiate its terms. The CBA meetings commenced. The Union
insisted that representatives from the Alyansa ng mga Unyon sa Coca-
Cola be allowed to sit down as observers in the CBA meetings. The
Union officers and members also insisted that their wages be based
on their work shift rates. For its part, the Company was of the view that
the members of the Alyansa were not members of the bargaining unit.
The Alyansa was a mere aggregate of employees of the Company in
its various plants; and is not a registered labor organization. Thus, an
impasse ensued.

The Union, its officers, directors and six shop stewards filed a
"Notice of Strike" with the National Conciliation and Mediation Board
(NCMB) on two grounds: (a) deadlock on CBA ground rules; and (b)
unfair labor practice arising from the company's refusal to bargain. The
Union then filed an Amended Notice of Strike on the following grounds:
(a) unfair labor practice for the company's refusal to bargain in good
faith; and (b) interference with the exercise of their right to self-
organization.

The Union decided to participate in a mass action organized by


the Alyansa ng mga Unyon sa Coca-Cola in front of the Company's
premises. Union members, officers and members of the Board of
Directors, and shop stewards, individually filed applications for leave
of absence. Certain that its operations in the plant would come to a
complete stop since there were no sufficient trained contractual

541
Page 542

employees who would take over, the Company disapproved all leave
applications and notified the applicants accordingly. The Company
filed a "Petition to Declare Strike Illegal".

ISSUES:
1. Whether the strike was legal

2. Assuming that that the strike was illegal, whether the


petitioner-shop stewards be considered officers of the union in the
determination of the proper penalty

RULING:
1. NO. A strike is the most powerful of the economic weapons
of workers which they unsheathe to force management to agree to an
equitable sharing of the joint product of labor and capital. The decision
to declare a strike must rest on a rational basis, free from emotionalism,
envisaged by the tempers and tantrums of a few hot heads, and finally
focused on the legitimate interests of the Union which should not,
however, be antithetical to the public welfare, and, to be valid, a strike
must be pursued within legal bounds. The right to strike as a means of
attainment of social justice is never meant to oppress or destroy the
employer.

For a strike to be valid, the following procedural requisites


provided by Art. 263 of the Labor Code must be observed: (a) a notice
of strike filed with the DOLE 30 days before the intended date thereof,
or 15 days in case of unfair labor practice; (b) strike vote approved by
a majority of the total union membership in the bargaining unit
concerned obtained by secret ballot in a meeting called for that
purpose, (c) notice given to the DOLE of the results of the voting at
least seven days before the intended strike. These requirements are
mandatory and the failure of a union to comply therewith renders the
strike illegal. It is clear in this case that petitioners totally ignored the
statutory requirements and embarked on their illegal strike.

It bears stressing that the law makes a distinction between union


members and union officers. A worker merely participating in an illegal
strike may not be terminated from employment. It is only when he
commits illegal acts during a strike that he may be declared to have
lost employment status. For knowingly participating in an illegal strike
or participates in the commission of illegal acts during a strike, the law
provides that a union officer may be terminated from employment. The
law grants the employer the option of declaring a union officer who
participated in an illegal strike as having lost his employment. It
possesses the right and prerogative to terminate the union officers
from service.

542
Page 543

2. YES. Under Section 501 (a) and (b) of the Landrum Griffin
Act of 1959, shop stewards are officers of the Union. There is no similar
provision in the Labor Code of the Philippines; nonetheless, petitioners
who are shop stewards are considered union officers.

Officers normally mean those who hold defined offices. An officer


is any person occupying a position identified as an office. An office may
be provided in the constitution of a labor union or by the union itself in
its CBA with the employer. An office is a word of familiar usage and
should be construed according to the sense of the thing.

A shop steward is appointed by the Union in a shop, department,


or plant serves as representative of the Union, charged with
negotiating and adjustment of grievances of employees with the
supervisor of the employer In fine, they are part and parcel of the
continuous process of grievance resolution designed to preserve and
maintain peace among the employees and their employer. They
occupy positions of trust and laden with awesome responsibilities.

In this case, instead of playing the role of "peacemakers" and


grievance solvers, the petitioners-shop stewards participated in the
strike. Thus, like the officers and directors of petitioner Union who
joined the strike, petitioners-shop stewards also deserve the penalty of
dismissal from their employment.

543
Page 544

Case Digests by: JACABA, JOYCE B.

MANILA HOTEL EMPLOYEES ASSOCIATION V. MANILA


HOTEL CORPORATION
G.R. No. 154591. March 5, 2007.
CHICO-NAZARIO, J.

DOCTRINE:
STRIKES IN VIOLATION OF RETURN-TO-WORK ORDER
Defiance of the assumption order or a return-to work order by a
striking employee, whether a union officer or a member, is an illegal
act and, therefore, a valid ground for loss of employment status.

FACTS:
The Manila Hotel Employees Association (MHEA) filed a Notice of
Strike with the National Conciliation and Mediation Board (NCMB)
against Manila Hotel on the grounds of unfair labor practices. The
Secretary of Labor and Employment (SOLE) certified the labor dispute
to the NLRC for compulsory arbitration. Specifically, the Order enjoined
any strike or lockout and the parties were ordered to cease and desist
from committing any acts that may exacerbate the situation. The
parties and their counsels were served copies of the said Order.

The case was set for mandatory conference. The parties were
advised of the certification order, which prohibited them from taking
any action that would exacerbate the situation. The MHEA conducted
a strike despite the clear terms of the Order. Thereafter, several
conferences were conducted by the NLRC, wherein both parties were
warned against aggravating the already volatile situation.

Manila Hotel filed a complaint with Prayer for Injunction and/or


Temporary Restraining Order, alleging that MHEA conducted an illegal
strike, blocked all ingress and egress of the hotel premises, harassed
and intimidated company officers, non-striking employees, customers
and suppliers. The NLRC issued an Order directing the striking
workers to return to work immediately and the hotel to accept them
back under the same terms and conditions of employment. The NLRC
received a copy of the Compliance filed by Manila Hotel, manifesting
that only six striking employees complied with the return-to-work Order
and were reinstated. The other striking employees had openly defied
the said Order.

ISSUE:
Whether the strike was legal

544
Page 545

RULING:
NO. The Court has consistently ruled in a long line of cases
spanning several decades that once the SOLE assumes jurisdiction
over a labor dispute, such jurisdiction should not be interfered with by
the application of the coercive processes of a strike or lockout.
Defiance of the assumption order or a return-to work order by a striking
employee, whether a union officer or a member, is an illegal act and,
therefore, a valid ground for loss of employment status.

Returning to work in this situation is not a matter of option or


voluntariness but of obligation. The worker must return to his job
together with his co-workers so the operations of the company can be
resumed and it can continue serving the public and promoting its
interest. This extraordinary authority given to the Secretary of Labor is
aimed at arriving at a peaceful and speedy solution to labor disputes,
without jeopardizing national interests. Regardless therefore of their
motives, or the validity of their claims, the striking workers must cease
and/or desist from any and all acts that tend to, or undermine this
authority of the Secretary of Labor, once an assumption and/or
certification order is issued. They cannot, for instance, ignore return-
to-work orders, citing unfair labor practices on the part of the company,
to justify their action.

545
Page 546

Case Digests by: JACABA, JOYCE B.

G&S TRANSPORT CORPORATION V. INFANTE


G.R. No. 160303. September 13, 2007.
TINGA, J.

DOCTRINE:
EXISTENCE OF LABOR DISPUTE IN STRIKES
Article 212 of the Labor Code defines strike as any temporary
stoppage of work by the concerted action of employees as a result of
an industrial or labor dispute. A valid strike therefore presupposes the
existence of a labor dispute.

FACTS:
Petitioner G & S Transport Corporation claimed to have received
from the NAIA Airport Taxi Service Employees Union-TUPAS (Union)
a letter-memorandum demanding the dismissal from employment of
Ricardo Gonzales (Gonzales) and Ephraim Alzaga (Alzaga), both
drivers of petitioner on the ground that they were found guilty of
committing acts of disloyalty, conduct unbecoming of a union member
and acts inimical to the interest of the Union. The Union based its
action on a petition filed by said employees calling for a local election.
The two employees were terminated by petitioner.

Several drivers of petitioner stopped driving their taxi cabs


apparently in sympathy with their dismissed colleagues. Petitioner
alleged that the work stoppage constituted an illegal strike at the work
premises. Furthermore, petitioner averred that various illegal acts,
such as stopping, barring and intimidating other employees wishing to
enter the work premises, were committed by the said drivers that
resulted in the paralyzation of petitioner's business operation.

Petitioner ordered the striking workers to return to work but some


of the drivers, including respondents, refused to do so. Petitioner filed
an action for illegal strike before the Labor Arbiter against thirty-seven
(37) drivers. Two days later, said drivers filed a case for illegal
dismissal against petitioner.

ISSUES:
1. Whether the strike was legal

2. Whether the respondents were validly terminated

546
Page 547

RULING:
1. NO. Article 212 of the Labor Code defines strike as any
temporary stoppage of work by the concerted action of employees as
a result of an industrial or labor dispute. A valid strike therefore
presupposes the existence of a labor dispute. The strike undertaken
by respondents took the form of a sit-down strike, or more aptly termed
as a sympathetic strike, where the striking employees have no
demands or grievances of their own, but they strike for the purpose of
directly or indirectly aiding others, without direct relation to the
advancement of the interest of the strikers. It is indubitable that an
illegal strike in the form of a sit-down strike occurred in petitioner's
premises, as a show of sympathy to the two employees who were
dismissed by petitioner. Apart from the allegations in its complaint for
illegal strike filed before the Labor Arbiter, petitioner presented the
affidavits and testimonies of their other employees which confirm the
participation of respondents in the illegal strike. Petitioner has
sufficiently established that respondents remained in the work
premises in the guise of waiting for orders from management to
resume operations when, in fact, they were actively participating in the
illegal strike.

2. NO. The Labor Code protects an ordinary, rank-and-file


union member who participated in such a strike from losing his job,
provided that he did not commit an illegal act during the strike. It can
be gleaned from Article 264 of the Labor Code that an ordinary striking
employee cannot be terminated for mere participation in an illegal
strike. There must be proof that he committed illegal acts during the
strike and the striker who participated in the commission of illegal act
must be identified. Proof beyond reasonable doubt is not required.
Substantial evidence available under the attendant circumstances,
which may justify the imposition of the penalty of dismissal, may
suffice.

In the case at bar, this Court is not convinced that the affidavits of
petitioner's witnesses constitute substantial evidence to establish that
illegal acts were committed by respondents. Nowhere in their affidavits
did these witnesses cite the particular illegal acts committed by each
individual respondent during the strike. Notably, no questions during
the hearing were asked relative to the supposed illegal acts.

547
Page 548

Case Digests by: JACABA, JOYCE B.

STEEL CORPORATION OF THE PHILIPPINES V. SCP


EMPLOYEES UNION-NATIONAL FEDERATION OF
LABOR UNIONS
G.R. Nos. 169829-30. April 16, 2008.
AZCUNA, J.

DOCTRINE:
AUTOMATIC NATURE OF RETURN-TO-WORK ORDER
The mere issuance of an assumption order by the Secretary of
Labor automatically carries with it a return-to-work order, even if the
directive to return to work is not expressly stated in the assumption
order.

FACTS:
SCP-Federated Union of the Energy Leaders-General and Allied
Services (FUEL-GAS) filed a petition for Certification Election in its bid
to represent the rank-and-file employees of the petitioner. A consent
election was conducted. Said election was declared a failure because
less than a majority of the rank-and-file employees cast their votes.
FUEL-GAS filed an Election Protest.

SCP Employees Union (SCPEU)-National Federation of Labor


Unions (NAFLU), the mother federation of respondent, filed a petition
for certification Election for and on behalf of its affiliate, seeking to
represent the rank-and-file employees of petitioner. The Med-Arbiter
denied the election protest of FUEL-GAS and granted the petition for
certification election.

The Department of Labor and Employment (DOLE)


Undersecretary ordered the conduct of a certification election with
"FUEL-GAS," respondent and "NO UNION" as choices. The
certification election proceeded. Respondent emerged as winner;
hence, the second election protest filed by FUEL-GAS. On appeal, the
CA rendered a Decision which annulled and set aside the decision and
resolution of the Undersecretary. The CA further directed the holding
of a certification election with "FUEL-GAS" and "NO UNION" as
choices, to the exclusion of respondent.

The Med-Arbiter dismissed FUEL-GAS' election protest but


deferred the request of respondent to be declared winner in the
certification election until final resolution of the pending petitions with
the CA. Respondent appealed to the Labor Secretary. The

548
Page 549

Undersecretary rendered a Decision certifying respondent as the


exclusive bargaining agent of petitioner's employees. As a
consequence, respondent sent to petitioner CBA proposals. Petitioner,
however, held in abeyance any action on the proposals in view of its
pending motion for reconsideration. Respondent filed a Notice of Strike
with the National Conciliation and Mediation Board (NCMB). The union
raised the issue of unfair labor practice (ULP) allegedly committed by
petitioner for the latter's refusal to bargain with it.

The labor dispute was certified to the National Labor Relations


Commission (NLRC) for compulsory arbitration. Another Notice of
Strike was filed by respondent for non-recognition as a certified union;
refusal to bargain; discrimination against union officers and members;
harassment and intimidation; and illegal dismissal. The NLRC issued
a Resolution, declaring petitioner as having no obligation to recognize
respondent as the certified bargaining agent; dismissing the charge of
unfair labor practice; declaring as illegal the strike held by the union;
and declaring the loss of employment of the officers of the union.

Respondent filed another Notice of Strike alleging as grounds,


petitioner's refusal to bargain and union busting. The notice was later
dismissed and respondent was enjoined from holding a strike.

Respondent filed another Notice of Strike on the grounds of refusal


to bargain and union busting. Respondent thereafter went on strike.
The Labor Secretary certified the dispute to the NLRC and directed the
employees to return to work. The NLRC rendered a Decision ordering
petitioner to bargain collectively with respondent as the duly certified
bargaining agent.

ISSUES:
Whether the strike was legal

RULING:
NO. The strike was illegal for having been conducted in utter
defiance of the Secretary's return-to-work order and after the dispute
had been certified for compulsory arbitration. Although ostensibly there
were several notices of strike successively filed by respondent, these
notices were founded on substantially the same grounds — petitioner's
continued refusal to recognize it as the collective bargaining
representative.

The moment the Secretary of Labor assumes jurisdiction over a


labor dispute in an industry indispensable to national interest, such
assumption shall have the effect of automatically enjoining the
intended or impending strike. It was not even necessary for the
Secretary of Labor to issue another order directing a return to work.

549
Page 550

The mere issuance of an assumption order by the Secretary of Labor


automatically carries with it a return-to-work order, even if the directive
to return to work is not expressly stated in the assumption order.

Respondent, in the instant case, after the assumption of


jurisdiction and certification of the dispute to the NLRC for compulsory
arbitration, filed notices of strike and staged the strike obviously
contrary to the provisions of labor laws. Worse, it filed not one but
several notices of strike which resulted in two certified cases which
were earlier consolidated. These disputes could have been averted
had respondent respected the CA's decision. That way, the collective
bargaining agent would have been determined and petitioner could
have been compelled to bargain. Respondent, through its officers,
instead opted to use the weapon of strike to force petitioner to
recognize it as the bargaining agent. The strike, having been staged
after the dispute had been certified for arbitration and contrary to the
return-to-work order, became a prohibited activity, and was thus illegal.

550
Page 551

Case Digests by: JACABA, JOYCE B.

CHRIS GARMENTS CORPORATION V. STO. TOMAS


G.R. No. 167426. January 12, 2009.
QUISUMBING, J.

DOCTRINE:
DOCTRINE OF RES JUDICATA
The elements of res judicata are: (1) the judgment sought to bar
the new action must be final; (2) the decision must have been rendered
by a court having jurisdiction over the subject matter and the parties;
(3) the disposition of the case must be a judgment on the merits; and
(4) there must be as between the first and second action, identity of
parties, subject matter, and causes of action.

FACTS:
Respondent Chris Garments Workers Union-PTGWO filed a
petition for certification election with the Med-Arbiter. The union sought
to represent petitioner's rank-and-file employees not covered by its
Collective Bargaining Agreement (CBA) with the Samahan Ng Mga
Manggagawa sa Chris Garments Corporation-Solidarity of Union in the
Philippines for Empowerment and Reforms (SMCGC-SUPER).

Petitioner moved to dismiss the petition. It argued that it has an


existing CBA with SMCGC-SUPER which bars any petition for
certification election prior to the 60-day freedom period. It also
contended that the union members are not its regular employees.

The Med-Arbiter dismissed the petition. The Med-Arbiter ruled that


there was no employer-employee relationship between the parties.
The Med-Arbiter also held that even if the union members are
considered direct employees of petitioner, the petition for certification
election will still fail due to the contract bar rule under Article 232 of the
Labor Code. Hence, a petition could only be filed during the 60-day
freedom period of the CBA.

The union filed a second petition for certification election. The


Med-Arbiter dismissed the petition on the ground that it was barred by
a prior judgment. On appeal, the Secretary of Labor and Employment
affirmed the decision of the Med-Arbiter.

The union filed a third petition for certification election. The Med-
Arbiter dismissed the petition on the grounds that no employer-
employee relationship exists between the parties and that the case

551
Page 552

was barred by a prior judgment. On appeal, the Secretary of Labor and


Employment granted the petition.

ISSUES:
1. Whether the case is barred by res judicata or
conclusiveness of judgment

2. Whether there is an employer-employee relationship


between petitioner and the union members

RULING:
1. NO. The doctrine of res judicata provides that a final
judgment or decree on the merits by a court of competent jurisdiction
is conclusive of the rights of the parties or their privies in all later suits
on points and matters determined in the former suit. The elements of
res judicata are: (1) the judgment sought to bar the new action must
be final; (2) the decision must have been rendered by a court having
jurisdiction over the subject matter and the parties; (3) the disposition
of the case must be a judgment on the merits; and (4) there must be
as between the first and second action, identity of parties, subject
matter, and causes of action.

Res judicata has a dual aspect: first, "bar by prior judgment" which
is provided in Rule 39, Section 47 (b) of the 1997 Rules of Civil
Procedure and second, "conclusiveness of judgment" which is
provided in Section 47 (c) of the same Rule. There is "bar by prior
judgment" when, as between the first case where the judgment was
rendered, and the second case that is sought to be barred, there is
identity of parties, subject matter, and causes of action. In this
instance, the judgment in the first case constitutes an absolute bar to
the second action. On the other hand, the doctrine of "conclusiveness
of judgment" provides that issues actually and directly resolved in a
former suit cannot again be raised in any future case between the
same parties involving a different cause of action.

In the instant case, there is no dispute as to the presence of the


first three elements of res judicata. The Resolution of the Secretary of
Labor and Employment on the first petition for certification election
became final and executory. It was rendered on the merits and the
Secretary of Labor and Employment had jurisdiction over the case.

The Secretary of Labor and Employment dismissed the first


petition as it was filed outside the 60-day freedom period. At that time
therefore, the union has no cause of action since they are not yet
legally allowed to challenge openly and formally the status of SMCGC-
SUPER as the exclusive bargaining representative of the bargaining
unit. Such dismissal, however, has no bearing in the instant case since

552
Page 553

the third petition for certification election was filed well within the 60-
day freedom period. Otherwise stated, there is no identity of causes of
action to speak of since in the first petition, the union has no cause of
action while in the third, a cause of action already exists for the union
as they are now legally allowed to challenge the status of SMCGC-
SUPER as exclusive bargaining representative.

2. YES. The matter of employer-employee relationship has


been resolved with finality by the Secretary of Labor and Employment
in the Resolution. Since petitioner did not appeal this factual finding,
then, it may be considered as the final resolution of such issue. To
reiterate, "conclusiveness of judgment" has the effect of preclusion of
issues.

553
Page 554

Case Digests by: JACABA, JOYCE B.

UNIVERSITY OF IMMACULATE CONCEPTION V.


SECRETARY OF LABOR
G.R. Nos. 178085-178086. September 14, 2015.
JARDELEZA, J.

DOCTRINE:
EXTENT OF JURISDICTION OF SECRETARY OF LABOR
The authority to create the tripartite committee flows from the
jurisdiction conferred by Article 263 (g) to the Secretary. A grant of
jurisdiction, in the absence of prohibitive legislation, implies the
necessary and usual incidental powers essential to effectuate it— also
referred to as "incidental jurisdiction."

FACTS:
The UIC Teaching and Non-Teaching Employees Union — FFW
(the "Union") filed a notice of strike on the grounds of bargaining
deadlock and unfair labor practice. The National Conciliation and
Mediation Board (NCMB) called the parties to a conference where they
agreed that an increase be granted to the workers.

On the same occasion, the UIC demanded the exclusion of


secretaries, registrars, accounting personnel and guidance counselors
from the bargaining unit, on account of their being confidential
employees. When the parties agreed to submit this particular issue to
voluntary arbitration, the arbitration panel sustained the UIC. The UIC
gave the affected employees, the "Respondent Employees", the option
to choose between keeping their positions or resigning from the Union.
When they elected to keep both their positions and their union
membership, UIC sent them notices of termination, which led into a
notice of strike filed by the Union. The Secretary suspended the effects
of the said termination and ordered UIC to reinstate the respondent
employees. This Order was modified by the Secretary directing the
payroll reinstatement of the respondent employees.

The Union filed its second notice of strike mostly on the grounds
of bargaining deadlock on the issues of computing the seventy percent
(70%) incremental proceeds and unfair labor practices. The Secretary
assumed jurisdiction over the dispute, issued a Return-to-Work Order.

The Secretary issued an Order directing the parties to execute a


collective bargaining agreement (CBA) embodying all items agreed
upon by the parties and the salary increases. The Secretary likewise

554
Page 555

upheld the validity of the strike declared by the Union. This Order was
challenged by UIC before the Court of Appeals and the Supreme
Court, both of which affirmed the same.

The Union moved before the Secretary for the creation of a


tripartite committee to compute the net proceeds of the tuition fee
increases. The parties signed the CBA. The CBA was submitted to the
Regional Labor Office. The DOLE issued an Order granting the motion
to create a tripartite committee.

UIC filed two separate Petitions. In the first petition, UIC assailed
the Secretary's order mandating the creation of a tripartite committee
for the purpose of computing the net incremental proceeds. In the
second petition, UIC assailed the Secretary's finding that the
Respondent Employees were illegally dismissed.

ISSUE:
Whether the Secretary of Labor has jurisdiction to order the
creation of a tripartite committee

RULING:
YES. The authority to create the tripartite committee flows from
the jurisdiction conferred by Article 263 (g) to the Secretary. A grant of
jurisdiction, in the absence of prohibitive legislation, implies the
necessary and usual incidental powers essential to effectuate it— also
referred to as "incidental jurisdiction." Incidental jurisdiction includes
the power and authority of an office or tribunal to do all things
reasonably necessary for the administration of justice within the scope
of its jurisdiction, and for the enforcement of its judgment and
mandates. Incidental jurisdiction is presumed to attach upon the
conferment of jurisdiction over the main case, unless explicitly withheld
by the legislature. In this regard, we find nothing in the Labor Code that
prohibits the Secretary from creating ad hoc committees to aid in the
resolution of labor disputes after he has assumed jurisdiction. The
primary objective of Article 263 (g) is not merely to terminate labor
disputes between private parties; rather, it is the promotion of the
common good considering that a prolonged strike or lockout in an
industry indispensable to the national interest can be inimical to the
economy. Hence, provided that the Secretary's orders are reasonably
connected with the objective of the law, as it is in this case, courts will
not disturb the same.

555
Page 556

Case Digests by: JACABA, JOYCE B.

RAMIREZ V. POLYSON INDUSTRIES, INC.


G.R. No. 207898. October 19, 2016.
PERALTA, J.

DOCTRINE:
NUMBER OF EMPLOYEES PARTICIPATING IN STRIKE IS
IMMATERIAL
It is not necessary that any fixed number of employees should quit
their work in order to constitute the stoppage a strike, and the number
of persons necessary depends in each case on the peculiar facts in
the case and no definite rule can be laid down.

FACTS:
The instant case arose from a labor dispute, between petitioners,
who were employees of Polyson Industries, Inc (Polyson) and were
officers of Obrero Pilipino (Obrero), the union of the employees of
Polyson, and respondent Polyson, which was certified by the Secretary
of the Department of Labor and Employment (DOLE) to the NLRC for
compulsory arbitration.

Polyson alleged that: it received a notice of hearing from the DOLE


with respect to the petition for certification election filed by Obrero;
Polyson met with the officers of Obrero; Obrero asked that it be
voluntarily recognized as the exclusive bargaining agent of the rank-
and-file employees of Polyson, but the latter refused; the Obrero
officers threatened the management that the union will show its
collective strength in the coming days; Polyson received a rush order
from one of its clients; the management of Polyson informed the
operators that they would be needing workers to work overtime
because of the said order; based on the usual practice of the company,
those who intend to perform overtime work were expected to sign the
"time sheet" indicating their willingness to work after their shift; the
supervisors approached the operators but were told that they would be
unable to work overtime because they have other commitments after
their shift; the supervisors then requested that the operators set aside
their time for the following day to work beyond their regular shift; five
(5) operators indicated their desire to work overtime; however, after
their regular shift, three of the five workers did not work overtime which
resulted in the delay in delivery of the client's order and eventually
resulted in the cancellation of the said order by reason of such delay;
when management asked the workers to indicate in the time sheet the
reason for their failure to do so, two of the three workers, namely, Visca
and Tuting gave the same reason, to wit: "Ayaw nila/ng iba na mag-OT

556
Page 557

[overtime] ako" ; the management then conducted an investigation and


a hearing where Visca affirmed his previous claim that petitioners were
the ones who pressured him to desist from rendering overtime work;
Tuting executed a written statement claiming that herein petitioners
induced or threatened them not to work overtime; the management
then gave notices to petitioners asking them to explain why no
disciplinary action would be taken against them; petitioners submitted
their respective explanations denying their liability; after evaluation, the
management informed petitioners that it has decided to terminate
petitioners' employment on the ground that they instigated an illegal
concerted activity resulting in losses to the company.

Petitioners denied the allegations of Polyson contending that they


were terminated from their employment because they established a
union which sought to become the exclusive bargaining agent of the
rank-and-file employees of Polyson; and that Polyson is guilty of unfair
labor practice.

Obrero filed a Notice of Strike with the National Conciliation and


Mediation Board (NCMB) which was predicated on various grounds,
among which was the alleged illegal dismissal of herein petitioners.
The DOLE Secretary certified the labor dispute to the NLRC for
immediate compulsory arbitration where the parties were required to
maintain the status quo, in accordance with Article 263 (g) of the Labor
Code.

ISSUE:
Whether there was an illegal strike

RULING:
YES. Petitioners are guilty of instigating their co-employees to
commit slowdown, an inherently and essentially illegal activity even in
the absence of a no-strike clause in a collective bargaining contract, or
statute or rule.

Nothing in the law requires that a slowdown be carefully planned


and that it be participated in by a large number of workers. The
essence of this kind of strike is that the workers do not quit their work
but simply reduce the rate of work in order to restrict the output or delay
the production of the employer. It is not necessary that any fixed
number of employees should quit their work in order to constitute the
stoppage a strike, and the number of persons necessary depends in
each case on the peculiar facts in the case and no definite rule can be
laid down. As discussed above, petitioners engaged in slowdown when
they induced two of their co-workers to quit their scheduled overtime
work and they accomplished their purpose when the slowdown
resulted in the delay and restriction in the output of Polyson.

557
Page 558

Case Digests by: JACABA, JOYCE B.

BIGG’S INC. V. BONCACAS


G.R. No. 200487. March 6, 2019.
CAGUIOA, J.

DOCTRINE:
REQUIREMENTS OF STRIKE BASED ON UNFAIR LABOR
PRACTICE TO BE VALID
In a strike grounded on unfair labor practice, the following are the
requirements: (1) the strike may be declared by the duly certified
bargaining agent or legitimate labor organization; (2) the conduct of the
strike vote in accordance with the notice and reportorial requirements
to the NCMB and subject to the seven-day waiting period; (3) notice of
strike filed with the NCMB and copy furnished to the employer, subject
to the 15-day cooling-off period. In cases of union busting, the 15-day
cooling-off period shall not apply.

FACTS:
Bigg's, Inc. (Bigg's) was the employer of union members. They are
represented by their union president Boncacas. Both parties have
contrasting versions of the incidents leading to the conflict between the
Bigg's management and the union members.

Bigg's alleges that around 50 union members staged an illegal "sit-


down strike" in Bigg's restaurant. The union did not comply with the
requirements of sending Notice of Strike to the National Conciliation
and Mediation Board (NCMB). Neither did the union obtain the "strike
vote" from its members. Moreover, the union belatedly filed a Notice of
Strike with the NCMB to conceal the illegality of the sit-down strike.
Bigg's issued a memorandum to the striking union members requiring
them to explain their actions within 24 hours from notice. The union
members did not comply with the company's order. Thus, they were
sent employment termination letters.

On the other hand, the union members accuse Bigg's of interfering


with union activities. Allegedly, union members were asked to withdraw
their membership under threat of losing their employment. Employees
Mariano Aycardo and Marilyn Jana were dismissed from service
purportedly due to their union membership. On the day of the alleged
sit-down strike, Boncacas and other union members were prevented
from entering the premises of Bigg's. On the same day, they filed a
Notice of Strike with the NCMB. They attempted to return to work, but
they were informed to obtain their respective memoranda. The
memoranda informed them of their suspension from work for
participating in a sit-down strike.

558
Page 559

The union members filed a complaint before the NCMB for unfair
labor practices, illegal dismissal, and damages. Bigg's also filed a
complaint before the NCMB for illegal strike against the union
members. The two complaints were consolidated and the NCMB
conducted mediation proceedings. When mediation reached an
impasse, the union conducted another strike.

ISSUE:
Whether the strikes were legal

RULING:
NO. The Labor Code and the IRR limit the grounds for a valid strike
to: (1) a bargaining deadlock in the course of collective bargaining, or
(2) the conduct of unfair labor practices by the employer. Only a
certified or duly recognized bargaining representative may declare a
strike in case of a bargaining deadlock. However, in cases of unfair
labor practices, the strike may be declared by any legitimate labor
organization.

In both instances, the union must conduct a "strike vote" which


requires that the actual strike is approved by majority of the total union
membership in the bargaining unit concerned. The union is required to
notify the regional branch of the NCMB of the conduct of the strike vote
at least 24 hours before the conduct of the voting. Thereafter, the union
must furnish the NCMB with the results of the voting at least seven
days before the intended strike or lockout. This seven-day period has
been referred to as the "seven-day strike ban" or "seven-day waiting
period."

In a strike grounded on unfair labor practice, the following are the


requirements: (1) the strike may be declared by the duly certified
bargaining agent or legitimate labor organization; (2) the conduct of the
strike vote in accordance with the notice and reportorial requirements
to the NCMB and subject to the seven-day waiting period; (3) notice of
strike filed with the NCMB and copy furnished to the employer, subject
to the 15-day cooling-off period. In cases of union busting, the 15-day
cooling-off period shall not apply.

The union did not file the requisite Notice of Strike and failed to
observe the cooling-off period. In an effort to legitimize the strike, the
union filed a Notice of Strike on the same day. This cannot be
considered as compliance with the requirement, as the cooling-off
period is mandatory. The cooling-off period is not merely a period
during which the union and the employer must simply wait. The
purpose of the cooling-off period is to allow the parties to negotiate and
seek a peaceful settlement of their dispute to prevent the actual

559
Page 560

conduct of the strike. In other words, there must be genuine efforts to


amicably resolve the dispute.

The union failed to prove with substantial evidence that Bigg's was
guilty of unfair labor practice as defined under Article 259 of the Labor
Code to allow the union, a non-certified bargaining agent to initiate the
strike. Likewise, the union failed to prove that there was union busting
to exempt compliance with the cooling-off period. The union did not
present any substantial evidence to prove its allegations that union
members were actually dismissed or threatened with dismissal for their
union membership. In fine, the union's failure to comply with the
mandatory requirements rendered the strike on illegal.

560
Page 561

Case Digests by: JACABA, JOYCE B.

ST. LUKE’S MEDICAL CENTER, INC. V. NOTARIO


G.R. No. 152166. October 20, 2010.
PERALTA, J.

DOCTRINE:
GROSS AND HABITUAL NEGLECT OF DUTIES AS A GROUND
FOR DISMISSAL FROM EMPLOYMENT
A single or isolated act of negligence does not constitute a just
cause for the dismissal of the employee.

FACTS:
St. Luke's Medical Center, Inc. (petitioner hospital) employed
respondent as In-House Security Guard. Respondent was on duty from
6:00 p.m. to 6:00 a.m. of the following day. His work consisted mainly
of monitoring the video cameras. In the evening of December 30, 1996,
Justin Tibon, a foreigner from Majuro, Marshall Island, then attending
to his 3-year-old daughter, Andanie De Brum, who was admitted,
reported to the management of petitioner hospital about the loss of his
mint green traveling bag, which was placed inside the cabinet.

The Security Department of petitioner hospital conducted an


investigation. When the tapes of video camera recorder (VCR) no. 3
covering the subject period were reviewed, it was shown that the VCR
was focused on camera no. 2 (Old Maternity Unit), from 2103H to
2215H [or 9:03 p.m. to 10:15 p.m.] of December 30, 1996, and camera
no. 1 (New Maternity Unit), from 0025H to 0600H [or 12:25 a.m. to 6:00
a.m.] of December 31, 1996. The cameras failed to record any incident
of theft at room 257.

Petitioner hospital issued a Memorandum to respondent, the


CCTV monitoring staff on duty, directing him to explain in writing, within
24 hours upon receipt thereof, why no disciplinary action should be
taken against him for violating the normal rotation/sequencing process
of the VCR and, consequently, failed to capture the theft of Tibon's
traveling bag at room 257.

Respondent explained that on the subject dates, he was the only


personnel on duty as nobody wanted to assist him. Because of this, he
decided to focus the cameras on the Old and New Maternity Units, as
these two units have high incidence of crime.

561
Page 562

Finding the written explanation of respondent to be unsatisfactory,


petitioner hospital served on respondent a copy of the Notice of
Termination dismissing him on the ground of gross
negligence/inefficiency under Section 1, Rule VII of its Code of
Discipline.

Respondent filed a Complaint for illegal dismissal against


petitioner hospital and its Chairman.

ISSUE:
Whether the respondent was illegally dismissed

RULING:
YES. To effectuate a valid dismissal from employment by the
employer, the Labor Code has set twin requirements, namely: (1) the
dismissal must be for any of the causes provided in Article 282 of the
Labor Code; and (2) the employee must be given an opportunity to be
heard and defend himself.

Under Article 282 (b) of the Labor Code, an employer may


terminate an employee for gross and habitual neglect of duties.
Neglect of duty, to be a ground for dismissal, must be both gross and
habitual. Gross negligence connotes want of care in the performance
of one's duties. Habitual neglect implies repeated failure to perform
one's duties for a period of time, depending upon the circumstances.
A single or isolated act of negligence does not constitute a just cause
for the dismissal of the employee. Under the prevailing circumstances,
respondent exercised his best judgment in monitoring the CCTV
cameras so as to ensure the security within the hospital premises.
Verily, assuming arguendo that respondent was negligent, although
this Court finds otherwise, the lapse or inaction could only be regarded
as a single or isolated act of negligence that cannot be categorized as
habitual and, hence, not a just cause for his dismissal.

The employee must be furnished two written notices: the first


notice apprises the employee of the particular acts or omissions for
which his dismissal is sought, and the second is a subsequent notice,
which informs the employee of the employer's decision to dismiss him.
The petitioner hospital failed to comply with the rule on twin notice and
hearing as it merely required respondent to give his written explanation
within 24 hours and, thereafter, ordered his dismissal.

562
Page 563

Case Digests by: JACABA, JOYCE B.

ALILING V. WORLD EXPRESS CORPORATION


G.R. No. 185829. April 25, 2012.
VELASCO, JR., J.

DOCTRINE:
SUBSTANTIVE AND PROCEDURAL DUE PROCESS AS A
REQUIREMENT FOR VALID DISMISSAL
To justify fully the dismissal of an employee, the employer must,
as a rule, prove that the dismissal was for a just cause and that the
employee was afforded due process prior to dismissal.

FACTS:
Respondent Wide Wide World Express Corporation (WWWEC)
offered to employ petitioner Armando Aliling (Aliling) as "Account
Executive (Seafreight Sales)". The offer came with a six (6)-month
probation period condition with this express caveat: "Performance
during [sic] probationary period shall be made as basis for confirmation
to Regular or Permanent Status."

Training then started. Instead of a Seafreight Sale assignment,


WWWEC asked Aliling to handle Ground Express (GX). Barely a
month after, Manuel F. San Mateo III (San Mateo), WWWEC Sales
and Marketing Director, emailed Aliling to express dissatisfaction with
the latter's performance.

Lariosa, Human Resources Manager of WWWEC, asked Aliling to


report to the Human Resources Department to explain his absence
taken without leave. Aliling denied being absent on the days in
question, attaching to his reply-letter a copy of his timesheet which
showed that he worked.

Aliling wrote San Mateo stating: "I hereby tender my resignation


effective October 15, 2004." While WWWEC took no action on his
tender, Aliling nonetheless demanded reinstatement and a written
apology, claiming in a subsequent letter to management that San
Mateo had forced him to resign.

Lariosa's response-letter informed Aliling that his case was still in


the process of being evaluated. Lariosa again wrote, this time to advise
Aliling of the termination of his services effective as of that date owing
to his "non-satisfactory performance" during his probationary period.

563
Page 564

Records show that Aliling, for the period indicated, was paid his
outstanding salary.

Aliling filed a Complaint for illegal dismissal due to forced


resignation, nonpayment of salaries as well as damages with the
NLRC against WWWEC.

ISSUES:
1. Whether Aliling was a regular employee

2. Whether Aliling was illegally dismissed

RULING:
1. YES. On the basis of documentary evidence adduced, that
respondent WWWEC did not inform petitioner Aliling of the reasonable
standards by which his probation would be measured against at the
time of his engagement.

Contrary to respondents' contention, San Mateo's email cannot


support their allegation on Aliling being informed of the standards for
his continued employment, such as the sales quota, at the time of his
engagement. As it were, the email message was sent to Aliling more
than a month after he signed his employment contract with WWWEC.
The aforequoted Section 6 of the Implementing Rules of Book VI, Rule
VIII-A of the Code specifically requires the employer to inform the
probationary employee of such reasonable standards at the time of his
engagement, not at any time later; else, the latter shall be considered
a regular employee. Thus, pursuant to the explicit provision of Article
281 of the Labor Code, Section 6 (d) of the Implementing Rules of Book
VI, Rule VIIIA of the Labor Code and settled jurisprudence, petitioner
Aliling is deemed a regular employee as of June 11, 2004, the date of
his employment contract.

2. YES. To justify fully the dismissal of an employee, the


employer must, as a rule, prove that the dismissal was for a just cause
and that the employee was afforded due process prior to dismissal.

ON SUBSTANTIVE DUE PROCESS: An employee's failure to


meet sales or work quotas falls under the concept of gross inefficiency,
which in turn is analogous to gross neglect of duty that is a just cause
for dismissal under Article 282 of the Code. However, in order for the
quota imposed to be considered a valid productivity standard and
thereby validate a dismissal, management's prerogative of fixing the
quota must be exercised in good faith for the advancement of its
interest. The duty to prove good faith, however, rests with WWWEC as
part of its burden to show that the dismissal was for a just cause.
WWWEC must show that such quota was imposed in good faith. This

564
Page 565

WWWEC failed to do, perceptibly because it could not. The fact of the
matter is that the alleged imposition of the quota was a desperate
attempt to lend a semblance of validity to Aliling's illegal dismissal.

ON PROCEDURAL DUE PROCESS: When the Labor Code


speaks of procedural due process, the reference is usually to the two
(2)-written notice rule envisaged in Section 2 (III), Rule XXIII, Book V
of the Omnibus Rules Implementing the Labor Code.

The adverted memo of WWWEC supposedly informing Aliling of


the likelihood of his termination and directing him to account for his
failure to meet the expected job performance would have had
constituted the "charge sheet," sufficient to answer for the first notice
requirement, but for the fact that there is no proof such letter had been
sent to and received by him. In fact, in his Complainant's Reply
Affidavit, Aliling goes on to tag such letter/memorandum as fabrication.
WWWEC did not adduce proof to show that a copy of the letter was
duly served upon Aliling. Clearly enough, WWWEC did not comply with
the first notice requirement.

Neither was there compliance with the imperatives of a hearing or


conference. The Court need not dwell at length on this particular
breach of the due procedural requirement. Suffice it to point out that
the record is devoid of any showing of a hearing or conference having
been conducted.

On the contrary, barely five (5) days after it served the notice of
termination, WWWEC acknowledged that it was still evaluating his
case. And the written notice of termination itself did not indicate all the
circumstances involving the charge to justify severance of
employment.

565
Page 566

Case Digests by: NG, KRISTOFFER MONICO S.

PEREZ v. PHILIPPINE TELEGRAPH AND TELEPHONE


G.R. No. 152048. April 7, 2009.
CORONA, J.

DOCTRINE:
RIGHT AGAINST ILLEGAL DISMISSAL
It is a cardinal rule in our jurisdiction that the employer must
furnish the employee with two written notices before the termination of
employment can be effected: (1) the first apprises the employee of the
particular acts or omissions for which his dismissal is sought; and (2)
the second informs the employee of the employer’s decision to dismiss
him.

FACTS:
Petitioners were employed in the PT&T Shipping Section by
respondent. Later, it was discovered that the shipping section showed
that the shipping documents have traces of tampering and alteration.
Thus, petitioners were placed on preventive suspension for 30 days for
the alleged incident. Further, their suspension was extended for 15
days twice.

A memorandum was then given informing the petitioners that


were being dismissed from service for having falsified company
documents. This prompted them to file a complaint for illegal
suspension and illegal dismissal. The LA ruled in favor of the
petitioners; while the NLRC reversed the decision.

ISSUE:
WON petitioners were dismissed for just cause and with the
observance of due process.

RULING:
NO. Respondents were not able to sufficiently establish the facts
from which the loss of confidence resulted. Other that the allegations
that such documents passed through the petitioners’ hands,
respondents did not have any evidence that there was indeed
alterations. The alterations on the shipping documents could not
reasonably be attributed to petitioners because it was never proven
that petitioners alone had control of or access to these documents.

566
Page 567

Respondents’ illegal act of dismissal was further aggravated by


failure to observe due process. It is a cardinal rule in our jurisdiction
that the employer must furnish the employee with two written notices
before the termination of employment can be effected: (1) the first
apprises the employee of the particular acts or omissions for which his
dismissal is sought; and (2) the second informs the employee of the
employer’s decision to dismiss him. The requirement of a hearing is
complied with as long as there was an opportunity to be heard, and not
necessarily that an actual hearing was conducted.

The court noted the conflict between the Labor Code and IRR.
The LC provides “ample opportunity to be heard” while the IRR
“requires a hearing and conference” Even if no hearing was conducted,
the requirement of due process had been met since they were
accorded a chance to explain their side of the controversy. The law
prevails over IRR.

Petitioners were neither apprised of the charges against them nor


given a chance to defend themselves. They were simply and arbitrarily
separated from work and served notices of termination.

567
Page 568

Case Digests by: NG, KRISTOFFER MONICO S.

DISTRIBUTION AND CONTROL PRODUCTS v. SANTOS


G.R. No. 212616. July 10, 2017.
PERALTA, J.

DOCTRINE:
TWIN REQUIREMENTS OF NOTICE AND HEARING
The employer must furnish the employee with two written notices
before the termination of employment can be effected: (1) the first
apprises the employee of the particular acts or omissions for which his
dismissal is sought; and (2) the second informs the employee of the
employer's decision to dismiss him. The requirement of a hearing is
complied with as long as there was an opportunity to be heard, and not
necessarily that an actual hearing was conducted.

FACTS:
The case stemmed from the filling of the complaint for
constructive dismissal and payment of separation pay by the
respondent. He alleged that he received a notice placing him under
preventive suspension for allegedly participating in the unlawful taking
of circuit breakers and electrical products. Further, he was never given
an opportunity to explain before he was suspended. He was later
dismissed.

ISSUE:
WON respondent was validly dismissed for loss of trust and
confidence

RULING:
NO. In order for the employer to properly invoke this ground,
the employer must satisfy two conditions.

First, the employer must show that the employee concerned


holds a position of trust and confidence. Jurisprudence provides for
two classes of positions of trust. The first class consists of managerial
employees, or those who, by the nature of their position, are entrusted
with confidential and delicate matters and from whom greater fidelity to
duty is correspondingly expected. The second class includes
"cashiers, auditors, property custodians, or those who, in the normal
and routine exercise of their functions, regularly handle significant
amounts of [the employer's] money or property."

568
Page 569

Second, the employer must establish the existence of an act


justifying the loss of trust and confidence. To be a valid cause for
dismissal, the act that betrays the employer's trust must be real, i.e.,
founded on clearly established facts, and the employee's breach of the
trust must be willful, i.e., it was done intentionally, knowingly and
purposely, without justifiable excuse. Moreover, with respect to rank-
and-file personnel, loss of trust and confidence, as ground for valid
dismissal, requires proof of involvement in the alleged events in
question, and that mere uncorroborated assertions and accusations by
the employer will not be sufficient.

As to whether or not respondent was afforded procedural due


process, the settled rule is that in termination proceedings of
employees, procedural due process consists of the twin requirements
of notice and hearing. The employer must furnish the employee with
two written notices before the termination of employment can be
effected: (1) the first apprises the employee of the particular acts or
omissions for which his dismissal is sought; and (2) the second informs
the employee of the employer's decision to dismiss him. The
requirement of a hearing is complied with as long as there was an
opportunity to be heard, and not necessarily that an actual hearing was
conducted.

The only notice given by petitioners to respondent was the notice


of his 30-day preventive suspension and, as found by the LA, nothing
therein indicated that he was required nor was given the opportunity to
explain his side, considering that he was being implicated in the theft
of the subject circuit breakers and other electrical products. It is true
that petitioners conducted their own investigation but the same was
made without the participation of respondent.

569
Page 570

Case Digests by: NG, KRISTOFFER MONICO S.

AGABON v. NLRC
G.R. No. 158693. November 17, 2004.
YNARES-SANTIAGO, J.

DOCTRINE:
PROCEDURAL DUE PROCESS
Procedurally, (1) if the dismissal is based on a just cause, the
employer must give the employee two written notices and a hearing or
opportunity to be heard if requested by the employee before
terminating the employment: a notice specifying the grounds for which
dismissal is sought a hearing or an opportunity to be heard and after
hearing or opportunity to be heard, a notice of the decision to dismiss;
and (2) if the dismissal is based on authorized causes, the employer
must give the employee and the DOLE written notices 30 days prior to
the effectivity of his separation.

FACTS:
Private respondent Riviera Home employed petitioners as
gypsum board and cornice installers until they were subsequently
dismissed for abandonment of work. Petitioners filed a complaint for
illegal dismissal. The LA found petitioners to be illegally dismissed.
While the NLRC & CA reversed the decision. They were not illegally
dismissed because they abandoned their employment.

ISSUE:
WON respondent were illegally dismissed

RULING:
To dismiss an employee, the law requires not only the existence
of a just and valid cause but also enjoins the employer to give the
employee the opportunity to be heard and to defend himself.

Abandonment is the deliberate and unjustified refusal of an


employee to resume his employment. It is a form of neglect of duty,
hence, a just cause for termination of employment by the employer.

For a valid finding of abandonment, these two factors should be


present: (1) the failure to report for work or absence without valid or
justifiable reason; and (2) a clear intention to sever employer-employee
relationship, with the second as the more determinative factor which is
manifested by overt acts from which it may be deduced that the
employees has no more intention to work. The intent to discontinue the

570
Page 571

employment must be shown by clear proof that it was deliberate and


unjustified.

In this case, petitioners were frequently absent having


subcontracted for an installation work for another company.
Subcontracting for another company clearly showed the intention to
sever the employer-employee relationship with private respondent.

Procedurally, (1) if the dismissal is based on a just cause, the


employer must give the employee two written notices and a hearing or
opportunity to be heard if requested by the employee before
terminating the employment: a notice specifying the grounds for which
dismissal is sought a hearing or an opportunity to be heard and after
hearing or opportunity to be heard, a notice of the decision to dismiss;
and (2) if the dismissal is based on authorized causes, the employer
must give the employee and the DOLE written notices 30 days prior to
the effectivity of his separation.

In this case, the dismissal is for just cause but due process was
not observed. While the procedural infirmity cannot be cured, it should
not invalidate the dismissal. Private respondent is subjected to pay an
indemnity of nominal damages amounting to 30,000.

571
Page 572

Case Digests by: NG, KRISTOFFER MONICO S.

KING OF KINGS TRANSPORT v. MAMAC


G.R. No. 166208. June 29, 2007.
VELASCO, JR., J.

DOCTRINE:
DUE PROCESS
For termination on just causes, the following must be observed:
(a) written notice served on the employee specifying the grounds for
termination giving him reasonable opportunity to explain his side; (b) a
hearing or conference; (c) written notice of termination served on the
employee indicating the grounds have been established to justify his
termination.

FACTS:
Respondent was a conductor for DMTC and was one of the few
people who organized DMTC Workers’ Union Pending the certification
election, respondent was transferred to petitioner company (KKTI).
The KKTI later establish a union to which respondent was elected
president.

Upon audit of the conductor’s report of respondent, KKTI noted


an irregularity which caused a loss of income totaling to 800 pesos.
While no irregularity report was given, KKTI asked the respondent to
explain the discrepancy. Subsequently, respondent received a letter
terminating his employment due to the alleged irregularity as fraud.
This prompted the respondent to filed for illegal dismissal

ISSUE:
WON verbal appraisal of charges against an employee constituted
a breach of procedural due process

RULING:
YES. Due process under the Labor Code involves two aspects:
first, substantive––the valid and authorized causes of termination of
employment under the Labor Code; and second, procedural––the
manner of dismissal.

For termination on just causes, the following must be observed:


(a) written notice served on the employee specifying the grounds for
termination giving him reasonable opportunity to explain his side; (b) a
hearing or conference; (c) written notice of termination served on the

572
Page 573

employee indicating the grounds have been established to justify his


termination.

Respondent was not issued a written notice charging him of


committing an infraction. The law is clear on the matter. A verbal
appraisal of the charges against an employee does not comply with
the first notice requirement. Jurisprudence states that consultations or
conferences are not a substitute for the actual observance of notice
and hearing.

No hearing was conducted. Regardless of respondent’s written


explanation, a hearing was still necessary in order for him to clarify and
present evidence in support of his defense.

573
Page 574

Case Digests by: NG, KRISTOFFER MONICO S.

PUNCIA v. TOYOTA SHAW


G.R. No. 214399. June 28, 2016.
PERLAS-BERNABE., J.

DOCTRINE:
PROCEDURAL DUE PROCESS
As to the procedural due process, the following must be
observed: (1) the first written notice to be served on the employees
should contain the specific causes or grounds for termination against
them; (2) the employers should schedule and conduct a hearing or
conference for the employees to be given the opportunity to respond
to the charge; and (3) a written notice of termination served on the
employee indicating that upon due consideration of all the
circumstances, grounds have been established to justify his
termination.

FACTS:
Puncia was hired as a marketing professional for Toyota with a
monthly sales quota of 7 vehicles. As he was having trouble complying
with the quota, Toyota extended him leniency by lowering his monthly
sales quota to 3 vehicles. But still, Puncia failed to comply. It was
shown that Puncia failed in satisfying his monthly sales quota, only
selling 5 vehicles out of 34 he was required to sell over the said period.

Toyota sent a notice to explain and after hearing was conducted,


Puncia failed to appear despite notice. Toyota then sent a notice of
termination dismissing him on the ground of insubordination. Puncia,
in turn, filed a complaint for illegal dismissal.

ISSUE:
WON Puncia was dismissed from employment for just cause

RULING:
YES. But Toyota failed to comply with the procedural due
process. It is settled that "for a dismissal to be valid, the rule is that the
employer must comply with both substantive and procedural due
process requirements. Substantive due process requires that the
dismissal must be pursuant to either a just or an authorized cause of
the LC. Procedural due process mandates that the employer must
observe the twin requirements of notice and hearing before a dismissal
can be effected.

574
Page 575

Jurisprudence dictates that an employer in entitled to impose


productivity standards for its employees, and the latter’s non-
compliance therewith can lead to his termination from work. Thus,
Toyota complied with the substantive due process as there was indeed
just cause for Puncia’s termination.

As to the procedural due process, the following must be


observed: (1) the first written notice to be served on the employees
should contain the specific causes or grounds for termination against
them; (2) the employers should schedule and conduct a hearing or
conference for the employees to be given the opportunity to respond
to the charge; and (3) a written notice of termination served on the
employee indicating that upon due consideration of all the
circumstances, grounds have been established to justify his
termination.

Toyota FAILED to comply with the procedural requirements


because records show that the notice of termination shows that Puncia
was dismissed not for the ground stated in the notice to explain (gross
inefficiency) but for gross insubordination. Toyota was directed to pay
30,000 nominal damages

575
Page 576

Case Digests by: NG, KRISTOFFER MONICO S.

JAKA FOOD PROCESSING v. PACOT


G.R. NO. 151378. March 28, 2005
GARCIA, J.

DOCTRINE:
REQUIRED DUE PROCESS FOR AUTHORIZED CAUSES
A dismissal for an authorized cause under Art. 298 does not
necessarily imply delinquency or culpability on the part of the
employee. Instead, the dismissal process is initiated by the employer’s
exercise of his management prerogative.

FACTS:
Herein respondents were terminated from their employment due
to financial straits experienced by Jaka. However, the termination was
effected without Jaka complying with the requirement under Art. 298 of
the LC on service of written notice upon the employees and DOLE at
least one month before the intended date of termination.

Respondents filed with the LA a complaint for illegal dismissal.


The LA & NLRC declared that termination was illegal. On the other
hand, the CA reversed the rulings of both tribunal.

ISSUE:
WON the dismissal was valid despite non-compliance with the
notice requirement

RULING:
YES. In this case, respondents were terminated on the ground
of retrenchment, which is one of the authorized causes enumerated
under the LC. But it was established that Jaka failed to comply with the
notice requirement under the same article. The dismissal is, therefore,
valid, but Jaka must pay nominal damages of 50,000.

A dismissal for an authorized cause under Art. 298 does not


necessarily imply delinquency or culpability on the part of the
employee. Instead, the dismissal process is initiated by the employer’s
exercise of his management prerogative.

The difference between Agabon and the instant case is that in


the former, the dismissal was based on a just cause under the Labor
Code while in the present case, respondents were dismissed due to

576
Page 577

retrenchment, which is one of the authorized causes under the same


Code.

The clear-cut distinction between a dismissal for just cause and


a dismissal for authorized cause is further reinforced by the fact that in
the first, payment of separation pay is not required; while in the second,
the law requires payment of separation pay.

577
Page 578

Case Digests by: NG, KRISTOFFER MONICO S.

ABBOTT LABORATORIES v. ALCARAZ


G.R. No. 123456. January 01, 2022.
Peralta, C. J.

DOCTRINE:
CONTRACTUAL DUE PROCESS
As to the termination procedure, a different procedure is applied
when terminating a probationary employee; the usual two-notice rule
does not govern. The IRR provides that if the termination is brought
about by the failure of an employee to meet the standards of the
employer, it shall be sufficient that a written notice is served the
employee, within a reasonable time from the effective date of
termination.

FACTS:
Abbott posted a publication for its need of a Medical and
Regulatory Affairs Manager. Respondent, who is working in another
company, showed interest in the position and submitted her
application.

In Abbott’s offer, it was stated that Alcaraz is on probation. She


then accepted the offer and received an email confirming her
acceptance together with her job description. During her orientation,
she was briefed on her duties and responsibilities and was later sent
an explanation of the procedure for her performance evaluation.
Alcaraz observed that some of the employees had disciplinary
problems. She would then reprimand them for unprofessional behavior
(violation of dress code, etc.) She was considered as too strict.

In a meeting, she was informed that she failed to meet the


regularization standards and that she would be terminated. This
prompted Alcaraz to file a complaint for illegal dismissal claiming that
she is now a regular employee for failure to inform her of the
reasonable standards for her regularization.

ISSUE:
WON Alcaraz was illegally dismissed

RULING:
NO. The services of an employee who has been engaged on
probationary basis may be terminated for any of the following: (a) a just
or authorized cause; and (b) when he fails to qualify as a regular

578
Page 579

employee in accordance with reasonable standards prescribed by the


employer.

As to the termination procedure, a different procedure is applied


when terminating a probationary employee; the usual two-notice rule
does not govern. The IRR provides that if the termination is brought
about by the failure of an employee to meet the standards of the
employer, it shall be sufficient that a written notice is served the
employee, within a reasonable time from the effective date of
termination.

Records show that Alcaraz’s dismissal was effected through a


letter wherein it stated the reason for her termination. This written
notice sufficiently meets the criteria set forth, thereby legitimizing the
cause and manner of the dismissal as a probationary employee under
the LC. Despite the existence of a sufficient ground to terminate
Alcaraz’s employment and Abbott’s compliance with the Labor Code
termination procedure, it is readily apparent that Abbott breached its
contractual obligation to Alcaraz when it failed to abide by its own
procedure in evaluating the performance of a probationary employee.

579
Page 580

Case Digests by: NG, KRISTOFFER MONICO S.

AGABON v. NLRC
G.R. No. 158693. November 17, 2004.
YNARES-SANTIAGO, J.

DOCTRINE:
PROCEDURAL DUE PROCESS
Procedurally, (1) if the dismissal is based on a just cause, the
employer must give the employee two written notices and a hearing or
opportunity to be heard if requested by the employee before
terminating the employment: a notice specifying the grounds for which
dismissal is sought a hearing or an opportunity to be heard and after
hearing or opportunity to be heard, a notice of the decision to dismiss;
and (2) if the dismissal is based on authorized causes, the employer
must give the employee and the DOLE written notices 30 days prior to
the effectivity of his separation.

FACTS:
Private respondent Riviera Home employed petitioners as
gypsum board and cornice installers until they were subsequently
dismissed for abandonment of work. Petitioners filed a complaint for
illegal dismissal. The LA found petitioners to be illegally dismissed.
While the NLRC & CA reversed the decision. They were not illegally
dismissed because they abandoned their employment.

ISSUE:
WON respondent were illegally dismissed

RULING:
To dismiss an employee, the law requires not only the existence
of a just and valid cause but also enjoins the employer to give the
employee the opportunity to be heard and to defend himself.

Abandonment is the deliberate and unjustified refusal of an


employee to resume his employment. It is a form of neglect of duty,
hence, a just cause for termination of employment by the employer.

For a valid finding of abandonment, these two factors should be


present: (1) the failure to report for work or absence without valid or
justifiable reason; and (2) a clear intention to sever employer-employee
relationship, with the second as the more determinative factor which is
manifested by overt acts from which it may be deduced that the
employees has no more intention to work. The intent to discontinue the

580
Page 581

employment must be shown by clear proof that it was deliberate and


unjustified.

In this case, petitioners were frequently absent having


subcontracted for an installation work for another company.
Subcontracting for another company clearly showed the intention to
sever the employer-employee relationship with private respondent.

Procedurally, (1) if the dismissal is based on a just cause, the


employer must give the employee two written notices and a hearing or
opportunity to be heard if requested by the employee before
terminating the employment: a notice specifying the grounds for which
dismissal is sought a hearing or an opportunity to be heard and after
hearing or opportunity to be heard, a notice of the decision to dismiss;
and (2) if the dismissal is based on authorized causes, the employer
must give the employee and the DOLE written notices 30 days prior to
the effectivity of his separation.

In this case, the dismissal is for just cause but due process was
not observed. While the procedural infirmity cannot be cured, it should
not invalidate the dismissal. Private respondent is subjected to pay an
indemnity of nominal damages amounting to 30,000.

581
Page 582

Case Digests by: NG, KRISTOFFER MONICO S.

JAKA FOOD PROCESSING v. PACOT


G.R. NO. 151378. March 28, 2005
GARCIA, J.

DOCTRINE:
REQUIRED DUE PROCESS FOR AUTHORIZED CAUSES
A dismissal for an authorized cause under Art. 298 does not
necessarily imply delinquency or culpability on the part of the
employee. Instead, the dismissal process is initiated by the employer’s
exercise of his management prerogative.

FACTS:
Herein respondents were terminated from their employment due
to financial straits experienced by Jaka. However, the termination was
effected without Jaka complying with the requirement under Art. 298 of
the LC on service of written notice upon the employees and DOLE at
least one month before the intended date of termination.

Respondents filed with the LA a complaint for illegal dismissal.


The LA & NLRC declared that termination was illegal. On the other
hand, the CA reversed the rulings of both tribunal.

ISSUE:
WON the dismissal was valid despite non-compliance with the
notice requirement

RULING:
YES. In this case, respondents were terminated on the ground
of retrenchment, which is one of the authorized causes enumerated
under the LC. But it was established that Jaka failed to comply with the
notice requirement under the same article. The dismissal is, therefore,
valid, but Jaka must pay nominal damages of 50,000.

A dismissal for an authorized cause under Art. 298 does not


necessarily imply delinquency or culpability on the part of the
employee. Instead, the dismissal process is initiated by the employer’s
exercise of his management prerogative.

The difference between Agabon and the instant case is that in


the former, the dismissal was based on a just cause under the Labor
Code while in the present case, respondents were dismissed due to

582
Page 583

retrenchment, which is one of the authorized causes under the same


Code.

The clear-cut distinction between a dismissal for just cause and


a dismissal for authorized cause is further reinforced by the fact that in
the first, payment of separation pay is not required; while in the second,
the law requires payment of separation pay.

583
Page 584

Case Digests by: NG, KRISTOFFER MONICO S.

CULILI v. EASTERN TELECOMMUNICATIONS PHILS


G.R. No. 165381. February 9, 2011.
LEONARDO-DE CASTRO, J.

DOCTRINE:
REDUNDANCY
An employer cannot simply declare that it has become
overmanned and dismiss its employees without producing adequate
proof to sustain its claim of redundancy. Among the requisites of a valid
redundancy program are: (1) the good faith of the employer in
abolishing the redundant position; and (2) fair and reasonable criteria
in ascertaining what positions are to be declared redundant, such as
but not limited to: preferred status, efficiency, and seniority.

FACTS:
Due to business troubles and losses, Eastern
Telecommunication Philippines, Inc. (ETPI) implemented a two-
phased right-sizing program: reduction of ETPI’s workforce and then a
companywide reorganization. ETPI offered a special retirement
program to employees who have been in service for at least 15 years.

Of all qualified employees, only Culili rejected the offer. Another


department absorbed Culili’s functions and his position was abolished
due to redundancy. He was later informed that his employment was
terminated and was dismissed.

ISSUE:
WON Culili’s dimisssal can be considered ULP

RULING:
NO. There is redundancy when the service capability of the
workforce is greater that what is reasonably required to meet the
demands of the business enterprise. This determination properly
belongs to the employer. However, there must be good faith of the
employer and fair and reasonable criteria in ascertaining what
positions are to be declared redundancy.

However, an employer cannot simply declare that it has become


overmanned and dismiss its employees without producing adequate
proof to sustain its claim of redundancy. Among the requisites of a valid
redundancy program are: (1) the good faith of the employer in
abolishing the redundant position; and (2) fair and reasonable criteria

584
Page 585

in ascertaining what positions are to be declared redundant, such as


but not limited to: preferred status, efficiency, and seniority.

This Court also held that the following evidence may be proffered
to substantiate redundancy: the new staffing pattern, feasibility studies/
proposal on the viability of the newly created positions, job description
and the approval by the management of the restructuring.

There is no showing that ETPI, in implementing its program was


motivated by ill-will, bad faith or malice or was aimed at interfering with
its employees’ right to self-organization.

585
Page 586

Case Digests by: NG, KRISTOFFER MONICO S.

SERRANO v. GALLANT MARITIME


G.R. No. 167614. March 24, 2009.
AUSTRIA-MARTINEZ, J.

DOCTRINE:
MONETARY BENEFITS IN CASE OF ILLEGAL DISMISSAL
All OFWs, regardless of contract periods or the unexpired portions
thereof, were treated alike in terms of the computation of their
monetary benefits in case of illegal dismissal. Their claims were
subjected to a uniform rule of computation: their basic salaries
multiplied by the entire unexpired portion of their employment
contracts.

FACTS:
Petitioner, a Filipino seafarer, was hired by Gallant Maritime
under a POEA-approved Contract of Employment.

During the petitioner’s departure for work, he was constrained to


accept a downgraded employment contract for the position of Second
Officer with a monthly salary of US$1,000.00, upon the assurance and
representation of respondents that he would be made Chief Officer by
the end of April 1998. The downgrade is contrary to the Chief Officer
post that he initially signed up for. (with higher salary rate).

Respondents did not deliver on their promise to make petitioner


Chief Officer. Hence, petitioner refused to stay on as Second Officer
and was repatriated to the Philippines on May 26, 1998.

Petitioner’s employment contract was for a period of 12 months,


but at the time of his repatriation he had served only two (2) months
and seven (7) days of his contract, leaving an unexpired portion of nine
(9) months and twenty-three (23) days. He then filed with the Labor
Arbiter (LA) a Complaint against respondents for constructive
dismissal and for payment of his money claims in the total amount of
US$26,442.7.

ISSUE:
WON the lump-salary should be awarded by reason of illegal
dismissal

RULING:

586
Page 587

NO. It is plain that prior to R.A. No. 8042, all OFWs, regardless
of contract periods or the unexpired portions thereof, were treated alike
in terms of the computation of their monetary benefits in case of illegal
dismissal. Their claims were subjected to a uniform rule of
computation: their basic salaries multiplied by the entire unexpired
portion of their employment contracts.

The enactment of the subject clause in R.A. No. 8042 introduced


a differentiated rule of computation of the money claims of illegally
dismissed OFWs based on their employment periods, in the process
singling out one category whose contracts have an unexpired portion
of one year or more and subjecting them to the peculiar disadvantage
of having their monetary awards limited to their salaries for 3 months
or for the unexpired portion thereof, whichever is less, but all the while
sparing the other category from such prejudice, simply because the
latter’s unexpired contracts fall short of one year.

Prior to R.A. No. 8042, a uniform system of computation of the


monetary awards of illegally dismissed OFWs was in place.
This uniform system was applicable even to local workers with fixed-
term employment.

The subject clause does not state or imply any definitive


governmental purpose; and it is for that precise reason that the clause
violates not just petitioner’s right to equal protection, but also her right
to substantive due process under Section 1, Article III of the
Constitution.

The subject clause being unconstitutional, petitioner is entitled to


his salaries for the entire unexpired period of nine months and 23 days
of his employment contract, pursuant to law and jurisprudence prior to
the enactment of R.A. No. 8042.

587
Page 588

Case Digests by: NG, KRISTOFFER MONICO S.

YAP v. THENAMARIS SHIP’S MANAGEMENT


G.R. No. 179532. May 30, 2011
NACHURA, J.

DOCTRINE:
SECTION 10 OF RA 8042
The Court has already declared in the case of Serrano that the
clause “salaries for the unexpired portion of his employment contract
for three months for every year of the unexpired term, whichever is
less” is unconstitutional for being violative of the rights of the OFWs to
equal protection of the laws.

FACTS:
Yap was employed as an electrician with a contract period of 12
months. Three months later, the vessel was sold. Yap received his
seniority bonus, vacation bonus, extra bonus along with scrapping
bonus. However, he insisted that he was entitle to the payment of the
unexpired portion of his contract since he was illegally dismissed from
employment. He alleged that he opted for immediate transfer but none
was made. This prompted Yap to file a complaint for illegal dismissal.
Respondent invoked R.A. 8042

ISSUE:
WON Section 10 of R.A. 8042 is constitutional

RULING:
NO. It is undisputed that Yap was indeed illegally dismissed. too,
the Court has already declared in the case of Serrano that the clause
“salaries for the unexpired portion of his employment contract for three
months for every year of the unexpired term, whichever is less” is
unconstitutional for being violative of the rights of the OFWs to equal
protection of the laws.

Moreover, this Court held therein that the subject clause does
not state or imply any definitive governmental purpose; hence, the
same violates not just therein petitioner’s right to equal protection, but
also his right to substantive due process under Section 1, Article III of
the Constitution. Consequently, petitioner therein was accorded his
salaries for the entire unexpired period of nine months and 23 days of
his employment contract, pursuant to law and jurisprudence prior to
the enactment of R.A. No. 8042.

588
Page 589

Following Serrano, we hold that this case should not be included


in the aforementioned exception. After all, it was not the fault of
petitioner that he lost his job due to an act of illegal dismissal committed
by respondents. To rule otherwise would be iniquitous to petitioner and
other OFWs, and would, in effect, send a wrong signal that
principals/employers and recruitment/manning agencies may violate
an OFW’s security of tenure which an employment contract embodies
and actually profit from such violation based on an unconstitutional
provision of law.

589
Page 590

Case Digests by: NG, KRISTOFFER MONICO S.

BANK OF LUBAO V. MANABAT


G.R. No. 188722. February 1, 2012
REYES, J.

DOCTRINE:
DOCTRINE OF STRAINED RELATIONS
Under the law and prevailing jurisprudence, an illegally dismissed
employee is entitled to reinstatement as a matter of right. However, if
reinstatement would only exacerbate the tension and strained relations
between the parties, or where the relationship between the employer
and the employee has been unduly strained by reason of their
irreconcilable differences, particularly where the illegally dismissed
employee held a managerial or key position in the company, it would
be more prudent to order payment of separation pay instead of
reinstatement.

FACTS:
Manabat was hired by petitioner Bank of Lubao, as an encoder,
which he manned together with two other employees, one of them is
teller Lingad.

An initial audit conducted by the petitioner revealed that there


was a misappropriation of funds in the amount of P3,000,000.00, more
or less. Apparently, there were transactions entered and posted in the
passbooks of the clients but were not entered in the bank’s book of
accounts. Further audit showed that there were various deposits which
were entered in the bank’s computer but were subsequently reversed
and marked as “error in posting”.

An administrative hearing was conducted by the bank’s


investigating committee where the respondent was further made to
explain his side, which concluded that the respondent conspired with
Lingad in making fraudulent entries disguised as error corrections in
the bank’s computer.

ISSUE:
WON the doctrine of strained relations be applied in this case

RULING:
YES. At the outset, it should be stressed that a determination of
the applicability of the doctrine of strained relations is essentially a
factual question and, thus, not a proper subject in the instant petition.

590
Page 591

The well-entrenched rule in our jurisdiction is that only questions


of law may be entertained by this Court in a petition for review on
certiorari. This rule, however, is not ironclad and admits certain
exceptions, such as when, inter alia, the findings of fact are conflicting.

Here, in view of the conflicting findings of the NLRC and the CA,
this Court is constrained to pass upon the propriety of the application
of the doctrine of strained relations to justify the award of separation
pay to the respondent in lieu of reinstatement.

Under the law and prevailing jurisprudence, an illegally dismissed


employee is entitled to reinstatement as a matter of right. However, if
reinstatement would only exacerbate the tension and strained relations
between the parties, or where the relationship between the employer
and the employee has been unduly strained by reason of their
irreconcilable differences, particularly where the illegally dismissed
employee held a managerial or key position in the company, it would
be more prudent to order payment of separation pay instead of
reinstatement.

Under the doctrine of strained relations, the payment of separation


pay is considered an acceptable alternative to reinstatement when the
latter option is no longer desirable or viable. On one hand, such
payment liberates the employee from what could be a highly
oppressive work environment. On the other hand, it releases the
employer from the grossly unpalatable obligation of maintaining in its
employ a worker it could no longer trust.

591
Page 592

Case Digests by: NG, KRISTOFFER MONICO S.

ST. MARY’S ACADEMY v. PALACIO


G.R. No. 164913. September 8, 2010
DEL CASTILLO, J.

DOCTRINE:
LIMITED BACKWAGES
It is incumbent upon this Court to afford full protection to labor.
Thus, while we take cognizance of the employer's right to protect its
interest, the same should be exercised in a manner which does not
infringe on the workers' right to security of tenure. "Under the policy of
social justice, the law bends over backward to accommodate the
interests of the working class on the humane justification that those
with less privilege in life should have more in law.

FACTS:
Petitioner hired respondents as teachers, and respondent
Palacio as guidance counselor. In accordance to DECS Memorandum
No. 10, S. 1998 pursuant to RA 7836, the Petitioner informed the
respondents that they cannot be re-accepted for the school year 2000-
2001 for not having passed the LET (Licensure Examinations for
Teachers), nor can they continue with their teaching profession.

They filed a complaint contesting that their termination as highly


irregular and premature. They averred their right to security of tenure
despite the requirements set by the PRC for they had special permits
to teach and the civil service eligibility required under the law. In
addition to this, the deadline for teachers to register under the
Memorandum was set to 19 September 2000, but the petitioner
decided to terminate them as early as 31 March 2000. Lastly, the
acceptance of the Petitioner of other teacher who do not also possess
the required eligibility under the Memorandum showed evident bad
faith. The LA, NLRC and CA found for the respondents ruling for the
illegality of their dismissal.

ISSUE:
WON the CA erred in holding that the dismissal of respondents
were premature

RULING:
NO. The dismissal was premature and defeated their right to
security of tenure except for one respondent whose dismissal has legal

592
Page 593

basis for lack of the required qualification needed for continued


practice of teaching.

It is incumbent upon the Court to afford full protection to labor. The


law has provided a specific timeframe within which respondents could
comply, petitioner has no right to deny them of this privilege accorded
to them by law. Insofar as Palacio, Calibod, Laquio, Santander and
Montedramos are concerned, being dismissed on March 2000 was
premature. However, Saile is not qualified to take the LET, therefore,
no prematurity is to speak of on her end. Petitioner’s intention and
desire not to put the students’ education and school operation in
jeopardy is neither a decisive consideration for respondents’
termination prior to the deadline set by law. The prejudice that
respondents’ retention would cause to the school’s operation is only
trivial.

593
Page 594

Case Digest by: PAJARILLAGA, JAN LAURENCE M.

TOYOTA MOTOR PHILS. CORP. WORKERS


ASSOCIATION v. NLRC
G.R. NOS. 158786 & 158789 : October 19, 2007
Velasco, Jr., J.

DOCTRINE:
ARTICLE 294 (Formerly Article 279)

A strike means any temporary stoppage of work by the concerted


action of employees as a result of an industrial or labor dispute. A labor
dispute, in turn, includes any controversy or matter concerning terms
or conditions of employment or the association or representation of
persons in negotiating, fixing, maintaining, changing, or arranging the
terms and conditions of employment, regardless of whether the
disputants stand in the proximate relation of the employer and the
employee.
With regard to the grant of severance compensation to legally
dismissed union members, the general rule is that when just causes
for terminating the services of an employee under Art. 282 of the Labor
Code exist, the employee is not entitled to separation pay. There are
however certain exceptions. One exception where separation pay is
given even though an employee is validly dismissed is when the court
finds justification in applying the principle of social justice well
entrenched in the 1987 Constitution.

FACTS:
In this case, Toyota Motor Philippines Corporation Workers
Association (hereinafter referred to as the Union) filed a petition for
certification election among the Toyota rank and file employees with
the National Conciliation and Mediation Board (NCMB). The Med-
Arbiter denied the petition, but, on appeal, the DOLE
Secretary granted the Union’s prayer, and, through an Order, directed
the immediate holding of the certification election. After Toyota’s plea
for reconsideration was denied, the certification election was
conducted. The Med-Arbiter’s Order certified the Union as the sole and
exclusive bargaining agent of all the Toyota rank and file
employees. Toyota challenged said Order via an appeal to the DOLE
Secretary.

In the meantime, the Union submitted its CBA proposals to Toyota, but
the latter refused to negotiate in view of its pending appeal.
Consequently, the Union filed a notice of strike with the NCMB based
on Toyota’s refusal to bargain. In connection with Toyota’s appeal,
Toyota and the Union were required to attend a hearing on before the

594
Page 595

Bureau of Labor Relations (BLR). The February 21, 2001 hearing was
cancelled and reset to February 22.

Going to the merits of this case, more than 200 employees of Toyota
were terminated from work after they participated in 2 strikes on
separate occasions, causing substantial losses on the part of Toyota
due to its inability to meet the production goals. It appears that on the
first instance, the Union requested that they be allowed to attend the
hearing of the pending mediation case, to which Toyota management
refused. Despite such, the employees-union members and officers still
did not report to work for 2 consecutive days. These two strikes as
discussed below became one of the main considerations as to the
controversy involved and to be resolved herein.

The first strike in this case was staged on February 21, 2001. 135
Union officers and members failed to render the required overtime
work, and instead marched to and staged a picket in front of the BLR
office. The Union, in a letter of the same date, also requested that its
members be allowed to be absent on February 22 to attend the
hearing and instead work on their next scheduled rest day. This
request however was denied by Toyota. And despite denial of the
Union’s request, more than 200 employees staged mass actions on
February 22 and 23 in front of the DOLE offices, to protest certain
labor issues such as the partisan and anti-union stance of Toyota. Due
to the deliberate absence of a little around 200 employees on
February 22 to 23, Toyota experienced acute lack of manpower in its
manufacturing and production lines, and was unable to meet its
production goals resulting in huge losses. On February 27, Toyota
sent individual letters to some 360 employees requiring them to
explain within 24 hours why they should not be dismissed for their
obstinate defiance of the company’s directive to render overtime work
on February 21, for their failure to report for work on February 22 and
23, and for their participation in the concerted actions which severely
disrupted and paralyzed the plant’s operations.
In response to the said notice to explain, Toyota Motor Philippines
Corporation Workers Association filed with the NCMB another notice
of strike for union busting amounting to unfair labor practice. On March
1, Toyota Motor Philippines Corporation Workers Association
nonetheless submitted an explanation in compliance with the February
27 notices sent by Toyota to the erring employees. Consequently, on
March 2 and 5, Toyota issued 2 memoranda to the concerned
employees to clarify whether or not they are adopting the March 1,
2001 Union’s explanation as their own. The employees were also
required to attend an investigative interview, but they refused to do so.
As a result, on March 16, Toyota terminated the employment of 227
employees for participation in concerted actions in violation of its Code
of Conduct and for misconduct under Article 282 of the Labor Code.

595
Page 596

A second and subsequent strike was thereafter staged, and this was
claimed to be in reaction to the dismissal of its union members and
officers, the Toyota Motor Philippines Corporation Workers Association
Union went on strike on March 17, 2001. Subsequently, from March
28, 2001 to April 12, 2001, the Union intensified its strike by barricading
the gates of Toyota’s Bicutan and Sta. Rosa plants. The strikers
prevented workers who reported for work from entering the plants. On
March 29, 2001, Toyota filed a petition for injunction with a prayer for
the issuance of a TRO with the NLRC. It sought free ingress to and
egress from its Bicutan and Sta. Rosa manufacturing plants. Acting on
said petition, the NLRC issued a TRO against the Union, ordering its
leaders and members as well as its sympathizers to remove their
barricades and all forms of obstruction to ensure free ingress to and
egress from the company’s premises. In addition, Toyota filed a
petition to declare the strike illegal with the NLRC arbitration branch,
and prayed that the erring Union officers, directors, and members be
dismissed. The DOLE Secretary assumed jurisdiction over the labor
dispute on April 10, 2001 and issued an Order certifying the labor
dispute to the NLRC. In said Order, the DOLE Secretary directed all
striking workers to return to work at their regular shifts by April 16,
2001. The DOLE Secretary ordered Toyota to accept the returning
employees under the same terms and conditions obtaining prior to the
strike or at its option, put them under payroll reinstatement and at the
same time, parties were also enjoined from committing acts that may
worsen the situation. The second strike staged by the Union ended the
strike on April 12, 2001 and the union members and officers tried to
return to work on April 16 but too unfortunate for them, they were told
that Toyota opted for payroll-reinstatement authorized by the Order of
the DOLE Secretary.

A third strike was staged on May 23, 2001, despite the issuance of the
DOLE Secretary’s certification Order, several payroll-reinstated
members of the Union staged a protest rally in front of Toyota’s Bicutan
Plant bearing placards and streamers in defiance of the April 10, 2001
DOLE order. Then, on May 28, 2001, several union members staged
another protest action in front of the Bicutan Plant. At the same time,
some payroll-reinstated employees picketed in front of the Santa Rosa
Plant’s main entrance, and were later joined by other Union members.
Notwithstanding the certification Order, the Union filed another notice
of strike on June 5, 2001.

The NLRC, on its part, ordered both parties to submit their respective
position papers on June 8, 2001. The union, however, requested
for abeyance of the proceedings considering that there is a
pending petition for certiorari with the CA assailing the validity of the
DOLE Secretary’s Assumption of Jurisdiction Order.

596
Page 597

But on June 19, 2001, the NLRC issued an Order, reiterating its
previous order for both parties to submit their respective position
papers on or before June 2, 2001. But it was only Toyota who
submitted its position paper. During the August 3, 2001 hearing, the
Union, despite several accommodations, still failed to submit its
position paper, but later claimed it filed its position paper by registered
mail. Subsequently, the NLRC, in its August 9, 2001 Decision, declared
the strikes staged by the Union on February 21 to 23, 2001 and May
23 and 28 as illegal for the Union failed to comply with the procedural
requirements of a valid strike under Art. 263 of the Labor Code. The
NLRC declared that the dismissal of the 227 who participated in the
illegal strike on February 21-23, 2001 is legal. The NLRC ruled that the
award of severance compensation may properly be given to the
dismissed Union members. The Union again staged strikes on May 23
and 28, 2001, after the DOLE Secretary assumed jurisdiction over the
Toyota dispute on April 10, 2001. The NLRC found the strikes illegal
as they violated Article 264 of the Labor Code which proscribes any
strike or lockout after jurisdiction is assumed over the dispute by the
President or the DOLE Secretary. The NLRC held that both parties
must have maintained the status quo after the DOLE Secretary issued
the certification order, and ruled that the union did not respect the
DOLE Secretary’s directive. Both parties filed motion for
reconsiderations, which the NLRC denied. Subsequently, both parties
questioned the Resolutions of the NLRC in separate petitions for
certiorari filed with the Court of Appeals, which consolidated the
petitions.

The Court of Appeals ruled that the union’s petition is defective in form
for its failure to append a proper verification and certificate of non-
forum shopping, given that, out of the 227 petitioners, only 159 signed
the verification and certificate of non-forum shopping. Despite the flaw,
the Court of Appeals proceeded to resolve the petitions on the merits
and affirmed the assailed NLRC Decision and Resolution with a
modification, however, of deleting the award of severance
compensation to the dismissed Union members. But in its June 20,
2003 Resolution, the Court of Appeals modified its February 27, 2003
Decision by reinstating severance compensation to the dismissed
employees based on social justice.

ISSUE:
(a) Were the strikes committed by the union on the above-mentioned
occasions are illegal.
(b) Should separation pay should be awarded to the union members
who participated in the illegal strikes.

RULING:

597
Page 598

(a) There is an illegal strike in the case at bar. A strike means any
temporary stoppage of work by the concerted action of employees as
a result of an industrial or labor dispute. A labor dispute, in turn,
includes any controversy or matter concerning terms or conditions of
employment or the association or representation of persons in
negotiating, fixing, maintaining, changing, or arranging the terms and
conditions of employment, regardless of whether the disputants stand
in the proximate relation of the employer and the employee.

Petitioner Union contends that the protests or rallies conducted on


February 21 and 23, 2001 are not within the ambit of strikes as defined
in the Labor Code, since they were legitimate exercises of their right to
peaceably assemble and petition the government for redress of
grievances. But the union’s position fails to convince the Supreme
Court. The Supreme Court ruled that the protest actions undertaken by
the Union officials and members on February 21 to 23, 2001
are not valid and proper exercises of their right to assemble and ask
government for redress of their complaints, but are illegal strikes in
breach of the Labor Code. The Union’s position is weakened by the
lack of permit from the City of Manila to hold rallies. Shrouded as
demonstrations, they were in reality temporary stoppages of work
perpetrated through the concerted action of the employees who
deliberately failed to report for work on the convenient excuse that they
will hold a rally at the BLR and DOLE offices on February 21 to 23. The
purported reason for these protest actions was to safeguard their rights
against any abuse which the med-arbiter may commit against their
cause. However, the Union failed to advance convincing proof that the
med-arbiter was biased against them. The acts of the med-arbiter in
the performance of his duties are presumed regular. Sans ample
evidence to the contrary, the Union was unable to justify the February
2001 mass actions. What comes to the fore is that the decision not to
work for two days was designed and calculated to cripple the
manufacturing arm of Toyota. It becomes obvious that the real and
ultimate goal of the Union is to coerce Toyota to finally acknowledge
the Union as the sole bargaining agent of the company. This is not a
legal and valid exercise of the right of assembly and to demand redress
of grievance. The concerted actions were undertaken without
satisfying the prerequisites for a valid strike under Art. 263 of the Labor
Code. The Union failed to comply with the requirement of a notice of
strike filed with the DOLE 30 days before the intended date of strike,
or 15 days in case of unfair labor practice, strike vote approved by a
majority of the total union membership in the bargaining unit concerned
obtained by secret ballot in a meeting called for that purpose and notice
given to the DOLE of the results of the voting at least seven days
before the intended strike. These requirements are mandatory and the
failure of a union to comply with them renders the strike illegal. The
evident intention of the law in requiring the strike notice and the strike-
vote report is to reasonably regulate the right to strike, which is

598
Page 599

essential to the attainment of legitimate policy objectives embodied in


the law. As they failed to conform to the law, the strikes on February
21, 22, and 23, 2001 or the one referred to as the first strike
were illegal.

With respect to the strikes committed from March 17 to April 12, 2001
or the one referred to as the second strike, those were initially legal as
the legal requirements were met. However, on March 28 to April 12,
2001, the Union barricaded the gates of the Bicutan and Sta. Rosa
plants and blocked the free ingress to and egress from the company
premises. Toyota employees, customers, and other people having
business with the company were intimidated and were refused entry to
the plants. As earlier explained, these strikes were illegal because
unlawful means were employed. The acts of the Union officers and
members are in palpable violation of Art. 264(e), which proscribes acts
of violence, coercion, or intimidation, or which obstruct the free ingress
to and egress from the company premises. Undeniably, the strikes
from March 28 to April 12, 2001, or the one referred to as the second
strike, were illegal.

Petitioner Union also posits that strikes were not committed on May 23
and 28, 2001 or the one referred to as the third strike. The Union
asserts that the rallies held on May 23 and 28, 2001, could not be
considered strikes, as the participants were the dismissed employees
who were on payroll reinstatement. It concludes that there was no work
stoppage. However, the Court ruled that this contention has no basis.
It is clear that once the DOLE Secretary assumes jurisdiction over the
labor dispute and certifies the case for compulsory arbitration with the
NLRC, the parties have to revert to the status quo ante or the state of
things as it was before. This was not heeded by the Union and the
individual respondents who staged illegal concerted actions on May 23
and 28, 2001, in contravention of the Order of the DOLE Secretary that
no acts should be undertaken by them to aggravate the already
deteriorated situation.

(b) With regard to the grant of severance compensation to legally


dismissed union members, the general rule is that when just causes
for terminating the services of an employee under Art. 282 of the Labor
Code exist, the employee is not entitled to separation pay. There are
however certain exceptions. One exception where separation pay is
given even though an employee is validly dismissed is when the court
finds justification in applying the principle of social justice well
entrenched in the 1987 Constitution. In one case, the Court laid down
the rule that severance compensation shall be allowed only when the
cause of the dismissal is other than serious misconduct or that which
reflects adversely on the employee’s moral character. Considering that
the dismissal of the employees was due to their participation in the

599
Page 600

illegal strikes as well as violation of the Code of Conduct of the


company, the same constitutes serious misconduct. A serious
misconduct is a transgression of some established and definite rule of
action, a forbidden act, a dereliction of duty, willful in character, and
implies wrongful intent and not mere error in judgment. Based on
existing jurisprudence, the award of separation pay to the Union
officials and members in the instant petitions cannot be sustained.

The Union contends that the NLRC violated its right to due process
when it disregarded its position paper in deciding Toyota’s petition to
declare the strike illegal. It is entirely the Union’s fault that its position
paper was not considered by the NLRC. Records readily reveal that
the NLRC was even too generous in affording due process to the
Union. It issued no less than 3 orders for the parties to submit its
position papers, which the Union ignored until the last minute. No
sufficient justification was offered why the Union belatedly filed its
position paper. In Datu Eduardo Ampo v. The Hon. Court of Appeals,
it was explained that a party cannot complain of deprivation of due
process if he was afforded an opportunity to participate in the
proceedings but failed to do so. If he does not avail himself of the
chance to be heard, then it is deemed waived or forfeited without
violating the constitutional guarantee. Thus, there was no violation of
the Union’s right to due process on the part of the NLRC. Union officers
are liable for unlawful strikes or illegal acts during a strike. Art. 264(a)
sanctions the dismissal of a union officer who knowingly participates in
an illegal strike or who knowingly participates in the commission of
illegal acts during a lawful strike.

Member’s liability depends on participation in illegal acts. Art. 264(a) of


the Labor Code provides that a member is liable when he knowingly
participates in an illegal act during a strike. No precise meaning was
given to the phrase “illegal acts.” It may encompass a number of acts
that violate existing labor or criminal laws, such as the violation of
Article 264(e) of the Labor Code which provides that “[n]o person
engaged in picketing shall commit any act of violence, coercion or
intimidation or obstruct the free ingress to or egress from the
employer’s premises for lawful purposes, or obstruct public
thoroughfares”, commission of crimes and other unlawful acts in
carrying out the strike; and violation of any order, prohibition, or
injunction issued by the DOLE Secretary or NLRC in connection with
the assumption of jurisdiction/certification Order under Art. 263(g) of
the Labor Code. This enumeration, however, is not exclusive and it
may cover other breaches of existing laws. However, there must be
proof that he committed illegal acts during the strike and the striker who
participated in the commission of illegal act[s] must be identified. But
proof beyond reasonable doubt is not required. Substantial evidence
available under the circumstances, which may justify the imposition of
the penalty of dismissal, may suffice.

600
Page 601

Case Digests by: PAJARILLAGA, JAN LAURENCE M.

BRISTOL MYERS SQUIBB (PHILS.), INC. vs. RICHARD


NIXON A. BABAN
G.R. No. 167449 December 17, 2008
REYES, R.T., J.:

DOCTRINE:

There are two requisites for a valid dismissal by the employer on the
ground of breach of trust and confidence under Article 297(c), and
those employees involved herein must be one holding a position of
trust and confidence, such as: (1) managerial employees, for they are
defined as those vested with the powers or prerogatives to lay down
management policies and to hire, transfer suspend, lay-off, recall,
discharge, assign or discipline employees or effectively recommend
such managerial actions. (2) Cashiers, auditors, property custodians,
for they are defined as those who in the normal and routine exercise
of their functions, regularly handle significant amounts of money or
property. In addition, there must be an act that would justify the loss of
trust and confidence. Loss of trust and confidence to be a valid cause
for dismissal must be based on a willful breach of trust and founded on
clearly established facts. The basis for the dismissal must be clearly
and convincingly established but proof beyond reasonable doubt is not
necessary.

FACTS:

In 1992, Bristol Myers Squibb Philippines, Inc. (BMSI) hired


Richard Nixon A. Baban (Baban) as district manager of the company.
Being a manager and a confidential employee, he was assigned to
handle the company's clients in Cagayan de Oro-Northern Mindanao
area and its immediate vicinities and to supervise territory managers
detailed in his district. He is also tasked to promote the nutritional
products of BMSI to medical practitioners and to sell products the
same to drug outlets.
Sometime in 1998, an auditor of BMSI found twenty (20) packs
of "Mamacare" samples in the baggage compartment of a company
car with an accompanying note with political overtones. Apparently,
Baban stapled the political "thank you" note with the intention of
distributing them to his father's supporters during the 1998 election.
In the course of the investigation, Baban admitted that he had
caused the attachment of the notes to the product samples. He argued
that there was no unauthorized distribution of the samples since he
intended to give them only to doctors who requested them. He then
admitted that he had committed a mistake and pleaded for
consideration for the lapse, insisting that he has not caused any

601
Page 602

damage nor injury to the image of the company as the samples were
not, in fact, distributed and that no gain was derived by him or his
family. Unconvinced with his explanation, Baban was dismissed,
hence this case for illegal dismissal.
The matter was first brought before the Labor Arbiter which ruled that
Baban’s Termination is valid. It was ruled that Baban has violated
company rules and regulations by his unauthorized use of its property.
BMSI is therefore justified to declare respondent unworthy of the trust
and confidence formerly imposed in him. The Labor Arbiter, however
dismissed respondent's complaint. The Labor Arbiter ordered BMSI to
pay Baban P297,009.84 representing the admitted monetary liabilities.
The matter was raised before the NLRC. The latter affirmed the LA
ruling. The NLRC further awarded financial assistance in favor of
Baban by way of separation pay equivalent to one (1) month pay for
every year of service covering the period from the date of his regular
employment up to 25 August 1998, a fraction of six (6) months being
considered one (1) year.
As the matter was raised before the Court of Appeals, the latter ruled
that Baban was illegally dismissed. The CA apparently granted his plea
for mercy when it ruled that his action while censurable did not merit
termination. The CA characterized his action as a mere lapse of human
frailty considering the elections were over. Moreover, the stapling of
the thank you notes did not give rise to any undue advantage to
respondent or his father.

ISSUE:
Is the CA order of reinstatement, with full backwages and damages,
of a confidential employee whom the company had found to be guilty
of breach of trust justifiable?

RULING:

Baban was rightfully dismissed. It cannot be argued that Baban is a


confidential employee. He is one holding a position of trust and
confidence. In this case, Baban was employed as district manager for
Cagayan de Oro-North Mindanao and its immediate vicinities. It is not
the job title but the actual work that the employee performs. He was
employed to handle products for distribution to medical practitioners
and sale to drug outlets. As a result of his handling of large amounts of
petitioner's samples, respondent is, by law, an employee with a
position of trust, falling under the second class.
It is equally apparent that there was an act that would justify the loss
of trust and confidence in this case. There is no doubt that Baban
willfully breached the trust and confidence reposed in him by not asking
for permission before using company property for his own or another's
benefit, as required in the Company Standards of Business Conduct.
Moreover, when Baban failed to turn over the samples left in his care
and stapled the political "thank you" note with the intention of

602
Page 603

distributing them to his father's supporters, he had, in effect


appropriated company property for personal gain and benefit.
However, the SC ruled that since Baban was validly dismissed
for a cause other than serious misconduct or those that negatively
reflect on his moral character, the award of separation pay as an
equitable relief is justifiable. This award is merely to coat the bitter
termination experienced by respondent with a little social justice.
Separation pay at the rate of one month salary for every year of
service is proper.

603
Page 604

Case Digests by: PAJARILLAGA, JAN LAURENCE M.

ARMANDO G. YRASUEGUI v. PHILIPPINE AIRLINES, INC


G.R. No. 168081 October 17, 2008
REYES, R.T., J.:

DOCTRINE:

The requisites for Bona Fide Occupational Qualification (BFOQ) are as


follows, (1) the employment qualification is reasonably related to the
essential operation of the job involved; and (2) that there is factual
basis for believing that all or substantially all persons meeting the
qualification would be unable to properly perform the duties of the job.
The dismissal of petitioner can be predicated on the bona fide
occupational qualification defense. Employment in particular jobs may
not be limited to persons of a particular sex, religion, or national origin
unless the employer can show that sex, religion, or national origin is
an actual qualification for performing the job. The qualification is
called a bona fide occupational qualification (BFOQ). In short, the test
of reasonableness of the company policy is used because it is
parallel to BFOQ. BFOQ is valid “provided it reflects an inherent
quality reasonably necessary for satisfactory job performance.”

FACTS:
The prescribed weight of PAL is a continuing BFOQ. On board
an aircraft, the body weight and size of a cabin attendant are important
factors to consider in case of emergency. Aircrafts have constricted
cabin space, and narrow aisles and exit doors. It would d be absurd to
require airline companies to reconfigure the aircraft in order to widen
the aisles and exit doors just to accommodate overweight cabin
attendants like petitioner herein. Thus, the failure to maintain such
weight is a valid ground for dismissal under the “analogous cases” of
Article 297(e) of the Labor Code. As a rule, a legally dismissed
employee is not entitled to separation pay. This may be deduced from
the language of Article 294 of the Labor Code that [a]n employee who
is unjustly dismissed from work shall be entitled to such. Exceptionally,
it is granted to a legally dismissed employee as an act "social justice,"
or based on "equity." In both instances, it is required that the dismissal
(1) was not for serious misconduct; and (2) does not reflect on the
moral character of the employee. Armando was a former international
flight steward of PAL. He stands five feet and eight inches (5’8") with a
large body frame. The proper weight for a man of his height and body
structure is from 147 to 166 pounds, the ideal weight being 166
pounds, as mandated by the Cabin and Crew Administration Manual
of PAL. In 1984, PAL gave Armando an extended vacation leave in
order for him to lose weight. After meeting the required weight,

604
Page 605

Armando was allowed to return to work. But Armando’s weight problem


recurred. He was 40 pounds beyond his ideal weight and his weight
kept on increasing in the course of his leave without pay from October
17, 1988 to February 1989. For a span of 5 years, PAL has been giving
Armando vacation leave without pay to give him time to get back in
shape. PAL even scheduled several weight checks to monitor
Armando’s weight. Armando, on the other hand, would always
reschedule and would only report for a couple of times. On the last 3
weight checks that he had attended, Armando still remained
overweight.
On November 13, 1992, PAL finally served Armando a Notice of
Administrative Charge for violation of company standards on weight
requirements. In his Answer, Armando did not deny being overweight
but claimed that his violation, if any, had already been condoned by
PAL since "no action has been taken by the company" regarding his
case "since 1988. He also claimed that PAL discriminated against him
because "the company has not been fair in treating the cabin crew
members who are similarly situated." During the clarificatory hearing
on December 1992, Armando manifested that he was undergoing a
weight reduction program to lose at least two (2) pounds per week so
as to attain his ideal weight. Six months thereafter, Armando was
formally informed by PAL that due to his inability to attain his ideal
weight, "and considering the utmost leniency" extended to him "which
spanned a period covering a total of almost five (5) years," his services
were considered terminated "effective immediately."
Armando then filed a complaint for illegal dismissal before the
Labor Arbiter which ruled in Armando’s favor and held the illegality of
the dismissal. The LA held that the weight standards of PAL are
reasonable in view of the nature of the job of Armando. However, the
weight standards need not be complied with under pain of dismissal
since his weight did not hamper the performance of his duties.
Assuming that it did, Armando could be transferred to other positions
where his weight would not be a negative factor. Notably, other
overweight employees, were promoted instead of being disciplined.
Both parties appealed to the matter to the NLRC. The NLRC
affirmed the decision of the Labor Arbiter. According to the NLRC,
"obesity, or the tendency to gain weight uncontrollably regardless of
the amount of food intake, is a disease in itself. As a consequence,
there can be no intentional defiance or serious misconduct by
petitioner to the lawful order of PAL for him to lose weight.
The CA reversed the NLRC opining that the weight standards of
PAL are meant to be a continuing qualification for an employee's
position. The failure to adhere to the weight standards is an analogous
cause for the dismissal of an employee under Article 282(e) of the
Labor Code in relation to Article 282(a).

ISSUE:

605
Page 606

Is obesity a just cause in termination of the employee’s services?

RULING:

The Court ruled in the affirmative stating that obesity of petitioner is a


ground for dismissal under Article 282(e) of the Labor Code. It is a
continuing qualification. Tersely put, an employee may be dismissed
the moment he is unable to comply with his ideal weight as prescribed
by the weight standards. The dismissal of the employee would thus
fall under Article 282(e) of the Labor Code. The standards violated in
this case were not mere "orders" of the employer; they were the
"prescribed weights" that a cabin crew must maintain in order to qualify
for and keep his or her position in the company. In this sense, the
failure to maintain these standards does not fall under Article 282(a)
whose express terms require the element of willfulness in order to be
a ground for dismissal. The failure to meet the employer's qualifying
standards is in fact a ground that falls under Article 282(e) - the "other
causes analogous to the foregoing."
By its nature, these "qualifying standards" are norms that apply prior
to and after an employee is hired. They apply prior to employment
because these are the standards a job applicant must initially meet in
order to be hired. They apply after hiring because an employee must
continue to meet these standards while on the job in order to keep his
job. Under this perspective, a violation is not one of the faults for which
an employee can be dismissed pursuant to pars. (a) to (d) of Article
282; the employee can be dismissed simply because he no longer
"qualifies" for his job irrespective of whether or not the failure to qualify
was willful or intentional.
The court held that the obesity of petitioner, when placed in the context
of his work as flight attendant, becomes an analogous cause under
Article 282(e) of the Labor Code that justifies his dismissal from the
service. His obesity may not be unintended, but is nonetheless
voluntary. As the CA correctly puts it, voluntariness basically means
that the just cause is solely attributable to the employee without any
external force influencing or controlling his actions. This element runs
through all just causes under Article 282, whether they be in the nature
of a wrongful action or omission. Gross and habitual neglect, a
recognized just cause, is considered voluntary although it lacks the
element of intent found in Article 282(a), (c), and (d).
The dismissal of petitioner can be predicated on the bona fide
occupational qualification defense. Employment in particular jobs may
not be limited to persons of a particular sex, religion, or national origin
unless the employer can show that sex, religion, or national origin is
an actual qualification for performing the job. The qualification is called
a bona fide occupational qualification (BFOQ). In short, the test of
reasonableness of the company policy is used because it is parallel to
BFOQ. BFOQ is valid “provided it reflects an inherent quality

606
Page 607

reasonably necessary for satisfactory job performance.” The business


of PAL is air transportation. As such, it has committed itself to safely
transport its passengers. In order to achieve this, it must necessarily
rely on its employees, most particularly the cabin flight deck crew who
are on board the aircraft. The weight standards of PAL should be
viewed as imposing strict norms of discipline upon its employees. The
primary objective of PAL in the imposition of the weight standards for
cabin crew is flight safety. Separation pay, however, should be
awarded in favor of the employee as an act of social justice or based
on equity. This is so because his dismissal is not for serious
misconduct. Neither is it reflective of his moral character.

607
Page 608

Case Digests by: PAJARILLAGA, JAN LAURENCE M.

DREAMLAND HOTEL RESORT and WESTLEY J.


PRENTICE vs. STEPHEN B. JOHNSON
G.R. No. 191455 March 12, 2014
REYES, J.:

DOCTRINE:
SEPARATION PAY AND STRAINED RELATIONS

There is constructive dismissal if an act of clear discrimination,


insensibility, or disdain by an employer becomes so unbearable on the
part of the employee that it would foreclose any choice by him except
to forego his continued employment. It exists where there is cessation
of work because continued employment is rendered impossible,
unreasonable or unlikely, as an offer involving a demotion in rank and
a diminution in pay. In such case, there is no abandonment of
employment nor a resignation in the real sense. A constructively
dismissed employee is an illegally dismissed employee who is entitled
to two reliefs: backwages and reinstatement. These are separate and
distinct. In instances where reinstatement is no longer feasible
because of strained relations between the employee and the
employer, separation pay is considered an acceptable alternative. In
effect, an illegally dismissed employee is entitled to either
reinstatement, if viable, or separation pay if reinstatement is no longer
viable, and backwages. In case at bar, Johnson is a permanent
resident exempted from the requirement of securing an AEP. There
was constructive dismissal and not abandonment of work because he
tendered his resignation due to the fact that despite working for 3
months, he received only an insignificant fraction of his salaries.

The accepted doctrine is that separation pay may avail in lieu of


reinstatement if reinstatement is no longer practical or in the best
interest of the parties. Separation pay in lieu of reinstatement may
likewise be awarded if the employee decides not to be reinstated.
Under the doctrine of strained relations, the payment of separation
pay is considered an acceptable alternative to reinstatement when
the latter option is no longer desirable or viable.

FACTS:

Dreamland is a corporation while Prentice is its current


President and Chief Executive Officer. Respondent Stephen B.
Johnson is an Australian citizen who came to the Philippines as a

608
Page 609

businessman/investor without the authority to be employed as the


employee/officer of any business as he was not able to secure his
Alien Employment Permit AEP. Prentice alleged that Johnson invest
in Dreamland and at the same time provide his services as
Operations Manager of Dreamland with a promise that he will secure
an AEP and TIN. Westley J. Prentice and Stephen B. Johnson
entered into an Employment Agreement, which stipulates among
others, that Johnson shall serve as Operations Manager of
Dreamland from August 1, 2007 and shall serve as such for a period
of three (3) years. From the start of August 2007, Johnson already
reported for work. However, he was instructed to supervise
construction and speak with potential guests. He also undertook the
overall preparation of the guestrooms and staff for the opening of the
hotel, even performing menial tasks. As he was remained unpaid
since August 2007 and he has loaned all his money to petitioners, he
asked for his salary after the resort was opened in October 2007 but
the same was not given to him by petitioners. He became very
alarmed with the situation as it appears that there was no intention to
pay him his salary, which he now depended on for his living as he
has been left penniless. He was also denied the benefits promised
him as part of his compensation such as service vehicles, meals and
insurance. He was also not given the authority due to him as resort
manager. Another argument posited by the petitioners is that the
employment contract executed by the parties is inefficacious because
the employment contract is subject to the presentation of Johnson of
his Alien Employment Permit (AEP) and Tax Identification Number
(TIN). Prentice countermanded his orders to the staff at every
opportunity. Worse, he would even be berated and embarrassed in
front of the staff. After another embarrassment was handed out by
Prentice in front of the staff, which highlighted his lack of real
authority in the hotel and the disdain for him by petitioners,
respondent Johnson was forced to submit his resignation. In
deference to the Employment Agreement signed, Johnson stated that
he was willing to continue work for the three-month period stipulated
therein. However, in an SMS or text message sent by Prentice to him
on the same day at around 8:20 pm, he was informed that his
resignation is considered immediate. Despite demand, petitioners
refused to pay Johnson the salaries and benefits due him. Johnson
filed a Complaint for illegal dismissal and non-payment of salaries,
separation pay, among others, against the petitioners. The matter
was brought before the Labor Arbiter which dismissed Johnson’s
complaint for lack of merit with the finding that he voluntarily resigned
from his employment and was not illegally dismissed. The NLRC,
however, reversed LA's ruling. In effect, the NLRC ruled that there’s
no abandonment of work but a constructive dismissal, which is
defined as an involuntary resignation resorted to when continued
employment is rendered impossible, unreasonable or unlikely.

609
Page 610

ISSUE:
Is Johnson entitled to his claims of unpaid salaries and
separation pay?

RULING:
Johnson is entitled to separation pay and for the other claims of unpaid
salaries. The petitioners contend that the employment of Johnson as
operations manager commenced only on October 8, 2007 and not on
August 1, 2007. However, the employment contract categorically
stated that the "term of employment shall commence on August 1,
2007." Furthermore, the factual allegations of Johnson that he actually
worked from August 1, 2007 were neither sufficiently rebutted nor
denied by the petitioners. For the petitioners’ failure to disprove that
Johnson started working on August 1, 2007, as stated on the
employment contract, payment of his salaries on said date, even prior
to the opening of the hotel is warranted. Johnson has adduced proof
that as a permanent resident, he is exempted from the requirement of
securing an AEP as expressed under Department Order No. 75-06,
Series of 2006 of the Department of Labor and Employment (DOLE).
Furthermore, Johnson submitted a Certification from DOLE Regional
Office III, stating that he is exempted from securing an AEP as a holder
of Permanent Resident Visa. Consequently, the condition imposed
upon Johnson’s employment, if there is any, is in truth without effect
to its validity. Anent the requirement of securing a TIN to make the
contract of employment efficacious, records show that Johnson
secured his TIN only on December 2007 after his resignation as
operations manager. Nevertheless, this does not negate the fact that
the contract of employment had already become effective even prior
to such date. In addition to the foregoing, there is no stipulation in the
employment contract itself that the same shall only be effective upon
the submission of AEP and TIN.
It cannot be argued any further that Johnson is an employee of
Dreamland. This is evidenced by their employment contract. The fact
that Johnson did not furnish AEP and TIN number will not invalidate
the contract since nowhere in the contract did it state that failure to
submit the said document would invalidate the employment. Another,
failure of submission of AEP will not invalidate the contract because
Johnson is exempted from such requirement. It is equally apparent as
well that Johnson was constructively dismissed. In this case, the SC
upheld the findings of NLRC that there’s no abandonment of work but
a constructive dismissal, which is defined as an involuntary resignation
resorted to when continued employment is rendered impossible,
unreasonable or unlikely.
There is constructive dismissal if an act of clear discrimination,
insensibility, or disdain by an employer becomes so unbearable on the
part of the employee that it would foreclose any choice by him except
to forego his continued employment. It exists where there is cessation

610
Page 611

of work because continued employment is rendered impossible,


unreasonable or unlikely, as an offer involving a demotion in rank and
a diminution in pay. It is impossible, unreasonable or unlikely that any
employee, such as Johnson would continue working for an employer
who does not pay him his salaries. The Court construes that the act of
the petitioners in not paying Johnson his salaries for three months has
become unbearable on the latter’s part that he had no choice but to
cede his employment with them. While it was Johnson who tendered
his resignation, it was due to the petitioners’ acts that he was
constrained to resign. The petitioners cannot expect Johnson to
tolerate working for them without any compensation. Since Johnson
was constructively dismissed, he was illegally dismissed. Thus, an
illegally dismissed employee is entitled to two reliefs: backwages and
reinstatement. The two reliefs provided are separate and distinct. In
instances where reinstatement is no longer feasible because of
strained relations between the employee and the employer, separation
pay is granted. In effect, an illegally dismissed employee is entitled to
either reinstatement, if viable, or separation pay if reinstatement is no
longer viable, and backwages.
The normal consequences of respondents’ illegal dismissal, then, are
reinstatement without loss of seniority rights, and payment of
backwages computed from the time compensation was withheld up to
the date of actual reinstatement. But in this case, NLRC found that due
to the strained relations between the parties, separation pay is to be
awarded to Johnson in lieu of his reinstatement. The NLRC held that
Johnson is entitled to backwages up to the finality of the decision,
separation pay equivalent to one month salary, and unpaid salaries.
Separation pay is computed from the commencement of employment
up to the time of termination, including the imputed service for which
the employee is entitled to backwages. As one-month salary is
awarded as separation pay for every year of service, including imputed
service, Johnson should be paid separation pay equivalent to his
three-month salary for the three-year contract.

In the present case, the NLRC found that due to the strained relations
between the parties, separation pay is to be awarded to Johnson in
lieu of his reinstatement. The NLRC held that Johnson is entitled to
backwages from November 3, 2007 up to the finality of the decision;
separation pay equivalent to one month salary; and unpaid salaries
from August 1, 2007 to November 1, 2007 amounting to a total of
P172,800.00. While the Court agrees with the NLRC that the award of
separation pay and unpaid salaries is warranted, the Court does not
lose sight of the fact that the employment contract states that
Johnson's employment is for a term of three years. Accordingly, the
award of backwages should be computed from November 3, 2007 to
August 1, 2010 - which is three years from August 1, 2007.

611
Page 612

Furthermore, separation pay is computed from the commencement of


employment up to the time of termination, including the imputed
service for which the employee is entitled to backwages.

612
Page 613

Case Digests by: PAJARILLAGA, JAN LAURENCE M.

MANILA WATER COMPANY vs. CARLITO DEL ROSARIO


G.R. No. 188747 January 29, 2014
PEREZ, J.:

DOCTRINE:
SEPARATION PAY

An employee who has been dismissed for any of the just causes
is not entitled to a separation pay. However, in exceptional cases, the
Court has granted separation pay to a legally dismissed employee as
an act of "social justice" or on "equitable grounds." In both instances,
it is required that the dismissal (1) was not for serious misconduct; and
(2) did not reflect on the moral character of the employee. A contrary
rule would have the effect of rewarding rather than punishing the erring
employee for his offense. If the employee who steals from the company
is granted separation pay even as he is validly dismissed, it is not
unlikely that he will commit a similar offense in his next employment
because he thinks he can expect a like leniency if he is again found
out. This kind of misplaced compassion is not going to do labor in
general any good as it will encourage the infiltration of its ranks by
those who do not deserve the protection and concern of the
Constitution.

If an employee's length of service is to be regarded as a justification


for moderating the penalty of dismissal, such gesture will actually
become a prize for disloyalty, distorting the meaning of social justice
and undermining the efforts of labor to cleanse its ranks of
undesirables. In exceptional cases, however, the Court has granted
separation pay to a legally dismissed employee as an act of "social
justice" or on "equitable grounds." In both instances, it is required that
the dismissal (1) was not for serious misconduct; and (2) did not reflect
on the moral character of the employee.

FACTS:

On 22 October 1979, Del Rosario was employed as Instrument


Technician by Metropolitan Waterworks and Sewerage System
(MWSS). Sometime in 1996, MWSS was reorganized pursuant to
Republic Act No. 8041 or the National Water Crisis Act of 1995, and
its implementing guidelines − Executive Order No. 286. Because of
the reorganization, Manila Water absorbed some employees of
MWSS including Del Rosario. On 1 August 1997, Del Rosario officially
became an employee of Manila Water.

613
Page 614

Sometime in May 2000, Manila Water discovered that 24 water meters


were missing in its stockroom. Upon investigation, it appeared that Del
Rosario and his co-employee, Danilo Manguera, were involved in the
pilferage and the sale of water meters to the company’s contractor.
Consequently, Manila Water issued a Memorandum, directing Del
Rosario to explain in writing within 72 hours why he should not be dealt
with administratively for the loss of the said water meters. In his letter
explanation, Del Rosario confessed his involvement in the act charged
and pleaded for forgiveness, promising not to commit similar acts in
the future. On 29 June 2000, Manila Water conducted a hearing to
afford Del Rosario the opportunity to personally defend himself and to
explain and clarify his defenses to the charge against him. During the
formal investigation Del Rosario was found responsible for the loss of
the water meters and therefore liable for violating Section 11.1 of the
Company’s Code of Conduct. Manila Water proceeded to dismiss Del
Rosario from employment on 3 July 2000. This prompted Del Rosario
to file an action for illegal dismissal claiming that his severance from
employment is without just cause. He averred that his admission to
the misconduct charged was not voluntary but was coerced by the
company. Such admission therefore, made without the assistance of
a counsel, could not be made basis in terminating his employment.
Invoking Section 11.1 of the Company’s Code of Conduct, Manila
Water averred that such act of stealing the company’s property is
punishable by dismissal. The company invited the attention of this
Court to the fact that Del Rosario himself confessed his involvement
to the loss of the water meters not only in his letter-explanation, but
also during the formal investigation, and in both instances, pleaded for
his employer’s forgiveness.The matter was brought before the Labor
Arbiter which dismissed the case for lack of merit the complaint filed
by Del Rosario who was, however, awarded separation pay.
According to the Labor Arbiter, Del Rosario’s length of service for 21
years, without previous derogatory record, warrants the award of
separation pay. The NLRC on the other hand, dismissed the appeal
brought before it interposed by Manila Water for its failure to append
a certification against forum shopping in its Memorandum of Appeal.
The motion for reconsideration was also denied.

The Court of Appeals, on the other hand, reversed the NLRC


Resolution and held that it committed a grave abuse of discretion
when it dismissed Manila Water’s appeal on mere technicality. The
appellate court, however, proceeded to affirm the decision of the Labor
Arbiter awarding separation pay to Del Rosario.

ISSUE:

Should del Rosario be awarded with separation pay?

614
Page 615

RULING:

The Court ruled in the negative. And rationated that as a general rule,
an employee who has been dismissed for any of the just causes
enumerated under Article 282 of the Labor Code is not entitled to a
separation pay. The said article of the Omnibus Rules implementing
the Labor Code provides that the just causes for terminating the
services of an employee shall be those provided in Article 282 of the
Code. The separation from work of an employee for a just cause does
not entitle him to the termination pay provided in the Code, without
prejudice, however, to whatever rights, benefits and privileges he may
have under the applicable individual or collective agreement with the
employer or voluntary employer policy or practice.
In exceptional cases, however, the Court has granted separation pay
to a legally dismissed employee as an act of "social justice" or on
"equitable grounds." In both instances, it is required that the dismissal
(1) was not for serious misconduct; and (2) did not reflect on the moral
character of the employee. Where the reason for the valid dismissal
is, for example, habitual intoxication or an offense involving moral
turpitude, like theft or illicit sexual relations with a fellow worker, the
employer may not be required to give the dismissed employee
separation pay, or financial assistance, or whatever other name it is
called, on the ground of social justice. A contrary rule would, as the
petitioner correctly argues, have the effect, of rewarding rather than
punishing the erring employee for his offense. And we do not agree
that the punishment is his dismissal only and that the separation pay
has nothing to do with the wrong he has committed. Of course it has.
Indeed, if the employee who steals from the company is granted
separation pay even as he is validly dismissed, it is not unlikely that
he will commit a similar offense in his next employment because he
thinks he can expect a like leniency if he is again found out. This kind
of misplaced compassion is not going to do labor in general any good
as it will encourage the infiltration of its ranks by those who do not
deserve the protection and concern of the Constitution. The policy of
social justice is not intended to countenance wrongdoing simply
because it is committed by the underprivileged. At best, it may mitigate
the penalty but it certainly will not condone the offense. Compassion
for the poor is an imperative of every humane society but only when
the recipient is not a rascal claiming an undeserved privilege. Social
justice cannot be permitted to be refuge of scoundrels any more than
can equity be an impediment to the punishment of the guilty. Those
who invoke social justice may do so only if their hands are clean and
their motives blameless and not simply because they happen to be
poor. This great policy of our Constitution is not meant for the
protection of those who have proved they are not worthy of it, like the
workers who have tainted the cause of labor with the blemishes of their
own character.

615
Page 616

That Del Rosario rendered 21 years of service to the company will not
save the day for hm. As discussed in the case of Central Pangasinan
Electric Cooperative, Inc. v. NLRC is on all fours, that although long
years of service might generally be considered for the award of
separation benefits or some form of financial assistance to mitigate the
effects of termination, this case is not the appropriate instance for
generosity under the Labor Code nor under our prior decisions. The
fact that private respondent served petitioner for more than twenty
years with no negative record prior to his dismissal, in our view of this
case, does not call for such award of benefits, since his violation
reflects a regrettable lack of loyalty and worse, betrayal of the
company. If an employee's length of service is to be regarded as a
justification for moderating the penalty of dismissal, such gesture will
actually become a prize for disloyalty, distorting the meaning of social
justice and undermining the efforts of labor to cleanse its ranks of
undesirables.

616
Page 617

Case Digests by: PAJARILLAGA, JAN LAURENCE M.

DARIO NACAR v. GALLERY FRAMES AND/OR FELIPE


BORDEY, JR.,
G.R. No. 189871 : August 13, 2013
PERALTA,J.:

DOCTRINE:
INTEREST RATES IN LABOR DISPUTE PROCEEDS

By the nature of an illegal dismissal case, the reliefs continue to


add up until full satisfaction, as expressed under Article 279 of the
Labor Code. The recomputation of the consequences of illegal
dismissal upon execution of the decision does not constitute an
alteration or amendment of the final decision being implemented. The
illegal dismissal ruling stands; only the computation of monetary
consequences of this dismissal is affected, and this is not a violation
of the principle of immutability of final judgments. That the amount
respondents shall now pay has greatly increased is a consequence
that it cannot avoid as it is the risk that it ran when it continued to
seek recourses against the Labor Arbiter's decision.

A recomputation (or an original computation, if no previous


computation has been made) is a part of the law – specifically, Article
279 of the Labor Code and jurisprudence – that is read into the
decision. By the nature of an illegal dismissal case, the reliefs
continue to add up until full satisfaction, as expressed under Article
279. The recomputation of the consequences of illegal dismissal
upon execution of the decision does not constitute an alteration or
amendment of the final decision being implemented. The illegal
dismissal ruling stands; only the computation of monetary
consequences of this dismissal is affected, and this is not a violation
of the principle of immutability of final judgments. In this case, a re-
computation is necessary as it essentially considered the labor
arbiter's original decision. The first part contains the finding of
illegality and its monetary consequences; the second part is the
computation of the awards or monetary consequences of the illegal
dismissal, computed from the original decision. Under the decision,
no essential change is made by a recomputation as this step is a
necessary consequence that flows from the nature of the illegality of
dismissal declared by the LA in that decision. That the amount
respondents shall now pay has greatly increased is a consequence
that it cannot avoid as it is the risk that it ran when it continued to
seek recourses against the LA’s decision. Article 279 provides for the
consequences of illegal dismissal in no uncertain terms, qualified only
by jurisprudence in its interpretation of when separation pay in lieu of

617
Page 618

reinstatement is allowed. When that happens, the finality of the illegal


dismissal decision becomes the reckoning point instead of the
reinstatement that the law decrees.

In the absence of an express stipulation as to the rate of interest


that would govern the parties, the rate of legal interest for loans or
forbearance of any money, goods or credits and the rate allowed in
judgments shall no longer be twelve percent (12%) per annum - as
reflected in the case of Eastern Shipping Lines40 and the Manual of
Regulations for Non-Bank Financial Institutions, before its
amendment by BSP-MB Circular No. 799 - but will now be six percent
(6%) per annum effective July 1, 2013. It should be noted,
nonetheless, that the new rate could only be applied prospectively
and not retroactively. Consequently, the twelve percent (12%) per
annum legal interest shall apply only until June 30, 2013. Come July
1, 2013 the new rate of six percent (6%) per annum shall be the
prevailing rate of interest when applicable.Add the doctrine here.

FACTS:
Dario Nacar filed a complaint for constructive against
respondents Gallery Frames (GF) and/or Felipe Bordey, Jr. On
October 15, 1998, the Labor Arbiter rendered a Decision in favor of
petitioner and found that he was dismissed from employment without
a valid or just cause. Thus, petitioner was awarded backwages and
separation pay in lieu of reinstatement in the amount of P158,919.92.

Respondents appealed to the NLRC, but it was dismissed for


lack of merit. Accordingly, the NLRC sustained the decision of the
Labor Arbiter. Respondents filed a motion for reconsideration, but it
was denied. Dissatisfied, respondents filed a Petition for Review on
Certiorari before the CA but it was likewise denied. Respondents then
sought relief before the Supreme Court. Finding no reversible error
on the part of the CA, this Court denied the petition in the Resolution
dated April 17, 2002.

An Entry of Judgment was later issued certifying that the


resolution became final and executory on May 27, 2002. The case
was, thereafter, referred back to the Labor Arbiter for execution.
Petitioner filed a Motion for Correct Computation, praying that his
backwages be computed from the date of his dismissal on January
24, 1997 up to the finality of the Resolution of the Supreme Court on
May 27, 2002. Upon recomputation, the Computation and
Examination Unit of the NLRC arrived at an updated amount in the
sum of P471,320.31.

618
Page 619

Respondents filed a Motion to Quash Writ of Execution, arguing,


among other things, that since the Labor Arbiter awarded separation
pay of P62,986.56 and limited backwages of P95,933.36, no more
recomputation is required to be made of the said awards. They
claimed that after the decision becomes final and executory, the
same cannot be altered or amended anymore. LA denied the motion
but the decision was reversed by the NLRC on appeal.

Petitioner appealed to the CA but was denied, stating that since


petitioner no longer appealed the October 15, 1998 Decision of the
Labor Arbiter, which already became final and executory, a belated
correction thereof is no longer allowed. The CA stated that there is
nothing left to be done except to enforce the said judgment.
Consequently, it can no longer be modified in any respect, except to
correct clerical errors or mistakes. Thus, petitioner filed this petition
for review on certiorari.

ISSUE:
Is the re-computation in the course of execution of the labor
arbiter's original computation of the awards made is legally proper?
What is the applicable interest rate?

RULING:
Yes, the re-computation in the course of execution of the labor
arbiter's original computation of the awards made is legally proper.
Interest of twelve percent (12%) per annum of the total monetary
awards, computed from May 27, 2002 to June 30, 2013 and six
percent (6%) per annum from July 1, 2013 until their full satisfaction.
It must be stressed that the decision consists essentially of two parts.
The first is that part of the decision that cannot now be disputed
because it has been confirmed with finality. This is the finding of the
illegality of the dismissal and the awards of separation pay in lieu of
reinstatement, backwages, attorney's fees, and legal interests. The
second part is the computation of the awards made. Clearly implied
from this original computation is its currency up to the finality of the
labor arbiter's decision. As we noted above, this implication is
apparent from the terms of the computation itself, and no question
would have arisen had the parties terminated the case and
implemented the decision at that point.

The petitioner appropriately sought to nullify the NLRC decision


on jurisdictional grounds through a timely filed Rule 65 petition for
certiorari. The CA decision, finding that NLRC exceeded its authority
in affirming the payment of 13th month pay and indemnity, lapsed to
finality and was subsequently returned to the labor arbiter of origin for
execution. It was at this point that the present case arose. Focusing
on the core illegal dismissal portion of the original labor arbiter's

619
Page 620

decision, the implementing labor arbiter ordered the award re-


computed; he apparently read the figures originally ordered to be paid
to be the computation due had the case been terminated and
implemented at the labor arbiter's level. Thus, the labor arbiter re-
computed the award to include the separation pay and the
backwages due up to the finality of the CA decision that fully
terminated the case on the merits. Unfortunately, the labor arbiter's
approved computation went beyond the finality of the CA decision
and included as well the payment for awards the final CA decision
had deleted - specifically, the proportionate 13th month pay and the
indemnity awards. Hence, the CA issued the decision now
questioned in the present petition.

We see no error in the CA decision confirming that a re-


computation is necessary as it essentially considered the labor
arbiter's original decision in accordance with its basic component
parts as we discussed above. To reiterate, the first part contains the
finding of illegality and its monetary consequences; the second part is
the computation of the awards or monetary consequences of the
illegal dismissal, computed as of the time of the labor arbiter's original
decision.

By the nature of an illegal dismissal case, the reliefs continue to


add up until full satisfaction, as expressed under Article 279 of the
Labor Code. The recomputation of the consequences of illegal
dismissal upon execution of the decision does not constitute an
alteration or amendment of the final decision being implemented. The
illegal dismissal ruling stands; only the computation of monetary
consequences of this dismissal is affected, and this is not a violation
of the principle of immutability of final judgments. That the amount
respondents shall now pay has greatly increased is a consequence
that it cannot avoid as it is the risk that it ran when it continued to
seek recourses against the Labor Arbiter's decision.The Court of
Appeals Decision was then reversed and set aside..

620
Page 621

Case Digests by: PAJARILLAGA, JAN LAURENCE M.

BANI RURAL BANK VS. DE GUZMAN


G.R. No. 170904
November 13, 2013
Brion, J.
.

DOCTRINE:
COMPUTATION

The computation of the respondents' backwages must be from the time


of the illegal dismissal from employment until the finality of the decision
ordering the payment of separation pay.

The awards of reinstatement and separation pay are exclusive


remedies the change of awards, from reinstatement to separation pay,
not only modified the awards granted, but also changed the manner
the respondents backwages is to be computed. The respondents’
backwages can no longer be computed up to the point of reinstatement
as there is no longer any award of reinstatement to speak of.

As a rule, "a final judgment may no longer be altered, amended or


modified, even if the alteration, amendment or modification is meant to
correct what is perceived to be an erroneous conclusion of fact or law
and regardless of what court, be it the highest Court of the land,
rendered it. Any attempt on the part of the entities charged with the
execution of a final judgment to insert, change or add matters not
clearly contemplated in the dispositive portion violates the rule on
immutability of judgments." An exception to this rule is the existence of
supervening events which refer to facts transpiring after judgment has
become final and executory or to new circumstances that developed
after the judgment acquired finality, including matters that the parties
were not aware of prior to or during the trial as they were not yet in
existence at that time. Under the circumstances of this case, the
existence of the strained relations between the petitioners and the
respondents was a supervening event that justified the NLRC’s
modification of its final March 17, 1995 resolution.
Under the circumstances of this case, the existence of the strained
relations between the petitioners and the respondents was a
supervening event that justified the NLRC's modification of its final
March 17, 1995 resolution. The NLRC concluded that the award of
reinstatement was no longer possible; thus, it awarded separation pay,
in lieu of reinstatement. Unless exceptional reasons are presented,
these above findings and conclusion can no longer be disturbed after
they lapsed to finality.

621
Page 622

FACTS:
The respondents were employees of Bani Rural Bank, Inc. and ENOC
Theatre I and II who filed complaint for illegal dismissal against the
petitioners. The complaint was initially dismissed by the LA. The NLRC
reversed the LA’s finding and in its March 17, 1995 resolution ordered
the petitioners to reinstate the two complainants to their former
positions, without loss of seniority rights and other benefits and
privileges, with backwages from the time of their constructive dismissal
until their actual reinstatement, less earnings elsewhere. The parties
did not file any motion for reconsideration or appeal. The March 17,
1995 resolution of the NLRC became final and executory and the
computation of the awards was remanded to the LA for execution
purposes. First, the LA deducted the earnings derived by the
respondents either from Bani Rural Bank, Inc. or ENOC Theatre I and
II. Second, the LA fixed the period of backwages from the respondents'
illegal dismissal until August 25 1995 or the date when the respondents
allegedly manifested that they no longer wanted to be reinstated.

The respondents appealed LA’s computation with the NLRC. In a


decision dated July 31, 1998, the NLRC modified the terms of the
March 17, 1995 resolution insofar as it clarified the phrase less
earnings elsewhere. The NLRC additionally awarded the payment of
separation pay, in lieu of reinstatement, on account of the strained
relations between the parties.

The respondents filed a motion for reconsideration on whether the


award of backwages was still included in the judgment. The NLRC
dismissed the motion for having been filed out of time. On January 29,
1999, the July 31, 1998 decision of the NLRC lapsed to finality and
became executory. In the course of the recomputation, the petitioners
filed before the LA a Motion to Quash Writ of Execution and Suspend
Further Execution they reiterated their position that the respondents’
backwages should be computed only up to August 25, 1995. In an
order dated July 12, 2000, the LA computed the respondents’
backwages only up to August 25, 1995. The respondents appealed the
July 12, 2000 order of Labor Arbiter Gambito to the NLRC, which
reversed LA’s order. In its decision dated September 28, 2001, the
NLRC ruled that the computation of the respondents’ backwages
should be until January 29 1999 which was the date when the July 31,
1998 decision attained finality. CA affirmed the decision of the NLRC.

ISSUE:
Was the respondents’ backwages correctly computed under the
decision dated September 28, 2001 of the NLRC, as confirmed by the
CA, in light of the circumstance that there were two final NLRC
decisions affecting the computation of the backwages?

RULING:

622
Page 623

The Court ruled in the affirmative. The computation of the respondents'


backwages must be from the time of the illegal dismissal from
employment until the finality of the decision ordering the payment of
separation pay. It is only when the NLRC rendered its July 31, 1998
decision ordering the payment of separation pay (which both parties
no longer questioned and which thereafter became final) that the issue
of the respondents' employment with the petitioners was decided with
finality, effectively terminating it. The respondents' backwages,
therefore, must be computed from the time of their illegal dismissal
until January 29, 1999, the date of finality of the NLRC's July 31, 1998
Decision. As a final point, the CA s ruling must be modified to include
legal interest commencing from the finality of the NLRC's July 31, 1998
decision. The CA failed to consider that the NLRC's July 31, 1998
decision, once final, becomes a judgment for money from which
another consequence flows -the payment of interest in case of delay.
Under the circumstances, the payment of legal interest of six percent
(6) upon the finality of the judgment is proper. It is not barred by the
principle of immutability of judgment as it is compensatory interest
arising from the final judgment.
The computation of backwages depends on the final awards adjudged
as a consequence of illegal dismissal, in that: First, when
reinstatement is ordered, the general concept under Article 279 of the
Labor Code, as amended, computes the backwages from the time of
dismissal until the employee’s reinstatement. The computation of
backwages (and similar benefits considered part of the backwages)
can even continue beyond the decision of the labor arbiter or NLRC
and ends only when the employee is actually reinstated.
Second, when separation pay is ordered in lieu of reinstatement (in
the event that this aspect of the case is disputed) or reinstatement is
waived by the employee (in the event that the payment of separation
pay, in lieu, is not disputed), backwages is computed from the time of
dismissal until the finality of the decision ordering separation pay.
Third, when separation pay is ordered after the finality of the decision
ordering the reinstatement by reason of a supervening event that
makes the award of reinstatement no longer possible (as in the case),
backwages is computed from the time of dismissal until the finality of
the decision ordering separation pay.
The above computation of backwages, when separation pay is
ordered, has been the Court’s consistent ruling. In Session Delights
Ice Cream and Fast Foods v. Court Appeals Sixth Division, we
explained that the finality of the decision becomes the reckoning point
because in allowing separation pay, the final decision effectively
declares that the employment relationship ended so that separation
pay and backwages are to be computed up to that point.

623
Page 624

We may also view the proper computation of backwages (whether


based on reinstatement or an order of separation pay) in terms of the
life of the employment relationship itself.
When reinstatement is ordered, the employment relationship
continues. Once the illegally dismissed employee is reinstated, any
compensation and benefits thereafter received stem from the
employee’s continued employment. In this instance, backwages are
computed only up until the reinstatement of the employee since after
the reinstatement, the employee begins to receive compensation from
his resumed employment.
When there is an order of separation pay (in lieu of reinstatement or
when the reinstatement aspect is waived or subsequently ordered in
light of a supervening event making the award of reinstatement no
longer possible), the employment relationship is terminated only upon
the finality of the decision ordering the separation pay. The finality of
the decision cuts-off the employment relationship and represents the
final settlement of the rights and obligations of the parties against each
other. Hence, backwages no longer accumulate upon the finality of the
decision ordering the payment of separation pay since the employee
is no longer entitled to any compensation from the employer by reason
of the severance of his employment.
As the records show, the contending parties did not dispute the
NLRC’s order of separation pay that replaced the award of
reinstatement on the ground of the supervening event arising from the
newly-discovered strained relations between the parties. The parties
allowed the NLRC’s July 31, 1998 decision to lapse into finality and
recognized, by their active participation in the second computation of
the awards, the validity and binding effect on them of the terms of the
July 31, 1998 decision.
Under these circumstances, while there was no express modification
on the period for computing backwages stated in the dispositive portion
of the July 31, 1998 decision of the NLRC, it is nevertheless clear that
the award of reinstatement under the March 17, 1995 resolution (to
which the respondents backwages was initially supposed to have been
computed) was substituted by an award of separation pay. As earlier
stated, the awards of reinstatement and separation pay are exclusive
remedies; the change of awards (from reinstatement to separation pay)
under the NLRC s July 31, 1998 not only modified the awards granted,
but also changed the manner the respondents backwages is to be
computed. The respondents’ backwages can no longer be computed
up to the point of reinstatement as there is no longer any award of
reinstatement to speak of.

624
Page 625

A decision in an illegal dismissal case consists essentially of two


components: The first is that part of the decision that cannot now be
disputed because it has been confirmed with finality. This is the finding
of the illegality of the dismissal and the awards of separation pay in
lieu of reinstatement, backwages. The second part is the computation
of the awards made. That there is already a final and executory March
17, 1995 resolution finding that respondents have been illegally
dismissed, and awarding backwages and reinstatement, is not
disputed. That there, too, is the existence of another final and
executory July 31, 1998 decision modifying the reinstatement aspect
of the March 17, 1995 resolution, by awarding separation pay, is
likewise beyond dispute.

As a rule, "a final judgment may no longer be altered, amended or


modified, even if the alteration, amendment or modification is meant to
correct what is perceived to be an erroneous conclusion of fact or law
and regardless of what court, be it the highest Court of the land,
rendered it. Any attempt on the part of the entities charged with the
execution of a final judgment to insert, change or add matters not
clearly contemplated in the dispositive portion violates the rule on
immutability of judgments." An exception to this rule is the existence of
supervening events which refer to facts transpiring after judgment has
become final and executory or to new circumstances that developed
after the judgment acquired finality, including matters that the parties
were not aware of prior to or during the trial as they were not yet in
existence at that time.

Under the circumstances of this case, the existence of the strained


relations between the petitioners and the respondents was a
supervening event that justified the NLRC s modification of its final
March 17, 1995 resolution. The NLRC, in its July 31, 1998 decision,
based its conclusion that strained relations existed on the conduct of
the parties during the first execution proceedings before the Labor
Arbiter. The NLRC considered the delay in the respondents’
reinstatement and the parties conflicting claims on whether the
respondents wanted to be reinstated. The NLRC also observed that
during the intervening period from the first computation (which was
done in 1995) to the appeal and resolution of the correctness of the
first computation (subject of the NLRC s July 31, 1998 decision),
neither party actually did anything to implement the respondents’
reinstatement. The NLRC considered these, actions as indicative of
the strained relations between the parties so that neither of them
actually wanted to implement the reinstatement decree in the March
17, 1995 resolution. The NLRC concluded that the award of
reinstatement was no longer possible; thus, it awarded separation pay,
in lieu of reinstatement. Unless exceptional reasons are presented,
these above findings and conclusion can no longer be disturbed after
they lapsed to finality.

625
Page 626

Case Digests by: PAJARILLAGA, JAN LAURENCE M.

LARA’S GIFT AND DECOR vs. MIDTOWN INDUSTRIAL


SALES, INC.
G.R. No. 225433, August 28, 2019,
J. Carpio

DOCTRINE:
REVISED RULES ON RATES OF INTEREST.

The general rule is that the interest stipulated by the parties shall apply,
provided it is not excessive and unconscionable. Absent any
stipulation, the Court has consistently held that the prevailing legal
interest prescribed by the Bangko Sentral ng Pilipinas applies to loans
or forbearance of money, goods or credits, as well as to judgments.
The rates of interest stated in the guidelines on the imposition of
interests, as laid down in the landmark case of Eastern Shipping Lines,
Inc. v. Court of Appeals have already been modified in Bangko Sentral
ng Pilipinas Monetary Board (BSP-MB) Circular No. 799, Series of
2013, which reduced the rate of legal interest from twelve percent
(12%) per annum to six percent (6%) per annum.

Clearly, under the law and jurisprudence, the prevailing legal interest
prescribed by the Bangko Sentral ng Pilipinas applies, in the absence
of stipulated interest, on the following: (1) loans; (2) forbearance of
any money, goods or credits; and (3) judgments in litigations
involving loans or forbearance of money, goods or credits.

If the rate of interest is stipulated, such stipulated interest shall


apply and not the legal interest, provided the stipulated interest is not
excessive and unconscionable. The stipulated interest shall be
applied until full payment of the obligation because that is the law
between the parties. The legal interest only applies in the absence of
stipulated interest. Moreover, there should be no compounding of
interest, whether stipulated or legal, unless compounding is expressly
agreed upon in writing by the parties or mandated by law or
regulation.

FACTS:
Petitioner Lara's Gifts & Decors, Inc. (Lara’s) is engaged in the
business of manufacturing, selling, and exporting handicraft products.
Midtown Industrial Sales, Inc. (Midtown) is engaged in selling
industrial and construction materials and Lara is one of its clients.
Lara’s made several purchases from Midtown on a sixty (60)-day
credit term, with the condition that 24% interest per annum would be
charged on all accounts overdue, as stated in the sales invoices.

626
Page 627

Midtown was issued several checks which apparently bounced more


than once, even after repeated demands from Lara. Lara interposed
that non-payment was due to the poor quality of products delivered
by Midtown, which caused its failing the standard of its US buyers,
raising also the economic recession in USA and the fire incident in its
factory. The RTC ordered Lara to pay the 1.2 Million with 24%
interest and 50,000 attorney’s fees. On appeal, the CA sustained the
RTC findings and further ruled that the 24% interest should be
applied considering that petitioner has not shown that it was placed at
a disadvantage in its contractual relation with respondent.

ISSUE:
Should the 24% interest rate considered valid?

RULING:

The Court held that Lara’s in indeed liable in this case and discussed
the matters crucial to the determination of proper imposable interest
rates for matters as such at bar. The Court ruled that Lara's Gifts &
Decors, Inc. should pay respondent Midtown Industrial Sales, Inc.
₱1,263,104.22 representing the principal amount plus stipulated
interest at 24% per annum to be computed from 22 January 2008, the
date of extrajudicial demand, until full payment. The Court reiterated
that the legal interest on the 24% per annum interest due on the
principal amount accruing as of judicial demand, at the rate of 12% per
annum from the date of judicial demand on 5 February 2008 until 30
June 2013, and thereafter at the rate of 6% per annum from 1 July
2013 until full payment. In addition, the Court provided that the sum of
₱50,000.00 as attorney's fees, plus legal interest thereon at the rate of
6% per annum to be computed from the finality of this Decision until
full payment.

The Court further discussed in this case that in Asian Construction and
Development Corporation v. Cathay Pacific Steel Corporation, the
Court upheld the validity of interest rate fixed at 24% per annum that
was expressly stipulated in the sales invoices. The Court held that
petitioner is presumed to have full knowledge of the terms and
conditions of the contract and that by not objecting to the stipulations
in the sales invoice, it also bound itself to pay not only the stated selling
price but also the interest of 24% per annum on overdue accounts and
the 25% of the unpaid invoice for attorney's fees.

In the present case, petitioner, which has been doing business since
1990 and has been purchasing various materials from respondent
since 2004, cannot claim to have been misled into agreeing to the 24%
interest rate which was expressly stated in the sales invoices. Besides,
this Court has already ruled in several cases that an interest rate of
24% per annum agreed upon between the parties is valid and binding

627
Page 628

and not excessive and unconscionable. Thus, the stipulated 24%


interest per annum is binding on petitioner.

Clearly, under the law and jurisprudence, the prevailing legal interest
prescribed by the Bangko Sentral ng Pilipinas applies, in the absence
of stipulated interest, on the following: (1) loans; (2) forbearance of any
money, goods or credits; and (3) judgments in litigations involving
loans or forbearance of money, goods or credits.

It should be noted that under Section 1 of P.D. No.116, the prevailing


legal interest prescribed by the Bangko Sentral ng Pilipinas applies to
"judgments" in the absence of stipulated interest. Forbearance of
goods includes the sale of goods on installment, requiring periodic
payment of money to the creditor. Forbearance of credits includes the
sale of anything on credit, where the full amount due can be paid at a
date after the sale. This case involves a forbearance of credit wherein
petitioner was granted a 60-day credit term on its purchases, with the
condition that a 24% interest per annum would be charged on all
accounts overdue. Since there was an extra judicial demand before
the complaint was filed, interest on the amount due begins to run not
from the filing of the complaint but from the date of such extrajudicial
demand. Thus, the unpaid principal obligation of Pl,263,104.22 shall
earn the stipulated interest of 24% per annum from the date of
extrajudicial demand on 22 January 2008 until full payment.

The rates of interest stated in the guidelines on the imposition of


interests, as laid down in the landmark case of Eastern Shipping Lines,
Inc. v. Court of Appeals have already been modified in Bangko Sentral
ng Pilipinas Monetary Board (BSP-MB) Circular No. 799, Series of
2013, which reduced the rate of legal interest from twelve percent
(12%) per annum to six percent (6%)per annum.

As previously discussed, the general rule is that the interest stipulated


by the parties shall apply, provided it is not excessive and
unconscionable. Absent any stipulation, the Court has consistently
held that the prevailing legal interest prescribed by the Bangko Sentral
ng Pilipinas applies to loans or forbearance of money, goods or credits,
as well as to judgments.

628
Page 629

Case Digests by: PAJARILLAGA, JAN LAURENCE M.

Session Delights Ice Cream And Fast Food vs Court of


Appeals
G.R. No. 172149; February 8, 2010
Brion, J.

DOCTRINE:
BACKWAGES IN RELATION TO IMMUTABILITY OF
JUDGEMENTS

A judgment should be implemented according to the terms of its


dispositive portion is a long and well-established rule. Otherwise
stated, it is the dispositive portion that categorically states the rights
and obligations of the parties to the dispute as against each other.
Thus, it is the dispositive portion which the entities charged with the
execution of a final judgment that must be enforced to ensure the
validity of the execution.

Save for recognized exceptions, a final judgment may no longer be


altered, amended or modified, even if the alteration, amendment or
modification is meant to correct what is perceived to be an erroneous
conclusion of fact or law and regardless of what court, be it the highest
Court of the land, renders it. Any attempt on the part of the responsible
entities charged with the execution of a final judgment to insert, change
or add matters not clearly contemplated in the dispositive portion
violates the rule on immutability of judgments.

FACTS:

The private respondent filed against the petitioner a complaint for


illegal dismissal. The labor arbiter decided the complaint on February
8, 2001, finding that the petitioner illegally dismissed the private
respondent. The decision awarded the private respondent backwages,
separation pay in lieu of reinstatement, indemnity, and attorney’s fees,
under a computation that the decision itself.

On the petitioner’s appeal, the NLRC affirmed the labor arbiter’s


decision in its resolutions dated May 31, 2002 and September 30,
2002. The petitioner continued to seek relief, this time by filing a
petition for certiorari before the CA. On July 4, 2003, the CA dismissed
the petition and affirmed with modification the NLRC decision by
deleting the awards for a proportionate 13th month pay and for
indemnity. The CA decision became final per Entry of Judgment dated
July 29, 2003.

629
Page 630

While the case was still with the Labor Arbiter or sometime in January
2004, and in the course of the execution of the above final judgment
pursuant to Section 3, Rule VIII of the then NLRC Rules of Procedure,
the Finance Analyst Labor Arbiter’s Office submitted an updated
computation of the monetary awards due the private respondent in the
total amount of ₱235,986.00. This updated computation included
additional backwages and separation pay due the private respondent
computed from March 1, 2001 to September 17, 2003. The petitioner
objected to the re-computation and appealed the labor arbiter’s order
to the NLRC. The petitioner claimed that the updated computation was
inconsistent with the dispositive portion of the labor arbiter’s February
8, 2001 decision.

The matter was brought before the NLRC. The NLRC disagreed with
the petitioner and affirmed the labor arbiter’s decision in a resolution
dated October 25, 2004. The NLRC also denied the petitioner’s motion
for reconsideration in its resolution dated January 31, 2005.

The Court of Appeals partially granted the petition in its decision of


December 19, 2005 (now challenged before us) by deleting the
awarded proportionate 13th month pay. The Court of Appeals
explained in this ruling that employees illegally dismissed are entitled
to reinstatement, full backwages, inclusive of allowances and other
benefits or their monetary equivalent, computed from the time actual
compensation was withheld from them, up to the time of actual
reinstatement. If reinstatement is no longer feasible, the backwages
shall be computed from the time of their illegal dismissal up to the
finality of the decision. The Court of Appeals reasoned that a re-
computation of the monetary awards was necessary to determine the
correct amount due the private respondent from the time his salary was
withheld from him until July 29, 2003 which is the date of finality of the
July 4, 2003 decision in CA-G.R. SP No. 74653, since the separation
pay, which was awarded in lieu of reinstatement, had not been paid by
the petitioner. The attorney’s fees likewise have to be re-computed in
light of the deletion of the proportionate 13th month pay and indemnity
awards. The petitioner timely filed a motion for reconsideration which
the CA denied in its resolution.

ISSUE:

Is it justifiable that the final and executory decision or the labor arbiter’s
decision of February 8, 2001, as affirmed with modification by the Court
of Appeals may be enforced beyond the terms decreed in its
dispositive portion?

RULING:

630
Page 631

The Court stated at the outset that, as a rule, any delay in the execution
of final and executory decisions are frowned upon, as the immediate
enforcement of the parties’ rights, confirmed by a final decision, is a
major component of the ideal administration of justice. The Court
further stated, however, that circumstances may transpire rendering
certain delays unavoidable. One such occasion is when the execution
of the final judgment is not in accord with what the final judgment
decrees in its dispositive portion. Just as the execution of a final
judgment is a matter of right for the winning litigant who should not be
denied the fruits of his or her victory, the right of the losing party to give,
perform, pay, and deliver only what has been decreed in the final
judgment should also be respected.

That a judgment should be implemented according to the terms of its


dispositive portion is a long and well-established rule. Otherwise
stated, it is the dispositive portion that categorically states the rights
and obligations of the parties to the dispute as against each other.
Thus, it is the dispositive portion which the entities charged with the
execution of a final judgment that must be enforced to ensure the
validity of the execution.

A companion to the above rule on the execution of a final judgment is


the principle of its immutability. Save for recognized exceptions, a final
judgment may no longer be altered, amended or modified, even if the
alteration, amendment or modification is meant to correct what is
perceived to be an erroneous conclusion of fact or law and regardless
of what court, be it the highest Court of the land, renders it. Any attempt
on the part of the responsible entities charged with the execution of a
final judgment to insert, change or add matters not clearly
contemplated in the dispositive portion violates the rule on immutability
of judgments.

In the present case, with the CA’s deletion of the proportionate 13th
month pay and indemnity awards in the labor arbiter’s February 8,
2001 decision, only the awards of backwages, separation pay, and
attorney’s fees remain. These are the awards subject to execution.

631
Page 632

Case Digests by: PAJARILLAGA, JAN LAURENCE M.

UNITED COCONUT CHEMICALS, INC. VS. VICTORIANO


B. VALMORES
G.R. No. 201018 12 July 2017
BERSAMIN, J.:

DOCTRINE:
INCLUSION IN BACKWAGES, EMPLOYER LIABILITY IN ILLEGAL
DISMISSAL DONE IN RELATION TO A UNION SECURITY
CLAUSE

The base figure for the computation of backwages should include not
only the basic salary but also the regular allowances being received,
such as the emergency living allowances and the 13th month pay
mandated by the law. The purpose for this is to compensate the worker
for what he has lost because of his dismissal, and to set the price or
penalty on the employer for illegally dismissing his employee.

The base figure to be used in reckoning full backwages is the salary


rate of the employee at the time of his dismissal. The amount does not
include the increases or benefits granted during the period of his
dismissal because time stood still for him at the precise moment of his
termination, and move forward only upon his reinstatement. Hence,
the respondent should only receive backwages that included the
amounts being received by him at the time of his illegal dismissal but
not the benefits granted to his coemployees after his dismissal.

The petitioner, as the employer effecting the unlawful dismissal, was


solely liable for the backwages of the respondent, its employee.
Jurisprudence provides that the employer must still observe
substantive and procedural due process before complying with the
demand of a union to dismiss the expelled union members from
service. Failure of GMC to carry out this obligation makes it liable for
illegal dismissal of Casio, et al.
.

FACTS:

UCCI hired the respondent as its Senior Utilities Inspector with a


monthly salary of P11,194.00. He then became a member of the
United Coconut Chemicals, Inc. Employees' Labor Organization
(UELO) until his expulsion sometime in 1995. Due to the expulsion,
UELO formally demanded that UCCI terminate the services of the
respondent pursuant to the union security clause of the CBA. UCCI
dismissed him on February 22, 1996. He then filed a complaint for
illegal dismissal in the NLRC. LA dismissed the respondent’s

632
Page 633

complaint. NLRC reversed the Labor Arbiter. It declared that


respondents liable for illegal dismissal and ordered them to reinstate
complainant to his former position without loss of seniority rights and
with full backwages from the date of dismissal on 22 February 1996
to the date of actual reinstatement. The parties moved for
reconsideration. The NLRC denied the motions for reconsideration of
the respondent and UELO, but partially granted UCCI's motion by
granting its prayer to be exempted from paying backwages. CA
affirmed the NLRC decision in all respects.

Petitioner appealed to the Court which denied the petition for review
on certiorari. The denial became final and executory on February 26,
2004. Respondent moved for the execution of the judgment in his
favor. LA Lontoc ruled that the backwages due to respondent should
be computed by the wage rate at the time of his dismissal and included
in the computation 13th month pay and SIL but excluded CBA granted
benefits. However, NLRC reversed the LA and ruled to include benefits
granted under the CBA. The Court of Appeals affirmed the NLRC
decision. UCCI posits that in determining the respondent's backwages,
the prospective increases in wages as well as the benefits provided in
the CBA should be excluded; that, as a consequence, the base figure
for computing the respondent's backwages should be his basic salary
prevailing at the time of his dismissal, unqualified by deductions or
increases.

ISSUE:

What should have been the correct basis for computing the backwages
of the respondent?

What is the nature of UCCI's liability for payment of full backwages?

Should the benefits provided under the CBA should be excluded?

RULING:

Labor Arbiter Lontoc opined that the backwages due to the respondent
should be computed by excluding the benefits under the CBA. While
the term "backwages" used in Article 279 of the Labor Code includes
the benefits which the complainant should have received had he not
been dismissed from work, benefits which are not prescribed by law of
those referring to benefits granted by the employer either pursuant to
the CBA or its benevolence, cannot be recognized unless duly proved.
While complainant attached a few pages of what purports to be their
collective bargaining agreement, the effectivity date thereof was never
presented for the NLRC and for us to determine the dates of their
applicability.

633
Page 634

Thus, complainant's entitlement to these benefits was not substantially


proven. For the same reason, we have no basis to consider the same.
Except for the bare allegation that he should have been paid these
benefits, no proof of such grant was presented by complainant.
However, the legally mandated benefits are factored in the
computation.

The matter was brought before the NLRC which affirmed the Labor
Arbiter decision with modification. The case was remanded to the
Arbitration Branch of origin only for the purpose of recomputation of
complainant's full backwages using the Collective Bargaining
Agreement for the covered period as basis of computation. Petitioner
was directed to furnish the office of the Labor Arbiter's copies of the
Collective Bargaining Agreement pertinent thereto. Incidentally, the
Court of Appeals upheld the NLRC decision as the matter was raised
to the former court.
It was reminded by the Court that as a rule, backwages include all
benefits previously enjoyed by the illegally dismissed employee. The
extent of the backwages to be awarded to an illegally dismissed
employee has been set in Article 279 of the Labor Code which provides
that in cases of regular employment, the employer shall not terminate
the services of an employee except for a just cause or when authorized
by this Title. An employee who is unjustly dismissed from work shall be
entitled to reinstatement without loss of seniority rights and other
privileges and to his full backwages, inclusive of allowances, and to his
other benefits or their monetary equivalent computed from the time his
compensation was withheld from him up to the time of his actual
reinstatement.

The base figure for the computation of backwages should include not
only the basic salary but also the regular allowances being received,
such as the emergency living allowances and the 13th month pay
mandated by the law. The purpose for this is to compensate the worker
for what he has lost because of his dismissal, and to set the price or
penalty on the employer for illegally dismissing his employee.

The base figure to be used in reckoning full backwages is the salary


rate of the employee at the time of his dismissal. The amount does not
include the increases or benefits granted during the period of his
dismissal because time stood still for him at the precise moment of his
termination, and move forward only upon his reinstatement. Hence,
the respondent should only receive backwages that included the
amounts being received by him at the time of his illegal dismissal but
not the benefits granted to his coemployees after his dismissal.

The petitioner, as the employer effecting the unlawful dismissal, was


solely liable for the backwages of the respondent, its employee.

634
Page 635

In General Milling Corporation v. Casio, the Court explained the liability


of the employer in case of the unlawful termination pursuant to the
union security provision of the CBA, and explained that despite a
closed shop provision in the CBA and the expulsion of Casio, et al.
from IBP-Local 31, law and jurisprudence imposes upon GMC the
obligation to accord Casio, et al. substantive and procedural due
process before complying with the demand of IBP-Local 31 to dismiss
the expelled union members from service. The failure of GMC to carry
out this obligation makes it liable for illegal dismissal of Casio, et al.

In the case of Malayang Samahan ng mga Manggagawa sa M.


Greenfield, the Court held that notwithstanding the fact that the
dismissal was at the instance of the federation and that the federation
undertook to hold the company free from any liability resulting from the
dismissal of several employees, the company may still be held liable if
it was remiss in its duty to accord the would-be dismissed employees
their right to be heard on the matter.

635
Page 636

Case Digests by: PAJARILLAGA, JAN LAURENCE M.

UNIVERSAL ROBINA CORPORATION and LANCE Y.


GOKONGWEI vs. WILFREDO Z. CASTILLO
G.R. No. 189686 July 10, 2013
PEREZ, J.:

DOCTRINE:

Separation pay “as a measure of social justice” is allowed in those


instances where the employee is validly dismissed for causes other
than serious misconduct or those reflecting on his moral character.
Jurisprudence has added some more instances for proper denial of
separation pay from that other than those under Art. 282 of the Labor
Code, like willful disobedience, gross and habitual neglect of duty,
fraud or willful breach of trust, and commission of a crime against the
employer or his family which would preclude award of separation pay.

To summarize, the award of separation pay is authorized in the


situations dealt with in Article 283 and 284 of the Labor Code, but not
in terminations of employment based on instances enumerated in
Article 282.

Courts must be most judicious and circumspect in awarding


separation pay or financial assistance as the constitutional policy to
provide full protection to labor is not meant to be an instrument to
oppress the employers. In fine, we should be more cautious in
awarding financial assistance to the undeserving and those who are
unworthy of the liberality of the law.

FACTS:
Respondent Wilfredo Castillo (Castillo) was hired by petitioner
Universal Robina Corporation (URC) as a truck salesman on March
23, 1983 with a monthly salary of P4,000. He rose from ranks and
became a Regional Sales Manager until his dismissal on January 12,
2006. The area of Castillos responsibility covered some parts of
Laguna, including Lianas Supermart (Liana) in Laguna.

On August 19, 2005, URCs Credit and Collection Department (CCD)


Analyst noted an outright deduction in the amount of P72,000 tagged
as Gift Certificate (GC). This finding prompted URCs Corporate
Internal Audit (CIA) to conduct a routine audit of the unresolved
accounts of Lianas account receivables. The CIA suspected that
respondent might have committed an act of fraud against the company

636
Page 637

and Lianas for his personal gain. Lianas Vice President for Marketing
confirmed the receipt of the GCs by respondent. Hence, respondent
was asked to explain in writing why the company should not institute
the appropriate disciplinary action against him. Respondent denied
accepting any GC.

He was subsequently dismissed for breach of trust and confidence and


acts inimical to the interest of URC after a finding that he signed 2 blank
Charge Invoices despite full knowledge that the same will be used by
Liana’s Supermarket to cause the deduction of P72,000.00 against the
account of URC for the gift certificate that Castillo unduly received.
Castillo filed a complaint for illegal dismissal against URC. And on May
30, 2006, respondent filed a complaint for illegal dismissal against
petitioners URC and its President and Chief Operating Officer (COO)
Lance Gokongwei.

LA declared Castillo to have been illegally dismissed and ordered the


payment of backwages and separation pay. The NLRC reversed the
decision of LA after finding that URC had more than sufficient proof
that respondent violated its trust. MR of Castillo was denied. On
appeal, the CA upheld his dismissal but awarded him separation pay
"as a form of equitable relief” considering that he honorably served the
company for about 23 years and this is his only and first offense.

ISSUE:
Whether Castillo is entitled to separation pay?

RULING:
The Court ruled that Castillo is not entitled to separation pay. As the
rule now stands, the award of separation pay is authorized in the
situations dealt with in Article 283 and 284 of the Labor Code, but not
in terminations of employment based on instances enumerated in
Article 282. Labor adjudicatory officials and the CA must demur the
award of separation pay based on social justice when an employee’s
dismissal is based on serious misconduct or willful disobedience;
gross and habitual neglect of duty; fraud or willful breach of trust; or
commission of a crime against the person of the employer or his
immediate family. They must be most judicious and circumspect in
awarding separation pay or financial assistance as the constitutional
policy to provide full protection to labor is not meant to be an
instrument to oppress the employers. In fine, we should be more
cautious in awarding financial assistance to the undeserving and those
who are unworthy of the liberality of the law.

637
Page 638

The case of Philippine Long Distance Telephone Co. v. NLRC


enunciated the ruling that separation pay as a measure of social justice
is allowed in those instances where the employee is validly dismissed
for causes other than serious misconduct or those reflecting on his
moral character. The case of Toyota Motor Phils. Corp. Workers
Association (TMPCWA) v. NLRC expanded the doctrine laid down in
PLDT by adding dismissals other than those under Art. 282 of the
Labor Code, like willful disobedience, gross and habitual neglect of
duty, fraud or willful breach of trust, and commission of a crime against
the employer or his family which would preclude award of separation
pay.

Indeed, Castillo has committed acts constituting willful breach of trust


and confidence reposed on him by URC. The principal charge against
petitioner Castillo was hinged upon "unauthorized arrangements"
which he allegedly entered into. His neglect in signing the blank charge
invoices and his improper receipt of gift certificates for his personal
gain, when taken together constitute a breach of the trust and
confidence reposed on petitioner Castillo by URC.
As a Regional Sales Manager, petitioner Castillo occupied a position
or responsibility and as such, he should have known that he placed the
interests of the company at a disadvantage by signing the blank charge
invoices. Because of such act, private respondent URC was prejudiced
by no less than P72,000.00. This alone is sufficient cause for breach
of trust and loss of confidence.

638
Page 639

Case Digests by: PAJARILLAGA, JAN LAURENCE M.

MINETTE BAPTISTA, BANNIE EDSEL SAN MIGUEL, and


MA. FEDAYON vs. ROSARIO VILLANUEVA, et. al.
G.R. No. 194709, July 10, 2013
MENDOZA J.:

DOCTRINE:

Due process, as a constitutional precept, is satisfied when a person


was notified of the charge against him and was given an opportunity
to explain or defend himself. In administrative proceedings, the filing
of charges and giving reasonable opportunity for the person so
charged to answer the accusations against him constitute the
minimum requirements of due process. The essence of due process
is simply to be heard, or as applied to administrative proceedings, an
opportunity to explain one’s side, or an opportunity to seek a
reconsideration of the action or ruling complained of. Mere absence of
a one-on-one confrontation between the petitioners and their
complainants does not automatically affect the validity of the
proceedings before the Committee. Not all cases necessitate a trial-
type hearing.

As in this case, what is indispensable is that a party be given the right


to explain one’s side, which was adequately afforded. They were
properly notified of the charges. Any supposed procedural flaw in the
proceedings before the Committee was deemed cured when
petitioners were given the opportunity to be heard.

In labor cases, as in other administrative proceedings, substantial


evidence or such relevant evidence as a reasonable mind might accept
as sufficient to support a conclusion is required. All the prohibited acts
constituting unfair labor practice should materially relate to the workers'
right to self-organization. Unfortunately, petitioners failed to discharge
the burden required to prove the charge of ULP against the
respondents. Aside from their self-serving allegations, petitioners were
not able to establish how they were restrained or coerced by their union
in a way that curtailed their right to self-organization.

FACTS:

Petitioners were former union members of Radio Philippines Network


Employees Union (RPNEU), a legitimate labor organization and the
sole and exclusive bargaining agent of the rank and file employees of

639
Page 640

Radio Philippines Network (RPN), while the respondents were the


union’s elected officers and members.
On suspicion of union mismanagement, petitioners filed a complaint
for impeachment of their union president, Reynato Siozon, before the
executive board of RPN, which was eventually abandoned. They later
re-lodged the impeachment complaint, this time, against all the union
officers and members of RPNEU before the DOLE and likewise filed
various petitions for audit covering the period from 2000 to 2004.
Thereafter, complaints were filed before the Chairman of RPNEU’s
Committee on Grievance and Investigation (hereinafter referred to as
the Committee) against petitioners citing as grounds the "commission
of an act which violates RPNEU Constitution and By-Laws,
specifically, Article IX, Section 2.5 for urging or advocating that a
member start an action in any court of justice or external investigative
body against the Union or its officer without first exhausting all internal
remedies open to him or available in accordance with the CBL.
The Committee submitted their recommendation of expulsion from the
union to RPNEU’s Board of Directors which was affirmed by the
RPNEU’s Board of Directors. Through a Memorandum, petitioners
were served an expulsion notice from the union. Consequently, RPN
informed petitioners of the termination of their employment enforcing
Article II, Section 2 also known as the union security clause of their
current CBA.
Petitioners filed a complaint for ULP against the respondents arguing
that that they were denied substantive and procedural due process of
law when they were expelled from the RPNEU. First, they posit that
the procedure that should have been followed by the respondents in
resolving the charges against them should be that provided in their
Constitution, which requires that members put their grievance in
writing to be submitted to their union president, who shall strive to have
the parties settle their differences amicably. Petitioners maintain that
any form of grievance would be referred only to the committee upon
failure of the parties to settle amicably. They also point out that they
were not given the opportunity to personally face and confront their
accusers, which were violative of their right to examine the
complainants and the supposed charges against them.

ISSUE:

(a) Was the labor organization is guilty of unfair labor practice when it
expels members pursuant to the Union’s CBL?

(b) Were petitioners deprived of substantive and procedural due


process in their expulsion from the union?

640
Page 641

RULING:

(a) The Court ruled to the negative and discussed the well-settled rule
that workers’ and employers’ organizations shall have the right to draw
up their constitutions and rules to elect their representatives in full
freedom, to organize their administration and activities and to
formulate their programs. In this case, RPNEU’s Constitution and By-
Laws expressly mandate that before a party is allowed to seek the
intervention of the court, it is a pre-condition that he should have
availed of all the internal remedies within the organization. Petitioners
were found to have violated the provisions of the union’s Constitution
and By-Laws when they filed petitions for impeachment against their
union officers and for audit before the DOLE without first exhausting
all internal remedies available within their organization. This act is a
ground for expulsion from union membership. Thus, petitioners’
expulsion from the union was not a deliberate attempt to curtail or
restrict their right to organize, but was triggered by the commission of
an act, expressly sanctioned by the union’s Constitution and By-Laws.

Unfortunately, petitioners failed to discharge the burden required to


prove the charge of ULP against the respondents. Aside from their self-
serving allegations, petitioners were not able to establish how they
were restrained or coerced by their union in a way that curtailed their
right to self-organization. The records likewise failed to sufficiently
show that the respondents unduly persuaded management into
discriminating against petitioners. Other than to bring to its attention
their expulsion from the union, which in turn, resulted in the
implementation of their CBA’ s union security clause. As earlier stated,
petitioners had the burden of adducing substantial evidence to support
its allegations of ULP, which burden they failed to discharge. In fact,
both the NLRC and the CA found that petitioners were unable to prove
their charge of ULP against the respondents.

(b) As to substantive due process, the SC ruled that the expulsion of


petitioners was meritorious. It is well-settled that workers’ and
employers’ organizations shall have the right to draw up their
constitutions. In this case, RPNEU’s Constitution and By-Laws
expressly mandate that before a party is allowed to seek the
intervention of the court, it is a pre-condition that he should have
availed of all the internal remedies within the organization. Petitioners
were found to have violated such when they filed petitions for
impeachment against their union officers and for audit before the
DOLE without first exhausting all internal remedies available within
their organization. This act is a ground for expulsion from union

641
Page 642

membership. Thus, petitioners’ expulsion from the union was not a


deliberate attempt to curtail or restrict their right to organize, but was
triggered by the commission of an act, expressly sanctioned by their
Constitution.
Going now to the procedural aspect, the SC declared that the
expulsion was free from infirmities. Under RPNEU’s Constitution, any
charge against the member must be submitted to the Board of
Directors and not to the Committee. Although the respondents filed the
charge before the Committee, this supposed procedural flaw was
deemed cured when petitioners were given the opportunity to be
heard. Due process, as a constitutional precept, is satisfied when a
person was notified of the charge against him and was given an
opportunity to explain or defend himself. In administrative proceedings,
the filing of charges and giving reasonable opportunity for the person
so charged to answer the accusations against him constitute the
minimum requirements of due process. The essence of due process is
simply to be heard, or as applied to administrative proceedings, an
opportunity to explain one’s side, or an opportunity to seek a
reconsideration of the action or ruling complained of. It cannot be
denied that petitioners were properly notified of the charges filed
against them and were equally afforded the opportunity to present their
side.

642
Page 643

Case Digests by: PAJARILLAGA, JAN LAURENCE M.

BPI EMPLOYEES UNION-DAVAO CITY-FUBU (BPIEU-


DAVAO CITY-FUBU) v. BANK OF THE PHILIPPINE
ISLANDS (BPI)
G.R. No. 174912 July 24, 2013
Mendoza, J

DOCTRINE:

ART. 261. Jurisdiction of Voluntary Arbitrators or panel of Voluntary


Arbitrators. – x x x Accordingly, violations of a Collective Bargaining
Agreement, except those which are gross in character, shall no longer
be treated as unfair labor practice and shall be resolved as grievances
under the Collective Bargaining Agreement. For purposes of this
article, gross violations of Collective Bargaining Agreement shall mean
flagrant and/or malicious refusal to comply with the economic
provisions of such agreement. Clearly, only gross violations of the
economic provisions of the CBA are treated as ULP. Otherwise, they
are mere grievances.

FACTS:

A service agreement between Bank of the Philippine Islands (BPI))


and BPI Operations Management Corporation (BOMC) was initially
implemented in BPI's Metro Manila branches. In this agreement,
BOMC undertook to provide services such as check clearing, delivery
of bank statements, fund transfers, card production, operations
accounting and control, and cash servicing, conformably with BSP
Circular No. 1388.

The service agreement was likewise implemented in Davao City.

Later, a merger between BPI and Far East Bank and Trust Company
(FEBTC) took effect on April 10, 2000 with BPI as the surviving
corporation.

Thereafter, BPI's cashiering function and FEBTC's cashiering,


distribution and bookkeeping functions were handled by BOMC.
Consequently, twelve (12) former FEBTC employees were transferred
to BOMC to complete the latter's service complement.

643
Page 644

The Union then filed a formal protest. During the Labor Management
Conference, BPI invoked management prerogative stating that the
creation of the BOMC was to preserve more jobs and to designate it
as an agency to place employees where they were most needed. On
the other hand, the Union charged that BOMC undermined the
existence of the union since it reduced or divided the bargaining unit.
While BOMC employees perform BPI functions, they were beyond the
bargaining unit's coverage. In contracting out FEBTC functions to
BOMC, BPI effectively deprived the union of the membership of
employees handling said functions as well as curtailed the right of
those employees to join the union.

Particularly in dispute is the validity of the transfer of twelve (12) former


FEBTC employees to BOMC, instead of being absorbed in BPI after
the corporate merger. The Union claims that a union shop agreement
is stipulated in the existing CBA. It is unfair labor practice for employer
to outsource the positions in the existing bargaining unit.

ISSUE:
Was the act of BPI to outsource the cashiering, distribution and
bookkeeping functions to BOMC is not in conformity with the law and
the existing CBA and thus, constitute unfair labor practice?

RULING:

It is incomprehensible how the "reduction of positions in the collective


bargaining unit" interferes with the employees' right to self-
organization because the employees themselves were neither
transferred nor dismissed from the service. In one case, the Court held
that it is management prerogative to farm out any of its activities,
regardless of whether such activity is peripheral or core in nature. What
is of primordial importance is that the service agreement does not
violate the employee's right to security of tenure and payment of
benefits to which he is entitled under the law. Violations of a Collective
Bargaining Agreement, except those which are gross in character,
shall no longer be treated as unfair labor practice and shall be resolved
as grievances under the Collective Bargaining Agreement. For
purposes of this article, gross violations of Collective Bargaining
Agreement shall mean flagrant and/or malicious refusal to comply with
the economic provisions of such agreement. Clearly, only gross
violations of the economic provisions of the CBA are treated as ULP.
Otherwise, they are mere grievances.

644
Page 645

In the present case, the alleged violation of the union shop agreement
in the CBA, even assuming it was malicious and flagrant, is not a
violation of an economic provision in the agreement. The provisions
relied upon by the Union were those articles referring to the recognition
of the union as the sole and exclusive bargaining representative of all
rank-and-file employees, as well as the articles on union security,
specifically, the maintenance of membership in good standing as a
condition for continued employment and the union shop clause. It
failed to take into consideration its recognition of the bank’s exclusive
rights and prerogatives, likewise provided in the CBA, which included
the hiring of employees, promotion, transfers, and dismissals for just
cause and the maintenance of order, discipline and efficiency in its
operations.

The Union, however, insists that jobs being outsourced to BOMC were
included in the existing bargaining unit, thus, resulting in a reduction of
a number of positions in such unit. The reduction interfered with the
employees’ right to self-organization because the power of a union
primarily depends on its strength in number. It is incomprehensible
how the reduction of positions in the collective bargaining unit
interferes with the employees’ right to self-organization because the
employees themselves were neither transferred nor dismissed from
the service.

As far as the twelve (12) former FEBTC employees are concerned, the
Union failed to substantially prove that their transfer, made to complete
BOMC’s service complement, was motivated by ill will, anti-unionism
or bad faith so as to affect or interfere with the employees’ right to self-
organization.

It is to be emphasized that contracting out of services is not illegal per


se. It is an exercise of business judgment or management prerogative.
Absent proof that the management acted in a malicious or arbitrary
manner, the Court will not interfere with the exercise of judgment by
an employer. In this case, bad faith cannot be attributed to BPI
because its actions were authorized by CBP Circular No. 1388, Series
of 199333 issued by the Monetary Board of the then Central Bank of
the Philippines. The circular covered amendments in Book I of the
Manual of Regulations for Banks and Other Financial Intermediaries,
particularly on the matter of bank service contracts. A finding of ULP
necessarily requires the alleging party to prove it with substantial
evidence. Unfortunately, the Union failed to discharge this burden. The
Court agrees with BPI that D.O. No. 10 is but a guide to determine
what functions may be contracted out, subject to the rules and

645
Page 646

established jurisprudence on legitimate job contracting and prohibited


labor-only contracting.

646
Page 647

Case Digests by: PAJARILLAGA, JAN LAURENCE M.

INTEGRATED MICROELECTRONICS, INC. vs PIONILLA


G.R. No. 200222 August 28, 2013
PERLAS-BERNABE, J.:

DOCTRINE:
As a general rule, an illegally dismissed employee is entitled to
reinstatement or separation pay, if reinstatement is not viable and
payment of full backwages. In certain cases, however, the Court has
carved out an exception to the foregoing rule and thereby ordered the
reinstatement of the employee without backwages on account of the
following: (a) the fact that dismissal of the employee would be too harsh
of a penalty; and (b) that the employer was in good faith in terminating
the employee.

FACTS:
Respondent Adonis Pionilla was hired by petitioner IMI as its
production worker. On May 5, 2005, Pionilla received a notice from IMI
requiring him to explain the incident which occurred the day before
where he was seen escorting a lady to board the company shuttle bus
at the Alabang Terminal. It was reported by the bus marshall that the
lady was wearing a company identification card (ID) – which serves as
a free pass for shuttle bus passengers – even if she was just a job
applicant at IMI. In this regard, Pionilla admitted that he lent his
temporary second ID to the lady who turned out to be his relative. He
further intimated that he risked lending her his ID to save on their
transportation expenses. Nevertheless, he apologized for his actions.
A Committee was formed to investigate the matter, and found that
Pionilla violated Article 6.12 of the Company Rules and Regulations
(CRR) which prohibits the lending of ones ID since the same is
considered a breach of its security rules and carries the penalty of
dismissal. Subsequently, Pionilla received a letter dated August 16,
2005 informing him of his dismissal from service. Three days after, he
filed a complaint for illegal dismissal with damages against IMI.LA ruled
that Pionilla has been illegally dismissed. On appeal, the NLRC
reversed the LAs ruling, finding Pionillas dismissal to be valid. It
pointed out that Pionillas act of lending his temporary ID was willful and
intentional. Dissatisfied, Pionilla filed a petition for certiorari before the
CA. The CA rendered a Decision granting Pionillas petition. It found
that while IMIs regulations on company IDs were reasonable, the
penalty of dismissal was too harsh and not commensurate to the
misdeed committed. In view of the CAs ruling, IMI filed a petition for
review on certiorari before the Supreme Court but the same was
denied. Hence, motion for reconsideration was filed.

647
Page 648

ISSUE:
Whether or not Pionilla is entitled to reinstatement with full back
wages.

RULING:
The Supreme Court partially granted the petition of IMI, holding that
while Pionilla should be reinstated, he is not entitled to backwages. As
a general rule, an illegally dismissed employee is entitled to
reinstatement or separation pay, if reinstatement is not viable and
payment of full backwages. In certain cases, however, the Court has
carved out an exception to the foregoing rule and thereby ordered the
reinstatement of the employee without backwages on account of the
following: (a) the fact that dismissal of the employee would be too
harsh of a penalty; and (b) that the employer was in good faith in
terminating the employee.

The Court is convinced that petitioner's guilt was substantially


established. Nevertheless, The Court agreed with respondent
Minister's order of reinstating petitioner without backwages instead of
dismissal which may be too drastic. The Court further stated that the
penalty of dismissal was too harsh of a penalty to be imposed against
Pionilla for his infractions and IMI was in good faith when it dismissed
Pionilla as his dereliction of its policy on ID usage was honestly
perceived to be a threat to the company's security. Denial of
backwages would sufficiently penalize the infraction at bar. The
emplyer acted in good faith. They should be exempt from the burden
of paying backwages. The good faith of the employer, when clear
under the circumstances, may preclude or diminish recovery of
backwages. Only employees discriminately dismissed are entitled to
backpay.

648
Page 649

Case Digests by: PALOMAR, ANDREI ANNE U.

GOLDEN ACE BUILDERS VS. TALDE


G.R. No. 187200. May 5, 2010.
Carpio Morales, J.

DOCTRINE:
ARTICLE 294 [279]; DOCTRINE OF STRAINED RELATIONS
Under the doctrine of strained relations, the payment of separation
pay is considered an acceptable alternative to reinstatement when the
latter option is no longer desirable or viable. On one hand, such
payment liberates the employee from what could be a highly
oppressive work environment. On the other hand, it releases the
employer from the grossly unpalatable obligation of maintaining in its
employ a worker it could no longer trust.

FACTS:
In 1990, respondent Jose Talde was hired as a carpenter by
petitioner Golden Ace Builders of which its co-petitioner Arnold Azul
(Azul) is the owner-manager. In February 1999, Azul, alleging the
unavailability of construction projects, stopped giving work
assignments to respondent, prompting the latter to file a complaint for
illegal dismissal.

The Labor Arbiter ruled in favor of Talde and ordered his


immediate reinstatement without loss of seniority rights and other
privileges, and with payment of full backwages and an additional
amount representing premium pay for rest days, service incentive
leave pay and 13th month pay.

Pending appeal to the NLRC, petitioners advised Talde to report


for work. However, Talde manifested that actual animosities existed
between him and petitioners and there had been threats to his life and
his family's safety, hence, he opted for the payment of separation pay.

The monetary liability of petitioner was recomputed which the


petitioner found to be exorbitant. It also contends that since respondent
refused to report back to work, he should be considered to have
abandoned the same.

The NLRC granted petitioners' motion and held that since


respondent did not appeal the Decision of the Labor Arbiter granting
him only reinstatement and backwages, not separation pay in lieu
thereof, he may not be afforded affirmative relief; and since he refused

649
Page 650

to go back to work, he may recover backwages only up to May 20,


2001, the day he was supposed to return to the job site.

ISSUE:
Whether or not respondent Talde may be denied of separation
pay, in lieu of reinstatement and backwages.

RULING:
No. Under the doctrine of strained relations, the payment of
separation pay is considered an acceptable alternative to
reinstatement when the latter option is no longer desirable or viable.
On one hand, such payment liberates the employee from what could
be a highly oppressive work environment. On the other hand, it
releases the employer from the grossly unpalatable obligation of
maintaining in its employ a worker it could no longer trust.

Strained relations must be demonstrated as a fact, however, to be


adequately supported by evidence — substantial evidence to show
that the relationship between the employer and the employee is indeed
strained as a necessary consequence of the judicial controversy.

In the present case, the Labor Arbiter found that actual animosity
existed between petitioner Azul and respondent as a result of the filing
of the illegal dismissal case. Such finding, especially when affirmed by
the appellate court as in the case at bar, is binding upon the Court,
consistent with the prevailing rules that this Court will not try facts anew
and that findings of facts of quasi-judicial bodies are accorded great
respect, even finality.

Clearly then, respondent is entitled to backwages and separation


pay as his reinstatement has been rendered impossible due to strained
relations. As correctly held by the appellate court, the backwages due
respondent must be computed from the time he was unjustly dismissed
until his actual reinstatement, or from February 1999 until June 30,
2005 when his reinstatement was rendered impossible without fault on
his part.

650
Page 651

Case Digests by: PALOMAR, ANDREI ANNE U.

METROGUARDS SECURITY AGENCY CORPORATION


VS. HILONGO
G.R. No. 215630. March 9, 2015.
Villarama, Jr., J.

DOCTRINE:
ARTICLE 294 [279]; RE-COMPUTATION OF MONETARY
CONSEQUENCES AFTER FINALITY OF JUDGMENT
The re-computation of the consequences of illegal dismissal upon
execution of the decision does not constitute an alteration or
amendment of the final decision being implemented. The illegal
dismissal ruling stands; only the computation of monetary
consequences of this dismissal is affected, and this is not a violation of
the principle of immutability of final judgments.

FACTS:
In the previous case of Alberto Hilongo v. Bee Guards
Corp./Milagros Chan, the Labor Arbiter ruled that respondent Alberto
N. Hilongo was illegally dismissed. However, it was reversed by the
NLRC. Upon appeal, the CA reversed the decision of the NLRC.

The petitioners did not appeal the CA’s decision. Hence, Hilongo
then filed a motion for entry of judgment and a motion for clarification
of Decision/Resolution praying that the CA's March 26, 2013
Resolution be clarified and interpreted to include the amount of the
award as stated in the Labor Arbiter's Decision dated April 30, 2010
and additional award computed from May 1, 2010 to March 26, 2013,
or the date the CA denied petitioners' motion for reconsideration.

The CA held that when an appellate court affirms the Labor


Arbiter's ruling, it is understood that awards due to the illegally
dismissed employee shall be recomputed in order to account for the
period of time that has lapsed from the rendition of the Labor Arbiter's
decision up to its finality.

On July 9, 2013, respondent Hilongo filed a motion for issuance of


writ of execution alleging that the June 11, 2013 CA Resolution had
confirmed that the amount of P170,520.31 awarded by the Labor
Arbiter is not sufficient, and that there is a need to compute additional
monetary awards reckoned from May 1, 2010 up to April 26, 2013 or
the date Hilongo presumed as the date of finality of the decision.

651
Page 652

The Labor Arbiter directed the issuance of a writ of execution and


ruled that the award of P170,520.31 as stated in the Labor Arbiter's
Decision dated April 30, 2010 prevails.

ISSUE:
Whether or not there should be additional monetary awards
reckoned from May 1, 2010 up to April 26, 2013 or the date Hilongo
presumed as the date of finality of the decision.

RULING:
Yes. We thus cannot agree with petitioners' contention that a
decision that has acquired finality becomes immutable and unalterable.
The re-computation of the consequences of illegal dismissal upon
execution of the decision does not constitute an alteration or
amendment of the final decision being implemented. The illegal
dismissal ruling stands; only the computation of monetary
consequences of this dismissal is affected, and this is not a violation of
the principle of immutability of final judgments.

The CA Decision dated September 7, 2012 became final and


executory on April 26, 2013. Thus, the April 30, 2010 Decision of the
Labor Arbiter which ordered the payment of separation pay in lieu of
reinstatement, effectively ended the employment relationship of the
parties on April 26, 2013, the date the CA decision became final. Since
the Labor Arbiter's computation of Hilongo's monetary award was up
to the date of his April 30, 2010 Decision only, the CA properly decreed
the computation of additional back wages and separation pay.

However, the CA incorrectly concluded that the April 30, 2010


Decision of the Labor Arbiter became final on June 11, 2013, contrary
to its own finding that it became final and executory on April 26, 2013.
This led to its erroneous computation of the additional back wages and
separation pay of Hilongo, as well as reckoning the date of the 12%
legal interest. Following the teaching of Nacar v. Gallery Frames that
the computation of the monetary consequences (back wages and
separation pay) of the illegal dismissal decision should be reckoned
from its finality, the additional back wages and separation pay of
Hilongo should be computed from May 1, 2010 to April 26, 2013.
Further, the payment of legal interest of 12% per annum should also
be from April 26, 2013 up to June 30, 2013. Thereafter, in accordance
with Bangko Sentral ng Pilipinas Monetary Board's Circular No. 799,
series of 2013, the legal interest computed from July 1, 2013 until the
monetary awards were fully satisfied will be 6% per annum.

652
Page 653

Case Digests by: PALOMAR, ANDREI ANNE U.

MAERSK-FILIPINAS CREWING INC. VS. AVESTRUZ


G.R. No. 207010. February 18, 2015.
Perlas-Bernabe, J.

DOCTRINE:
ARTICLE 294 [279]; TERMINATION OF EMPLOYMENT FOR JUST
CAUSE
Insubordination, as a just cause for the dismissal of an employee,
necessitates the concurrence of at least two requisites: (1) the
employee's assailed conduct must have been willful, that is,
characterized by a wrongful and perverse attitude; and (2) the order
violated must have been reasonable, lawful, made known to the
employee, and must pertain to the duties which he had been engaged
to discharge.

FACTS:
Avestruz was hired as Chief Cook on board vessel M/V Nedlloyd
Drake by petitoenr Maersk. Avestruz had an altercation with Captain
Woodward which led to his dismissal.

Hence, he filed a complaint for illegal dismissal, payment for the


unexpired portion of his contract, damages, and attorney's fees against
Maersk, among others. Avestruz alleges that no investigation or
hearing was conducted nor was he given the chance to defend himself
before he was dismissed, and that Captain Woodward failed to
observe the provisions under Section 17 of the POEA-SEC on
disciplinary procedures. Also, he averred that he was not given any
notice stating the ground for his dismissal. Additionally, he claimed that
the cost of his airfare in the amount of US$606.15 was deducted from
his wages.

The petitioners argued that during his stint on the vessel, Avestruz
failed to perform his duties properly and when asked to comply, he
became angry and snapped. As a result, Captain Woodward initiated
disciplinary proceedings and informed Avestruz during the hearing of
the offenses he committed. Thereafter, he was informed of his
dismissal from service due to insubordination. Relative thereto,
Captain Woodward sent two e-mails to Maersk explaining the decision
to terminate Avestruz's employment and requesting for Avestruz's
replacement.

653
Page 654

Petitioners maintained that Avestruz was dismissed for a just and


valid cause and is, therefore, not entitled to recover his salary for the
unexpired portion of his contract. They likewise claimed that they were
justified in deducting his airfare from his salary, and that the latter was
not entitled to moral and exemplary damages and attorney's fees.

ISSUE:
Whether or not Avestruz was legally dismissed.

RULING:
No. Insubordination, as a just cause for the dismissal of an
employee, necessitates the concurrence of at least two requisites: (1)
the employee's assailed conduct must have been willful, that is,
characterized by a wrongful and perverse attitude; and (2) the order
violated must have been reasonable, lawful, made known to the
employee, and must pertain to the duties which he had been engaged
to discharge.

In this case, the contents of Captain Woodward's e-mails do not


establish that Avestruz's conduct had been willful, or characterized by
a wrongful and perverse attitude.

Conversely, apart from Captain Woodward's e-mails, no other


evidence was presented by the petitioners to support their claims.
While rules of evidence are not strictly observed in proceedings before
administrative bodies, petitioners should have offered additional proof
to corroborate their statements.

It was incumbent upon the petitioners to present other substantial


evidence to bolster their claim that Avestruz committed acts that
constitute insubordination as would warrant his dismissal. At the least,
they could have offered in evidence entries in the ship's official logbook
showing the infractions or acts of insubordination purportedly
committed by Avestruz, the ship's logbook being the official repository
of the day-to-day transactions and occurrences on board the vessel.
Having failed to do so, their position that Avestruz was lawfully
dismissed cannot be sustained.

654
Page 655

Case Digests by: PALOMAR, ANDREI ANNE U.

VILLENA VS. BATANGAS II ELECTRIC


G.R. No. 205735. February 4, 2015.
Perl;as-Bernabe, J.

DOCTRINE:
ARTICLE 294 [279]; INCLUDED IN “OTHER BENEFITS”
• ON RETIREMENT PAY: In order for her retirement pay claim to
be considered, Villena's complaint should have contained
substantial allegations which would show that she (a) had
applied for the same, and (b) her application squares with the
requirements of entitlement under the terms of the company's
retirement plan which, in fact, was issued on September 20,
2003, or after the August 31, 2001 CA Decision had already
attained finality.
• ON TRANSPORTATION, REPRESENTATION, AND
CELLULAR PHONE USAGE ALLOWANCES: It is clear from
BATELEC II's pleadings and submissions that representation
allowance, transportation allowance, and cellular phone usage
allowance are given to the Finance Manager/Department
Manager as part of their benefits, unlike the separate entitlement
to retirement pay which may be recovered only upon a
meritorious subsequent application when the employee decides
to retire. Consequently, these allowances ought to be included in
the "other benefits pertaining to the position of Finance Manager"
to which Villena is entitled to and which were awarded to her
under the final and executory CA Decision and NLRC Resolution.

FACTS:
Villena was hired by respondent BATELEC II as bookkeeper in
1978. She rose from the ranks and was promoted as Finance Manager
in 1985. In 1994, she was demoted to the position of Auditor, which
caused her to file a complaint for constructive dismissal before the
Labor Arbiter.

The NLRC reversed the ruling of the LA and declared that Villena
was illegally dismissed. It ordered BATELEC II to reinstate her to her
former position as Finance Manager, or its equivalent, and to pay her
salary differentials. However, the NLRC's judgment was silent on the
payment of allowances, benefits, and attorney's fees. Hence, Villena
moved for reconsideration, but was denied.

Upon appeal, the CA declared Villena to be "entitled to the


difference between the salary of the Finance Manager and that of the

655
Page 656

auditor, plus allowances and any other benefits pertaining to the


position of Finance Manager at the time she was removed therefrom
up to the date of her actual reinstatement." It also granted her
attorney's fees in the amount of 10% of the total monetary award. The
case was remanded to the LA for the computation of the total amount
due to Villena.

The LA declared that Villena was entitled only to "salary


differentials, 13th month pay, unused sick leave, leave of absence"
amounting to P1,078,890.14, excluding from the computation claims
for bonus, representation allowance, transportation benefits, and
attorney's fees. Moreover, her claim for separation pay in lieu of
reinstatement was denied.

ISSUE:
Whether or not (a) retirement pay, and (b) representation,
transportation, and cellular phone usage allowances should be
awarded in favor of Villena as part of “other benefits”.

RULING:
A. ON RETIREMENT PAY.

As the Court sees it, the "other benefits" mentioned in these rulings
cannot be construed to include retirement pay for the primary reason
that they adjudged awards relative to Villena's illegal dismissal
complaint, which remains barren of a specific cause of action for
retirement pay. In order for her retirement pay claim to be considered,
Villena's complaint should have contained substantial allegations
which would show that she (a) had applied for the same, and (b) her
application squares with the requirements of entitlement under the
terms of the company's retirement plan which, in fact, was issued on
September 20, 2003, or after the August 31, 2001 CA Decision had
already attained finality. However, based on the records, what she
sought for in her illegal dismissal complaint were the reliefs of
reinstatement, payment of salary differentials, all benefits and
allowances that she may have received as Finance Manager,
attorney's fees, and damages. Thus, as the matter left for
determination is whether or not the aforesaid rulings, when executed,
should include retirement pay and representation, transportation, and
cellular phone usage allowances, the Court will harken back only to the
context of the illegal dismissal complaint from which such awards of
"other benefits" stemmed from.

B. ON TRANSPORTATION, REPRESENTATION, AND CELLULAR


PHONE USAGE ALLOWANCES.

656
Page 657

It is clear from BATELEC II's pleadings and submissions that


representation allowance, transportation allowance, and cellular phone
usage allowance are given to the Finance Manager/Department
Manager as part of their benefits, unlike the separate entitlement to
retirement pay which may be recovered only upon a meritorious
subsequent application when the employee decides to retire.
Consequently, these allowances ought to be included in the "other
benefits pertaining to the position of Finance Manager" to which Villena
is entitled to and which were awarded to her under the final and
executory CA Decision and NLRC Resolution.

657
Page 658

Case Digests by: PALOMAR, ANDREI ANNE U.

SANGWOO PHILIPPINES, INC. VS. SANGWOO


PHILIPPINES, INC. EMPLOYEES UNION-OLALIA
G.R. No. 173154. December 9, 2013.
Perlas-Bernabe, J.

DOCTRINE:
ARTICLE 294 [279]; BUSINESS CLOSURE DUE TO SERIOUS
BUSINESS LOSSES; SEPARATION BENEFITS
Article [297] of the Labor Code does not obligate an employer to
pay separation benefits when the closure is due to serious losses. To
require an employer to be generous when it is no longer in a position
to do so, in our view, would be unduly oppressive, unjust, and unfair to
the employer.

FACTS:
On July 25, 2003, during CBA SPEU and SPI, the latter filed with
the DOLE a letter-notice of temporary suspension of operations for one
month due to lack of orders from its buyers. SPEU was furnished a
copy of the said letter. Negotiations on the CBA, however, continued
and the parties signed a handwritten Memorandum of Agreement,
which, among others, specified the employees' wages and benefits for
the next two years, and that in the event of a temporary shutdown, all
machineries and raw materials would not be taken out of the SPI
premises.

In 2004, SPI declared its permanent closure and cessation of


business operations. SPI offered separation benefits of one-half (1/2)
month pay for every year of service to each of its employees. 234
employees of SPI accepted the offer, received the said sums and
executed quitclaims. Those who refused the offer, (the minority
employees), were nevertheless given until March 25, 2004 to accept
their checks and correspondingly, execute quitclaims. However, the
minority employees did not claim the said checks.

ISSUE:
Whether or not the minority employees are entitled to separation
pay.

RULING:
No. Closure of business, as an authorized cause for termination of
employment, aims to prevent further financial drain upon an employer
who cannot pay anymore his employees since business has already

658
Page 659

stopped. In such a case, the employer is generally required to give


separation benefits to its employees, unless the closure is due to
serious business losses. As explained in the case of Galaxie Steel
Workers Union (GSWU-NAFLU-KMU) v. NLRC:

The Constitution, while affording full protection to labor,


nonetheless, recognizes "the right of enterprises to reasonable
returns on investments, and to expansion and growth." In line with
this protection afforded to business by the fundamental law, Article
[297] of the Labor Code clearly makes a policy distinction. It is only
in instances of "retrenchment to prevent losses and in cases of
closures or cessation of operations of establishment or
undertaking not due to serious business losses or financial
reverses" that employees whose employment has been
terminated as a result are entitled to separation pay. In other
words, Article [297] of the Labor Code does not obligate an
employer to pay separation benefits when the closure is due
to serious losses. To require an employer to be generous
when it is no longer in a position to do so, in our view, would
be unduly oppressive, unjust, and unfair to the employer.
Ours is a system of laws, and the law in protecting the rights
of the working man, authorizes neither the oppression nor the
self-destruction of the employer.

In this case, the LA, NLRC, and the CA all consistently found that
SPI indeed suffered from serious business losses which resulted in its
permanent shutdown and accordingly, held the company's closure to
be valid. The Court thus holds that SPI is not obliged to give separation
benefits to the minority employees pursuant to Article 297 of the Labor
Code as interpreted in the case of Galaxie.

659
Page 660

Case Digests by: PALOMAR, ANDREI ANNE U.

PAPERTECH, INC. VS. KANDO


G.R. No. 236020. January 8, 2020.
Carandang, J.

DOCTRINE:
ARTICLE 294 [279]; STRAINED RELATIONS
In the case of Digital Telecommunications Philippines, Inc. v.
Digitel Employees Union, We held that the length of time from the
occurrence of the incident to its resolution and the demonstrated
litigiousness of the parties showed that their relationship is strained.

FACTS:
Katando was hired by Papertech as a machine operator. In 2008,
Ppaertech filed a complaint for illegal strike against Katando and other
participants and dismissed the latter. However, the NLRC ruled for
their reinstatement.

In 2013, Katando received a memorandum from Papertech stating


that due to urgency of business, she will be transferred to its Makati
office. Katando was suspended due to to her refusal. She then filed a
complaint for illegal suspension before the NLRC.

Papertech repeatedly asked Katando to explain her refusal to


comply with her transfer. Despite submitting her explanation,
Papertech dismissed Katando of her insubordination. Katando filed a
complaint for illegal dismissal, moral and exemplary damages, and
attorney's fees against Papertech.

The LA ruled in favor of Katando. The NLRC agreed with the Labor
Arbiter that separation pay should be given to Katando in lieu of her
reinstatement.

On August 18, 2017, the CA granted Katando's petition and


ordered Papertech to immediately reinstate her to her previous position
without loss of seniority rights in addition to the award of backwages.

The CA ruled that the doctrine of strained relations cannot apply


to Katando as she is part of the rank and file workforce and does not
occupy a managerial or key position in the company. She even asked
for her reinstatement. In addition, there is no proof of strained relations
between her and Papertech.

660
Page 661

ISSUE:
Whether or not Katando should be reinstated instead of granting
her separation pay.

RULING:
No. In Balaquezon, the Court awarded backwages as severance
pay based on equity. The Court explained, "[t]his means that a
monetary award is to be paid to the striking employees as an
alternative to reinstatement which can no longer be effected in view of
the long passage of time or because of the "realities of the situation."
After Balaquezon, the Court further expounded on the doctrine of
strained relations in the case of Globe-Mackay Cable and Radio Corp.
v. National Labor Relations Commission, wherein We discussed the
following considerations in applying the doctrine of strained relations:
(1) the employee must occupy a position where he or she enjoys the
trust and confidence of his or her employer; (2) it is likely that if
reinstated, an atmosphere of antipathy and antagonism may be
generated as to adversely affect the efficiency and productivity of the
employee concerned; (3) it cannot be applied indiscriminately because
some hostility is invariably engendered between the parties as a result
of litigation; and (4) it cannot arise from a valid and legal act of
asserting one's right. After Globe-Mackay, We clarified that the
doctrine cannot apply when the employee has not indicated an
aversion to returning to work, or does not occupy a position of trust and
confidence in, or has no say in the operation of, the employer's
business. In addition, strained relations between the parties must be
proven as a fact.

Although Katando does not occupy a position of trust and


confidence as a machine operator, the circumstances of this case
nonetheless calls for the application of the doctrine of strained
relations. It is true that litigation between the parties per se should not
bar the reinstatement of an employee. However, as observed by the
NLRC, this is not the only case involving Papertech and Katando. They
have been in conflict since 2008, or for 11 years now. In the case of
Digital Telecommunications Philippines, Inc. v. Digitel Employees
Union, We held that the length of time from the occurrence of the
incident to its resolution and the demonstrated litigiousness of the
parties showed that their relationship is strained. Similarly, the
protracted litigation between the parties here sufficiently demonstrate
that their relationship is strained. It is notable that Papertech has not
even bothered to appeal the ruling of the Labor Arbiter, and even stated
that "in order not to prolong the proceedings, and for both parties to
peacefully move on from this unwanted situation, Papertech is willing
to pay the judgment award of separation pay." Clearly, Papertech does
not want Katando back as its employee.

661
Page 662

Case Digests by: PALOMAR, ANDREI ANNE U.

OLYMPIA HOUSING, INC. VS. LAPASTORA


G.R. No. 187691. January 13, 2016.
Reyes, J.

DOCTRINE:
ARTICLE 294 [279]; REINSTATEMENT NO LONGER POSSIBLE
DUE TO CLOSURE
While the finding of illegal dismissal in favor of Lapastora subsists,
his reinstatement was rendered a legal impossibility with OHI's closure
of business. Considering the impossibility of Lapastora's
reinstatement, the payment of separation pay, in lieu thereof, is proper.

GRANT OF BACKWAGES AND SEPARATION PAY TO RUN UP TO


DATE OF CLOSURE
The amount of separation pay to be given to Lapastora must be
computed from March 1995, the time he commenced employment with
OHI, until the time when the company ceased operations in October
2000.

FACTS:
To establish employer-employee relationship with OHI, Lapastora
and Ubalubao alleged that they were directly hired by the company and
received salaries directly from its operations clerk. They also claimed
that OHI exercised control over them. It was also OHI which terminated
their employment after they petitioned for regularization. Prior to their
dismissal, they were subjected to investigations for their alleged
involvement in the theft of personal items and cash belonging to hotel
guests and were summarily dismissed by OHI despite lack of evidence.

For their part, OHI and Limcaoco alleged that Lapastora and
Ubalubao were not employees of the company but of Fast Manpower,
with which it had a contract of services, particularly, for the provision of
room attendants. They claimed that Fast Manpower is an independent
contractor. Reinforcing OHI's claims, Fast Manpower reiterated that it
is a legitimate manpower agency and that it had a valid contract of
services with OHI.

The Labor Arbiter ruled that Lapastora and Abalubao were regular
employees of OHI and that they were illegally dismissed. OHI assailed
that the reinstatement of Lapastora and Ubalubao was no longer
possible in view of the transfer of the management of the OER to HSAI-
Raintree. The NLRC and the CA both ruled in the negative.

662
Page 663

Unyielding, OHI filed the instant petition, reiterating its arguments


before the CA. It added that, even assuming that the facts warrant a
finding of illegal dismissal, the cessation of operations of the company
is a supervening event that should limit the award of backwages to
Lapastora and Ubalubao until October 1, 2000 only and justify the
deletion of the order of reinstatement. After all, it complied with the
notice requirements of the DOLE for a valid closure of business.

ISSUE:
Whether or not reinstatement is possible in this case.

RULING:
No. The issue of employer-employee relationship between OHI
and Lapastora had been deliberated and ruled upon by the LA and the
NLRC in the affirmative on the basis of the evidence presented by the
parties. However, while the finding of illegal dismissal in favor of
Lapastora subsists, his reinstatement was rendered a legal
impossibility with OHI's closure of business. Considering the
impossibility of Lapastora's reinstatement, the payment of separation
pay, in lieu thereof, is proper. The amount of separation pay to be given
to Lapastora must be computed from March 1995, the time he
commenced employment with OHI, until the time when the company
ceased operations in October 2000. As a twin relief, Lapastora is
likewise entitled to the payment of backwages, computed from the time
he was unjustly dismissed, or from February 24, 2000 until October 1,
2000 when his reinstatement was rendered impossible without fault on
his part.

663
Page 664

Case Digests by: PALOMAR, ANDREI ANNE U.

CLAUDIA'S KITCHEN, INC. VS. TANGUIN


G.R. No. 221096. June 28, 2017.
Mendoza, J.

DOCTRINE:
ARTICLE 294 [279]; SEPARATION PAY IN LIEU OF
REINSTATEMENT, WHEN PROPER
In sum, separation pay is only awarded to a dismissed employee
in the following instances: 1) in case of closure of establishment under
Article 298 [formerly Article 283] of the Labor Code; 2) in case of
termination due to disease or sickness under Article 299 [formerly
Article 284] of the Labor Code; 3) as a measure of social justice in
those instances where the employee is validly dismissed for causes
other than serious misconduct or those reflecting on his moral
character; 4) where the dismissed employee's position is no longer
available; 5) when the continued relationship between the employer
and the employee is no longer viable due to the strained relations
between them; or 6) when the dismissed employee opted not to be
reinstated, or the payment of separation benefits would be for the best
interest of the parties involved.

In all of these cases, the grant of separation pay presupposes that


the employee to whom it was given was dismissed from employment,
whether legally or illegally. In fine, as a general rule, separation pay in
lieu of reinstatement could not be awarded to an employee whose
employment was not terminated by his employer.

FACTS:
Respondent Tanguin was employed by petitioner Claudia's
Kitchen as a billing supervisor. Tanguin averred that she was placed
on preventive suspension for allegedly forcing her co-employees to
buy silver jewelry from her during office hours and inside the company
premises. During her suspension, the petitioners discovered her
habitual tardiness and gross negligence in the computation of the total
number of hours worked by her co-employees. Subsequently, she
received notices, requiring her to explain the charges against her;
reminding that she is still an employee of Claudia’s Kitchen; and
directing her to report back to work. Tanguin, however, failed to act on
these notices.

The LA ruled that Tanguin's preventive suspension was justified


and stressed that she was not illegally dismissed. Nevertheless, the
LA ordered the petitioners to pay Tanguin her unpaid salary.

664
Page 665

The NLRC, however, found that Tanguin did not abandon her work
when she failed to report for work despite notice. It stated that the filing
of the complaint for illegal dismissal negated the claim of
abandonment. The NLRC concluded that there was neither dismissal
nor abandonment. Thus, she should be reinstated to her former
position, but without backwages.

The CA said that reinstatement was not proper because such


remedy was applicable only to illegally dismissed employees. It added
that the petitioners did not dismiss her from employment as evidenced
by several notices sent to her requiring her to report back to work and
to explain the charges against her. The CA, however, applied the
doctrine of strained relations and ordered the payment of separation
pay to Tanguin instead of compelling the petitioners to accept her in
their employ.

ISSUE:
Whether or not separation pay in lieu of reinstatement may be
awarded to an employee who was not dismissed from employment.

RULING:
No. The payment of separation pay and reinstatement are
exclusive remedies. The payment of separation pay replaces the legal
consequences of reinstatement to an employee who was illegally
dismissed. To award separation pay in lieu of reinstatement to an
employee who was never dismissed by his employer would only give
imprimatur to the unacceptable act of an employee who is facing
charges related to his employment, but instead of addressing the
complaint against him, he opted to file an illegal dismissal case against
his employer.

In sum, separation pay is only awarded to a dismissed employee


in the following instances:

1) in case of closure of establishment under Article 298 [formerly


Article 283] of the Labor Code;
2) in case of termination due to disease or sickness under Article
299 [formerly Article 284] of the Labor Code;
3) as a measure of social justice in those instances where the
employee is validly dismissed for causes other than serious
misconduct or those reflecting on his moral character;
4) where the dismissed employee's position is no longer available;
5) when the continued relationship between the employer and the
employee is no longer viable due to the strained relations
between them; or

665
Page 666

6) when the dismissed employee opted not to be reinstated, or the


payment of separation benefits would be for the best interest of
the parties involved.

In all of these cases, the grant of separation pay presupposes that


the employee to whom it was given was dismissed from employment,
whether legally or illegally. In fine, as a general rule, separation pay in
lieu of reinstatement could not be awarded to an employee whose
employment was not terminated by his employer.

666
Page 667

Case Digests by: PALOMAR, ANDREI ANNE U.

DUMAPIS VS. LEPANTO


G.R. No. 204060. September 15, 2020.
Lazaro-Javier, J.

DOCTRINE:
COMPUTATION OF AWARD OF SEPARATION PAY AND
BACKWAGES
The award of separation pay and backwages for illegally
dismissed employees should be computed from the time they got
illegally dismissed until the finality of the decision ordering payment of
their separation pay, in lieu of reinstatement.

ID; INCLUDED ITEMS


Verily, the Court now ordains the uniform rule that the award of
backwages and/or separation pay due to illegally dismissed employees
shall include all salary increases and benefits granted under the law
and other government issuances, Collective Bargaining Agreements,
employment contracts, established company policies and practices,
and analogous sources which the employees would have been entitled
to had they not been illegally dismissed. On the other hand, salary
increases and other benefits which are contingent or dependent on
variables such as an employee's merit increase based on performance
or longevity or the company's financial status shall not be included in
the award.

FACTS:
An initial complaint for illegal dismissal between the parties was
dismissed by the NLRC. However the decision was reversed insofar
as three of the complainants, now petitioners, Moreno Dumapis,
Francisco Liagao and Elmo Tundagui were concerned. Lepanto
elevated the case to the CA wherein the latter affirmed the decision of
the NLRC.

Following the finality of the decision, the labor arbiter issued the
corresponding writ of execution in the total amount of P897,412.95
covering petitioners' backwages and separation pay. Petitioners then
sought a recomputation of this award which the labor arbiter granted
through his Order dated May 27, 2009, increasing the award to
P2,602,856.21.

Lepanto moved to quash the writ of execution, insisting that the


computation should be reckoned from the date of dismissal up until the

667
Page 668

NLRC rendered its Decision dated August 30, 2002. Lepanto further
claimed that the parties had already agreed to satisfy the original
monetary award of P897,412.95, for which, an initial amount of
P100,000.00 was already deposited into the account of petitioners'
counsel.

In their Partial Motion for Reconsideration/Memorandum of


Appeal, petitioners asserted that the cut-off date for the computation of
the award was November 23, 2008 when this Court's Decision in G.R.
No. 163210 became final and executory. Petitioners cited Surima v.
NLRC and Carlos v. CA. They also argued that the monetary award
should include salary increases granted under the CBA as the same
should have accrued to them had they not been illegally terminated.
Lastly, petitioners reported that out of Lepantos's P100,000.00 deposit,
only P75,000.00 went to them as the P25,000.00 went to another
complainant who was also their counsel's client.

ISSUE:
What is the correct formula for computing the award of separation
pay and backwages to petitioners?

RULING:
In CICM Mission Seminaries, et al. v. Perez citing Bani Rural
Bank, Inc. v. De Guzman, the Court through the Second Division laid
down the rule that the award of separation pay and backwages for
illegally dismissed employees should be computed from the time they
got illegally dismissed until the finality of the decision ordering payment
of their separation pay, in lieu of reinstatement.

In accordance with CICM Mission Seminaries, petitioners'


backwages and separation pay here should, therefore, be computed
from September 22, 2000 when they got illegally dismissed until
November 25, 2008, when this Court's Decision dated August 13, 2008
became final and executory.

On what exactly these backwages ought to include, the Court's


relevant rulings may be categorized into two (2):

The first category delves on the inclusion or non-inclusion in the


award of salary increases and benefits which are contingent on the
fulfillment of certain conditions such as merit increase based on
performance, company's fiscal position, or management's benevolent
initiative.

668
Page 669

On the other hand, the second category delves on guaranteed


salary increases and benefits. Their grant is either mandated by law,
standard company policy, or Collective Bargaining Agreement (CBA).

Given the Court's repetitive self-contradictions in the award of


backwages or separation pay owing to illegally dismissed employees
and the consequent instability they have caused to our labor law
jurisprudence, the time has come to settle these contradictions,
once and for all.

We keenly note that there is no provision in the Labor Code which


mandates the exclusion of salary increases and benefits accruing to
the dismissed employee. Article 279 (now Art. 292) in fact grants
illegally dismissed employees the right to full backwages, inclusive of
allowances, and other benefits or their monetary equivalent computed
from the time their compensation was withheld up to the time of their
actual reinstatement.

When the law does not distinguish, we should not distinguish.

Verily, the Court now ordains the uniform rule that the award of
backwages and/or separation pay due to illegally dismissed employees
shall include all salary increases and benefits granted under the law
and other government issuances, Collective Bargaining Agreements,
employment contracts, established company policies and practices,
and analogous sources which the employees would have been entitled
to had they not been illegally dismissed. On the other hand, salary
increases and other benefits which are contingent or dependent on
variables such as an employee's merit increase based on performance
or longevity or the company's financial status shall not be included in
the award.

669
Page 670

Case Digests by: PALOMAR, ANDREI ANNE U.

LYNVIL FISHING ENTERPRISES, INC. VS. ARIOLA


G.R. No. 181974. February 1, 2012.
Perez, J.

DOCTRINE:
ARTICLE 295 [280]; REGULAR EMPLOYEE
Textually, the provision that: "NA ako ay sumasang-ayon na
maglingkod at gumawa ng mga gawain sang-ayon sa patakarang "por
viaje" na magmumula sa pagalis sa Navotas papunta sa pangisdaan
at pagbabalik sa pondohan ng lantsa sa Navotas, Metro Manila" is for
a fixed period of employment. In the context, however, of the facts that:
(1) the respondents were doing tasks necessarily to Lynvil's fishing
business with positions ranging from captain of the vessel to
bodegero ; (2) after the end of a trip, they will again be hired for another
trip with new contracts; and (3) this arrangement continued for more
than ten years, the clear intention is to go around the security of tenure
of the respondents as regular employees. And respondents are so by
the express provisions of the second paragraph of Article 280, thus:

. . . Provided, That any employee who has rendered at least one


year of service, whether such service is continuous or broken,
shall be considered a regular employee with respect to the activity
in which he is employed and his employment shall continue while
such activity exists.

FACTS:
Lynvil is a company engaged in deep-sea fishing. Respondent
Ariola, among others were employees engaged on a per trip basis or
"por viaje" which terminates at the end of each trip. Lynvil received a
report that the respondents stole eight (8) tubs of "pampano" and
"tangigue" fish and delivered them to another vessel. The respondents
were notified to explain. Failing to explain as required, respondents'
employment was terminated.

Aggrieved, the employees filed a complaint for illegal dismissal


with claims for backwages, salary differential reinstatement, service
incentive leave, holiday pay and its premium and 13th month pay from
1996 to 1998. They also claimed for moral, exemplary damages and
attorney's fees for their dismissal with bad faith.

The LA ruled in favor of the employees’ charge of illegal dismissal.


The NLRC reversed and set aside the decision of the LA.

670
Page 671

The CA reinstated the decision of the LA except as to the award


of attorney's fees. The appellate court held that the allegation of theft
did not warrant the dismissal of the employees since there was no
evidence to prove the actual quantities of the missing kinds of fish
loaded to Analyn VIII. It also reversed the finding of the NLRC that the
dismissed employees were merely contractual employees and added
that they were regular ones performing activities which are usually
necessary or desirable in the business and trade of Lynvil. Finally, it
ruled that the two-notice rule provided by law and jurisprudence is
mandatory and non-compliance therewith rendered the dismissal of
the employees illegal.

ISSUE:
Whether or not the respondent employees were regular
employees.

RULING:
Yes. Jurisprudence, laid two conditions for the validity of a fixed-
contract agreement between the employer and employee:

First, the fixed period of employment was knowingly and


voluntarily agreed upon by the parties without any force, duress,
or improper pressure being brought to bear upon the employee
and absent any other circumstances vitiating his consent; or

Second, it satisfactorily appears that the employer and the


employee dealt with each other on more or less equal terms with
no moral dominance exercised by the former or the latter.

Textually, the provision that: "NA ako ay sumasang-ayon na


maglingkod at gumawa ng mga gawain sang-ayon sa patakarang "por
viaje" na magmumula sa pagalis sa Navotas papunta sa pangisdaan
at pagbabalik sa pondohan ng lantsa sa Navotas, Metro Manila" is for
a fixed period of employment. In the context, however, of the facts that:
(1) the respondents were doing tasks necessarily to Lynvil's fishing
business with positions ranging from captain of the vessel to
bodegero ; (2) after the end of a trip, they will again be hired for another
trip with new contracts; and (3) this arrangement continued for more
than ten years, the clear intention is to go around the security of tenure
of the respondents as regular employees. And respondents are so by
the express provisions of the second paragraph of Article 280, thus:

. . . Provided, That any employee who has rendered at least


one year of service, whether such service is continuous or broken,
shall be considered a regular employee with respect to the activity

671
Page 672

in which he is employed and his employment shall continue while


such activity exists.

The same set of circumstances indicate clearly enough that it was


the need for a continued source of income that forced the employees'
acceptance of the "por viaje" provision.

672
Page 673

Case Digests by: PALOMAR, ANDREI ANNE U.

SONZA VS. ABS-CBN BROADCASTING


CORPORATION
G.R. No. 138051. June 10, 2004.
Carpio, J.

DOCTRINE:
ARTICLE 295 [280]; TALENTS ARE NOT EMPLOYEES
The hiring of exclusive talents is a widespread and accepted
practice in the entertainment industry. This practice is not designed to
control the means and methods of work of the talent, but simply to
protect the investment of the broadcast station. The broadcast station
normally spends substantial amounts of money, time and effort "in
building up its talents as well as the programs they appear in and thus
expects that said talents remain exclusive with the station for a
commensurate period of time." Normally, a much higher fee is paid to
talents who agree to work exclusively for a particular radio or television
station. In short, the huge talent fees partially compensates for
exclusivity, as in the present case.

FACTS:
Respondent ABS-CBN signed an Agreement with Mel and Jay
Management and Development Corporation ("MJMDC"). The
Agreement listed the services SONZA would render to ABS-CBN, as
follows:

a. Co-host for Mel & Jay radio program, 8:00 to 10:00 a.m.,
Mondays to Fridays;
b. Co-host for Mel & Jay television program, 5:30 to 7:00 p.m.,
Sundays.

ABS-CBN agreed to pay for SONZA's services a monthly talent fee


of P310,000 for the first year and P317,000 for the second and third
year of the Agreement. ABS-CBN would pay the talent fees on the 10th
and 25th days of the month.

SONZA complained that ABS-CBN did not pay his salaries,


separation pay, service incentive leave pay, 13th month pay, signing
bonus, travel allowance and amounts due under the Employees Stock
Option Plan ("ESOP").

ABS-CBN filed a Motion to Dismiss on the ground that no employer-


employee relationship existed between the parties. SONZA filed an

673
Page 674

Opposition to the motion. Meanwhile, ABS-CBN continued to remit


SONZA's monthly talent fees and other payments due him under the
Agreement.

ISSUE:
Whether or not an employer-employee relationship between
Sonza and ABS-CBN exists.

RULING:
No. Case law has consistently held that the elements of an
employer-employee relationship are: (a) the selection and
engagement of the employee; (b) the payment of wages; (c) the power
of dismissal; and (d) the employer's power to control the employee on
the means and methods by which the work is accomplished. The last
element, the so called "control test", is the most important element.

A. Selection and Engagement of Employee - Independent


contractors often present themselves to possess unique skills,
expertise or talent to distinguish them from ordinary employees.
The specific selection and hiring of SONZA, because of his
unique skills, talent and celebrity status not possessed by
ordinary employees, is a circumstance indicative, but not
conclusive, of an independent contractual relationship.

B. Payment of Wages - All the talent fees and benefits paid to


SONZA were the result of negotiations that led to the Agreement.
If SONZA were ABS-CBN's employee, there would be no need
for the parties to stipulate on benefits such as "SSS, Medicare, .
. . and 13th month pay" which the law automatically incorporates
into every employer-employee contract. Whatever benefits
SONZA enjoyed arose from contract and not because of an
employer-employee relationship. SONZA's talent fees,
amounting to P317,000 monthly in the second and third year, are
so huge and out of the ordinary that they indicate more an
independent contractual relationship rather than an employer-
employee relationship.

C. Power of Dismissal - For violation of any provision of the


Agreement, either party may terminate their relationship. SONZA
failed to show that ABS-CBN could terminate his services on
grounds other than breach of contract, such as retrenchment to
prevent losses as provided under labor laws.

D. Power of Control - Applying the control test to the present case,


we find that SONZA is not an employee but an independent
contractor. We find that ABS-CBN was not involved in the actual
performance that produced the finished product of SONZA's

674
Page 675

work. A radio broadcast specialist who works under minimal


supervision is an independent contractor.

Being an exclusive talent does not by itself mean that


SONZA is an employee of ABS-CBN. Even an independent
contractor can validly provide his services exclusively to the
hiring party. In the broadcast industry, exclusivity is not
necessarily the same as control.

The hiring of exclusive talents is a widespread and accepted


practice in the entertainment industry. This practice is not designed to
control the means and methods of work of the talent, but simply to
protect the investment of the broadcast station. The broadcast station
normally spends substantial amounts of money, time and effort "in
building up its talents as well as the programs they appear in and thus
expects that said talents remain exclusive with the station for a
commensurate period of time." Normally, a much higher fee is paid to
talents who agree to work exclusively for a particular radio or television
station. In short, the huge talent fees partially compensates for
exclusivity, as in the present case.

675
Page 676

Case Digests by: PALOMAR, ANDREI ANNE U.

CONSOLIDATED BROADCASTING SYSTEM, INC. VS.


OBERIO
G.R. No. 168424. June 8, 2007.
Ynares-Santiago, J.

DOCTRINE:
ARTICLE 295 [280]; REGULAR EMPLOYMENT TEST
The test to determine whether employment is regular or not is the
reasonable connection between the particular activity performed by the
employee in relation to the usual business or trade of the employer.
Also, if the employee has been performing the job for at least one year,
even if the performance is not continuous or merely intermittent, the
law deems the repeated and continuing need for its performance as
sufficient evidence of the necessity, if not indispensability of that
activity to the business.

FACTS:
Respondents alleged that they were employed as drama talents
by DYWB-Bombo Radyo, a radio station owned and operated by
petitioner Consolidated Broadcasting System, Inc. They reported for
work daily for six days in a week and were required to record their
drama production in advance. Some of them were employed by
petitioner since 1974, while the latest one was hired in 1997.

In 1998, petitioner reduced the number of its drama productions


from 14 to 11, but was opposed by respondents. After the negotiations
failed, the latter sought the intervention of the DOLE. It is revealed that
petitioner is guilty of violation of labor standard laws.

Petitioner contended that respondents are not its employees.


Vexed by the respondents' complaint, petitioner allegedly pressured
and intimidated respondents. Respondents Oberio and Delta were
suspended for minor lapses and the payment of their salaries were
purportedly delayed. Eventually, pending the outcome of the inspection
case with the Regional Director, respondents were barred by petitioner
from reporting for work; thus, the former claimed constructive
dismissal.

ISSUE:
Whether or not an employer-employee relationship exists.

RULING:

676
Page 677

Yes. Respondents' employment with petitioner passed the "four-


fold test" on employer-employee relations, namely: (1) the selection
and engagement of the employee, or the power to hire; (2) the payment
of wages; (3) the power to dismiss; and (4) the power to control the
employee.

Petitioner failed to controvert with substantial evidence the


allegation of respondents that they were hired by the former on various
dates from 1974 to 1997. If petitioner did not hire respondents and if it
was the director alone who chose the talents, petitioner could have
easily shown, being in possession of the records, a contract to such
effect. However, petitioner merely relied on its contention that
respondents were piece rate contractors who were paid by results.

The test to determine whether employment is regular or not is the


reasonable connection between the particular activity performed by the
employee in relation to the usual business or trade of the employer.
Also, if the employee has been performing the job for at least one year,
even if the performance is not continuous or merely intermittent, the
law deems the repeated and continuing need for its performance as
sufficient evidence of the necessity, if not indispensability of that
activity to the business. Thus, even assuming that respondents were
initially hired as project/contractual employees who were paid per
drama or per project/contract, the engagement of their services for 2
to 25 years justify their classification as regular employees, their
services being deemed indispensable to the business of petitioner.

As to the payment of wages, it was petitioner who paid the same


as shown by the payroll bearing the name of petitioner company in the
heading with the respective salaries of respondents opposite their
names.

All these, taken together, unmistakably show the existence of an


employer-employee relationship. Not only did petitioner possess the
power of control over their work but also the power to discipline them
through the imposition of fines and suspension for violation of company
rules and policies.

677
Page 678

Case Digests by: PALOMAR, ANDREI ANNE U.

OROZCO VS. COURT OF APPEALS


G.R. No. 155207. August 13, 2008.
Nachura, J.

DOCTRINE:
ARTICLE 295 [280]; CONTROL
The main determinant therefore is whether the rules set by the
employer are meant to control not just the results of the work but also
the means and method to be used by the hired party in order to achieve
such results.

ID; ID; RULE IN DETERMINING EMPLOYER-EMPLOYEE


RELATIONSHIP
There is no inflexible rule to determine if a person is an employee
or an independent contractor; thus, the characterization of the
relationship must be made based on the particular circumstances of
each case. There are several factors that may be considered by the
courts, but as we already said, the right to control is the dominant factor
in determining whether one is an employee or an independent
contractor.

FACTS:
Philippine Daily Inquirer (PDI) engaged the services of petitioner
Wilhelmina Orozco to write a weekly column for its Lifestyle section.
She religiously submitted her articles every week, except for a six-
month stint in New York City when she, nonetheless, sent several
articles through mail. She received compensation of P250.00 — later
increased to P300.00 — for every column published. Orozco claims
that the PDI editor-in-chief wanted to stop publishing her column for no
reason at all.

Meanwhile, PDI claims that they decided to cut down the number
of columnists by keeping only those whose columns were well-written,
with regular feedback and following. In their judgment, petitioner's
column failed to improve, continued to be superficially\ and poorly
written, and failed to meet the high standards of the newspaper.
Hence, they decided to terminate petitioner's column.

Aggrieved by the newspaper's action, petitioner filed a complaint


for illegal dismissal, backwages, moral and exemplary damages, and
other money claims before the NLRC.

678
Page 679

ISSUE:
Whether or not a newspaper columnist is an employee of the
newspaper which publishes the column.

RULING:
No. Not all rules imposed by the hiring party on the hired party
indicate that the latter is an employee of the former. Rules which serve
as general guidelines towards the achievement of the mutually desired
result are not indicative of the power of control.

The main determinant therefore is whether the rules set by the


employer are meant to control not just the results of the work but also
the means and method to be used by the hired party in order to achieve
such results.

A careful examination reveals that the factors enumerated by the


petitioner are inherent conditions in running a newspaper. In other
words, the so-called control as to time, space, and discipline are
dictated by the very nature of the newspaper business itself. Contrary
to petitioner's protestations, it does not appear that there was any
actual restraint or limitation on the subject matter — within the Lifestyle
section — that she could write about. Contrary to petitioner's
protestations, it does not appear that there was any actual restraint or
limitation on the subject matter — within the Lifestyle section — that
she could write about.

Aside from the control test, this Court has also used the economic
reality test. The economic realities prevailing within the activity or
between the parties are examined, taking into consideration the totality
of circumstances surrounding the true nature of the relationship
between the parties. This is especially appropriate when, as in this
case, there is no written agreement or contract on which to base the
relationship. In our jurisdiction, the benchmark of economic reality in
analyzing possible employment relationships for purposes of applying
the Labor Code ought to be the economic dependence of the worker
on his employer.

There is no inflexible rule to determine if a person is an employee


or an independent contractor; thus, the characterization of the
relationship must be made based on the particular circumstances of
each case. There are several factors that may be considered by the
courts, but as we already said, the right to control is the dominant factor
in determining whether one is an employee or an independent
contractor.

679
Page 680

Case Digests by: PALOMAR, ANDREI ANNE U.

WILLIAM UY CONSTRUCTION CORP. VS. TRINIDAD


G.R. No. 183250. March 12, 2010.
Abad, J.

DOCTRINE:
ARTICLE 925 [280]; TEST IN DETERMINING PROJECT
EMPOLYEE AND REGULAR EMPLOYEE
The test for distinguishing a "project employee" from a "regular
employee" is whether or not he has been assigned to carry out a
"specific project or undertaking," with the duration and scope of his
engagement specified at the time his service is contracted.

ID; STATUS OF EMPLOYMENT; CONSTRUCTION INDUSTRY


Generally, length of service provides a fair yardstick for
determining when an employee initially hired on a temporary basis
becomes a permanent one, entitled to the security and benefits of
regularization. But this standard will not be fair, if applied to the
construction industry, simply because construction firms cannot
guarantee work and funding for its payrolls beyond the life of each
project. And getting projects is not a matter of course. Construction
companies have no control over the decisions and resources of project
proponents or owners. There is no construction company that does not
wish it has such control but the reality, understood by construction
workers, is that work depended on decisions and developments over
which construction companies have no say.

FACTS:
Respondent Jorge Trinidad filed a complaint for illegal dismissal
and unpaid benefits against petitioner William Uy Construction
Corporation. Trinidad claimed that he had been working with the latter
company for 16 years since 1988 as driver of its service vehicle, dump
truck, and transit mixer. He had signed several employment contracts
with the company that identified him as a project employee although
he had always been assigned to work on one project after another with
some intervals. He further alleged that in December 2004 petitioner
company terminated him from work after it shut down operations
because of lack of projects. He learned later, however, that although it
opened up a project in Batangas, it did not hire him back for that
project.

Petitioner company countered that by the nature of its business, it


had to hire and engage the services of project construction workers,
including respondent Trinidad, whose employments had to be co-

680
Page 681

terminous with the completion of specific company projects. For this


reason, every time the company employed Trinidad, he had to execute
an employment contract with it, called Appointment as Project Worker.

ISSUE:
Whether or not petitioner company's repeated rehiring of
respondent Trinidad over several years as project employee for its
various projects automatically entitled him to the status of a regular
employee.

RULING:
No. The test for distinguishing a "project employee" from a "regular
employee" is whether or not he has been assigned to carry out a
"specific project or undertaking," with the duration and scope of his
engagement specified at the time his service is contracted. Here, it is
not disputed that petitioner company contracted respondent Trinidad's
service by specific projects with the duration of his work clearly set out
in his employment contracts. He remained a project employee
regardless of the number of years and the various projects he worked
for the company.

Generally, length of service provides a fair yardstick for


determining when an employee initially hired on a temporary basis
becomes a permanent one, entitled to the security and benefits of
regularization. But this standard will not be fair, if applied to the
construction industry, simply because construction firms cannot
guarantee work and funding for its payrolls beyond the life of each
project. And getting projects is not a matter of course. Construction
companies have no control over the decisions and resources of project
proponents or owners. There is no construction company that does not
wish it has such control but the reality, understood by construction
workers, is that work depended on decisions and developments over
which construction companies have no say.

In this case, respondent Trinidad's series of employments with


petitioner company were co-terminous with its projects. When its Boni
Serrano-Katipunan Interchange Project was finished in December
2004, Trinidad's employment ended with it. He was not dismissed. His
employment contract simply ended with the project for which he had
signed up. His employment history belies the claim that he
continuously worked for the company. Intervals or gaps separated one
contract from another.

681
Page 682

Case Digests by: PARIL, JOSHUA F.

DM Consunji Inc. v. Jamin


G.R. No. 192514. April 18, 2012.
Brion J.

DOCTRINE:
ARTICLE 295
Once a project or work pool employee has been: (1) continuously,
as opposed to intermittently, rehired by the same employer for the
same tasks or nature of tasks; and (2) these tasks are vital, necessary
and indispensable to the usual business or trade of the employer, then
the employee must be deemed a regular employee.

FACTS:
D.M. Consunji, Inc. (DMCI), a construction company, hired
respondent Estelito L. Jamin as a laborer. Sometime in 1975, Jamin
became a helper carpenter. Since his initial hiring, Jamins employment
contract had been renewed a number of times. On March 20, 1999, his
work at DMCI was terminated due to the completion of the SM Manila
project. This termination marked the end of his employment with DMCI
as he was not rehired again.

Jamin filed a complaintfor illegal dismissal, with several money


claims (including attorneys fees), against DMCI and its
President/General Manager, David M. Consunji. Jamin alleged that
DMCI terminated his employment without a just and authorized cause
at a time when he was already 55 years old and had no independent
source of livelihood. He claimed that he rendered service to DMCI
continuously for almost 31 years.

DMCI denied liability. It argued that it hired Jamin on a project-to-


project basis, from the start of his engagement in 1968 until the
completion of its SM Manila project on March 20, 1999 where Jamin
last worked. With the completion of the project, it terminated Jamins
employment.

The LA dismissed the complaint for lack of merit. On appeal, the NLRC
affirmed the decision of the LA. On further appeal, the CA reversed the
NLRC decision and ruled that Jamin was a regular employee. Hence,
DMCI seeks a reversal of the CA rulings on the ground that the
appellate court committed a grave error in annulling the decisions of
the labor arbiter and the NLRC.

682
Page 683

ISSUE:
Whether or not Jamin is a regular employee

RULING:
YES. Once a project or work pool employee has been: (1)
continuously, as opposed to intermittently, rehired by the same
employer for the same tasks or nature of tasks; and (2) these tasks are
vital, necessary and indispensable to the usual business or trade of the
employer, then the employee must be deemed a regular employee.

While the contracts indeed show that Jamin had been engaged as a
project employee, there was an almost unbroken string of Jamin’s
rehiring from December 17, 1968 up to the termination of his
employment on March 20, 1999. While the history of Jamin’s
employment (schedule of projects) relied upon by DMCI shows a gap
of almost four years in his employment for the period between July 28,
1980 (the supposed completion date of the Midtown Plaza project) and
June 13, 1984 (the start of the IRRI Dorm IV project), the gap was
caused by the company’s omission of the three projects above
mentioned.

To reiterate, Jamin’s employment history with DMCI stands out for his
continuous, repeated and successive rehiring in the company’s
construction projects. In all the 38 projects where DMCI engaged
Jamin’s services, the tasks he performed as a carpenter were
indisputably necessary and desirable in DMCIs construction business.
He might not have been a member of a work pool as DMCI insisted
that it does not maintain a work pool, but his continuous rehiring and
the nature of his work unmistakably made him a regular employee.

Further, as we stressed in Liganza, respondent capitalizes on our


ruling in D.M. Consunji, Inc. v. NLRC which reiterates the rule that the
length of service of a project employee is not the controlling test of
employment tenure but whether or not the employment has been fixed
for a specific project or undertaking the completion or termination of
which has been determined at the time of the engagement of the
employee."

"Surely, length of time is not the controlling test for project employment.
Nevertheless, it is vital in determining if the employee was hired for a
specific undertaking or tasked to perform functions vital, necessary and
indispensable to the usual business or trade of the employer. Here,
private respondent had been a project employee several times over.
His employment ceased to be coterminous with specific projects when
he was repeatedly re-hired due to the demands of petitioners business.
Without doubt, Jamin’s case fits squarely into the employment situation
just quoted.

683
Page 684

Case Digests by: PARIL, JOSHUA F.

Aro et al v. NLRC
G.R. No. 174792. March 7, 2012
Peralta, J.

DOCTRINE
ARTICLE 295
The length of service or the re-hiring of construction workers on a
project-to-project basis does not confer upon them regular employment
status, since their re-hiring is only a natural consequence of the fact
that experienced construction workers are preferred.

FACTS:

Several employees of private respondent Benthel Development


Corporation, including the petitioners, filed a Complaint for illegal
dismissal with various money claims and prayer for damages against
the latter. The Labor Arbiter rendered a decision finding private
respondent guilty of illegal dismissal.

The employees, including the petitioners herein, appealed from the


said decision. The NLRC affirmed the decision of Labor Arbiter
Carreon with the modification that private respondent pay backwages
computed from the respective dates of dismissal until finality of the
decision.

Private respondent, unsatisfied with the modification made by the


NLRC, filed a motion for reconsideration with the contention that, since
it has been found by the Labor Arbiter and affirmed in the assailed
decision that the employees were project employees, the computation
of backwages should be limited to the date of the completion of the
project and not to the finality of the decision. The NLRC, however,
denied the motion ruling that private respondent failed to establish the
date of the completion of the project.

ISSUE:
Whether or not employees herein are regular or project
employees?

RULING:

They are project employees. They were hired for the construction
of the Cordova Reef Village Resort in Cordova, Cebu.

684
Page 685

Article 280 of the Labor Code distinguishes a "project employee" from


a "regular employee," thus:

Article 280. Regular and Casual Employment − The provisions of


written agreement to the contrary notwithstanding and regardless of
the oral agreement of the parties, an employment shall be deemed to
be regular where the employee has been engaged to perform activities
which are usually necessary or desirable in the usual business or trade
of the employer, except where the employment has been fixed for a
specific project or undertaking the completion or termination of which
has been determined at the time of the engagement of the employee
or where the work or service to be performed is seasonal in nature and
the employment is for the duration of the season.

An employment shall be deemed to be casual if it is not covered by the


preceding paragraph: Provided, That, any employee who has rendered
at least one year service, whether such service is continuous or
broken, shall be considered a regular employee with respect to the
activity in which he is employed and his employment shall continue
while such activity exists.

In Hanjin Heavy Industries and Construction Co. Ltd. v. Ibañez this


Court extensively discussed the above distinction, thus:

x x x [T]he principal test for determining whether particular employees


are properly characterized as "project employees" as distinguished
from "regular employees" is whether or not the project employees were
assigned to carry out a "specific project or undertaking," the duration
and scope of which were specified at the time the employees were
engaged for that project.

In a number of cases, the Court has held that the length of service or
the re-hiring of construction workers on a project-to-project basis does
not confer upon them regular employment status, since their re-hiring
is only a natural consequence of the fact that experienced construction
workers are preferred. Employees who are hired for carrying out a
separate job, distinct from the other undertakings of the company, the
scope and duration of which has been determined and made known to
the employees at the time of the employment , are properly treated as
project employees and their services may be lawfully terminated upon
the completion of a project Should the terms of their employment fail
to comply with this standard, they cannot be considered project
employees.

This Court agrees with the findings of the CA that petitioners were
project employees. It is not disputed that petitioners were hired for the
construction of the Cordova Reef Village Resort in Cordova, Cebu. By
the nature of the contract alone, it is clear that petitioners' employment
was to carry out a specific project.

685
Page 686

Therefore, being project employees, petitioners are only entitled to full


backwages, computed from the date of the termination of their
employment until the actual completion of the work.

686
Page 687

Case Digests by: PARIL, JOSHUA F.

Universal Robina Sugar Milling Corp v. Acibo et. al


G.R. No. 186439. January 15, 2014.
Brion J.

DOCTRINE:
ARTICLE 295
The period denominated in the contract of employment is not the basis
in determining whether an employee is seasonal or regular.
FACTS:
Ferdinand Acibo, et al. were employees of UNIVERSAL ROBINA
SUGAR MILLING CORPORATION (URSUMCO). Acibo, et al. signed
contracts of employment for a given period and after its expiration,
URSUMCO repeatedly hired these employees to perform the same
duties and obligations.

Acibo, et al. filed a complaint before the Labor Arbiter for


regularization however it was denied because the LA argued that they
were seasonal employees. Seven of the 22 complainants filed an
appeal to the NLRC. The latter reversed the LA’s ruling claiming that
they were regular employees. The CA affirmed NLRC’s decision but
excluded the Acibo, et al. from monetary benefits under the CBA.

ISSUE:
Whether or not Acibo, et al. are regular employees of URSUMCO

RULING:
Plantation workers or mill employees only work on seasonal
basis. This, however, does not exclude them from the benefits of
regularization. Being in such nature, Acibo, et al. are considered to be
regular employees.

Regular employment means that there was an arrangement


between the employee and the employer that the former will be
engaged to perform activities which are necessary or desirable to the
usual business or trade of the latter. On the other hand, a project
employment is an arrangement for a specific project or undertaking
whose termination is determined by the completion of the project.

The nature of the employment does not depend solely on the will
or word of the employer or on the procedure for hiring and the manner
of designating the employee. Rather, the nature of the employment
depends on the nature of the activities to be performed by the
employee, considering the nature of the employer’s business, the

687
Page 688

duration and scope to be done. Accordingly, Acibo, et al. are neither


project nor seasonal employees.

Acibo, et al. were made to perform tasks that does not pertain to
milling operations of URSUMCO. However, their duties are regularly
and habitually needed in URSUMCO’s operation. Moreover, they were
regularly and repeatedly hired to perform the same tasks. Being
repeatedly hired for the same purpose makes them regularized
employees.

The plantation workers or the mill employees do not work


continuously for 1 whole year but only for the duration of the growing
or the sugarcane or the milling season. Their seasonal work, however,
does not detract from considering them in regular employment.

688
Page 689

Case Digests by: PARIL, JOSHUA F.

GMA Network Inc. v. Pabriga


G.R. No. 176419. November 27, 2013.
Leonardo De Castro J.

DOCTRINE:
ARTICLE 295
The principal test for determining whether particular employees are
properly characterized as "project employees" as distinguished from
"regular employees," is whether or not the "project employees" were
assigned to carry out a "specific project or undertaking," the duration
(and scope) of which were specified at the time the employees were
engaged for that project.

FACTS:
Respondents are television technicians assigned to the following
tasks:
o Manning of Technical Operations Center
o Acting as Transmitter/VTR men
o Acting as Maintenance staff
o Acting as cameramen

On July 19, 1999 due to the miserable working conditions private


respondents, television technicians, were forced to file a complaint
against GMA before the NLRC Regional Arbitration Branch No. VII
Cebu City. On August 4, 1999, GMA received a notice of hearing of
the complaint. The following day, petitioner’s Engineering Manager,
Roy Villacastin, confronted the private respondents about said
complaint.

On August 9, 1999, private respondents were summoned to the office


of GMA’s Area Manager, Mrs. Susan Alino, and they were made to
explain why they filed the complaint. The next day, private respondents
were barred from entering and reporting for work without any notice
stating the reasons therefor. On August 13, 1999, private respondents,
through their counsel, wrote a letter to Alino, requesting that they be
recalled back to work

On August 23, 1999, a reply from Mr. Bienvenido Bustria, GMA’s head
of Personal and Labor Relations Division, admitted the non-payment
of benefits but did not mention the request of private respondents to
be allowed to return to work. On 15 September 1999, private
respondents sent another letter to Mr. Bustria reiterating their request
to work but the same was totally ignored. On 8 October 1999, private
respondents filed an amended complaint raising the following

689
Page 690

additional issues: 1) Unfair Labor Practice; 2) Illegal dismissal; and 3)


Damages and Attorney’s fees. On 23 September 1999, a mandatory
conference was set to amicably settle the dispute between the parties,
however, the same proved to be futile. As a result, both of them were
directed to file their respective position papers. On 10 November 1999,
private respondents filed their position paper and on 2 March 2000,
they received a copy of petitioner’s position paper. The following day,
the Labor Arbiter issued an order considering the case submitted for
decision.

LA dismissed the complaint for illegal dismissal and unfair labor


practice, but held petitioner liable for 13th month pay. Respondents
appealed to NLRC. NLRC reversed LA, saying a) All complainants
are regular employees with respect to the particular activity to which
they were assigned, until it ceased to exist. As such, they are entitled
to payment of separation pay computed at one (1) month salary for
every year of service; b) They are not entitled to overtime pay and
holiday pay; and c) They are entitled to 13th month pay, night shift
differential and service incentive leave pay. For purposes of accurate
computation, the entire records are REMANDED to the Regional
Arbitration Branch of origin which is hereby directed to require from
respondent the production of additional documents where necessary.

ISSUE:
Whether or not respondents are regular employees and not project
employees

RULING:

They are regular employees. The principal test for determining whether
particular employees are properly characterized as "project
employees" as distinguished from "regular employees," is whether or
not the "project employees" were assigned to carry out a "specific
project or undertaking," the duration (and scope) of which were
specified at the time the employees were engaged for that project.

Employers claiming that their workers are project employees should


not only prove that the duration and scope of the employment was
specified at the time they were engaged, but also that there was indeed
a project. The project could either be (1) a particular job or undertaking
that is within the regular or usual business of the employer company,
but which is distinct and separate, and identifiable as such, from the
other undertakings of the company; or (2) a particular job or
undertaking that is not within the regular business of the corporation.

In the case at bar, respondents’ jobs and undertakings are clearly


within the regular or usual business of GMA, and are not identifiably
distinct or separate from the other undertakings of the company.

690
Page 691

Petitioner’s allegation that respondents were merely substitutes or


what they call pinch-hitters (which means that they were employed to
take the place of regular employees of petitioner who were absent or
on leave) does not change the fact that their jobs cannot be considered
projects within the purview of the law. Every industry, even public
offices, has to deal with securing substitutes for employees who are
absent or on leave. Such tasks, whether performed by the usual
employee or by a substitute, cannot be considered separate and
distinct from the other undertakings of the company. While it is
management’s prerogative to device a method to deal with this issue,
such prerogative is not absolute and is limited to systems wherein
employees are not ingeniously and methodically deprived of their
constitutionally protected right to security of tenure. We are not
convinced that a big corporation such as petitioner cannot device a
system wherein a sufficient number of technicians can be hired with a
regular status who can take over when their colleagues are absent or
on leave, especially when it appears from the records that petitioner
hires so-called pinch-hitters regularly every month.

Nowhere in the records is there any showing that petitioner reported


the completion of its projects and the dismissal of private respondents
in its finished projects to the nearest Public Employment Office as per
Policy Instruction No. 2015 of the Department of Labor and
Employment [DOLE]. Jurisprudence abounds with the consistent rule
that the failure of an employer to report to the nearest Public
Employment Office the termination of its workers’ services every time
a project or a phase thereof is completed indicates that said workers
are not project employees. A project employee may also attain the
status of a regular employee if there is a continuous rehiring of project
employees after the stoppage of a project; and the activities performed
are usual [and] customary to the business or trade of the employer.
The circumstances set forth by law and jurisprudence is present in this
case. Even if private respondents are to be considered as project
employees, they attained regular employment status, just the same.

691
Page 692

Case Digests by: PARIL, JOSHUA F.


G.R. No. 192394. July 3, 2013.
Villarama Jr. J.

DOCTRINE:
ARTICLE 295
Failure of an employer to file termination reports after every
project completion proves that an employee is not a project
employee.

FACTS:
Roy D. Pasos worked on several projects for Philippine National
Construction Corporation on April 26, 1996 which should end on July
25, 1996 instead it was extended for up to 2 more years until August
1998. He was then rehired on November 1998 which was then
continuously being extended until October 19, 2000. He was then
asked to report to his superior for another reemployment. For purposes
of reemployment he went under medical examination which revealed
he had a pneumonitis and advised to take a 14day sick leave. After the
sick leave he had another medical examination that revealed he had a
Tuberculosis and to take a 60 day leave of absence. Petitioner claimed
that after he presented his medical clearance to the Project Personnel
Officer he was informed that his services were already terminated. This
prompted petitioner to file a case of illegal dismissal.

ISSUE:
Whether or not petitioner is a regular employee and not a mere
project employee.

RULING:
Petitioner is a regular employee. In the instant case, the
appointments issued to petitioner indicated that he was hired for
specific projects. This Court is convinced however that although he
started as a project employee, he eventually became a regular
employee of PNCC.

Under Article 280 of the Labor Code, as amended, a project employee


is one whose "employment has been fixed for a specific project or
undertaking the completion or termination of which has been
determined at the time of the engagement of the employee or where
the work or services to be performed is seasonal in nature and the
employment is for the duration of the season." Thus, the principal test
used to determine whether employees are project employees is
whether or not the employees were assigned to carry out a specific
project or undertaking, the duration or scope of which was specified at

692
Page 693

the time the employees were engaged for that project.

In the case at bar, petitioner worked continuously for more than two
years after the supposed three-month duration of his project
employment for the NAIA II Project. While his appointment for said
project allowed such extension since it specifically provided that in
case his "services are still needed beyond the validity of the contract,
the Company shall extend his services," there was no subsequent
contract or appointment that specified a particular duration for the
extension. His services were just extended indefinitely until "Personnel
Action Form Project Employment" dated July 7, 1998 was issued to
him which provided that his employment will end a few weeks later or
on August 4, 1998. While for first three months, petitioner can be
considered a project employee of PNCC, his employment thereafter,
when his services were extended without any specification of as to the
duration, made him a regular employee of PNCC. And his status as a
regular employee was not affected by the fact that he was assigned to
several other projects and there were intervals in between said projects
since he enjoys security of tenure.

Failure of an employer to file termination reports after every project


completion proves that an employee is not a project employee.

693
Page 694

Case Digests by: PARIL, JOSHUA F.

Gapayao v. Fulo
G.R. No. 193493. June 13, 2013.
Sereno, CJ.

DOCTRINE:
ARTICLE 295
Pakyaw workers are considered employees for as long as their
employers exercise control over them

FACTS:

Jaime Fulo had been working in a farm owned by Jaime Gapayao since
1983. In November 1997, Jaime Fulo was electrocuted while working
in the said farm. Jaime Fulo died. Thereafter, Rosario Fulo, the widow
of Jaime Fulo, filed a claim for death benefits before the SSS (Social
Security System). It turned out however that Jaime Fulo was never
registered with the SSS. Eventually, SSS ordered Gapayao, as the
employer, to pay the SSS contributions due with penalty.

Gapayao averred he cannot be made liable to pay the SSS


contributions because according to him there was no employer-
employee relationship between him and Jaime Fulo. He argued,
among others, that Jaime Fulo was not his employee because:

1. he did not work regular hours as he was only called when needed
and that Fulo can even look for other jobs elsewhere if he wanted to.
In fact, Fulo also worked for some other people;

2. he was only an “extra” in the farm;

3. Gapayao had no control over him (lack of control);

ISSUE:
Whether or not Jaime Fulo was an employee of Jaime Gapayao.

RULING:

Yes. Fulo was a regular employee and was thus entitled to receive
SSS benefits, among others. The Supreme Court agreed with the
Court of Appeals in ruling that it “does not follow that a person who
does not observe normal hours of work cannot be deemed an
employee.” It is also not material that Gapayao never supervised Fulo.

694
Page 695

In this case, the number of hours worked is not material. Gapayao is


considered a pakyaw worker. Pakyaw workers are considered regular
employees for as long as their employers have control over them. The
power of the employer to control the work of the employee is
considered the most significant determinant of the existence of an
employer-employee relationship. This is the so-called control test and
is premised on whether the person for whom the services are
performed reserves the right to control both the end achieved and the
manner and means used to achieve that end. It should be remembered
that the control test merely calls for the existence of the right to control,
and not necessarily the exercise thereof. It is not essential that the
employer actually supervises the performance of duties by the
employee. It is enough that the former has a right to wield the
power.

695
Page 696

Case Digests by: PARIL, JOSHUA F.

Millennium Erectors Corp. v. Magallanes


G.R. No. 184362. November 15, 2010.
Carpio-Morales, J.

DOCTRINE:
ARTICLE 295
A project employee is one whose "employment has been fixed for
a specific project or undertaking, the completion or termination of
which has been determined at the time of the engagement of the
employee or where the work or service to be performed is seasonal in
nature and the employment is for the duration of the season."

FACTS:
Magallanes is a utility man working for Tiu, the CEO of
Respondent. In July 2004, he was dismissed because of old age,
which prompted him to file an illegal dismissal complaint before the
Labor Arbiter. Petitioner filed a position paper arguing that respondent
was a project employee whom it hired for a building project in Libis on
January 30, 2003, to prove which it submitted the employment contract
signed by him; that on August 3, 2004, respondents services were
terminated as the project was nearing completion; and he was given
financial assistance in the amount ofP2,000, for which he signed a
quitclaim and waiver.

The Labor Arbiter ruled in favor of petitioner, holding that respondent


knew of his status as project employee, and that the project was
completed. On appeal to the NLRC, it held that Respondent was a
regular employee, and because of the payrolls, it is evident that
Respondent was employed for 16 years. The NLRC thus concluded
that while respondents work as a utility man may not have been
necessary or desirable in the usual business of petitioner as a
construction company, that he performed the same functions
continuously for 16 years converted an otherwise casual employment
to regular employment, hence, his termination without just or
authorized cause amounted to illegal dismissal.

ISSUE:
Whether or not Magallanes is a regular employee

RULING:
Yes. A project employee is one whose "employment has been
fixed for a specific project or undertaking, the completion or
termination of which has been determined at the time of the

696
Page 697

engagement of the employee or where the work or service to be


performed is seasonal in nature and the employment is for the duration
of the season."

The service of project employees are co-terminus with the project and
may be terminated upon the end or completion of that project or project
phase for which they were hired. Regular employees, in contrast, enjoy
security of tenure and are entitled to hold on to their work or position
until their services are terminated by any of the modes recognized
under the Labor Code.

Petitioner’s various payrolls dating as early as 2001 show that


respondent had been employed by it. As aptly observed by the
appellate court, these documents, rather than sustaining petitioner’s
argument, only serve to support respondent’s contention that he had
been employed in various projects, if not for 16 years, at the very least
two years prior to his dismissal.

Assuming arguendo that petitioner hired respondent initially on a per


project basis, his continued rehiring, as shown by the sample payrolls
converted his status to that of a regular employee.

697
Page 698

Case Digests by: PARIL, JOSHUA F.

Caparoso v. CA
G.R. No. 155505. February 15, 2007.
Carpio J.

DOCTRINE:
ARTICLE 295
An employee who is allowed to work after a probationary period
shall be considered a regular employee.

FACTS:
Composite Enterprises is a supplier of confectioneries Caparoso
and Quindipan were Composite’s deliverymen. Petitioners were
dismissed from the service and subsequently filed illegal dismissal
complaint. Respondents alleged that petitioners were both hired as
deliverymen, initially for three months and then on a month-to-month
basis and the termination from employment resulted from the
expiration of their contracts of employment on 8 October 1999.

Labor Arbiter - petitioners are regular employees of respondents


NLRC- Reversed LA decision CA- affirmed the NLRC’s decision. The
Court of Appeals held that respondents’ manpower requirement varies
from month to month depending on the demand from their clients for
their products. Respondents’ manpower requirement determines the
period of their employees’ services. Respondents employed
petitioners for the purpose of addressing a temporary manpower
shortage.

ISSUE:
Whether petitioners are regular employees of Composite
Enterprises

RULING:
Petitioners are Not Regular Employees Under Article 280 of the
Labor Code, a regular employee is (1) one who is engaged to perform
activities that are necessary or desirable in the usual trade or business
of the employer, or (2) a casual employee who has rendered at least
one year of service, whether continuous or broken, with respect to the
activity in which he is employed.

However, even if an employee is engaged to perform activities that


are necessary or desirable in the usual trade or business of the
employer, it does not preclude the fixing of employment for a definite

698
Page 699

period. We agree with the Court of Appeals that in this case, the fixed
period of employment was knowingly and voluntarily agreed upon by
the parties.

The Court of Appeals noted that there was no indication of force,


duress, or improper pressure exerted on petitioners when they signed
the contracts. Further, there was no proof that respondents were
regularly engaged in hiring workers for work for a minimum period of
five months to prevent the regularization of their employees.
Petitioners’ Employment is akin to Probationary Employment At most,
petitioners’ employment for less than six months can be considered
probationary.

Article 281 of the Labor Code provides: Art. 281. Probationary


Employment. - Probationary employment shall not exceed six (6)
months from the date the employee started working, unless it is
covered by an apprenticeship agreement stipulating a longer period.
The services of an employee who has been engaged on a probationary
basis may be terminated for a just cause or when he fails to qualify as
a regular employee in accordance with reasonable standards made
known by the employer to the employee at the time of his engagement.
An employee who is allowed to work after a probationary period shall
be considered a regular employee. Petitioners were hired on 11 May
1999, initially for three months. After the expiration of their contracts,
petitioners were hired on a month-to-month basis. Their contracts of
employment ended on 8 October 1999. Hence, they were employed
for a total of five months. Their employment did not even exceed six
months to entitle them to become regular employees.

699
Page 700

Case Digests by: PARIL, JOSHUA F.

Spouses Lim v. Legaspi Hope Christian School et. al


G.R. No. 172818. March 31, 2009.
Quisumbing J.

DOCTRINE:
ARTICLE 295
For a private school teacher to acquire permanent status in
employment, the following requisites must concur: (1) the teacher is a
full-time teacher; (2) the teacher must have rendered three consecutive
years of service; and (3) such service must have been satisfactory.

FACTS:

Petitioner-spouses Alwyn Ong Lim and Evelyn Lukang Lim were hired
in June 1999. Alwyn was assigned to teach Mathematics, Geometry,
Algebra and Trigonometry subjects in the high school department of
Legazpi Hope Christian School. Evelyn, on the other hand, was
assigned to teach Chinese Language 1 and 2 and Chinese Math
subjects in the elementary department of the same school.4

On April 4, 2002, respondent Helen Sia, head teacher of the school's


Chinese department, verbally informed petitioners that their
employment with the school were to be terminated, without giving the
reasons therefor.5 Thus, petitioners filed their complaints for illegal
dismissal and monetary claims against the school and its officials on
April 5, 2002.6 On May 31, 2002, respondent Ramon Sia, Vice
Chairman of the school's Board of Directors, sent a letter to the
petitioners stating that their three-year probation had expired and that
the school management had decided to discontinue their
employment.7

Before the Labor Arbiter, respondents claimed that petitioners were


merely part-time teachers and thus they can be dismissed even without
waiting for the three-year probation period to lapse, as they never
acquired permanent status.8

The Labor Arbiter ruled in favor of petitioners.

Respondents appealed the decision of the Labor Arbiter to the NLRC.


The NLRC found that petitioners were only part-time teachers who did
not acquire permanent status; hence, their dismissal was legal.

Petitioners contend that they were not issued any formal written
probationary contract. They also contend that they were never

700
Page 701

informed of reasonable standards under which they would be


evaluated or rated in connection with their supposed probationary
period of employment. Thus, in the absence of a written contract of
employment, upon their satisfactory completion of their three-year
probationary period they contend that they are considered as, and
became, regular and permanent teaching personnel of the respondent
school.

ISSUE:
Whether or not the petitioners were hired as permanent teaching
personnel.

RULING:

No. In University of Sto. Tomas v. NLRC, we ruled that for a private


school teacher to acquire permanent status in employment, the
following requisites must concur: (1) the teacher is a full-time teacher;
(2) the teacher must have rendered three consecutive years of service;
and (3) such service must have been satisfactory.

The burden is on petitioners to prove their affirmative allegation that


they are permanent teaching personnel. However, there is not enough
evidence on record to show that their total working day is devoted to
the school. There is no showing of what the regular work schedule of
a regular teacher in respondent school is. What is clear in the records
is that Evelyn and Alwyn spent two hours and four hours, respectively,
but not the entire working day, at the respondent school. They do not
meet requirement "c" of Section 45 of the Manual. Hence, we sustain
the findings of the Court of Appeals that the petitioners are part-time
teachers. Being part-time teachers, in accordance with University of
Sto. Tomas v. NLRC, they cannot acquire permanent status.

701
Page 702

Case Digests by: PARIL, JOSHUA F.

D.M CONSUNJI INC V ANTONIO GOBRES


G.R. No. 169170. August 9, 2010
Peralta, J.

DOCTRINE:
ARTICLE 295
A project employee is defined under Article 280 of the LC as one
whose employment has been fixed or a specific project or undertaking
the completion or termination of which has been determined at the time
of the engagement of the employee or where the work or services to
be performed is seasonal in nature and the employment is for the
duration of the season.

FACTS:
Respondents Gobres et al., worked as carpenters in the construction
projects of petitioner DM Consunji on several occasions and/or at
various times. Their termination from employment for each project was
reported to the Department of Labor and Employment (DOLE).
Respondents’ last assignment was at Quad 4-Project in Glorietta
where they started working on September 1, 1998. However, in
October, respondents saw their names included in the Notice of
Termination posted on the bulletin board at the project premises.
Hence, respondents filed a complaint with the NLRC against petitioner
for illegal dismissal, and non-payment of 13th month pay, 5 days
service incentive leave pay, and damages.

However, petitioner contends that respondents, being project


employees, are covered by Policy Instruction No. 20, as superseded
by Department Order No. 19, series of 1993 with respect to their
separation or dismissal. Further, respondents were allegedly
employed per project undertaken by petitioner and within varying
estimated periods indicated in their respective project employment
contracts. Citing the employment record of each respondent, petitioner
averred that respondents’ services were terminated when their phases
of work for which their services were engaged were completed or when
the projects themselves were completed. As such, the termination of
the respondents was warranted and legal. Moreover, petitioner
claimed that respondents have been duly paid their service incentive
leave pay and 13th month pay through their respective bank accounts,
as evidenced by bank remittances.
Respondents replied that the Quad 4-Project at Glorietta, Ayala,
Makati City was estimated to take two years to finish, but they were
dismissed within the two-year period. They had no prior notice of their

702
Page 703

termination. Hence, granting that they were project employees, they


were still illegally dismissed for non-observance of procedural due
process.

LABOR ARBITER: Dismissed complaint, finding that respondents


were project employees and that they were dismissed from the last
project they were assigned to when their respective phases of work
were completed, and that petitioner reported their termination of
services to the DOLE in accordance with the requirements of law

NLRC: Affirmed decision of LA

CA: Sustained decision of NLRC. However, it stated that although


respondents were project employees, they were still entitled to know
the reason for their dismissal and to be heard on whatever claims they
might have. Further, their right to statutory due process was violated
for lack of advance notice of their termination, even if they were validly
terminated for having completed the phases of work for which they
were hired. Hence, ordering the petitioner to pay respondents P20,000
each as nominal damages for lack of advance notice of their
termination.

Petitioners contend that the award of nominal damages to each


respondent is unwarranted under Sec. 2 (III), Rule XXIII, Book V of the
Omnibus Rules Implementing the Labor Code, which states that if the
termination is brought about by the completion of the contract or
phrase thereof, no prior notice is required.

ISSUE:
Whether respondents, as project employees, are entitled to
nominal damages for lack of advance notice of their dismissal.

RULING:
No. A project employee is defined under Article 280 of the LC as one
whose employment has been fixed or a specific project or undertaking
the completion or termination of which has been determined at the time
of the engagement of the employee or where the work or services to
be performed is seasonal in nature and the employment is for the
duration of the season.

The Court held that the Agabon vs. NLRC case is not applicable in the
case at bar because it involved the dismissal of regular employees for
abandonment of work, which is a just cause for dismissal under Art.
282 of the LC. Although the dismissal was for a cause, the employer
therein was required to observe the standard of due process for
termination of employment based on just cause. Since the employer
therein failed to comply with the twin requirements of notice and

703
Page 704

hearing, the Court ordered the employer to pay the employees nominal
damages for failure to observe procedural due process.
Unlike in Agabon, respondents, in this case, were not terminated for
just cause under Art. 282 of the LC. Dismissal based on just causes
contemplate acts or omissions attributable to the employee. Instead,
respondents were terminated due to the completion of the phases of
work for which their services were engaged.
Records show that respondents were dismissed after the expiration of
their respective project employment contracts, and due to the
completion of the phases of work respondents were engaged for.
Hence, the cited provisions requirements of due process or prior notice
when an employee is dismissed prior to the completion of the project
or phase thereof do not apply in this case.
In the case at bar, the LA, NLRC, and CA all found that respondents
were validly terminated due to the completion of the phases of work for
which respondents’ services were engaged. Section 2 (III), Rule XXIII,
Book V of the Omnibus Rules Implementing the Labor Code provides
that if the termination is brought about by the completion of the contract
or phase thereof, no prior notice is required. In the case of Cioco, Jr.
vs. C.E. Construction Corporation, it is explained that this is because
completion of the work or project automatically terminates the
employment, in which case, the employer is, under the law, only
obliged to render a report to the DOLE on the termination of the
employment.
Hence, prior or advance notice of termination is not part of procedural
due process if the termination is brought about by the completion of
the contract or phase thereof for which the employee was engaged.
Petitioner, therefore, did not violate any requirement of procedural due
process by failing to give respondents advance notice of their
termination; thus, there is no basis for the payment of nominal
damages.

704
Page 705

Case Digests by: PARIL, JOSHUA F.

Mercado v AMA
G.R. No. 183572. April 13, 2010.
Brion J.

DOCTRINE:
ARTICLE 295
The Labor Code is supplemented with respect to the period of
probation by special rules found in the Manual of Regulations for
Private Schools. The use of employment for fixed periods during the
teachers’ probationary period is likewise an accepted practice in the
teaching profession.

FACTS:
Petitioners were faculty members of AMA Computer College,
Paranaque city since 1998. During the school year 2000-2001,
AMACC implemented new faculty screening guidelines. Under the new
screening guidelines, teachers were to be hired or maintained based
on extensive teaching experience, capability, potential, high academic
qualifications and research background. The petitioners failed to obtain
a passing rating based on the performance standards; hence AMACC
did not give them any salary increase. Petitioners filed a complaint with
the Arbitration Branch of the NLRC on July 25, 2000, for underpayment
of wages, non-payment of overtime and overload compensation, 13th
month pay, and for discriminatory practices.
On September 7, 2000, petitioners were then given a notice of Non-
renewal of Contract.

ISSUE:
Whether or not the teachers’ probationary status should be
disregarded simply because the contracts were fixed terms.

RULING:
A reality we have to face in the consideration of employment on
probationary status of teaching personnel is that they are not governed
purely by the Labor Code. The Labor Code is supplemented with
respect to the period of probation by special rules found in the Manual
of Regulations for Private Schools.27 On the matter of probationary
period, Section 92 of these regulations provides:
“Section 92. Probationary Period.—Subject in all instances to
compliance with the Department and school requirements, the
probationary period for academic personnel shall not be more than
three (3) consecutive years of satisfactory service for those in the

705
Page 706

elementary and secondary levels, six (6) consecutive regular


semesters of satisfactory service for those in the tertiary level, and nine
(9) consecutive trimesters of satisfactory service for those in the
tertiary level where collegiate courses are offered on a trimester basis.”

The provision on employment on probationary status under the


Labor Code is a primary example of the fine balancing of interests
between labor and management that the Code has institutionalized
pursuant to the underlying intent of the Constitution.

On the one hand, employment on probationary status affords


management the chance to fully scrutinize the true worth of hired
personnel before the full force of the security of tenure guarantee of
the Constitution comes into play. Based on the standards set at the
start of the probationary period, management is given the widest
opportunity during the probationary period to reject hirees who fail to
meet its own adopted but reasonable standards. These standards,
together with the just and authorized causes for termination of
employment the Labor Code expressly provides, are the grounds
available to terminate the employment of a teacher on probationary
status.

Labor, for its part, is given the protection during the probationary
period of knowing the company standards the new hires have to meet
during the probationary period, and to be judged on the basis of these
standards, aside from the usual standards applicable to employees
after they achieve permanent status. Under the terms of the Labor
Code, these standards should be made known to the teachers on
probationary status at the start of their probationary period, or at the
very least under the circumstances of the present case, at the start of
the semester or the trimester during which the probationary standards
are to be applied. Of critical importance in invoking a failure to meet
the probationary standards, is that the school should show—as a
matter of due process —how these standards have been applied.

The school, however, cannot forget that its system of fixed-term


contract is a system that operates during the probationary period and
for this reason is subject to the terms of Article 281 of the Labor Code.
Unless this reconciliation is made, the requirements of this Article on
probationary status
would be fully negated as the school may freely choose not to renew
contracts simply because their terms have expired. The inevitable

706
Page 707

effect of course is to wreck the scheme that the Constitution and the
Labor Code established to balance relationships between labor and
management.

Given the clear constitutional and statutory intents, we cannot


but conclude that in a situation where the probationary status overlaps
with a fixed-term contract not specifically used for the fixed term it
offers, Article 281 should assume primacy and the fixed-period
character of the contract must give way. This conclusion is
immeasurably strengthened by the petitioners’ and the AMACC’s
hardly concealed expectation that the employment on probation could
lead to permanent status, and that the contracts are renewable unless
the petitioners fail to pass the school’s standards.

While we can grant that the standards were duly communicated


to the petitioners and could be applied beginning the 1st trimester of
the school year 2000-2001, glaring and very basic gaps in the school’s
evidence still exist. The exact terms of the standards were never
introduced as evidence; neither does the evidence show how these
standards were applied to the petitioners. Without these pieces of
evidence (effectively, the finding of just cause for the non-renewal of
the petitioners’ contracts), we have nothing to consider and pass upon
as valid or invalid for each of the petitioners. Inevitably, the non-
renewal (or effectively, the termination of employment of employees on
probationary status) lacks the supporting finding of just cause that the
law requires and, hence, is illegal.

In this light, the CA decision should be reversed. Thus, the LA’s


decision, affirmed as to the results by the NLRC, should stand as the
decision to be enforced, appropriately re-computed to consider the
period of appeal and review of the case up to our level.

707
Page 708

Case Digests by: PARIL, JOSHUA F.

Brent School v. Zamora


G.R. No. L-48494. February 5, 1990.
Narvasa, J.

DOCTRINE:
ARTICLE 295
Article 280 of the Labor Code should have no application to
instances where a fixed period of employment was agreed upon
knowingly and voluntarily by the parties, without any force, duress or
improper pressure being brought to bear upon the employee and
absent any other circumstances vitiating his consent, or where it
satisfactorily appears that the employer and employee dealt with each
other on more or less equal terms with no moral dominance whatever
being exercised by the former over the latter.

FACTS:
Private respondent Doroteo R. Alegre was engaged as athletic
director by petitioner Brent School, Inc. at a yearly compensation of
P20,000.00. The contract fixed a specific term for its existence, five (5)
years, i.e., from July 18, 1971, the date of execution of the agreement,
to July 17, 1976. Subsequent subsidiary agreements dated March 15,
1973, August 28, 1973, and September 14, 1974 reiterated the same
terms and conditions, including the expiry date, as those contained in
the original contract of July 18, 1971.

On April 20,1976, Alegre was given a copy of the report filed by Brent
School with the Department of Labor advising of the termination of his
services effective on July 16, 1976. The stated ground for the
termination was "completion of contract, expiration of the definite
period of employment." Although protesting the announced termination
stating that his services were necessary and desirable in the usual
business of his employer, and his employment lasted for 5 years -
therefore he had acquired the status of regular employee - Alegre
accepted the amount of P3,177.71, and signed a receipt therefor
containing the phrase, "in full payment of services for the period May
16, to July 17, 1976 as full payment of contract."

The Regional Director considered Brent School's report as an


application for clearance to terminate employment (not a report of
termination), and accepting the recommendation of the Labor
Conciliator, refused to give such clearance and instead required the
reinstatement of Alegre, as a "permanent employee," to his former
position without loss of seniority rights and with full back wages

708
Page 709

ISSUE:
Whether or not the provisions of the Labor Code, as amended,
have anathematized "fixed period employment" or employment for a
term.

RULING:
NO. The employment contract between Brent School and Alegre
was executed on July 18, 1971, at a time when the Labor Code of the
Philippines (P.D. 442) had not yet been promulgated. At that time, the
validity of term employment was impliedly recognized by the
Termination Pay Law, R.A. 1052, as amended by R.A. 1787. Prior,
thereto, it was the Code of Commerce (Article 302) which governed
employment without a fixed period, and also implicitly acknowledged
the propriety of employment with a fixed period. The Civil Code of the
Philippines, which was approved on June 18, 1949 and became
effective on August 30,1950, itself deals with obligations with a period.
No prohibition against term-or fixed-period employment is contained in
any of its articles or is otherwise deducible therefrom.

It is plain then that when the employment contract was signed between
Brent School and Alegre, it was perfectly legitimate for them to include
in it a stipulation fixing the duration thereof Stipulations for a term were
explicitly recognized as valid by this Court.

The status of legitimacy continued to be enjoyed by fixed-period


employment contracts under the Labor Code (PD 442), which went into
effect on November 1, 1974.

Accordingly, and since the entire purpose behind the development of


legislation culminating in the present Article 280 of the Labor Code
clearly appears to have been, as already observed, to prevent
circumvention of the employee's right to be secure in his tenure, the
clause in said article indiscriminately and completely ruling out all
written or oral agreements conflicting with the concept of regular
employment as defined therein should be construed to refer to the
substantive evil that the Code itself has singled out: agreements
entered into precisely to circumvent security of tenure. It should have
no application to instances where a fixed period of employment was
agreed upon knowingly and voluntarily by the parties, without any
force, duress or improper pressure being brought to bear upon the
employee and absent any other circumstances vitiating his consent, or
where it satisfactorily appears that the employer and employee dealt
with each other on more or less equal terms with no moral dominance
whatever being exercised by the former over the latter.

Alegre's employment was terminated upon the expiration of his last


contract with Brent School on July 16, 1976 without the necessity of
any notice. The advance written advice given the Department of Labor

709
Page 710

with copy to said petitioner was a mere reminder of the impending


expiration of his contract, not a letter of termination, nor an application
for clearance to terminate which needed the approval of the
Department of Labor to make the termination of his services effective.
In any case, such clearance should properly have been given, not
denied.

Alegre's contract of employment with Brent School having lawfully


terminated with and by reason of the expiration of the agreed term of
period thereof, he is declared not entitled to reinstatement

710
Page 711

Case Digests by: PARIL, JOSHUA F

PUREFOODS CORP V NLRC


G.R. No. 122653. December 12, 1997.
Davide Jr, J.

DOCTRINE:
ARTICLE 295
Under Art. 280, there are two kinds of regular employees are (1)
those who are engaged to perform activities which are necessary or
desirable in the usual business or trade of the employer; and (2) those
casual employees who have rendered at least one year of service,
whether continuous or broken, with respect to the activity in which they
are employed.

FACTS:
The private respondents were hired by petitioner Pure Foods to
work for a fixed period of five months at its tuna cannery plant in
General Santos City. After the expiration of their respective contracts
of employment, their services were terminated. They forthwith
executed a "Release and Quitclaim" stating that they had no claim
whatsoever against the petitioner. Private respondents then filed
before the NLRC-Sub-RAB a complaint for illegal dismissal against the
petitioner.

The Labor Arbiter dismissed the complaint on the ground that the
private respondents were mere contractual workers, and not regular
employees; hence, they could not avail of the law on security of tenure.
The termination of their services by reason of the expiration of their
contracts of employment was, therefore, justified.

The private respondents appealed the decision to the NLRC which


affirmed the LA’s decision. However, on private respondents' motion
for reconsideration, the NLRC rendered another decision holding that
the private respondent and their co-complainants were regular
employees. It declared that the contract of employment for five months
was a "clandestine scheme employed by the petitioner to stifle private
respondents' right to security of tenure" and should therefore be struck
down and disregarded for being contrary to law, public policy, and
morals. Hence, their dismissal on account of the expiration of their
respective contracts was illegal. Its motion for reconsideration having
been denied, the petitioner came to this Court contending that
respondent NLRC committed grave abuse of discretion amounting to
lack of jurisdiction in reversing the decision of the Labor Arbiter.

711
Page 712

ISSUE:
Whether or not private respondents are regular employees of
petitioner company or mere contractual employees.

RULING:
The SC held that the petition devoid of merit. Under Art. 280, there are
two kinds of regular employees are (1) those who are engaged to
perform activities which are necessary or desirable in the usual
business or trade of the employer; and (2) those casual employees
who have rendered at least one year of service, whether continuous or
broken, with respect to the activity in which they are employed.

In the instant case, the private respondents' activities consisted in the


receiving, skinning, loining, packing, and casing-up of tuna fish which
were then exported by the petitioner. Indisputably, they were
performing activities which were necessary and desirable in
petitioner's business or trade. Contrary to petitioner's submission, the
private respondents could not be regarded as having been hired for a
specific project or undertaking. The term "specific project or
undertaking" under Article 280 of the Labor Code contemplates an
activity which is not commonly or habitually performed or such type of
work which is not done on a daily basis but only for a specific duration
of time or until completion; the services employed are then necessary
and desirable in the employer's usual business only for the period of
time it takes to complete the project. The fact that the petitioner
repeatedly and continuously hired workers to do the same kind of work
as that performed by those whose contracts had expired negates
petitioner's contention that those workers were hired for a specific
project or undertaking only.

Although, this Court has upheld the legality of fixed-term employment,


none of the criteria had been met in the present case. It could not be
supposed that private respondent and all other so-called "casual"
workers of the petitioner KNOWINGLY and VOLUNTARILY agreed to
the 5-month employment contract. Cannery workers are never on
equal terms with their employers. Almost always, they agree to any
terms of an employment contract just to get employed considering that
it is difficult to find work given their ordinary qualifications. Their
freedom to contract is empty and hollow because theirs is the freedom
to starve if they refuse to work as casual or contractual workers.
Indeed, to the unemployed, security of tenure has no value. It could
not then be said that petitioner and private respondents "dealt with
each other on more or less equal terms with no moral dominance
whatever being exercised by the former over the latter.

The petitioner does not deny or rebut private respondents' averments


(1) that the main bulk of its workforce consisted of its so-called "casual"

712
Page 713

employees; (2) that as of July 1991, "casual" workers numbered 1,835;


and regular employee, 263; (3) that the company hired "casual" every
month for the duration of five months, after which their services were
terminated and they were replaced by other "casual" employees on the
same five-month duration; and (4) that these "casual" employees were
actually doing work that were necessary and desirable in petitioner's
usual business. This scheme of the petitioner was apparently designed
to prevent the private respondents and the other "casual" employees
from attaining the status of a regular employee. It was a clear
circumvention of the employees' right to security of tenure and to other
benefits like minimum wage, cost-of-living allowance, sick leave,
holiday pay, and 13th month pay. Indeed, the petitioner succeeded in
evading the application of labor laws. Also, it saved itself from the
trouble or burden of establishing a just cause for terminating
employees by the simple expedient of refusing to renew the
employment contracts.

The five-month period specified in private respondents'


employment contracts having been imposed precisely to circumvent
the constitutional guarantee on security of tenure should, therefore, be
struck down or disregarded as contrary to public policy or morals. To
uphold the contractual arrangement between the petitioner and the
private respondents would, in effect, permit the former to avoid hiring
permanent or regular employees by simply hiring them on a temporary
or casual basis, thereby violating the employees' security of tenure in
their jobs.

713
Page 714

Case Digests by: PARIL, JOSHUA F.

Leyte Geothermal Power Progressive Employees Union


v. PNOC
G.R. No. 170351. March 30, 2011.
Nachura, J.

DOCTRINE:
ARTICLE 295
The litmus test to determine whether an individual is a project
employee lies in setting a fixed period of employment involving a
specific undertaking which completion or termination has been
determined at the time of the particular employee's engagement.

FACTS:
Among PNOCs geothermal projects is the Leyte Geothermal
Power Project located at the Greater Tongonan Geothermal
Reservation in Leyte. Thus, the PNOC hired and employed hundreds
of employees on a contractual basis, whereby, their employment was
only good up to the completion or termination of the project and would
automatically expire upon the completion of such project. Majority of
the employees hired by PNOC in its Leyte Geothermal Power Projects
had become members of petitioner. In view of that circumstance, the
petitioner demands from the PNOC for recognition of it as the collective
bargaining agent of said employees and for a CBA negotiation with it,
which PNOC refused.

When the project was about to be completed, the PNOC served


Notices of Termination of Employment upon the employees who are
members of the petitioner. On December 28, 1998, the petitioner filed
a Notice of Strike with DOLE on the ground of purported commission
by the latter of unfair labor practice for "refusal to bargain collectively,
union busting and mass termination." On the same day, the petitioner
declared a strike and staged such strike. Efforts to settle the dispute
amicably failed.

ISSUE:
Whether or not the officers and members of petitioner Union are
project employees of respondent

RULING:
In accordance with Article 280 of the Labor Code, and as
explained in ALU-TUCP v. NLRC, the litmus test to determine whether
an individual is a project employee lies in setting a fixed period of
employment involving a specific undertaking which completion or

714
Page 715

termination has been determined at the time of the particular


employee's engagement. In this case, the officers and the members of
petitioner Union were specifically hired as project employees for
respondents Leyte Geothermal Power Project. Consequently, upon
the completion of the project or substantial phase thereof, the officers
and the members of petitioner Union could be validly terminated.

Petitioner Union is adamant, however, that the lack of interval in


the employment contracts its officer and members negates the latter's
status as mere project employees. However, petitioner Union's
members employment for more than a year does not equate to their
regular employment with respondent, as explained in Mercado, Sr. v.
NLRC, which stated that the proviso in Article 280, deeming all those
who had rendered service for more than one year as regular
employees, only applies to casual employees, and not project
employees.

715
Page 716

Case Digests by: John Ranier A. Rafael


Salazar vs. National Labor Relations Commission
G.R. No. 109210. April 17, 1996
Kapunan, J.

DOCTRINE:
TERMINATION OF EMPLOYMENT

Separation pay is not granted where the dismissed worker is a


project employee. Project employees are not entitled to termination
pay if they are terminated as a result of the completion of the project
or any phase thereof in which they are employed, regardless of the
number of projects in which they have been employed by a particular
construction company.

FACTS:
Salazar was employed by respondent H.L. Carlos Construction as
a construction/project engineer for the construction of the Monte de
Piedad building in Cubao, Quezon city on April 17 1990, A year later,
respondents project manager sent a memorandum to Salazar
informing him of the termination of his services. Salazar after receiving
the said memorandum filed a complaint against his former employer,
one of reliefs asked for was separation pay be granted to him. The
Labor arbiter declared that Salazar was a project employee and his
services were terminated due to the completion of the project. The
NLRC affirmed the assailed decision. The subsequent motion for
reconsideration was denied for lack of merit. Undaunted, petitioner
then raised his issue with the court

ISSUE:
Whether Salazar is entitled to separation pay or not

RULING:
No, as a project employee which the petitioner’s services are
deemed co-terminous with the project which means that his services
may be terminated as soon as the project for which he was hired is
completed. In the case at bench, there is no dispute as the Court held
that Salazar’s cause of dismissal was due to the completion of the
construction of the Monte De Piedad building. He himself stated that it
took him until May 15 1991 to complete the finishing touches on the
building.

716
Page 717

Case Digests by: John Ranier A. Rafael

Fonterra Brands Phils., Inc. vs. Leonardo Largado and


Teotimo Estrallado
G.R. No.205300. March 18,2015
Velasco,JR.,J

DOCTRINE:
FIXED TERM EMPLOYMENT CONTRACTS

Fixed-term employment contracts are not limited, as they are under


the present Labor Code, to those by nature seasonal or for specific
projects with predetermined dates of completion; they also include
those to which the parties by free choice have assigned a specific
date of termination.The determining factor of such contracts is not the
duty of the employee but the day certain agreed upon by the parties
for the commencement and termination of the employment
relationship.

FACTS:
Petitioner Fonterra contracted the services of Zytron Corporation
for the marketing and promotion of its milk and dairy products.
Pursuant to this contract, Zytron provided Fonterra trade
merchandising representative who are the respondent in this case,
Leonardo Largado and Teotimo Estrallado. Fonterra informed zytron
through a letter of its intention of terminating its promotion contract, the
effect of this was the termination of the services of both respondents.
The petitioner company instead engaged a different marketing
company named A.C. Sicat Marketing, Respondent due to their
intention to keep their employment with Fonterra, they applied for a
marketing position with A.C Sicat which hired them for five months.
Upon the lapse of 5 months from the date of engagement, their
employment contracts was not renewed. This prompted respondent to
file for illegal dismissal, regularization, non payment of service
incentive e leave and 13th month pay against the two marketing
companies. The Labor Arbiter dismissed the complaint and rule that
the respondent were not dismissed, but they were the one who refused
to renew their contract.

The NLRC affirmed the Labor Arbiter, finding that respondents'


separation from Zytron was brought about by the execution of the
contract between Fonterra and A.C. Sicat where the parties agreed to
absorb Zytron's personnel, including respondents. Too, respondents
failed to present any evidence that they protested this set-up.
Furthermore, respondents failed to refute the allegation that they

717
Page 718

voluntarily refused to renew their contract with A.C. Sicat. Also,


respondents did not assert any claim against Zytron and A.C. Sicat.

The CA found that A.C. Sicat satisfies the requirements of legitimate


job contracting, but Zytron does not. According to the CA: (1) Zytron's
paid-in capital of P250,000 cannot be considered as substantial
capital; (2) its Certificate of Registration was issued by the DOLE
months after respondents' supposed employment ended; and (3) its
claim that it has the necessary tools and equipment for its business is
unsubstantiated. Therefore, according to the CA, respondents were
Fonterra's employees.

The CA however held that respondent were illegally dismissed since


Fonterra failed to prove that their dismissal is lawful.

ISSUE:
Whether the Court of Appeals erred in ruling that Zytron was a
mere labor only contractor and whether the respondent were illegally
dismissed or not.

RULING:

The determination of whether Zytron is legitimate job contractor or not


is immaterial to the resolution of the case because respondents
voluntarily terminated their employment with zytron and applied for the
same position with A.C Sicat. The Labor arbiter and the NLRC are
correct with their ruling that the termination of employment by the those
respondent was due to the cessation of their respective contracts. By
refusing to renew their contracts espondents voluntarily terminated
their employment with Zytron by refusing to renew their employment
contracts with the latter, applying with A.C. Sicat, and working as the
latter's employees, thereby abandoning their previous employment
with Zytron. Too, it is well to mention that for obvious reasons,
resignation is inconsistent with illegal dismissal. This being the case,
Zytron cannot be said to have illegally dismissed respondents, contrary
to the findings of the CA.

As to A.C Sicat it is clear that respondents were employed by A.C.


Sicat as project employees. In their employment contract with the
latter, it is clearly stated that A.C Sicat temporarily employing
respondents as marketing representative effective June 6[, 2006]
under the following terms and conditions: The need for your service
being only for a specific project, your temporary employment will be for
the duration only of said project of our client, namely to promote

718
Page 719

FONTERRA BRANDS products which is expected to be finished on


or before Nov. 06, 2006

719
Page 720

Case Digests by: John Ranier A. Rafael

Basan vs. Coca-Cola Bottlers


G.R. No. 174365-66. Feb 04, 2015.
Peralta, C. J.

DOCTRINE:
Regular employees

In determining whether an employment should be considered


regular or non-regular, the applicable test is the reasonable
connection between the particular activity performed by the employee
in relation to the usual business or trade of the employer

The standard, supplied by the law itself, is whether the work


undertaken is necessary or desirable in the usual business or trade of
the employer, a fact that can be assessed by looking into the nature
of the services rendered and its relation to the general scheme under
which the business or trade is pursued in the usual course.

The repeated rehiring of respondent workers and the continuing


need for their services clearly attest the necessity or desirability of
their services in the regular conduct of the business or trade of
petitioner company.

FACTS:

On February 18, 1997, petitioners Romeo Basan, Danilo Dizon,


Jaime L. Tumabiao, Jr., Roberto Dela Rama, Jr., Ricky S. Nicolas,
Crispulo D. Donor, Galo Falguera filed a complaint for illegal dismissal
with money claims against respondent Coca-Cola Bottlers Philippines,
alleging that respondent dismissed them without just cause and prior
written notice required by law.

Respondent countered that they hired petitioners in a temporary


capacity, particularly to act as substitutes for its absent regular route
helpers to act as substitute for its absent regular workers, the
petitioners knew that their assignments as route helpers was
temporary in duration, hence they have no basis for the action.

The labor arbiter ruled in favor of petitioners and found they were
performing activities necessary and desirable to the usual business of
petitioner for more than the period for regularization hence they are
considered as regular employees and their dismissal was done
contrary to law in the absence of just cause and prior written notice.
The NLRC affirmed the Labor Arbiters decision and rejected

720
Page 721

respondent’s contention that petitioners were employed for a specific


project or undertaking.

Undaunted and persistent, respondent filed two petitions for


certiorari and reversed the rulings of the NLRC and Labor arbiter,
according to the appellate court the act of coca-cola of repeated hiring
for various period would not automatically categorize them as regular
employees

ISSUE:
Whether the petitioners are regular employees and entitled to
procedural and substantive due process?

RULING:

Yes, regular employees are classified into: (1) regular employees


by nature of work; and (2) regular employees by years of service. The
former refers to those employees who perform a particular activity
which is necessary or desirable in the usual business or trade of the
employer, regardless of their length of service; while the latter refers to
those employees who have been performing the job, regardless of the
nature thereof, for at least a year. They fall within the first classification
of employees

While fixed term employment is not per se illegal or against public


policy, the criteria above must first be established to the satisfaction of
this Court. Yet, the records of this case reveal that for years, petitioners
were repeatedly engaged to perform functions necessary to
respondent's business for fixed periods short of the six-month
probationary period of employment. If there was really no intent to
circumvent security of tenure, respondent should have made it clear to
petitioners that they were being hired only for fixed periods in an
agreement freely entered into by the parties. To this Court,
respondent's act of hiring and re-hiring petitioners for periods short of
the legal probationary period evidences its intent to thwart petitioner's
security of tenure

And as stated in the case of Brent v. Zamora It should have no


application to instances where a fixed period of employment was
agreed upon knowingly and voluntarily by the parties, without any
force, duress or improper pressure being brought to bear upon the
employee and absent any other circumstances vitiating his consent, or
where it satisfactorily appears that the employer and employee dealt
with each other on more or less equal terms with no moral dominance
whatever being exercised by the former over the latter

721
Page 722

Case Digests by: John Ranier A. Rafael

Convoy Marketing Corporation vs. Albia


G.R. No. 194969. October 17, 2015.
Peralta, C. J.

DOCTRINE:
REGULAR EMPLOYMENT- Indications or criteria which the term
employment cannot be said to be in circumvention of the law on
security of tenure.

1) The fixed period of employment was knowingly and voluntarily


agreed upon by the parties without any force, duress, or improper
pressure being brought to bear upon the employee and absent any
other circumstances vitiating his consent; or

2) It satisfactorily appears that the employer and the employee dealt


with each other on more or less equal terms with no moral dominance
exercised by the former or the latter.

Where from the circumstances it is apparent that the period has been
imposed to preclude acquisition of tenurial security by the employees,
they should be struck down as contrary to public policy.

FACTS:

Oliver Albiaa started working as a common laborer for the respondent


Convoy Marketing, a distributor of bottled wines, liquor and bottled
water, in 2001. He was assigned the job of a pahinante, or one who
loads and unloads cargoes transported to customers by the delivery
vehicles of the company. A year later, he was promoted to delivery van
driver.

On July 22, 2004, he did something that cost him his job. He smelled
of liquor upon his arrival from the delivery route. He gave the
explanation that after completing the delivery, he and his two
pahinantes decided to rest a little in a store outside the company
compound. They drank several bottles of beer before going back to the
compound to start loading for the next morning's delivery

This prompted Albia to protest his dismissal, by filing a complaint for


illegal dismissal. The company asserts that Albia is in an independent
contractor. at the end of every service period stated in the contracts,
the petitioner was studiedly made to sign a quitclaim and release in
which he acknowledged receiving a certain sum, at most P5,172.28, in
satisfaction of all claims that he may have against the company, and

722
Page 723

confirmed the termination of the agreement due to the expiration of the


stated period.

The Labor arbiter dismissed Albia’s complaint for lack of merit, stating
that the quitclaim is valid and binding upon the parties. The NLRC
dismissed the appeal and affirmed the Labor Arbiter’s decision.

However Court of appeals set aside and reversed the NLRC ruling

ISSUE:
Whether Albia is a regular employee of Convoy or not?

RULING:

He is a regular employee and thus entitled to all the rights of a


regular employee. Any employee who has rendered at least one year
of service, whether such services is continuous or broken shall be
considered a regular employee with respect to the activity in which he
is employed and his employment shall continue while such activity
exists. That Albia has become a regular employee is evident from the
Delivery Agency Agreements (For Driver)executed for the periods of
November 22, 2002 to April 22, 2003, May 29, 2003 to October 29,
2003, November 11, 2003 to April 10, 2004, and April 13, 2004 to
September 13, 2004 — which indicate that he had rendered at least
one year of broken service with respect to the same activity in which
he was employed from the time he was hired as a driver on November
22, 2002 until he was terminated on July 23, 2004. T

Neither could Albia be deemed a fixed-term contractual


employee, as the Delivery Agency Agreements executed between him
and Convoy fall short of the requisites for such fixed-term contracts to
be valid.

Considered to be legitimate under the Labor Code, fixed-term


employment contracts terminate by their own terms at the end of a
definite period. The fact that the service rendered by the employees is
usually necessary and desirable in the business operations of the
employer will not impair the validity of such contracts. For, the decisive
determinant in the term employment is not the activities that the
employee is called to perform, but the day certain agreed upon by the
parties for the commencement and termination of their employment
relationship

723
Page 724

Case Digests by: John Ranier A. Rafael

Jamias vs. NLRC


G.R. No. 159350. March 16, 2016.
Bersamin

DOCTRINE:
Fixed period employment

The test to determine whether a particular employee is engaged


as a project or regular employee is whether or not the employee is
assigned to carry out a specific project or undertaking, the duration or
scope of which was specified at the time of his engagement. There
must be a determination of, or a clear agreement on, the completion or
termination of the project at the time the employee is engaged.
Otherwise put, the fixed period of employment must be knowingly and
voluntarily agreed upon by the parties, without any force, duress or
improper pressure being brought to bear upon the employee and
absent any other circumstances vitiating his consent, or it must
satisfactorily appear that the employer and employee dealt with each
other on more or less equal terms with no moral dominance
whatsoever being exercised by the former on the latter.

FACTS:

Respondent Innodata Philippines, Inc. (Innodata), a domestic


corporation engaged in the business of data processing and
conversion for foreign clients, 14 individuals including the 3 petitioners
here Jamias, Matuguinas and Cruz was hired by Innodata.

After their respective contracts expired, the aforenamed


individuals filed a complaint for illegal dismissal claiming that Innodata
had made it appear that they had been hired as project employees in
order to prevent them from becoming regular employees

On September 8, 1998, Labor Arbiter (LA) Vicente Layawen


rendered his decision dismissing the complaint for lack of merit. He
found and held that the petitioners had knowingly signed their
respective contracts in which the durations of their engagements were
clearly stated; and that their fixed term contracts, being exceptions to
Article 280 of the Labor Code, precluded their claiming regularization.
The NLRC affirmed the LA’s decision opining that the labor code did
not prohibit employment contracts with fixed periods provided the
contracts had been voluntarily entered into by the parties.

724
Page 725

The Court of Appeals affirmed the judgment of the NLRC, this


was appealed by only 3 of the original complainants.

ISSUE:
Whether the fixed period employment contract between the
petitioner and respondent contravenes the Labor code

RULING:

The employment of the petitioners who were engaged as project


employees for a fixed term legally ended upon the expiration of their
contract. Their complaint for illegal dismissal was plainly lacking in
merit.

The contracts of the petitioners indicated the one-year duration


of their engagement as well as their respective project assignments
(i.e., Jamias being as assigned to the CD-ROM project; Cruz and
Matuguinas to the TSET project). here is no indication that the
petitioners were made to sign the contracts against their will. Neither
did they refute Innodata's assertion that it did not employ force,
intimidate or fraudulently manipulate the petitioners into signing their
contracts, and that the terms thereof had been explained and made
known to them. Hence, the petitioners knowingly agreed to the terms
of and voluntarily signed their respective contracts

The fixing by Innodata of the period specified in the contracts of


employment did not also indicate its ill-motive to circumvent the
petitioners' security of tenure. Indeed, the petitioners could not
presume that the fixing of the one-year term was intended to evade or
avoid the protection to tenure under Article 280 of the Labor Code in
the absence of other evidence establishing such intention. This
presumption must ordinarily be based on some aspect of the
agreement other than the mere specification of the fixed term of the
employment agreement, or on evidence aliunde of the intent to evade

725
Page 726

Case Digests by: Rafael, John Ranier A

Gadia vs Sykes Asia INC


G.R. No. 209499. January 28, 2015
Pernas J.

DOCTRINE:
PROJECT BASED EMPLOYEES

For an employee to be considered project-based, the employer


must show compliance with two (2) requisites, namely that: (a) the
employee was assigned to carry out a specific project or undertaking;
and (b) the duration and scope of which were specified at the time they
were engaged for such project.

FACTS:

Sykes Asia is a corporation engaged in Business Process


Outsourcing (BPO) which provides support to its international clients
from various sectors (e.g., technology, telecommunications, retail
services) by carrying on some of their operations, governed by service
contracts that it enters with them. On September 2, 2003, Alltel
Communications, Inc. (Alltel), a United States- based
telecommunications firm, contracted Sykes Asia's services to
accommodate the needs and demands of Alltel clients for its postpaid
and prepaid services (Alltel Project). Thus, on different dates, Sykes
Asia hired petitioners as customer service representatives, team
leaders, and trainers for the Alltel Project

Initially the partnership went smoothly until Alltel sent two letters
to Sykes that it was terminating all support services provided by Sykes
Asia related to the Alltell project, consequently Sykes sent end of life
notices to their employees. Aggrieved the employees filed separated
complaints for illegal dismissal

In their defense, the respondent employer averred that


petitioners were not regular employees but rather project based
employees and the termination of the project is a valid ground for their
dismissal

The Labor Arbiter ruled in favor of respondents and accordingly


dismissed the petitioners complaints for lack of merit.

The NLRC modified the LA ruling, that they are regular


employees but were validly terminated due to redundancy, the ruling
is based on the fact that it was neither determined nor made know to

726
Page 727

petitioners, at the time of the hiring when the project would end, be
terminated or completed.

The court of appeals annulled and set aside the ruling and
reinstated that of the LA, because the contracts readily shows that they
were hired exclusively for the Alltel project

ISSUE:

Whether or not the CA correctly granted respondents' petition for


certiorari, thereby setting aside the NLRC's decision holding that
petitioners were regular employees and reinstating the LA ruling that
petitioners were merely project-based employees, and thus, validly
dismissed from service.

RULING:

The Court finds that the CA correctly granted respondents'


certiorari petition before it, since the NLRC gravely abused its
discretion in ruling that petitioners were regular employees of Sykes
Asia when the latter had established by substantial evidence that they
were merely project-based.

In this case, records reveal that Sykes Asia adequately informed


petitioners of their employment status at the time of their engagement,
as evidenced by the latter's employment contracts which similarly
provide that they were hired in connection with the Alltel Project, and
that their positions were "project-based and as such is co-terminus to
the project." In this light, the CA correctly ruled that petitioners were
indeed project-based employees, considering that: (a) they were hired
to carry out a specific undertaking,i.e., the Alltel Project; and (b) the
duration and scope of such project were made known to them at the
time of their engagement, i.e., "co-terminus with the project when the
Alltel Project was terminated, petitioners no longer had any project to
work on, and hence, Sykes Asia may validly terminate them from
employment.

727
Page 728

Case Digests by: John Ranier A. Rafael

Innodata knowledge services vs. Inting


G.R. No. 211892, December 06, 2017.
Peralta, C. J.

DOCTRINE:
PROJECT EMPLOYEES AND FIXED TERM EMPLOYEES

Project employment and fixed-term employment are not the same.


While the former requires a particular project, the duration of a fixed-
term employment agreed upon by the parties may be any day certain,
which is understood to be "that which must necessarily come
although it may not be known when." The decisive determinant in
fixed-term employment is not the activity that the employee is called
upon to perform but the day certain agreed upon by the parties for the
commencement and termination of the employment relationship

FACTS:

Petitioner Innodata Knowledge Services, Inc.(IKSI) is a company


engaged in data processing, encoding, indexing, abstracting,
typesetting, imaging, and other processes in the capture, conversion,
and storage of data and information. At one time, Applied Computer
Technologies (ACT), a company based in the United States of
America, hired IKSI to review various litigation documents. Due to the
nature of the job, ACT required IKSI to hire lawyers, or at least, law
graduates, to review various litigation documents, classify said
documents into the prescribed categories, and ensure that outputs
are delivered on time. For this purpose, IKSI engaged the services of
respondents as senior and junior reviewers with a contract duration of
five (5) years.

On January 7, 2010, however, respondents received a Notice


of Forced Leave from IKSI informing them that they shall be placed
on indefinite forced leave due to changes in business conditions,
client requirements and specifications.

IKSI sent respondents separate notices informing them that


their project employment contracts would have to be terminated. This
prompted the respondent to file a case for illegal dismissal. The labor
arbiter declared there was no illegal dismissal which was affirmed by
the NLRC. However the CA granted the petition of respondents and
reversed the assailed NLRC ruling

728
Page 729

ISSUE:

Whether or not the CA committed an error when it reversed the


NLRC, which declared that respondent employees, as mere project
employees, were validly placed on floating status and, therefore,
were not illegally dismissed.

RULING:

The court rules in the negative, IKSI was able to show the
presence of a specific project, the ACT Project, in the contract and the
alleged duration of the same, it failed to prove, however, that
respondents were in reality made to work only for that specific project
indicated in their employment documents and that it adequately
informed them of the duration and scope of said project at the time their
services were engaged, the Court has declared that where from the
circumstances it is apparent that the periods have been imposed to
preclude acquisition of tenurial security by the employee, they should
be struck down as contrary to public policy or morals

There were no valid fixed-term or project contracts and


respondents were IKSI's regular employees who could not be
dismissed except for just or authorized causes. Any ambiguity in said
contracts must be resolved against the company, especially because
under Article 1702 of the Civil Code, in case of doubt, all labor contracts
shall be construed in favor of the worker. The Court cannot simply allow
IKSI to construe otherwise what appears to be clear from the wordings
of the contract itself. The interpretation which IKSI seeks to conjure is
wholly unacceptable, as it would result in the violation of respondents'
right to security of tenure guaranteed in Section 3 of Article XIII of the
Constitution and in Article 294

729
Page 730

Case Digests by: John Ranier A. Rafael

Pacific Metals vs. Tamayo


G.R. No.226920. December 5,2019
Lazaro-Javier

DOCTRINE:
PROJECT EMPLOYEE WHEN DEEMED A REGULAR EMPLOYEE

When an employee's services are extended without any specification


as to the duration, he is deemed to have become a regular employee
of the company. In the same vein, when Tamayo was re-hired by
PAMCO for an unspecified period and continuously worked for the
project for more than a year, he is deemed to have become a regular
employee of PAMCO

FACTS:

Petitioner Pacific Metals is a foreign company engaged n the


importation of nickel ore mined in the Philippines. In line with its desire
to purchase high quality nickel ore from its target area, PAMCO
negotiated to enter into an exploration agreement with Eramen
Minerals, Inc. (ERAMEN) for the development of a target area covered
by the latter's Mineral Production and Sharing Agreement (MPSA ) In
preparation for its joint venture business with ERAMEN, PAMCO
engaged the services of respondent Edgar Allan Tamayo, a licensed
and registered geologist. Tamayo signed up for a two-month
employment contract, commencing on September 2010. In turn,
PAMCO agreed to pay Tamayo P90,000.00 per month for his services.
According to PAMCO, Tamayo's two-month engagement was
extended for another two (2) months, or until January 31, 2011
Tamayo was designated manager for the ERAMEN/PAMCO
Exploration Project. As such, he was in charge of preparing the project
reports and updates, and budget requests for approval of Fernandez,
ERAMEN's president. There is no showing, however, that Tamayo's
engagement with the ERAMEN/PAMCO Exploration Project was
covered by an employment contract.

Subsequently, by letter dated November 29, 2011, Tamayo was


informed that his services as exploration manager was terminated
effective December 31, 2011 in view of the completion of the
exploration aspect of the project.

730
Page 731

On December 12, 2012, Tamayo filed a complaint for illegal


dismissal against PAMCO and ERAMEN. He prayed for backwages,
separation pay, 13th month pay, moral and exemplary damages, and
attorney's fees.

The labor arbiter rules that Tamayo was not a regular employee
but a project employee, The NLRC affirmed the decision of the LA that
Tamayo was not illegally dismissed but was terminated due to project
completion. The Court of Appeals however reversed the ruling that he
is a regular employee who had been illegally dismissed. According to
the Appellate court when Tamayo was rehired after the expiration of
his service contract, he ceased to be a project employee

Where it was ruled that when an employee's services are


extended without any specification as to the duration, he is deemed to
have become a regular employee of the company. In the same vein,
when Tamayo was re-hired by PAMCO for an unspecified period and
continuously worked for the project for more than a year, he is deemed
to have become a regular employee of PAMCO.

ISSUE:
Whether or not Tamayo is a regular or project employee

RULING:

He is a regular employee. That the exploration project was


allegedly already completed does not suffice to convince that indeed
the project had reached its conclusion. For no proof was adduced to
substantiate this allegation. It is quite unconvincing that the exploration
project was alleged to have already been completed or was even
nearing completion, only one year after its commencement,
considering that the project was actually good for five years. Surely, a
project good for five years could not have been accomplished for such
short period of one year.

He is a regular employee because he performs work which are


usually necessary and desirable to the business of the employer.
Tamayo is a licensed and registered geologist.

Although PAMCO persistently claims that Tamayo was only re-


hired for two (2) more months following the expiration of his first two-
month contract with the company, records bear that Tamayo rendered
service much longer than two (2) months. He was made to stay on for
a year for the work he rendered was in fact necessary and
indispensable to PAMCO's usual trade or business.

731
Page 732

Case Digests by: John Ranier A. Rafael

Claret School of Quezon City vs. Sinday


G.R. No. 226358. October 9, 2019.
Leonen,J

DOCTRINE:
FIXED TERMED CONTRACTS- WHEN SHOULD IT BE
DISREGARDED.

The Civil Code and the Labor Code allow the execution of fixed-term
employment contracts. However, in cases where periods are imposed
to prevent an employee from acquiring security of tenure, such
contracts must be disregarded for being contrary to public policy and
morals. Brent's application is limited to cases where the employer and
the employee are more or less on an equal footing when they enter
into the contract

FACTS:

Claret School of Quezon City (Claret) is an educational institution


located on Mahinhin Street, UP Village, Quezon City. Sinday is the wife
of Wencil Sinday, one (1) of Claret's longtime drivers.

On February 18, 2014, Sinday filed her Complaint for illegal


dismissal against the school. Sinday narrated that in April 2010, Claret
engaged her as a releasing clerk in its book sale, tasking her with the
inventory and release of books to Claret's students

In July 2010, Sinday worked as a filing clerk at Claret's Human


Resources Department, where she updated employees' files, delivered
memoranda to different departments, and assisted in school programs.
In April 2011, she was posted back as a releasing clerk. She held this
position until July 14, 2011.

Before her job as releasing clerk expired, Sinday applied for work
at one (1) of Claret's departments, Claret Technical-Vocational
Training Center (Claretech), which taught vocational and technical
skills to underprivileged students. On July 15, 2011, she started her
new work as secretary, preparing materials, assisting in the delivery of
correspondence to other departments, and encoding and filing
documents, among other tasks. To classify her as a regular employee.
She was classified under the non-teaching or non-academic school
employees.

732
Page 733

In May 2013, Claret asked Sinday to sign a Probationary


Employment Contract covering the period of January 16, 2013 to July
15, 2013. When the contract expired, Sinday asked Leticia Perez, the
Human Resources head of Claret, regarding her employment status,
but she was told that her tenure would expire on July 31, 2013 because
of the change in school administration

Sinday filed her Complaint, claiming that she had been a regular
employee as she performed various jobs that were usually necessary
and desirable in the usual business of Claret

On the other hand, Claret denied Sinday's claims averring that


she was merely a part-time fixed-term contractual employee whom the
school accommodated because her husband was its longtime driver.
It also argued that Sinday was well aware of her fixed-term
employment as confirmed by her application letters and biodata, which
showed her employment's duration.

The labor arbiter found that Sinday was illegally dismissed, The
Labor Arbiter ruled that the repeated hiring of Sinday for around three
(3) years conferred her with regular employment status. The NLRC
reversed the Labor arbiters decision and found that Sinday was not
illegally dismissed.

The Appellate court found that Sinday was Illegally dismissed,


Hence the present petition

ISSUE:
Whether or not respondent Sinday is a regular employee or a
fixed term employee

RULING:

Yes. Sinday is a regular employee, to be a fixed term employee,


Brent laid down the criteria under which a fixed- term employment
cannot be deemed in circumvention of the security of tenure:

(1) When the parties have knowingly and voluntarily agreed upon a
fixed period of employment "without any force, duress[,] or improper
pressure being brought to bear upon the employee and absent any
other circumstances vitiating his consent"; or

(2) When "it satisfactorily appears that the employer and employee
dealt with each other on more or less equal terms" with the employer
not having exercised any moral dominance over the employee.

The criteria limit the application of Brent to particular cases where


the employer and the employee are on a more or less equal footing in
entering into the contract. If none of the aforementioned criteria are

733
Page 734

present, this Court will strike down a fixed-term employment contract,


which was done by the court in this case.

There is no genuine freedom to contract when a fixed-term


employment is used as a vehicle to exploit the economic disadvantage
of workers like respondent. Plain wage earners should not be faulted
for tolerating jobs they desperately need. Brent recognized the validity
of fixed- term employments only within the context that employers and
employees are on an equal footing

734
Page 735

Case Digests by: John Ranier A. Rafael

Gapayao vs. Fulo


G.R. No193494. June 13,2013.
Sereno, C. J.

DOCTRINE:
SEASONAL EMPLOYEES

Farm workers generally fall under the definition of seasonal


employees. We have consistently held that seasonal employees may
be considered as regular employees. Regular seasonal employees
are those called to work from time to time. The nature of their
relationship with the employer is such that during the off season, they
are temporarily laid off; but reemployed during the summer season or
when their services may be needed. They are in regular employment
because of the nature of their job, and not because of the length of
time they have worked.

Pakyaw workers are considered employees for as long as their


employers exercise control over them

FACTS:

On 4 November 1997, Jaime Fulo (deceased) died of "acute


renal failure secondary to 1st degree burn 70% secondary
electrocution" while doing repairs at the residence and business
establishment of petitioner Jaime Gapayao

Allegedly moved by his Christian faith, petitioner extended some


financial assistance to private respondent. On 16 November 1997, the
latter executed an Affidavit of Desistance stating that she was not
holding them liable for the death of her late husband, Jaime Fulo, and
was thereby waiving her right and desisting from filing any criminal or
civil action against petitioner and subsequently both parties executed
a Compromise Agreement.

Private respondent filed a claim for social security benefits with


the Social Security System (SSS) — Sorsogon Branch. However, upon
verification and evaluation, it was discovered that the deceased was
not a registered member of the SSS.

Upon the insistence of private respondent that her late husband


had been employed by petitioner from January 1983 up to his untimely
death on 4 November 1997, the SSS conducted a field investigation to

735
Page 736

clarify his status of employment. SSS’s findings stated that the


deceased was an employee of the petitioners in this case.

Private respondent filed a Petition before the SSC on 17


February 2003. In her Petition, she sought social security coverage
and payment of contributions in order to avail herself of the benefits
accruing from the death of her husband. petitioner filed an Answer
disclaiming any liability on the premise that the deceased was not the
former's employee, but was rather an independent contractor whose
tasks were not subject to petitioner's control and supervision

The SSC rendered a Resolution that the deceased was an


employee of the petitioner, was ordered to pay P45,315.95
representing the unpaid SS contributions due on behalf of deceased
Jaime Fulo, the amount of P217,710.33 as 3% per month penalty for
late remittance thereof, computed as of March 30, 2006, without
prejudice to the collection of additional penalty accruing thereafter, and
the sum of P230,542.20 (SSS) and P166,000.00 (EC) as damages for
the failure of the respondent to report the deceased Jaime Fulo for SS
coverage prior to his death pursuant to Section 24(a) of the SS Law.

Petitioner appealed to the Court of Appeals, which was denied.


Upon appeal to the Supreme Court, petitioner alleges that the
deceased was engaged on pakyaw basis and worked for a short period
of time, in the nature of a farm worker every season

ISSUE:

The sole issue presented the Court is whether or not there


exists between the deceased Jaime Fulo and petitioner an employer-
employee relationship that would merit an award of benefits in favor
of private respondent under social security laws.

RULING:

There exists an employer-employee relationship between the


deceased and the petitioner . A reading of the records reveals that the
deceased was indeed a farm worker who was in the regular employ of
petitioner. From year to year, starting January 1983 up until his death,
the deceased had been working on petitioner's land by harvesting
abaca and coconut, processing copra, and clearing weeds. His
employment was continuous in the sense that it was done for more
than one harvesting season. Moreover, no amount of reasoning could
detract from the fact that these tasks were necessary or desirable in
the usual business of petitioner.

The other tasks allegedly done by the deceased outside his usual
farm work only bolster the existence of an employer-employee
relationship. Pakyaw workers are considered employees for as long as

736
Page 737

their employers exercise control over them. It should be remembered


that the control test merely calls for the existence of the right to control,
and not necessarily the exercise thereof.

737
Page 738

Case Digests by: John Ranier A. Rafael

Paz vs Northern Tobacco


G.R. No. 199554 February 18,2015
Leonon,J

DOCTRINE:
REGULAR SEASONAL EMPLOYEES.

Article 280 of the Labor Code and jurisprudence identified three types
of employees, namely: “(1) regular employees or those who have
been engaged to perform activities which are usually necessary or
desirable in the usual business or trade of the employer; (2) project
employees or those whose employment has been fixed for a specific
project or undertaking, the completion or termination of which has
been determined at the time of the engagement of the employee or
where the work or service to be performed is seasonal in nature and
the employment is for the duration of the season; and (3) casual
employees or those who are neither regular nor project employees.”

Jurisprudence also recognizes the status of regular seasonal


employees.

FACTS:

Northern Tobacco Redrying Co., Inc. (NTRCI), a flue-curing and


redrying of tobacco leaves business, employs approximately 100
employees with seasonal workers "tasked to sort, process, store and
transport tobacco leaves during the tobacco season of March to
September".

NTRCI hired Zenaida Paz (Paz) sometime in 1974 as a seasonal


sorter, paid P185.00 daily. NTRCI regularly re-hired her every tobacco
season since then. She signed a seasonal job contract at the start of
her employment and a pro-forma application letter prepared by NTRCI
in order to qualify for the next season.

On May 18, 2003, Paz was 63 years old when NTRCI informed her
that she was considered retired under company policy. A year later,
NTRCI told her she would receive P12,000.00 as retirement pay. Paz,
with two other complainants, filed a Complaint for illegal dismissal
against NTRCI on March 4, 2004. She amended her Complaint on April
27, 2004 into a Complaint for payment of retirement benefits,
damages, and attorney's fees as P12,000.00 seemed inadequate for
her 29 years of service.

738
Page 739

NTRCI countered that no Collective Bargaining Agreement (CBA)


existed between NTRCI and its workers. Thus, it computed the
retirement pay of its seasonal workers based on Article 287 of the
Labor Code.

NTRCI raised the requirement of at least six months of service a year


for that year to be considered in the retirement pay computation. It
claimed that Paz only worked for at least six months in 1995, 1999,
and 2000 out of the 29 years she rendered service. Thus, Paz's
retirement pay amounted to P12,487.50 after multiplying her P185.00
daily salary by 22 1/2 working days in a month, for three years.

The labor arbiter confirmed and sided with the employer, stating it’s
the correct computation. The NLRC modified the decision stating that
Zenaida Paz retirement pay should be computed pursuant to RA
7641 and that all the months she was engaged to work for
respondent for the last twenty eight years should be added and
divide by six (for a fraction of six months is considered as one year)
to get the number of years for her retirement pay

and the appellate court dismissed the petition and modified the lower
tribunals decision however it awarded financial assistance to Zenaida
Paz in the amount of 60,356.25 pesos.

Paz now comes before the Supreme Court asking that the computation
of the NLRC to reinstate the decision of the NLRC with respect to the
computation of the retirement pay.

Petitioners argument is that Article 287, as amended by Republic Act


No. 7641, applies and entitles her to retirement pay equivalent to
atleast one-half month salary for every year of service, a fraction of at
least six (6) months being considered as one whole year. She adds
that she was then 63 years old, and while one may opt to retire at 60
years old, the compulsory retirement age is 65 years old under Article
287, as amended.

respondent NTRCI argues that unlike regular employees, seasonal


workers like petitioner Paz can offer their services to other employers
during off-season. Thus, the six-month rule avoids the situation where
seasonal workers receive retirement pay twice — an even more
favorable position compared with regular employees

739
Page 740

ISSUE:
Whether there was illegal dismissal, what is the proper
computation for the retirement pay and what kind of employee is the
petitioner

RULING:
Paz is considered as a regular seasonal employee because she
performed the same task every season for several years. As such,
she is entitled to the rights under Art 279 of the Labor Code.

The Supreme Court further ruled that Petitioner Paz was illegally
dismissed from her employment despite the amendment to her
Complaint because she maintained in her pleadings that she had
been made to retire even before reaching the age of 65, pursuant to
Article 287, as amended. Petitioner Paz was only 63 years old when
she was informed by NTRCI that she was considered retired under
company policy. However, NTRCI failed to prove a valid company
retirement policy. Hence, Paz is deemed entitled to her full
backwages from May 18, 2003 until she reached the compulsory
retirement age of 65 in 2005.

The Labor Code requires employers to comply with both


procedural and substantive due process in dismissing employees. As
provided in several cases decided by this Court, if the dismissal is
based on just causes, the employer must give the employee two
written notices and an opportunity to be heard. If the dismissal is
based on authorized causes, the employer must give the employee
and the DOLE written notices at least 30 days prior to the date of
effectivity.

In the case at bar, NTRCI had considered Paz as retired at the


age of 63. This does not fall under the just causes enumerated in Art.
282; under authorized causes in Art. 283; nor disease as a ground for
termination in Art. 284. There was also no showing that the
respondent had complied with due process requisites. Hence,
consistent with jurisprudence, Paz is entitled to nominal damages in
the amount of P30,000.00.

Applying the formula in Art. 287 of the Labor Code, the


retirement pay was correctly computed at P12,487.50. The Court also
ruled that indeed, the retirement pay is too meager to support Paz
who has become old, weak, and unable to find gainful employment.
Hence, she is entitled to financial assistance amounting to
P60,356.25.

740
Page 741

Case Digests by: John Ranier A. Rafael

Kimberly Independent Labor Union v. Drilon


G.R. Nos. 77629 & 7879, May 9, 1990.
Regalado, J.

DOCTRINE:
CASUAL EMPLOYEES
The law thus provides for two kinds of regular employees,
namely: (1) those who are engaged to perform activities which are
usually necessary or desirable in the usual business or trade of the
employer; and (2) those who have rendered at least one year of
service, whether continuous or broken, with respect to the activity in
which they are employed. The individual petitioners herein who have
been adjudged to be regular employees fall under the second
category. These are the mechanics, electricians, machinists, machine
shop helpers, warehouse helpers, painters, carpenters, pipefitters
and masons. It is not disputed that these workers have been in the
employ of KIMBERLY for more than one year at the time of the filing
of the petition for certification election by KILUSAN-OLALIA.

Owing to their length of service with the company, these workers


became regular employees, by operation of law, one year after they
were employed by KIMBERLY through RANK.

FACTS:
Kimberly-Clark Philippines, Inc. (KIMBERLY) executed a three-
year collective bargaining agreement with United Kimberly-Clark
Employees Union-Philippine Transport and General Workers’
Organization (UKCEUPTGWO) which expired on June 30, 1986.
During the freedom period, some members of the bargaining unit
formed another union called Kimberly Independent Labor Union for
Solidarity, Activism and Nationalism-Organized Labor Association in
Line Industries and Agriculture (KILUSAN-OLALIA). The new union
filed a petition for certification election to which Kimberly and
UKCEUPTGWO did not object but objected to the inclusion of
contractual workers whose employment was coursed through an
independent contractor as qualified voters. However, during the
pendency of the petition, KILUSAN-OLALIA filed a notice of strike
and eventually staged a strike until then Minister Sanchez issued an
assumption order. During the pre-election conference, 64 casual
workers were challenged by KIMBERLY and UKCEU-PTGWO on the
ground that they are not employees of KIMBERLY but of RANK. It
was agreed by all the parties that the 64 voters shall be allowed to

741
Page 742

cast their votes but that their ballots shall be segregated and subject
to challenge proceedings.

Later on, Minister Sanchez ruled that (1) the service contract for
janitorial and yard maintenance services between KIMBERLY and
RANK was declared legal; (2) that the other casual employees not
performing janitorial and yard maintenance services were deemed
labor-only contractuals and since labor-only contracting is prohibited,
such employees were held to have attained the status of regular
employees, the regularization being effective as of the date of the
decision; and (3) that UKCEU-PTGWO, having garnered more votes
than KILUSAN-OLALIA, was certified as the exclusive bargaining
representative of KlMBERLY’s employees.

KILUSAN-OLALIA filed a motion for reconsideration questioning


the authority of the Minister of Labor to assume jurisdiction over the
representation issue.

In a petition, KILUSAN-OLALIA avers that the respondent


Secretary of Labor and/or the former Minister of Labor have acted
with grave abuse of discretion and/or without jurisdiction in ruling on
the issue of bargaining representation and declaring respondent
UKCEU-PTGWO as the collective bargaining representative of all
regular rank-and-file employees of the respondent company; in
holding that the 64 employees are not entitled to vote in the
certification election; and that considering the regularization of
petitioners (who are not janitors and maintenance employees) to be
effective only on the date of the disputed decision of Minister
Sanchez. KILUSAN-OLALIA contends that after finding that the 64
workers are regular employees of KIMBERLY, Minister Sanchez
should have remanded the representation case to the med-arbiter,
instead of declaring UKCEU-PTGWO as the winner in the certification
election and setting aside the med-arbiter’s order which allowed the
64 casual workers to cast their votes.

ISSUE:
Whether or not the casual employees not performing janitorial
and yard maintenance service who has rendered at least one year of
service is considered as regular employees

Whether or not the 64 casual employees are qualified to vote in


the certification election

RULING:
The Supreme Court ruled that the former labor minister gravely
abused his discretion in holding that those workers not engaged in

742
Page 743

janitorial or yard maintenance service attained the status of regular


employees only on November 13, 1986, which thus deprived them of
their constitutionally protected right to vote in the certification election
and choose their rightful bargaining representative.

The law provides for two kinds of regular employees, namely: (1)
those who are engaged to perform activities which are usually
necessary or desirable in the usual business or trade of the employer;
and (2) those who have rendered at least one year of service,
whether continuous or broken, with respect to the activity in which
they are employed. The individual petitioners herein who have been
adjudged to be regular employees fall under the second category. It
is not disputed that these workers have been in the employ of
KIMBERLY for more than one year at the time of the filing of the
petition for certification election by KILUSAN-OLALIA.

Owing to their length of service with the company, these workers


became regular employees, by operation of law, one year after they
were employed by KIMBERLY through RANK.

The Court ruled that as a consequence of their status as regular


employees, those workers are entitled to the payment of salary
differential, cost of living allowance, 13th month pay, and such other
benefits extended to regular employees under the CBA, from the day
immediately following their first year of service in the company. These
regular employees are likewise entitled to vote in the certification
election held in July 1, 1986. As such, to finally determine the certified
bargaining representative, the votes cast by those employees not
performing janitorial and yard maintenance service, which form part
of the 64 challenged votes, should be opened, counted and
considered.

743
Page 744

Case Digests by: John Ranier A. Rafael

Philippine Geothermal Inc. v. NLRC


G.R. No. 82643-67. August 30, 1990.
Paras, J.

DOCTRINE:
CASUAL EMPLOYEES
Assuming therefore, that an employee could properly be regarded
as a casual (as distinguished from a regular employee) he becomes
entitled to be regarded as a regular employee of the employer as soon
as he has completed one year of service. Under the circumstances,
employers may not terminate the service of a regular employee except
for a just cause or when authorized under the Labor Code. It is not
difficult to see that to uphold the contractual arrangement between the
employer and the employee would in effect be to permit employers to
avoid the necessity of hiring regular or permanent employees
indefinitely on a temporary or casual status, thus to deny them security
of tenure in their jobs.

FACTS:
Philippine Geothermal, Inc is a US corporation engaged in the
exploration and development of geothermal energy resources as an
alternative source of energy. On the other hand, the private
respondents of the case are employees of the petitioner occupying
various positions ranging from carpenter to clerk, who had worked for
the company under individual contracts from 15 days to 3 months. The
contractual employment were regularly renewed to the extent that the
private respondents had rendered service from 3 to 5 years until 1983
and 1984 when they were notified that the petitioner will no longer
renew their individual contracts.

Since the private respondents were excluded in the bargaining unit


of the regular rank and file employees, they organized a separate labor
union. They filed a petition for certification election. However, they
alleged that the company started harassing them and replaced them
with contract workers. Hence, they filed a case for illegal lock-out and
unfair labor.

The LA ruled in favor of the respondents, declaring them as


regular and permanent employees. The NLRC affirmed.

744
Page 745

ISSUE:
Whether or not the private respondents are regular and permanent
employees

RULING:
YES. In the recent case of Kimberly Independent Labor Union for
Solidarity, Activism, and Nationalism-Olalia vs. Hon. Franklin M. Drilon,
G.R. Nos. 77629 and 78791 promulgated last May 9, 1990, this Court
classified the two kinds of regular employees, as: 1) those who are
engaged to perform activities which are usually necessary or desirable
in the usual business or trade of the employer; and 2) those who have
rendered at least one (1) year of service, whether continuous or broken
with respect to the activity in which they are employed. While the actual
regularization of these employees entails the mechanical act of issuing
regular appointment papers and compliance with such other operating
procedures, as may be adopted by the employer, it is more in keeping
with the intent and spirit of the law to rule that the status of regular
employment attaches to the casual employee on the day immediately
after the end of his first year of service.

Even an employee is regarded as only a casual employee, he


becomes entitled to be regarded as regular employee as soon as he
has completed one year of service. To uphold the contractual
arrangement between the employer and the employee would in effect
be to permit employers to avoid the necessity of hiring regular or
permanent employees indefinitely on a temporary or casual status,
thus to deny them security of tenure in their jobs.

In the case at bar, the private respondents are deemed regular


and permanent employees of Philippine Geothermal, Inc.

745
Page 746

Case Digests by: John Ranier A. Rafael

Samonte v. La Salle Greenhills


G.R. No. 199683. February 10, 2016.
Perez, J.

DOCTRINE:
Fixed-term Employment
A fixed-term employment is allowable under the Labor Code
wherein the parties agree upon the day certain for the commencement
and termination of their employment relationship. A day certain being
understood to be "that which must necessarily come, although it may
not be known when. Furthermore, the term must be voluntarily and
knowingly entered into by the parties who must have dealt with each
other on equal terms not one exercising moral dominance over the
other.

FACTS:
La Salle Greenhills, Inc. (LSGI) contracted the services of medical
professionals, from 1989 and for 15 years thereafter, to comprise its
Health Service Team. The members of the team signed a uniform
Contract of Retainer for a specific academic year, which terminates at
the end of the school year. After 15 years of continuous renewal, the
petitioners were informed by LSGI Head Administrator that their
contracts will no longer be renewed for the upcoming school year.
Hence, the petitioners and their colleagues requested fir the payment
of their separation pay. When such was denied by LSGI, they filed a
complaint for illegal dismissal with prayer for separation pay, damages
and attorney’s fees.

The petitioners contend that they were regular employees. As


such, they could only be removed for just or authorized causes. They
contend that they were receiving a regular monthly salary; 13th month
pay; automatic yearly salary increase; they were subjected to various
performance rating; and that they serve an average of nine hours a
week but beyond the work hours, they were on call for medical
emergencied of the La Sallian community. They further alleged that
they are required to attend staff meetings and to participate in
formulation of policies and programs designed to enhance the School
services.

On the other hand, LSGI contends that the complainants were


professional physicials and dentists on a retainer basis and were paid
their monthly retainer fees; that LSGI had no power to impose
disciplinary measures upon them; and that LSGI had no power of

746
Page 747

control over how complainants actually performed their professional


services. LSGI invoked the doctrine pronounced in the case of Sonza
v. ABS-CBN to justify its stance that the petitioners were independent
contractors and not regular employees.

The LA ruledthat the complainants were independent contractors.


However, the LA awarded separation pay to the petitioners on the
ground of compassionate social justice.

The NLRC disagreed and ruled that the Contract of Retainers


between the complainants and LSGI are valid fixed-term employment.
As such, the petitioners are not entitled to reinstatement nor to
payment of separation pay. The Court of Appeals likewise ruled
against petitioners’ claim of regular employment.

ISSUE:
Whether or not the complainants are regular employees who may
only be dismissed for just and authorized causes

RULING:
The Supreme Court ruled that the petitioners are all regular
employees of LSGI. The uniform one-page Contracts of Retainer
signed by petitioners were prepared by LSGI alone. Petitioners,
medical professionals as they were, were still not on equal footing with
LSGI as they obviously did not want to lose their jobs that they had
stayed in for fifteen (15) years. There is no specificity in the contracts
regarding terms and conditions of employment that would indicate that
petitioners and LSGI were on equal footing in negotiating it. The
Contract of Retainer very clearly spelled out that LSGI had the power
of control over petitioners.

Time and again we have held that the power of control refers to
the existence of the power and not necessarily to the actual exercise
thereof, nor is it essential for the employer to actually supervise the
performance of duties of the employee.17 It is enough that the
employer has the right to wield that power.

Jurisprudence had established that a fixed-term employment is


allowable under the Labor Code only if the term was voluntarily and
knowingly entered into by the parties who must have dealt with each
other on equal terms not one exercising moral dominance over the
other. Further, a fixed-term contract is an employment contract, the
repeated renewals of which make for a regular employment. In the
case at bar, the Court of Appeals disregarded the repeated renewals
of the Contracts of Retainer of petitioners spanning a decade and a
half.

747
Page 748

In the case at bar, the Supreme Court ruled that due to the
repeated renewal of the contract for 15 years, interrupted only by the
close of the school year; the necessity of the work performed by the
petitioners; and the existence of power of control, the petitioners are
deemed regular employees entitled to security of tenure who could
only be dismissed for just and authorized causes.

Since the petitioners were illegally dismissed, the Court ordered


separation pay in lieu of reinstatement.

748
Page 749

Case Digests by: MIKHAILA KLAUDINE A. ROSALES

ST. LUKE’S MEDICAL CENTER v. SANCHEZ


G.R. No. 212054. March 11, 2015.
Perlas-Bernabe, J.

DOCTRINE:
RIGHT TO DISCIPLINE
Among the employer’s management prerogatives is the right to
prescribe reasonable rules and regulations necessary or proper for the
conduct of its business or concern, to provide certain disciplinary
measures to implement said rules and to assure that the same would
be complied with. Any willful or intentional disobedience to reasonable
rules justifies the termination of the employment of the employee.

FACTS:
Respondent Sanchez was hire by petitioner St. Luke’s Medical
Center as Staff Nurse in its Pediatric Unit. At the end of her shift, she
passed through the entrance/exit standard inspection. The security
guard noticed a pouch in her bag and asked her to open it. The security
guard found that the pouch contained medical stocks. Sanchez was
asked to write an incident report and a letter of apology. In the letter,
Sanchez categorically stated that despite knowing it was prohibited,
she still took the medical stocks. Sanchez argued that as a practice,
she saved these excess stocks to be used in immediate procedures,
and that she only failed to return the pouch on the day of the incident.
Petitioner terminated her services for violation of the Code of Discipline
and for acts of dishonesty.

Sanchez filed a complaint for illegal dismissal. The LA ruled that


Sanchez was validly dismissed. However, the NLRC reversed the LA
decision and ruled that Sanchez was illegally dismissed, as petitioner
failed to prove ill-will on the part of Sanchez. The CA upheld the NLRC
ruling, hence this petition.

ISSUE:
Whether respondent’s dismissal was a valid exercise of
petitioner’s management prerogative to discipline its employees

RULING:

Yes, respondent’s dismissal was a valid exercise of petitioner’s


right to discipline its employees.

749
Page 750

Among the employer’s management prerogatives is the right to


prescribe reasonable rules and regulations necessary or proper for the
conduct of its business or concern, to provide certain disciplinary
measures to implement said rules and to assure that the same would
be complied with. At the same time, the employee has the corollary
duty to obey all reasonable rules, orders, and instructions of the
employer; and willful or intentional disobedience thereto, as a general
rule, justifies termination of the contract of service and the dismissal of
the employee.

In this case, the Court finds that Sanchez was validly dismissed by
SLMC for her willful disregard and disobedience of Section 1, Rule I of
the SLMC Code of Discipline, which reasonably punishes acts of
dishonesty, i.e., “theft, pilferage of hospital or co-employee property”
with termination from employment. Such act is obviously connected
with Sanchez’s work, who, as a staff nurse, is tasked with the proper
stewardship of medical supplies. As it is clear that the company
policies subject of this case are reasonable and lawful, sufficiently
known to the employee, and evidently connected with the latter’s work,
the Court concludes that SLMC dismissed Sanchez for a just cause.

750
Page 751

Case Digests by: MIKHAILA KLAUDINE A. ROSALES

PHILIPPINE SPAN ASIA CARRIERS CORP. v. PELAYO


G.R. No. 212003. February 28, 2018.
Leonen, J.

DOCTRINE:
RIGHT TO DISCIPLINE; INVESTIGATION
Disciplining employees does not only entail the demarcation of
permissible and impermissible conduct through company rules and
regulations, and the imposition of appropriate sanctions. It also
involves intervening mechanisms to assure that employers' rules
would be complied with. These mechanisms include the conduct of
investigations to address employee wrongdoing.

FACTS:
Respondent Pelayo was employed by petitioner Suplicio Lines,
now Philippine Span Asia Carriers Corporation, as Accounting Clerk.
Sulpicio Lines discovered several anomalous transactions in its office.
During the investigation, Pelayo was interviewed as she was the one
who personally prepared the cash vouchers and checks. During the
follow-up interview, Pelayo walked out, claiming that she was being
coerced to admit complicity. Pelayo was later admitted to a hospital
because of depression. She eventually stopped reporting for work.
Petitioner issued a memorandum asking Pelayo for an explanation.
She was also placed on preventive suspension. However, instead of
responding, Pelayo filed a complaint for constructive dismissal.

The LA ruled that Pelayo was constructively dismissed and


faulted petitioner for harassing Pelayo. The NLRC reversed the LA
decision, and ruled that the matter of disciplining employees was a
management prerogative and that respondent’s involvement in the
investigation does not necessarily amount to harassment. The CA
reversed the NLRC, and agreed with the findings of the LA, hence this
petition.

ISSUE:
Whether respondent’s participation in the investigation
constituted harassment, which renders her dismissal illegal

RULING:

No, respondent’s participation in the investigation does not


amount to harassment.

751
Page 752

An employer who conducts investigations following the discovery


of misdeeds by its employees is not being abusive when it seeks
information from an employee involved in the workflow which
occasioned the misdeed. An employee's involvement in such an
investigation will naturally entail difficulty. This difficulty does not mean
that the employer is creating an inhospitable employment atmosphere
so as to ease out the employee involved in the investigation.

In this case, this Court fails to see how the petitioner's


investigation amounted to respondent's constructive dismissal. There
is no objective proof demonstrating how the interview in Cebu actually
proceeded. Other than respondent's bare allegation, there is nothing
to support the claim that her interviewers were hostile, distrusting, and
censorious, or that the interview was a mere pretext to pin her down.
Respondent's recollection is riddled with impressions, unsupported by
independently verifiable facts. These impressions are subjective
products of nuanced perception, personal interpretation, and ingrained
belief that cannot be appreciated as evidencing "the truth respecting a
matter of fact." Respondent's subsequent hospitalization does not
prove harassment or coercion to make an admission either. The mere
fact of its occurrence is not an attestation that respondent's interview
proceeded in the manner that she claimed it did.

752
Page 753

Case Digests by: MIKHAILA KLAUDINE A. ROSALES

ZUELLIG FREIGHT AND CARGO SYSTEMS v. NLRC


G.R. No. 157900. July 22, 2013.
Bersamin, J.

DOCTRINE:
CHANGE OF NAME NOT BONA FIDE CLOSURE OF BUSINESS
The changing of the name of a corporation is no more the
creation of a corporation than the changing of the name of a natural
person is begetting of a natural person. The act, in both cases, would
seem to be what the language which we use to designate it imports –
a change of name, and not a change of being.

FACTS:
Private respondent San Miguel was a checker/customs
representative of Zeta, now petitioner. He and other employees were
informed that their services were being terminated as Zeta would
cease operations. San Miguel filed a complaint for illegal dismissal. He
contended that the amendments of the articles of incorporation of Zeta
were for the purpose of changing the corporate name, broadening the
primary functions, and increasing the capital stock; and that such
amendments could not mean that it had been thereby dissolved.
Petitioner countered that San Miguel’s termination from employment
had been for a cause authorized by the Labor Code.

The LA ruled that San Miguel was illegally dismissed, and held
that there was merely a change of business name and primary purpose
and upgrading of stocks of the corporation. The NLRC affirmed the
LA’s decision. The CA also affirmed the NLRC decision, hence this
petition.

ISSUE:
Whether the closure of the business operations of petitioner was
bona fide, thereby justifying the dismissal of private respondent

RULING:
No, the closure of the business of operations of petitioner was
not bona fide, thereby resulting in the illegal dismissal of private
respondent.

The amendments of the articles of incorporation of Zeta to


change the corporate name to Zuellig Freight and Cargo Systems, Inc.
did not produce the dissolution of the former as a corporation. The
effect of the change of name was not a change of the corporate being.

753
Page 754

The changing of the name of a corporation is no more the creation of


a corporation than the changing of the name of a natural person is
begetting of a natural person. The act, in both cases, would seem to
be what the language which we use to designate it imports – a change
of name, and not a change of being.

Zeta and petitioner remained one and the same corporation. The
change of name did not give petitioner the license to terminate
employees of Zeta like San Miguel without just or authorized cause. he
dismissal of San Miguel from employment on the pretext that petitioner,
being a different corporation, had no obligation to accept him as its
employee, was illegal and ineffectual.

754
Page 755

Case Digests by: MIKHAILA KLAUDINE A. ROSALES

PECKSON v. ROBINSONS SUPERMARKET


CORPORATION
G.R. No. 198534. July 03, 2013.
Reyes, J.

DOCTRINE:
RIGHT TO TRANSFER
If the transfer of an employee is not unreasonable, or
inconvenient, or prejudicial to him, and it does not involve a
demotion in rank or a diminution of his salaries, benefits and
other privileges, the employee may not complain that it amounts
to a constructive dismissal.

FACTS:
Petitioner Peckson was hired by respondent Robinsons
Supermarket Corporation as Sales Clerk. When she was already
holding the position of Category Buyer, the Assistant Vice President
reassigned her to the position of Provincial Coordinator. Peckson
claims that her new assignment was a demotion because it was non-
supervisory. She refused to turn over her responsibilities to the new
Category Buyer. She also refused to accept her new responsibilities.
Respondent denied that the transfer was a demotion. It claimed that
the position has the same work conditions and salaries. Respondent
also claimed that Peckson did not possess the traits of punctuality and
diligence required of a Category Buyer.

The LA ruled in favor of respondent and held that job


reassignment or classification is a strict prerogative of employer and
that petitioner cannot refuse her transfer. The NLRC sustained the
findings of the LA. The CA affirmed the NLRC ruling, hence this
petition.

ISSUE:
Whether the reassignment or transfer of petitioner was a valid
exercise of respondent’s management prerogative

RULING:

Yes, the reassignment or transfer of petitioner was a valid


exercise of respondent’s management prerogative.

The exercise of management’s prerogative concerning the


employees’ work assignments is based on its assessment of the

755
Page 756

qualifications, aptitudes and competence of its employees, and by


moving them around in the various areas of its business operations it
can ascertain where they will function with maximum benefit to the
company. When the transfer of an employee is not unreasonable, or
inconvenient, or prejudicial to him, and it does not involve a demotion
in rank or a diminution of his salaries, benefits and other privileges, the
employee may not complain that it amounts to a constructive dismissal.

In this case, the respondents have discharged the burden of


proof that the transfer of the petitioner was not tantamount to
constructive dismissal. there is substantial showing that the transfer
of the petitioner from Category Buyer to Provincial Coordinator was not
unreasonable, inconvenient, or prejudicial to her. The petitioner failed
to dispute that the job classifications of Category Buyer and Provincial
Coordinator are similar, or that they command a similar salary structure
and responsibilities. We agree with the NLRC that the Provincial
Coordinator’s position does not involve mere clerical functions but
requires the exercise of discretion from time to time, as well as
independent judgment, since the Provincial Coordinator gives
appropriate recommendations to management and ensures the faithful
implementation of policies and programs of the company. It even has
influence over a Category Buyer because of its recommendatory
function that enables the Category Buyer to make right decisions on
assortment, price and quantity of the items to be sold by the store.

756
Page 757

Case Digests by: MIKHAILA KLAUDINE A. ROSALES

GATBONTON v. NLRC
G.R. No. 146779. January 23, 2006.
Austria-Martinez, J.

DOCTRINE:
PREVENTIVE SUSPENSION
Preventive suspension is a disciplinary measure for the
protection of the company’s property pending investigation of any
alleged malfeasance or misfeasance committed by the employee. The
employer may place the worker concerned under preventive
suspension if his continued employment poses a serious and imminent
threat to the life or property of the employer or of his co-workers.
However, when it is determined that there is no sufficient basis to justify
an employee’s preventive suspension, the latter is entitled to the
payment of salaries during the time of preventive suspension.

FACTS:
Petitioner Gatbonton was an associate professor of respondent
Mapua Institute of Technology (MIT), Faculty of Civil Engineering. A
civil engineering student of respondent MIT filed a letter-complaint
against petitioner for unfair/unjust grading system, sexual harassment
and conduct unbecoming of an academician. MIT, through its
Committee on Decorum and Investigation placed petitioner under a 30-
day preventive suspension. The Committee believed that petitioner’s
continued stay during the investigation would affect his performance
as a faculty member, as well as the student’s learning. The Committee
also believed that the suspension will allow petitioner to prepare
himself for the investigation and will prevent his influences on other
members of the community. Petitioner filed with the NLRC a complaint
for illegal suspension, damages and attorney’s fees.

The LA ruled that the preventive suspension was illegal, and that
respondents shall pay petitioner’s wages during the period of his
preventive suspension. The NLRC reversed the decision of the LA.
The CA affirmed the ruling of the NLRC.

ISSUE:
Whether the imposition of the preventive suspension against
petitioner is legal

RULING:
No, the imposition of the preventive suspension against
petitioner is illegal.

757
Page 758

The Labor Code provides that the employer may place the
worker concerned under preventive suspension if his continued
employment poses a serious threat to the life or property of the
employer or of his co-workers.

In this case, there is nothing on record which shows that


respondent MIT imposed the preventive suspension on petitioner as
his continued employment poses a serious threat to the life or property
of the employer or of his coworkers; therefore, his preventive
suspension is not justified. The Mapua Rules is one of those issuances
that should be published for its effectivity, since its purpose is to
enforce and implement R.A. No. 7877, which is a law of general
application. At the time of the imposition of petitioners preventive
suspension on January 11, 1999, the Mapua Rules were not yet legally
effective, and therefore the suspension had no legal basis.

758
Page 759

Case Digests by: MIKHAILA KLAUDINE A. ROSALES

AUTOMATIC APPLIANCES, INC. v. DEGUIDOY


G.R. No. 228088. December 4, 2019.
Reyes, Jr., J.

DOCTRINE:
RIGHT TO TRANSFER
Transfer of an employment and designation of tasks is
management prerogative, unless there is clear cut discrimination or
intended to make an employee unable to perform of her function due
to her transfer, or is a diminution of benefits, then management
prerogative principle should not be set aside.

FACTS:
Petitioner Automatic Appliances, Inc. (AAI) is a corporation
organized and existing under the laws of the Philippines. Petitioners
Lim, Buenaventura and Pontillas are the former President, Vice
President for Human Resource and Tutuban Branch Manager
respectively of the said corporation. AAI hired Deguidoy as a regular
sales coordinator. She was tasked with selling merchandise and was
required to maintain branch sales quota. AAI suffered decline in its
sales and experienced economic difficulties. It then implemented cost
cutting measures, which included closing some of its branches. AAI
issued a memorandum, informing its employees of their re-shuffling
and re-assignment to AAI’s various brances. As a result, Deguidoy was
re-assigned from the Cubao Branch to the Tutuban Branch. She
accepted her re-assignment. She failed to reach her sales quota. She
also incurred 29 days of unexplained absences.

Upon hearing the deficiencies of Deguidoy, AAI issued


attendance infraction memo of inefficiency and gross negligence.
Deguido was placed under one month suspension. Upon return, AAI
verbally informed her of an intended transfer to its Ortigas branch,
Dismayed, Deguidoy left during her lunch break, and never returned.
AAI sent Deguidoy a letter requiring her to explain her failure to report
for work. Deguidoy ignored the said letter twice. Still the same was
unheeded. Unknown to AAI, Deguidoy filed a case for illegal dismissal.

ISSUE:
Whether the transfer was a valid exercise of management
prerogative

RULING:
Yes, the transfer was a valid exercise of management
prerogative.

759
Page 760

Transfer of an employment and designation of tasks is


management prerogative, unless there is clear cut discrimination or
intended to make an employee unable to perform of her function due
to her transfer, or is a diminution of benefits, then management
prerogative principle should not be set aside.

In this case, AAI’s decision to transfer respondent to its Ortigas


branch was the result of an assiduous review of the latter’s work
performance balanced alongside the company’s business needs. It
was backed by evidence consisting of Deguidoy’s sales output and
attendance records. In the same vein, AAI’s reassignments were not a
spur of the moment move. It began as a series of measures to
streamline its operations. Deguidoy was not singled out or
discriminated against.

760
Page 761

Case Digests by: MIKHAILA KLAUDINE A. ROSALES

TELUS INTERNATIONAL PHILIPPINES, INC. v. DE


GUZMAN
G.R. No. 202676. December 4, 2019.
Hernando, J.

DOCTRINE:
RIGHT TO TRANSFER; FLOATING STATUS
The temporary lay-off wherein the employees cease to work
should not exceed six months. Moreover, placing employees in a valid
"floating status" presupposes that there are more employees than
work.
FACTS:
Respondent was hired by petitioner as Inbound Sales Associate.
He was charged with insulting or showing discourtesy, disrespect, or
arrogance towards superiors or co-team members and abusive
behavior language. He was placed on preventive suspension. He was
found not liable for the said offenses. However, he was removed from
his current designation and was transferred to another practice. De
Guzman was forced to apply for vacation leave. After exhausting his
vacation leaves, he was informed that he was not yet required to return
to work. He was considered as a floater and will not get paid unless his
floating status has been lifted. De Guzman filed a complaint for illegal
dismissal.

The LA ruled that petitioner is guilty of constructively dismissing


De Guzman, as he was not immediately reinstated to his former
position after his preventive suspension. This was reversed by the
NLRC, which found that De Guzman filed to prove that he was
constructively dismissed. On appeal, the CA ruled that De Guzman
was constructively dismissed, hence this petition.

ISSUE:
Whether respondent was validly placed on floating status

RULING:
No, respondent was not validly placed on floating status.

While there is no specific provision in the Labor Code which


governs the "floating status" or temporary "off detail" of workers
employed by agencies, it is implicitly recognized in Article 301 of the
Labor Code which speaks of situations of temporary retrenchment or
lay-off due to valid operation issues. This situation applies not only in
security services but also in other industries. Relevantly, it has been
held that in all cases however, the temporary lay-off wherein the

761
Page 762

employees cease to work should not exceed six months. Moreover,


placing employees in a valid "floating status" presupposes that there
are more employees than work.

In this case, this Court cannot subscribe to the argument of the


company that placing De Guzman on "floating status" was perfectly
acceptable under the labor laws. Telus compared De Guzman's
circumstances to that of security guard on "off detail" and insists that
the call center industry is on all fours with that of a security agency or
bus companies to their drivers wherein placing the employees on
floating status without salaries or financial benefit for an indefinite time
is a valid recourse so long as it does not exceed six months. Telus did
not provide any valid justification or presented proof that there was
indeed a deficit of account that bars the immediate transfer of De
Guzman or that the company was sustaining losses that would justify
placing De Guzman on floating status. Hence, the unwarranted acts of
Telus evidently constitute proof of the constructive dismissal of De
Guzman. To say that Telus merely exercised its rights and that any
inconvenience or injury that De Guzman may have suffered resulted
merely in damnum absque injuria which cannot legally give rise to a
cause of action for constructive dismissal, is abhorrent considering the
fact that his being placed on a "floating status" without valid reasons
violated his security of tenure and resulted in unfavorable economic
consequences to De Guzman

762
Page 763

Case Digests by: MIKHAILA KLAUDINE A. ROSALES

PUNCIA v. TOYOTA SHAW/PASIG, INC.


G.R. No. 214399. June 28, 2016.
Perlas-Bernabe, J.

DOCTRINE:
RIGHT TO IMPOSE PRODUCTIVITY STANDARD
An employer is entitled to impose productivity standards for its
employees, and the latter's non-compliance therewith can lead to his
termination from work.

FACTS:
Puncia was hired by Toyota as Marketing Professional. He was
tasked to sell 7 vehicles per month as quote. Puncia was able to sell
only 1 vehicle in July and none in August. Puncia was asked to explain,
but he failed to appear during the hearing. His services were later on
terminated on the ground of insubordination. Puncia filed a complaint
for illegal dismissal. He argued that his dismissal was merely due to
Toyota’s discovery that he was a director of the Union. On the other
hand, Toyota argued that Puncia’s dismissal was due to his failure to
comply with the company’s strict requirements on sales quota.

The LA dismissed Puncia’s complaint for lack of merit. It ruled


that he was terminated for a just cause due to his inefficiency brought
about by his failure to meet the required monthly quota. The NLRC
reversed the LA ruling and held that Puncia was illegally dismissed. On
appeal, the CA reinstated the decision of the LA, hence this petition.

ISSUE:
Whether petitioner’s failure to meet the required monthly sales
quota justifies his dismissal from employment

RULING:
Yes, petitioner’s failure to meet the required monthly sales quota
justifies his dismissal from employment.

An employer is entitled to impose productivity standards for its


employees, and the latter's non-compliance therewith can lead to his
termination from work. The petitioners' failure to meet the sales quota
assigned to each of them constitute a just cause of their dismissal,
regardless of the permanent or probationary status of their
employment. Failure to observe prescribed standards of work, or to
fulfill reasonable work assignments due to inefficiency may constitute
just cause for dismissal. Such inefficiency is understood to mean

763
Page 764

failure to attain work goals or work quotas, either by failing to complete


the same within the allotted reasonable period, or by producing
unsatisfactory results.

In this case, records reveal that as a Marketing Professional for


Toyota, Puncia had a monthly sales quota of seven (7) vehicles from
March 2011 to June 2011. As he was having trouble complying with
said quota, Toyota even extended him a modicum of leniency by
lowering his monthly sales quota to just three (3) vehicles for the
months of July and August 2011; but even then, he still failed to
comply. In that six (6)-month span, Puncia miserably failed in satisfying
his monthly sales quota, only selling a measly five (5) vehicles out of
the 34 he was required to sell over the course of said period. Verily,
Puncia's repeated failure to perform his duties - i.e., reaching his
monthly sales quota - for such a period of time falls under the concept
of gross inefficiency.

764
Page 765

Case Digests by: MIKHAILA KLAUDINE A. ROSALES

DUNCAN ASSOCIATION OF DETAILMAN-PTGWO v.


GLAXO WELLCOME PHILIPPINES, INC.
G.R. No. 162994. September 17, 2004.
Tinga, J.

DOCTRINE:
RULE ON MARRIAGE
The prohibition against personal or marital relationships with
employees of competitor companies is reasonable under the
circumstances because relationships of that nature might compromise
the interests of the company.

FACTS:
Respondent Glaxo is a pharmaceutical company. Petitioner
Tecson was hired by Glaxo as medical representative. In their contract
of employment, it was stipulated that he shall disclose to the
management any existing or future relationship with co-employees or
employees of competing drug companies. It was also stated in the
contract that if the management finds that the relationship poses a
conflict of interest, Tecson shall resign from the company. Tecson later
married Betsy, the Branch Coordinator of Astra, a competitior of Glaxo.
Glaxo asked Tecson if he would resign, but he merely asked for more
time to decide. Tecson was transferred to another branch. Later, he
was offered his separation pay.

Tecson claimed that he was constructively dismissed. On the


other hand, Glaxo argued that the company policy prohibiting its
employees from having a relationship with and/or marrying an
employee of a competitor company is a valid exercise of its
management prerogatives.

ISSUE:
Whether the company policy prohibiting employees from having
a relationship with employees of a competitor company is a valid
exercise of management prerogative

RULING:
Yes, the company policy is a valid exercise of management
prerogative.

The prohibition against personal or marital relationships with


employees of competitor companies upon Glaxo’s employees is
reasonable under the circumstances because relationships of that

765
Page 766

nature might compromise the interests of the company. In laying down


the assailed company policy, Glaxo only aims to protect its interests
against the possibility that a competitor company will gain access to its
secrets and procedures.

Glaxo’s policy prohibiting an employee from having a relationship


with an employee of a competitor company is a valid exercise of
management prerogative. Glaxo has a right to guard its trade secrets,
manufacturing formulas, marketing strategies and other confidential
programs and information from competitors, especially so that it and
Astra are rival companies in the highly competitive pharmaceutical
industry. That Glaxo possesses the right to protect its economic
interests cannot be denied. No less than the Constitution recognizes
the right of enterprises to adopt and enforce such a policy to protect its
right to reasonable returns on investments and to expansion and
growth.

766
Page 767

Case Digests by: MIKHAILA KLAUDINE A. ROSALES

TIU v. PLATINUM PLANS PHIL., INC.


G.R. No. 163512. February 28, 2007.
Quisumbing, J.

DOCTRINE:
POST-EMPLOYMENT BAN
A non-involvement clause is not necessarily void for being in
restraint of trade as long as there are reasonable limitations as to time,
trade, and place.

FACTS:
Respondent Platinum Plans Phil., Inc. is a corporation engaged
in the pre-need industry. Petitioner Tiu was hired by respondent as its
Division Marketing Director. She was thereafter re-hired as Senior
Assistance Vice President for a period of 5 years. Just after 2 years,
petitioner stopped reporting to work. In the same year, she became the
Vice President for Sales of Professional Pensions Plan, a corporation
also engaged in the pre-need industry. Respondent filed a complaint
for damages. Respondent argued that petitioner violated the non-
involvement clause in her contract, which provides that in case of
separation, she will not be involved with any corporation engaged in
the same business for the next 2 years. On the other hand, petitioner
argued that the non-involvement clause is offensive to public policy.

The trial court ruled that a contract in restraint of trade is valid


provided that there is a limitation upon either time or place. In the case
of the pre-need industry, the trial court found that the 2-year restriction
was valid and reasonable, hence this petition.

ISSUE:
Whether the non-involvement clause is valid

RULING:
Yes, non-involvement clause is valid.

A non-involvement clause is not necessarily void for being in


restraint of trade as long as there are reasonable limitations as to time,
trade, and place. In any event, Article 1306 of the Civil Code provides
that parties to a contract may establish such stipulations, clauses,
terms and conditions as they may deem convenient, provided they are
not contrary to law, morals, good customs, public order, or public
policy.

767
Page 768

In this case, the non-involvement clause has a time limit: two


years from the time petitioner’s employment with respondent ends. It
is also limited as to trade, since it only prohibits petitioner from
engaging in any pre-need business akin to respondent’s.1awphi1.net
More significantly, since petitioner was the Senior Assistant Vice-
President and Territorial Operations Head in charge of respondent’s
Hongkong and Asean operations, she had been privy to confidential
and highly sensitive marketing strategies of respondent’s business. To
allow her to engage in a rival business soon after she leaves would
make respondent’s trade secrets vulnerable especially in a highly
competitive marketing environment. In sum, we find the non-
involvement clause not contrary to public welfare and not greater than
is necessary to afford a fair and reasonable protection to respondent.

768
Page 769

Case Digests by: MIKHAILA KLAUDINE A. ROSALES

THE PHILIPPINE GEOTHERMAL, INC. EMPLOYEES


UNION v. UNOCAL PHILIPPINES, INC.
G.R. No. 190187. September 28, 2016.
Leonen, J.

DOCTRINE:
EFFECT OF MERGER
The merger of a corporation with another does not operate to
dismiss the employees of the corporation absorbed by the surviving
corporation. The employment of the absorbed employees subsists.

FACTS:
Petitioner is the bargaining agent of the rank-and-file employees
of respondent. Respondent is a foreign corporation incorporated under
the laws of the State of California. Respondent executed a merger
agreement with Chevron Texaco Corporation and Blue Merger. Under
the agreement, Blue Merger would become the surviving corporation.
Chevron then became the parent corporation of the merged
corporations. Thereafter, Blue Merger changed its name to Unocal
Corporation. Unocal executed a CBA with petitioner. Petitioner wrote
respondent asking for the separation benefits provided under the CBA.
Respondent denied their request and asserted that the merger did not
result in the cessation of its operations. The dispute was submitted for
voluntary arbitration.

The SOLE ruled in favor of petitioner and held that the union
members were impliedly terminated from employment as a result of
the merger. On appeal, the CA reversed the SOLE decision, and held
that the merger did not affect the employees.

ISSUE:
Whether the merger resulted in the implied dismissal of the
employees

RULING:
No, the merger did not result in the implied dismissal of the
employees.

There is no implied dismissal of its employees as a consequence


of the merger. A merger is a consolidation of two or more corporations,
which results in one or more corporations being absorbed into one
surviving corporation. The separate existence of the absorbed
corporation ceases, and the surviving corporation retains its identity

769
Page 770

and takes over the rights, privileges, franchises, properties, claims,


liabilities and obligations of the absorbed corporations. If respondent is
a subsidiary of Unocal California, which, in turn, is a subsidiary of
Unocal Corporation, then the merger of Unocal Corporation with Blue
Merger and Chevron does not affect respondent or any of its
employees. Respondent has a separate and distinct personality from
its parent corporation. Nonetheless, if respondent is indeed a party to
the merger, the merger still does not result in the dismissal of its
employees. The merger of Unocal Corporation with Blue Merger and
Chevron does not result in an implied termination of the employment
of petitioner's members. Assuming respondent is a party to the merger,
its employment contracts are deemed to subsist and continue by "the
combined operation of the Corporation Code and the Labor Code
under the backdrop of the labor and social justice provisions of the
Constitution.

770
Page 771

Case Digests by: MIKHAILA KLAUDINE A. ROSALES

SME BANK INC. v. DE GUZMAN


G.R. No. 184517. October 8, 2013.
Sereno, C. J.

DOCTRINE:
EFFECT OF CHANGE IN SHAREHOLDERS
There was no transfer of the business establishment to speak of,
but merely a change in the new majority shareholders of the
corporation. Following the rule in stock sales, employees may not be
dismissed except for just or authorized causes under the Labor Code.

FACTS:
Respondent employees were employees of SME Bank.
Originally, the principal shareholders and corporate directors of the
bank were Agustin and De Guzman. SME Bank experienced financial
difficulties. To remedy the situation, the bank officials proposed its sale
to Samson. Letter Agreements were sent to Agustin and De Guzman.
Espiritu, then the general manager of SME Bank, held a meeting with
all the employees and persuaded them to tender their resignations,
with the promise that they would be rehired upon reapplication. Relying
on these representations, some tendered their resignations. As it
turned out, respondent employees, except for Simeon, Jr., were not
rehired. Respondent-employees demanded the payment of their
respective separation pays, but their requests were denied. Aggrieved
by the loss of their jobs, respondent employees filed a Complaint
before NLRC.

The LA ruled that the buyer of an enterprise is not bound to


absorb its employees, unless there is an express stipulation to the
contrary. However, he also found that respondent employees were
illegally dismissed, because they had involuntarily executed their
resignation letters after relying on representations that they would be
given their separation benefits and rehired by the new management.
The NLRC affirmed the decision of the LA. The CA also affirmed the
decision of the NLRC, hence this petition.

ISSUE:
Whether the new majority shareholders are entitled to dismiss
the employees

RULING:
No, they are not entitled to dismiss the employees absent a just
or authorized cause.

771
Page 772

Contrary to petitioner banks argument, there was no transfer of


the business establishment to speak of, but merely a change in the
new majority shareholders of the corporation. There are two types of
corporate acquisitions : asset sales and stock sales. In asset sales, the
corporate entity sells all or substantially all of its assets to another
entity. In stock sales, the individual or corporate shareholders sell a
controlling block of stock to new or existing shareholders. In contrast
with asset sales, in which the assets of the selling corporation are
transferred to another entity, the transaction in stock sales takes place
at the shareholder level. Because the corporation possesses a
personality separate and distinct from that of its shareholders, a shift
in the composition of its shareholders will not affect its existence and
continuity. Thus, notwithstanding the stock sale, the corporation
continues to be the employer of its people and continues to be liable
for the payment of their just claims. Furthermore, the corporation or its
new majority shareholders are not entitled to lawfully dismiss corporate
employees absent a just or authorized cause.

In this case, what is involved is a stock sale. Following the rule


in stock sales, respondent employees may not be dismissed except for
just or authorized causes under the Labor Code. It is thus erroneous
on the part of the corporation to consider the employees as terminated
from their employment when the sole reason for so doing is a change
of management by reason of the stock sale.

772
Page 773

Case Digests by: MIKHAILA KLAUDINE A. ROSALES

TAMSON’S ENTERPRISES, INC. v. CA


G.R. No. 192881. November 16, 2011.
Mendoza, J.

DOCTRINE:
PROBATIONARY EMPLOYMENT
In all cases of probationary employment, the employer shall
make known to the employee the standards under which he will qualify
as a regular employee at the time of his engagement. Where no
standards are made known to the employee at that time, he shall be
deemed a regular employee.

FACTS:
Sy was hired by Tamsons as Assistant to the President. Four
days before she completed her 6th month of working in Tamsons, Sy
was informed that her services would be terminated due to inefficiency.
She was asked to sign a letter of resignation and quitclaim. She was
told not to report for work anymore because her services were no
longer needed. Petitioners asserted that before Sy was hired, she was
apprised that she was being hired as a probationary employee for six
months, subject to extension as a regular employee conditioned on her
meeting the standards of permanent employment set by the company.
Her work performance was thereafter monitored and evaluated. She
was later formally informed that her employment would end because
she failed to meet the company's standards. Sy to file a case for illegal
dismissal.

The LA ruled that Sy was illegally dismissed, and held that a


termination, notwithstanding the probationary status, must be for a just
cause. The NLRC reversed the LA decision. The CA reinstated the LA
decision, finding that the termination of Sy’s services was illegal, hence
this petition.

ISSUE:
Whether the dismissal of Sy, a probationary employee, was legal

RULING:
No, the dismissal of Sy was illegal.

Even if probationary employees do not enjoy permanent status,


they are accorded the constitutional protection of security of tenure.
They may only be terminated for a just cause or when they otherwise
fail to qualify as regular employees in accordance with reasonable

773
Page 774

standards made known to them by the employer at the time of their


engagement. One of the conditions before an employer can terminate
a probationary employee is dissatisfaction on the part of the employer
which must be real and in good faith, not feigned so as to circumvent
the contract or the law.

In this case, absent any proof showing that the work performance
of petitioner was unsatisfactory, We cannot conclude that petitioner
failed to meet the standards. This absence of proof leads Us to infer
that their dissatisfaction with her work performance was contrived so
as not to regularize her employment. he petitioners failed to convey to
Sy the standards upon which she should measure up to be considered
for regularization and how the standards had been applied in her case.
Petitioners dissatisfaction was at best self-serving and dubious as they
could not present concrete and competent evidence establishing her
alleged incompetence. Failure on the part of the petitioners to
discharge the burden of proof is indicative that the dismissal was not
justified. The standards under which she would qualify as a regular
employee not having been communicated to her at the start of her
probationary period, Sy qualified as a regular employee.

774
Page 775

Case Digests by: MIKHAILA KLAUDINE A. ROSALES

HACIENDA PRIMERA DEVELOPMENT CORPORATION v.


VILLEGAS
G.R. No. 186243. April 11, 2011.
Nachura, J.

DOCTRINE:
PROBATIONARY EMPLOYMENT
In all cases of probationary employment, the employer shall
make known to the employee the standards under which he will qualify
as a regular employee at the time of his engagement. Where no
standards are made known to the employee at that time, he shall be
deemed a regular employee. Failure to make known the standard to
be met by the probationary employee shall make such an employee a
regular employee from the day of hiring.

FACTS:
Petitioner Hacienda hired respondent Michael S. Villegas as
General Manager of Amorita Resort. He was hired as a probationary
employee for 3 months. Respondent started working on Jan. 1, 2007,
but on Mar. 4, 2007, he received a call from Paramount Consultancy
and Management telling him to report back to Manila. He was informed
that his services had been terminated. He requested a written notice
of termination, but none was given, prompting Villegas to file an action
for Illegal Dismissal before the Labor Arbiter. Petitioner contended that
respondent's services were terminated because he failed to qualify for
regular employment. Specifically, it claimed that respondent failed to
conceptualize and complete financial budgets, sales projection, room
rates, website development, and marketing plan in coordination with
the Sales and Marketing Manager.

The LA held that the dismissal was illegal, and ordered the
reinstatement of Villegas. The NLRC reversed the LA decision. The
CA reinstated the ruling of the LA, but without an order for
reinstatement because of the strained relations between the parties,
hence this petition.

ISSUE:
Whether respondent Villegas was validly dismissed

RULING:
No, respondent Villegas was not validly dismissed for failure of
petitioner to specify the reasonable standards by which respondent’s
performance was evaluated.

775
Page 776

The dismissal of a probationary employee presupposes that the


standards to be met were made known to the employee. There are two
grounds to legally terminate a probationary employee. It may be done
either: a) for a just cause; or b) when the employee fails to qualify as a
regular employee in accordance with reasonable standards made
known by the employer to the employee at the start of the employment.

In this case, petitioner Hacienda fails to specify the reasonable


standards by which respondent's alleged poor performance was
evaluated, much less to prove that such standards were made known
to him at the start of his employment. Thus, he is deemed to have been
hired from day one as a regular employee.

776
Page 777

Case Digests by: LEX ANGELO A. ROSARIO

UNIVERSIDAD DE STA. ISABEL vs. SAMBAJON, JR.


G.R. Nos. 196280 & 196286. April 2, 2014
Illarama, Jr., J

DOCTRINE:
PROBATIONARY EMPLOYMENT OF TEACHERS
IN PRIVATE SCHOOLS
The probationary employment of teachers in private schools is
not governed purely by the Labor Code. The Labor Code is
supplemented with respect to the period of probation by special rules
found in the Manual of Regulations for Private Schools. It is the Manual
of Regulations for Private Schools, and not the Labor Code, that
determines whether or not a faculty member in an educational
institution has attained regular or permanent status

FACTS:
Universidad de Sta. Isabel (petitioner) is a non-stock, non-profit
religious educational institution in Naga City. Petitioner hired Marvin-
Julian L. Sambajon, Jr. (respondent) as a full-time college faculty
member with the rank of Assistant Professor on probationary status,
as evidenced by an Appointment Contract dated November 1, 2002,
effective November 1, 2002 up to March 30, 2003.
After the aforesaid contract expired, petitioner continued to give
teaching loads to respondent who remained a full-time faculty member
of the Department of Religious Education for the two semesters of
school-year (SY) 2003-2004 (June 1, 2003 to March 31, 2004); and
two semesters of SY 2004-2005 (June 2004 to March 31, 2005).
Sometime in June 2003, after respondent completed his master’s
degree, respondent's salary was increased, as reflected in his pay slips
starting October 1-15, 2004. He was likewise re-ranked from Assistant
Professor to Associate Professor.
In a letter dated October 15, 2004 addressed to the President of
petitioner, Sr. Ma. Asuncion G. Evidente, D.C., respondent vigorously
argued that his salary increase should be made effective as of June
2003 and demanded the payment of his salary differential. The school
administration thru Sr. Purita Gatongay, D.C., replied by explaining its
policy on re-ranking of faculty members that that teachers in the
Universidad are not re-ranked during their probationary period. The
Faculty Manual as revised for school year 2002-2003 provides (page
38) "Re-ranking is done every two years, hence the personnel hold
their present rank for two years. Those undergoing probationary period
and those on part-time basis of employment are not covered by this
provision."

777
Page 778

On February 26, 2005, respondent’s services was terminated. On


April 14, 2005, respondent filed a complaint for illegal dismissal against
the petitioner.
In his Decision dated August 22, 2006, Labor Arbiter Jesus
Orlando M. Quinones ruled that there was no just or authorized cause
in the termination of respondent's probationary employment.
Consequently, petitioner was found liable for illegal dismissal.
On appeal to the NLRC, NLRC rendered its Decision affirming the
Labor Arbiter and holding that respondent had acquired a permanent
status. CA likewise sustained the conclusion of the NLRC that
respondent had already acquired permanent status when he was
allowed to continue teaching after the expiration of his first
appointment-contract on March 30, 2003.

ISSUE:
(1) Whether respondent Sambajon, Jr. has acquired a permanent
status when he was allowed to continue teaching after the
expiration of his first appointment-contract on March 30, 2003
(2) Whether respondent was illegally dismissed

RULING:
(1) NO. He is still a probationary employee.
The probationary employment of teachers in private schools is not
governed purely by the Labor Code (Article 296, LC). The Labor Code
is supplemented with respect to the period of probation by special rules
found in the Manual of Regulations for Private Schools. On the matter
of probationary period, Section 92 of the 1992 Manual of Regulations
for Private Schools regulations states:
Section 92. Probationary Period. — Subject in all instances
to compliance with the Department and school requirements,
the probationary period for academic personnel shall not
be more than three (3) consecutive years of satisfactory
service for those in the elementary and secondary levels, six
(6) consecutive regular semesters of satisfactory service
for those in the tertiary level, and nine (9) consecutive
trimesters of satisfactory service for those in the tertiary level
where collegiate courses are offered on a trimester basis.
(Emphasis supplied.)
Thus, it is the Manual of Regulations for Private Schools, and not
the Labor Code, that determines whether or not a faculty member in
an educational institution has attained regular or permanent status.
Section 93 of the 1992 Manual of Regulations for Private Schools
provides that full-time teachers who have satisfactorily completed their
probationary period shall be considered regular or permanent.
In this case, the CA sustained the NLRC's ruling that respondent
was illegally dismissed considering that he had become a regular
employee when petitioner allowed him to work beyond the date
specified in his first probationary appointment contract which expired
on March 30, 2003. According to the CA:

778
Page 779

. . . As can be gleaned from Section 92 of the 1992 Manual of


Regulations for Private Schools, the probationary period applicable in
this case is not more than six (6) consecutive regular semesters of
satisfactory service. In other words, the probationary period for
academic personnel in the tertiary level runs from one (1) semester to
six (6) consecutive regular semesters of satisfactory service. In the
instant case, records reveal that Sambajon, Jr. only signed two
appointment contracts. The first appointment-contract which he signed
was dated November 2002 for the period November 1, 2002 to March
30, 2003, as Assistant Professor 10 on probationary status. . . . The
second appointment-contract which Sambajon, Jr. executed was
dated February 26, 2004, for the period November 1, 2003 to March
31, 2004. . . . Compared with the first appointment-contract, it was not
indicated in the February 26, 2004 appointment-contract that
Sambajon, Jr. was hired on probationary status, which explains the
NLRC's conclusion that Sambajon, Jr. already attained permanent
status. At this juncture, it is worthy to emphasize that other than the
period provided under Article 281 of the Labor Code, the following
quoted portion of Article 281 of the Labor Code still applies:
"ART. 281. PROBATIONARY EMPLOYMENT. — . . . An
employee who is allowed to work after a probationary period shall be
considered a regular employee."

SC disagrees.

The third appointment contract dated February 26, 2004


reads:

February 26, 2004


MR. MARVIN JULIAN SAMBAJON
Religious Education Department
Dear Mr. Sambajon,
I am pleased to inform you that you are designated and
commissioned to be an Apostle of Love and Service, Unity and
Peace as you dedicate and commit yourself in the exercise of
your duties and responsibilities as a:
FULL TIME FACULTY MEMBER
of the Religious Education Department from November 1,
2003 to March 31, 2004. HTDAac
Unless otherwise renewed in writing, this
designation automatically terminates as of the date
expiration above states without further notice.

Since it was explicitly provided in the above contract that unless


renewed in writing respondent's appointment automatically expires at
the end of the stipulated period of employment, the CA erred in
concluding that simply because the word "probationary" no longer
appears below the designation (Full-Time Faculty Member),
respondent had already become a permanent employee.

779
Page 780

(2) YES. He was illegally dismissed.

Notwithstanding the limited engagement of probationary


employees, they are entitled to constitutional protection of security of
tenure during and before the end of the probationary period. The
services of an employee who has been engaged on probationary basis
may be terminated for any of the following: (a) a just or (b) an
authorized cause; and (c) when he fails to qualify as a regular
employee in accordance with reasonable standards prescribed by the
employer.
In a letter dated February 26, 2005, petitioner terminated the
services of respondent stating that his probationary employment as
teacher will no longer be renewed upon its expiry on March 31, 2005,
respondent's fifth semester of teaching. No just or authorized cause
was given by petitioner. Prior to this, respondent had consistently
achieved above average rating based on evaluation by petitioner's
officials and students. He had also been promoted to the rank of
Associate Professor after finishing his master's degree course on his
third semester of teaching. Clearly, respondent's termination after five
semesters of satisfactory service was illegal.
Respondent therefore is entitled to continue his three-year
probationary period, such that from March 31, 2005, his probationary
employment is deemed renewed for the following semester (1st
semester of SY 2005-2006). However, given the discordant relations
that had arisen from the parties' dispute, it can be inferred with certainty
that petitioner had opted not to retain respondent in its employ beyond
the three-year period. Therefore, the award of backwages as a
consequence of the finding of illegal dismissal in favor of respondent
should be confined to the three-year probationary period.

780
Page 781

Case Digests by: LEX ANGELO A. ROSARIO

UNIVAC DEVELOPMENT, INC. vs. SORIANO


G.R. No. 182072. June 19, 2013
Peralta, J.

DOCTRINE:
TERMINATION OF PROBATIONARY EMPLOYEE’S SERVICES
A probationary employee can only be dismissed from employment for
a just cause or when he fails to qualify as a regular employee in
accordance with reasonable standards made known to him by the
employer at the time of his engagement.

FACTS:
The case stemmed from the Complaint for Illegal Dismissal filed
by respondent against petitioner, the company's Chairperson Sadamu
Watanabe (Watanabe), and the Head of the Engineering Department
Johnny Castro (Castro). Admittedly, respondent was hired on August
23, 2004 by petitioner on probationary basis as legal assistant of the
company with a monthly salary of P15,000.00. Respondent claimed
that on February 15, 2005, or eight (8) days prior to the completion of
his six months probationary period, Castro allegedly informed him that
he was being terminated from employment due to the company's cost-
cutting measures. He allegedly asked for a thirty-day notice but his
termination was ordered to be effective immediately. Thus, he was left
with no choice but to leave the company.
Petitioner, on the other hand, denied the allegations of respondent
and claimed instead that prior to his employment, respondent was
informed of the standards required for regularization. Petitioner also
supposedly informed him of his duties and obligations which included
safekeeping of case folders, proper coordination with the company's
lawyers, and monitoring of the status of the cases filed by or against
the company. Petitioner recalled that on January 5, 2005, a company
meeting was held where respondent allegedly expressed his intention
to leave the company because he wanted to review for the bar
examinations. It was also in that meeting where he was informed of his
unsatisfactory performance in the company. Thus, when respondent
did not report for work on February 16, 2005, petitioner assumed that
he pushed through with his plan to leave the company. In other words,
petitioner claimed that respondent was not illegally dismissed from
employment, rather, he in fact abandoned his job by his failure to report
for work.
LA dismissed respondent's complaint for lack of merit. The LA
held that respondent was informed of his unsatisfactory performance.
As a law graduate and a master's degree holder, respondent was
presumed to know that his probationary employment would soon end.

781
Page 782

On appeal, the NLRC affirmed the LA decision in its entirety. On appeal


to the CA, CA reversed NLRC and LA’s ruling and held that respondent
Soriano to have been illegally dismissed from work.

ISSUE:
Whether respondent Soriano has been illegally dismissed from work

RULING:
Yes. Respondent Soriano was illegally dismissed.

Article 281 (now Article 296) of the Labor Code and its
Implementing Rules describe probationary employment and set the
guidelines to be followed by the employer and employee.
It is undisputed that respondent was hired as a probationary
employee. As such, he did not enjoy a permanent status.
Nevertheless, he is accorded the constitutional protection of security
of tenure which means that he can only be dismissed from employment
for a just cause or when he fails to qualify as a regular employee in
accordance with reasonable standards made known to him by the
employer at the time of his engagement.
It is primordial that at the start of the probationary period, the
standards for regularization be made known to the probationary
employee. In this case, as held by the CA, petitioner failed to present
adequate evidence to substantiate its claim that respondent was
apprised of said standards. It is evident from the LA and NLRC
decisions that they merely relied on surmises and presumptions in
concluding that respondent should have known the standards
considering his educational background as a law graduate. Equally
important is the requirement that in order to invoke "failure to meet the
probationary standards" as a justification for dismissal, the employer
must show how these standards have been applied to the subject
employee. In this case, aside from its bare allegation, it was not shown
that a performance evaluation was conducted to prove that his
performance was indeed unsatisfactory.
Indeed, the power of the employer to terminate a probationary
employee is subject to three limitations, namely: (1) it must be
exercised in accordance with the specific requirements of the contract;
(2) the dissatisfaction on the part of the employer must be real and in
good faith, not feigned so as to circumvent the contract or the law; and
(3) there must be no unlawful discrimination in the dismissal. In this
case, not only did petitioner fail to show that respondent was apprised
of the standards for regularization but it was likewise not shown how
these standards had been applied in his case.
Pursuant to well-settled doctrine, petitioner's failure to specify the
reasonable standards by which respondent's alleged poor
performance was evaluated as well as to prove that such standards
were made known to him at the start of his employment, makes

782
Page 783

respondent a regular employee. In other words, because of this


omission on the part of petitioner, respondent is deemed to have
been hired from day one as a regular employee.
To justify the dismissal of an employee, the employer must, as a
rule, prove that the dismissal was for a just cause and that the
employee was afforded due process prior to dismissal. We find no
reason to depart from the CA conclusion that respondent's termination
from employment is without just and valid ground. Neither was due
process observed, making his termination illegal. He is, therefore,
entitled to the twin relief of reinstatement and backwages granted
under the Labor Code.

783
Page 784

Case Digests by: LEX ANGELO A. ROSARIO

ABBOTT LABORATORIES, PHILIPPINES vs. ALCARAZ


G.R. No. 192571. July 23, 2013
Perlas-Bernabe, J.

DOCTRINE:
EMPLOYER’S LIABILITY TO NOMINAL DAMAGES
Case law has settled that an employer who terminates an employee
for a valid cause but does so through invalid procedure is liable to pay
the latter nominal damages.
FACTS:
On June 27, 2004, petitioner Abbott Laboratories, Philippines
(Abbott) caused the publication in a major broadsheet newspaper of its
need for a Medical and Regulatory Affairs Manager (Regulatory Affairs
Manager) who would: (a) be responsible for drug safety surveillance
operations, staffing, and budget; (b) lead the development and
implementation of standard operating procedures/policies for drug
safety surveillance and vigilance; and (c) act as the primary interface
with internal and external customers regarding safety operations and
queries. Alcaraz — who was then a Regulatory Affairs and Information
Manager at Aventis Pasteur Philippines, Incorporated (another
pharmaceutical company like Abbott) — showed interest and
submitted her application on October 4, 2004.
On February 12, 2005, Alcaraz signed an employment contract
which stated, inter alia, that she was to be placed on probation for a
period of six (6) months beginning February 15, 2005 to August 14,
2005.
During Alcaraz's pre-employment orientation, petitioner Allan G.
Almazar (Almazar), Hospira's Country Transition Manager, briefed her
on her duties and responsibilities as Regulatory Affairs Manager.
On March 3, 2005, petitioner Maria Olivia T. Yabut-Misa (Misa),
Abbott's Human Resources (HR) Director, sent Alcaraz an e-mail
which contained an explanation of the procedure for evaluating the
performance of probationary employees and further indicated that
Abbott had only one evaluation system for all of its employees. Alcaraz
was also given copies of Abbott's Code of Conduct and Probationary
Performance Standards and Evaluation (PPSE) and Performance
Excellence Orientation Modules (Performance Modules) which she
had to apply in line with her task of evaluating the Hospira ALSU staff.

784
Page 785

During the course of her employment, Alcaraz noticed that some of the
staff had disciplinary problems. Thus, she would reprimand them for
their unprofessional behavior such as non-observance of the dress
code, moonlighting, and disrespect of Abbott officers. However,
Alcaraz's method of management was considered by Walsh to be "too
strict."

On May 16, 2005, Alcaraz was called to a meeting with Walsh and
Terrible where she was informed that she failed to meet the
regularization standards for the position of Regulatory Affairs Manager.
Thereafter, Walsh and Terrible requested Alcaraz to tender her
resignation, else they be forced to terminate her services.

On May 23, 2005, Walsh, Almazar, and Bernardo personally handed


to Alcaraz a letter stating that her services had been terminated
effective May 19, 2005. On May 27, 2005, Alcaraz received another
copy of the said termination letter via registered mail

Alcaraz felt that she was unjustly terminated from her employment and
thus, filed a complaint for illegal dismissal and damages against Abbott
and its officers, namely, Misa, Bernardo, Almazar, Walsh, Terrible, and
Feist. She claimed that she should have already been considered as a
regular and not a probationary employee given Abbott's failure to
inform her of the reasonable standards for her regularization upon her
engagement as required under Article 295 of the Labor Code.

Labor Arbiter dismissed Alcaraz’s complaint for lack of merit. On


appeal by Alcaraz to the NLRC, the NLRC reversed the findings of the
LA and ruled that there was no evidence showing that Alcaraz had
been apprised of her probationary status and the requirements which
she should have complied with in order to be a regular employee. On
appeal by Abbott to CA, CA affirmed the ruling of the NRLC.

ISSUE:
(3) Whether Alcaraz’s services was legally terminated
(4) Whether the termination procedure for the services of
probationary employees was met in the case of Alcaraz

RULING:
(3) Yes. Alcaraz was not illegally dismissed.
The services of an employee who has been engaged on
probationary basis may be terminated for any of the following: (a) a just
or (b) an authorized cause; and (c) when he fails to qualify as a regular
employee in accordance with reasonable standards prescribed by the
employer.

785
Page 786

Corollary thereto, Section 6 (d), Rule I, Book VI of the Implementing


Rules of the Labor Code provides that if the employer fails to inform
the probationary employee of the reasonable standards upon which
the regularization would be based on at the time of the engagement,
then the said employee shall be deemed a regular employee.
In other words, the employer is made to comply with two (2)
requirements when dealing with a probationary employee: first, the
employer must communicate the regularization standards to the
probationary employee; and second, the employer must make such
communication at the time of the probationary employee's
engagement. If the employer fails to comply with either, the employee
is deemed as a regular and not a probationary employee.
The exception to the foregoing is when the job is self-descriptive
in nature, for instance, in the case of maids, cooks, drivers, or
messengers.
In this case, petitioners contend that Alcaraz was terminated
because she failed to qualify as a regular employee according to
Abbott's standards which were made known to her at the time of her
engagement. Contrarily, Alcaraz claims that Abbott never apprised her
of these standards and thus, maintains that she is a regular and not a
mere probationary employee.
The Court finds petitioners' assertions to be well-taken.
A punctilious examination of the records reveals that Abbott had
indeed complied with the above-stated requirements. This conclusion
is largely impelled by the fact that Abbott clearly conveyed to Alcaraz
her duties and responsibilities as Regulatory Affairs Manager prior to,
during the time of her engagement, and the incipient stages of her
employment.
In fine, the Court rules that Alcaraz's status as a probationary employee
and her consequent dismissal must stand. Consequently, in holding
that Alcaraz was illegally dismissed due to her status as a regular and not a
probationary employee, the Court finds that the NLRC committed a grave
abuse of discretion.

(4) YES. The termination procedure for services of probationary


employees was met in the case of Alcaraz.

A different procedure is applied when terminating a probationary


employee; the usual two-notice rule does not govern. Section 2, Rule
I, Book VI of the Implementing Rules of the Labor Code states that "[i]f
the termination is brought about by the . . . failure of an employee to
meet the standards of the employer in case of probationary
employment, it shall be sufficient that a written notice is served the
employee, within a reasonable time from the effective date of
termination."
As the records show, Alcaraz's dismissal was effected through a
letter dated May 19, 2005 which she received on May 23, 2005 and
again on May 27, 2005. Stated therein were the reasons for her

786
Page 787

termination, i.e., that after proper evaluation, Abbott determined that


she failed to meet the reasonable standards for her regularization
considering her lack of time and people management and decision-
making skills, which are necessary in the performance of her functions
as Regulatory Affairs Manager.

Nonetheless, despite the existence of a sufficient ground to


terminate Alcaraz's employment and Abbott's compliance with the
Labor Code termination procedure, it is readily apparent that Abbott
breached its contractual obligation to Alcaraz when it failed to abide by
its own procedure in evaluating the performance of a probationary
employee.
Veritably, a company policy partakes of the nature of an implied
contract between the employer and employee.
Hence, given such nature, company personnel policies create an
obligation on the part of both the employee and the employer to abide
by the same.
Records show that Abbott's PPSE procedure mandates, inter alia,
that the job performance of a probationary employee should be
formally reviewed and discussed with the employee at least twice: first
on the third month and second on the fifth month from the date of
employment. Abbott is also required to come up with a Performance
Improvement Plan during the third month review to bridge the gap
between the employee's performance and the standards set, if any. In
addition, a signed copy of the PPSE form should be submitted to
Abbott's HRD as the same would serve as basis for recommending the
confirmation or termination of the probationary employment.
In this case, it is apparent that Abbott failed to follow the above-
stated procedure in evaluating Alcaraz. For one, there lies a hiatus of
evidence that a signed copy of Alcaraz's PPSE form was submitted to
the HRD. It was not even shown that a PPSE form was completed to
formally assess her performance. Neither was the performance
evaluation discussed with her during the third and fifth months of her
employment. Nor did Abbott come up with the necessary Performance
Improvement Plan to properly gauge Alcaraz's performance with the
set company standards.
In this light, while there lies due cause to terminate Alcaraz's
probationary employment for her failure to meet the standards required
for her regularization, and while it must be further pointed out that
Abbott had satisfied its statutory duty to serve a written notice of
termination, the fact that it violated its own company procedure renders
the termination of Alcaraz's employment procedurally infirm,
warranting the payment of nominal damages. A further exposition is
apropos.
Case law has settled that an employer who terminates an
employee for a valid cause but does so through invalid procedure is
liable to pay the latter nominal damages.

787
Page 788

Case Digests by: LEX ANGELO A. ROSARIO

COLEGIO DEL SANTISIMO ROSARIO V. ROJO


G.R. 170388. September 4, 2013
Del Castillo, J.

DOCTRINE:
PROBATIONARY EMPLOYMENT OF TEACHERS
IN PRIVATE SCHOOLS
Cases dealing with employment on probationary status of teaching
personnel are not governed solely by the Labor Code as the law is
supplemented, with respect to the period of probation, by special rules
found in the Manual of Regulations for Private Schools (the Manual).
FACTS:
Petitioner Colegio del Santisimo Rosario (CSR) hired respondent
as a high school teacher on probationary basis for the school years
1992-1993, 1993-1994 and 1994-1995.
On April 5, 1995, CSR, through petitioner Sr. Zenaida S. Mofada,
OP (Mofada), decided not to renew respondent's services.
Thus, on July 13, 1995, respondent filed a Complaint for illegal
dismissal. He alleged that since he had served three consecutive
school years which is the maximum number of terms allowed for
probationary employment, he should be extended permanent
employment. Citing paragraph 75 of the 1970 Manual of Regulations
for Private Schools (1970 Manual), respondent asserted that "full-time
teachers who have rendered three (3) consecutive years of satisfactory
services shall be considered permanent."
On the other hand, petitioners argued that respondent knew that
his Teacher's Contract for school year 1994-1995 with CSR would
expire on March 31, 1995. Accordingly, respondent was not dismissed
but his probationary contract merely expired and was not renewed.
Petitioners also claimed that the "three years" mentioned in paragraph
75 of the 1970 Manual refer to "36 months," not three school years.
And since respondent served for only three school years of 10 months
each or 30 months, then he had not yet served the "three years" or 36
months mentioned in paragraph 75 of the 1970 Manual.
The LA ruled that "three school years" means three years of 10
months, not 12 months. Considering that respondent had already
served for three consecutive school years, then he has already
attained regular employment status. Thus, the non-renewal of his
contract for school year 1995-1996 constitutes illegal dismissal. On
appeal by petitioner to NLRC, it held that after serving three school
years, respondent had attained the status of regular employment
especially because CSR did not make known to respondent the
reasonable standards he should meet. On appeal to the CA, CA ruled
that respondent has attained the status of a regular employee after he

788
Page 789

was employed for three consecutive school years as a full-time teacher


and had served CSR satisfactorily

ISSUE:
Whether respondent is considered as regular employee of petitioner

RULING:
Yes.

In Mercado v. AMA Computer College-Parañaque City, Inc., we


had occasion to rule that cases dealing with employment on
probationary status of teaching personnel are not governed solely by
the Labor Code as the law is supplemented, with respect to the period
of probation, by special rules found in the Manual of Regulations for
Private Schools (the Manual). With regard to the probationary period,
Section 92 of the 1992 Manual provides:
Section 92. Probationary Period. — Subject in all instances to
compliance with the Department and school requirements, the
probationary period for academic personnel shall not be more
than three (3) consecutive years of satisfactory service for those
in the elementary and secondary levels, six (6) consecutive regular
semesters of satisfactory service for those in the tertiary level, and nine
(9) consecutive trimesters of satisfactory service for those in the
tertiary level where collegiate courses are offered on a trimester basis.

In this case, petitioners' teachers who were on probationary


employment were made to enter into a contract effective for one school
year. Thereafter, it may be renewed for another school year, and the
probationary employment continues. At the end of the second fixed
period of probationary employment, the contract may again be
renewed for the last time.
Such employment for fixed terms during the teachers'
probationary period is an accepted practice in the teaching profession.
In Magis Young Achievers' Learning Center v. Manalo, we noted that:
The common practice is for the employer and the teacher to enter into
a contract, effective for one school year.
xxx xxx xxx
At the end of this third year, the employer may now decide whether to
extend a permanent appointment to the employee, primarily on the
basis of the employee having met the reasonable standards of
competence and efficiency set by the employer. For the entire duration
of this three-year period, the teacher remains under probation.
However, this scheme "of fixed-term contract is a system that operates
during the probationary period and for this reason is subject to Article
281 (now 296) of the Labor Code," which provides:
. . . The services of an employee who has been engaged on a
probationary basis may be terminated for a just cause or when he
fails to qualify as a regular employee in accordance with
reasonable standards made known by the employer to the
employee at the time of his engagement. An employee who is

789
Page 790

allowed to work after a probationary period shall be considered a


regular employee.
In Mercado, we held that "[u]nless this reconciliation is made, the
requirements of [Article 281] on probationary status would be fully
negated as the school may freely choose not to renew contracts simply
because their terms have expired." This will have an unsettling effect
in the equilibrium vis-a-vis the relations between labor and
management that the Constitution and Labor Code have worked hard
to establish.
That teachers on probationary employment also enjoy the
protection afforded by Article 281 of the Labor Code is supported by
Section 93 of the 1992 Manual which provides:
Sec. 93. Regular or Permanent Status. — Those who have served
the probationary period shall be made regular or permanent. Full-
time teachers who have satisfactorily completed their
probationary period shall be considered regular or permanent.
(Emphasis supplied)
The above provision clearly provides that full-time teachers
become regular or permanent employees once they have satisfactorily
completed the probationary period of three school years. The use of
the term satisfactorily necessarily connotes the requirement for
schools to set reasonable standards to be followed by teachers on
probationary employment. For how else can one determine if
probationary teachers have satisfactorily completed the probationary
period if standards therefor are not provided?
As such, "no vested right to a permanent appointment shall accrue
until the employee has completed the prerequisite three-year period
necessary for the acquisition of a permanent status. [However, it must
be emphasized that] mere rendition of service for three consecutive
years does not automatically ripen into a permanent appointment. It is
also necessary that the employee be a full-time teacher, and that the
services he rendered are satisfactory."

790
Page 791

Case Digests by: LEX ANGELO A. ROSARIO

PHILIPPINE DAILY INQUIRER, INC. vs. MAGTIBAY, JR


G.R. No. 164532. July 27, 2007
Garcia, J.

DOCTRINE:
TERMINATION OF PROBATIONARY EMPLOYEE’S SERVICES
The services of an employee who has been engaged on a probationary
basis may be terminated for a just cause or when he fails to qualify as
a regular employee in accordance with reasonable standards made
known by the employer to the employee at the time of his engagement.
An employee who is allowed to work after a probationary period shall
be considered a regular employee.

FACTS:
Leonardo Magtibay was hired on a contractual basis by the
Philippine Daily Inquirer for a period of 5 months. Before the expiration
of the said contract, they agreed to extend it for another 15 days. After
Magtibay’s contractual employment expired, PDI then announced that
they created an available position for a second telephone operator,
who will undergo a probationary employment. Since it was a practice
of PDI to give preference to its regular employees for vacancies, Ms.
Regina Layague, a regular employee and a member of the Philippine
Daily Inquirer Employees Union (PDIEU), applied for the said available
position but later withdrew his application prompting Magtibay to apply
for the same.
PDI then hired Magtibay on a probationary basis for a period of 6
months. A week prior to the expiration of the 6 month probationary
period, Magtibay was handed his termination paper for alleged failure
to meet company standards.
For such, he filed a complaint for illegal dismissal and damages
before the Labor Arbiter. PDIEU later joined the case by filing a
supplemental complaint for unfair labor practice.
The Labor Arbiter ruled in favor of PDI. The Labor Arbiter, in ruling
in favor of PDI relied on the abstract language provided for in the
termination paper which stated that: “you did not meet the standards
of the company”, to wit: (1) he repeatedly violated the company rule
prohibiting unauthorized persons from entering the telephone
operators room; (2) he intentionally omitted to indicate in his
application form his having a dependent child; and (3) he exhibited lack
of sense of responsibility by locking the door of the telephone operators
room on March 10, 1996 without switching the proper lines to the
company guards so that incoming calls may be answered by them.
However, on appeal to the NLRC, it ruled in favor of Magtibay which
prompted PDI to file with it a motion for reconsideration which NLRC

791
Page 792

denied. PDI then went to CA via a petition for certiorari. The CA denied
due course on PDI’s petition. Hence this appeal.

ISSUE:
Whether respondent Magtibay was illegally dismissed

RULING:
No.

Within the limited legal six-month probationary period,


probationary employees are still entitled to security of tenure. It is
expressly provided in the afore-quoted Article 281 (now 279, Labor
Code) that a probationary employee may be terminated only on two
grounds: (a) for just cause, or (b) when he fails to qualify as a regular
employee in accordance with reasonable standards made known by
the employer to the employee at the time of his engagement.
PDI invokes the second ground under the premises. In claiming
that it had adequately apprised Magtibay of the reasonable standards
against which his performance will be gauged for purposes of
permanent employment, PDI cited the one-on-one seminar between
Magtibay and its Personnel Assistant, Ms. Rachel Isip-Cuzio. PDI also
pointed to Magtibay's direct superior, Benita del Rosario, who diligently
briefed him about his responsibilities in PDI. These factual assertions
were never denied nor controverted by Magtibay. Neither did he belie
the existence of a specific rule prohibiting unauthorized persons from
entering the telephone operator's booth and that he violated that
prohibition. This notwithstanding, the NLRC and the CA proceeded
nonetheless to rule that the records of the case are bereft of any
evidence showing that these rules and regulations form part of the so-
called company standards.
SC did not agree with the appellate court when it cleared the
NLRC of commission of grave abuse of discretion despite the latter's
disregard of clear and convincing evidence that there were reasonable
standards made known by PDI to Magtibay during his probationary
employment. It is on record that Magtibay committed obstinate
infractions of company rules and regulations, which in turn constitute
sufficient manifestations of his inadequacy to meet reasonable
employment norms. The suggestion that Magtibay ought to have been
made to understand during his briefing and orientation that he is
expected to obey and comply with company rules and regulations
strains credulity for acceptance. The CA's observation that "nowhere
can it be found in the list of Basic Responsibility and Specific Duties
and Responsibilities of respondent Magtibay that he has to abide by
the duties, rules and regulations that he has allegedly violated" is a
strained rationalization of an unacceptable conduct of an employee.
Common industry practice and ordinary human experience do not
support the CA's posture. All employees, be they regular or

792
Page 793

probationary, are expected to comply with company-imposed rules and


regulations, else why establish them in the first place. Probationary
employees unwilling to abide by such rules have no right to expect,
much less demand, permanent employment. We, therefore find
sufficient factual and legal basis, duly established by substantial
evidence, for PDI to legally terminate Magtibay's probationary
employment effective upon the end of the 6-month probationary period.
It is undisputed that PDI apprised Magtibay of the ground of his
termination, i.e., he failed to qualify as a regular employee in
accordance with reasonable standards made known to him at the time
of engagement, only a week before the expiration of the six-month
probationary period. Given this perspective, does this make his
termination unlawful for being violative of his right to due process of
law?
It does not.
Unlike under the first ground for the valid termination of
probationary employment which is for just cause, the second ground
does not require notice and hearing. Due process of law for this second
ground consists of making the reasonable standards expected of the
employee during his probationary period known to him at the time of
his probationary employment. By the very nature of a probationary
employment, the employee knows from the very start that he will be
under close observation and his performance of his assigned duties
and functions would be under continuous scrutiny by his superiors. It
is in apprising him of the standards against which his performance shall
be continuously assessed where due process regarding the second
ground lies, and not in notice and hearing as in the case of the first
ground.

793
Page 794

Case Digests by: LEX ANGELO A. ROSARIO

ALCIRA VS. NATIONAL LABOR RELATIONS


COMMISSION
G.R. No. 149859. June 9, 2004
Corona, J.

DOCTRINE:
(1) The computation of the 6-month probationary period is reckoned
from the date of appointment up to the same calendar date of the
6th month following.
(2) An employer is deemed to substantially comply with the rule on
notification of standards if he apprises the employee that he will
be subjected to a performance evaluation on a particular date
after his hiring.
(3) The limited security of tenure accorded to probationary
employees ends on the date of the expiration of the probationary
period.

FACTS:
Respondent Middleby Philippines Corporation (Middleby) hired
petitioner as engineering support services supervisor on a
probationary basis for six months. Apparently unhappy with petitioner's
performance, respondent Middleby terminated petitioner's services.
The bone of contention centered on whether the termination occurred
before or after the six-month probationary period of employment.
The parties, presenting their respective copies of Alcira's
appointment paper, claimed conflicting starting dates of employment:
May 20, 1996 according to petitioner and May 27, 1996 according to
respondent. Both documents indicated petitioner's employment status
as "probationary (6 mos.)" and a remark that "after five months
(petitioner's) performance shall be evaluated and any adjustment in
salary shall depend on (his) work performance."
Petitioner asserts that, on November 20, 1996, in the presence of
his co-workers and subordinates, a senior officer of respondent
Middleby in bad faith withheld his time card and did not allow him to
work. Considering this as a dismissal "after the lapse of his
probationary employment," petitioner filed on November 21, 1996 a
complaint in the National Labor Relations Commission (NLRC) against
respondent Middleby contending that he had already become a regular
employee as of the date he was illegally dismissed.
In their defense, respondents claim that, during petitioner's
probationary employment, he showed poor performance in his
assigned tasks, incurred ten absences, was late several times and
violated company rules on the wearing of uniform. Since he failed to
meet company standards, petitioner's application to become a regular
employee was disapproved and his employment was terminated.

794
Page 795

LA dismissed the complaint. The NLRC affirmed the decision of


the labor arbiter. On appeal, the CA affirmed the judgment of the NLRC

ISSUES:
(1) Whether petitioner was allowed to work beyond his probationary
period and was therefore already a regular employee at the time of
his alleged dismissal
(2) Whether respondent Middleby informed petitioner of the
standards for "regularization" at the start of his employment
(3) Whether petitioner was illegally dismissed when respondent
Middleby opted not to renew his contract on the last day of his
probationary employment.

RULING:

(1) NO.
Petitioner claims that under the terms of his contract, his
probationary employment was only for five months as indicated by the
remark "Please be informed that after five months, your performance
shall be evaluated and any adjustment in salary shall depend on your
work performance." The argument lacks merit. As correctly held by the
labor arbiter, the appointment contract also stated in another part
thereof that petitioner's employment status was "probationary (6
mos.)." The five-month period referred to the evaluation of his work.
Petitioner insists that he already attained the status of a regular
employee when he was dismissed on November 20, 1996 because,
having started work on May 20, 1996, the six-month probationary
period ended on November 16, 1996. According to petitioner's
computation, since Article 13 of the Civil Code provides that one month
is composed of thirty days, six months total one hundred eighty days.
As the appointment provided that petitioner's status was "probationary
(6 mos.)" without any specific date of termination, the 180th day fell on
November 16, 1996. Thus, when he was dismissed on November 20,
1996, he was already a regular employee.
Petitioner's contention is incorrect. In CALS Poultry Supply
Corporation, et al. vs. Roco, et al., this Court dealt with the same issue
of whether an employment contract from May 16, 1995 to November
15, 1995 was within or outside the six-month probationary period. We
ruled that November 15, 1995 was still within the six-month
probationary period. We reiterate our ruling in CALS Poultry Supply:
(O)ur computation of the 6-month probationary period is reckoned
from the date of appointment up to the same calendar date of the 6th
month following.
In short, since the number of days in each particular month was
irrelevant, petitioner was still a probationary employee when
respondent Middleby opted not to "regularize" him on November 20,
1996

795
Page 796

(2) NO.

Section 6(d) of Rule 1 of the Implementing Rules of Book VI of the


Labor Code (Department Order No. 10, Series of 1997) provides that:
xxx xxx xxx
(d) In all cases of probationary employment, the employer shall make
known to the employee the standards under which he will qualify as a
regular employee at the time of his engagement. Where no standards
are made known to the employee at that time, he shall be deemed a
regular employee.
xxx xxx xxx
SC held that respondent Middleby substantially notified petitioner
of the standards to qualify as a regular employee when it apprised him,
at the start of his employment, that it would evaluate his supervisory
skills after five months.
An employer is deemed to substantially comply with the rule on
notification of standards if he apprises the employee that he will be
subjected to a performance evaluation on a particular date after his
hiring.

(3) NO.

It is settled that even if probationary employees do not enjoy


permanent status, they are accorded the constitutional protection of
security of tenure. This means they may only be terminated for just
cause or when they otherwise fail to qualify as regular employees in
accordance with reasonable standards made known to them by the
employer at the time of their engagement.
But we have also ruled in Manlimos, et al. vs. National Labor
Relations Commission that this constitutional protection ends on the
expiration of the probationary period. On that date, the parties are free
to either renew or terminate their contract of employment. Manlimos
concluded that "(t)his development has rendered moot the question of
whether there was a just cause for the dismissal of the petitioners . . .."
In the case at bar, respondent Middleby exercised its option not to
renew the contract when it informed petitioner on the last day of his
probationary employment that it did not intend to grant him a regular
status.
Although we can regard petitioner's severance from work as
dismissal, the same cannot be deemed illegal.

796
Page 797

Case Digests by: LEX ANGELO A. ROSARIO

MERCADO, ET AL. vs. AMA COMPUTER COLLEGE-


PARAÑAQUE CITY, INC
G.R. No. 183572. April 13, 2010
Brion, J.

DOCTRINE:
COMPANY STANDARDS THAT THE PROBATIONARY
EMPLOYEES SHOULD MEET MUST BE COMMUNICATED TO
THEM
Labor, for its part, is given the protection during the probationary period
of knowing the company standards the new hires have to meet during
the probationary period, and to be judged on the basis of these
standards, aside from the usual standards applicable to employees
after they achieve permanent status. Under the terms of the Labor
Code,these standards should be made known to the teachers on
probationary status at the start of their probationary period, or at the
very least under the circumstances of the present case, at the start of
the semester or the trimester during which the probationary standards
are to be applied. Of critical importance in invoking a failure to meet
the probationary standards, is that the school should show — as a
matter of due process — how these standards have been applied.

FACTS:
AMACC is an educational institution engaged in computer-based
education in the country. One of AMACC's biggest schools in the
country is its branch at Parañaque City. The petitioners Mercado, et al.
were faculty members who started teaching at AMACC on May 25,
1998. The petitioners executed individual Teacher's Contracts for each
of the trimesters that they were engaged to teach, with the following
common stipulation:
1. POSITION. The TEACHER has agreed to accept a non-tenured
appointment to work in the College of . . . effective . . . to . . . or for the
duration of the last term that the TEACHER is given a teaching load
based on the assignment duly approved by the DEAN/SAVP-COO.
For the school year 2000-2001, AMACC implemented new faculty
screening guidelines, set forth in its Guidelines on the Implementation
of AMACC Faculty Plantilla. Under the new screening guidelines,
teachers were to be hired or maintained based on extensive teaching
experience, capability, potential, high academic qualifications and
research background. The performance standards under the new
screening guidelines were also used to determine the present faculty
members' entitlement to salary increases. The petitioners failed to
obtain a passing rating based on the performance standards; hence
AMACC did not give them any salary increase.

797
Page 798

Because of AMACC's action on the salary increases, the


petitioners filed a complaint with the Arbitration Branch of the NLRC on
July 25, 2000, for underpayment of wages, non-payment of overtime
and overload compensation, 13th month pay, and for discriminatory
practices.
On September 7, 2000, the petitioners individually received a
memorandum from AMACC, through Human Resources Supervisor
Mary Grace Beronia, informing them that with the expiration of their
contract to teach, their contract would no longer be renewed.
The petitioners amended their labor arbitration complaint to
include the charge of illegal dismissal against AMACC. The petitioners
also contended that AMACC failed to give them adequate notice;
hence, their dismissal was ineffectual.
LA ruled that the petitioners had been illegally dismissed, and
ordered AMACC to reinstate them to their former positions. On appeal,
NLRC affirmed in toto the LA's ruling. On appeal to the CA, the CA
reverse LA and NLRC ruling, granted AMACC's petition for certiorari,
and dismissed the petitioners' complaint for illegal dismissal

ISSUE:
Whether petitioners were illegally dismissed

RULING:
YES.

A reality we have to face in the consideration of employment on


probationary status of teaching personnel is that they are not governed
purely by the Labor Code. The Labor Code is supplemented with
respect to the period of probation by special rules found in the Manual
of Regulations for Private Schools. On the matter of probationary
period, Section 92 of these regulations provides:
Section 92. Probationary Period. — Subject in all instances to
compliance with the Department and school requirements, the
probationary period for academic personnel shall not be more than
three (3) consecutive years of satisfactory service for those in the
elementary and secondary levels, six (6) consecutive regular
semesters of satisfactory service for those in the tertiary level, and
nine (9) consecutive trimesters of satisfactory service for those
in the tertiary level where collegiate courses are offered on a
trimester basis.
The CA pointed this out in its decision (as the NLRC also did), and we
confirm the correctness of this conclusion. Other than on the period,
the following quoted portion of Article 281 (now Article 279) of the
Labor Code still fully applies:
. . . The services of an employee who has been engaged on a
probationary basis may be terminated for a just cause when he fails to
qualify as a regular employee in accordance with reasonable
standards made known by the employer to the employee at the time
of his engagement. An employee who is allowed to work after a
probationary period shall be considered a regular employee.

798
Page 799

The use of employment for fixed periods during the teachers'


probationary period is likewise an accepted practice in the teaching
profession. We mentioned this in passing in Magis Young Achievers'
Learning Center v. Adelaida P. Manalo, albeit a case that involved
elementary, not tertiary, education, and hence spoke of a school year
rather than a semester or a trimester. SC noted in this case:
The common practice is for the employer and the teacher to enter
into a contract, effective for one school year. At the end of the
school year, the employer has the option not to renew the contract,
particularly considering the teacher's performance. If the contract is
not renewed, the employment relationship terminates. If the contract
is renewed, usually for another school year, the probationary
employment continues. Again, at the end of that period, the parties
may opt to renew or not to renew the contract. If renewed, this second
renewal of the contract for another school year would then be the last
year — since it would be the third school year — of probationary
employment. At the end of this third year, the employer may now
decide whether to extend a permanent appointment to the
employee, primarily on the basis of the employee having met the
reasonable standards of competence and efficiency set by the
employer. For the entire duration of this three-year period, the
teacher remains under probation. Upon the expiration of his
contract of employment, being simply on probation, he cannot
automatically claim security of tenure and compel the employer
to renew his employment contract. It is when the yearly contract is
renewed for the third time that Section 93 of the Manual becomes
operative, and the teacher then is entitled to regular or permanent
employment status.
SC held that in a situation where the probationary status overlaps
with a fixed-term contract not specifically used for the fixed term it
offers, Article 281 should assume primacy and the fixed-period
character of the contract must give way. This conclusion is
immeasurably strengthened by the petitioners' and the AMACC's
hardly concealed expectation that the employment on probation could
lead to permanent status, and that the contracts are renewable unless
the petitioners fail to pass the school's standards.
To highlight what SC meant by a fixed-term contract specifically
used for the fixed term it offers, a replacement teacher, for example,
may be contracted for a period of one year to temporarily take the place
of a permanent teacher on a one-year study leave. The expiration of
the replacement teacher's contracted term, under the circumstances,
leads to no probationary status implications as she was never
employed on probationary basis; her employment is for a specific
purpose with particular focus on the term and with every intent to end
her teaching relationship with the school upon expiration of this term.
If the school were to apply the probationary standards (as in fact
it says it did in the present case), these standards must not only be
reasonable but must have also been communicated to the teachers at
the start of the probationary period, or at the very least, at the start of
the period when they were to be applied. These terms, in addition to

799
Page 800

those expressly provided by the Labor Code, would serve as the just
cause for the termination of the probationary contract. As explained
above, the details of this finding of just cause must be communicated
to the affected teachers as a matter of due process.
AMACC, by its submissions, admits that it did not renew the
petitioners' contracts because they failed to pass the Performance
Appraisal System for Teachers (PAST) and other requirements for
regularization that the school undertakes to maintain its high academic
standards. The evidence is unclear on the exact terms of the
standards, although the school also admits that these were standards
under the Guidelines on the Implementation of AMACC Faculty
Plantilla put in place at the start of school year 2000-2001.
While we can grant that the standards were duly communicated
to the petitioners and could be applied beginning the 1st trimester of
the school year 2000-2001, glaring and very basic gaps in the school's
evidence still exist. The exact terms of the standards were never
introduced as evidence; neither does the evidence show how these
standards were applied to the petitioners. Without these pieces of
evidence (effectively, the finding of just cause for the non-renewal of
the petitioners' contracts), we have nothing to consider and pass upon
as valid or invalid for each of the petitioners. Inevitably, the non-
renewal (or effectively, the termination of employment of employees on
probationary status) lacks the supporting finding of just cause that the
law requires and, hence, is illegal.

800
Page 801

Case Digests by: LEX ANGELO A. ROSARIO

OYSTER PLAZA HOTEL VS. MELIVO


G.R. No. 217455. October 5, 2016
Mendoza, J.

DOCTRINE:
WHEN A PROBATIONARY EMPLOYEE DEEMED A REGULAR
EMPLOYEE
An employee allowed to work beyond the probationary period is
deemed a regular employee.
FACTS:
On October 22, 2009, respondent Errol O. Melivo (Melivo) filed
before the NLRC a Complaint for illegal dismissal with prayers for
reinstatement and payment of back wages, holiday pay, overtime pay,
service incentive leave, and, 13th month pay against petitioners Oyster
Plaza Hotel (Oyster Plaza), Rolito Go (Go), and Jennifer Ampel
(Ampel).
At the February 17, 2010 hearing, however, only Melivo appeared.
On even date, Melivo filed his Position Paper, alleging the
following: that Oyster Plaza was a business entity engaged in the
business of hotel operation, under the ownership/management of Go
and Ampel; that in August 2008, Oyster Plaza hired him as a trainee
room boy; that in November 2008, Oyster Plaza hired him as a
probationary room boy and he was made to sign an employment
contract but he was not furnished a copy, that the said contract expired
in March 2009 and his work ended; that on April 7, 2009, Oyster Plaza
hired him again as a room boy, but without any employment contract
or document; and that in September 2009, his supervisor Ampel
verbally told him that his contract was expiring, thus, he must stop
reporting for work.
The LA ruled that Melivo was illegally dismissed. Considering that
Melivo had already rendered six (6) months of service for Oyster Plaza,
the LA held that he had become a regular employee by operation of
law. The NLRC affirmed the decision of the LA. On appeal, CA affirmed
the decision of the NLRC.

ISSUE:
Whether Melivo was illegally dismissed

RULING:
YES.

An employee allowed to work beyond the probationary period is


deemed a regular employee.

801
Page 802

In Holiday Inn Manila vs. NLRC (Holiday Inn), the Court


considered therein complainant's 3-week on-the-job training (OJT)
period as her probationary employment period. The Court explained
that the complainant was certainly under observation during her 3-
week OJT such that if her services proved unsatisfactory, she could
have been dropped anytime during said period. On the other hand,
when her services were continued after her training, the employer in
effect recognized that she had passed probation and was qualified to
be a regular employee. Thus, the Court ruled that the complainant
therein attained regular employment status when she was formally
placed under probation after her OJT.
The present case involves substantially the same factual
considerations as that of Holiday Inn. In this case, Melivo was first hired
as a trainee in August 2008. His training lasted for three (3) months.
As a room boy, his performance was certainly under observation.
Thus, it can be reasonably deduced that Melivo's probationary
employment actually started in August 2008, at the same time he
started working as a trainee. Therefore, when he was re-hired as room
boy after his training period sometime in November 2008 he attained
regular employment status.
Assuming arguendo that the 3-month training period could be
considered a probationary period, the conclusion would still be the
same. It should be remembered that Melivo was again employed as a
room boy in November 2008 under probationary status for five (5)
months or until March 2009. Records would show that Melivo had
completed his probationary employment. Thus, when Oyster Plaza re-
hired him for the third time on April 7, 2009, he became its regular
employee thereof.
The petitioners' contention that Melivo was hired as a project
employee is untenable. Under Article 280 of the Labor Code, as
amended, a project employee is one whose employment has been
fixed for a specific project or undertaking, the completion or termination
of which has been determined at the time of the engagement of the
employee. Here, the contract of employment failed to indicate the
specific project or undertaking for which Oyster Plaza sought Melivo's
services. Moreover, as correctly noted by the NLRC, the petitioners
failed to submit a report of Melivo's termination to the nearest public
employment office, as required under Section 2 of D.O. No. 19.
As a regular employee, Melivo could only be dismissed for just or
authorized causes after affording him the procedural requirement of
notice and hearing. The petitioners failed to adduce evidence that
Melivo's dismissal was for a just or authorized cause, or that he was
sufficiently notified and given opportunity to be heard why his
employment should not be terminated. Hence, Melivo's dismissal was
illegal.

802
Page 803

Case Digests by: LEX ANGELO A. ROSARIO

IMASEN PHILIPPINE MANUFACTURING CORP. V.


ALCON
G.R. No. 194884. October 22, 2014
Brion, J.

DOCTRINE:
DISMISSAL BECAUSE OF MISCONDUCT BY HAVING SEXUAL
ACTS WITHIN THE COMPANY PREMISES AND DURING
WORKING HOURS
Sexual acts and intimacies between two consenting adults belong, as
a principled ideal, to the realm of purely private relations. Whether
aroused by lust or inflamed by sincere affection, sexual acts should be
carried out at such place, time and circumstance that, by the generally
accepted norms of conduct, will not offend public decency nor disturb
the generally held or accepted social morals. Under these parameters,
sexual acts between two consenting adults do not have a place in the
work environment
.
FACTS:
Respondents Ramonchito Alcon and Joann Papa reported for work at
petitioner Imasen Philippine Manufacturing Corporation’s company
premises — from 8:00 pm to 5:00 am. At around 12:40 am, Cyrus
Altiche, Imasen's security guard, saw the respondents having sexual
intercourse on the floor, using a piece of carton as mattress at the "Tool
and Die" section. Altiche immediately relayed what he saw to Danilo
Ogana, another security guard, who made a follow-up inspection.
Altiche then submitted a handwritten report of the incident to Imasen's
Finance and Administration Manager. Thereafter, Imasen informed
respondents of Altiche's report and directed them to submit their
individual explanation.
The respondents claimed that they were merely sleeping in the "Tool
and Die" section at the time of the incident. They also claimed that
other employees were near the area, making the commission of the
act charged impossible.
Imasen directed them to appear at the formal hearing of the
administrative charge against them. In a memorandum, Imasen
terminated their services. It found the respondents guilty of the act
charged which it considered as "gross misconduct contrary to the
existing policies, rules and regulations of the company." Thus,
respondents filed a complaint for illegal dismissal.
The Labor Arbiter and NLRC dismissed the respondents' complaint
since respondents' dismissal was valid, i.e., for the just cause of gross
misconduct and with due process. CA, however, nullified the NLRC's
ruling. Despite agreeing with the infraction charged, it disagreed with
the conclusion that the respondents' sexual intercourse inside

803
Page 804

company premises constituted serious misconduct that the Labor


Code considers sufficient to justify the penalty of dismissal. It pointed
out that the respondents' act, while provoked by "reckless passion in
an inviting environment and time," was not done with wrongful intent or
with the grave or aggravated character that the law requires..

ISSUE:
Whether engaging in sexual intercourse inside company premises
during work hours amounts to serious misconduct justifying their
dismissal.

RULING:
YES.
Misconduct is an improper or wrong conduct. It is a transgression
of some established and definite rule of action, a forbidden act, a
dereliction of duty, willful in character, and implies wrongful intent and
not mere error in judgment. For misconduct or improper behavior to be
a just cause for dismissal, the following elements must concur: (a) the
misconduct must be
serious; (b) it must relate to the performance of the employee's
duties showing that the employee has become unfit to continue
working for the employer; and (c) it must have been performed with
wrongful intent.
Sexual acts and intimacies between two consenting adults belong,
as a principled ideal, to the realm of purely private relations. Whether
aroused by lust or inflamed by sincere affection, sexual acts should be
carried out at such place, time and circumstance that, by the generally
accepted norms of conduct, will not offend public decency nor disturb
the generally held or accepted social morals. Under these parameters,
sexual acts between two consenting adults do not have a place in the
work environment.
In the case, indisputably, the respondents engaged in sexual
intercourse inside company premises and during work hours. These
circumstances, by themselves, are already punishable misconduct.
Added to these considerations, however, is the implication that the
respondents did not only disregard company rules but flaunted their
disregard in a manner that could reflect adversely on the status of
ethics and morality in the company.
Additionally, the respondents engaged in sexual intercourse in an
area where co-employees or other company personnel have ready and
available access.
The respondents likewise committed their act at a time when the
employees were expected to be and had, in fact, been at their
respective posts, and when they themselves were supposed to be, as
all other employees had in fact been,

804
Page 805

working. Thus, respondents' misconduct is of grave and


aggravated character so that the company was justified in imposing
the highest penalty available —
dismissal. Their infraction transgressed the bounds of socially and
morally accepted human public behavior, and at the same time showed
brazen disregard for the respect that their employer expected of them
as employees. By their misconduct, the respondents, in effect, issued
an open invitation for others to commit the same infraction, with like
disregard for their employer's rules, for the respect owed to their
employer, and for their co-employees' sensitivities. Taken together,
these considerations reveal a depraved disposition that the Court
cannot but consider as a valid cause for dismissal.

805
Page 806

Case Digests by: LEX ANGELO A. ROSARIO

NORTHWEST AIRLINES, INC. V. DEL ROSARIO


G.R. No. 157633. September 10, 2014
Bersamin, J.

DOCTRINE:
DEFINITION OF FIGHT; FIGHTING AS SERIOUS MISCONDUCT
In several rulings where the meaning of fight was decisive, the Court
has observed that the term fight was considered to be different from
the term argument.
.
FACTS:
During the boarding preparations, Kathleen Gamboa, a flight
attendant assigned at the First Class Section of Flight NW 26, needed
to borrow a wine bottle opener from her fellow attendants. Respondent
Ma. Concepcion Del Rosario, a flight attendant assigned at the
Business Class Section of the same flight, remarked that any flight
attendant who could not bring a wine bottle opener had no business
working in the First-Class Section. Apparently, Gamboa overheard Del
Rosario's remarks, and later on verbally confronted her. Their
confrontation escalated into a heated argument. Escaño intervened but
the two ignored her, prompting her to rush outside the aircraft to get
Morales, the Assistant Base Manager, to pacify them. Del Rosario
claimed that only an animated discussion had transpired between her
and Gamboa. However, since respondent refused to fly on the
condition that they would have to stay away from each other during the
flight, Morales decided not to allow both of them on the flight.
After an investigation, respondent was informed of her termination
from the service. Northwest stated that based on the results of the
investigation, Del Rosario and Gamboa had engaged in a fight on
board the aircraft, even if there had been no actual physical contact
between them; and that because fighting was strictly prohibited by
Northwest, Northwest considered her dismissal from the service
justified and in accordance with the Rules of Conduct for Employees.
Labor Arbiter held that the dismissal of Del Rosario had been
justified and valid upon taking into account that Northwest had been
engaged in the airline business in which a good public image had been
demanded, and in which flight attendants had been expected to
maintain an image of sweetness and amiability; that fighting among its
employees even in the form of heated arguments or discussions were
very contradictory to that expected image.
NLRC reversed the decision of the Labor Arbiter and declared that
the incident between her and Gamboa could not be considered as
synonymous with fighting since fighting is the existence of an
underlying hostility between the parties which is so intense that there
is an imminent danger of a physical conflict (if there is none yet). At the

806
Page 807

very least, they should project a general appearance of wanting to


physically strike each other. NLRC stated that the witnesses refer to
the incident as "arguing" or a "serious or animated discussion,” which
is merely talking or debating a certain issue.
Northwest, however, argued that Del Rosario was dismissed on
the grounds of serious misconduct and willful disobedience.

ISSUE:
Whether the incident between respondent and Gamboa amounts to
fighting to justify respondent’s dismissal.

RULING:
NO. Respondent was thus illegally dismissed.
Misconduct refers to the improper or wrong conduct that
transgresses some established and definite rule of action, a forbidden
act, a dereliction of duty, willful in character, and implies wrongful intent
and not mere error in judgment. But misconduct or improper behavior,
to be a just cause for termination of employment, must: (a) be serious;
(b) relate to the performance of the employee's duties; and (c) show
that the employee has become unfit to continue working for the
employer.
The last two elements of misconduct were present in the case of
Del Rosario. The cause of her dismissal related to the performance of
her duties as a flight attendant, and she became unfit to continue
working for Northwest. However, the first element is lacking.
The term fight was considered to be different from the term
argument. In People v. Asto, the Court characterized fight as not just a
merely verbal tussle but a physical combat between two opposing
parties. In Pilares, Sr. v. People,fight was held to be more than just an
exchange of words that usually succeeded the provocation by either
party.
In the case, the incident involving Del Rosario and Gamboa could
not be justly considered as akin to the fight contemplated by Northwest.
Del Rosario and Gamboa were arguing but not fighting. The
understanding of fight as one that required physical combat was
absent. Moreover, even assuming arguendo that the incident was the
kind of fight prohibited by Northwest's Rules of Conduct, the same
could not be considered as of such seriousness as to warrant Del
Rosario's dismissal from the service. The gravity of the fight, which was
not more than a verbal argument between them, was not enough to
tarnish or diminish Northwest's public image.

807
Page 808

Case Digests by: LEX ANGELO A. ROSARIO

CITIBANK V. NATIONAL LABOR RELATIONS


COMMISSION
G.R. No. 159302. February 6, 2008
Carpio-Morales, J.

DOCTRINE:
ATTITUDE PROBLEM AS SERIOUS MISCONDUCT
When an employee, despite repeated warnings from the employer,
obstinately refuses to curtail a bellicose inclination such that it erodes
the morale of co-employees, the same may be a ground for dismissal
for serious misconduct.

FACTS:
On August 8, 1979, Rosita Tan Paragas joined herein petitioner
Citibank in various positions and capacities. In early 1993, as a result
of the
reorganization, Citibank declared certain officers and employees,
or their positions/functions, redundant. Among these affected was
Paragas. However,
her employment was not terminated but was assigned to Records
Management Unit of the Quality Assurance Division as bank statement
retriever, a filing clerk job described by complainant as “non-brainer
job.”
In July 1994, Paragas was assigned to file Universal Account
Opening Forms (UAOF) in file boxes and retrieving such UAOFs from
the file boxes upon internal customers’ request from time to time.
Accordingly, starting February 21, 1995, her job in the bank was to file
and retrieve UAOFs.
Subsequently, Paragas was assigned to undertake the special
project of reorganizing the UAOF’s from December 13, 1996 to May
15, 1997. AVP Narciso Ferrera issued two memos to Paragas, calling
her attention on various misfiling of the reorganized UAOF files. Failing
to correct such misfiling and to finish the project on time despite the
extension of time for her to do so, Paragas was directed by Ferrera to
explain in writing why her employment should not be terminated.
After an administrative conference, Citibank ruled that the
explanation offered by Paragas, as well as those she raised during the
conference, were found self-serving. Consequently, her employment
was terminated on the ground of serious misconduct, willful
disobedience, gross and habitual neglect of duties and gross
inefficiency.
Paragas filed a complaint for illegal dismissal, praying for
reinstatement, backwages, damages and attorney’s fees. The labor
arbiter dismissed the complaint for lack of merit, finding that her
dismissal on the ground of work inefficiency was valid. On appeal, the

808
Page 809

NLRC affirmed the decision of the labor arbiter with the modification
that respondent should be paid separation pay “as a form of equitable
relief” in view of her length of service with petitioner.
Paragas filed a motion for partial reconsideration, praying that
petitioner Citibank be ordered to pay her the “Provident Fund” benefits
under its retirement plan for which she claimed to be qualified pursuant
to petitioner’s “Working Together” Manual. Respondent, claiming that
the labor arbiter upheld her dismissal on the ground of merely “work
inefficiency” and not for any misconduct on her part, asserted that she
is entitled to 90% of the retirement benefits.
The NLRC granted respondent’s motion for partial
reconsideration. Pursuant to this, Citibank filed a petition for certiorari
with the CA, which the latter denied. Hence, this petition.

ISSUE:
Whether Paragas was validly dismissed for serious misconduct.

RULING:
YES.
In support of its ruling that respondent’s dismissal was valid, the
labor arbiter relied on the performance appraisals of respondent from
July to December 1994, from January to June 1995, and from July to
December 1996, all of which were submitted by petitioner’s Assistant
Vice-President, Narciso M. Ferrera. These performance appraisals,
however, did not merely show that respondent was not able to meet
performance targets. More relevantly, they also consistently noted
significant behavioral and attitudinal problems in respondent. In
particular, respondent was found to be very argumentative; she had
difficulty working with others; she was hard to deal with; and she never
ceased being the subject of complaints from co-workers.
For the appraisal period from June to December 1995,
respondent’s performance appraisal report stated that her attitude
towards her work, the bank, and superiors needed reformation. The
report for January to June 1996 made the same observation, indicating
that there was no improvement on her part. The performance appraisal
report of respondent for the period of January to June 1997, besides
stating that she was still “hard to deal with,” described her as
“belligerent,” one who had “a negative presence which affects the
morale of the entire unit,” and who “pick[ed] fights with peers and other
employees even without provocation.”
In sum, it is respondent’s obstinate refusal to reform herself which
ultimately persuades this Court to find that her dismissal on the ground
of serious misconduct was valid.

809
Page 810

Case Digests by: LEX ANGELO A. ROSARIO

MIRANT (PHILIPPINES) CORP. V. CARO


G.R. No. 181490. April 23, 2014
Villarama, Jr., J.

DOCTRINE:
REFUSAL TO UNDERGO DRUG TEST NOT NECESSARILY
SERIOUS MISCONDUCT
The Court rules that Mirant’s Anti-Drugs Policy is excessive in
terminating an employee for his “unjustified refusal” to subject himself
to the random drug test on first offense, without clearly defining what
amounts to an “unjustified refusal.”
It is not a mere jurisprudential principle, but an enshrined provision
of law, that all doubts shall be resolved in favor of labor. Mirant’s Anti-
Drugs Policy being ambiguous as to what constitutes “unjustified
refusal”, it is not fair for this Court to allow an ambiguous policy to
prejudice the rights of an employee against illegal dismissal. To hold
otherwise and sustain the stance of petitioner corporation would be to
adopt an interpretation that goes against the very grain of labor
protection in this jurisdiction.

FACTS:
Petitioner corporation and its related companies, Mirant Saul
Corporation and Mirant Pagbilao Corporation, maintain around 2,000
employees. Petitioner Edgardo Bautista was the president of Mirant
when respondent Caro’s employment was terminated.
Respondent Caro was hired by Mirant Pagbilao as Logistics
Officer on January 3, 1994. On November 3, 2004, petitioner
corporation conducted a random drug test where respondent was
randomly chosen among its employees who would be tested for illegal
drug use. Respondent was duly notified that he was scheduled to be
tested after lunch on that day.
However, respondent Caro failed to take the drug test as
scheduled. He cited as reason a phone call from his wife’s colleague
informing him that a bombing incident occurred near his wife’s
workplace in Tel Aviv, Israel. Allegedly, Caro informed his
department’s secretary that he will attend to his predicament first and
return to work after. After missing his scheduled drug test, Caro offered
to take it on another day, at his own expense.
On November 8, 2004, respondent received a Show Cause Notice
from petitioner corporation through Jaime Dulot, his immediate
supervisor, requiring him to explain in writing why he should not be
charged with “unjustified refusal to submit to random drug testing.”
Respondent submitted his written explanation. Thereafter, an
Investigating Panel was formed. The Panel ruled that respondent Caro
was guilty of “unjustified refusal to submit to random drug testing” and

810
Page 811

recommended a penalty of four working weeks suspension without


pay.
However, Mirant’s Assistant Vice President for Material
Management Department recommended that respondent be
terminated from employment instead of merely being suspended,
arguing that Caro avoided the drug testing, and that “avoidance” is
synonymous with “refusal.” Caro’s employment was subsequently
terminated.
Respondent Caro filed a complaint for illegal dismissal averring
that Mirant did not comply with the twin requirements of notice and
hearing.
The Labor Arbiter found Caro to have been illegally dismissed,
saying that there was no just cause for the termination of Caro’s
employment. The NLRC ruled that Caro was properly dismissed for
cause, and he was also properly accorded his constitutional right to
due process. Despite its ruling, the NLRC granted financial assistance
to respondent on equitable grounds. It invoked the past decisions of
this Court which allowed the award of financial assistance due to
factors such as long years of service or the Court’s concern and
compassion towards labor where the infraction was not so serious.
The CA disagreed with the NLRC, ruling that it was immaterial
whether respondent failed, refused, or avoided being tested. The CA
ruled that the material fact was that respondent did not get himself
tested, in clear disobedience of company instructions and policy.
However, the CA considered the penalty of dismissal to be too harsh
to be imposed on respondent.

ISSUE:
Whether respondent Caro’s failure to undergo the scheduled random
drug testing constitutes serious misconduct.

RULING:
NO.

The Court agrees with the CA that Caro was illegally dismissed.
To the Court, petitioner Mirant’s subject Anti-Drugs Policy fell short of
being fair and reasonable.
The policy was not clear on what constitutes “unjustified refusal”
when the subject drug policy prescribed that an employee’s “unjustified
refusal” to submit to a random drug test shall be punishable by the
penalty of termination for the first offense. To be sure, the term
“unjustified refusal” could not possibly cover all forms of “refusal” as
the employee’s resistance, to be punishable by termination, must be
“unjustified.” To the mind of the Court, it is on this area where petitioner
corporation had fallen short of making it clear to its employees – as
well as to management – as to what types of acts would fall under the

811
Page 812

purview of “unjustified refusal.” Even petitioner corporation’s own


Investigating Panel recognized this ambiguity.
The fact that petitioner corporation’s own Investigating Panel and
its Vice President for Operations differed in their recommendations
regarding respondent’s case are first-hand proof that there, indeed, is
ambiguity in the interpretation and application of the subject drug
policy. The fact that petitioner corporation’s own personnel had to
dissect the intended meaning of “unjustified refusal” is further proof that
it is not clear on what context the term “unjustified refusal” applies to.
Moreover, the Court held that the penalty of termination imposed
by petitioner corporation upon respondent fell short of being
reasonable. Company policies and regulations are generally valid and
binding between the employer and the employee unless shown to be
grossly oppressive or contrary to law – as in the case at bar.
To be sure, the unreasonableness of the penalty of termination as
imposed in this case is further highlighted by a fact admitted by
petitioner corporation itself: that for the ten-year period that respondent
had been employed by petitioner corporation, he did not have any
record of a violation of its company policies.
In sum, the Court ruled that Caro was illegally dismissed.

812
Page 813

Case Digests by: LEX ANGELO A. ROSARIO

NACAGUE V. SULPICIO LINES


G.R. No. 172589. August 9, 2010
Carpio, J.

DOCTRINE:
REQUIRED PROOF FOR DISMISSAL FOR USE OF DRUGS, DRUG
USE AS SERIOUS MISCONDUCT
To constitute valid dismissal from employment, two requisites
must concur: (1) the dismissal must be for a just or authorized cause;
and (2) the employee must be afforded an opportunity to be heard and
to defend himself.
Section 36 of R.A. No. 9165 provides that drug tests shall be
performed only by authorized drug testing centers, and that drug
testing shall consist of both the screening test and the confirmatory
test.

FACTS:
On June 15, 1995, petitioner Jeffrey Nacague was hired by
respondent Sulpicio Lines as “hepe de viaje” or its representative on
board its vessel, M/V Princess of the World.
Sulpicio Lines received an anonymous letter regarding the use of
illegal drugs on board the ship. A housekeeper on the ship, Ceasar
Chico, reported finding drug paraphernalia inside the Mopalla Suite
Room, as well as the threat made
to him by Nacague and one Reynaldo Doroon after finding said
drug paraphernalia.
Subsequently, Sulpicio Lines sent a notice of investigation to
Nacague informing him of the charges against him for use of illegal
drugs and threatening a co-employee.
Some crew members, including Nacague, were subjected to a
random drug testing at S.M. Lazo Clinic as soon as the ship docked in
the port of Manila. The result of the drug test showed that Nacague
was positive for shabu. Thus, Sulpicio Lines subjected Nacague to a
formal investigation, where the latter denied using illegal drugs.
Nacague even submitted a drug test result he
voluntary took, where he yielded negative for use of illegal drugs.
However, on March 7, 2003, Sulpicio Lines sent Nacague a
memorandum, finding him culpable of grave misconduct and
terminating him from service.
After an illegal dismissal complaint was filed, the Labor Arbiter
found Nacague to have been illegally dismissed, saying that the
termination of employment of employees found positive for using illegal
drugs should not be exercised indiscriminately and thoughtlessly. The
Labor Arbiter agreed with Nacague that since S.M. Lazo Clinic was not

813
Page 814

accredited by the Dangerous Drugs Board, the results of the drug test
conducted by said clinic are questionable.
The NLRC reversed the Labor Arbiter’s decision and dismissed
Nacague’s complaint for lack of merit. According to the NLRC, since
Nacague, who was performing a task involving trust and confidence,
was found positive for using illegal drugs, he was guilty of serious
misconduct and loss of trust and confidence. The NLRC added that
Sulpicio Lines’ Code of Conduct specified that the penalty for the use
and illegal possession of prohibited drugs is dismissal.
The CA ruled that Sulpicio Lines complied with both the procedural
and substantive requirements of the law when it terminated the
employment of Nacague. Moreover, the CA said that the positive result
of the S.M. Lazo Clinic drug test was the main basis of Sulpicio Lines
in terminating Nacague’s employment, and that the evidence
presented by Sulpicio Lines was sufficient to justify the conclusion that
Nacague committed serious misconduct and a breach of trust and
confidence warranting his dismissal from employment.

ISSUE:
Whether Nacague was illegally dismissed from employment

RULING:
YES.
The Labor Code provides that to constitute valid dismissal from
employment, two requisites must concur: (1) the dismissal must be for
a just or authorized cause; and (2) the employee must be afforded an
opportunity to be heard and to defend himself. Additionally, Section 36
of R.A. No. 9165 provides that drug tests shall be performed only by
authorized drug testing centers, and that drug testing shall consist of
both the screening test and the confirmatory test.
The Court finds that Sulpicio Lines failed to clearly show that
Nacague was guilty of using illegal drugs. The Court agrees with the
Labor Arbiter that the lack of accreditation of S.M. Lazo Clinic made its
drug test results doubtful.
Sulpicio Lines failed to prove that S.M. Lazo Clinic is an accredited
drug testing center. Also, only a screening test was conducted to
determine if Nacague was guilty of using illegal drugs. Sulpicio Lines
did not confirm the positive result of the screening test with a
confirmatory test. Sulpicio Lines failed to indubitably prove that
Nacague was guilty of using illegal drugs amounting to serious
misconduct and loss of trust and confidence. Sulpicio Lines failed to
clearly show that it had a valid and legal cause for terminating
Nacague’s employment. When the alleged valid cause for the
termination of employment is not clearly proven, as in this case, the
law considers the matter a case of illegal dismissal.

814
Page 815

Case Digests by: LEX ANGELO A. ROSARIO

LEUS V. ST. SCHOLASTICA'S COLLEGE WESTGROVE


G.R. No. 187226. January 28, 2015
Reyes, J.

DOCTRINE:
WHEN THE LAW SPEAKS OF IMMORAL, THE LAW PERTAINS TO
SECULAR MORALITY
The determination of whether a conduct is disgraceful or immoral
involves a two-step process: first, a consideration of the totality of the
circumstances surrounding the conduct; and second, an assessment
of the said circumstances vis-à-vis the prevailing norms of conduct,
i.e., what the society generally considers moral and respectable.
The Court held that public and secular morality should determine
the prevailing norms of conduct, not religious morality. Accordingly,
when the law speaks of immoral or, necessarily, disgraceful conduct,
it pertains to public and secular morality; it refers to those conducts
which are proscribed because they are detrimental to conditions upon
which depend the existence and progress of human society.
FACTS:
Petitioner Cheryll Santos Leus was hired by respondent St.
Scholastica’s College Westgrove (SSCW), a Catholic educational
institution, as a nonteaching personnel.
In 2003, petitioner Leus got pregnent. When SSCW learned of
Leus’ pregnancy, SSCW Directress, Sr. Edna Quiambao, advised
Leus to file a
resignation letter effective June 1, 2003. However, Leus replied
that she would not resign from her employment solely because she got
pregnant before marriage.
Subsequently, the directress directed the petitioner to explain in
writing why she should not be dismissed for engaging in pre-marital
sexual relations, and even getting pregnant as a result, amounting to
serious misconduct and conduct unbecoming of an employee of a
Catholic school.
In response, Leus explained that her pregnancy out of wedlock
does not amount to serious misconduct or conduct unbecoming of an
employee. Moreover, she is unaware of any school policy stating that
being pregnant out of wedlock is considered as a serious misconduct,
and thus, a ground for dismissal.
Consequently, SSCW informed Leus that her employment with
SSCW is terminated on the ground of serious misconduct. Thus, Leus
filed a complaint for illegal dismissal with the Regional Arbitration
Branch of NLRC-Quezon City.
The Labor Arbiter dismissed the complaint, saying that petitioner’s
pregnancy out of wedlock amounts to a “disgraceful and immoral
conduct”, which is a just cause for her dismissal. The NLRC affirmed

815
Page 816

the decision of the Labor Arbiter. The CA ruled that petitioner’s


dismissal was a valid exercise of
SSCW’s management prerogatives to discipline and impose
penalties on erring employees pursuant to its policies, rules and
regulations. The CA also agrees with the NLRC’s conclusion that
petitioner’s pregnancy out of wedlock is a “disgraceful and immoral
conduct”, and thus, a ground for
dismissal under the applicable law, Section 94(e) of the 1992
Manual of Regulations for Private Schools (1992 MRPS).

ISSUE:
Whether petitioner Leus’ pregnancy out of wedlock constitutes a valid
ground to terminate her employment

RULING:
NO.

In resolving the issue, the Court assessed the matter from a strictly
neutral and secular point of view – the relationship between SSCW as
employer and the petitioner as an employee, the causes provided for
by law in the termination of such relationship, and the evidence on
record. The ground cited for the petitioner’s dismissal, i.e., pre-marital
sexual relations and, consequently, pregnancy out of wedlock, were
assessed as to whether the same constitutes a valid ground for
dismissal pursuant to Section 94(e) of the 1992 MRPS.
The labor tribunals concluded that the petitioner’s pregnancy out
of wedlock, per se, is “disgraceful and immoral” considering that she is
employed in a Catholic educational institution. In arriving at such
conclusion, the labor tribunals merely assessed the fact of the
petitioner’s pregnancy vis-à-vis the totality of the circumstances
surrounding the same.
However, the Court finds no substantial evidence to support the
aforementioned conclusion arrived at by the labor tribunals. The fact of
the petitioner’s pregnancy out of wedlock, without more, is not enough
to characterize the petitioner’s conduct as disgraceful or immoral.
There must be substantial evidence to establish that pre-marital sexual
relations and, consequently, pregnancy out of wedlock, are indeed
considered disgraceful or immoral.
In Chua-Qua v. Clave, the Court stressed that to constitute
immorality, the circumstances of each particular case must be
holistically considered and evaluated in light of the prevailing norms of
conduct and applicable laws.
Otherwise stated, it is not the totality of the circumstances
surrounding the conduct per se that determines whether the same is
disgraceful or immoral, but the conduct that is generally accepted by
society as respectable or moral.

816
Page 817

That the petitioner was employed by a Catholic educational


institution per se does not absolutely determine whether her pregnancy
out of wedlock is disgraceful or immoral. There is still a necessity to
determine whether the petitioner’s pregnancy out of wedlock is
considered disgraceful or immoral in accordance with the prevailing
norms of conduct.
The Court held that public and secular morality should determine
the prevailing norms of conduct, not religious morality. Accordingly,
when the law speaks of immoral or, necessarily, disgraceful conduct,
it pertains to public and secular morality; it refers to those conducts
which are proscribed because they are detrimental to conditions upon
which depend the existence and progress of human society.
The petitioner’s pregnancy out of wedlock is not a disgraceful or
immoral conduct since she and the father of her child have no
impediment to marry each other. The Court does not find any
circumstance in this case which would lead the Court to conclude that
the petitioner committed a disgraceful or immoral conduct. It bears
stressing that the petitioner and her boyfriend, at the time they
conceived a child, had no legal impediment to marry. Indeed, even
prior to her dismissal, the petitioner married her boyfriend, the father of
her child.
Having established that the petitioner was illegally dismissed, the
Court now determines the reliefs that she is entitled to and their extent.
In view of the particular circumstances of this case, it would be more
prudent to direct SSCW to pay the petitioner separation pay in lieu of
actual reinstatement. The continued employment of the petitioner with
SSCW would only serve to intensify the atmosphere of antipathy and
antagonism between the parties.
Consequently, the Court awards separation pay to the petitioner
equivalent to one (1) month pay for every year of service, with a fraction
of at least six (6) months considered as one (1) whole year, from the
time of her illegal dismissal up to the finality of this judgment, as an
alternative to reinstatement. Moreover, the Court awards petitioner full
backwages from the time of her illegal dismissal up to the finality of this
Decision and attorney’s fees equivalent to ten percent (10%) of the
total monetary award.

817
Page 818

Case Digests by: TANGHAL, JUAN MIGUEL P.


CADIZ v. BRENT HOSPITAL
G.R. No. 187417, February 24, 2016
Reyes, J.
Immorality as a just cause for termination of employment

The Court ruled that the determination of whether a conduct is


disgraceful or immoral involves a two-step process: first, a
consideration of the totality of the circumstances surrounding the
conduct; and second, an assessment of the said circumstances vis-a-
vis the prevailing norms of conduct, i.e., what the society generally
considers moral and respectable.

Jurisprudence has already set the standard of morality with


which an act should be gauged - it is public and secular, not religious.
Whether a conduct is considered disgraceful or immoral should be
made in accordance with the prevailing norms of conduct, which, as
stated in Leus, refer to those conducts which are proscribed because
they are detrimental to conditions upon which depend the existence
and progress of human society.

FACTS:
Cadiz was the Human Resource Officer of respondent Brent Hospital
and Colleges, Inc. (Brent) at the time of her indefinite suspension from
employment in 2006. The cause of suspension was Cadiz's
Unprofessionalism and Unethical Behavior Resulting to Unwed Pregnancy.
It appears that Cadiz became pregnant out of wedlock, and Brent imposed
the suspension until such time that she marries her boyfriend in accordance
with law.

Cadiz then filed with the Labor Arbiter (LA) a complaint for Unfair
Labor Practice, Constructive Dismissal, Non-Payment of Wages and
Damages with prayer for Reinstatement.

In its Decision dated April 12, 2007, the LA found that Cadiz's indefinite
suspension amounted to a constructive dismissal; nevertheless, the LA
ruled that Cadiz was not illegally dismissed as there was just cause for her
dismissal, that is, she engaged in premarital sexual relations with her
boyfriend resulting in a pregnancy out of wedlock.

LA further stated that her "immoral conduct x x x [was] magnified as


serious misconduct... by the fact that Brent is an institution of the Episcopal
Church in the Philippines operating both a hospital and college where
[Cadiz] was employed.

818
Page 819

Cadiz appealed to the National Labor Relations Commission (NLRC),


which affirmed the LA decision... he CA, however, dismissed her petition
outright due to technical defects in the petition: (1) incomplete statement
of material dates; (2) failure to attach registry receipts; and (3) failure to
indicate the place of issue of counsel's PTR and IBP official receipts.[11]
Cadiz sought reconsideration of the assailed CA Resolution dated July 22,
2008 but it was denied in the assailed Resolution dated February 24,
2009.[12] The CA further ruled that "a perusal of the petition will reveal
that public respondent NLRC committed no grave abuse of discretion
amounting to lack or excess of jurisdiction x x x holding [Cadiz's] dismissal
from employment valid

ISSUE:
Whether the honorable [NLRC] gravely abused its discretion
when it held that Cadiz's impregnation outside of wedlock is a ground
for the termination of employment.

RULING:
Both the LA and the NLRC upheld Cadiz's dismissal as. one
attended with just cause.

The Court, however, cannot subscribe to the labor tribunals'


conclusions.

Its Employee's Manual of Policies, meanwhile, enumerates "acts


of immorality such as scandalous behaviour, acts of lasciviousness
against any person (patient, visitors, co-workers) within hospital
premises" as a ground for discipline and discharge. Brent also relied
on Section 94 of the Manual of Regulations for Private Schools
(MRPS), which lists "disgraceful or immoral conduct" as a cause for
terminating employment.

Leus involved the same personal circumstances as the case at


bench, albeit the employer was a Catholic and sectarian educational
institution and the petitioner, Cheryl 1 Santos Leus (Leus), worked as
an assistant to the school's Director of the Lay Apostolate and
Community Outreach Directorate. Leus was dismissed from
employment by the school for having borne a child out of wedlock.
The Court ruled in Leus that the determination of whether a conduct
is disgraceful or immoral involves a two-step process: first, a
consideration of the totality of the circumstances surrounding the
conduct; and second, an assessment of the said circumstances vis-a-
vis the prevailing norms of conduct, i.e., what the society generally
considers moral and respectable.

819
Page 820

In this case, the surrounding facts leading to Cadiz's dismissal


are straightforward - she was employed as a human resources officer
in an educational and medical institution of the Episcopal Church of
the Philippines; she and her boyfriend at that time were both single;
they engaged in premarital sexual relations, which resulted into
pregnancy. The labor tribunals characterized these as constituting
disgraceful or immoral conduct. They also sweepingly concluded that
as Human Resource Officer, Cadiz should have been the epitome of
proper conduct and her indiscretion "surely scandalized the Brent
community.

Jurisprudence has already set the standard of morality with


which an act should be gauged - it is public and secular, not religious.
Whether a conduct is considered disgraceful or immoral should be
made in accordance with the prevailing norms of conduct, which, as
stated in Leus, refer to those conducts which are proscribed because
they are detrimental to conditions upon which depend the existence
and progress of human society. The fact that a particular act does not
conform to the traditional moral views of a certain sectarian institution
is not sufficient reason to qualify such act as immoral unless it,
likewise, does not conform to public and secular standards. More
importantly, there must be substantial evidence to establish that
premarital sexual relations and pregnancy out of wedlock is
considered disgraceful or immoral

The totality of the circumstances of this case does not justify the
conclusion that Cadiz committed acts of immorality. Similar to Leus,
Cadiz and her boyfriend were both single and had no legal
impediment to marry at the time she committed the alleged immoral
conduct.

Aside from these, the labor tribunals' respective conclusion that


Cadiz's "indiscretion" "scandalized the Brent community" is
speculative, at most, and there is no proof adduced by Brent to
support such sweeping conclusion.

The fact that Brent is a sectarian institution does not


automatically subject Cadiz to its religious standard of morality absent
an express statement in its manual of personnel policy and
regulations, prescribing such religious standard as gauge as these
regulations create the obligation on both the employee and the
employer to abide by the same

Brent, likewise, cannot resort to the MRPS because the Court


already stressed in Leus that "premarital sexual relations between
two consenting adults who have no impediment to marry each other,

820
Page 821

and, consequently, conceiving a child out of wedlock, gauged from a


purely public and secular view of morality, does not amount to a
disgraceful or immoral conduct under Section 94(e) of the 1992
MRPS."

WHEREFORE, the petition is GRANTED. The Resolutions dated


July 22, 2008 and February 24, 2009 of the Court of Appeals in CA-
G.R. SP No. 02373-M1N are REVERSED and SET ASIDE, and a
NEW ONE ENTERED finding petitioner Christine Joy Capin-Cadiz to
have been dismissed without just cause.

821
Page 822

Case Digests by: TANGHAL, JUAN MIGUEL P.


STERLING PAPER PRODUCTS v. KMM-KATIPUNAN
G.R. No. 221493. August 2, 2017
Mendoza, J.
Misconduct or improper behavior as just cause for dismissal
For misconduct or improper behavior to be a just cause for
dismissal, the following elements must concur: (a) the misconduct
must be serious; (b) it must relate to the performance of the
employee's duties showing that the employee has become unfit to
continue working for the employer; and (c) it must have been
performed with wrongful intent.

FACTS:
Petitioner Sterling averred that on June 26, 2010, their supervisor
Mercy Vinoya (Vinoya), found Respondent Raymond Esponga and his co-
employees about to take a nap on the sheeter machine. She called their
attention and prohibited them from taking a nap thereon for safety
reasons.

Esponga and his co-employees then transferred to the mango tree near the
staff house. When Vinoya passed by the staff house, she heard Esponga
utter, "Huwag maingay, puro bawal. " She then confronted Esponga, who
responded in a loud and disrespectful tone, "Puro kayo bawal, bakit bawal
ba magpahinga?” When Vinoya turned away, Esponga gave her the "dirty
finger" sign in front of his co-employees and said "Wala ka pala eh, puro ka
dakdak. Baka pag ako nagsalita hindi mo kayanin. "

After being served a notice to explain and several hearings, Sterling


dismissed Esponga for gross and serious misconduct. In the illegal dismissal
case filed by Esponga, the Labor Arbiter ruled in favor of Esponga, stating
that Sterling failed to discharge the burden of proof. NLRC reversed the
ruling, stating that the acts of Esponga were all violations of the Company
Code of Conduct. On appeal, the Court of Appeals reversed NLRC’s ruling,
stating that the utterances and gesture did not constitute gross
misconduct.

ISSUE:
Whether the cause of Esponga’s dismissal amounts to serious
misconduct.

RULING:
Yes.

822
Page 823

Under Article 282 (a) of the Labor Code, serious misconduct by the
employee justifies the employer in terminating his or her employment.

For misconduct or improper behavior to be a just cause for dismissal,


the following elements must concur: (a) the misconduct must be
serious; (b) it must relate to the performance of the employee's duties
showing that the employee has become unfit to continue working for
the employer; and (c) it must have been performed with wrongful
intent.

Primarily, the utterance of obscene, insulting or offensive words


against a superior is not only destructive of the morale of his co-
employees and a violation of the company rules and regulations, but
also constitutes gross misconduct. Further, Esponga's assailed
conduct was related to his work. Vinoya did not prohibit him from
taking a nap. She merely reminded him that he could not do so on the
sheeter machine for safety reasons. Esponga's acts reflect an
unwillingness to comply with reasonable management directives.

Finally, Esponga was motivated by wrongful intent. He committed the


acts in front of his co-employees, which evidently showed that he
intended to disrespect and humiliate his supervisor.

823
Page 824

Case Digests by: TANGHAL, JUAN MIGUEL P.


MARIBAGO BLUE WATER BEACH RESORT v. DUAL
G.R. No. 180660. July 20, 2010.
Perez, J.

Serious misconduct as just cause for termination


Theft committed by an employee is a serious misconduct which is a
just cause for termination under the law. Although as a rule this Court
leans over backwards to help workers and employees continue with
their employment or to mitigate the penalties imposed on them, acts of
dishonesty in the handling of company property, petitioner’s income in
this case, is a different matter.

While the Constitution is committed to the policy of social justice and


the protection of the working class, it should not be supposed that
every labor dispute will be automatically decided in favor of labor. The
management also has its own rights, as such, are entitled to respect
and enforcement in the interest of fair play.

FACTS:
Maribago Bluewater Beach Resort, Inc. (Maribago), a corporation
operating a resort hotel and restaurant in Brgy. Maribago, Lapu-Lapu
City, hired Nito Dual (Dual) as a waiter and promoted him later as
outlet cashier.

The case filed by Dual against Maribago was grounded on


Maribago’s alleged illegal dismissal of Dual for dishonesty when it
found out the discrepancy in a receipt dated January 9, 2005 at 10:00
p.m. It presented as evidence said receipt which showed that only
P3,036.00 was remitted by Dual corresponding to 6 sets of dinner
ordered by Japanese tourists, when in fact the order slip
corresponding to said receipt showed a total of 12 sets of dinner that
were ordered. It also showed evidence that it had given Dual and
other concerned employees the opportunity to explain themselves by
requesting them to attend clarificatory hearings.

Maribago argued that it only dismissed Dual after it conducted the


investigation. For his part, Dual admitted that he indeed issued the
receipt presented by Maribago as evidence but argued that the order
slip given to him was already altered to show that instead of 12 sets
of dinner, only 6 sets were served. Hence, because of the
cancellation, the guests only paid the amount reflected in the receipt
for 6 sets of dinner instead of the P10,100.00 for 12 sets. The Labor
Arbiter (LA) ruled that Dual was illegally dismissed and ordered for
Dual’s reinstatement but the NLRC reversed the LA’s decision on the

824
Page 825

ground that Dual’s act of depriving Maribago of its lawful revenue is


tantamount to fraud against the company which warrants dismissal
from the service. Falsification of commercial documents as a means
to malverse company funds constitutes fraud against the company.

ISSUE:
Whether Dual’s alleged action constitutes serious misconduct to
warrant his dismissal.

RULING:
YES.

Dual’s acts constitute serious misconduct which is a just cause for


termination under the law. Theft committed by an employee is a
serious misconduct which is a just cause for termination under the law.
Although as a rule this Court leans over backwards to help workers
and employees continue with their employment or to mitigate the
penalties imposed on them, acts of dishonesty in the handling of
company property, petitioner’s income in this case, is a different
matter.

In the present case, what was damning to the cause of Dual is the
receipt which he admittedly issued. The receipt was issued long after
the guests had left and after the alteration of the order slip was done.
Such fact led the Court to conclude that he consented and participated
in the anomaly.

While the Constitution is committed to the policy of social justice and


the protection of the working class, it should not be supposed that
every labor dispute will be automatically decided in favor of labor. The
management also has its own rights, as such, are entitled to respect
and enforcement in the interest of fair play.

825
Page 826

Case Digests by: TANGHAL, JUAN MIGUEL P.


BENITEZ v. SANTA FE MOVING AND RELOCATION SERVICES
G.R. No. 208163. April 20, 2015
Brion, J.

Serious misconduct as a just cause for termination of


employment

Serious misconduct is a just cause for termination of employment


under the law.29 Article 282 of the Labor Code provides: "An
employer may terminate an employment for any of the following
causes: (a) Serious misconduct or willful disobedience by the
employee of the lawful orders of his employer or representative in
connection with his work, x x x."

Benitez and his union stand firm on their position that he was not
liable for serious misconduct on account of his display of unruly
behavior during the company's Christmas Party on December 18,
2010 for reasons earlier discussed. On the other hand, the
respondents maintain that he committed a serious misconduct that
warranted his dismissal.

FACTS:
Benitez alleged that on December 20, 2010, the company served him a
memorandum advising him not to report for work effective immediately,
thereby terminating his employment, supposedly on grounds of serious
misconduct or willful disobedience. He allegedly uttered abusive words
against Kurangil during the company's Christmas Party on December 18,
2010. He bewailed that he was not given the opportunity to defend himself.

The incident happened in front of the company’s employees, their families,


as well as company clients and guests.

Petitioner filed a complaint for unfair labor practice and illegal dismissal,
with money claims. The LA dismissed the complaint for lack of merit, that
he is holding a position of trust and confidence, by hurling obscene,
insulting or offensive language against a superior thereby losing the trust
and confidence of his employer. Thus, he committed serious misconduct.

NLRC dismissed the appeal, likewise for lack of merit. However, it ruled that
the company failed to comply with the two-notice requirement. Hence, this
petition. Petitioner submits that he should not be dismissed for the serious
misconduct allegedly committed since it was not in connection with his

826
Page 827

work as a moving and relocation operator. Moreover, his misconduct is not


serious.

ISSUE:
Whether petitioner committed serious misconduct that would
warrant his dismissal.

RULING:
YES.

There is substantial evidence that Benitez maligned the company's


managing director and the company itself during their Christmas
Party on December 18, 2010. With such a big audience in front of
him, the court cannot imagine how Benitez could get away with his
claim that he did not malign nor disrespect Kurangil and the others.

Substantial evidence is such relevant evidence as a reasonable mind


might accept as adequate to support a conclusion, even if other
minds equally reasonable might conceivably opine otherwise.

They cite Samson v. NLRC39 as authority for their submission


that "misconduct, however serious, must nevertheless be in
connection with the employee's work to constitute just cause for his
separation.

The CA committed no reversible error in not applying


the Samson ruling in this case. Samson's outburst occurred during an
informal Christmas gathering of company sales officials and staff and
his maligned superior was not present during the gathering.

On the other hand, Benitez went up the stage and confronted his
superior with a verbal abuse. Also, the petitioners
cited Samson selectively and concealed its real thrust.

Further, it appears that in Samson, the company was ambivalent for a


while on what to do with Samson's offense as it took several weeks
after the last incident on January 3, 1994 before it asked him to
explain. Moreover, the company official maligned merely admonished
Samson during a meeting on January 4, 1994.

In contrast, the company acted swiftly and decisively in Benitez's


case, obviously and understandably, because of the gravity and high
visibility of his offense, which not only constituted a frontal verbal, and

827
Page 828

nearly physical (the attempted beer bottle throwing), assault against


Kurangil. Needless to say, Benitez's outburst also caused grave
embarrassment for the audience who witnessed the incident,
including company officials whom he likewise maligned, as well as
company clients and guests.

828
Page 829

Case Digests by: TANGHAL, JUAN MIGUEL P.


DOMINGO v. RAYALA
G.R. No. 155831. February 18, 2008.
Nachura, J.

Sexual Harassment

It is true that Sec. 3 of RA 7877 calls for a “demand, request or


requirement of a sexual favor.” But it is not necessary that the
demand, request or requirement of a sexual favor be articulated in a
categorical oral or written statement. It may be discerned, with equal
certitude, from the acts of the offender. Holding and squeezing
Domingo’s shoulders, running his fingers across her neck and tickling
her ear, having inappropriate conversations with her, giving her
money allegedly for school expenses with a promise of future
privileges, and making statements with unmistakable sexual
overtones – all these acts of Rayala resound with deafening clarity
the unspoken request for a sexual favor.

FACTS:
Ma. Lourdes T. Domingo (Domingo), then Stenographic Reporter III at
the NLRC, filed a Complaint for sexual harassment against Rayala, the
chairman of NLRC.

She alleged that Rayala called her in his office and touched her
shoulder, part of her neck then tickled her ears. Rayala argued that his acts
does not constitute sexual harassment because for it to exist, there must be
a demand, request or requirement of sexual favor.

ISSUE:
Whether or not Rayala commit sexual harassment.

RULING:
Yes.

The law penalizing sexual harassment in our jurisdiction is RA


7877. Section 3 thereof defines work-related sexual harassment in
this wise:

Sec. 3. Work, Education or Training-related Sexual Harassment


Defined. – Work, education or training-related sexual harassment is
committed by an employer, manager, supervisor, agent of the
employer, teacher, instructor, professor, coach, trainor, or any other
person who, having authority, influence or moral ascendancy over

829
Page 830

another in a work or training or education environment, demands,


requests or otherwise requires any sexual favor from the other,
regardless of whether the demand, request or requirement for
submission is accepted by the object of said Act.

(a) In a work-related or employment environment, sexual


harassment is committed when:

(1) The sexual favor is made as a condition in the hiring or in the


employment, re-employment or continued employment of said
individual, or in granting said individual favorable compensation,
terms, conditions, promotions, or privileges; or the refusal to grant the
sexual favor results in limiting, segregating or classifying the
employee which in a way would discriminate, deprive or diminish
employment opportunities or otherwise adversely affect said
employee;

. (2) The above acts would impair the employee’s rights or


privileges under existing labor laws; or

. (3) The above acts would result in an intimidating, hostile, or


offensive environment for the employee.

Even if we were to test Rayala’s acts strictly by the standards set


in Section 3, RA 7877, he would still be administratively liable. It is
true that this provision calls for a “demand, request or requirement of
a sexual favor.” But it is not necessary that the demand, request or
requirement of a sexual favor be articulated in a categorical oral or
written statement. It may be discerned, with equal certitude, from the
acts of the offender. Holding and squeezing Domingo’s shoulders,
running his fingers across her neck and tickling her ear, having
inappropriate conversations with her, giving her money allegedly for
school expenses with a promise of future privileges, and making
statements with unmistakable sexual overtones – all these acts of
Rayala resound with deafening clarity the unspoken request for a
sexual favor.

830
Page 831

Case Digests by: TANGHAL, JUAN MIGUEL P.


COCA COLA EXPORT CORP. v. GACAYAN
G.R. No. 149433. June 22, 2011
Leonardo-Castro, J.

Loss of trust and confidence constitutes a just and valid cause


for an employee’s termination

It has oft been held that loss of confidence should not be used as a
subterfuge for causes which are illegal, improper and unjustified. It
must be genuine, not a mere afterthought to justify an earlier action
taken in bad faith. It bears stressing that what is at stake here are the
sole means of livelihood, the name and the reputation of the
employee. Thus, petitioner company must sufficiently and
convincingly show that the loss of trust and confidence in respondent
Gacayan was founded on clearly established facts, incidents and
substantial evidence.

Although the amounts involved in the subject receipts were relatively


small, or only the dates and/or items ordered were altered or
tampered with, respondent Gacayan act of submitting fraudulent
items of expense adversely reflected on her integrity and honesty,
which is ample basis for petitioner company to lose its trust and
confidence in her.

FACTS:
Employees of Coca-Cola were allowed reimbursement of meal and
transportation expenses incurred when the employee worked overtime for
at least four hours on a Saturday, Sunday, or holiday, and for at least two
hours on weekdays. It was in connection with this company policy that
respondent Gacayan, then a Senior Financial Accountant, was made to
explain the alleged alterations in three (3) receipts which she submitted to
support her claim for reimbursement of meal expenses, to wit: 1)
McDonald Receipt No. 875493 dated October 1, 1994 for P111.00; 2)
Shakey Pizza Parlor Receipt No. 122658 dated November 20, 1994
forP174.06; and 3) Shakey Pizza Parlor Receipt No. 41274 dated July 19,
1994 for P130.50.

Petitioner company sent respondent Gacayan several memoranda requiring


her to explain why her claims for reimbursement should not be considered
fraudulent since there were alterations in the dates and prices thereof.
Consequently, respondent Gacayan submitted her explanation denying any
personal knowledge in the commission of the alterations on the subject
receipts.

831
Page 832

Petitioner company then conducted a hearing and formal investigation,


such eventually led to the dismissal of Gacayan for fraudulently submitting
tampered and/or altered receipts in support of her petty cash
reimbursements in gross violation of the company rules and regulations.

On June 6, 1995, respondent Gacayan filed a complaint with the NLRC. In a


Decision dated June 17, 1996, the Labor Arbiter dismissed respondent
Gacayan complaint for lack of merit. This was affirmed by the NLRC in its
Resolution dated April 14, 1998.

On appeal, the Court of Appeals reversed the NLRC and ruled that the
penalty imposed on respondent Gacayan was too harsh. The Court of
Appeals ordered the immediate reinstatement of respondent Gacayan to
her former position or to a substantially equivalent position without loss of
seniority rights and with full backwages.

In a petition for review on certiorari, the Supreme Court denied Coca-cola


petition and ruled that Clarita P. Gacayan was illegally dismissed from her
employment with petitioner company.

Petitioner company now begs the Supreme Court to reconsider this


pronouncement, arguing that respondent Gacayan position as a "Senior
Financial Accountant with the Job Description of a Financial Project
Analyst" has duties which clearly qualify her as one occupying a position of
trust and responsibility.

ISSUE:
Whether or not Gacayan occupies a position of trust and
responsibility.

RULING:
This Motion for Reconsideration is impressed with merit.

It is well-settled that loss of trust and confidence constitutes a just


and valid cause for an employee termination.

Respondent Gacayan was the Senior Financial Accountant of


petitioner company. While respondent Gacayan denies that she is
handling or has custody of petitioner funds, a re-examination of the
records of this case reveals that she indeed handled delicate and
confidential matters in the financial analyses and evaluations of the
action plans and strategies of petitioner company. Respondent
Gacayan was also privy to the strategic and operational decision-
making of petitioner company, a sensitive and delicate position

832
Page 833

requiring the latter utmost trust and confidence. As such, she should
be considered as holding a position of responsibility or of trust and
confidence.

The findings of the Labor Arbiter, as affirmed by the NLRC indicates


that respondent Gacayan betrayed the trust and confidence reposed
on her when she, ironically a Senior Financial Accountant tasked with
ensuring financial reportorial/regulatory compliance from others,
repeatedly submitted tampered or altered receipts to support her
claim for meal reimbursements, in gross violation of the rules and
regulations of petitioner company. Upon review, even the Court of
Appeals did not absolve respondent Gacayan of wrongdoing but
rather merely held that dismissal was too harsh a penalty for her
infraction.

It has oft been held that loss of confidence should not be used as a
subterfuge for causes which are illegal, improper and unjustified. It
must be genuine, not a mere afterthought to justify an earlier action
taken in bad faith. It bears stressing that what is at stake here are the
sole means of livelihood, the name and the reputation of the
employee. Thus, petitioner company must sufficiently and
convincingly show that the loss of trust and confidence in respondent
Gacayan was founded on clearly established facts, incidents and
substantial evidence.

In its motion for reconsideration, petitioner company emphasized the


clear and convincing evidence on record that respondent Gacayan
breached the trust and confidence reposed in her when she
repeatedly submitted tampered or altered receipts to support her
claim for meal reimbursement. Petitioner company maintained that
respondent Gacayan cannot mistakenly file a claim for overtime meal
allowance reimbursement for a day she knew she was not entitled to,
as she did not actually render overtime work. Petitioner company
reiterated its evidence showing that respondent Gacayan acted with
wrongful, malicious and fraudulent intent when she repeatedly
submitted tampered or altered receipts.

Respondent Gacayan intentionally, knowingly, purposely, and without


justifiable excuse, submitted tampered or altered receipts to support
her claim for meal reimbursement. Respondent Gacayan failed to
sufficiently refute the charges against her for the submission of said
fraudulent items of expense. All she did was to deny any personal
knowledge in the commission of the alterations in the subject receipts
and to point fingers at other people who may have done the
alterations.

833
Page 834

Although the amounts involved in the subject receipts were relatively


small, or only the dates and/or items ordered were altered or
tampered with, respondent Gacayan act of submitting fraudulent
items of expense adversely reflected on her integrity and honesty,
which is ample basis for petitioner company to lose its trust and
confidence in her.

834
Page 835

Case Digests by: TANGHAL, JUAN MIGUEL P.


HOLCIM PHILIPPINES v. RENANTE OBRA
G.R. No. 220998. August 8, 2016
Renante, J.

Misconduct as a just cause for dismissal

To constitute a valid cause for dismissal within the text and meaning
of Art. 282 (now Article 297) of the Labor Code, the employee’s
misconduct must be serious, i.e., of such grave and aggravated
character and not merely trivial or unimportant. As in this case, where
the item respondent tried to take out was practically of no value to
petitioner. Moreover, ill will or wrongful intent cannot be ascribed to
respondent, considering that, while he asked Castillo not to report the
incident to the management, he also volunteered the information that
he had a piece of scrap wire in his bag and offered to return it if the
same could not possible be brought outside the company premises
sans a gate pass.

FACTS:
Renante Obra was employed as packhouse operator by Holcim which
ensures the safety and efficient operation of rotopackers, auto bag placers
and cariramats, as well as their auxiliaries. On July 10, 2013, at around 4
o'clock in the afternoon, respondent was about to exit Gate 2 of
petitioner's La Union Plant when the security guard on duty, Kristian
Castillo (Castillo), asked him to submit himself and the backpack he was
carrying for inspection. Respondent refused and confided to Castillo that he
has a piece of scrap electrical wire in his bag. He also requested Castillo not
to report the incident to the management, and asked the latter if
respondent could bring the scrap wire outside the company premises;
otherwise, he will return it to his locker in the Packhouse Office. However,
Castillo did not agree, which prompted respondent to turn around and
hurriedly go back to the said office where he took the scrap wire out of his
bag. Soon thereafter, a security guard arrived and directed him to go to the
Security Office where he was asked to write a statement regarding the
incident. On July 16, 2013, respondent received a Notice of Gap20 requiring
him to explain within five (5) days therefrom why no disciplinary action,
including termination, should be taken against him on account of the
above-mentioned incident.21 He was also placed on preventive suspension
for thirty (30) days effective immediately.

On August 8, 2013, petitioner issued a Decision/Resolution Memo


dismissing from service respondent for serious misconduct. Petitioner

835
Page 836

found no merit in respondent's claim that he was unaware that a gate pass
is required to take out a piece of scrap wire, pointing out that the same is
incredulous since he had been working thereat for nineteen (19) years
already. It also drew attention to the fact that respondent refused to
submit his bag for inspection, which, according to petitioner, confirmed his
intention to take the wire for his personal use. Further, petitioner
emphasized that respondent's actions violated its rules which, among
others, limit the use of company properties for business purposes only and
mandate the employees, such as respondent, to be fair, honest, ethical,
and act responsibly and with integrity.

Respondent filed a complaint before the NLRC for illegal dismissal and
money claims.

ISSUE:
Whether Obra was illegally dismissed.

RULING:
YES.

As correctly observed by the NLRC, while there is no dispute that


respondent took a piece of wire from petitioner’s La Union Plant and
tried to bring it outside the company premises, he did so in the belief
that the same was already for disposal, petitioner did not suffer any
damage from the incident, given that after being asked to submit
himself and his bag for inspection, respondent had a change of heart
and decided to just return the wire to the office. Respondent also
showed remorse for his mistake, pleading repeatedly with petitioner
to reconsider the penalty imposed upon him.

As in the foregoing cases, herein respondent deserves compassion


and humane understanding more than condemnation, especially
considering that he had been in petitioner’s employ for 19 years
already and this is the first time that he had been involved in taking
company property, which item, at the end of the day, is practically of
no value. Besides, respondent did not occupy a position of trust and
confidence, the loss of which would have justified his dismissal over
the incident.

Neither can respondent’s infraction can be characterized as a serious


misconduct which, under Article 282 (now Article 297) of the Labor
Code is as a just cause for dismissal.

To constitute a valid cause for dismissal within the text and meaning
of Art. 282 (now Article 297) of the Labor Code, the employee’s

836
Page 837

misconduct must be serious, i.e., of such grave and aggravated


character and not merely trivial or unimportant. As in this case, where
the item respondent tried to take out was practically of no value to
petitioner. Moreover, ill will or wrongful intent cannot be ascribed to
respondent, considering that, while he asked Castillo not to report the
incident to the management, he also volunteered the information that
he had a piece of scrap wire in his bag and offered to return it if the
same could not possible be brought outside the company premises
sans a gate pass.

837
Page 838

Case Digests by: TANGHAL, JUAN MIGUEL P.


WATERFRONT CEBU v LEDESMA
G.R. No. 197556. March 25, 2015
Villarama, Jr., J.

Disregard of procedural flaw serves substantial justice

There is no substantial justice that may be served here in


disregarding the procedural flaw committed by Ledesma because the
NLRC correctly found him guilty of misconduct or improper behavior
in committing lascivious conduct and demanding sexual favors from
Christe Mandal and Rosanna Lofranco.

The CA ruled in favor of Ledesma since it believed his version that


the complainants merely invented the accusations against him
because Waterfront failed to present as evidence the CCTV footages
of the alleged lascivious conduct of Ledesma inside the elevator and
the conference room. But this argument was not even raised by
Ledesma himself and it was only the CA which utilized this as a
justification to bolster its findings that Ledesma did not commit any
infraction. This being a labor case, the evidence required is only
substantial evidence which was adequately established here by the
positive and credible testimonies of the complainants.

FACTS:
Respondent was employed as a House Detective at Waterfront located
at Salinas Drive, Cebu City.

On the basis of the complaints filed before Waterfront by Christe6


Mandal, a supplier of a concessionaire of Waterfront, and Rosanna
Lofranco, who was seeking a job at the same hotel, Ledesma was dismissed
from employment.7 From the affidavits8 and testimonies9 of Christe
Mandal and Rosanna Lofranco during the administrative hearings
conducted by Waterfront, the latter found, among others, that Ledesma
kissed and mashed the breasts of Christe Mandal inside the hotel’s
elevator, and exhibited his penis and asked Rosanna Lofranco to
masturbate him at the conference room of the hotel.

On August 12, 2008, Ledesma filed a complaint10 for illegal dismissal


which was docketed as NLRC RAB-VII Case No. 08-1887-08. The LA found
that the allegations leveled against Ledesma are mere concoctions, and
concluded that Ledesma was illegally dismissed.

838
Page 839

ISSUE:
Whether the dismissal of Ledesma is valid

RULING:
YES

There is no substantial justice that may be served here in


disregarding the procedural flaw committed by Ledesma because the
NLRC correctly found him guilty of misconduct or improper behavior
in committing lascivious conduct and demanding sexual favors from
Christe Mandal and Rosanna Lofranco.

The CA ruled in favor of Ledesma since it believed his version that


the complainants merely invented the accusations against him
because Waterfront failed to present as evidence the CCTV footages
of the alleged lascivious conduct of Ledesma inside the elevator and
the conference room. But this argument was not even raised by
Ledesma himself and it was only the CA which utilized this as a
justification to bolster its findings that Ledesma did not commit any
infraction. This being a labor case, the evidence required is only
substantial evidence which was adequately established here by the
positive and credible testimonies of the complainants.

Notably, Ledesma never refuted, at the administrative investigation


level at Waterfront, and even at the proceedings before the LA,
NLRC, and the CA, the allegations leveled against him by Rosanna
Lofranco that, after deluding her to perform a massage on him,
Ledesma exhibited to her his penis and requested that he be
masturbated while inside the conference room of the hotel. If not for
the position of Ledesma as a House Detective, he will not have
access to the conference room nor will he know that the premises is
not monitored through a closed-circuit television, thus giving him the
untrammeled opportunity to accomplish his lewd design on the
unsuspecting victim. Such acts of Ledesma constituted misconduct
or improper behavior which is a just cause for his dismissal.

839
Page 840

Case Digests by: TANGHAL, JUAN MIGUEL P.


CESAR NAGUIT v. SAN MIGUEL CORPORATION
G.R. No. 188839. June 22, 2015
Peralta, J

Fighting as a valid cause for dismissal

The settled rule is that fighting within company premises is a valid


ground for the dismissal of an employee. Moreover, the act of
assaulting another employee is serious misconduct which justifies the
termination of employment. Where the totality of the evidence was
sufficient to warrant the dismissal of the employees, the law warrants
their dismissal without making any distinction between a first offender
and a habitual delinquent.

FACTS:
Petitioner was an employee of San Miguel Corporation Metal Closure
and Lithography Plant, a division of Respondent Corporation (SMC).
Sometime in 23 September 2002, Renato Regala and Petitioner got
involved in an altercation in Respondent Corporation’s Canlubang Plant.
Petitioner claims that the altercation sprung from an event when Regala
distributed libellous materials against the union which Petitioner is a union
steward. Upon investigation of the Respondent Company’s Human
Resource Department, petitioner chose to remain silent and did not
address the charges against him. He was later terminated.

LA: The Labor Arbiter dismissed Petitioner’s complaint for illegal dismissal
for lack of merit.

NLRC: The NLRC dismissed the Petitioner’s appeal and affirmed the
Decision of the Labor Arbiter.

ISSUE:
Whether or not Petitioner had been illegally dismissed and is entitled
to reinstatement and full back wages.

RULING:
NO. As noted by the Labor Arbiter, other than his bare allegations,
petitioner did not submit proof to support his allegations nor did he
provide evidence to counter those which were submitted by
respondent. The Supreme Court did not agree with petitioner's
argument that the penalty of dismissal imposed upon him is too harsh
and is not commensurate to the infraction he has committed,
considering that he has been in respondent's employ for fifteen years

840
Page 841

and that this is just his first offense of this nature. The settled rule is
that fighting within company premises is a valid ground for the
dismissal of an employee. Moreover, the act of assaulting another
employee is serious misconduct which justifies the termination of
employment. Where the totality of the evidence was sufficient to
warrant the dismissal of the employees, the law warrants their
dismissal without making any distinction between a first offender and
a habitual delinquent.

841
Page 842

Case Digests by: TANGHAL, JUAN MIGUEL P.


ADAMSON UNIVERSITY FACULTY v ADAMSON UNIVERSITY
G.R. No. 227070. March 9, 2020
Leonen, J.

Casual expression of surprise for serious misconduct

The use of expletives as a casual expression of surprise or


exasperation is not serious misconduct per se that warrants an
employee's dismissal. However, the employee's subsequent acts
showing willful and wrongful intent may be considered in determining
whether there is a just cause for their employment termination.

FACTS:
Delos Reyes was a university professor and the assistant chairperson
of the Social Sciences Department of Adamson University (Adamson). He
was also the president of the Adamson University Faculty and Employees
Union (the Union), a duly registered labor union and the sole and exclusive
bargaining agent of Adamson's faculty and non-academic personnel.

Paula Mae was holding the doorknob on her way out of the office, while
Delos Reyes held the doorknob on the other side. When Paula Mae stepped
aside, Delos Reyes allegedly exclaimed the words "anak ng puta" and
walked on without any remorse. This caused emotional trauma to Paula
Mae.

ISSUE:
Whether the expression justifies the dismissal of the employee

RULING:
NO.

Misconduct is not considered serious or grave when it is not


performed with wrongful intent. If the misconduct is only simple, not
grave, the employee cannot be validly dismissed.[76]

A teacher exclaiming "anak ng puta" after having encountered a


student is an unquestionable act of misconduct. However, whether it
is serious misconduct that warrants the teacher's dismissal will
depend on the context of the phrase's use. "Anak ng puta" is similar
to "putang ina" in that it is an expletive sometimes used as a casual
expression of displeasure, rather than a personal attack or insult.

842
Page 843

A review of the records reveals that the utterance in question, "anak


ng puta," was an expression of annoyance or exasperation. Both
petitioner and Paula Mae were pulling from each side of the door,
prompting the professor to exclaim frustration without any clear intent
to maliciously damage or cause emotional harm upon the student.
That they had not personally known each other before the incident,
and that petitioner had no personal vendetta against Paula Mae as to
mean those words to insult her, confirm this conclusion.

843
Page 844

Case Digests by: TANGHAL, JUAN MIGUEL P.


COFFEE BEAN AND TEA LEAF v. ARENAS
G.R. No. 208908. March 11, 2015
Brion, J.

Willful disobedience as valid cause for dismissal

For willful disobedience to be a valid cause for dismissal, these two


elements must concur: (1) the employee’s assailed conduct must
have been willful, that is, characterized by a wrongful and perverse
attitude; and (2) the order violated must have been reasonable,
lawful, made known to the employee, and must pertain to the duties
which he had been engaged to discharge.

FACTS:
On April 1, 2008, the Coffee Bean and Tea Leaf Philippines, Inc. (CBTL) hired
Rolly P. Arenas (Arenas) to work as a "barista" at its Paseo Center Branch.
His principal functions included taking orders from customers and
preparing their ordered food and... beverages. Upon signing the
employment contract, Arenas was informed of CBTL's existing employment
policies.

After inspection, Basallo prepared a store manager's report which listed


Arenas' recent infractions, as follows:

Leaving the counter unattended and eating chips in an unauthorized area


while on duty (March 30, 2009);

Reporting late for work on several occasions (April 1, 3 and 22); and

Placing an iced tea bottle in the ice bin despite having knowledge of
company policy prohibiting the same (April 28, 2009).

Based on the mystery guest shopper and duty manager's reports, Arenas
was required to explain his alleged violations. However, CBTL found Arenas'
written explanation unsatisfactory, hence CBTL terminated his
employment.

Arenas filed a complaint for illegal dismissal.

LA RULING:. The LA ruled in his favor, declaring that he had been illegally
dismissed.

844
Page 845

NLRC RULING:

On appeal, the NLRC affirmed the LA's decision.

CBTL filed a petition for certiorari under Rule 65 before the CA. CBTL
insisted that Arenas' infractions amounted to serious misconduct or willful
disobedience, gross and habitual neglect of duties, and breach of trust and
confidence. To support these allegations,... CBTL presented Arenas' letter
where he admitted his commission of the imputed violations.

CA RULING... the CA issued its decision dismissing the petition. The CA


ruled that Arenas' offenses fell short of the required legal standards to
justify his dismissal; and that these do not constitute serious misconduct or
willful disobedience, and gross... negligence, to merit his termination from
service.

ISSUE:
Whether Arenas’ act constitutes a valid dismissal

RULING:
NO.

For willful disobedience to be a valid cause for dismissal, these two


elements must concur: (1) the employee’s assailed conduct must
have been willful, that is, characterized by a wrongful and perverse
attitude; and (2) the order violated must have been reasonable,
lawful, made known to the employee, and must pertain to the duties
which he had been engaged to discharge.

Tested against these standards, it is clear that Arenas’ alleged


infractions do not amount to such a wrongful and perverse attitude.
Though Arenas may have admitted these wrongdoings, these do not
amount to a wanton disregard of CBTL’s company policies. As
Arenas mentioned in his written explanation, he was on a scheduled
break when he was caught eating at CBTL’s al fresco dining area.
During that time, the other service crews were the one in charge of
manning the counter. Notably, CBTL’s employee handbook imposes
only the penalty of written warning for the offense of eating non-CBTL
products inside the store’s premises.

CBTL also imputes gross and habitual neglect of duty to Arenas for
coming in late in three separate instances.

845
Page 846

Gross negligence implies a want or absence of, or failure to exercise


even a slight care or diligence, or the entire absence of care. It
evinces a thoughtless disregard of consequences without exerting
any effort to avoid them.18 There is habitual neglect if based on the
circumstances, there is a repeated failure to perform one’s duties for
a period of time.

In light of the foregoing criteria, we rule that Arenas’ three counts of


tardiness cannot be considered as gross and habitual neglect of duty.
The infrequency of his tardiness already removes the character of
habitualness. These late attendances were also broadly spaced out,
negating the complete absence of care on Arenas’ part in the
performance of his duties. Even CBTL admitted in its notice to explain
that this violation does not merit yet a disciplinary action and is only
an aggravating circumstance to Arenas’ other violations.

To further justify Arenas’ dismissal, CBTL argues that he committed


serious misconduct when he lied about using the ice bin as cooler for
his bottled iced tea. Under CBTL’s employee handbook, dishonesty,
even at the first instance, warrants the penalty of termination from
service.

For misconduct or improper behavior to be a just cause for dismissal,


(a) it must be serious; (b) it must relate to the performance of the
employee’s duties; and (c) it must show that the employee has
become unfit to continue working for the employer.

846
Page 847

Case Digests by: TANGHAL, JUAN MIGUEL P.


LORES REALTY v PACIA
G.R. No. 171189. March 9, 2011
Mendoza, J.

Requisites for willful disobedience

The offense of willful disobedience requires the concurrence of two


(2) requisites:

(1) the employee's assailed conduct must have been willful, that is
characterized by a wrongful and perverse attitude; and

(2) the order violated must have been reasonable, lawful, made
known to the employee and must pertain to the duties which he had
been engaged to discharge.

FACTS:
Respondent Virginia E. Pacia(Pacia)was hired by Lores Realty Enterprises,
Inc. (LREI). At the time of her dismissal, she was the assistant manager and
officer-in-charge of LREI's Accounting Department under the Finance
Administrative Division.

LREI's acting general manager, Sumulong, ordered Pacia to prepare a check


amounting to P150,000.00, an order which she was slow to follow. Pacia
eventually complied with the order, nevertheless. Pacia was ordered to
prepare another check, this time amounting to P175,000.00. She was again
slow to comply, but eventually the order was complied with. To explain her
refusal to immediately follow the directive, Pacia reasoned out that the
funds in LREIs account were not sufficient to cover the amounts to be
indicated in the checks.

Thereafter Pacia received a notice of termination stating, among others,


that she was being dismissed because of her willful disobedience and their
loss of trust and confidence in her.

Pacia then filed a Complaint for Unfair Labor Practice due to Harassment,
Constructive Dismissal, Moral and Exemplary Damages against LREI and
Sumulong. Subsequently, Pacia filed an Amended Complaint to include the
charges of illegal dismissal and non-payment of salaries. The Labor
Arbiter(LA)rendered a decision finding that the dismissal of Pacia was for a

847
Page 848

just and valid cause but ordering payment of what was due her.On appeal,
the NLRC in its decision reversed the LA's Decision and found LREI and
Sumulong guilty of illegal dismissal. The case was elevated to the CA. The
CA held that LREI and Sumulong failed to establish with substantial
evidence that the dismissal of Pacia was for a just cause.It found that Pacias
initial reluctance to obey the orders of her superiors was for a good reason
- to shield the company from liability in the event that the checks would be
dishonored for insufficiency of funds.

ISSUE:
Whether or not Pacias termination was justified under the
circumstances

RULING:
The petition has no merit.

At the outset, it must be emphasized that the issues raised in this


petition are questions of fact which are not proper subjects of an
appeal by certiorari.A disharmony between the factual findings of the
LA and the NLRC, however,opens the door to a review by this
Court.Factual findings of administrative agencies are not infallible and
will be set aside when they fail the test of arbitrariness. Moreover,
when the findings of the NLRC contradict those of the LA, this Court,
in the exercise of its equity jurisdiction, may look into the records of
the case and re-examine the questioned findings.

Article 282 of the Labor Code enumerates the just causes for which
an employer may terminate the services of an employee, to wit:

(a)Serious misconduct or willful disobedience by the employee of the


lawful orders of his employer or representative in connection with his
work;

(b)Gross and habitual neglect by the employee of his duties;

(c)Fraud or willful breach by the employee of the trust reposed in him


by his employer or duly authorized representative;

(d)Commission of a crime or offense by the employee against the


person of his employer or any immediate member of his family or his
duly authorized representative; and

(e)Other causes analogous to the foregoing.

848
Page 849

The offense of willful disobedience requires the concurrence of two


(2) requisites:

(1) the employee's assailed conduct must have been willful, that is
characterized by a wrongful and perverse attitude; and

(2) the order violated must have been reasonable, lawful, made
known to the employee and must pertain to the duties which he had
been engaged to discharge.

The Court finds nothing unlawful in the directive of Sumulong to


prepare checks in payment of LREI's obligations. The availability or
unavailability of sufficient funds to cover the check is immaterial in the
physical preparation of the checks. Pacias initial reluctance to
prepare the checks, however, which was seemingly an act of
disrespect and defiance, was for honest and well intentioned reasons.
Protecting LREI and Sumulong from liability under the Bouncing
Checks Law. was foremost in her mind.It was not wrongful or willful.
Neither can it be considered an obstinate defiance of company
authority.The Court takes into consideration that Pacia, despite her
initial reluctance, eventually did prepare the checks on the same day
she was tasked to do it. Pacias apprehension was justified when the
check was dishonored.This clearly affirms her assertion that she was
just being cautious and circumspect for the company's sake.Thus,
her actuation should not be construed as improper conduct.

Petition is DENIED.

849
Page 850

Case Digests by: TANGHAL, JUAN MIGUEL P.


PACIFIC v. CABANSAY
G.R. No. 167345. November 23, 2007
Nachura, J.

Refusal to comply with lawful order as valid cause for dismissal

Established in our labor law jurisprudence is the principle that while


compassion and human consideration should guide the disposition of
cases involving termination of employment, since it affects one's
means of livelihood, it should not be overlooked that the benefits
accorded to labor do not include compelling an employer to retain the
services of an employee who has been shown to be a gross liability
to the employer.

Willful disobedience or insubordination necessitates the concurrence


of at least two requisites: (1) the employee's assailed conduct must
have been willful, that is, characterized by a wrongful and perverse
attitude; and (2) the order violated must have been reasonable,
lawful,... made known to the employee and must pertain to the duties
which he had been engaged to discharge.

Indeed, by refusing to postpone the presentation and implementation


of the new training process, respondent intentionally, knowingly and
purposely, without justifiable excuse, breached the trust and
confidence reposed in her by her employer.

FACTS:
Respondent Ma. Lourdes Cabansay (Cabansay) was hired as Senior Traning
Manager of ePacific Global Contact Center, Inc. with a monthly salary of
P38,000.00 on April 18, 2001[4] and became a regular employee on August
1, 2001. In

March 2002, respondent was tasked to prepare a new training process for
the company's Telesales Trainees

After reviewing the training module prepared by respondent, Mr. Rosendo


S. Ballesteros (Ballesteros), the company's Senior Vice President-Business

850
Page 851

Development Group, found that the same did not contain any changes and
that they were not ready to present it.[6] He thus instructed respondent
through an electronic mail (e-mail) to postpone the presentation and the
implementation of the new training process.[7] Ballesteros further
emphasized that the Department needed more time to teach the trainees
on... how to get leads, focus on developing their telemarketing skills and
acquire proper motivation.

In response to Ballesteros's e-mail instructions, Cabansay wrote, also via e-


mail,... This is a very simple presentation and I WILL NOT POSTPONE it
today, it's very easy to comprehend and as per YOUR INSTRUCTION we will
be implementing it next week, so when should we present this to the TLs?

Let's not make SIMPLE THINGS COMPLICATED.

I will go on with the presentation this afternoon.

Adversely reacting to respondent's attitude, Ballesteros sent Cabansay a


memo on April 6, 2002, informing the latter that he found her message to
be a clear act of insubordination, causing him to lose his trust and
confidence in her as Manager of the Training Department. He then asked
respondent to explain in writing why she should not be terminated as a
consequence of her acts... no presentation of the training module was
made on April 5, 2002 because the Senior Manager for Telesales, Ms. Lorna
Garcia, on instruction of Ballesteros, informed all the participants that the
same was postponed because Management was not yet ready to present
the... module.[... on April 11, 2002, the same day she submitted her second
explanation, Cabansay received a memorandum from the HR
Department/Office of the President notifying her that she had been
terminated from the service effective immediately for having committed an
act of... insubordination resulting in the management's loss of trust and
confidence in her.[

Respondent, thus, filed a case for illegal dismissal

On September 2, 2002, Labor Arbiter (LA) Madjayran H. Ajan rendered his


Decision dismissing the complaint. The Labor Arbiter ruled that reading
Cabansay's e-mail message between the lines would clearly show that she
willfully disobeyed the order of Ballesteros.

On January 10, 2005, the appellate court rendered its Decision[30] granting
the petition. The CA ruled that Cabansay's termination could be justified

851
Page 852

neither by insubordination nor loss of trust and confidence. A perusal of the


e-mail instructions sent by

Ballesteros to her would show that, although the alleged order to postpone
the presentation of the training module was reasonable and lawful, it was
not clearly made known to her. The phrase "I don't think [we are ready to
present this to all TL]" could not be deemed an... order as it merely
suggested an opinion.[31] Moreover, the e-mail reply of Cabansay cannot
be considered an act of willful defiance or insubordination. The language
used was not harsh and no rude remarks or demeaning statements were
made. She was only... explaining her view on the matter, which could not
be considered unlawful considering that she was also a managerial
employee clothed with discretionary powers. Clearly, her acts did not
constitute the "wrongful and perverse attitude" that otherwise would
sanction dismissal. And... even if she were guilty of insubordination, such
minor infraction should not merit the ultimate and supreme penalty of
dismissal.

ISSUE:
Whether or not respondent Cabansay was illegally dismissed.

RULING:

After a careful review of the records and considering the arguments


of the parties, the Court finds the petition impressed with merit.

In the instant case, we find that the labor tribunal did not arbitrarily
disregard or misapprehend the evidence. Its finding that respondent
was validly dismissed is likewise warranted by substantial evidence.
Thus, we agree with petitioner's stance that the findings of the LA,...
as affirmed by the NLRC, should not have been set aside by the
appellate court. Deference to the expertise acquired by the labor
tribunal and the limited scope granted in the exercise of certiorari
jurisdiction restrain any probe into the correctness of the LA's and the

NLRC's evaluation of evidence.

Willful disobedience or insubordination necessitates the concurrence


of at least two requisites: (1) the employee's assailed conduct must
have been willful, that is, characterized by a wrongful and perverse
attitude; and (2) the order violated must have been reasonable,
lawful,... made known to the employee and must pertain to the duties
which he had been engaged to discharge.[43] On the other hand,
loss of trust and confidence, to be a valid ground for dismissal, must

852
Page 853

be based on a willful breach of trust and founded on clearly...


established facts. A breach is willful if it is done intentionally,
knowingly and purposely, without justifiable excuse, as distinguished
from an act done carelessly, thoughtlessly, heedlessly or
inadvertently. It must rest on substantial grounds and not on the
employer's... arbitrariness, whims, caprices or suspicion; otherwise,
the employee would eternally remain at the mercy of the employer.
Loss of confidence must not also be indiscriminately used as a shield
by the employer against a claim that the dismissal of an employee
was arbitrary. And,... in order to constitute a just cause for dismissal,
the act complained of must be work-related and show that the
employee concerned is unfit to continue working for the employer.

In the case at bar, the reasonableness and lawfulness of


Ballesteros's order is not in question, so is its relation to the duties of
respondent. What is disputed herein is rather its clarity. Respondent
Cabansay contends that the directive was not clearly made known...
to her: Ballesteros's order was to postpone the implementation but
not the presentation of the new training process/module to the team
leaders.

While respondent Cabansay was a managerial employee, a Senior


Training Manager entrusted with the delicate matter of molding the
minds and characters of call center agents and team leaders, and
clothed with discretion to determine what was in the best interest of
the company,... her managerial discretion was not without limits. Its
parameters were contained the moment her discretion was exercised
and then opposed by the immediate superior officer/employer for
being against the policies and welfare of the company. Hence, any
action in pursuit of the... discretion thus opposed ceased to be
discretionary and could be considered as willful disobedience.[45]

Indeed, by refusing to postpone the presentation and implementation


of the new training process, respondent intentionally, knowingly and
purposely, without justifiable excuse, breached the trust and
confidence reposed in her by her employer. To present and discuss a
training... module, which is deemed by management as still
inadequate in its content, will certainly not only waste the time, effort
and energy of the participants in the discussion but will also entail
losses on the part of the company.

853
Page 854

Case Digests by: TANGHAL, JUAN MIGUEL P.


TONGKO v. THE MANUFACTURER’S LIFE
G.R. No. 167622. January 25, 2011
Velasco, Jr., J.

“Laggard performance” does not justify dismissal on the


grounds of gross and habitual neglect of duty

When there is no showing of a clear, valid and legal cause for the
termination of employment, the law considers the matter a case of
illegal dismissal and the burden is on the employer to prove that the
termination was for a valid or authorized cause. This burden of proof
appropriately lies on the shoulders of the employer and not on the
employee because a worker's job has some of the characteristics of
property rights and is therefore within the constitutional mantle of
protection. No person shall be deprived of life, liberty or property
without due process of law, nor shall any person be denied the equal
protection of the laws.

FACTS:
Manufacturers Life Insurance, Co. is a domestic corporation engaged in life
insurance business. De Dios was its President and Chief Executive Officer.
Petitioner Tongko started his relationship with Manulife in 1977 by virtue of
a Career Agent's Agreement.

De Dios subsequently sent Tongko a letter of termination in accordance


with Tongko's Agents Contract. Tongko filed a complaint with the NLRC
against Manulife for illegal dismissal, alleging that he had an employer-
employee relationship with De Dios instead of a revocable agency by
pointing out that the latter exercised control over him through directives
regarding how to manage his area of responsibility and setting objectives
for him relating to the business. Tongko also claimed that his dismissal was
without basis and he was not afforded due process.

Manulife argues that Tongkos’s employment was validly terminated on the


ground of gross and habitual neglect of duties, inefficiency, as well as willful
disobedience of the lawful orders of Manulife. Manulife stated:

In the instant case, private respondent, despite the written reminder from
Mr. De Dios refused to shape up and altogether disregarded the latter's
advice resulting in his laggard performance clearly indicative of his willful
disobedience of the lawful orders of his superior.

854
Page 855

As private respondent has patently failed to perform a very fundamental


duty, and that is to yield obedience to all reasonable rules, orders and
instructions of the Company, as well as gross failure to reach at least
minimum quota, the termination of his engagement from Manulife is highly
warranted and therefore, there is no illegal dismissal to speak of.

ISSUE:
Whether Tongko’s dismissal was valid

RULING:
NO.

It is readily evident from the above-quoted portions of Manulife's


petition that it failed to cite a single iota of evidence to support its
claims. Manulife did not even point out which order or rule that
Tongko disobeyed. More importantly, Manulife did not point out the
specific acts that Tongko was guilty of that would constitute gross and
habitual neglect of duty or disobedience. Manulife merely cited
Tongko's alleged "laggard performance," without substantiating such
claim, and equated the same to disobedience and neglect of duty.

We cannot, therefore, accept Manulife's position.

In Quebec, Sr. v. National Labor Relations Commission, we ruled


that:

When there is no showing of a clear, valid and legal cause for the
termination of employment, the law considers the matter a case of
illegal dismissal and the burden is on the employer to prove that the
termination was for a valid or authorized cause. This burden of proof
appropriately lies on the shoulders of the employer and not on the
employee because a worker's job has some of the characteristics of
property rights and is therefore within the constitutional mantle of
protection. No person shall be deprived of life, liberty or property
without due process of law, nor shall any person be denied the equal
protection of the laws.

Apropos thereto, Art. 277, par. (b), of the Labor Code mandates in
explicit terms that the burden of proving the validity of the termination
of employment rests on the employer. Failure to discharge this
evidential burden would necessarily mean that the dismissal was not
justified, and, therefore, illegal.

855
Page 856

We again ruled in Times Transportation Co., Inc. v. National Labor


Relations Commission that:

The law mandates that the burden of proving the validity of the
termination of employment rests with the employer. Failure to
discharge this evidentiary burden would necessarily mean that the
dismissal was not justified, and, therefore, illegal. Unsubstantiated
suspicions, accusations and conclusions of employers do not provide
for legal justification for dismissing employees. In case of doubt, such
cases should be resolved in favor of labor, pursuant to the social
justice policy of our labor laws and Constitution.

This burden of proof was clarified in Community Rural Bank of San


Isidro (N.E.), Inc. v. Paez to mean substantial evidence, to wit:

The Labor Code provides that an employer may terminate the


services of an employee for just cause and this must be supported by
substantial evidence. The settled rule in administrative and quasi-
judicial proceedings is that proof beyond reasonable doubt is not
required in determining the legality of an employer's dismissal of an
employee, and not even a preponderance of evidence is necessary
as substantial evidence is considered sufficient. Substantial evidence
is more than a mere scintilla of evidence or relevant evidence as a
reasonable mind might accept as adequate to support a conclusion,
even if other minds, equally reasonable, might conceivably opine
otherwise.

Here, Manulife failed to overcome such burden of proof. It must be


reiterated that Manulife even failed to identify the specific acts by
which Tongko's employment was terminated much less support the
same with substantial evidence. To repeat, mere conjectures cannot
work to deprive employees of their means of livelihood. Thus, it must
be concluded that Tongko was illegally dismissed.

856
Page 857

Case Digests by: TORRES, PAULA GAIL I.

PHARMACIA AND UPJOHN, INC. (NOW PFIZER


PHILIPPINES, INC.) vs. RICARDO ALBAYDA, JR.
G.R. No. 172724. August 23, 2010.
Peralta, J.

DOCTRINE:
GROSS INSUBORDINATION.
It is a management prerogative to transfer or assign employees
from one office or area of operation to another, provided there is no
demotion in rank or diminution of salary, benefits, and other privileges,
and the action is not motivated by discrimination, made in bad faith, or
effected as a form of punishment or demotion without sufficient cause.
The objection to the transfer being grounded solely upon the personal
inconvenience or hardship that will be caused to the employee by
reason of the transfer is not a valid reason to disobey an order of
transfer.

FACTS:
Ricardo Albayda, Jr. was designated by Pharmacia as District
Sales Manager assigned in Western Visayas area after the merger
between Pharmacia and Upjohn.

In 1999, he received a Memorandum that he was being


reassigned as a District Sales Manager in the Northern Mindanao area.
In response, he said that he has always been assigned to the Western
Visayas area and that he felt that he could not improve the sales of
products if he was assigned to an unfamiliar territory. He argued that
he will be dislocated from his family should he transfer and that his
family’s income would be affected because his wife earns around
P50,000 per month in Bacolod.

It was explained to him that the transfer was because of his


expertise; that the district performed dismally in 1999 and, therefore,
they were confident that under his leadership, he can implement new
ways to increase its production; and stressed that the decision was
purely a business decision. Thereafter, he was reassigned to Manila,
and he was directed to report to work within 5 days, but this was
unheeded. Because of this, petitioner dismissed him from work through
a letter due to AWOL and insubordination.

Respondent filed a case for constructive dismissal.

857
Page 858

ISSUE:
Was the dismissal based on insubordination valid?

RULING:
Yes, Albayda was legally dismissed.

Jurisprudence recognizes the exercise of management


prerogative to transfer or assign employees from one office or area of
operation to another, provided there is no demotion in rank or
diminution of salary, benefits, and other privileges, and the action is
not motivated by discrimination, made in bad faith, or effected as a
form of punishment or demotion without sufficient cause. In this case,
the transfer from one district to another is duly explained.

The allegation of complainant that respondent’s family’s income


will be affected, if true, should not be taken in consideration. What is
contemplated is the diminution of salary of the complainant but not his
wife. Besides, even if complainant may accept his new assignment, his
wife may still continue to do her business in Bacolod City. Anyway,
Bacolod City and Manila is just one (1) hour travel by plane. Also, in
respondent’s contract of employment, he agreed to be assigned to any
work or workplace as may be determined by the company whenever
the operations require such assignment. As such, he is bound thereby.

The Supreme Court has long stated that the objection to the
transfer being grounded solely upon the personal inconvenience or
hardship that will be caused to the employee by reason of the transfer
is not a valid reason to disobey an order of transfer. Such being the
case, respondent cannot adamantly refuse to abide by the order of
transfer without exposing himself to the risk of being dismissed. Hence,
his dismissal was for just cause in accordance with Article 282(a) (now
Art. 297) of the Labor Code.

858
Page 859

Case Digests by: TORRES, PAULA GAIL I.

ST. LUKE’S MEDICAL CENTER, INC. vs. MARIA


THERESA SANCHEZ
G.R. No. 212054. March 11, 2015.
Perlas-Bernabe, J.

DOCTRINE:
WILLFUL DISOBEDIENCE.
For an employee to be validly dismissed on willful disobedience,
the employer’s orders, regulations, or instructions must be: (1)
reasonable and lawful, (2) sufficiently known to the employee, and (3)
in connection with the duties which the employee has been engaged
to discharge.

FACTS:
Maria Theresa Sanchez was hired by SLMC as a Staff Nurse. She
passed through the Entrance/Exit where she was subjected to the
standard inspection procedure by the security personnel. The security
guard on-duty noticed a pouch in her bag and asked her to open the
same. When opened, said pouch contained an assortment of medical
stocks which were subsequently confiscated. She was brought to the
SLMC In-House Security Department (IHSD) where she was directed
to write an Incident Report explaining why she had the questioned
items in her possession. She complied with the directive and also
submitted an undated handwritten letter of apology. An initial
investigation was also conducted which thereafter served Sanchez a
Notice to Explain. Sanchez submitted an Incident Report Addendum,
explaining that the questioned items came from the medication
drawers of patients who had already been discharged, and, as similarly
practiced by the other staff members, she started saving these items
as excess stocks in her pouch, along with other basic items that she
uses during her shift.

Sanchez was placed under preventive suspension until the


conclusion of the investigation which, thereafter, required her to
explain why she should not be terminated from service for “acts of
dishonesty” due to her possession of the questioned items. In
response, she submitted a letter which merely reiterated her claims in
her previous letter. She likewise requested for a case conference,
which SLMC granted. After hearing her side, SLMC informed Sanchez
of its decision to terminate her employment.

Sanchez filed for illegal dismissal. SLMC contended that Sanchez


was validly dismissed for just cause as she had committed theft in

859
Page 860

violation of Section 1, Rule I of the SLMC Code of Discipline, which


punishes acts of dishonesty, i.e., robbery, theft, pilferage, and
misappropriation of funds, with termination from service.

ISSUE:
Was the dismissal based on willful disobedience valid?

RULING:
Yes, she was legally dismissed.

Among the employer’s management prerogatives is the right to


prescribe reasonable rules and regulations necessary or proper for the
conduct of its business or concern, to provide certain disciplinary
measures to implement said rules and to assure that the same would
be complied with. At the same time, the employee has the corollary
duty to obey all reasonable rules, orders, and instructions of the
employer; and willful or intentional disobedience thereto, as a general
rule, justifies termination of the contract of service and the dismissal of
the employee.

Article 297 (formerly Art. 282) of the Labor Code provides: Article
297. Termination by Employer.—An employer may terminate an
employment for any of the following causes: (a) Serious misconduct or
willful disobedience by the employee of the lawful orders of his
employer or his representative in connection with his work; x x x x Note
that for an employee to be validly dismissed on this ground, the
employer’s orders, regulations, or instructions must be: (1) reasonable
and lawful, (2) sufficiently known to the employee, and (3) in
connection with the duties which the employee has been engaged to
discharge.”

The SLMC Code of Discipline is further supplemented by the


company policy requiring the turn-over of excess medical
supplies/items for proper handling and providing a restriction on taking
and bringing such items out of the SLMC premises without the proper
authorization or “pass” from the official concerned, which Sanchez was
equally aware thereof.

860
Page 861

Case Digests by: TORRES, PAULA GAIL I.

ST. LUKE’S MEDICAL CENTER, INC. vs. ESTRELITO


NOTARIO
G.R. No. 152166. October 20, 2010.
Peralta, J.

DOCTRINE:
GROSS AND HABITUAL NEGLECT OF DUTY.
Neglect of duty, to be a ground for dismissal, must be both gross
and habitual. Gross negligence connotes want of care in the
performance of one’s duties, while habitual neglect implies repeated
failure to perform one’s duties for a period of time, depending upon the
circumstances; A single or isolated act of negligence does not
constitute a just cause for the dismissal of the employee.

FACTS:
SLMC employed Estrelito Notario as an In-House Security Guard
with his work focused mainly on monitoring the CCTVs installed in the
hospital. One evening, as Justin Tibon was attending to his 3-year-old
daughter who then admitted in the hospital, he reported to the
management that he had lost his mint green traveling bag. Acting on
this, the Security Dept. investigated. When the tapes were reviewed, it
was shown the cameras failed to record any incident of theft in the
room.

SLMC issued a Memorandum to Notario, who was the CCTV


monitoring staff on duty at that time. He explained that he was the only
personnel on duty that time as nobody wanted to assist him. Because
of this, he decided to focus the cameras on the Old and New Maternity
Units, as these two units have a high incidence of crime. Finding the
written explanation of respondent to be unsatisfactory, he was
dismissed due to gross negligence/inefficiency.

Notario filed a case for illegal dismissal but petitioners countered


that they validly dismissed respondent for gross negligence and
observed due process before terminating his employment.

ISSUE:
Was respondent validly dismissed for gross negligence?

RULING:
No, Notario was illegally dismissed.

861
Page 862

Under Article 282(b) (now Art. 297) of the Labor Code, an


employer may terminate an employee for gross and habitual neglect of
duties. Neglect of duty, to be a ground for dismissal, must be both
gross and habitual. Gross negligence connotes want of care in the
performance of one's duties. Habitual neglect implies repeated failure
to perform one's duties for a period of time, depending upon the
circumstances. A single or isolated act of negligence does not
constitute a just cause for the dismissal of the employee.

Under the prevailing circumstances, respondent exercised his


best judgment in monitoring the CCTV cameras so as to ensure the
security within the hospital premises. Respondent exercised his best
judgment in monitoring the CCTV cameras so as to ensure the security
within the hospital premises. Verily, assuming arguendo that
respondent was negligent, although the Court finds otherwise, the
lapse or inaction could only be regarded as a single or isolated act of
negligence that cannot be categorized as habitual and, hence, not a
just cause for his dismissal.

862
Page 863

Case Digests by: TORRES, PAULA GAIL I.

LBC EXPRESS – METRO MANILA, INC. vs. JAMES


MATEO
G.R. No. 168215. June 09, 2009.
Corona, J.

DOCTRINE:
GROSS AND HABITUAL NEGLECT OF DUTY.
The services of a regular employee may be terminated only for
just or authorized causes, including gross and habitual negligence
under Article 282 (now Art. 297), paragraph (b) of the Labor Code.
Gross negligence is characterized by want of even slight care, acting
or omitting to act in a situation where there is a duty to act, not
inadvertently but willfully and intentionally with a conscious indifference
to the consequences insofar as other persons may be affected.

FACTS:
Respondent James Mateo, designated as a customer associate,
was a regular employee of petitioner LBC. Mateo arrived at LBC's
Escolta office to drop off packages coming from various LBC airposts.
He parked his motorcycle directly in front of the LBC office, switched
off the engine and took the key with him. However, he did not lock the
steering wheel because he allegedly was primarily concerned with the
packages, including a huge sum of money that needed to be
immediately secured inside the LBC office. He returned promptly within
three to five minutes but the motorcycle was gone. He immediately
reported the loss to his superiors at LBC and to the nearest police
station.

LBC directed Mateo to appear in his office for formal investigation


and to explain his side. As directed, Mateo appeared and presented
his side. After investigation, he received a notice of termination from
LBC. Mateo thereafter filed a complaint for illegal dismissal.

ISSUE:
Was the dismissal based on gross negligence valid?

RULING:
Yes, Mateo’s infraction amounted to gross negligence.

The services of a regular employee may be terminated only for


just or authorized causes, including gross and habitual negligence
under Article 282, paragraph (b) of the Labor Code. Gross negligence

863
Page 864

is characterized by want of even slight care, acting or omitting to act in


a situation where there is a duty to act, not inadvertently but willfully
and intentionally with a conscious indifference to the consequences
insofar as other persons may be affected.

Mateo was undisputedly negligent when he left the motorcycle


without locking it despite clear, specific instructions to do so. His
argument that he stayed inside the LBC office for only three to five
minutes was of no moment. It only proved that he did not exercise even
the slightest degree of care during that very short time. Mateo
deliberately did not heed the employer's very important precautionary
measure to ensure the safety of company property. Regardless of the
reasons advanced, the exact evil sought to be prevented by LBC in
repeatedly directing its customer associates to lock their motorcycles
occurred, resulting in a substantial loss to LBC.

Although Mateo's infraction was not habitual, we must take into


account the substantial amount lost. In this case, LBC lost a motorcycle
with a book value of P46,000 which by any means could not be
considered a trivial amount. Mateo was entrusted with a great
responsibility to take care of and protect company property and his
gross negligence should not allow him to walk away from that incident
as if nothing happened and, worse, to be rewarded with backwages to
boot.

864
Page 865

Case Digests by: TORRES, PAULA GAIL I.

MANSION PRINTING CENTER vs. DIOSDADO BITARA,


JR.
G.R. No. 168120. January 25, 2012.
Perez, J.

DOCTRINE:
GROSS AND HABITUAL NEGLECT OF DUTY.
Multiple attendance delinquencies may be characterized as
habitual and are sufficient justifications to terminate the complainant’s
employment. The totality of infractions or the number of violations
committed during the period of employment shall be considered in
determining the penalty to be imposed upon an erring employee. The
offenses committed by him should not be taken singly and separately
but in their totality.

FACTS:
Petitioner Mansion is engaged in the printing of quality self-
adhesive labels, brochures, posters, stickers, packaging and the like.
Respondent Bitara was engaged as a helper (kargador) and was later
promoted as the company’s sole driver tasked to pick-up raw materials
for the printing business, collect account receivables and deliver the
products to the clients within the delivery schedules. Mansion avers
that the timely delivery of the products to the clients is one of the
foremost considerations material to the operation of the business. It
being so, his attendance was closely monitored. They noted his
habitual tardiness and absenteeism.

Thus, Mansion issued a Memorandum requiring Bitara to submit


a written explanation as to why no administrative sanction should be
imposed on him for his habitual tardiness. Despite his undertaking to
report on time, however, he continued to disregard attendance policies.
His absences without prior notice and approval from March 11-16,
2000 were considered to be the most serious infraction of all because
of its adverse effect on business operations.

Consequently, another Memorandum (Notice to Explain) was


issued requiring respondent to explain why his services should not be
terminated. It was personally handed to respondent but the latter, after
reading the directive, refused to acknowledge receipt thereof. He did
not submit any explanation and, thereafter, never reported for work.
Another Memorandum (Notice of Termination) was personally served
upon him informing him that the company found him grossly negligent
of his duties, for which reason, his services were terminated.

865
Page 866

Bitara filed a complaint for illegal dismissal.

ISSUE:
Was the habitual tardiness/absenteeism of respondent a valid
ground for dismissal?

RULING:
Yes, the dismissal in this case was justified.

The imputed absence and tardiness of the complainant are


documented. He faltered on his attendance 38 times of the 66 working
days. His last absences on 11, 13, 14, 15 and 16 March 2000 were
undertaken without even notice/permission from management. These
attendance delinquencies may be characterized as habitual and are
sufficient justifications to terminate the complainant’s employment. On
this score, Valiao v. Court of Appeals (2004), is instructive: xxx It bears
stressing that petitioner’s absences and tardiness were not isolated
incidents but manifested a pattern of habituality. xxx The totality of
infractions or the number of violations committed during the period of
employment shall be considered in determining the penalty to be
imposed upon an erring employee. The offenses committed by him
should not be taken singly and separately but in their totality. Fitness
for continued employment cannot be compartmentalized into tight little
cubicles of aspects of character, conduct, and ability separate and
independent of each other.

In Valiao, this Court defined gross negligence as “want of care in


the performance of one’s duties” and habitual neglect as “repeated
failure to perform one’s duties for a period of time, depending upon the
circumstances.” These are not overly technical terms, which, in the first
place, are expressly sanctioned by the Article 297 (previously Art. 282)
Labor Code of the Philippines, to wit: ART. 282. Termination by
employer.—An employer may terminate an employment for any of the
following causes: (a) xxx (b) Gross and habitual neglect by the
employee of his duties; xxx Clearly, even in the absence of a written
company rule defining gross and habitual neglect of duties,
respondent’s omissions qualify as such warranting his dismissal from
the service.

866
Page 867

Case Digests by: TORRES, PAULA GAIL I.

CAVITE APPAREL, INCORPORATED vs. MICHELLE


MARQUEZ
G.R. No. 172044. February 06, 2013.
Brion, J.

DOCTRINE:
GROSS AND HABITUAL NEGLECT OF DUTY.
Neglect of duty, to be a ground for dismissal under Article 282 of
the Labor Code, must be both gross and habitual. Gross negligence
implies want of care in the performance of one’s duties. Habitual
neglect imparts repeated failure to perform one’s duties for a period of
time, depending on the circumstances.

FACTS:
Michelle Marquez was hired as a regular employee in the Finishing
Department of Cavite Apparel. Prior to her dismissal, she incurred and
was already sanctioned for three AWOLs. On the 4th instance, she got
sick and did not report for work but when she returned, she submitted
a medical certificate – which Cavite denied receiving. Nonetheless,
Cavite Apparel suspended Michelle for 6 days. When Michelle again
returned to work, Cavite Apparel dismissed her for habitual
absenteeism.

Michelle filed for illegal dismissal with a prayer for reinstatement.


Cavite cites Michelle’s AWOLS as gross and habitual neglect of duty
and was prejudicial to its business operations.

ISSUE:
Was the dismissal valid?

RULING:
No, Michelle’s four absences were not habitual.

Neglect of duty, to be a ground for dismissal under Article 297 of


the Labor Code, must be both gross and habitual. Gross negligence
implies want of care in the performance of one’s duties. Habitual
neglect imparts repeated failure to perform one’s duties for a period of
time, depending on the circumstances.

Four absences in her six years of service cannot be considered


gross and habitual neglect of duty, especially so since the absences
were spread out over a six-month period. Michelle’s penalty of

867
Page 868

dismissal too harsh or not proportionate to the infractions she


committed. Although Michelle was fully aware of the company rules
regarding leaves of absence, and her dismissal might have been in
accordance with the rules, it is well to stress that the Supreme Court is
not bound by such rules. While previous infractions may be used to
support an employee’s dismissal from work in connection with a
subsequent similar offense, penalties must be commensurate to the
offense involved and to the degree of the infraction.

868
Page 869

Case Digests by: TORRES, PAULA GAIL I.

ESSENCIA MANARPIIS vs. TEXAN PHILIPPINES, INC.


G.R. No. 197011. January 28, 2015.
Villarama, Jr. J.

DOCTRINE:
GROSS AND HABITUAL NEGLECT OF DUTY.
Abandonment as a just ground for dismissal requires
the deliberate, unjustified refusal of the employee to perform his
employment responsibilities. It is well-settled that the filing by an
employee of a complaint for illegal dismissal with a prayer for
reinstatement is proof enough of his desire to return to work, thus,
negating the employer’s charge of abandonment.

FACTS:
Respondent TPI hired Essencia Manarpiis as Sales and Marketing
Manager of TPI’s Aroma Division. Claiming insurmountable losses, TPI
served a written notice addressed to all employees that TPI will cease
operations. On the same day, TPI barred her from reporting for work
and paid her last salary even if there was still a month left before
company closure. Manarpiis filed for illegal dismissal.

Manarpiis received a Notice of Investigation to explain her alleged


violations of company rules and fraudulent acts. Manarpiis’s counsel
said that there was no point in the investigation as TPI already
dismissed her. Manarpiis was then served a Notice of Termination due
to [dishonesty, loss of confidence, and] abandonment of work since
she went AWOL.

TPI was able to remain in business but still claims that Manarpiis
was found to have committed infractions constituting loss of confidence
which resulted in her termination.

ISSUE:
Was the dismissal for abandonment valid?

RULING:
No, her dismissal was without cause as there was no
abandonment.

Two elements must concur for a valid abandonment:

869
Page 870

(1) the failure to report to work or absence without valid or


justifiable reason, and

(2) a clear intention to sever the employer-employee


relationship, with the second element as the more determinative factor
being manifested by some overt acts.

Abandonment as a just ground for dismissal requires deliberate,


unjustified refusal to perform his employment responsibilities. Mere
absence or failure to work, even after notice to return, is not tantamount
to abandonment. It is well-settled that the filing by an employee of an
illegal dismissal complaint with reinstatement is proof enough of his
desire to return to work, thus, negating the employer’s charge of
abandonment. Manarpiis did not abandon her work but was told not to
report after being served a written notice of company closure.

The announced cessation of business operations was a


subterfuge for getting rid of Manarpiis. Here, the subsequent
investigation and termination of Manarpiis based on [dishonesty, loss
of confidence and] abandonment of work, clearly appears as an
afterthought as it was done only after Manarpiis had filed an illegal
dismissal case.

870
Page 871

Case Digests by: TORRES, PAULA GAIL I.

SCHOOL OF THE HOLY SPIRIT OF QUEZON CITY vs.


CORAZON TAGUIAM
G.R. No. 165565. July 14, 2008.
Quisumbing, J.

DOCTRINE:
GROSS AND HABITUAL NEGLECT OF DUTY.
Gross negligence implies a want or absence of or a failure to
exercise slight care or diligence, or the entire absence of care. It
evinces a thoughtless disregard of consequences without exerting any
effort to avoid them. Habitual neglect implies repeated failure to
perform one’s duties for a period of time, depending upon the
circumstances. The sufficiency of the evidence as well as the resultant
damage to the employer should be considered in the dismissal of the
employee.

FACTS:
Respondent Taguiam was the class adviser of Grade 5 –
Esmeralda of petitioner School. Student Chiara Federico’s permit for
an authorized year-end class celebration was unsigned, but Taguiam
still allowed her to join because her mother personally brought her to
the school with her packed lunch and swimsuit. Taguiam warned the
pupils who did not know how to swim to avoid the deeper area.
However, while the pupils were swimming, two of them sneaked out.
Taguiam went after them to verify where they were going. While she
was away, Chiara Mae drowned and thereafter, died.

Petitioner School dismissed Taguiam on the ground of gross


negligence resulting to loss of trust and confidence, prompting the
latter to file a complaint for illegal dismissal.

ISSUE:
Was the dismissal valid?

RULING:
Yes.

Under Article 282 (now Art. 297) of the Labor Code, gross and
habitual neglect of duties is a valid ground for an employer to terminate
an employee. Gross negligence implies a want or absence of or a
failure to exercise slight care or diligence, or the entire absence of care.
It evinces a thoughtless disregard of consequences without exerting

871
Page 872

any effort to avoid them. Habitual neglect implies repeated failure to


perform one’s duties for a period of time, depending upon the
circumstances.

While an employee’s negligence, although gross, was not


habitual, in view of the considerable resultant damage, the Court is in
agreement that the cause is sufficient to dismiss her—in this case, the
damage went as far as claiming the life of a child; The sufficiency of
the evidence as well as the resultant damage to the employer should
be considered in the dismissal of the employee.

872
Page 873

Case Digests by: TORRES, PAULA GAIL

PHILIPPINE AIRLINES, INC. vs. NATIONAL LABOR


RELATIONS COMMISSION, MARCELITO PESCANTE
G.R. No. 126805. March 16, 2000.
Quisumbing, J.

DOCTRINE:
FRAUD.
That an employee attempted to deprive his, employer of its lawful
revenue is already tantamount to fraud against the company, which
warrants dismissal from the service.

FACTS:
PAL employees Pescante and Vicente were assigned to handle
PAL’s flight for Cebu as load controller and check-in clerk, respectively.
When passenger Cominero was checking in for her flight, Pescante
called Vicente who willingly cooperated in checking-in Cominero.

Vicente reflected a lighter weight of baggage of passenger


Cominero to make it seem as if it was within the allowable level by
pooling the same with other passengers with lesser baggage weight or
no baggage at all, in violation of PAL’s Code of Discipline (illegal
pooling of baggage).

For this, Cominero gave Vicente P1,000. The scheme then having
been discovered, Vicente used the P1,000 to pay the excess baggage
but upon arrival in Cebu, Cominero was still made to pay the remaining
amount due.

An administrative charge for “fraud against the company” was then


filed against Pescante wherein he was found guilty and was dismissed
from service. Pescante filed a complaint for illegal dismissal.

ISSUE:
Was the dismissal based on fraud valid?

RULING:
Yes, an attempt to defraud is already tantamount to fraud against
his/her employer.

There is substantial evidence in showing that Pescante had direct


involvement in the illegal pooling of baggage. Obviously, Pescante’s

873
Page 874

act is inexcusable as it constitutes a serious offense under PAL’s Code


of Discipline.

The fact that PAL failed to show that it suffered losses in revenue
as a consequence of Pescante’s questioned act is immaterial. It must
be stressed that actual defraudation is not necessary in order that an
employee may be held liable. The attempt to deprive PAL of its lawful
revenue is already tantamount to fraud against the company, which
warrants dismissal from service.

874
Page 875

Case Digests by: TORRES, PAULA GAIL I.

JULIETA B. STA. ANA vs. MANILA JOCKEY CLUB, INC.


G.R. No. 208459. February 15, 2017.
Del Castillo, J.

DOCTRINE:
LOSS OF CONFIDENCE / BREACH OF TRUST.
To legally dismiss an employee on the ground of loss of trust, the
employer must establish that a) the employee occupied a position of
trust and confidence, or has been routinely charged with the care and
custody of the employer’s money or property; b) the employee
committed a willful breach of trust based on clearly established facts;
and c) such loss of trust relates to the employee’s performance of
duties. There must be actual breach of duty on the part of the employee
to justify his or her dismissal on the ground of loss of trust and
confidence.

FACTS:
Petitioner Julieta Sta. Ana was dismissed from her position as
outlet teller of Manila Jockey Club (MJCI) when it was found out after
due investigation that she conspired with other tellers in illegally
misappropriating funds of MJCI and lending it out to employees of
MJCI. Petitioner was charged by MJCI with dishonesty and other
fraudulent acts, which all warranted dismissal on the ground of loss of
trust and confidence.

Petitioner filed a complaint for illegal dismissal. She alleged that


she has been engaged in lending business to augment her income.

ISSUE:
Was the dismissal based on loss of trust and confidence valid?

RULING:
No, the dismissal was not proper.

To legally dismiss an employee on the ground of loss of trust, the


employer must establish that:

(a) the employee occupied a position of trust and confidence,


or has been routinely charged with the care and custody of the
employer's money or property;

875
Page 876

(b) the employee committed a willful breach of trust based on


clearly established facts; and

(c) such loss of trust relates to the employee's performance of


duties.

In fine, there must be actual breach of duty on the part of the


employee to justify his or her dismissal on the ground of loss of trust
and confidence.

It is a cardinal rule that loss of trust and confidence should be


genuine, and not simulated; it must arise from dishonest or deceitful
conduct, and must not be arbitrarily asserted in the face of
overwhelming contrary evidence. While proof beyond reasonable
doubt is not required, loss of trust must have some basis or such
reasonable ground for one to believe that the employee committed the
infraction, and the latter’s participation makes him or her totally
unworthy of the trust demanded by the position.

Here, MJCI failed to prove that Sta. Ana committed willful breach
of its trust. Particularly, it failed to establish that Sta. Ana used its
employee for her personal business during office hours, and used its
money, without authority, to lend money to another. Hence, to dismiss
her on the ground of loss of trust and confidence is unwarranted. The
allegations against Sta. Ana were not supported by clear and
convincing evidence after it was established that Sta. Ana has obtained
bank loans secured by real property mortgage, which then shows that
she has funds derived from sources other than her monthly salary.
There was no direct linkage shown between Sta. Ana’s lending
business and stolen funds of MJCI.

876
Page 877

Case Digests by: TORRES, PAULA GAIL I.

REXIE HORMILLOSA vs. COCA-COLA BOTTLERS


PHILS., INC.
G.R. No. 198699. October 09, 2013.
Mendoza, J.

DOCTRINE:
LOSS OF CONFIDENCE / BREACH OF TRUST.
There are two (2) classes of positions of trust. The first class
consists of managerial employees. They are defined as those vested
with the powers or prerogatives to lay down management policies and
to hire, transfer suspend, lay-off, recall, discharge, assign or discipline
employees or effectively recommend such managerial actions. The
second class consists of cashiers, auditors, property custodians, etc.
They are defined as those who in the normal and routine exercise of
their functions, regularly handle significant amounts of money or
property. There is a high degree of trust and confidence reposed on
route salesmen, and when confidence is breached, the employer may
take proper disciplinary action on them.

FACTS:
As a route salesman of respondent Coca-Cola Bottlers, petitioner
Hormillosa sells Coca-Cola’s products, either in cash or credit, collects
payments and is authorized to issue sales invoices. He also has the
duty of forwarding the sales invoices to the Finance Department of the
company for accounting and auditing.

Hormillosa was placed on grounded status when the District Sales


Supervisor found out that the petitioner issued invoices for fictitious
sales, and committed other violations of the company rules and
regulations: falsification of company records, fictitious issuances of
credit sales, issuance and non-issuance of invoices, receipts, etc.

After recommendation of the District Sales Manager, he was


terminated with the approval of the Regional Sales Manager.

ISSUE:
Was the dismissal valid?

RULING:
Yes, breach of trust reposed on him was duly proven.

877
Page 878

To legally dismiss an employee on the ground of loss of trust, the


employer must establish that:

(a) the employee occupied a position of trust and confidence,


or has been routinely charged with the care and custody of the
employer's money or property;

(b) the employee committed a willful breach of trust based on


clearly established facts; and

(c) such loss of trust relates to the employee's performance of


duties.

For the first requisites – there are 2 classifications of positions of


trusts: (a) managerial employees, and (b) employees such as cashiers,
auditors, property custodians, etc. Specifically, the second class of
employees are those who in the normal and routine exercise of their
functions, regularly handle significant amounts of money or property.
The second requisite is that there must be an act that would justify the
loss of trust and confidence. Loss of trust and confidence to be a valid
cause for dismissal must be based on a willful breach of trust and
founded on clearly established facts. The basis for the dismissal must
be clearly and convincingly established but proof beyond reasonable
doubt is not necessary.

Hormillosa, being a route salesman, falls under the second class.


By selling soft drink products and collecting payments for the same, he
was considered an employee who regularly handled significant
amounts of money and property in the normal and routine exercise of
his functions. As an employee holding a position of trust, there was a
high degree of trust and confidence reposed on him, and when this
trust is breached, the employer is justified in taking appropriate
disciplinary action. Salesmen are highly individualistic personnel who
have to be trusted and left essentially on their own. A high degree of
confidence is reposed on them because they are entrusted with funds
or properties of their employer.

878
Page 879

Case Digests by: TORRES, PAULA GAIL I.

CECILIA MANESE, et al. vs. JOLLIBEE FOODS


CORPORATION
G.R. No. 170454. October 11, 2012.
Peralta, J.

DOCTRINE:
LOSS OF CONFIDENCE / BREACH OF TRUST.
The mere existence of a basis for the loss of trust and confidence
justifies the dismissal of the managerial employee because when an
employee accepts a promotion to a managerial position or to an office
requiring full trust and confidence, such employee gives up some of
the rigid guaranties available to ordinary workers.

FACTS:
Petitioners were a team of Jollibee employees tasked to open a
new Jollibee branch in Festival Mall, Alabang. Prior to the opening of
the branch, Cruz (one of the petitioners) caused the Commissary
Warehouse and Distribution to deliver 450 packs of Jollibee
Chickenjoy. These Chickenjoys were then placed in the freezer.

Cruz attempted to return 150 pieces of rejects to the Commissary,


but the driver of the Commissary refused to accept them due to
discoloration and deteriorated condition. After a meeting, the
petitioners decided to soak and clean the rejects with soda water, pack
them again, and then return them again to the freezer.

After an inspection conducted by the area manager, the rejects


were discovered inside the freezer. Petitioners were charged of
extreme serious misconduct, gross negligence, product tampering,
falsification of company records and insubordination in connection with
the rejects kept in the freezer, which could have caused contamination
and threat to food safety.

After due investigation, all of the petitioners were dismissed for


gross negligence and incompetence, and for breach of trust and
confidence as managerial employees.

ISSUE:
Was the dismissal valid on the ground of loss of trust and
confidence?

RULING:

879
Page 880

Yes, the dismissal was valid.

The mere existence of a basis for the loss of trust and confidence
justifies the dismissal of the managerial employee because when an
employee accepts a promotion to a managerial position or to an office
requiring full trust and confidence, such employee gives up some of
the rigid guaranties available to ordinary workers. Infractions, which if
committed by others would be overlooked or condoned or penalties
mitigated, may be visited with more severe disciplinary action. Proof
beyond reasonable doubt is not required provided there is a valid
reason for the loss of trust and confidence, such as when the employer
has a reasonable ground to believe that the managerial employee
concerned is responsible for the purported misconduct and the nature
of his participation renders him unworthy of the trust and confidence
demanded by his position.

Managerial employees enjoy security of tenure and, although the


standards for their dismissal are less stringent, the loss of trust and
confidence must be substantial and founded on clearly established
facts sufficient to warrant the managerial employee’s separation from
the company. Substantial evidence is of critical importance and the
burden rests on the employer to prove it.

Here, the acts or omissions enumerated were valid bases for their
termination, which was grounded on gross negligence and/or loss of
trust and confidence.

880
Page 881

Case Digests by: TORRES, PAULA GAIL I.

GRAND ASIAN SHIPPING LINES, INC. vs. WILFREDO


GALVEZ, et al.
G.R. No. 178184. January 29, 2014.
Del Castillo, J.

DOCTRINE:
LOSS OF CONFIDENCE / BREACH OF TRUST.
With respect to rank-and-file personnel, loss of trust and
confidence, as ground for valid dismissal, requires proof of involvement
in the alleged events; while for managerial employees, the mere
existence of a basis for believing that such employee has breached the
trust of his employer would suffice for his dismissal.

FACTS:
A formal complaint for qualified theft was filed against respondents
due to pilferage of fuel oil whole on board the vessel. Respondents
crewmembers of one of Grand Asian Shipping Lines,Inc.’s vessels,
M/T Dorothy Uno, with the following designations: Wilfredo Galvez
(Galvez) as Captain; Joel Sales (Sales) as Chief Mate; Cristito Gruta
(Gruta) as Chief Engineer; Danilo Arguelles (Arguelles) as Radio
Operator; Renato Batayola (Batayola), Patricio Fresmillo (Fresmillo)
and Jovy Noble (Noble) as Able Seamen; Emilio Dominico (Dominico)
and Benny Nilmao (Nilmao) as Oilers; and Jose Austral (Austral) as
2nd Engineer.

During investigation, petitioner GASL placed respondents under


preventive suspension and after conducting administrative hearings,
GASLI terminated respondents for (1) serious misconduct, (2) willful
breach of trust and (3) commission of crime or offense against the
employer. In turn, respondents filed for illegal dismissal.

ISSUE:
Was the dismissal valid based on loss of trust and confidence?

RULING:
Yes, the dismissal as regards the managerial employees were
valid.

The ship captain and chief engineer, both managerial employees,


were validly dismissed on the ground of loss of trust and confidence.
GASLI, on the other hand, failed to substantiate adequately the
charges of pilferage against the respondents who were classified as

881
Page 882

rank-and-file personnel. The mere filing of a formal charge, to our mind,


does not automatically make the dismissal valid. Evidence submitted
to support the charge should be evaluated to see if the degree of proof
is met to justify respondents’ termination.

As to the loss of trust and confidence, distinction should be made


between managerial and rank and file employees. With respect to
rank-and-file personnel, loss of trust and confidence, as ground for
valid dismissal, requires proof of involvement in the alleged events,
while for managerial employees, the mere existence of a basis for
believing that such employee has breached the trust of his employer
would suffice for his dismissal.

In the case before us, Galvez, as the ship captain, is considered a


managerial employee since his duties involve the governance, care
and management of the vessel. Gruta, as chief engineer, is also a
managerial employee for he is tasked to take complete charge of the
technical operations of the vessel. As captain and as chief engineer,
Galvez and Gruta perform functions vested with authority to execute
management policies and thereby hold positions of responsibility over
the activities in the vessel. Indeed, their position requires the full trust
and confidence of their employer for they are entrusted with the
custody, handling and care of company property and exercise authority
over it.

The fact that there was an overstatement of fuel consumption and


that there was loss of a considerable amount of diesel fuel oil remained
unrefuted. Their failure to account for this loss of company property
betrays the trust reposed and expected of them. They had violated
petitioners’ trust and for which their dismissal is justified on the ground
of breach of confidence.

882
Page 883

Case Digests by: TORRES, PAULA GAIL I.

BLUER THAN BLUE JOINT VENTURES COMPANY vs.


GLYZA ESTEBAN
G.R. No. 192582. April 07, 2014.
Reyes, J.

DOCTRINE:
LOSS OF CONFIDENCE / BREACH OF TRUST.
Loss of trust and confidence is premised on the fact that the
employee concerned holds a position of responsibility, trust and
confidence. The employee must be invested with confidence on
delicate matters, such as the custody, handling, care and protection of
the employer’s property and funds. It is not the job title but the actual
work that the employee performs that determines whether he or she
occupies a position of trust and confidence.

Loss of trust and confidence to be a valid cause for dismissal must


be work related such as would show the employee concerned to be
unfit to continue working for the employer and it must be based on a
willful breach of trust and founded on clearly established facts.

FACTS:
Respondent Glyza Esteban was employed as Sales Clerk, and
assigned at petitioner’s EGG boutique in SM City Marilao, Bulacan,
beginning the year 2006. Part of her primary tasks were attending to
all customer needs, ensuring efficient inventory, coordinating orders
from clients, cashiering and reporting to the accounting department.
Esteba received a report that several employees have access to its
point-of-sale (POS) system through a universal password. Upon
investigation, it was discovered that it was Esteban who gave the
password. In her explanation, Esteban admitted that she used the
universal password three times on the same day after she learned of it
from two other employees who she saw browsing through the
petitioner's sales inquiry. She inquired how the employees were able
to open the system and she was told that they used the "123456"
password.

Esteban’s preventive suspension was lifted after 10 days, but at


the same time, a notice of termination was sent to her, finding her
explanation unsatisfactory and terminating her employment
immediately on the ground of loss of trust and confidence. Esteban
filed for illegal dismissal.

883
Page 884

ISSUE:
Was the dismissal on the ground of loss of trust and confidence
valid?

RULING:
No, the cause was insufficient to justify termination.

Loss of trust and confidence is premised on the fact that the


employee concerned holds a position of responsibility, trust and
confidence. The employee must be invested with confidence on
delicate matters, such as the custody, handling, care and protection of
the employer’s property and funds. “[W]ith respect to rank-and-file
personnel, loss of trust and confidence as ground for valid dismissal
requires proof of involvement in the alleged events in question, and
that mere uncorroborated assertions and accusations by the employer
will not be sufficient.”

As consistently ruled by the Court, it is not the job title but the
actual work that the employee performs that determines whether he or
she occupies a position of trust and confidence. In Esteban's case,
given that she had in her care and custody the store's property and
funds, she is considered as a rank-and-file employee occupying a
position of trust and confidence.

Loss of trust and confidence to be a valid cause for dismissal must


be work related such as would show the employee concerned to be
unfit to continue working for the employer and it must be based on a
willful breach of trust and founded on clearly established facts. Such
breach is willful if it is done intentionally, knowingly, and purposely,
without justifiable excuse as distinguished from an act done carelessly,
thoughtlessly, heedlessly or inadvertently. The loss of trust and
confidence must spring from the voluntary or willful act of the
employee, or by reason of some blameworthy act or omission on the
part of the employee. In this case, the Court finds that the acts
committed by Esteban do not amount to a willful breach of trust. She
admitted that she accessed the POS system with the use of the
unauthorized "123456" password. She did so, however, out of curiosity
and without any obvious intention of defrauding the petitioner.

884
Page 885

Case Digests by: Vergara, Jean Colleen M.

CONCEPCION V. MINEX IMPORT CORP.


G.R. No. 153569. January 24, 2012.
Bersamin, J.

DOCTRINE:
LOSS OF CONFIDENCE / BREACH OF TRUST
The employer may validly dismiss for loss of trust and confidence
an employee who commits an act of fraud prejudicial to the interest of
the employer. Neither a criminal prosecution nor a conviction beyond
reasonable doubt for the crime is a requisite for the validity of the
dismissal. Nonetheless, the dismissal for a just or lawful cause must
still be made upon compliance with the requirements of due process
under the Labor Code; otherwise, the employer is liable to pay
nominal damages as indemnity to the dismissed employee.

FACTS:
Respondent Minex Import-Export Corporation (Minex) employed
the petitioner initially as a salesgirl and then made her a supervisor in
July 1997 with no salary increase. On Oct. 23, 1997, respondent Vina
Mariano, an Assistant Manager of Minex, assigned the petitioner to
the SM Harrison Plaza kiosk with the instruction to hold the keys of
the kiosk. Working under her supervision there were salesgirls
Cristina Calung and Lida Baquilar. On Nov. 9, 1997, a Sunday, the
petitioner and her salesgirls conducted a cash-count of their sales
proceeds and determined their total for the last three days to be
P50,912. The petitioner wrapped the amount in a plastic bag and
deposited it in the drawer of the locked wooden cabinet of the kiosk.
At about 9:30 am of the next day, the petitioner phoned Vina to report
that the P50,912 was missing, explaining how she had found upon
reporting to work that morning that the contents of the cabinet were in
disarray and the money already missing. Later, while the petitioner
was giving a detailed statement on the theft to the security
investigator of Harrison Plaza, Vina and Sylvia Mariano, her
superiors, arrived with a policeman who immediately placed the
petitioner under arrest and brought her to the where the police
investigated her. She was detained for a day and was released only
because the inquest prosecutor instructed so. On Nov.12, 1997, the
petitioner complained against the respondents for illegal dismissal in
the DOLE. On Nov.14, 1997, Minex filed a complaint for qualified
theft against the petitioner in the Office of the City Prosecutor in
Manila. Vina claims that the petitioner did not call the office of Minex
for the pick-up of the P39,194.50 cash sales on Nov. 9, 1997, in
violation of the standard operating procedure (SOP) requiring cash

885
Page 886

proceeds exceeding P10,000 to be reported for pick-up if the amount


could not be deposited in the bank. The RTC ruled in favor of
Concepcion but the NLRC (as sustained by the CA) reversed it by
reason of her dismissal is justifiable for loss of trust and confidence in
the light of the finding of probable cause by the DOJ and the City
Prosecutor and the filing of the information for qualified theft against
her.

ISSUE:
Whether the petitioner was terminated for a just and valid cause

RULING:
YES. To dismiss an employee, the law requires the existence of
a just and valid cause. Article 282 of the Labor Code enumerates the
just causes for termination by the employer: (a) serious misconduct or
willful disobedience by the employee of the lawful orders of his
employer or the latter's representative in connection with the
employee's work; (b) gross and habitual neglect by the employee of
his duties; (c) fraud or willful breach by the employee of the trust
reposed in him by his employer or his duly authorized representative;
(d)commission of a crime or offense by the employee against the
person of his employer or any immediate member of his family or his
duly authorized representative; and (e) other causes analogous to the
foregoing.

Yet, even as we now say that the respondents had a just or valid
cause for terminating the petitioner, it becomes unavoidable to ask
whether or not they complied with the requirements of due process
prior to the termination which we find in the negative. The petitioner
demonstrated how quickly and summarily her dismissal was carried
out without first requiring her to explain anything in her defense. The
fair and reasonable opportunity required to be given to the employee
before dismissal encompassed not only the giving to the employee of
notice of the cause and the ability of the employee to explain, but also
the chance to defend against the accusation. In view of the foregoing,
we impose on the respondents the obligation to pay to the petitioner
an indemnity in the form of nominal damages of P30,000.00,
conformably with Agabon v. NLRC

886
Page 887

Case Digests by: Vergara, Jean Colleen M.

MANILA JOCKEY CLUB v. TRAJANO


G.R. No. 160982. June 26, 2013
Bersamin, J.

DOCTRINE:
LOSS OF CONFIDENCE / BREACH OF TRUST
The loss of trust and confidence, to be a valid ground for
dismissal, must be based on a willful breach of trust and confidence
founded on clearly established facts. Moreover, the loss of trust and
confidence must be related to the employee's performance of duties.

FACTS:
MJCI had employed Aimee O. Trajano as a selling teller of
betting tickets since November 1989. On April 25, 1998, two regular
bettors gave her their respective lists of bets (rota) and money for the
bets for Race 14. Although the bettors suddenly left her, she entered
their bets in the selling machine and segregated the tickets for pick
up by the two bettors upon their return. Before closing time, one of
the bettors (requesting bettor) returned and asked her to cancel one
of his bets worth P2,000. Since she was also operating the negative
machine on that day, she obliged and immediately cancelled the bet
as requested.

The reliever-supervisor later approached Trajano and told her to


submit a written explanation about the ticket cancellation incident.
The next day she submitted the handwritten explanation to Atty.
Galit, Assistant Racing Supervisor. Later that day, she received an
inter-office correspondence signed by Atty. Galit informing her that
she was being placed under preventive suspension effective April 28,
1998, for an unstated period of time. When Trajano reported for work
at the end of thirty days of her suspension, she was no longer
admitted. She then learned that she had been dismissed when she
read a copy of an inter-office correspondence about her termination
posted in a selling station of MJCI. Trajano instituted a complaint for
illegal dismissal against MJCI in the DOLE. She claimed that her
dismissal was not based on any of the grounds enumerated under
Article 282 of the Labor Code; that her dismissal on the ground of
unauthorized cancellation of ticket had no basis because she was
also the operator of the negative machine on the day in question with
the authority to cancel tickets as requested; that the cancellation was
not intentional on her part but resulted from an honest mistake that
did not amount to dishonesty.

887
Page 888

MJCI maintained that Trajano's dismissal was justified because


the unauthorized cancellation of the ticket had constituted a serious
violation of company policy amounting to dishonesty; that her action
had also constituted a just cause for terminating her employment
under Article 282 of the Labor Code, particularly paragraph (a) on
serious misconduct or willful disobedience and paragraph (b) on
gross and habitual neglect of duty.

ISSUE:
Whether or not there was just cause when MJCI dismissed
Trajano from the service

RULING:
NO. The valid termination of an employee may either be for just
causes under Article 282 or for authorized causes under Article 283
and Article 284, all of the Labor Code. Specifically, loss of the
employer's trust and confidence is a just cause under Article 282 (c),
a provision that ideally applies only to cases involving an employee
occupying a position of trust and confidence, or to a situation where
the employee has been routinely charged with the care and custody
of the employer's money or property. But the loss of trust and
confidence, to be a valid ground for dismissal, must be based on a
willful breach of trust and confidence founded on clearly established
facts. Moreover, the loss of trust and confidence must be related to
the employee's performance of duties.

As a selling teller, Trajano held a position of trust and


confidence. The nature of her employment required her to handle and
keep in custody the tickets issued and the bets made in her assigned
selling station. The bets were funds belonging to her employer.
Although the act complained of — the unauthorized cancellation of
the ticket (i.e., unauthorized because it was done without the consent
of the bettor) — was related to her work as a selling teller, MJCI did
not establish that the cancellation of the ticket was intentional,
knowing and purposeful on her part in order for her to have breached
the trust and confidence reposed in her by MJCI, instead of being
only out of an honest mistake. Still, to justify the supposed loss of its
trust and confidence in Trajano, MJCI contends that the unauthorized
cancellation of the ticket could have greatly prejudiced MJCI for
causing damage to both its income and reputation. We consider the
contention of MJCI unwarranted. As the records indicate, MJCI's
prejudice remained speculative and unrealized. To dismiss an
employee based on speculation as to the damage the employer could
have suffered would be an injustice. The injustice in the case of
Trajano would be greater if the supposed just cause for her dismissal
was not even sufficiently established.

888
Page 889

Case Digests by: Vergara, Jean Colleen M.

RENO FOODS v. NAGKAKAISANG LAKAS NG


MANGGAGAWA(NLM)-KATIPUNAN on behalf of its
member, NENITA CAPOR,
G.R. No. 164016. March 15, 2010.
Del Castillo, J.

DOCTRINE:
COMMISSION OF A CRIME
There is no legal or equitable justification for awarding financial
assistance to an employee who was dismissed for stealing company
property. Social justice and equity are not magical formulas to erase
the unjust acts committed by the employee against his employer.
While compassion for the poor is desirable, it is not meant to coddle
those who are unworthy of such consideration.

FACTS:
Respondent Nenita Capor (Capor) was an employee of Reno
Foods until her dismissal on October 27, 1998. It is a standard
operating procedure of petitioner-company to subject all its
employees to reasonable search of their belongings upon leaving the
company premises. On October 19, 1998, the guard on duty found
six Reno canned goods wrapped in nylon leggings inside Capor's
fabric clutch bag. The only other contents of the bag were money bills
and a small plastic medicine container. Petitioners accorded Capor
several opportunities to explain her side, often with the assistance of
the union officers of Nagkakaisang Lakas ng Manggagawa (NLM)-
Katipunan. In fact, after petitioners sent a Notice of Termination to
Capor, she was given yet another opportunity for reconsideration
through a labor-management grievance. Unfortunately, petitioners did
not find reason to change its earlier decision to terminate Capor's
employment with the company. On December 8, 1998, petitioners
filed a complaint-affidavit against Capor for qualified theft. On April 5,
1999, a Resolution was issued finding probable cause for the crime
charged. Meanwhile, (NLM)-Katipunan filed on behalf of Capor a
complaint for illegal dismissal and money claims against petitioners
with the Head Arbitration Office of the NLRC for NCR. The complaint
prayed that Capor be paid her full backwages as well as moral and
exemplary damages.

The LA found that theft of company property is tantamount to


serious misconduct; as such, Capor is not entitled to reinstatement
and backwages, as well as moral and exemplary damages.
Moreover, the LA ruled that consistent with prevailing jurisprudence,

889
Page 890

an employee who commits theft of company property may be validly


terminated and consequently, the said employee is not entitled to
separation pay.

On appeal, the NLRC affirmed the factual findings and monetary


awards of the LA but added an award of financial assistance. The CA
affirmed the NLRC's award of financial assistance to Capor wherein it
stressed that the laborer's welfare should be the primordial and
paramount consideration when carrying out and interpreting
provisions of the Labor Code.

ISSUE:
Whether the NLRC committed grave abuse of discretion
amounting to lack or excess of jurisdiction in granting financial
assistance to an employee who was validly dismissed for theft of
company property.

RULING:
YES. We find no justification for the award of separation pay to
Capor. This award is a deviation from established law and
jurisprudence. The law is clear. Separation pay is only warranted
when the cause for termination is not attributable to the employee's
fault, such as those provided in Articles 283 and 284 of the Labor
Code, as well as in cases of illegal dismissal in which reinstatement is
no longer feasible. It is not allowed when an employee is dismissed
for just cause, such as serious misconduct. Jurisprudence has
classified theft of company property as a serious misconduct and
denied the award of separation pay to the erring employee. We see
no reason why the same should not be similarly applied in the case of
Capor. She attempted to steal the property of her long-time employer.
For committing such misconduct, she is definitely not entitled to an
award of separation pay. While we sympathize with Capor's plight,
being of retirement age and having served petitioners for 39 years,
we cannot award any financial assistance in her favor because it is
not only against the law but also a retrogressive public policy.

890
Page 891

Case Digests by: Vergara, Jean Colleen M.

LYNVIL FISHING ENTERPRISES, INC. v. ARIOLA


G.R. No. 181974. February 1, 2012.
Del Castillo, J.

DOCTRINE:
COMMISSION OF A CRIME
Whichever way the public prosecutor disposes of a complaint,
the finding does not bind the labor tribunal.

FACTS:
On 1 August 1998, Lynvil received a report from Romanito
Clarido, one of its employees, that on 31 July 1998, he witnessed that
while on board the company vessel Analyn VIII, respondent Lynvil
employees, namely: Andres G. Ariola (Ariola), the captain; Jessie D.
Alcovendas (Alcovendas), Chief Mate; Jimmy B. Calinao (Calinao),
Chief Engineer; Ismael G. Nubla (Nubla), cook; Elorde Bañez
(Bañez), oiler; and Leopoldo D. Sebullen (Sebullen), bodegero,
conspired with one another and stole 8 tubs of "pampano" and
"tangigue" fish and delivered them to another vessel.

Lynvil, through De Borja, filed a criminal complaint against the


dismissed employees for violation of P.D. 532, or the Anti-Piracy and
Anti- Highway Robbery Law of 1974 before the Office of the City
Prosecutor of Malabon City. On 12 November 1998, First Assistant
City Prosecutor Rosauro Silverio found probable cause for the
indictment of the dismissed employees for the crime of qualified theft
under the RPC. The respondent employees filed with the Arbitration
Branch of the NLRC-NCR on 25 August 1998 a complaint for illegal
dismissal.

The Labor Arbiter found that there was no evidence showing that
the private respondents received the 41 bañeras of "pampano" as
alleged by De Borja in his reply-affidavit; and that no proof was
presented that the 8 bañeras of pampano [and tangigue] were
missing at the place of destination. 18 The Labor Arbiter disregarded
the Resolution of Assistant City Prosecutor Rosauro Silverio on the
theft case. He reasoned out that the Labor Office is governed by
different rules for the determination of the validity of the dismissal of
employees.

Although the NLRC reversed the decision of the LA, the CA


however reinstated the ruling of the LA and held that the allegation of

891
Page 892

theft did not warrant the dismissal of the employees since there was
no evidence to prove the actual quantities of the missing kinds of fish
loaded to Analyn VIII.

ISSUE:
Whether the CA erred in ruling that the termination of
respondent’s employment was not supported by substantial evidence.

RULING:
YES. In Nasipit Lumber Co. v. NLRC, we ruled that proof beyond
reasonable doubt of an employee's misconduct is not required when
loss of confidence is the ground for dismissal. It is sufficient if the
employer has "some basis" to lose confidence or that the employer
has reasonable ground to believe or to entertain the moral conviction
that the employee concerned is responsible for the misconduct and
that the nature of his participation therein rendered him absolutely
unworthy of the trust and confidence demanded by his position. It
added that the dropping of the qualified theft charges against the
respondent is not binding upon a labor tribunal. While in Nicolas v.
NLRC, we held that a criminal conviction is not necessary to find just
cause for employment termination. Otherwise stated, an employee's
acquittal in a criminal case, especially one that is grounded on the
existence of reasonable doubt, will not preclude a determination in a
labor case that he is guilty of acts inimical to the employer's interests.
In the reverse, the finding of probable cause is not followed by
automatic adoption of such finding by the labor tribunals.

In other words, whichever way the public prosecutor disposes of


a complaint, the finding does not bind the labor tribunal. Thus, Lynvil
cannot argue that since the Office of the Prosecutor found probable
cause for theft the Labor Arbiter must follow the finding as a valid
reason for the termination of respondents' employment. The proof
required for purposes that differ from one and the other are likewise
different.

892
Page 893

Case Digests by: Vergara, Jean Colleen M.

JOHN HANCOCK LIFE INSURANCE CORP. v. DAVIS


G.R. No. 169549. September 3, 2008.
Corona, J.

DOCTRINE:
OTHER ANALOGOUS CASES
A cause analogous to serious misconduct is a voluntary and/or
willful act or omission attesting to an employee's moral depravity.
Theft committed by an employee against a person other than his
employer, if proven by substantial evidence, is a cause analogous to
serious misconduct.

FACTS:
Respondent Joanna Cantre Davis was agency administration
officer of petitioner John Hancock Life Insurance Corporation. On
October 18, 2000, Patricia Yuseco, petitioner's corporate affairs
manager, discovered that her wallet was missing. She immediately
reported the loss of her credit cards to AIG and BPI Express. To her
surprise, she was informed that "Patricia Yuseco" had just made
substantial purchases using her credit cards in various stores in the
City of Manila. She was also told that a proposed transaction in
Abenson's-Robinsons Place was disapproved because "she" gave
the wrong information upon verification. Because loss of personal
property among its employees had become rampant in its office,
petitioner sought the assistance of the NBI. The NBI, in the course of
its investigation, obtained a security video from Abenson's showing
the person who used Yuseco's credit cards. Yuseco and other
witnesses positively identified the person in the video as respondent.
Consequently, the NBI and Yuseco filed a complaint for qualified theft
against respondent. But because the affidavits presented by the NBI
(identifying respondent as the culprit) were not properly verified, the
city prosecutor dismissed the complaint due to insufficiency of
evidence. Meanwhile, petitioner placed respondent under preventive
suspension and instructed her to cooperate with its ongoing
investigation. Instead of doing so, however, respondent filed a
complaint for illegal dismissal alleging that petitioner terminated her
employment without cause. The LA as affirmed by the NLRC found
that respondent committed serious misconduct (she was the principal
suspect for qualified theft committed inside petitioner's office during
work hours). There was a valid cause for her dismissal. The CA found
that the LA and NLRC merely adopted the findings of the NBI
regarding respondent's culpability.

893
Page 894

Before the SC, Petitioner argues that the ground for an


employee's dismissal need only be proven by substantial evidence.
Thus, the dropping of charges against an employee (specially on a
technicality such as lack of proper verification) or his subsequent
acquittal does not preclude an employer from dismissing him due to
serious misconduct.

ISSUE:
Whether or not petitioner substantially proved the presence of
valid cause for respondent's termination.

RULING:
YES. Petitioner dismissed respondent based on the NBI's finding
that the latter stole and used Yuseco's credit cards. But since the
theft was not committed against petitioner itself but against one of its
employees, respondent's misconduct was not work-related and
therefore, she could not be dismissed for serious misconduct.
Nonetheless, Article 282 (e) of the Labor Code talks of other
analogous causes or those which are susceptible of comparison to
another in general or in specific detail. A cause analogous to serious
misconduct is a voluntary and/or willful act or omission attesting to an
employee's moral depravity. Theft committed by an employee against
a person other than his employer, if proven by substantial evidence,
is a cause analogous to serious misconduct.

The labor arbiter and the NLRC relied not only on the affidavits of
the NBI's witnesses but also on that of respondent. They likewise
considered petitioner's own investigative findings. Clearly, they did
not merely adopt the findings of the NBI but independently assessed
evidence presented by the parties. Their conclusion (that there was
valid cause for respondent's separation from employment) was
therefore supported by substantial evidence.

894
Page 895

Case Digests by: Vergara, Jean Colleen M.

YRASUEGUI v. PAL
G.R. No. 168081. October 17, 2008.
Reyes, R.T., J

DOCTRINE:
OTHER ANALOGOUS CASES
An employee may be dismissed the moment he is unable to
comply with his ideal weight as prescribed by the weight standards.
The dismissal of the employee would thus fall under Article 282 (e) of
the Labor Code.

FACTS:
Petitioner Armando G. Yrasuegui was a former international flight
steward of Philippine Airlines, Inc. (PAL). He stands five feet and
eight inches (5'8") with a large body frame. The proper weight for a
man of his height and body structure is from 147 to 166 pounds, the
ideal weight being 166 pounds, as mandated by the Cabin and Crew
Administration Manual of PAL.

On June 15, 1993, petitioner was formally informed by PAL that


due to his inability to attain his ideal weight, "and considering the
utmost leniency" extended to him "which spanned a period covering a
total of almost five (5) years", his services were considered
terminated "effective immediately". His MR having been denied,
petitioner filed a complaint for illegal dismissal against PAL. To
buttress his stance, he argues that (1) his dismissal does not fall
under 282 (e) of the Labor Code; (2) continuing adherence to the
weight standards of the company is not a bona fide occupational
qualification; and (3) he was discriminated against because other
overweight employees were promoted instead of being disciplined.

ISSUE:
Whether or not the CA gravely erred in holding that petitioner’s
obesity can be a ground for dismissal under para (e) of article 282 of
the labor code

RULING:
NO. The obesity of petitioner is a ground for dismissal under
Article 282 (e) of the Labor Code. A reading of the weight standards
of PAL would lead to no other conclusion than that they constitute a
continuing qualification of an employee in order to keep the job.
Tersely put, an employee may be dismissed the moment he is unable

895
Page 896

to comply with his ideal weight as prescribed by the weight standards.


The dismissal of the employee would thus fall under Article 282 (e) of
the Labor Code. As explained by the CA: . . .

[T]he standards violated in this case were not mere "orders" of


the employer; they were the "prescribed weights" that a cabin crew
must maintain in order to qualify for and keep his or her position
in the company. In other words, they were standards that establish
continuing qualifications for an employee's position. In this sense,
the failure to maintain these standards does not fall under Article 282
(a) whose express terms require the element of willfulness in order to
be a ground for dismissal. The failure to meet the employer's
qualifying standards is in fact a ground that does not squarely fall
under grounds (a) to (d) and is therefore one that falls under Article
282(e) — the "other causes analogous to the foregoing". By its
nature, these "qualifying standards" are norms that apply prior to
and after an employee is hired. They apply prior to employment
because these are the standards a job applicant must initially meet in
order to be hired. They apply after hiring because an employee must
continue to meet these standards while on the job in order to keep his
job. Under this perspective, a violation is not one of the faults for
which an employee can be dismissed pursuant to pars. (a) to (d) of
Article 282; the employee can be dismissed simply because he no
longer "qualifies" for his job irrespective of whether or not the failure
to qualify was willful or intentional. . . .

We hold that the obesity of petitioner, when placed in the context


of his work as flight attendant, becomes an analogous cause under
Article 282 (e) of the Labor Code that justifies his dismissal from the
service. His obesity may not be unintended, but is nonetheless
voluntary. As the CA correctly puts it, "[v]oluntariness basically
means that the just cause is solely attributable to the employee
without any external force influencing or controlling his actions. This
element runs through all just causes under Article 282, whether they
be in the nature of a wrongful action or omission. Gross and habitual
neglect, a recognized just cause, is considered voluntary although it
lacks the element of intent found in Article 282 (a), (c), and (d)."

896
Page 897

Case Digests by: Vergara, Jean Colleen M.

SON ET AL. V. UST


G.R. No. 211273. April 18, 2018
Del Castillo, J.

DOCTRINE:
OTHER ANALOGOUS CAUSES
It cannot be said that by agreeing to the tenure by default
provision in the CBA, the employer is deemed to be in estoppel or
have waived the application of the requirement under a CHED
Memorandum Order. Such a waiver is precisely contrary to law.

FACTS:
UST is an educational institution operating under the authority of
CHED. Petitioners Raymond A. Son, Raymond S. Antiola, and
Wilfredo E. Pollarco are full time professors of US and are members
of the UST Faculty Union, with which UST at the time had a
Collective Bargaining Agreement (CBA). Under their respective
appointment papers, petitioners were designated as "faculty
member[s] on PROBATIONARY status," whose "accession to tenure
status is conditioned by [sic] your meeting all the requirements
provided under existing University rules and regulations and other
applicable laws including, among others, possession of the
[prerequisite] graduate degree before the expiration of the
probationary period " The UST-UST Faculty Union CBA provided
that, “if he is made to serve the University further, in spite of the lack
of a master's degree, he shall be deemed to have attained tenure.”
The cited CBA provision relative to the requirement of a Master's
degree in the faculty member's field of instruction is in line with the
requirement laid down in the 1992 Revised Manual of Regulations for
Private Schools issued by then Department of Education, Culture,
and Sports (DECS), and the CHED's Memorandum Order No. 40-08
— or Manual of Regulations for Private Higher Education of 2008 —
stating that, “Section 35. Minimum Faculty Qualifications . — The
minimum qualifications of a faculty in a higher education institution
shall be as follows: 1. For undergraduate program a. Holder of a
master's degree”

Petitioners did not possess the required Master's degree, but


were nonetheless hired by UST on the condition that they fulfill the
requirement within the prescribed period. Petitioners enrolled in the
Master's program, but were unable to finish the same. Acting on a
CHED Memorandum, UST wrote the petitioners and other affected
faculty members, informing them of the university's decision to cease

897
Page 898

re-appointment of those who failed to complete their Master's


degrees, but allow a written appeal from the concerned faculty
members who are due for thesis defense/completion of their Master's
degrees. Petitioners did not make a written appeal, operating under
the belief that they have been vested tenure under the CBA for their
continued employment despite failure to obtain the required Master's
degree. Petitioners now claim that they were illegally dismissed.

ISSUE:
Whether petitioner were illegally dismissed as faculty members

RULING:
NO. When CHED Memorandum Order No. 40-08 came out, it
merely carried over the requirement of a masteral degree for faculty
members of undergraduate programs contained in the 1992 Revised
Manual of Regulations for Private Schools. It cannot therefore be said
that the requirement of a master's degree was retroactively applied in
petitioners' case, because it was already the prevailing rule with the
issuance of the 1992 Revised Manual of Regulations for Private
Schools. Thus, going by the requirements of law, it is plain to see that
petitioners are not qualified to teach in the undergraduate programs
of UST. And while they were given ample time and opportunity to
satisfy the requirements by obtaining their respective master's
degrees, they failed in the endeavor. Petitioners knew this — that
they cannot continue to teach for failure to secure their master's
degrees — and needed no reminding of this fact; "those who are
seeking to be educators are presumed to know these mandated
qualifications."

It cannot be said either that by agreeing to the tenure by default


provision in the CBA, respondents are deemed to be in estoppel or
have waived the application of the requirement under CHED
Memorandum Order No. 40-08. Such a waiver is precisely contrary to
law. Moreover, a waiver would prejudice the rights of the students
and the public, who have a right to expect that UST is acting within
the bounds of the law, and provides quality education by hiring only
qualified teaching personnel. Under Article 6 of the Civil Code,
"[r]ights may be waived, unless the waiver is contrary to law, public
order, public policy, morals, or good customs, or prejudicial to a third
person with a right recognized by law." On the other hand, there
could be no acquiescence — amounting to estoppel — with respect
to acts which constitute a violation of law. "The doctrine of estoppel
cannot operate to give effect to an act which is otherwise null and
void or ultra vires. " "[N]o estoppel can be predicated on an illegal
act."

898
Page 899

Case Digests by: Vergara, Jean Colleen M.

MALLO vs. SOUTHEAST ASIAN COLLEGE, INC. (SACI)


G.R. No. 212861. October 14, 2015.
Perlas-Bernabe, J

DOCTRINE:
ABANDONMENT
Abandonment of position is a matter of intention and cannot be
lightly inferred, much less legally presumed, from certain equivocal
acts.

FACTS:
A complaint for unfair labor practice and illegal dismissal was
filed by Mallo against respondents Southeast Asian College, Inc.
(SACI) and its Executive President/CEO, Edita F. Enatsu before the
NLRC. Mallo alleged that SACI first hired him as a Probationary Full-
Time Faculty Member of its College of Nursing and Midwifery with the
rank of Asst. Professor C for the Second Semester of SY 2007-2008
and, thereafter, his employment was renewed for the next semesters
until the Summer Semester of SY 2010-2011. In June 2011, Mallo
inquired about his teaching load for the First Semester of SY 2011-
2012, However, the Dean of the College of Nursing, Dr. Curato,
simply retorted that the school was under no obligation to give him
any teaching loads for the semester because he was merely a
contractual employee. As such, Mallo was constrained to file the
instant complaint against respondents. Respondents claim that as
early as April 2011 as evidenced by Dr. Curato's letter to the Medical
Center Chief II of the National Center for Mental Health (NCMH),
SACI gave Mallo his teaching load for the First Semester of SY 2011-
2012 — as Clinical Instructor for the College of Nursing's
Preceptorship Program to be conducted at NCMH. Unfortunately,
Mallo twice failed the qualifying test required for the job. This
notwithstanding, SACI gave Mallo a teaching load by appointing him
as a Clinical Instructor for Preceptorship Program to be conducted at
the United Doctors Medical Center (UDMC) instead, beginning June
23, 2011, which he accepted. However, a day before he was set to
start as a Clinical Instructor at UDMC, Mallo asked for a change in
schedule, which was denied as it would entail a reshuffle of the entire
NLRE schedule of the school. On June 23 to 25, 2011, Mallo did not
attend his classes at UDMC which prompted SACI to contact Mallo if
he would report for work the following day, to which the latter replied
in the negative as his schedule with SACI conflicted with his new
employment. Thereafter, SACI never heard from Mallo again.

899
Page 900

ISSUE:
Whether the CA correctly ruled that Mallo abandoned his job.

RULING:
YES. While the Court concurs with the CA that Mallo was not
illegally dismissed, the Court does not agree that he had abandoned
his work. The concept of abandonment in labor law had been
thoroughly discussed in Tan Brothers Corporation of Basilan City v.
Escudero: As defined under established jurisprudence, abandonment
is the deliberate and unjustified refusal of an employee to resume his
employment. It constitutes neglect of duty and is a just cause for
termination of employment under paragraph (b) of Article 282 [now
Article 296] 46 of the Labor Code. To constitute abandonment,
however, there must be a clear and deliberate intent to
discontinue one's employment without any intention of
returning. In this regard, two elements must concur: (1) failure to
report for work or absence without valid or justifiable reason;
and (2) a clear intention to sever the employer-employee
relationship, with the second element as the more determinative
factor and being manifested by some overt acts. Otherwise
stated, absence must be accompanied by overt acts unerringly
pointing to the fact that the employee simply does not want to work
anymore. It has been ruled that the employer has the burden of proof
to show a deliberate and unjustified refusal of the employee to
resume his employment without any intention of returning.

In this case, records are bereft of any indication that Mallo's


absence from work was deliberate, unjustified, and with a clear intent
to sever his employment relationship with SACI. While respondents
claim to have assigned Mallo as Clinical Instructor at UDMC after
failing the qualifying tests at NCMH, which assignment the latter
initially accepted, but eventually declined, there is no proof that Mallo
was informed of such assignment. It bears stressing that a party
alleging a critical fact must support his allegation with substantial
evidence for any decision based on unsubstantiated allegation cannot
stand as it will offend due process.

More importantly, Mallo's filing of a complaint for illegal dismissal,


coupled with his prior acts of actively inquiring about his teaching
load, negate any intention on his part to sever his employment.
Indeed, it is simply absurd for Mallo to provide continuous service to
SACI for more than three (3) years in order to attain a regular status,
only to leave his job without any justifiable reason and, thereafter, file
a case in an attempt to recover the same. To reiterate, abandonment
of position is a matter of intention and cannot be lightly inferred, much
less legally presumed, from certain equivocal acts.

900
Page 901

Case Digests by: Vergara, Jean Colleen M.

MANARPIIS v. TEXAN PHILIPPINES, INC. (TPI)


G.R. No. 197011. January 28, 2015.
Villarama, JR., J

DOCTRINE:
ABANDONMENT
The two elements which must concur for a valid abandonment,
viz.: (1) the failure to report to work or absence without valid or
justifiable reason, and (2) a clear intention to sever the employer-
employee relationship, with the second element as the more
determinative factor being manifested by some overt acts.
Abandonment as a just ground for dismissal requires the deliberate,
unjustified refusal of the employee to perform his employment
responsibilities. Mere absence or failure to work, even after notice to
return, is not tantamount to abandonment. Furthermore, filing by an
employee of a complaint for illegal dismissal with a prayer for
reinstatement is proof enough of his desire to return to work, thus,
negating the employer's charge of abandonment. An employee who
takes steps to protest his dismissal cannot logically be said to have
abandoned his work.

FACTS:
Texan Philippines, Inc. (TPI) which is owned and managed by
Catherine Rialubin-Tan and her Singaporean husband Richard Tan
(respondents) hired Essencia Q. Manarpiis (petitioner) as Sales and
Marketing Manager of the company's Aroma Division with a monthly
salary of P33,800.00. Claiming insurmountable losses, respondents
served a written notice (July 27, 2000) addressed to all their
employees that TPI will cease operations by August 31, 2000. On
August 7, 2000, petitioner filed a complaint for illegal dismissal, non-
payment of overtime pay, holiday pay, service incentive leave pay,
unexpired vacation leave and 13th month pay and with prayer for
moral and actual damages.

ISSUE:
Whether Manarpiis abandoned her job

RULING:
NO. We have laid down the two elements which must concur for
a valid abandonment, viz.: (1) the failure to report to work or absence
without valid or justifiable reason, and (2) a clear intention to sever
the employer-employee relationship, with the second element as the
more determinative factor being manifested by some overt acts.

901
Page 902

Abandonment as a just ground for dismissal requires the deliberate,


unjustified refusal of the employee to perform his employment
responsibilities. Mere absence or failure to work, even after notice to
return, is not tantamount to abandonment. Furthermore, it is well-
settled that the filing by an employee of a complaint for illegal
dismissal with a prayer for reinstatement is proof enough of his desire
to return to work, thus, negating the employer's charge of
abandonment. An employee who takes steps to protest his dismissal
cannot logically be said to have abandoned his work.

Abandonment in this case was a trumped up charge, apparently


to make it appear that petitioner was not yet terminated when she
filed the illegal dismissal complaint and to give a semblance of truth
to the belated investigation against the petitioner. Petitioner did not
abandon her work but was told not to report for work anymore after
being served a written notice of termination of company closure on
July 27, 2000 and turning over company properties to respondent
Rialubin-Tan.

902
Page 903

Case Digests by: Vergara, Jean Colleen M.

ARABIT v. JARDINE PACIFIC FINANCE, INC.


(FORMERLY MB FINANCE)
G.R. No. 181719. April 21, 2014.
Brion, J.

DOCTRINE:
REDUNDANCY
The fact that redundancy and retrenchment are found together in
just one provision does not necessarily give rise to the conclusion
that the difference between them is immaterial. Retrenchment and
redundancy are two different concepts; they are not synonymous;
thus, they should not be used interchangeably.

In implementing redundancy, there are two levels. The first level


based on Asian Alcohol is broader as the case recognized
distinctions on a per position basis. The second level, derived from
Golden Thread, is more specific where the distinction narrows down
to the particular employees occupying the same positions which were
already declared to be redundant.

FACTS:
Petitioners were former regular employees of respondent Jardine
Pacific Finance, Inc. (formerly MB Finance). On the claim of financial
losses, Jardine decided to reorganize and implement a redundancy
program among its employees. The petitioners were among those
affected by the redundancy program. Jardine thereafter hired
contractual employees to undertake the functions these employees
used to perform. The LA (as affirmed by the NLRC) ruled that hiring
of contractual employees to replace the petitioners directly
contradicts the concept of redundancy which involves the trimming
down of the workforce because a task is being carried out by too
many people. The CA reversed the rulings of the lower courts
justifying the act of Jardine as that of a management prerogative.

When petitioners argued that there is a difference between


financial loss and decline of earnings, Jardine claimed that the
distinction between redundancy and retrenchment is not material
contending that employers resort to these causes of dismissal for
purely economic considerations. Jardine added that the immateriality
of the distinction between these just causes for dismissal is shown by
the fact that redundancy and retrenchment are found and lumped
together in just one single provision of the Labor Code (Article 283).

903
Page 904

ISSUE(1):
Whether the distinction between redundancy and retrenchment is not
material for being lumped together in just one single provision of the
Labor Code (Article 283)

RULING:
NO. The fact that redundancy and retrenchment are found
together in just one provision does not necessarily give rise to the
conclusion that the difference between them is immaterial.
Retrenchment and redundancy are two different concepts; they are
not synonymous; thus, they should not be used interchangeably.

In Andrada, et al., v. NLRC, citing the case of Sebuguero v.


NLRC, this Court clarified: Redundancy exists where the services of
an employee are in excess of what is reasonably demanded by the
actual requirements of the enterprise. A position is redundant where it
is superfluous, and superfluity of a position or positions may be the
outcome of a number of factors, such as overhiring of workers,
decreased volume of business, or dropping of a particular product
line or service activity previously manufactured or undertaken by the
enterprise. Retrenchment, on the other hand, is used interchangeably
with the term "lay-off." It is the termination of employment initiated by
the employer through no fault of the employee's and without prejudice
to the latter, resorted to by management during periods of business
recession, industrial depression, or seasonal fluctuations, or during
lulls occasioned by lack of orders, shortage of materials, conversion
of the plant for a new production program or the introduction of new
methods or more efficient machinery, or of automation. Simply put, it
is an act of the employer of dismissing employees because of losses
in the operation of a business, lack of work, and considerable
reduction on the volume of his business, a right consistently
recognized and affirmed by this Court.

These rulings appropriately clarify that redundancy does not


need to be always triggered by a decline in the business. Primarily,
employers resort to redundancy when the functions of an employee
have already become superfluous or in excess of what the business
requires. Thus, even if a business is doing well, an employer can still
validly dismiss an employee from the service due to redundancy if
that employee's position has already become in excess of what the
employer's enterprise requires. From this perspective, it is illogical for
Jardine to terminate the petitioners' employment and replace them
with contractual employees. The replacement effectively belies
Jardine's claim that the petitioners' positions were abolished due to
superfluity. Redundancy could have been justified if the functions of
the petitioners were transferred to other existing employees of the
company. To dismiss the petitioners and hire new contractual

904
Page 905

employees as replacements necessarily give rise to the sound


conclusion that the petitioners' services have not really become in
excess of what Jardine's business requires.

ISSUE(2):
Whether redundancy was properly implemented by Jardine in
dismissing the petitioners

RULING:
NO. The employer must follow certain guidelines to dismiss
employees due to redundancy. These guidelines aim to ensure that
the dismissal is not implemented arbitrarily and is not tainted with bad
faith against the dismissed employees. In Golden Thread Knitting
Industries, Inc. v. NLRC, this Court laid down the principle that the
employer must use fair and reasonable criteria in the selection of
employees who will be dismissed from employment due to
redundancy. Such fair and reasonable criteria may include the
following, but are not limited to: (a) less preferred status (e.g.,
temporary employee); (b) efficiency; and (c) seniority. The presence
of these criteria used by the employer shows good faith on its part
and is evidence that the implementation of redundancy was
painstakingly done by the employer in order to properly justify the
termination from the service of its employees. While in Asian Alcohol
Corp. v. NLRC, the court also laid down guidelines for redundancy to
be characterized as validly undertaken by the employer: For the
implementation of a redundancy program to be valid, the employer
must comply with the following requisites: (1) written notice served on
both the employees and the Department of Labor and Employment at
least one month prior to the intended date of retrenchment; (2)
payment of separation pay equivalent to at least one month pay or at
least one month pay for every year of service, whichever is higher; (3)
good faith in abolishing the redundant positions; and (4) fair and
reasonable criteria in ascertaining what positions are to be
declared redundant and accordingly abolished.

Based on the guidelines in Golden Thread and Asian Alcohol, we


find that at two levels, Jardine failed to set the required fair and
reasonable criteria in the termination of the petitioners' employment
and thus was arbitrary and in bad faith. The first level, based on
Asian Alcohol, where the case recognized distinctions on a per
position basis, Jardine failed to explain why it chose the petitioners'
posts as the ones which have already become redundant and
terminable. The second more specific level derived from Golden
Thread, where the distinction narrows down to the particular
employees occupying the same positions already declared to be
redundant, Jardine failed to explain why among all of its employees,
the petitioners were the ones selected to be dismissed.

905
Page 906

Case Digests by: Vergara, Jean Colleen M.

SHIMIZU PHILS. CONTRACTORS, INC. v. CALLANTA


G.R. No. 165923. September 29, 2010.
Del Castillo, J

DOCTRINE:
REDUNDANCY
The purpose of the one-month prior notice rule is to give DOLE
an opportunity to ascertain the veracity of the cause of termination.
Non-compliance with this rule clearly violates the employee's right to
statutory due process.

FACTS:
Petitioner, a corporation engaged in the construction business,
employed respondent on August 23, 1994 as Safety Officer assigned
at petitioner's Yutaka-Giken Project and eventually as Project
Administrator of petitioner's Structural Steel Division in 1995. In a
Memorandum dated June 7, 1997, respondent was informed that his
services will be terminated effective July 9, 1997 due to the lack of
any vacancy in other projects and the need to re-align the company's
personnel requirements brought about by the imperatives of
maximum financial commitments. Respondent then filed an illegal
dismissal complaint against petitioner assailing his dismissal as
without any valid cause.

Petitioner advanced that respondent's services was terminated


in accordance with a valid retrenchment program being implemented
by the company since 1996 due to financial crisis that plague the
construction industry. Respondent claimed that petitioner failed to
comply with the requirements called for by law before implementing a
retrenchment program thereby rendering it legally infirmed. First, it
did not comply with the provision of the Labor Code mandating the
service of notice of retrenchment. He pointed out that the notice sent
to him never mentioned retrenchment but only project completion as
the cause of termination. Also, the notice sent to the Department of
Labor and Employment (DOLE) did not conform to the 30-day prior
notice requirement. Second, petitioner failed to use fair and
reasonable criteria in determining which employees shall be
retrenched or retained. As shown in the termination report submitted
to DOLE, he was the only one dismissed out of 333 employees.

ISSUE:
Whether petitioner corporation complied with the statutory due
process in dismissing the respondent due to an authorized cause.

906
Page 907

RULING:
NO. Although there was authorized cause to dismiss respondent
from the service, we find that petitioner did not comply with the 30-
day notice requirement. Petitioner maintains that it substantially
complied with the requirement of the law in that it, in fact, submitted
two notices or reports with the DOLE. However, petitioner admitted
that the reports were submitted 21 days, in the case of the first notice,
and 16 days, in the case of the second notice, before the intended
date of respondent's dismissal. The purpose of the one month prior
notice rule is to give DOLE an opportunity to ascertain the veracity of
the cause of termination. Non- compliance with this rule clearly
violates the employee's right to statutory due process. To be
consistent with our ruling in Jaka Food Processing Corporation v.
Pacot, the indemnity in the form of nominal damages should be fixed
in the amount of P50,000.00.

907
Page 908

Case Digests by: Vergara, Jean Colleen M.

SPI TECHNOLOGIES, INC. v. MAPUA


G.R. No. 191154. April 7, 2014.
Reyes, J.

DOCTRINE:
REDUNDANCY
The Court remains steadfast on its stand that the determination
of the continuing necessity of a particular officer or position in a
business corporation is a management prerogative, and the courts
will not interfere unless arbitrary or malicious action on the part of
management is shown.

FACTS:
Victoria K. Mapua (Mapua) alleged that she was hired in 2003 by
SPI Technologies, Inc. (SPI) and was the Corporate Development's
Research/Business Intelligence Unit Head and Manager of the
company. On March 21, 2007, Raina informed Mapua over the phone
that her position was considered redundant and that she is
terminated from employment effective immediately. Villanueva
notified Mapua that she should cease reporting for work the next day.
Her laptop computer and company mobile phone were taken right
away and her office phone ceased to function.

On March 27, 2007, Mapua filed with the Labor Arbiter (LA) a
complaint for illegal dismissal, claiming reinstatement or if deemed
impossible, for separation pay. Afterwards, she went to a meeting
with SPI, where she was given a second termination letter, the
contents of which were similar to the first one. On April 25, 2007,
Mapua received through mail, a third Notice of Termination dated
March 21, 2007 but the date of effectivity of the termination was
changed from March 21 to April 21, 2007. It further stated that her
separation pay will be released on May 20, 2007 and a notation was
inscribed, "refused to sign and acknowledge" with unintelligible
signatures of witnesses.

ISSUE:
Whether Mapua was accorded due process in her dismissal due
to redundancy.

RULING:
NO. Article 283 of the Labor Code provides:

908
Page 909

ART. 283. Closure of establishment and reduction of personnel. —


The employer may also terminate the employment of any employee
due to installation of labor-saving devices, redundancy,
retrenchment to prevent losses or the closing or cessation of
operation of the establishment or undertaking unless the closing is for
the purpose of circumventing the provisions of this Title, by serving a
written notice on the worker and the Department of Labor and
Employment at least one (1) month before the intended date
thereof. In case of termination due to installation of labor- saving
devices or redundancy, the worker affected thereby shall be
entitled to a separation pay equivalent to at least one (1) month pay
or to at least one (1) month pay for every year of service, whichever
is higher. In case of retrenchment to prevent losses and in cases of
closures or cessation of operations of establishment or undertaking
not due to serious business losses and financial reverses, the
separation pay shall be equivalent to one (1) month pay or at least
one-half (1/2) month pay for every year of service, whichever is
higher. A fraction of at least six (6) months shall be considered as
one (1) whole year.

Anent the first requirement which is written notice served on both


the employee and the DOLE at least one month prior to the intended
date of termination, SPI had discharged the burden of proving that it
submitted a notice to the DOLE on March 21, 2007, stating therein
that the effective date of termination is on April 21, 2007. It is,
however, quite peculiar that two kinds of notices were served to
Mapua. One termination letter stated that its date of effectivity is on
the same day, March 21, 2007. The other termination letter sent
through mail to Mapua's residence stated that the effective date of
her termination is on April 21, 2007. Hence, SPI must shoulder the
consequence of causing the confusion brought by the variations of
termination letters given to Mapua. Award of moral and exemplary
damages for an illegally dismissed employee is proper where the
employee had been harassed and arbitrarily terminated by the
employer. Here, the Court awarded the amount of P50,000.00 each
as moral and exemplary damages.

909
Page 910

Case Digests by: Vergara, Jean Colleen M.

CULILI v. EASTERN TELECOMMUNICATIONS


PHILIPPINES, INC. (ETPI)
G.R. No. 165381. February 9, 2011.
Leonardo-De Castro, J.

DOCTRINE:
REDUNDANCY
Among the requisites of a valid redundancy program are: (1) the
good faith of the employer in abolishing the redundant position; and
(2) fair and reasonable criteria in ascertaining what positions are to
be declared redundant, such as but not limited to: preferred status,
efficiency, and seniority. The following evidence may be proffered to
substantiate redundancy: the new staffing pattern, feasibility
studies/proposal on the viability of the newly created positions, job
description and the approval by the management of the restructuring.

FACTS:
Respondent Eastern Telecommunications Philippines, Inc.
(ETPI) is a telecommunications company engaged mainly in the
business of establishing commercial telecommunications systems
and leasing of international datalines or circuits that pass through the
international gateway facility (IGF). Petitioner Nelson A. Culili was
employed by ETPI as a Technician in its Field Operations
Department on January 27, 1981. In 1998, due to business troubles
and losses, ETPI was compelled to implement a Right-Sizing
Program which consisted of two phases: the first phase involved the
reduction of ETPI's workforce to only those employees that were
necessary and which ETPI could sustain; the second phase entailed
a company-wide reorganization which would result in the transfer,
merger, absorption or abolition of certain departments of ETPI. As
part of the first phase, ETPI, on December 10, 1998, offered to its
employees who had rendered at least fifteen years of service, the
Special Retirement Program, which consisted of the option to
voluntarily retire at an earlier age and a retirement package
equivalent to two and a half (2 1/2) months' salary for every year of
service. This offer was initially rejected by the Eastern
Telecommunications Employees' Union (ETEU), ETPI's duly
recognized bargaining agent, which threatened to stage a strike.
ETPI explained to ETEU the exact details of the Right-Sizing
Program and the Special Retirement Program and after consultations
with ETEU's members, ETEU agreed to the implementation of both
programs. Thus, on February 8, 1999, ETPI re-offered the Special
Retirement Program and the corresponding retirement package to the

910
Page 911

120 qualified employees for the program. Of all the employees who
qualified to avail of the program, only Culili rejected the offer.

The second phase necessitated the abolition, transfer, and


merger of a number of ETPI's departments. Among the departments
abolished was Culili's unit. As a result, Culili's position was abolished
due to redundancy.

ISSUE:
Whether ETPI complied with the requisites of a valid redundancy
program

RULING:
YES. An employer cannot simply declare that it has become
overmanned and dismiss its employees without producing adequate
proof to sustain its claim of redundancy. Among the requisites of a
valid redundancy program are: (1) the good faith of the employer in
abolishing the redundant position; and (2) fair and reasonable criteria
in ascertaining what positions are to be declared redundant, such as
but not limited to: preferred status, efficiency, and seniority. This
Court also held that the following evidence may be proffered to
substantiate redundancy: the new staffing pattern, feasibility
studies/proposal on the viability of the newly created positions, job
description and the approval by the management of the restructuring.

In the case at bar, ETPI was upfront with its employees about its
plan to implement a Right-Sizing Program. Even in the face of initial
opposition from and rejection of the said program by ETEU, ETPI
patiently negotiated with ETEU's officers to make them understand
ETPI's business dilemma and its need to reduce its workforce and
streamline its organization. This evidently rules out bad faith on the
part of ETPI.

In deciding which positions to retain and which to abolish, ETPI


chose on the basis of efficiency, economy, versatility and flexibility. It
needed to reduce its workforce to a sustainable level while
maintaining functions necessary to keep it operating. The records
show that ETPI had sufficiently established not only its need to
reduce its workforce and streamline its organization, but also the
existence of redundancy in the position of a Senior Technician. ETPI
explained how it failed to meet its business targets and the factors
that caused this, and how this necessitated it to reduce its workforce
and streamline its organization. ETPI also submitted its old and new
tables of organization and sufficiently described how limited the
functions of the abolished position of a Senior Technician were and
how it decided on whom to absorb these functions.

911
Page 912

Case Digests by: Vergara, Jean Colleen M.

ASUFRIN v. SAN MIGUEL CORPORATION


G.R. No. 156658. March 10, 2004.
Ynares-Santiago, J.

DOCTRINE:
REDUNDANCY
Whether it be by redundancy or retrenchment or any of the other
authorized causes, no employee may be dismissed without
observance of the fundamentals of good faith.

FACTS:
Coca Cola Plant, then a department of respondent San Miguel
Beer Corporation (SMC), hired petitioner as a utility/miscellaneous
worker in February 1972. On November 16, 1981, he became a
monthly paid employee promoted as Stock Clerk. Sometime in 1984,
the sales office and operations at the Sum-ag, Bacolod City Sales
Office were reorganized. Several positions were abolished including
petitioner’s position as Stock Clerk. After reviewing petitioner’s
qualifications, he was designated warehouse checker at the Sum-ag
Sales Office. On April 1, 1996, respondent SMC implemented a new
marketing system known as the “pre-selling scheme” at the Sum-ag
Beer Sales Office. As a consequence, all positions of route sales and
warehouse personnel were declared redundant. Respondent notified
the DOLE Director of Region VI that personnel of the Sales
Department of the Negros Operations Center 1 would be retired
effective March 31, 1995. SMC thereafter wrote a letter to petitioner
informing him that, owing to the implementation of the “pre-selling
operations” scheme, all positions of route and warehouse personnel
will be declared redundant and the Sum-ag Sales Office will be
closed effective April 30, 1996. Thus, from April 1, 1996 to May 15,
1996, petitioner reported to SMC’s Personnel Department at the Sta.
Fe Brewery, pursuant to a previous directive. Thereafter, the
employees of Sum-ag sales force were informed that they can avail
of respondent’s early retirement package pursuant to the
retrenchment program, while those who will not avail of early
retirement would be redeployed or absorbed at the Brewery or other
sales offices. Petitioner opted to remain and manifested to Acting
Personnel Manager Abadesco his willingness to be assigned to any
job, considering that he had 3 children in college. Petitioner was
surprised when he was informed by Abadesco that his name was
included in the list of employees who availed of the early retirement
package. Petitioner then filed a complaint for illegal dismissal.

912
Page 913

ISSUE:
Whether or not the dismissal of petitioner is based on a just and
authorized cause

RULING:
NO. The determination that employee's services are no longer
necessary or sustainable and, therefore, properly terminable is an
exercise of business judgment of the employer. The wisdom or
soundness of this judgment is not subject to discretionary review of
the LA and the NLRC, provided there is no violation of law and no
showing that it was prompted by an arbitrary or malicious act. In other
words, t is not enough for a company to merely declare that it has
become overmanned. It must produce adequate proof that such is the
actual situation to justify the dismissal of the affected employees for
redundancy.

Persuasive as the explanation proffered by respondent to justify


the dismissal of petitioner, a number of disturbing circumstances
leave us unconvinced. First, of the SMC employees assigned at the
Sum-ag Sales Office/Warehouse, petitioner was among those who
did not accept the offer of early retirement and only him clearly
manifested, through several letters, that he was even willing to accept
a demotion just to continue his employment. Second, Petitioner was
in the payroll of the Sta. Fe Brewery although he was actually posted
at the Sum-ag Warehouse. Thus, even assuming that his position in
the Sum- ag Warehouse became redundant, he should have been
returned to the Sta. Fe Brewery where he was actually assigned and
where there were vacant positions to accommodate him.

In selecting employees to be dismissed, a fair and reasonable


criteria must be used, such as but not limited to (a) less preferred
status, e.g. temporary employee; (b) efficiency; and (c) seniority. In
the case at bar, no criterion whatsoever was adopted by respondent
in dismissing petitioner. Furthermore, respondent has not shown how
the cessation of operations of the Sum-ag Sales Office contributed to
the ways and means of improving effectiveness of the organization
with the end in view of efficiency and cutting distribution overhead
and other related costs. Respondent, thus, clearly resorted to
sweeping generalization[s] in dismissing complainant.

Indeed, petitioner's predicament may have something to do with


an incident where he incurred the ire of an immediate superior in the
Sales Logistics Unit for exposing certain irregularities committed by
the latter. It bears stressing that whether it be by redundancy or
retrenchment or any of the other authorized causes, no employee
may be dismissed without observance of the fundamentals of good
faith.

913
Page 914

Case Digest by: BACOLOD, JAMIELOU C.

OCEAN EAST AGENCY CORP., v. LOPEZ


G.R. No. 194410. October 14, 2015.
Peralta, C. J.

DOCTRINE:
ARTICLE 298 REDUNDANCY
The presence of fair and reasonable criteria used by the employer
shows good faith on its part and is evidence that the implementation of
redundancy was painstakingly done by the employer in order to
properly justify the termination from the service of its employees.
Conversely, the absence of criteria in the selection of an employee to
be dismissed and the erroneous implementation of the criterion
selected, both render invalid the redundancy because both have the
ultimate effect of illegally dismissing an employee.

An employer cannot simply declare that it has become


overmanned and dismiss its employees without adequate proof to
sustain its claim of redundancy. The following evidence may be
proffered to substantiate redundancy, to wit: the new staffing pattern,
feasibility studies/proposal on the viability of the newly-created
positions, job description and the approval by the management of the
restructuring.

FACTS:
Petitioner Ocean East Agency Corporation (Ocean East) is a
manning agency engaged in recruitment and deployment of Filipino
seamen for overseas principals. Respondent Allan I. Lopez was
employed as Documentation Officer assigned to Ocean East's
Operations Department. Prior to his employment, Ocean East had
already engaged the services of one Grace Reynolds as
Documentation Clerk.

The Documentation Clerks and Officer were tasked to perform the


following functions: prepare the line-up of request crew by various
principals in close coordination with the Port Captain; assist in
attending to various operational expenses and disbursements;
coordinate closely with deserving former crew members for pooling
and/or immediate employment, if so required; and supervise the
preparation of the crew documents, such as travel documents and
clearances.

914
Page 915

Ocean East served notice to Lopez that effective thirty (30) days
later, his services will be terminated on the ground of redundancy, as
his position as Documentation Officer is but a duplication of those
occupied by its two (2) other personnel who were also exercising
similar duties and functions. Lopez filed a complaint for illegal
dismissal.

Petitioners claim to have clearly established that the functions of


Lopez as a Documentation Officer is virtually a duplication of the duties
and responsibilities performed by Ocean East's two (2) other
Documentation Clerks.

ISSUE:
Whether respondent was legally dismissed.

RULING:
No. Redundancy exists when the service capability of the
workforce is in excess of what is reasonably needed to meet the
demands of the enterprise. For the implementation of a redundancy
program to be valid, the employer must comply with these requisites:
(1) written notice served on both the employee and the Department of
Labor and Employment at least one month prior to the intended date
of retrenchment; (2) payment of separation pay equivalent to at least
one month pay or at least one month pay for every year of service,
whichever is higher; (3) good faith in abolishing the redundant
positions; and (4) fair and reasonable criteria in ascertaining what
positions are to be declared redundant and accordingly abolished.

The presence of fair and reasonable criteria used by the employer


shows good faith on its part and is evidence that the implementation of
redundancy was painstakingly done by the employer in order to
properly justify the termination from the service of its employees.
Conversely, the absence of criteria in the selection of an employee to
be dismissed and the erroneous implementation of the criterion
selected, both render invalid the redundancy because both have the
ultimate effect of illegally dismissing an employee.

An employer cannot simply declare that it has become


overmanned and dismiss its employees without adequate proof to
sustain its claim of redundancy. The following evidence may be
proffered to substantiate redundancy, to wit: the new staffing pattern,
feasibility studies/proposal on the viability of the newly-created
positions, job description and the approval by the management of the
restructuring.

915
Page 916

In this case, petitioners were able to establish through Ocean


East's Quality Procedures Manual that Lopez' position as a
Documentation Officer was redundant because its duties and functions
were similar to those of the Documentation Clerks in its operations
department. However, they failed to prove by substantial evidence their
observance of the fair and reasonable criteria of seniority and
efficiency in ascertaining the redundancy of the position of
Documentation Officer, as well as good faith on their part in abolishing
such position.

916
Page 917

Case Digest by: Bacolod, Jamielou C.

ASIAN ALCOHOL CORPORATION v. NLRC


G.R. No. 131108. March 25, 1999.
Puno, J.

DOCTRINE:
ARTICLE 298 RETRENCHMENT
The requirements for valid retrenchment which must be proved by
clear and convincing evidence are: (1) that the retrenchment is
reasonably necessary and likely to prevent business losses which, if
already incurred, are not merely de minimis, but substantial, serious,
actual and real, or if only expected, are reasonably imminent as
perceived objectively and in good faith by the employer; (2) that the
employer served written notice both to the employees and to the
Department of Labor and Employment at least one month prior to the
intended date of retrenchment; (3) that the employer pays the
retrenched employees separation pay equivalent to one month pay or
at least 1/2 month pay for every year of service, whichever is higher;
(4) that the employer exercises its prerogative to retrench employees
in good faith for the advancement of its interest and not to defeat or
circumvent the employees' right to security of tenure; and (5) that the
employer used fair and reasonable criteria in ascertaining who would
be dismissed and who would be retained among the employees, such
as status (i.e., whether they are temporary, casual, regular or
managerial employees),efficiency, seniority, physical fitness, age, and
financial hardship for certain workers.The presence of fair and
reasonable criteria used by the employer shows good faith on its part
and is evidence that the implementation of redundancy was
painstakingly done by the employer in order to properly justify the
termination from the service of its employees.

FACTS:
In September, 1991, the Parsons family, who originally owned the
controlling stocks in Asian Alcohol, were driven by mounting business
losses to sell their majority rights to Prior Holdings, Inc. The next
month, Prior Holdings took over its management and operation.

To thwart further losses, Prior Holdings implemented a


reorganizational plan and other cost-saving measures. Some one
hundred seventeen (117) employees out of a total workforce of three
hundred sixty (360) were separated. Seventy two (72) of them
occupied redundant positions that were abolished. The six (6) private
respondents are among those union members 5 whose positions were

917
Page 918

abolished due to redundancy. They were all assigned at the Repair


and Maintenance Section of the Pulupandan plant.

On December 18, 1992, the six (6) private respondents filed with
the NLRC complaints for illegal dismissal with a prayer for
reinstatement with backwages, moral damages and attorney’s
fees. They alleged that Asian Alcohol used the retrenchment program
as a subterfuge for union busting. They claimed that they were singled
out for separation by reason of their active participation in the union.
They also asseverated that Asian Alcohol was not bankrupt as it has
engaged in an aggressive scheme of contractual hiring.

ISSUE:
Whether or not the respondents were validly dismissed.

RULING:
Yes. The requirements for valid retrenchment which must be
proved by clear and convincing evidence are: (1) that the retrenchment
is reasonably necessary and likely to prevent business losses which, if
already incurred, are not merely de minimis, but substantial, serious,
actual and real, or if only expected, are reasonably imminent as
perceived objectively and in good faith by the employer; (2) that the
employer served written notice both to the employees and to the
Department of Labor and Employment at least one month prior to the
intended date of retrenchment; (3) that the employer pays the
retrenched employees separation pay equivalent to one month pay or
at least ½ month pay for every year of service, whichever is higher; (4)
that the employer exercises its prerogative to retrench employees in
good faith for the advancement of its interest and not to defeat or
circumvent the employees’ right to security of tenure; and (5) that the
employer used fair and reasonable criteria in ascertaining who would
be dismissed and who would be retained among the employees, such
as status (i.e., whether they are temporary, casual, regular or
managerial employees), efficiency, seniority, physical fitness, age, and
financial hardship for certain workers.

It should be observed that Article 283 of the Labor Code uses the
phrase "retrenchment to prevent losses." In its ordinary connotation,
this phrase means that retrenchment must be undertaken by the
employer before losses are actually sustained. The Court have
interpreted the law to mean that the employer need not keep all his
employees until after his losses shall have materialized. Otherwise,
the law could be vulnerable to attack as undue taking of property for
the benefit of another.

918
Page 919

In the case at bar, Prior Holdings took over the operations of Asian
Alcohol in October 1991. Plain to see, the last quarter losses in 1991
were already incurred under the new management. There were no
signs that these losses would abate. Irrefutable was the fact that losses
have bled Asian Alcohol incessantly over a span of several years. They
were incurred under the management of the Parsons family and
continued to be suffered under the new management of Prior Holdings.
Ultimately, it is Prior Holdings that will absorb all the losses, including
those incurred under the former owners of the company. The law gives
the new management every right to undertake measures to save the
company from bankruptcy.

In the case at bar, private respondents failed to proffer any proof


that the management acted in a malicious or arbitrary manner in
engaging the services of an independent contractor to operate the
Laura wells. Absent such proof, the Court has no basis to interfere with
the bona fide decision of management to effect more economic and
efficient methods of production.

919
Page 920

Case Digest by: BACOLOD, JAMIELOU C.

FASAP v. PHILIPPINE AIRLINES


G.R. No. Ja178083. March 13, 2018.
Peralta, C. J.

DOCTRINE:
ARTICLE 298 RETRENCHMENT.
In determining the validity of a retrenchment, judicial notice may
be taken of the financial losses incurred by an employer undergoing
corporate rehabilitation. In such a case, the presentation of audited
financial statements may not be necessary to establish that the
employer is suffering from severe financial losses.

FACTS:
The Third Division thereby differed from the decision of the Court
of Appeals (CA), which had pronounced in its appealed decision that
the remaining issue between the parties concerned the manner by
which PAL had carried out the retrenchment program. Instead, the
Third Division disbelieved the veracity of PAL’s claim of severe
financial losses, and concluded that PAL had not established its severe
financial losses because of its non-presentation of audited financial
statements. It further concluded that PAL had implemented the
retrenchment program in bad faith, and had not used fair and
reasonable criteria in selecting the employees to be retrenched.

Not satisfied, PAL filed the Motion for Reconsideration, in its motion
PAL insists that considering that the Labor Arbiter, the NLRC and the
CA unanimously found PAL to have experienced financial losses, the
Court should have accorded such unanimous findings with respect and
finality; that its being placed under suspension of payments and
corporate rehabilitation and receivership already sufficiently indicated
its grave financial condition; and that the Court should have also taken
judicial notice of the suspension of payments and monetary claims filed
against PAL that had reached and had been consequently resolved by
the Court PAL describes the Court's conclusion that it was not suffering
from tremendous financial losses because it was on the road to
recovery a year after the retrenchment as a mere obiter dictum that
was relevant only in rehabilitation proceedings.

ISSUE:
Whether there was a valid retrenchment.

RULING:

920
Page 921

Yes. After having been placed under corporate rehabilitation and


its rehabilitation plan having been approved by the SEC on June 23,
2008, PAL’s dire financial predicament could not be doubted.
Incidentally, the SEC’s order of approval came a week after PAL had
sent out notices of termination to the affected employees. It is thus
difficult to ignore the fact that PAL had then been experiencing difficulty
in meeting its financial obligations long before its rehabilitation.

In that regard, Section 2, Rule 129 of the Rules of


Court recognizes that the courts have discretionary authority to take
judicial notice of matters that are of public knowledge, or are capable
of unquestionable demonstration, or ought to be known to judges
because of their judicial functions. The principle is based on
convenience and expediency in securing and introducing evidence on
matters that are not ordinarily capable of dispute and are not bona
fide disputed.

Indeed, the Labor Arbiter properly took cognizance of PAL’s


substantial financial losses during the Asian financial crisis of 1997. On
its part, the NLRC recognized the grave financial distress of PAL based
on its ongoing rehabilitation/receivership. The CA likewise found that
PAL had implemented a retrenchment program to counter its
tremendous business losses that the strikes of the pilot's union had
aggravated. Such recognitions could not be justly ignored or denied,
especially after PAL's financial and operational difficulties had
attracted so much public attention that even President Estrada had to
intervene in order to save PAL as the country’s flag carrier.

The presentation of the audited financial statements should not the


sole means by which to establish the employer's serious financial
losses. The presentation of audited financial statements, although
convenient in proving the unilateral claim of financial losses, is not
required for all cases of retrenchment. The evidence required for each
case of retrenchment really depends on the particular circumstances
obtaining.

In short, to require a distressed corporation placed under


rehabilitation or receivership to still submit its audited financial
statements may become unnecessary or superfluous.

921
Page 922

Case Digest by: BACOLOD, JAMIELOU C.

SEBUGUERO v. NLRC
G.R. No. 115394. September 27, 1995.
Davide, Jr.,J.

DOCTRINE:
ARTICLE 298 RETRENCHMENT.
Under Article 283 of the Labor Code, there are three basic
requisites for a valid retrenchment:
(1) the retrenchment is necessary to prevent losses and such
losses are proven;
(2) written notice to the employees and to the Department of Labor
and Employment at least one month prior to the intended date of
retrenchment; and
(3) payment of separation pay equivalent to one month pay or at
least 1/2 month pay for every year of service, whichever is higher.

The lack of written notice to the petitioners and to the DOLE does
not, however, make the petitioners' retrenchment illegal such that they
are entitled to the payment of back wages and separation pay in lieu
of reinstatement as they contend. Their retrenchment, for not having
been effected with the required notices, is merely defective.

FACTS:
The petitioners were among the thirty-eight (38) regular
employees of private respondent GTI Sportswear Corporation (GTI), a
corporation engaged in the manufacture and export of ready-to-wear
garments, who were given "temporary lay-off" notices by the latter on
22 January 1991 due to alleged lack of work and heavy losses caused
by the cancellation of orders from abroad and by the garments
embargo of 1990.

Believing that their "temporary lay-off" was a ploy to dismiss


them, that GTI resorted to because of their union activities and was in
violation of their right to security of tenure since there was no valid
ground therefor, the 38 laid-off employees filed with the Labor Arbiter's
office in the National Capital Region complaints for illegal dismissal,
unfair labor practice, underpayment of wages, and non-payment of
overtime pay and 13th month pay.

Private respondent GTI denied the claim of illegal dismissal and


asserted that it was its prerogative to lay-off its employees temporarily
for a period not exceeding six months to prevent losses due to lack of

922
Page 923

work or job orders from abroad, and that the lay-off affected both union
and non-union members. It justified its failure to recall the 38 laid-off
employees after the lapse of six months because of the subsequent
cancellations of job orders made by its foreign principals, a fact which
was communicated to the petitioners and the other complainants who
were all offered their severance pay.

ISSUE:
Whether petitioners were validly retrenched or dismissed from
their employment.

RULING:

Yes. Under Article 283 of the Labor Code, there are three basic
requisites for a valid retrenchment:(1) the retrenchment is necessary
to prevent losses and such losses are proven; (2) written notice to the
employees and to the Department of Labor and Employment at least
one month prior to the intended date of retrenchment; and(3) payment
of separation pay equivalent to one month pay or at least 1/2 month
pay for every year of service, whichever is higher.

In this case, it is undisputed that the petitioners were given notice


of the temporary lay-off. There is, however, no evidence that any
written notice to permanently retrench them was given at least one
month prior to the date of the intended retrenchment. The NLRC found
that GTI conveyed to the petitioners the impossibility of recalling them
due to the continued unavailability of work. But what the law requires
is a written notice to the employees concerned and that requirement is
mandatory. That they were already on temporary lay-off at the time
notice should have been given to them is not an excuse to forego the
one-month written notice because by this time, their lay-off is to
become permanent and they were definitely losing their employment.

The lack of written notice to the petitioners and to the DOLE does
not, however, make the petitioners' retrenchment illegal such that they
are entitled to the payment of back wages and separation pay in lieu
of reinstatement as they contend. Their retrenchment, for not having
been effected with the required notices, is merely defective. In those
cases where we found the retrenchment to be illegal and ordered the
employees' reinstatement and the payment of back wages, the validity
of the cause for retrenchment, that is the existence of imminent or
actual serious or substantial losses, was not proven. But here, such a
cause is present as found by both the Labor Arbiter and the NLRC.
There is only a violation by GTI of the procedure prescribed in Article
283 of the Labor Code in effecting the retrenchment of the petitioner.

923
Page 924

It is now settled that where the dismissal of an employee is in fact


for a just and valid cause and is so proven to be but he is not accorded
his right to due process, i.e., he was not furnished the twin
requirements of notice and the opportunity to be heard, the dismissal
shall be upheld but the employer must be sanctioned for non-
compliance with the requirements of or for failure to observe due
process.

924
Page 925

Case Digest by: BACOLOD, JAMIELOU C.

BLUE EAGLE MANAGEMENT, INC., v. NAVAL


G.R. No. 192488. April 19, 2016
Leonardo- De Castro, J.

DOCTRINE:
ARTICLE 298 RETRENCHMENT.
The requirements for valid retrenchment which must be proved by
clear and convincing evidence are:
(1) that the retrenchment is reasonably necessary and likely to prevent
business losses which, if already incurred, are not merely de minimis,
but substantial, serious, actual and real, or if only expected, are
reasonably imminent as perceived objectively and in good faith by the
employer;
(2) that the employer served written notice both to the employees and
to the Department of Labor and Employment at least one month prior
to the intended date of retrenchment;
(3) that the employer pays the retrenched employees separation pay
equivalent to one month pay or at least 1/2 month pay for every year
of service, whichever is higher;
(4) that the employer exercises its prerogative to retrench employees
in good faith for the advancement of its interest and not to defeat or
circumvent the employees' right to security of tenure; and
(5) that the employer used fair and reasonable criteria in ascertaining
who would be dismissed and who would be retained among the
employees, such as status (i.e., whether they are temporary, casual,
regular or managerial employees), efficiency, seniority, physical
fitness, age, and financial hardship for certain workers.

FACTS:
Petitioner BEMI suffered financial losses, so in an attempt to reduce its
financial losses, petitioner BEMI resolved to decrease the operational
expenses of the company. Since the gross income of petitioner BEMI
was not even enough to cover the costs of the salaries, wages, and
other benefits of its employees, one of the measures the management
intended to implement was the downsizing of its workforce. Pursuant
to such decision, petitioners Bonoan and Dela Rama as the general
manager and human resource manager, evaluated and identified
several employees who could be the subject of retrenchment
proceedings, taking into consideration the employees' positions and
tenures at petitioner BEMI. After their evaluation, petitioners identified
five employees for retrenchment. Respondent was included in the list
because she was one of the employees with the shortest tenures.

925
Page 926

Before actually commencing retrenchment proceedings, petitioner


Dela Rama separately met with each of the five aforementioned
employees and presented to them the option of resigning instead. The
employees who would choose to resign would no longer be required to
report for work after their resignation but would still be paid their full
salary and their pro-rated 13th month pay, plus financial assistance in
the amount of one month salary for every year of service at petitioner
BEMI. This option would also give the employees free time to seek
other employment while still receiving salary from petitioner BEMI.
Since all the five employees identified for retrenchment decided to
voluntarily resign instead and avail themselves of the financial package
offered by petitioner BEMI, there was no more need for the company
to initiate retrenchment proceedings.

But because respondent was finding it difficult to find new employment,


she asked if it was possible for her to return to work for petitioner BEMI.
However, petitioner Bonoan replied that respondent's resignation had
long been approved and that petitioner BEMI would not be able to
rehire respondent given the difficult financial position of the company.
Petitioner Bonoan advised respondent to just receive the amount she
was entitled to by reason of her voluntary resignation, respondent
refused to receive the same. On the afternoon, respondent filed a
complaint for illegal dismissal against petitioners before the NLRC.

ISSUE:
Whether respondent was illegally dismissed.

RULING:
No. Under Article 283 of the Labor Code, retrenchment is one of
the authorized causes for termination of employment which the law
accords an employer who is not making good in its operations in order
to cut back on expenses for salaries and wages by laying off some
employees. The purpose of retrenchment is to save a financially ailing
business establishment from eventually collapsing.

The requirements for valid retrenchment which must be proved by


clear and convincing evidence are:
(1) that the retrenchment is reasonably necessary and likely to
prevent business losses which, if already incurred, are not merely de
minimis, but substantial, serious, actual and real, or if only expected,
are reasonably imminent as perceived objectively and in good faith by
the employer;
(2) that the employer served written notice both to the employees
and to the Department of Labor and Employment at least one month
prior to the intended date of retrenchment;

926
Page 927

(3) that the employer pays the retrenched employees separation


pay equivalent to one month pay or at least 1/2 month pay for every
year of service, whichever is higher;
(4) that the employer exercises its prerogative to retrench
employees in good faith for the advancement of its interest and not to
defeat or circumvent the employees' right to security of tenure; and
(5) that the employer used fair and reasonable criteria in
ascertaining who would be dismissed and who would be retained
among the employees, such as status (i.e., whether they are
temporary, casual, regular or managerial employees), efficiency,
seniority, physical fitness, age, and financial hardship for certain
workers

The evaluation and identification of the employees to be retrenched


were jointly undertaken by petitioners Bonoan and Dela Rama, as the
General Manager and HR Manager, respectively, of petitioner BEMI,
based on fair and reasonable criteria, i.e., the employees' positions
and tenures at the company. Respondent was included in the final list
of five employees to be retrenched because she was one of the
employees with the shortest tenures.

Because the five employees to be retrenched opted to voluntarily


resign instead and avail themselves of the financial package offered,
there was no more need for petitioner BEMI to comply with the notice
requirement to the Department of Labor and Employment. Said five
employees were to receive more benefits than what the law prescribed
in case of retrenchment, particularly: (a) full salary for February 2006
although they were no longer required to report to work after
submission of their resignation letters in mid-February 2006; (b) pro-
rated 13th month pay; and (c) financial assistance equivalent to one-
month salary for every year of service.

The foregoing circumstances persuade the Court that no fraud or


deception was employed upon respondent to resign because petitioner
BEMI was indeed about to implement in good faith a retrenchment of
its employees in order to advance its interest and not merely to defeat
or circumvent the respondent's right to security of tenure.

927
Page 928

Case Digest by: BACOLOD, JAMIELOU C.

LA CONSOLACION COLLEGE v. PASCUA


G.R. No. 214744. April 19, 2016.
Leonen, J.

DOCTRINE:
ARTICLE 298 RETRENCHMENT.
When termination of employment is occasioned by retrenchment to
prevent losses, an employer must declare a reasonable cause or
criterion for retrenching an employee. Retrenchment that disregards
an employee's record and length of service is an illegal termination of
employment.

FACTS:
On January 10, 2000, Pascua's services as school physician
were engaged by La Consolacion. She started working part-time
before serving full-time from 2008. On September 29, 2011, Pascua
was handed an Inter-Office Memo from Manalili, La Consolacion's
Human Resources Division Director, inviting her to a meeting
concerning her "working condition. In that meeting, Pascua was
handed a termination of employment letter, explaining the reasons for
and the terms of her dismissal, including payment of separation pay.

Pascua wrote to Sr. Mora, pointing out that the part-time school
physician, Dr. Venus Dimagmaliw (Dr. Dimagmaliw), should have
been considered for dismissal first. She also noted that rather than
dismissing her outright, La Consolacion could have asked her to revert
to part-time status instead. Sr. Mora replied to Pascua's letter. She
indicated the futility of her response considering that Pascua had opted
to file a complaint in the interim. She nevertheless answered Pascua's
queries "as a matter of courtesy." She explained that Pascua in
particular was retrenched because her position, the highest paid in the
health services division, was dispensable.

ISSUE:
Whether or not respondent Virginia Pascua's retrenchment was
valid.

RULING:
No. The Labor Code recognizes retrenchment as an authorized
cause for terminating employment. It is an option validly available to
an employer to address "losses in the operation of the enterprise, lack
of work, or considerable reduction on the volume of business.

928
Page 929

As to the substantive requisites, an employer must first show that


the retrenchment is reasonably necessary and likely to prevent
business losses which, if already incurred, are not merely de minimis,
but substantial, serious, actual and real, or if only expected, are
reasonably imminent as perceived objectively and in good faith by the
employer. Second, an employer must also show that it exercises its
prerogative to retrench employees in good faith for the advancement
of its interest and not to defeat or circumvent the employees' right to
security of tenure. Third, an employer must demonstrate that it used
fair and reasonable criteria in ascertaining who would be dismissed
and who would be retained among the employees, such as status (i.e.,
whether they are temporary, casual, regular or managerial
employees), efficiency, seniority, physical fitness, age, and financial
hardship for certain workers.

La Consolacion's failure was non-compliance with the third


substantive requisite of using fair and reasonable criteria that
considered the status and seniority of the retrenched employee.

There is no dispute here about respondent's seniority and preferred


status. Petitioners acknowledge that she had been employed by La
Consolacion since January 2000, initially as a part-time physician then
serving full-time beginning 2008. It is also not disputed that while
respondent was a full-time physician, La Consolacion had another
physician, Dr. Dimagmaliw, who served part-time. Precisely,
respondent's preeminence is a necessary implication of the very
criteria used by La Consolacion in retrenching her, i.e., that she was
the highest paid employee in health services division. La Consolacion's
disregard of respondent's seniority and preferred status relative to a
part-time employee indicates its resort to an unfair and unreasonable
criterion for retrenchment.

929
Page 930

Case Digest by: BACOLOD, JAMIELOU C.

SANGWOO PHILIPPINES v. SANGWOO PHILIPPINES


EMPLOYEES UNION
G.R. No. 173154.December 9, 2013.
Perlas-Bernabe, J.

DOCTRINE:
ARTICLE 298 CLOSURE OF BUSINESS
Article 297 of the Labor Code provides that before any employee
is terminated due to closure of business, it must give a one (1) month
prior written notice to the employee and to the DOLE. In this relation,
case law instructs that it is the personal right of the employee to be
personally informed of his proposed dismissal as well as the reasons
therefor; and such requirement of notice is not a mere technicality or
formality which the employer may dispense with. Since the purpose of
previous notice is to, among others, give the employee some time to
prepare for the eventual loss of his job, the employer has the positive
duty to inform each and every employee of their impending termination
of employment. To this end, jurisprudence states that an employer’s
act of posting notices to this effect in conspicuous areas in the
workplace is not enough. Verily, for something as significant as the
involuntary loss of one’s employment, nothing less than an
individually-addressed notice of dismissal supplied to each worker is
proper.

FACTS:
During the collective bargaining agreement (CBA) negotiations
between Sangwoo Philippines, Inc. Employees Union- Olalia (SPEU)
and Sangwoo Philippines, Inc. (SPI), the latter filed with the
Department of Labor and Employment (DOLE) a letter-notice of
temporary suspension of operations for one (1) month due to lack of
orders from its buyers. Negotiations on the CBA, however, continued
and on September 10, 2003, the parties signed a handwritten
Memorandum of Agreement.

On September 15, 2003, SPI temporarily ceased operations.


Thereafter, it successively filed two (2) letters with the DOLE for the
extension of the temporary shutdown until March 15, 2004.

Meanwhile, on October 28, 2003, SPEU filed a complaint for unfair


labor practice, illegal closure, illegal dismissal, damages and
attorney’s fees before the Regional Arbitration Branch IV of the NLRC.
Subsequently, SPI posted, in conspicuous places within the company

930
Page 931

premises, notices of its permanent closure and cessation of business


operations, effective March 16, 2004, due to serious economic losses
and financial reverses. Forthwith, SPI offered separation benefits of
one-half (1/2) month pay for every year of service to each of its
employees. 234 employees of SPI accepted the offer, received the
said sums and executed quitclaims. Those who refused the offer, i.e.,
the minority employees, were nevertheless given until March 25, 2004
to accept their checks and correspondingly, execute quitclaims.
However, the minority employees did not claim the said checks.

The LA found that SPI was indeed suffering from serious business
losses as evidenced by financial statements which were never
contested by SPEU –and, as such, validly discontinued its operations.
LA ruled that since SPI’s closure of business was due to serious
business losses, it was not mandated by law to grant separation
benefits to the minority employees.

ISSUE:
Whether or not petitioner, SPI, complied with the notice
requirement of Article 297 of the Labor Code.

RULING:
No. Closure of business is the reversal of fortune of the employer
whereby there is a complete cessation of business operations and/or
an actual locking-up of the doors of establishment, usually due to
financial losses. Closure of business, as an authorized cause for
termination of employment, aims to prevent further financial drain upon
an employer who cannot pay anymore his employees since business
has already stopped. In such a case, the employer is generally
required to give separation benefits to its employees, unless the
closure is due to serious business losses. In this case, the LA, NLRC,
and the CA all consistently found that SPI indeed suffered from serious
business losses which resulted in its permanent shutdown and
accordingly, held the company’s closure to be valid, the Court thus
holds that SPI is not obliged to give separation benefits to the minority
employees pursuant to Article 297 of the Labor Code.

Article 297 of the Labor Code provides that before any employee
is terminated due to closure of business, it must give a one (1) month
prior written notice to the employee and to the DOLE. In this relation,
case law instructs that it is the personal right of the employee to be
personally informed of his proposed dismissal as well as the reasons
therefor; and such requirement of notice is not a mere technicality or
formality which the employer may dispense with. Since the purpose of
previous notice is to, among others, give the employee some time to
prepare for the eventual loss of his job, the employer has the positive

931
Page 932

duty to inform each and every employee of their impending termination


of employment. To this end, jurisprudence states that an employer’s
act of posting notices to this effect in conspicuous areas in the
workplace is not enough. Verily, for something as significant as the
involuntary loss of one’s employment, nothing less than an
individually-addressed notice of dismissal supplied to each worker is
proper.

SPI was required to serve written notices of termination to its


employees, which it, however, failed to do. It is well to stress that while
SPI had a valid ground to terminate its employees, i.e., closure of
business, its failure to comply with the proper procedure for termination
renders it liable to pay the employee nominal damages for such
omission.

In this case, considering that SPI closed down its operations due
to serious business losses and that said closure appears to have been
done in good faith, the Court – similar to the case of Industrial Timber
– deems it just to reduce the amount of nominal damages to be
awarded to each of the minority employees from P50,000.00 to
P10,000.00.

932
Page 933

Case Digest by: BACOLOD, JAMIELOU C.

NAVOTAS SHIPYARD v. MONTALLANA


G.R. No. 190053. March 24, 2014.
Brion, J.

DOCTRINE:
TEMPORARY CLOSURE RIPENING TO CLOSURE DUE TO
SERIOUS BUSINESS LOSSES.
The petitioners undertook a temporary shutdown. In fact, the
company notified the DOLE of the shutdown and filed an
Establishment Termination Report containing the names of the
affected employees. The petitioners expected the company to recover
before the end of the six-month shutdown period, but unfortunately, no
recovery took place. Thus, the shutdown became permanent. Under
the circumstances, we cannot say that the company’s employees were
illegally dismissed; rather, they lost their employment because the
company ceased operations after failing to recover from their financial
reverses.

FACTS:
The case arose when respondents Innocencio Montallana, Alfredo
Bautista, Teodoro Judloman, Guillermo Bongas, Rogelio Bongas,
Diosdado Busante, Emiliano Badu and Rosendo Subing-Subing filed
a complaint for illegal (constructive) dismissal, with money claims,
against the petitioners, Navotas Shipyard Corporation (company) and
its President/General Manager, Jesus Villaflor.

The company’s employees (about 100) were called to a meeting


where Villaflor told them: “Magsasara na ako ng negosyo, babayaran
ko na lang kayo ng separation pay dahil wala na akong pangsweldo sa
inyo. Marami akong mga utang sa krudo, yelo, at iba pa.” Since then,
they were not allowed to report for work but Villaflor’s promise to give
them separation pay never materialized despite their persistent
demands and follow-ups.

The petitioners, on the other hand, claimed that due to the seasonal
lack of fish caught and uncollected receivables, the company suffered
financial reverses. It reported the temporary shutdown to the
Department of Labor and Employment, National Capital Region
(DOLE-NCR) and filed an Establishment Termination Report.

ISSUE:
Whether or not respondents were illegally dismissed.

933
Page 934

RULING:
No. The petitioners undertook a temporary shutdown. In fact, the
company notified the DOLE of the shutdown and filed an
Establishment Termination Report containing the names of the
affected employees. The petitioners expected the company to recover
before the end of the six-month shutdown period, but unfortunately, no
recovery took place. Thus, the shutdown became permanent.
According to the petitioners, they gave the company’s employees their
separation pay. Under the circumstances, we cannot say that the
company’s employees were illegally dismissed; rather, they lost their
employment because the company ceased operations after failing to
recover from their financial reverses.

The respondents’ verbal account of what happened during the


meeting, particularly the company’s imminent closure, to our mind,
confirmed the company’s dire situation. The temporary shutdown, it
appears, was a last ditch effort on the part of Villaflor to make the
company’s operations viable but, as it turned out, the effort proved
futile. The shutdown became permanent as the CA itself
acknowledged. Considering that the company’s closure was due
to serious financial reverses, it is not legally bound to give the
separated employees separation pay.

In the present case, the evidence on hand substantially shows that


the company closed down due to serious business reverses, an
authorized cause for termination of employment. The failure to notify
the respondents in writing of the closure of the company will not
invalidate the termination of their employment, but the company has to
pay them nominal damages for the violation of their right to procedural
due process.

934
Page 935

Case Digest by: BACOLOD, JAMIELOU C.

VETERANS FEDERATION v. MONTENEJO


G.R. No.184819. November 29, 2017.
Velasco, Jr., J.

DOCTRINE:
ARTICLE 298 CLOSURE OF BUSINESS.
One of the authorized causes for dismissal recognized under the
Labor Code is the bona fide cessation of business or operations by the
employer. Article 298 of the Labor Code explicitly sanctions
terminations due to the employer's cessation of business or
operations, as long as the cessation is bona fide or is not made for the
purpose of circumventing the employees' right to security of tenure.

FACTS:
VFP entered into a management agreement with VMDC. Under the
said agreement, VMDC was to assume exclusive management and
operation of the VFPIA.

In managing and operating the VFPIA, VMDC hired its own


personnel and employees. Among those hired by VMDC were
respondents Eduardo L. Montenejo, Mylene M. Bonifacio, Evangeline
E. Valverde and Deana N. Pagal (hereafter collectively referred to as
"Montenejo, et al.").

On November 1999, the VFP board passed a resolution terminating


the management agreement effective December 31, 1999. VMDC
conceded to the termination and eventually agreed to turn over to VFP
the possession of all buildings, equipment and other properties
necessary to the operation of the VFPIA. Subsequently, the President
of VMDC issued a memorandum informing the company's employees
of the termination of their services effective at the close of office hours
on January 31, 2000 "in view of the termination of the management
agreement."

Contending in the main that their dismissals had been effected


without cause and observance of due process, Montenejo, et al. filed
before the LA a complaint for illegal dismissal, money claims and
damages.

VMDC, for its part, denied the contention. It argued that the
dismissals of Montenejo, et al. were valid as they were due to an

935
Page 936

authorized cause - the cessation or closure of its business. VMDC


claimed that the cessation of its operations was but the necessary
consequence of the termination of such agreement.

ISSUE:
Whether or not Montenejo, et al. were illegally dismissed.

RULING:
No. One of the authorized causes for dismissal recognized under
the Labor Code is the bona fide cessation of business or operations by
the employer. Article 298 of the Labor Code explicitly sanctions
terminations due to the employer's cessation of business or
operations, as long as the cessation is bona fide or is not made for the
purpose of circumventing the employees' right to security of tenure.

To unmask the true intent of an employer when effecting a closure


of business, it is important to consider not only the measures adopted
by the employer prior to the purported closure but also the actions
taken by the latter after the fact. The employer's subsequent acts
of suddenly reviving a business it had just closed or surreptititiously
continuing with its operation after announcing a shutdown are telltale
badges that the employer had no real intent to cease its business or
operations and only seeks an excuse to terminate employees
capriciously.

The mere fact that VMDC could have chosen to continue operating
despite the termination of its management agreement with VFP is also
of no consequence. The decision of VMDC to cease its operations after
the termination of the management agreement is, under the law, a
lawful exercise by the company's leadership of its management
prerogative that must perforce be upheld where, as in this case, there
is an absence of showing that the cessation was made for prohibited
purposes.

The validity of the closure of VMDC necessarily validates the


dismissals of Montenejo, et al. that resulted therefrom. The dismissals
cannot be regarded as illegal because they were predicated upon an
authorized cause recognized by law.

Verily, the failure of VMDC to file a notice of closure with the DOLE
does not render the dismissals of Montenejo, et al., which were based
on an authorized cause, illegal. Following Agabon and Jaka, such
failure only entitles Montenejo, et al. to recover nominal damages from
VMDC in the amount of P50,000 each, on top of the separation pay
they already received.

936
Page 937

Case Digest by: BACOLOD, JAMIELOU C.

PNCC SKYWAY CORP. v. SECRETARY OF LABOR


G.R. No. 196110. February 6, 2017.
Peralta, J.

DOCTRINE:
ARTICLE 298 CLOSURE OF BUSINESS
Procedurally, Article 298 (formerly, Article 283) of the Labor Code,
as amended provides for three (3) requirements to properly effectuate
termination on the ground of closure or cessation of business
operations. These are: (a) service of a written notice to the employees
and to the DOLE at least one (1) month before the intended date of
termination; (b) the cessation of business must be bona fide in
character; and (c) payment to the employees of termination pay
amounting to one (1) month pay or at least one-half month pay for
every year of service, whichever is higher. Case law has settled that
an employer who terminates an employee for a valid cause but does
so through invalid procedure is liable to pay the latter nominal
damages.

FACTS:
The Philippine National Construction Corporation (PNCC) was
awarded by the Toll Regulatory Board (TRB) with the franchise of
constructing, operating and maintaining the north and south
expressways, including the South Metro Manila Skyway (Skyway). On
December 15, 1998, it created petitioner PNCC Skyway Corporation
(PSC) for the purpose of taking charge of its traffic safety, maintaining
its facilities and collecting toll.

Citra Metro Manila Tollway Corporation (Citra), a private investor


under a build-and-transfer scheme, entered into an agreement with the
TRB and the PNCC to transfer the operation of the Skyway from
petitioner PSC to the Skyway O & M Corporation (SOMCO). Three (3)
days before the full transfer of the operation of the Skyway to SOMCO,
petitioner PSC served termination letters to its employees, many of
whom were members of private respondent PNCC Skyway Traffic
Management and Security Division Worker's Organization (Union).
According to the letter, PSC has no choice but to close its operations
resulting in the termination of its employees effective January 31, 2008.

Private respondent Union, immediately upon receipt of the


termination letters, filed a Notice of Strike before the DOLE alleging
that the closure of the operation of PSC is tantamount to union-busting
because it is a means of terminating employees who are members

937
Page 938

thereof. Furthermore, the notices of termination were served on its


employees three (3) days before petitioner PSC ceases its operations,
thereby violating the employees' right to due process. Private
respondent Union, thus, prayed that petitioner PSC be held guilty of
unfair labor practice and illegal dismissal.

ISSUE:
Whether or not PSC violated Art. 283 (now Art. 298) of the Labor
Code with regard to procedural requirements stated therein.

RULING:
Yes. Procedurally, Article 298 (formerly, Article 283) of the Labor
Code, as amended provides for three (3) requirements to properly
effectuate termination on the ground of closure or cessation of
business operations. These are: (a) service of a written notice to the
employees and to the DOLE at least one (1) month before the intended
date of termination; (b) the cessation of business must be bona fide in
character; and (c) payment to the employees of termination pay
amounting to one (1) month pay or at least one-half month pay for
every year of service, whichever is higher. Case law has settled that
an employer who terminates an employee for a valid cause but does
so through invalid procedure is liable to pay the latter nominal
damages.

In the instant case, while both the SOLE and the appellate court
found the closure of PSC's business operation to be bona fide, the
required notices were, however, served on the employees and the
DOLE only three (3) days before the closure of the company. The
required written notice under Article 283 of the Labor Code is to inform
the employees of the specific date of termination or closure of business
operations, and must be served upon them at least one (1) month
before the date of effectivity to give them sufficient time to make the
necessary arrangements. The purpose of this requirement is to give
employees time to prepare for the eventual loss of their jobs, as well
as to give DOLE the opportunity to ascertain the veracity of the alleged
cause of termination. Thus, considering that the notices of termination
were given merely three (3) days before the cessation of the PSC's
operation, it defeats the very purpose of the required notice and the
mandate of Article 283 of the Labor Code. Neither the payment of
employees' salaries for the said one- month period nor the employees'
alleged actual knowledge of the ASTOA is sufficient to replace the
formal and written notice required by the law.

938
Page 939

Case Digest by: BACOLOD, JAMIELOU C.

PENAFRANCIA TOURS AND TRAVEL TRANSPORT v.


SARMIENTO
G.R. No. 178397. October 20, 2010.
Nachura, J.

DOCTRINE:
ARTICLE 298 CLOSURE OF BUSINESS.
Closure of business is the reversal of fortune of the employer
whereby there is a complete cessation of business operations and/or
an actual locking-up of the doors of the establishment, usually due to
financial losses. Closure of business, as an authorized cause for
termination of employment, aims to prevent further financial drain upon
an employer who can no longer pay his employees since business has
already stopped.

FACTS:
Respondents Joselito Sarmiento and Ricardo Catimbang were
employed by Petitioner Penafrancia Tours as bus inspectors. On
October 30th, 2002, both Sarmiento and Catimbang were dismissed
from work on the ground of irreversible business losses. Prior to their
dismissal, they were introduced to one Alfredo Perez, the owner of
ALPS Transportation, who allegedly bought Penafrancia and was now
its new owner. Sarmiento and Catimbang filed complaints for illegal
dismissal, underpayment of wages, and unfair labor practice.

Traversing the complaint, petitioner admitted that respondents were


among its bus inspectors. It asseverated, however, that due to severe
business losses, petitioner made the painful decision to stop its
operation and sell the business enterprise to the Perez family of ALPS
Transportation. It alleged that due notice was given to the Department
of Labor and Employment, and that all its employees were duly notified
and were paid their 13th corresponding separation pay, as well as their
month pay. The new owners maintained the business name of
petitioner, and the management of petitioner was entrusted to the new
owners in October 2002, with Edilberto Perez as Vice-President for
Finance and Operations. Subsequently, several memoranda were
issued by Edilberto Perez in behalf of petitioner. Petitioner argued that
the matter of rehiring respondents rested on the sound discretion of its
new owners, and the latter could not be compelled to absorb
petitioner’s former employees since the same was not part of the deal.
Petitioner alleged that respondents submitted their application for
reemployment but, after evaluation, the new owners opted not to hire
respondents.

939
Page 940

During the pendency of the illegal dismissal case, the new owner
(Perez) issued a notice that the management of the company shall
revert back to its former owners because of a rescission of sale.

The NLRC and the CA both found that petitioner failed to establish
its allegation that it was suffering from business losses, and that
petitioner did not actually sell its business to the Perez family. As such,
both ruled in favour of the respondent.

ISSUE:
Whether or not respondents were legally terminated from
employment by reason of the sale of the business enterprise and the
consequent change or transfer of ownership/management.

RULING:
No. The Supreme Court held that the respondents were illegally
dismissed. Closure of business is the reversal of fortune of the
employer whereby there is a complete cessation of business
operations and/or an actual locking-up of the doors of the
establishment, usually due to financial losses. Closure of business, as
an authorized cause for termination of employment, aims to prevent
further financial drain upon an employer who can no longer pay his
employees since business has already stopped.

On this ground, petitioner terminated the employment of


respondents. However, what petitioner apparently made was a transfer
of ownership. It is true that, as invoked by petitioner, in Manlimos, et
al. v. NLRC, et al., we held that a change of ownership in a business
concern is not proscribed by law. Lest petitioner forget, however, we
also held therein that the sale or disposition must be motivated by good
faith as a condition for exemption from liability. Thus, where the charge
of ownership is done in bad faith, or is used to defeat the rights of labor,
the successor-employer is deemed to have absorbed the employees
and is held liable for the transgressions of his or her predecessor.

But, in this case, there is no successor-employer because there was


no actual change of ownership. We sustain the uniform factual finding
of both the NLRC and the CA that no actual sale transpired and, as
such, there is no closure or cessation of business that can serve as an
authorized cause for the dismissal of respondents.

940
Page 941

Case Digest by: BACOLOD, JAMIELOU C.

SY ET AL. v. COURT OF APPEALS


G.R. No.142293. February 27, 2003.
Quisumbing, J.

DOCTRINE:
ARTICLE 299
Since the burden of proving the validity of the dismissal of the
employee rests on the employer, the latter should likewise bear the
burden of showing that the requisites for a valid dismissal due to a
disease have been complied with. In the absence of the required
certification by a competent public health authority, this Court has ruled
against the validity of the employees dismissal.

FACTS:
Sometime in 1958, private respondent Jaime Sahot started
working as a truck helper for petitioners family-owned trucking
business named Vicente Sy Trucking. In 1965, he became a truck
driver of the same family business, renamed T. Paulino Trucking
Service, later 6Bs Trucking Corporation in 1985, and thereafter known
as SBT Trucking Corporation since 1994. Throughout all these
changes in names and for 36 years, private respondent continuously
served the trucking business of petitioners.

In April 1994, Sahot was already 59 years old. He had been


incurring absences as he was suffering from various ailments.
Particularly causing him pain was his left thigh, which greatly affected
the performance of his task as a driver. He inquired about his medical
and retirement benefits with the Social Security System (SSS) on April
25, 1994, but discovered that his premium payments had not been
remitted by his employer.

At the end of his week-long absence, Sahot applied for extension


of his leave for the whole month of June, 1994. It was at this time when
petitioners allegedly threatened to terminate his employment should he
refuse to go back to work.

At this point, Sahot found himself in a dilemma. He was facing


dismissal if he refused to work, But he could not retire on pension
because petitioners never paid his correct SSS premiums. The fact
remained he could no longer work as his left thigh hurt abominably.
Petitioners ended his dilemma. They carried out their threat and
dismissed him from work, effective June 30, 1994. He ended up sick,

941
Page 942

jobless and penniless. Sahot filed with the NLRC NCR Arbitration
Branch, a complaint for illegal dismissal.

ISSUE:
Whether or not respondent was illegally dismissed.

RULING:
Yes. Article 284 of the Labor Code authorizes an employer to
terminate an employee on the ground of disease. However, in order to
validly terminate employment on this ground, Book VI, Rule I, Section
8 of the Omnibus Implementing Rules of the Labor Code requires:

Sec. 8. Disease as a ground for dismissal- Where the employee


suffers from a disease and his continued employment is prohibited by
law or prejudicial to his health or to the health of his co-employees, the
employer shall not terminate his employment unless there is a
certification by competent public health authority that the disease is of
such nature or at such a stage that it cannot be cured within a period
of six (6) months even with proper medical treatment. If the disease or
ailment can be cured within the period, the employer shall not
terminate the employee but shall ask the employee to take a leave.
The employer shall reinstate such employee to his former position
immediately upon the restoration of his normal health.

As this Court stated in Triple Eight integrated Services, Inc. vs.


NLRC, the requirement for a medical certificate under Article 284 of
the Labor Code cannot be dispensed with; otherwise, it would sanction
the unilateral and arbitrary determination by the employer of the gravity
or extent of the employees illness and thus defeat the public policy in
the protection of labor.

In the case at bar, the employer clearly did not comply with the
medical certificate requirement before Sahots dismissal was effected.
In the same case of Sevillana vs. I.T. (International) Corp., we ruled:
Since the burden of proving the validity of the dismissal of the
employee rests on the employer, the latter should likewise bear the
burden of showing that the requisites for a valid dismissal due to a
disease have been complied with. In the absence of the required
certification by a competent public health authority, this Court has ruled
against the validity of the employees dismissal. It is therefore
incumbent upon the private respondents to prove by the quantum of
evidence required by law that petitioner was not dismissed, or if
dismissed, that the dismissal was not illegal; otherwise, the dismissal
would be unjustified. This Court will not sanction a dismissal premised
on mere conjectures and suspicions, the evidence must be substantial

942
Page 943

and not arbitrary and must be founded on clearly established facts


sufficient to warrant his separation from work.

943
Page 944

Case Digest by: BACOLOD, JAMIELOU C.

UNION MOTORS CORP. v. NLRC


G.R. No. 159738.December 9, 2004.
Callejo, Sr., J

DOCTRINE:
ARTICLE 299
While it is true that the petitioner had objected to the veracity of
the medical certificates because of lack of notarization, it has been said
that verification of documents is not necessary in order that the said
documents could be considered as substantial evidence. The medical
certificates were properly signed by the physicians; hence, they bear
all the earmarks of regularity in their issuance and are entitled to full
probative weight.

FACTS:
On October 23, 1993, the respondent, Alejandro Etis, was hired
by the petitioner as an automotive mechanic at the service department
in the latter’s Paco Branch. In 1994, he was transferred to the Caloocan
City Branch, where his latest monthly salary was P6,330.00. During his
employment, he was awarded the “Top Technician” for the month of
May in 1995 and Technician of the Year (1995). He also became a
member of the Exclusive P40,000.00 Club and received the Model
Employee Award in the same year.

On September 22, 1997, the respondent made a phone call to


Rosita dela Cruz, the company nurse, and informed her that he had to
take a sick leave as he had a painful and unbearable toothache. The
next day, he again phoned Dela Cruz and told her that he could not
report for work because he still had to consult a doctor. Finding that
the respondent’s ailment was due to a tooth inflammation, the doctor
referred him to a dentist for further management. Dr. Rodolfo Pamor,
a dentist, then scheduled the respondent’s tooth extraction on
September 27, 1997, hoping that, by that time, the inflammation would
have subsided.

On October 2, 1997, the petitioner issued an Inter Office


Memorandum through Angelo B. Nicolas, the manager of its Human
Resources Department, terminating the services of the respondent for
having incurred more than five (5) consecutive absences without
proper notification. The petitioner considered the consecutive
absences of the respondent as abandonment of office under Section
6.1.1, Article III of the Company Rules.

944
Page 945

On October 4, 1997, Dr. Pamor successfully extracted the


respondent’s tooth. As soon as he had recovered, the respondent
reported for work, but was denied entry into the company’s premises.
He was also informed that his employment had already been
terminated. Left with no other recourse, the respondent filed, on May
18, 1999, a complaint for illegal dismissal before the arbitration branch
of the NLRC against the petitioner and/or Benito Cua.

ISSUE:
Whether or not respondent was validly dismissed.

RULING:
No. The termination by respondent-appellee of complainant’s
service despite knowledge of complainant’s ailment, as shown by the
telephone calls made by the latter to the company nurse and the actual
confirmation made by respondent’s company guard, who personally
visited complainant’s residence, clearly establishes the illegality of
complainant’s dismissal.

The company rules do not require that the notice of an employee’s


absence and the reasons therefor be in writing and for such notice to
be given to any specific office and/or employee of the petitioner.
Hence, the notice may be verbal; it is enough then that an officer or
employee of the petitioner, competent and responsible enough to
receive such notice for and in behalf of the petitioner, was informed of
such absence and the corresponding reason.

While it is true that the petitioner had objected to the veracity of


the medical certificates because of lack of notarization, it has been said
that verification of documents is not necessary in order that the said
documents could be considered as substantial evidence. The medical
certificates were properly signed by the physicians; hence, they bear
all the earmarks of regularity in their issuance and are entitled to full
probative weight.

945
Page 946

Case Digest by: BACOLOD, JAMIELOU C.

VILLARUEL v. YEO HAN GUAN


G.R. No. 169191.June 01, 2011.
Peralta, J.

DOCTRINE:
ARTICLE 299
There is no provision in the Labor Code which grants separation
pay to voluntarily resigning employees. In fact, the rule is that an
employee who voluntarily resigns from employment is not entitled to
separation pay, except when it is stipulated in the employment contract
or CBA, or it is sanctioned by established employer practice or policy.

FACTS:
This case is for payment of separation pay filed with the NLRC by
Romeo Villaruel against Yuhans Enterprises. Petitioner, Romeo
Villaruel, alleged that in June 1963, he was employed as a machine
operator by Ribonette Manufacturing Company, an enterprise
engaged in the business of manufacturing and selling PVC pipes and
is owned and managed by herein respondent Yeo Han Guan. Over a
period of almost twenty (20) years, the company changed its name four
times. Starting in 1993 up to the time of the filing of petitioner's
complaint in 1999, the company was operating under the name of
Yuhans Enterprises. Despite the changes in the company's name,
petitioner remained in the employ of respondent.

On Oct. 5, 1998 petitioner got sick and was confined in a hospital,


when he reported for work on Dec. 12, 1998, he alleged that
respondent no longer permitted him to go back because of his illness.
He asked respondent to allow him to continue working but to assign
him a lighter kind of work which was denied. He was offered a
separation pay of P15,000 corresponding to the period of 1993 to
1999. Petitioner prays that he be granted separation pay computed
from his first of employment in June 1963, but respondent refused.

On the other hand respondent averred in his position paper that


petitioner never showed up after he recovered from his illness.
Respondent was later caught by surprise when petitioner filed the
instant case for recovery of separation pay. Respondent claims that he
never terminated petitioner’s employment and that during their
mandatory conference he even told petitioner that he could go back to
work at anytime but petitioner clearly manifested that he was no longer
interested in returning to work.

946
Page 947

ISSUE:
Whether or not petitioner is entitled to his separation pay.

RULING:
No. The Court agrees with the CA in its observation of the
following circumstances as proof that respondent did not terminate
petitioner's employment: first, the only cause of action in petitioner's
original complaint is that he was “offered a very low separation pay”;
second, there was no allegation of illegal dismissal, both in petitioner's
original and amended complaints and position paper; and, third, there
was no prayer for reinstatement.

In consonance with the above findings, the Court finds that


petitioner was the one who initiated the severance of his employment
relations with respondent. It is evident from the various pleadings filed
by petitioner that he never intended to return to his employment with
respondent on the ground that his health is failing. Indeed, petitioner
did not ask for reinstatement. In fact, he rejected respondent's offer for
him to return to work. This is tantamount to resignation.

There is no provision in the Labor Code which grants separation


pay to voluntarily resigning employees. In fact, the rule is that an
employee who voluntarily resigns from employment is not entitled to
separation pay, except when it is stipulated in the employment contract
or CBA, or it is sanctioned by established employer practice or policy.
In the present case, neither the abovementioned provisions of the
Labor Code and its implementing rules and regulations nor the
exceptions apply because petitioner was not dismissed from his
employment and there is no evidence to show that payment of
separation pay is stipulated in his employment contract or sanctioned
by established practice or policy of herein respondent, his employer.

Since petitioner was not terminated from his employment and,


instead, is deemed to have resigned therefrom, he is not entitled to
separation pay under the provisions of the Labor Code.

947
Page 948

Case Digest by: BRODIT, VYEL MARIE C.

MALIG-ON v. EQUITABLE GENERAL SERVICES, INC.


G.R. No. 185269. June 29, 2010.
Abad, J.

DOCTRINE:
ARTICLE 300. TERMINATION BY AN EMPLOYEE
The rule in termination cases is that the employer bears the
burden of proving that he dismissed his employee for a just cause.
And, when the employer claims that the employee resigned from work,
the burden is on the employer to prove that he did so willingly. Whether
that is the case would largely depend on the circumstances
surrounding such alleged resignation. Those circumstances must be
consistent with the employee’s intent to give up work.

FACTS:
Petitioner, Elsa Malig-on was hired by respondent, Equitable
General Services, Inc. as a janitress on March 1996. After six years of
service, Malig-on’s immediate supervisor told her that the company
would be assigning her to another client. But it never did despite
several follow-ups that she made. Eight months later the company told
Malig-on that she had to file a resignation letter before it would reassign
her. She complied but the company reneged on its undertaking,
prompting Malig-on to file a complaint against it for illegal dismissal.

Respondents denied Malig-on’s allegations and claimed that she


just stopped reporting for work on February 2002 without giving any
reason. Consequently, the company wrote her two letters, first on
August 23, 2002 and again on September 2, 2002, asking her to
explain her continued absence. On October 15, 2002 Malig-on showed
up at the company’s office and submitted her resignation letter.

ISSUE:
Did respondent company constructively dismiss Malig-on?

RULING:
Yes, respondent Equitable General Services, Inc. constructively
dismissed Petitioner Malig-on.

The rule in termination cases is that the employer bears the


burden of proving that he dismissed his employee for a just cause.
And, when the employer claims that the employee resigned from work,
the burden is on the employer to prove that he did so willingly. Whether

948
Page 949

that is the case would largely depend on the circumstances


surrounding such alleged resignation. Those circumstances must be
consistent with the employee’s intent to give up work.

The Supreme Court ruled that the company evidently placed


Malig-on on floating status after being relieved as janitress in a client’s
workplace. But, as the Court has repeatedly ruled, such act of "off-
detailing" Malig-on was not the equivalent of dismissal so long as her
floating status did not continue beyond a reasonable time. But, when it
ran up to more than six months, the company may be considered to
have constructively dismissed her from work, that is, as of August 16,
2002. Thus, her purported resignation on October 15, 2002 could not
have been legally possible.

The company of course claims that it gave Malig-on notices on


August 23, 2002 and September 2, 2002, asking her to explain her
failure to report for work and informing her that the company would
treat such failure as lack of interest in it, respectively. But these notices
cannot possibly take the place of the notices required by law. They
came more than six months after the company placed her on floating
status and, consequently, the company gave her those notices after it
had constructively dismissed her from work.

949
Page 950

Case Digest by: BRODIT, VYEL MARIE C.

CHIANG KAI SHEK COLLEGE v. TORRES


G.R. No. 189456. April 2, 2014.
Perez, J.

DOCTRINE:
ARTICLE 300. TERMINATION BY AN EMPLOYEE
There is constructive dismissal when there is cessation of work,
because continued employment is rendered impossible, unreasonable
or unlikely, as an offer involving a demotion in rank or a diminution in
pay and other benefits.

FACTS:
Respondent, Rosalinda Torres, had been employed as a grade
school teacher of the school from July 1970 until 31 May 2003. She
was accused of leaking a copy of a special quiz given to Grade 5
students of HEKASI. An administrative hearing was then conducted on
wherein respondent and Mrs. Anduyan were asked questions by the
Investigating Committee relative to the leakage of test paper.

The Investigating Committee held a meeting and found


respondent and Mrs. Anduyan guilty of committing a grave offense of
the school policies by leaking a special quiz. The Committee decided
to impose the penalty of one-month suspension without pay on
respondent and forfeiture of all the benefits scheduled to be given on
Teacher’s Day. According to petitioners, their Investigating Committee
had actually decided to terminate respondent and had in fact prepared
a memorandum of termination, but respondent allegedly pleaded for a
change of punishment in a short letter.

Petitioners, through counsel, wrote to respondent’s counsel


asserting that respondent was being terminated but the latter
requested that "she be suspended instead on condition that she will
tender her voluntary resignation at the end of the school year."

On 10 June 2003, respondent filed a complaint for constructive


dismissal and illegal suspension with the Labor Arbiter.

ISSUE:
Does the school’s act of imposing the penalty of suspension
instead of immediate dismissal from service at the behest of the erring
employee, in exchange for the employee’s resignation at the end of the
school year, constitute constructive dismissal?

950
Page 951

RULING:
No, respondent and Mrs. Anduyan were not constructively
dismissed.

Given the indications of voluntary resignation, the Supreme Court


ruled that that there is no constructive dismissal in this case. There is
constructive dismissal when there is cessation of work, because
continued employment is rendered impossible, unreasonable or
unlikely, as an offer involving a demotion in rank or a diminution in pay
and other benefits. Aptly called a dismissal in disguise or an act
amounting to dismissal but made to appear as if it were not,
constructive dismissal may, likewise, exist if an act of clear
discrimination, insensibility, or disdain by an employer becomes so
unbearable on the part of the employee that it could foreclose any
choice by him except to forego his continued employment. There was
here no discrimination committed by petitioners. While respondent did
not tender her resignation wholeheartedly, circumstances of her own
making did not give her any other option. With due process, she was
found to have committed the grave offense of leaking test questions.
Dismissal from employment was the justified equivalent penalty.
Having realized that, she asked for, and was granted, not just a
deferred imposition of, but also an acceptable cover for the penalty.

Respondent’s profession, the gravity of her infraction, and the fact


that she waited until the close of the school year to challenge her
impending resignation demonstrate that respondent had bargained for
a graceful exit and is now trying to renege on her obligation.

951
Page 952

Case Digest by: BRODIT, VYEL MARIE C.

OPINALDO v. RAVINA
G.R. No. 196573. October 16, 2013.
Villarama, Jr., J.

DOCTRINE:
ARTICLE 300. TERMINATION BY AN EMPLOYEE
It is a time-honored legal principle that the employer has the onus
probandi to show that the dismissal or termination was for a just and
authorized cause under the Labor Code.

FACTS:
Respondent Narcisa Ravina is the general manager and sole
proprietor of St. Louisse Security Agency (the Agency). Petitioner
Victorino Opinaldo is a security guard who had worked for the Agency
until his alleged illegal dismissal by respondent on December 22, 2006.
Opinaldo was detailed to PAIJR Furniture Accessories (PAIJR) in
Mandaue City.

In a letter dated August 15, 2006, the owner of PAIJR submitted a


written complaint to respondent stating his request to relieve
respondent as one of the guards of PAIJR for the reason that he is no
longer physically fit to perform his duties and responsibilities as a
company guard because of his health condition.

Acceding to PAIJR’s request, respondent relieved petitioner from


his work. Respondent also required petitioner to submit a medical
certificate to prove that he is physically and mentally fit for work as
security guard. On September 2006, Petitioner was reassigned to
Gomez Construction at Mandaue City. After working for a period of two
weeks for Gomez Construction and upon receipt of his salary for
services rendered within the said two-week period, petitioner ceased
to report for work.

Petitioner then filed a complaint against respondent with the DOLE


for underpayment of salary and nonpayment of other labor standard
benefits. The parties agreed to settle and reached a compromise
agreement.

After almost four weeks from the settlement of the case, petitioner
returned to respondent’s office and asked respondent to sign an SSS
Sickness Notification which he was going to use in order to avail of the
discounted fees for a medical check-up, respondent allegedly refused

952
Page 953

and informed him that he was no longer an employee of the Agency.


Respondent allegedly told him that when he signed the quitclaim and
release form at the DOLE Regional Office, she already considered him
to have quit his employment.

Thereafter, petitioner filed a Complaint for Illegal Dismissal with a


prayer for the payment of separation pay in lieu of reinstatement
against respondent and the Agency before the NLRC. The NLRC ruled
that respondent and the Agency liable for illegal dismissal. On appeal,
respondent and the agency that they required Opinaldo to undergo a
medical examination and such is a lawful exercise of management
prerogative on Ravina’s part considering the charges that Opinaldo
was not only suffering from hypertension but was also sleeping while
on duty.

ISSUE:
Was there valid exercise of respondent’s management
prerogative to prevent petitioner’s continued employment with the
Agency unless he presents the required medical certificate?

RULING:
No, respondent did not properly exercise her management
prerogative when she withheld petitioner’s employment without due
process.

The Supreme Court reiterated the basic principle of labor


protection: that a worker cannot be deprived of his job without
satisfying the requirements of due process. Labor is property and the
right to make it available is next in importance to the rights of life and
liberty. As enshrined under the Bill of Rights, no person shall be
deprived of life, liberty or property without due process of law. The due
process requirement in the deprivation of one’s employment is
transcendental that it limits the exercise of the management
prerogative of the employer to control and regulate the affairs of the
business. In the case at bar, all that respondent employer needed to
prove was that petitioner employee was notified that his failure to
submit the required medical certificate will result in his lack of work
assignment – and eventually the termination of his employment – as a
security guard. There is no iota of evidence in the records, save for the
bare allegations of respondent, that petitioner was notified of such
consequence for non-submission.

Respondent failed to prove that she has notified petitioner that her
continuous refusal to provide him any work assignment was due to his
non-submission of the medical certificate. Had respondent exercised
the rules of fair play, petitioner would have had the option of complying

953
Page 954

or not complying with the medical certificate requirement – having full


knowledge of the consequences of his actions. Respondent failed to
do so and she cannot now hide behind the defense that there was no
illegal termination because petitioner cannot show proof that he had
been illegally dismissed. It is a time-honored legal principle that the
employer has the onus probandi to show that the dismissal or
termination was for a just and authorized cause under the Labor Code.
Respondent failed to show that the termination was justified and
authorized, nor was it done as a valid exercise of management
prerogative. Given the circumstances in the case at bar, it is not fair to
shift the burden to petitioner, and rule that he failed to prove his claim,
when respondent had successfully terminated the employer-employee
relationship without leaving a paper trail in a clear case of illegal
dismissal.

954
Page 955

Case Digest by: BRODIT, VYEL MARIE C.

WILLI HAHN ENTERPRISES v. MAGHUYOP


G.R. NO. 160348. December 17, 2004.
Ynares-Santiago, J.

DOCTRINE:
ARTICLE 300. TERMINATION BY AN EMPLOYEE.
The rule that the filing of a complaint for illegal dismissal is
inconsistent with resignation, is not applicable to the instant case. The
filing of an illegal dismissal case by respondent was evidently a mere
afterthought. It was filed not because she wanted to return to work but
to claim separation pay and backwages.

FACTS:
In 1982, Respondent, Lilia Maghuyop was hired by Petitioner Willi
Hahn as nanny of one of his sons. She was then employed as a sales
clerk of Willi Hahn Enterprises in 1986. In 1996, she was promoted as
store manager in their SM Cebu Branch.

In 1998, petitioner conducted an Inventory Report and discovered


that its SM Cebu branch incurred stock shortages and non-remittances
in the total amount of P27,727.39. Petitioner decided to terminate the
services of respondent, however, before he could do so, the latter
tendered her resignation. Believing the good faith of respondent in
resigning, petitioner decided not to file charges against her anymore.

On the other hand, respondent claimed that on July 22, 1998,


while she was in SM Cebu branch, she was approached by Tony Abu
and Cesar Araneta who ordered her to close shop and to write a letter
to Mr. and Mrs. Hahn thanking them for the years she had been in their
employ and for all the benefits she received from them. She refused to
obey the order, but Tony Abu typed the letter of resignation and asked
her to sign the same. Respondent admitted that she read and affixed
her signature on the letter. Thereafter, she was allegedly told to pack
her belongings and to vacate the housing unit provided by the
company for her family. Thereafter, respondent filed a complaint with
the NLRC.

ISSUE:
Did respondent voluntarily resign as manager of the SM Cebu
branch?

RULING:

955
Page 956

Yes, respondent voluntarily resigned as manager of the SM Cebu


branch.

In deciding the case, the Supreme Court cited Callanta v. National


Labor Relations Commission, where a national-promoter salesman of
Distilleria Limtuaco Co. resigned after he was found to have a shortage
of P49,005.49 in a "spot audit" conducted by the company. He later
filed an illegal dismissal case claiming that his consent to the
resignation was vitiated as he signed the company's ready-made
resignation letter because the latter threatened to file an estafa case
against him. It was ruled that a salesman-promoter could not have
been confused, coerced or intimidated into signing the resignation
letter. Furthermore, the Court concluded that he affixed his signature
in the said letter of his own free will with full knowledge of the
consequences thereof.

The Court held that the failure of petitioner to pursue the


termination proceedings against respondent and to make her pay for
the shortage incurred did not cast doubt on the voluntary nature of her
resignation. A decision to give a graceful exit to an employee rather
than to file an action for redress is perfectly within the discretion of an
employer. It is not uncommon that an employee is permitted to resign
to save face after the exposure of her malfeasance. Under the
circumstances, the failure of petitioner to file action against the
respondent should be considered as an act of compassion for one who
used to be a trusted employee and a close member of the household.

Respondent's unsubstantiated and self-serving claim that she was


coerced into signing the resignation letter does not deserve credence.
It is a basic rule in evidence that the burden of proof is on the part of
the party who makes the allegations. Respondent failed to discharge
this burden.

Furthermore, the rule that the filing of a complaint for illegal


dismissal is inconsistent with resignation, is not applicable to the
instant case. The filing of an illegal dismissal case by respondent was
evidently a mere afterthought. It was filed not because she wanted to
return to work but to claim separation pay and backwages.

Settled is the rule that factual findings of labor officials who are
deemed to have acquired expertise in matters within their respective
jurisdiction are generally accorded not only respect, but even finality,
and bind the Supreme Court when supported by substantial evidence.

956
Page 957

Case Digest by: BRODIT, VYEL MARIE C.

SKIPPERS UNITED PACIFIC, INC. v. DOZA ET. AL.,


G.R. No. 175558. February 8, 2012.
Carpio, J.

DOCTRINE:
ARTICLE 300. TERMINATION BY AN EMPLOYEE.
For a worker’s dismissal to be considered valid, it must comply
with both procedural and substantive due process. The legality of the
manner of dismissal constitutes procedural due process, while the
legality of the act of dismissal constitutes substantive due process.

Article 285 of the Labor Code recognizes termination by the


employee of the employment contract by "serving written notice on the
employer at least one (1) month in advance." Given that provision, the
law contemplates the requirement of a written notice of resignation.

FACTS:
This case arose from consolidated labor case filed by seafarers
Napoleon De Gracia (De Gracia), Isidro L. Lata (Lata), Charlie Aprosta
(Aprosta), and Nathaniel Doza (Doza) against local manning agency
Skippers United Pacific, Inc. and its foreign principal, Skippers
Maritime Services, Inc., Ltd. (Skippers) for unremitted home allotment
for the month of December 1998, salaries for the unexpired portion of
their employment contracts, moral damages, exemplary damages, and
attorney’s fees.

Skippers answered with a claim for reimbursement of De Gracia,


Aprosta and Lata’s repatriation expenses, as well as award of moral
damages and attorney’s fees. Skippers further alleged that on January
1999, Aprosta, De Gracia, Lata and Doza, arrived in the master’s cabin
and demanded immediate repatriation because they were not satisfied
with the ship. They threatened that they may become crazy any
moment and demanded for all outstanding payments due to them.
Skippers also admitted non-payment of home allotment for the month
of December 1998, but prayed for the offsetting of such amount with
the repatriation expenses.

Since De Gracia, et al. pre-terminated their contracts, Skippers


claimed they are liable for their repatriation expenses in accordance
with Section 19(G) of Philippine Overseas Employment Administration
(POEA) Memorandum Circular No. 55, series of 1996 which states:

957
Page 958

G. A seaman who requests for early termination of his contract


shall be liable for his repatriation cost as well as the transportation
cost of his replacement. The employer may, in case of
compassionate grounds, assume the transportation cost of the
seafarer’s replacement.

ISSUE:
Whether respondents, De Gracia, Apostra, Lata, and Doza were
validly dismissed from service?

RULING:
No, respondents were not validly dismissed. The Supreme Court
ruled that for a worker’s dismissal to be considered valid, it must
comply with both procedural and substantive due process. The legality
of the manner of dismissal constitutes procedural due process, while
the legality of the act of dismissal constitutes substantive due process.

Procedural due process in dismissal cases consists of the twin


requirements of notice and hearing. The employer must furnish the
employee with two written notices before the termination of
employment can be effected: (1) the first notice apprises the employee
of the particular acts or omissions for which his dismissal is sought;
and (2) the second notice informs the employee of the employer’s
decision to dismiss him. Before the issuance of the second notice, the
requirement of a hearing must be complied with by giving the worker
an opportunity to be heard. It is not necessary that an actual hearing
be conducted.

Substantive due process, on the other hand, requires that


dismissal by the employer be made under a just or authorized cause
under Articles 282 to 284 of the Labor Code.

In this case, there was no written notice furnished to De Gracia, et


al. regarding the cause of their dismissal. Cosmoship furnished a
written notice (telex) to Skippers, the local manning agency, claiming
that De Gracia, et al. were repatriated because the latter voluntarily
pre-terminated their contracts. This telex was given credibility and
weight by the Labor Arbiter and NLRC in deciding that there was pre-
termination of the employment contract "akin to resignation" and no
illegal dismissal. However, as correctly ruled by the CA, the telex
message is "a biased and self-serving document that does not satisfy
the requirement of substantial evidence." If, indeed, De Gracia, et al.
voluntarily pre-terminated their contracts, then De Gracia, et al. should
have submitted their written resignations.

958
Page 959

Article 285 of the Labor Code recognizes termination by the


employee of the employment contract by "serving written notice on the
employer at least one (1) month in advance." Given that provision, the
law contemplates the requirement of a written notice of resignation. In
the absence of a written resignation, it is safe to presume that the
employer terminated the seafarers. In addition, the telex message
relied upon by the Labor Arbiter and NLRC bore conflicting dates of 22
January 1998 and 22 January 1999, giving doubt to the veracity and
authenticity of the document. In 22 January 1998, De Gracia, et al.
were not even employed yet by the foreign principal. For these
reasons, the dismissal of De Gracia, et al. was illegal.

959
Page 960

Case Digest by: BRODIT, VYEL MARIE C.

MORALES v. HARBOUR CENTRE PORT TERMINAL, INC.


G.R. No. 174208. January 25, 2012.
Perez, J.

DOCTRINE:
ARTICLE 300. TERMINATION BY AN EMPLOYEE.

As a just and valid ground for dismissal, at any rate, abandonment


requires the deliberate, unjustified refusal of the employee to resume
his employment, without any intention of returning. Since an employee
like Morales who takes steps to protest his dismissal cannot logically
be said to have abandoned his work, it is a settled doctrine that the
filing of a complaint for illegal dismissal is inconsistent with
abandonment of employment.

FACTS:
On 16 May 2000, petitioner Jonathan V. Morales (Morales) was
hired by respondent Harbour Centre Port Terminal, Inc. (HCPTI) as an
Accountant and Acting Finance Officer, with a monthly salary of
₱18,000.00. Regularized on 17 November 2000, Morales was
promoted to Division Manager of the Accounting Department, for which
he was compensated a monthly salary of ₱33,700.00, plus allowances.
Subsequent to HCPTI’s transfer to its new offices at Vitas, Tondo,
Manila, an inter-office memorandum was issued reassigning Morales
to Operations Cost Accounting.

On 31 March 2003, Morales wrote Singson, protesting that his


reassignment was a clear demotion since the position to which he was
transferred was not even included in HCPTI’s plantilla. In response to
Morales’ grievance, Singson issued an inter-office memorandum
placing Morales on floating status. For the whole of the ensuing month
Morales was absent from work and/or tardy. Singson issued to Morales
a 29 April 2003 inter-office memorandum denominated as a First
Warning. The memorandum reminded Morales that, as an employee
of HCPTI, he was subject to its rules and regulations and could be
disciplinarily dealt with pursuant to its Code of Conduct. In view of the
absences Morales continued to incur, HCPTI issued a Second
Warning dated 6 May 200311 and a Notice to Report for Work and
Final Warning dated 22 May 2003.

960
Page 961

Thereafter, Morales filed a complaint dated 25 April 2003 against


HCPTI, Filart and Singson, for constructive dismissal, moral and
exemplary damages as well as attorney’s fees.

ISSUE:
Was Petitioner Morales constructively dismissed from work?

RULING:
Yes, Petitioner Morales was constructively dismissed from work.
The Supreme Court ruled that constructive dismissal exists where
there is cessation of work because "continued employment is rendered
impossible, unreasonable or unlikely, as an offer involving a demotion
in rank or a diminution in pay" and other benefits. Aptly called a
dismissal in disguise or an act amounting to dismissal but made to
appear as if it were not, constructive dismissal may, likewise, exist if
an act of clear discrimination, insensibility, or disdain by an employer
becomes so unbearable on the part of the employee that it could
foreclose any choice by him except to forego his continued
employment. In cases of a transfer of an employee, the rule is settled
that the employer is charged with the burden of proving that its conduct
and action are for valid and legitimate grounds such as genuine
business necessity and that the transfer is not unreasonable,
inconvenient or prejudicial to the employee. If the employer cannot
overcome this burden of proof, the employee’s transfer shall be
tantamount to unlawful constructive dismissal.

The right of employees to security of tenure does not give them


vested rights to their positions to the extent of depriving management
of its prerogative to change their assignments or to transfer them. By
management prerogative is meant the right of an employer to regulate
all aspects of employment, such as the freedom to prescribe work
assignments, working methods, processes to be followed, regulation
regarding transfer of employees, supervision of their work, lay-off and
discipline, and dismissal and recall of workers. Although jurisprudence
recognizes said management prerogative, it has been ruled that the
exercise thereof, while ordinarily not interfered with, is not absolute and
is subject to limitations imposed by law, collective bargaining
agreement, and general principles of fair play and justice. Thus, an
employer may transfer or assign employees from one office or area of
operation to another, provided there is no demotion in rank or
diminution of salary, benefits, and other privileges, and the action is
not motivated by discrimination, made in bad faith, or effected as a
form of punishment or demotion without sufficient cause. Indeed,
having the right should not be confused with the manner in which that
right is exercised.

961
Page 962

Although much had been made about Morales’ supposed refusal


to heed his employer’s repeated directives for him to return to work,
our perusal of the record also shows that HCPTI’s theory of
abandonment of employment cannot bear close scrutiny. While
ostensibly dated 6 May 2003, the Inter-Office Memorandum labeled as
a Second Warning was sent to Morales thru the JRS Express only on
9 May 200357 or two (2) days after summons were served on HCPTI,
Filart and Singson on 7 May 2003. Sent to Morales on 26 May 2003 or
after the parties’ initial conference before the Labor Arbiter on 19 May
2003, there was obviously even less reason for HCPTI’s 22 May 2003
letter denominated as Notice to Report for Work and Final Warning. As
a just and valid ground for dismissal, at any rate, abandonment
requires the deliberate, unjustified refusal of the employee to resume
his employment, without any intention of returning. Since an employee
like Morales who takes steps to protest his dismissal cannot logically
be said to have abandoned his work, it is a settled doctrine that the
filing of a complaint for illegal dismissal is inconsistent with
abandonment of employment.

962
Page 963

Case Digest by: BRODIT, VYEL MARIE C.

SHS PERFORATED MATERIALS, INC. v. DIAZ


G.R. No. 185814. October 13, 2010.
Mendoza, J.

DOCTRINE:
ARTICLE 300. TERMINATION BY AN EMPLOYEE.

1. Management prerogative refers "to the right of an employer to


regulate all aspects of employment, such as the freedom to
prescribe work assignments, working methods, processes to
be followed, regulation regarding transfer of employees,
supervision of their work, lay-off and discipline, and dismissal
and recall of work." Although management prerogative refers
to "the right to regulate all aspects of employment," it cannot be
understood to include the right to temporarily withhold
salary/wages without the consent of the employee.
2. Probationary employees who are unjustly dismissed during the
probationary period are entitled to reinstatement and payment
of full backwages and other benefits and privileges from the
time they were dismissed up to their actual reinstatement.

FACTS:
Respondent Manuel F. Diaz was hired by petitioner SHS as
Manager for Business Development on probationary status from July
18, 2005 to January 18, 2006, with a monthly salary of P100,000.00 to
which he was required to perform sales/marketing functions. Petitioner
Winfried Hartmannshenn (Hartmannshenn), was the president of SHS
Perforated Materials, Inc.

During meetings with the respondent, Hartmannshenn expressed


his dissatisfaction over respondent’s poor performance. Respondent
allegedly failed to make any concrete business proposal or implement
any specific measure to improve the productivity of the SHS office and
plant or deliver sales except for a meagre P2,500.00 for a sample
product.

On November 29, 2005, Hartmannshenn instructed Taguiang not


to release respondent’s salary. Later that afternoon, respondent called
and inquired about his salary. Taguiang informed him that it was being
withheld and that he had to immediately communicate with
Hartmannshenn. Again, respondent denied having received such
directive.

963
Page 964

The next day, on November 30, 2005, respondent served on SHS


a demand letter and a resignation letter. In said letter, respondent
alleged of illegal and unfair labor practices as the reason for his
resignation. In the evening of the same day, respondent met with
Hartmannshenn in Alabang. The latter told him that he was extremely
disappointed for the following reasons: his poor work performance; his
unauthorized leave and malingering from November 16 to November
30, 2005; and failure to immediately meet Hartmannshenn upon his
arrival from Germany.

Parties were unable to settle the issue amicable and on December


9, 2005, respondent filed a Complaint against the petitioners for illegal
dismissal.

ISSUES:
1. Was Petitioner Company’s act of withholding respondent’s salary
in view of his failure to render actual work a valid exercise of
management prerogative?
2. Was Respondent Diaz constructively dismissed?

RULING:
1. No, Petitioner Company’s act was an invalid exercise of
management prerogative. Petitioners’ evidence insufficient to
prove that respondent did not work from November 16 to
November 30, 2005.

The Supreme Court held that management prerogative refers "to


the right of an employer to regulate all aspects of employment,
such as the freedom to prescribe work assignments, working
methods, processes to be followed, regulation regarding transfer
of employees, supervision of their work, lay-off and discipline,
and dismissal and recall of work." Although management
prerogative refers to "the right to regulate all aspects of
employment," it cannot be understood to include the right to
temporarily withhold salary/wages without the consent of the
employee.

As correctly pointed out by the LA, "absent a showing that the


withholding of complainants wages falls under the exceptions
provided in Article 113, the withholding thereof is thus unlawful."

As the nature of respondent’s job did not allow close supervision


and monitoring by petitioners. Neither was there any prescribed
daily monitoring procedure established by petitioners to ensure
that respondent was doing his job. Therefore, granting that
respondent failed to answer Hartmannshenn’s mobile calls and
to reply to two electronic mail messages and given the fact that

964
Page 965

he admittedly failed to report to work at the SHS plant twice each


week during the subject period, such cannot be taken to signify
that he did not work from November 16 to November 30, 2005.

Furthermore, the electronic mail reports sent to Hartmannshenn


and the receipt presented by respondent as evidence of his
having worked during the subject period were not controverted
by petitioners. Although it cannot be determined with certainty
whether respondent worked for the entire period from November
16 to November 30, 2005, the consistent rule is that if doubt
exists between the evidence presented by the employer and that
by the employee, the scales of justice must be tilted in favor of
the latter in line with the policy mandated by Articles 2 and 3 of
the Labor Code to afford protection to labor and construe doubts
in favor of labor. For petitioners’ failure to satisfy their burden of
proof, respondent is presumed to have worked during the period
in question and is, accordingly, entitled to his salary. Therefore,
the withholding of respondent’s salary by petitioners is contrary
to Article 116 of the Labor Code and, thus, unlawful.

2. Yes, Respondent Diaz was constructively dismissed. There is


constructive dismissal if an act of clear discrimination,
insensibility, or disdain by an employer becomes so unbearable
on the part of the employee that it would foreclose any choice by
him except to forego his continued employment. It exists where
there is cessation of work because continued employment is
rendered impossible, unreasonable or unlikely, as an offer
involving a demotion in rank and a diminution in pay.

What made it impossible, unreasonable or unlikely for


respondent to continue working for SHS was the unlawful
withholding of his salary. For said reason, he was forced to
resign.

Although respondent was a probationary employee, he was still


entitled to security of tenure. Section 3 (2) Article 13 of the
Constitution guarantees the right of all workers to security of
tenure. In using the expression "all workers," the Constitution
puts no distinction between a probationary and a permanent or
regular employee. This means that probationary employees
cannot be dismissed except for cause or for failure to qualify as
regular employees.

This Court has held that probationary employees who are


unjustly dismissed during the probationary period are entitled to
reinstatement and payment of full backwages and other benefits
and privileges from the time they were dismissed up to their
actual reinstatement. Respondent is, thus, entitled to

965
Page 966

reinstatement without loss of seniority rights and other privileges


as well as to full backwages, inclusive of allowances, and other
benefits or their monetary equivalent computed from the time his
compensation was withheld up to the time of actual
reinstatement. Respondent, however, is not entitled to the
additional amount for 13th month pay, as it is clearly provided in
respondents Probationary Contract of Employment that such is
deemed included in his salary.

966
Page 967

Case Digest by: BRODIT, VYEL MARIE C.

SAN MIGUEL PROPERTIES PHILIPPINES, INC. v.


GUCABAN
G.R. No. 153982. July 18, 2011.
Peralta, C. J.

DOCTRINE:
ARTICLE 300. TERMINATION BY AN EMPLOYEE.

Resignation - the formal pronouncement or relinquishment of a position


or office - is the voluntary act of an employee who is in a situation
where he believes that personal reasons cannot be sacrificed in favor
of the exigency of the service, and he has then no other choice but to
disassociate himself from employment. The intent to relinquish must
concur with the overt act of relinquishment; hence, the acts of the
employee before and after the alleged resignation must be considered
in determining whether he in fact intended to terminate his
employment. In illegal dismissal cases, fundamental is the rule that
when an employer interposes the defense of resignation, on him
necessarily rests the burden to prove that the employee indeed
voluntarily resigned.

FACTS:
Respondent Gwendellyn Rose Gucaban (Gucaban) was a project
development manager of petitioner San Miguel Properties Philippines,
Inc. (SMPI). She had been in continuous service in the latter capacity
since 1991 until her severance from the company in February 1998.

In her complaint for illegal dismissal filed on June 26, 1998,


Gucaban alleged that her separation from service was practically
forced upon her by management. She claimed that on January 27,
1998, she was informed by SMPI's President and Chief Executive
Officer, Federico Gonzalez (Gonzalez), that the company was planning
to reorganize its manpower in order to cut on costs, and that she must
file for resignation or otherwise face termination. Three days later, the
Human Resource Department allegedly furnished her a blank
resignation form which she refused to sign. From then on, she had
been hounded by Gonzalez to sign and submit her resignation letter.

Gucaban complained of the ugly treatment which she had since


received from Gonzalez and the management supposedly on account
of her refusal to sign the resignation letter. She claimed she had been
kept off from all the meetings of the management committee, and that

967
Page 968

on February 12, 1998, she received an evaluation report signed by


Gonzalez showing that for the covered period she had been negligent
and unsatisfactory in the performance of her duties. It was supposedly
the extreme humiliation and alienation that impelled her to submit a
signed resignation letter on February 18, 1998.

ISSUE:
Was Respondent Gucaban constructively dismissed?

RULING:
Yes, Respondent Gucaban was constructively dismissed. The
resignation of Gucaban lacked the element of voluntariness. She had
been constructively and, hence, illegally dismissed as indeed her
continued employment is rendered impossible, unreasonable or
unlikely under the circumstances.

Resignation - the formal pronouncement or relinquishment of a


position or office - is the voluntary act of an employee who is in a
situation where he believes that personal reasons cannot be sacrificed
in favor of the exigency of the service, and he has then no other choice
but to disassociate himself from employment. The intent to relinquish
must concur with the overt act of relinquishment; hence, the acts of the
employee before and after the alleged resignation must be considered
in determining whether he in fact intended to terminate his
employment. In illegal dismissal cases, fundamental is the rule that
when an employer interposes the defense of resignation, on him
necessarily rests the burden to prove that the employee indeed
voluntarily resigned. Guided by these principles, we agree with the
Court of Appeals that with the availing evidence, SMPI was unable to
discharge this burden.

While indeed the abolition of Gucaban's position as a


consequence of petitioner's supposed reorganization plan is not the
ground invoked in this case of termination, still, the question of whether
or not there was such reorganization plan in place at the time of
Gucaban's separation from the company, is material to the
determination of whether her resignation was of her own volition as
claimed by SMPI, inasmuch as the facts of this case tell that Gucaban
could not have filed for resignation had Gonzalez not communicated to
her the alleged reorganization plan for the company.

Based on the records, shortly prior to and at the time of Gucaban's


alleged resignation, there was actually no genuine corporate
restructuring plan in place as yet. In other words, although the
company might have been suffering from losses due to market decline
as alleged, there was still no concrete plan for a corporate

968
Page 969

reorganization at the time Gonzalez presented to Gucaban the


seemingly last available alternative options of voluntary resignation
and termination by abolition of her office. Certainly, inasmuch as the
necessity of corporate reorganization generally lies within the exclusive
prerogative of management, Gucaban at that point had no facility to
ascertain the truth behind it, and neither was she in a position to
question it right then and there. Indeed, she could not have chosen to
file for resignation had SMPI not broached to her the possibility of her
being terminated from service on account of the supposed
reorganization.

Besides, whether there have been negotiations or not, the


irreducible fact remains that Gucaban's separation from the company
was the confluence of the fraudulent representation to her that her
office would be declared redundant, coupled with the subsequent
alienation which she suffered from the company by reason of her
refusal to tender resignation. The element of voluntariness in her
resignation is, therefore, missing. She had been constructively and,
hence, illegally dismissed as indeed her continued employment is
rendered impossible, unreasonable or unlikely under the
circumstances.

In Samaniego, one of the issues addressed by the Court is


whether the resignation of petitioners therein was voluntary; but while
the matter of reorganization was indeed raised as a peripheral issue,
nevertheless, the same has dealt merely with the validity thereof. As
in the cases of Domondon and Guerzon, the Court, in Samaniego, did
not tackle the matter of the existence or non-existence of a genuine
and bona fide reorganization at the time the option to resign was
presented to the employee as would affect his decision to voluntarily
resign or not. And in Sicangco, the Court dismissed the allegation of
involuntary resignation by a well-educated employee because there
was no proven fraud, intimidation or undue influence that could support
it. In the instant case, the pressing matter is whether there was in place
a genuine reorganization plan awaiting immediate implementation in
good faith at or about the time Gucaban resolved to hand in her
resignation letter. This issue is primordial, because to reiterate,
Gucaban indeed would not have opted to resign without the company
having laid out to her its prospect of a corporate restructuring - which
SMPI failed to establish as existing at the time - as well as the certainty
of a consequent termination should she not resign.

969
Page 970

Case Digest by: BRODIT, VYEL MARIE C.

BMG RECORDS (PHILS.), INC. v. APARECIO


G.R. NO. 153290. September 5, 2007.
Azcuna, J.

DOCTRINE:
ARTICLE 300. TERMINATION BY AN EMPLOYEE.

A resigned employee who desires to take his job back has to re-apply
therefor, and he shall have the status of a stranger who cannot
unilaterally demand an appointment. He cannot arrogate unto himself
the same position which he earlier decided to leave. To allow him to
do so would be to deprive the employer of his basic right to choose
whom to employ.

FACTS:
Private respondent Aida C. Aparecio (Aparecio) was hired by
Petitioner BMG Records (Phils), Inc. (BMG) on September 2, 1990 as
one of the promo girls in its Cebu Branch. On May 25, 1998, Aparecio
filed a complaint against BMG and its Branch Manager, Jose Yap, Jr.,
co-petitioner herein, for illegal dismissal. In her complaint, she alleged
that before she was illegally dismissed, she was asked by respondent
to resign and will be paid (sic) all her benefits due.

Petitioners, on the other hand alleged that Aparecio was initially


performing well as an employee but as years passed she seemed to
be complacent in the performance of her job and had been comparing
the salaries of promo girls in other companies. It appeared that she
was no longer interested in her job. In April 1998, Aparecio and two
other promo girls, Jovelina V. Soco and Veronica P. Mutya, intimated
to their supervisor that they were intending to resign and were
requesting for some financial assistance. BMG made it clear that, as a
company policy, an employee who resigns from service is not entitled
to financial assistance, but considering the length of their service and
due to humanitarian consideration it would accede to the request after
they secure their respective clearances.

ISSUE:
Was the resignation of respondent Aparecio valid?

RULING:

970
Page 971

Yes, respondent Aparecio’s resignation was valid. There is


nothing to support Aparecio's allegation that fraud was employed on
her to resign.

The Supreme Court ruled that based on the the pleadings, this
Court finds nothing to support Aparecio's allegation that fraud was
employed on her to resign. Fraud exists only when, through insidious
words or machinations, the other party is induced to act and without
which, the latter would not have agreed to. This Court has held that the
circumstances evidencing fraud and misrepresentation are as varied
as the people who perpetrate it, each assuming different shapes and
forms and may be committed in as many different ways. Fraud and
misrepresentation are, therefore, never presumed; it must be proved
by clear and convincing evidence and not mere preponderance of
evidence. Hence, this Court does not sustain findings of fraud upon
circumstances which, at most, create only suspicion; otherwise, it
would be indulging in speculations and surmises.

Moreover, Aparecio did not adduce any competent evidence to


prove that force or threat was applied by petitioners. For intimidation to
vitiate consent, the following requisites must be present: (1) that the
intimidation caused the consent to be given; (2) that the threatened act
be unjust or unlawful; (3) that the threat be real or serious, there being
evident disproportion between the evil and the resistance which all
men can offer, leading to the choice of doing the act which is forced on
the person to do as the lesser evil; and (4) that it produces a well-
grounded fear from the fact that the person from whom it comes has
the necessary means or ability to inflict the threatened injury to his
person or property. In the instant case, not one of these essential
elements was amply proven by Aparecio. Bare allegations of threat or
force do not constitute substantial evidence to support a finding of
forced resignation.

Resignation is the voluntary act of an employee who is in a


situation where one believes that personal reasons cannot be
sacrificed in favor of the exigency of the service, and one has no other
choice but to dissociate oneself from employment. It is a formal
pronouncement or relinquishment of an office, with the intention of
relinquishing the office accompanied by the act of relinquishment. As
the intent to relinquish must concur with the overt act of relinquishment,
the acts of the employee before and after the alleged resignation must
be considered in determining whether in fact, he or she intended to
sever from his or her employment.

The Court agrees with petitioners' contention that the


circumstances surrounding Aparecio's resignation should be given due

971
Page 972

weight in determining whether she had intended to resign. In this case,


such intent is very evident:

First, Aparecio already communicated to other people that she


was about to resign to look for a better paying job since she had been
complaining that employees like her in other companies were earning
much more;

Second, prior to the submission of her resignation letter, Aparecio


and two other promo girls, Soco and Mutya, approached their
supervisor, intimated their desire to resign, and requested that they be
given financial assistance, which petitioners granted on the condition
that deductions would be made in case of shortage after inventory;

Third, Aparecio, Soco, and Mutya submitted their duly signed


resignation letters, which were accepted by petitioners; and

Fourth, Aparecio already initiated the processing of her clearance;


thus, she was able to receive her last salary, 13th month pay, and tax
refund but refused to receive the financial assistance less the
deductions made.

The foregoing facts were affirmatively narrated and attested to in


the notarized affidavit of Soco and Cinco and have remained
incontrovertible as they were never denied by Aparecio.

Now, the acceptance by petitioners of Aparecio's resignation


rendered the same effective. Upon such acceptance, it may not be
unilaterally withdrawn without the consent of petitioners. When the
employee later signified the intention of continuing his or her work, it
was already up to the employer to accept the withdrawal of his or her
resignation. The mere fact that the withdrawal was not accepted does
not constitute illegal dismissal, the acceptance of the withdrawal of the
resignation being the employer's sole prerogative. In Philippine Today,
Inc. v. NLRC, it was held that:

A resigned employee who desires to take his job back has to re-
apply therefor, and he shall have the status of a stranger who
cannot unilaterally demand an appointment. He cannot arrogate
unto himself the same position which he earlier decided to leave.
To allow him to do so would be to deprive the employer of his basic
right to choose whom to employ. Such is tantamount to undue
oppression of the employer. It has been held that an employer is
free to regulate, according to his own discretion and judgment, all
aspects of employment including hiring. The law, in protecting the

972
Page 973

rights of the laborer, impels neither the oppression nor self-


destruction of the employer.

Therefore, there was no illegal dismissal in this case. What


transpired here was caused by an employee's error of judgment and
not by the employer's application of means vitiating the consent to
resign.

973
Page 974

Case Digest by: BRODIT, VYEL MARIE C.

TATEL v. JLFP INVESTIGATION SECURITY AGENCY,


INC.,
G.R. No. 206942. February 25, 2015.
Perlas – Bernabe, J.

DOCTRINE:
ARTICLE 300. TERMINATION BY AN EMPLOYEE.

Temporary "off-detail" or "floating status" is the period of time when


security guards are in between assignments or when they are made to
wait after being relieved from a previous post until they are transferred
to a new one. It does not constitute a dismissal, as the assignments
primarily depend on the contracts entered into by the security agencies
with third parties, so long as such status does not continue beyond a
reasonable time. When such a "floating status" lasts for more than six
(6) months, the employee may be considered to have been
constructively dismissed.

FACTS:
Petitioner, Vicente C. Tatel was hired by respondent company
JLFP as one of its security guards. He was posted at BaggerWerken
Decloedt En Zoon where he was required to work twelve (12) hours
everyday from Mondays through Sundays and received only
P12,400.00 as monthly salary. On October 14, 2009, Tatel filed a
complaint10 before the NLRC against JLFP for underpayment of
salaries and wages, non-payment of other benefits, 13th month pay,
and attorney's fees (underpayment case).

On October 24, 2009, Tatel was placed on "floating status"; thus,


on May 4, 2010, or after the lapse of six (6) months therefrom, without
having been given any assignments, he filed another complaint against
JLFP and its officers, or illegal dismissal, reinstatement, backwages,
refund of cash bond deposit amounting to P25,400.00, attorney's fees,
and other money claims (illegal dismissal case).

In their defense, respondents denied that Tatel was dismissed and


averred that they removed the latter from his post at BaggerWerken on
August 24, 2009 because of several infractions he committed while on
duty. Notwithstanding the pendency of the underpayment case,
respondents sent a Memorandum directing Tatel to report back to
work, noting that the latter last reported to the office on October 26,
2009. However, despite receipt of the said memorandum, respondents

974
Page 975

averred that Tatel ignored the same and failed to appear; hence, he
was deemed to have abandoned his work.

In his reply, admitted having received the subject memorandum


but when he he went to the JLFP office, he was merely advised to "wait
for possible posting." He repeatedly went back to the office for
reassignment, but to no avail.

ISSUE:
Was Petitioner Tatel constructively dismissed from work?

RULING:
Yes, Petitioner Tatel was constructively dismissed from work. The
Supreme Court held that Tatel was constructively, not actually,
dismissed after having been placed on "floating status" for more than
six (6) months, reckoned from October 24, 2009, the day following his
removal from his last assignment with IPVG.

In Superstar Security Agency, Inc. and/or Col. Andrada v. NLRC,


the Court ruled that placing an employee on temporary "off-detail" is
not equivalent to dismissal provided that such temporary inactivity
should continue only for a period of six (6) months. In security agency
parlance, being placed "off-detail" or on "floating status" means
"waiting to be posted." In Salvaloza v. NLRC, the Court further
explained the nature of the "floating status," to wit:

Temporary "off-detail" or "floating status" is the period of time


when security guards are in between assignments or when they
are made to wait after being relieved from a previous post until
they are transferred to a new one. It takes place when the security
agency's clients decide not to renew their contracts with the
agency, resulting in a situation where the available posts under its
existing contracts are less than the number of guards in its roster.
It also happens in instances where contracts for security services
stipulate that the client may request the agency for the
replacement of the guards assigned to it even for want of cause,
such that the replaced security guard may be placed on temporary
"off-detail" if there are no available posts under the agency's
existing contracts. During such time, the security guard does not
receive any salary or any financial assistance provided by law. It
does not constitute a dismissal, as the assignments primarily
depend on the contracts entered into by the security agencies with
third parties, so long as such status does not continue beyond a
reasonable time. When such a "floating status" lasts for more than
six (6) months, the employee may be considered to have been
constructively dismissed.

975
Page 976

Constructive dismissal exists when an act of clear discrimination,


insensibility, or disdain, on the part of the employer has become so
unbearable as to leave an employee with no choice but to forego
continued employment, or when there is cessation of work because
continued employment is rendered impossible, unreasonable, or
unlikely, as an offer involving a demotion in rank and a diminution in
pay.

In this case, respondents themselves claimed that after having


removed Tatel from his post at BaggerWerken on August 24, 2009 due
to several infractions committed thereat, they subsequently reassigned
him to SKI from September 16, 2009 to October 12, 2009 and then to
IPVG from October 21 to 23, 2009. Thereafter, and until Tatel filed the
instant complaint for illegal dismissal six (6) months later, or on May 4,
2010, he was not given any other postings or assignments. While it
may be true that respondents summoned him back to work through the
November 26, 2009 Memorandum, which Tatel acknowledged to have
received on December 11, 2009, records are bereft of evidence to
show that he was given another detail or assignment. As the "off-detail"
period had already lasted for more than six (6) months, Tatel is
therefore deemed to have been constructively dismissed.

976
Page 977

Case Digest by: BRODIT, VYEL MARIE C.

PAREDES v. FEED THE CHILDREN PHILIPPINES, INC.,


G.R. No. 184397. September 09, 2015.
Peralta, C. J.

DOCTRINE:
ARTICLE 300. TERMINATION BY AN EMPLOYEE.

The act of the employer moving the effectivity of the resignation is not
an act of harassment. The 30-day notice requirement for an
employee’s resignation is actually for the benefit of the employer who
has the discretion to waive such period. Its purpose is to afford the
employer enough time to hire another employee if needed and to see
to it that there is proper turn-over of the tasks which the resigning
employee may be handling.

Such rule requiring an employee to stay or complete the 30-day period


prior to the effectivity of his resignation becomes discretionary on the
part of management as an employee who intends to resign may be
allowed a shorter period before his resignation becomes effective.

FACTS:
Respondent Feed the Children Philippines, Inc. (FTCP) is a
nonstock, non-profit, and non-government organization duly
incorporated under the Philippine laws in 1989. Petitioner Rosalinda
Paredes was FTCP's National Director. Due to complaints (such
seeking exemption from policies which she herself had approved;
withholding organization funds despite approval of its release;
procuring health insurance for herself without paying her share of the
premium; and receiving additional fees contrary to the terms of her
contract) involving Petitioner Paredes, the FTCP decided to hire an
independent professional management and financial auditor.

But because of Paredes’ indifferent attitude and unjustified refusal


to submit to an audit, the Board resolved to suspend her. Before it
could be implemented, respondent FTCP received her resignation
letter stating that she can only serve the organization up to December
31, 2005. The Board accepted her resignation with the condition that
its effectivity be moved to November 30, 2005.

On November 2, 2005, petitioner filed a Complaint for illegal


dismissal, claiming that she was forced to resign, thus, was

977
Page 978

constructively dismissed, and impleaded Lao, Paradiang and Escobia


in their personal capacities.

ISSUE:
Was Petitioner Paredes constructively dismissed from work?

RULING:
No, petitioner cannot be deemed constructively dismissed from
work. The Supreme Court ruled that constructive dismissal occurs
when there is cessation of work because continued employment is
rendered impossible, unreasonable or unlikely; when there is a
demotion in rank or diminution in pay or both; or when a clear
discrimination, insensibility, or disdain by an employer becomes
unbearable to the employee. The test is whether a reasonable person
in the employee's position would have felt compelled to give up his
position under the circumstances.

In this case, petitioner cannot be deemed constructively


dismissed. She failed to present clear and positive evidence that
respondent FTCP, through its Board of Trustees, committed acts of
discrimination, insensibility, or disdain towards her which rendered her
continued employment unbearable or forced her to terminate her
employment from the respondent. As settled, bare allegations of
constructive dismissal, when uncorroborated by the evidence on
record, cannot be given credence.

Furthermore, the fact that the effectivity of her resignation was


moved to November is not sufficient to support the claim that Lao,
Paradiang and Escobia were eager to get rid of her. The act of the
employer moving the effectivity of the resignation is not an act of
harassment. The 30-day notice requirement for an employee’s
resignation is actually for the benefit of the employer who has the
discretion to waive such period. Its purpose is to afford the employer
enough time to hire another employee if needed and to see to it that
there is proper turn-over of the tasks which the resigning employee
may be handling.

Such rule requiring an employee to stay or complete the 30-day


period prior to the effectivity of his resignation becomes discretionary
on the part of management as an employee who intends to resign may
be allowed a shorter period before his resignation becomes effective.

Thus, the act of respondents moving the effectivity date of


petitioner’s resignation to a date earlier than what she had stated
cannot be deemed malicious. This cannot be viewed as an act of
harassment but merely the exercise of respondent's management

978
Page 979

prerogative. We cannot expect employers to maintain in their employ


employees who intend to resign, just so the latter can have continuous
work as they look for a new source of income.

979
Page 980

Case Digest by: BRODIT, VYEL MARIE C.

SILVERTEX WEAVING CORPORATION v. CAMPO


G.R. No. 211411. March 16, 2016.
Reyes, J.

DOCTRINE:
ARTICLE 300. TERMINATION BY AN EMPLOYEE.

Even granting that such document (Waiver, Release and Quitclaims


Statement) was actually executed by the respondent, its execution was
not fatal to the respondent's case for illegal dismissal. The finding of
illegal dismissal could still stand, as jurisprudence provides that "[a]n
employee's execution of a final settlement and receipt of amounts
agreed upon do not foreclose his right to pursue a claim for illegal
dismissal."

FACTS:
The case stems from a complaint for illegal dismissal and
monetary claims filed by Teodora F. Campo (respondent) against the
petitioners, wherein she claimed that she worked for Silvertex Weaving
Corporation (STWC) as a weaving machine operator beginning June
11, 1999, until she was unlawfully dismissed from employment on
November 21, 2010. Prior to her dismissal, she was suspended for one
week beginning November 14, 2010 after a stitching machine that she
was operating overheated and emitted smoke on November 13, 2010.
When the respondent tried to report back to work on November 21,
2010, she was denied entry by the STWC's security guard, reportedly
upon the instructions of Arcenal.

For their defense, the petitioners argued that the respondent, who
was hired only in June 2009, voluntarily resigned from STWC after she
was reprimanded for poor job performance. They submitted a
handwritten resignation letter allegedly executed by the respondent on
November 13, 2010, together with the Waiver, Release and Quitclaims
Statement that she supposedly signed following her receipt of
P30,000.00 from STWC.

ISSUE:
Was respondent Campos illegally dismissed from work?

RULING:
Yes, respondent was illegally dismissed from work. The Supreme
Court ruled that it is well-settled by jurisprudence that in labor cases,

980
Page 981

"the employer has the burden of proving that the employee was not
dismissed, or, if dismissed, that the dismissal was not illegal." The
NLRC's pronouncement that it was incumbent upon the respondent to
dispute the genuineness of her signature on the resignation letter was
then clearly misplaced. As the Court emphasized in San Miguel
Properties Philippines, Inc. v. Gucaban:

Resignation - the formal pronouncement or relinquishment of a


position or office - is the voluntary act of an employee who is in a
situation where he believes that personal reasons cannot be
sacrificed in favor of the exigency of the service, and he has then
no other choice but to disassociate himself from employment. The
intent to relinquish must concur with the overt act of
relinquishment; hence, the acts of the employee before and after
the alleged resignation must be considered in determining
whether he in fact intended to terminate his employment. In illegal
dismissal cases, fundamental is the rule that when an employer
interposes the defense of resignation, on him necessarily rests the
burden to prove that the employee indeed voluntarily resigned.

The petitioners attempted to discharge the burden of proving the


respondent's resignation by referring mainly to a letter allegedly
executed by the respondent. However, the authenticity and due
execution of the undated Waiver, Release and Quitclaims Statement
purportedly signed by the respondent was also not sufficiently
established. The QDR (Question Document Report) was not
conclusive on the issue of its genuineness. Even granting that such
document was actually executed by the respondent, its execution was
not fatal to the respondent's case for illegal dismissal. The finding of
illegal dismissal could still stand, as jurisprudence provides that "[a]n
employee's execution of a final settlement and receipt of amounts
agreed upon do not foreclose his right to pursue a claim for illegal
dismissal."

981
Page 982

Case Digest by: BRODIT, VYEL MARIE C.

DIVINE WORD COLLEGE OF LAOAG v. MINA


G.R. No. 195155. April 13, 2016.
Reyes, J.

DOCTRINE:
ARTICLE 300. TERMINATION BY AN EMPLOYEE.

There is demotion when an employee occupying a highly technical


position requiring the use of one's mental faculty is transferred to
another position, where the employee performed mere mechanical
work — virtually a transfer from a position of dignity to a servile or
menial job.

FACTS:
The Society of Divine Word Educational Association (DWEA)
established a Retirement Plan to provide retirement benefits. It
contained a clause bout the portability of benefits, to wit:

When a member who resigns or is separated from employment


from one Participating Employer and who is employed by another
Participating Employer, the member will carry the credit he
earned under his former Participating Employer to his new
Employer and the length of service in both will be taken into
consideration in determining his total years of continuous service
on the following conditions:

The transfer is approved by both the Participating


Employer whose service he is leaving and the new
Participating Employer;

The Retirement Board is notified of the transfer; and

The member is employed by another Participating


Employer on the next working day after his resignation.

Respondent Delfin Mina was employed in 1971 as a high school


teacher, and later on a high school principal, at the Academy of St.
Joseph (ASJ), a school run by the SVD. He was subsequently
transferred in 2002 to DWCL's college department as an Associate
Professor III. Thereafter, on June 1, 2003, Mina was assigned as the
College Laboratory Custodian of the School of Nursing and was
divested of his teaching load, effective June 1, 2003 until May 31, 2004,

982
Page 983

subject to automatic termination and without need for any further


notification.

In June 2004, he was offered early retirement but he declined the offer
because of his family's dependence on him for support. He later
received a Memorandum from the Office of the Dean enumerating
specific acts of gross or habitual negligence, insubordination, and
reporting for work under the influence of alcohol. He answered the
allegations against him; sensing, however, that it was pointless to
continue employment with DWCL, he requested that his retirement
date be adjusted to September 2004 to enable him to avail of the 25-
year benefits. He also requested for the inclusion of his eight years of
service in ASJ, to make his total years of service to 33 years pursuant
to the portability clause of the retirement plan, which was denied by
DWCL. Instead, he was paid P275,513.10 as retirement pay. It was
made to appear that his services were terminated by reason of
redundancy to avoid any tax implications. Mina was also made to sign
a deed of waiver and quitclaim stating that he no longer has any claim
against DWCL with respect to any matter arising from his employment
in the school.

On September 2004, he filed a case for illegal dismissal and recovery


of separation pay and other monetary claims.

ISSUE:
Was Respondent Mina constructively dismissed from work?

RULING:
Yes, Mina's transfer clearly amounted to a constructive dismissal
from work. The Supreme Court ruled that constructive dismissal is a
dismissal in disguise. There is cessation of work in constructive
dismissal because '"continued employment is rendered impossible,
unreasonable or unlikely, as an offer involving a demotion in rank or a
diminution in pay' and other benefits." To be considered as such, an
act must be a display of utter discrimination or insensibility on the part
of the employer so intense that it becomes unbearable for the
employee to continue with his employment. The law recognizes and
resolves this situation in favor of employees in order to protect their
rights and interests from the coercive acts of the employer.

Mina's transfer clearly amounted to a constructive dismissal. For


almost 22 years, Mina was a high school teacher enjoying a permanent
status in DWCL's high school department. In 2002, he was appointed
as an associate professor at the college department but shortly
thereafter, or on June 1, 2003, he was appointed as a college
laboratory custodian, which is a clear relegation from his previous

983
Page 984

position. Not only that. He was also divested of his teaching load. His
appointment even became contractual in nature and was subject to
automatic termination after one year "without any further notification."
Aside from this, Mina was the only one among the high school teachers
transferred to the college department who was divested of teaching
load. More importantly, DWCL failed to show any reason for Mina's
transfer and that it was not unreasonable, inconvenient, or prejudicial
to him.

Mina's appointment as laboratory custodian was a demotion.


There is demotion when an employee occupying a highly technical
position requiring the use of one's mental faculty is transferred to
another position, where the employee performed mere mechanical
work — virtually a transfer from a position of dignity to a servile or
menial job. The assessment whether Mina's transfer amounted to a
demotion must be done in relation to his previous position, that is, from
an associate college professor, he was made a keeper and inventory-
taker of laboratory materials. Clearly, Mina's new duties as laboratory
custodian were merely perfunctory and a far cry from his previous
teaching job, which involved the use of his mental faculties. And while
there was no proof adduced showing that his salaries and benefits
were diminished, there was clearly a demotion in rank. As was stated
in Blue Dairy Corporation v. NLRC, "[i]t was virtually a transfer from a
position of dignity to a servile or menial job."

984
Page 985

Case Digests by: CALVO, KRISKA ANGELA A.

COKIA INDUSTRIES HOLDINGS MANAGEMENT, INC. vs.


BEATRIZ C. BUG-OS
G.R. No. 236322. November 27, 2019
CARANDANG, J.

DOCTRINE:
CONSTRUCTIVE DISMISSAL vs. VOLUNTARY RESIGNATION
There is a difference between constructive dismissal and voluntary
resignation. Constructive dismissal exists if an act of clear
discrimination, insensibility, or disdain by an employer becomes so
unbearable on the part of the employee that it could foreclose any
choice by him or her except to forego his or her continued employment.
It is the employee who bears the burden of proving the same.
Resignation refers to the voluntary act of an employee who is in a
situation where one believes that personal reasons cannot be
sacrificed in favor of the exigency of the service, and one has no other
choice but to dissociate oneself from employment.

FACTS:
Respondent Bug-os was employed as CIHMI's (Cokia) accounting
personnel. Following the discovery by the CFO of Cokia of several
irregularities including forgeries and falsifications on the loans falsely
obtained, Bug-os was sent an office memorandum directing her to
explain her participation in securing the false loans. Respondent
denied the same and resigned 2 days after. She filed a complaint for
constructive dismissal, claiming that George and his mother (Officer
and CFO of Cokia, respectively) subjected her to harsh treatment the
moment the irregular transactions were discovered. This made working
for CIHMI unbearable and compelled her to resign.

Petitioner CIHMI denied the allegations and averred that


respondent's resignation was voluntary. The LA dismissed the claim of
Bug-os with prejudice and for lack of merit. This was reversed by the
NLRC whose decision was then affirmed by the CA.

ISSUE:
Whether or not Bug-os was constructively dismissed.

RULING:
Negative. There is a difference between constructive dismissal
and voluntary resignation. Constructive dismissal exists if an act of
clear discrimination, insensibility, or disdain by an employer becomes
so unbearable on the part of the employee that it could foreclose any

985
Page 986

choice by him or her except to forego his or her continued employment.


It is the employee who bears the burden of proving the same.
Resignation refers to the voluntary act of an employee who is in a
situation where one believes that personal reasons cannot be
sacrificed in favor of the exigency of the service, and one has no other
choice but to dissociate oneself from employment. The employer has
the burden of proving that an employee voluntarily resigned. The test
for determining if an employee was constructively dismissed is whether
a reasonable person in the employee's position would feel compelled
to give up his or her employment under the prevailing circumstances.
The acts of the employee before and after the alleged resignation must
be considered in determining whether he or she, in fact, intended to
sever his or her employment.

Applying the same in this case, it is apparent that Bug-os was not
constructively dismissed. Not only was respondent unable to prove her
dismissal allegations, her claim also lacked credibility. Respondent
alleged that it was the strong words and harsh treatment of the
petitioners that led to her resignation but she failed to substantiate the
same. Strong words from the employer do not necessarily make the
working environment unbearable. When these are uttered "without
palpable reason or are expressed only for the purpose of degrading
the dignity of the employee, then a hostile work environment will be
created.

Moreover, Bug-Os resigned merely two days after she was given
the Office Memorandum. It is incredulous that in that short span of time,
she was subjected to so much harassment that it made working for
CIHMI unbearable. While there is no fixed period for constructive
dismissal, the period from the time Bug-Os was asked to explain the
irregularities discovered until the time she resigned simply does not
lend credibility to her claim that she was constructively dismissed.

986
Page 987

Case Digests by: CALVO, KRISKA ANGELA A.

YUSHI KONDO vs. TOYOTA BOSHOKU CORPORATION


G.R. No. 201396. September 11, 2019
Jardeleza, J.

DOCTRINE:
CONSTRUCTIVE DISMISSAL
The employee bears the burden to prove by substantial evidence
the fact of his dismissal from employment. Absent any showing of an
overt or positive act proving that the employer had dismissed the
employee, the claim of illegal dismissal cannot be sustained as it would
be self-serving, conjectural, and of no probative value.

FACTS:
Petitioner Kondo is a Japanese citizen who was locally hired by
Toyota Philippines as Assistant General Manager for Marketing,
Procurement and Accounting. Aside from his salary, he was also
receiving other benefits including a service car, local driver and Caltex
card for gasoline expenses. This was during the time of Fuhimiko Ito,
then President of Toyota Philippines. The company's Japan
headquarters discovered anomalies committed by the latter and he
was eventually replaced by respondent Mamoru Matsunaga.

Kondo thereafter alleged the following:


1.) He was given an unfair performance evaluation rating which
coincided with the discovery of the anomalies committed by Ito.
2.) He was assigned the oldest company car and prevented from
using other company cars tor business travels. His driver was
withdrawn.
3.) He was prevented from further using his Caltex card for
gasoline expenses, and instructed to pay for gas expenses with his
own money, subject to reimbursement.
4.) Petitioner was transferred to the Production Control, Technical
Development and Special Project department as Assistant Manager,
an area where he had no knowledge, skills, and experience.

Basically, petitioner alleges that the benefits he previously


received, which he deemed essential requisites for his continued
employment, were diminished thereby constraining him to file a
complaint for illegal dismissal. According to petitioner, without said
benefits, he cannot report to work hence, he was constructively
dismissed. The allegations were denied by respondent Matsunaga
who argued that the entitlement for the service car and driver were only
for a period of 1 year. The Caltex card for gas expenses is a benefit
exclusively given to Japanese expatriates which the petitioner cannot

987
Page 988

avail of, being a local hire. It is still Toyota who pays for the expenses
of Kondo since the same is subject to reimbursement. Lastly, anent the
transfer, Matsunaga stated that it was an exercise of management
prerogative. Toyota had no intention of dismissing Kondo; it was the
latter who refused to report to work.

The LA ruled in favor of Kondo but the said decision was reversed
by the NLRC; the reversal was upheld by the CA.

ISSUE:
1.) Whether or not there was diminution of benefits.
2.) Whether or not Petitioner was constructively dismissed.

RULING:
1.) Negative. There is diminution of benefits when the following are
present: (a) the grant or benefit is founded on a policy or has ripened
into a practice over a long period of time; (b) the practice is consistent
and deliberate; (c) the practice is not due to error in the construction or
application of a doubtful or difficult question of law; and (d) the
diminution or discontinuance is done unilateral1y by the employer.
Under the first requisite, the benefit must be based on express
policy, a written contract or has ripened into a practice. In this case,
the grant of service car and local driver to petitioner was based neither
on express policy or a written contract nor can the same be considered
a company practice. Petitioner failed to prove that the car and driver
benefits were also being enjoyed by other employees who held
positions equivalent to his position, or that the benefits were given by
the company itself with voluntary and deliberate intent. What is clear
from the records is that the benefits were granted to Kondo through a
verbal agreement which is more of an accommodation that cannot be
demanded.
Anent the Caltex card, petitioner did not show his entitlement to
the same. Even if the same was withdrawn, Kondo's gasoline
expenses were still subject to reimbursement. Hence, at the end of the
day, it was still Toyota that paid for his gasoline consumption.

2.) Negative. Petitioner did not allege and prove specific facts that
would indicate his inability to function fully in the new department as a
result of his lack of expertise, or that his transfer constituted dear
discrimination or harassment. He also did not address Toyota's
assertion that his new function required him merely to oversee the
department and carry out management policies, rather than participate
in production and technical development. The mere fact of petitioner's
transfer to the new department does not support his claim of
constructive dismissal. The employee bears the burden to prove by
substantial evidence the fact of his dismissal from employment. Absent
any showing of an overt or positive act proving that the employer had

988
Page 989

dismissed the employee, the latter's claim of illegal dismissal cannot


be sustained as it would be self-serving, conjectural, and of no
probative value.

989
Page 990

Case Digests by: CALVO, KRISKA ANGELA A.

LBC EXPRESS-VIS, INC. vs. MONICA C. PALCO


G.R. No. 217101. February 12, 2020
LEONEN, J.

DOCTRINE:
CONSTRUCTIVE DISMISSAL – SEXUAL HARRASMENT
Constructive dismissal occurs when an employer makes an
employee's continued employment impossible, unreasonable or
unlikely, or has made an employee's working conditions or
environment harsh, hostile and unfavorable, such that the employee
feels obliged to resign from his or her employment.

An employee is considered constructively dismissed if he or she


was sexually harassed by her superior and her employer failed to act
on his or her complaint with prompt and sensitivity.

FACTS:
Respondent Palco is an employee of petitioner LBC Danao of
which Batucan is the Branch's Team Leader and OIC. While employed
at LBC, Palco had initially noticed that Batucan would often flirt with
her, which made her uncomfortable. The latter started sexually
harassing her; the last straw happened when Batucan attempted to
kiss her and eventually succeeded when he tried for a 2nd time.

Due to the incident, Palco excused herself from work. On May 5,


2010, she reported the same to LBC Head Office who in turn advised
her to transfer to another team while the matter was being investigated.
On May 14, 2010, Palco resigned as the management failed to act
promptly on the issue. She was forced to quit since she no longer felt
safe at work. It wasn't until June 15, 2010 that Batucan was served
with a Notice to Explain; administrative hearing for the incident was
held only on July 27, 2010, the same day that Pace filed a complaint
for illegal dismissal. Batucan received a suspension letter on
September 27, 2010.

The LA ruled in favor of Palco; this was affirmed by the NLRC and
CA on appeal. LBC contended that a period of 4 months do not
constitute an unreasonable period to resolve a sexual harassment
case. It also argued that it should not be held liable for constructive
dismissal since it did not commit any act of discrimination, insensibility,
or disdain towards respondent. Neither did it establish a harsh, hostile
or unfavorable work environment for her. On the other hand,
respondent maintains that she was constructively dismissed. She
avers that Batucan's acts towards her "created a hostile, intimidating

990
Page 991

and offensive environment, rendering her continued employment in the


company impossible, unreasonable or unlikely. She also contends that
petitioner was insensible and acted in bad faith in failing to immediately
act on her complaint; management took 4 months and 3 weeks to
resolve the matter and only when a constructive dismissal case has
been filed.

ISSUE:
Whether or not LBC should be held liable for constructive
dismissal.

RULING:
Affirmative. Constructive dismissal occurs when an employer
makes an employee's continued employment impossible,
unreasonable or unlikely, or has made an employee's working
conditions or environment harsh, hostile and unfavorable, such that the
employee feels obliged to resign from his or her employment. It does
not always involve forthright dismissal or diminution in rank,
compensation, benefit and privileges. There may be constructive
dismissal if an act of clear discrimination, insensibility, or disdain by an
employer becomes so unbearable on the part of the employee that it
could foreclose any choice by him except to forego his continued
employment. The gauge to determine whether there is constructive
dismissal is whether a reasonable person would feel constrained to
resign from his or her employment because of the circumstances,
conditions, and environment created by the employer for the
employee.

One of the ways by which a hostile or offensive work environment


is created is through the sexual harassment of an employee.
Workplace sexual harassment occurs when a supervisor, or agent of
an employer, or any other person who has authority over another in a
work environment, imposes sexual favors on another, which creates in
an intimidating, hostile, or offensive environment for the latter.

In this case, it is clear that Batucan's acts were sexually


suggestive. He held respondent's hand, put his hand on her lap and
shoulder, pulled her bra strap, joked about making a baby with her,
attempted to kiss her, and eventually scored one. These acts are not
only inappropriate, but are offensive and invasive enough to result in
an unsafe work environment for respondent. Petitioner's argument,
that it was Batucan who created the hostile work environment and not
the company itself, does not hold water. While it is true that Batucan,
acting in his personal capacity, cannot be deemed to have acted on
petitioner's behalf in committing the acts of sexual harassment hence
his illegal acts cannot be deemed authorized or sanctioned by the

991
Page 992

company, LBC's failure to take action reinforced the hostile


environment created by Batucan.

It is to be emphasized that the illegal acts should have been


committed by the employer against the employee. Unlawful acts
committed by a co-employee will not bring the matter within the ambit
of constructive dismissal. This is consistent with the established rule in
labor law that the complainant must first establish the employer-
employee relationship to be able to claim that he or she was illegally
dismissed. The distinction between the employer and an erring
managerial officer is likewise present in sexual harassment cases.
Under Section 5 of the Anti-Sexual Harassment Act, the employer is
only solidarity liable for damages with the perpetrator in case an act of
sexual harassment was reported and it did not take immediate action
on the matter. This provision thus illustrates that the employer must
first be informed of the acts of the erring managerial officer before it
can be held liable for the latter's acts. Conversely, if the employer has
been informed of the acts of its managerial staff, and does not contest
or question it, it is deemed to have authorized or be complicit to the
acts of its erring employee.

Petitioner's insensibility to respondent's sexual harassment case


is a ground for constructive dismissal. In this instance, it cannot be
denied that respondent was compelled to leave her employment
because of the hostile and offensive work environment created and
reinforced by Batucan and petitioner. She was thus clearly
constructively dismissed.

992
Page 993

Case Digests by: CALVO, KRISKA ANGELA A.

SKM ART CRAFT CORPORATION vs. Efren Bauca


G.R. No. 171282. November 27, 2013
Villarama, Jr. J.

DOCTRINE:
ARTICLE 286 – BONAFIDE SUSPENSION OF BUSINESS
OPERATIONS
Under Article 286 of the Labor Code, the bona fide suspension of
the operation of a business or undertaking for a period not exceeding
six months shall not terminate employment. If the employee was
forced to remain without work or assignment for a period exceeding six
months, then he is in effect constructively dismissed.

FACTS:
Petitioner SKM is engaged in the handicraft business. As a result
of the fire that gutted its inspection and receiving area and to prevent
further losses, SKM had to temporarily suspend its operations hence
respondents, who are employees of the said company, received a
notification on the same. Eight (8) days after receipt of the notification,
the respondents filed a case for illegal dismissal which petitioner
contested on the ground that under Article 286 of the Labor Code, bona
fide suspension of a business or undertaking for a period not
exceeding six months, is allowed.

The Labor Arbiter ruled that respondents were illegally dismissed


since the suspension exceeded the 6-months period allowed by the
law. This decision was reinstated by the CA after the NLRC reversed
the same for being premature, having been filed within the 6-month
period under Article 286.

ISSUE:
Whether or not the respondents were illegally dismissed.

RULING:
Affirmative. While it is true that the complaint was originally
prematurely filed (8 days after notification was received regarding
temporary suspension of operations), at the time the case was
decided, respondents were already considered illegally dismissed
since petitioner failed to recall them after six months. Under Article 286
of the Labor Code, the bona fide suspension of the operation of a
business or undertaking for a period not exceeding six months shall
not terminate employment. Consequently, when the bona fide
suspension of the operation of a business or undertaking exceeds six

993
Page 994

months, then the employment of the employee shall be deemed


terminated. By the same token and applying said rule by analogy, if
the employee was forced to remain without work or assignment for a
period exceeding six months, then he is in effect constructively
dismissed.

994
Page 995

Case Digests by: CALVO, KRISKA ANGELA A.

EMERITUS SECURITY AND MAINTENANCE SYSTEMS,


INC. vs. JANRIE C. DAILIG
G.R. No. 204761. April 2, 2014
CARPIO, J.

DOCTRINE:
FLOATING STATUS
The temporary inactivity or "floating status" of security guards
should continue only for six months, otherwise, the security agency
concerned could be liable for constructive dismissal.

FACTS:
Petitioner ESMSI hired respondent Dailig as one of its security
guards. During his employment, respondent was assigned to
petitioner's various clients however, on December 10, 2005, he was
relieved from his post. Dalig went to ESMSI's office to follow-up his
next assignment but more than 6 months has already passed and he
is still on floating status. This served as a ground for respondent to file
a complaint for illegal dismissal and payment for separation pay before
the NLRC. Petitioner denied the allegations and averred that it asked
respondent to report to the head office but the latter failed to comply.
This shows that respondent is no longer interested to continue his
employment.

The LA, NLRC and CA affirmed the finding of illegal dismissal


against ESMSI.

ISSUE:
Whether or not respondent was illegally dismissed.

RULING:
Affirmative. The temporary inactivity or "floating status" of
security guards should continue only for six months, otherwise, the
security agency concerned could be liable for constructive dismissal.
Petitioner admits relieving respondent from his post as security guard
on December 10, 2005 after which he remained on floating status at
the time he filed his complaint for illegal dismissal on June 16, 2006. In
other words, respondent was on floating status for more than six
months. Petitioner’s allegation of sending respondent a notice
sometime in January 2006, requiring him to report for work, is
unsubstantiated, and thus, self-serving. The failure of petitioner to give
respondent a work assignment beyond the reasonable six-month
period makes it liable for constructive dismissal.

995
Page 996

Case Digests by: CALVO, KRISKA ANGELA A.

NIPPON HOUSING PHIL. INC. vs. MAIAH ANGELA


LEYNES
G.R. No. 177816. August 3, 2011
PEREZ, J.

DOCTRINE:
ARTICLE 286 – FLOATING STATUS / OFF-DETAILING
Traditionally invoked by security agencies when guards are
temporarily sidelined from duty while waiting to be transferred or
assigned to a new post or client, Article 286 has been applied to other
industries when, as a consequence of the bona fide suspension of the
operation of a business or undertaking, an employer is constrained to
put employees on floating status for a period not exceeding six months.
"Off-detailing" is not equivalent to dismissal, so long as such status
does not continue beyond a reasonable time and that it is only when
such a "floating status" lasts for more than six months that the
employee may be considered to have been constructively dismissed.
A complaint for illegal dismissal filed prior to the lapse of said six-month
and/or the actual dismissal of the employee is generally considered as
prematurely filed.

FACTS:
Petitioner Nippon Housing (NHPI) recently ventured into the
business of building management and has a lone client - BGCC. It
hired respondent Leynes for the position of Property Manager.
Following Leynes’ misunderstanding with Engr. Cantuba, the Building
Engineer assigned at the project, NHPI Vice President Hiroshi Takada
issued a memorandum attributing the incident to "simple personal
differences" and directing Leynes to allow Engr. Cantuba to report back
for work (Leynes previously denied the latter entry to the project). In
view of the said decision, Leynes submitted a letter asking for an
emergency leave of absence for the purpose of coordinating with her
lawyer regarding her resignation letter. She later called of her planned
resignation and expressed her intention to return to work.

Considering the sensitive nature of Leynes’ position and the


critical stage of the Project’s business development, NHPI was
constrained to relay the situation to BGCC who, in turn, requested the
immediate appointment of a new Property Manager. As a result,
Petitioner offered the position to Engr. Jose and served Leynes with a
letter and memorandum relieving her from her position and directing
her to report to NHPI’s main office while she was on floating status.
Respondent then filed a complaint for illegal dismissal against NHPI
and its Officers. During the pendency of the case, however, Reyes

996
Page 997

(NHPI HR Head) eventually served DOLE and Leynes with a notice


terminating her services on the ground of redundancy or lack of a
posting commensurate to her position at the Project. The LA ruled in
favor of Leynes stating that her floating status was equivalent to
termination from employment without just cause and compliance with
the twin requirements of notice and hearing. The NLRC reversed the
said decision. Eventually, the CA found NHPI liable for illegal
dismissal.

ISSUE:
Whether or not NHPI's decision to put Leynes on floating status is
tantamount to constructive dismissal.

RULING:
Negative. NHPI validly placed Leynes on floating status pursuant
to Article 286 of the Labor Code. Traditionally invoked by security
agencies when guards are temporarily sidelined from duty while
waiting to be transferred or assigned to a new post or client, the said
provision has been applied to other industries when, as a consequence
of the bona fide suspension of the operation of a business or
undertaking, an employer is constrained to put employees on floating
status for a period not exceeding six months.

In this case, Leynes had been signifying her intention to resign.


Since her position is critical, and, as requested by BGCC, the client of
NHPI who has a contractually guaranteed right to ask for her relief,
NHPI exercised its management prerogative to put Leynes on floating
status until such time that another project could be secured. When
Engr. Jose filled in the position, there was no other Property Manager
position available to Leynes since NHPI only had one client. Petitioner
was then constrained to terminate Leynes. The immediate hiring of
Engr. Jose was brought about by Leynes’ own rash announcement of
her intention to resign from her position. She evidently had only herself
to blame for precipitately setting in motion the events which led to
NHPI’s hiring of her own replacement.

Considering that labor laws discourage intrusion in the employers’


judgment concerning the conduct of their business, courts often
decline to interfere in their legitimate business decisions, absent
showing of illegality, bad faith or arbitrariness. The right of employees
to security of tenure does not give them vested rights to their positions
to the extent of depriving management of its prerogative to change
their assignments or to transfer them. The record shows that Leynes
filed the complaint for actual illegal dismissal from which the case
originated immediately upon being placed on floating status as a
consequence of NHPI’s hiring of a new Property Manager for the
Project. The rule is settled, however, that "off-detailing" is not

997
Page 998

equivalent to dismissal, so long as such status does not continue


beyond a reasonable time and that it is only when such a "floating
status" lasts for more than six months that the employee may be
considered to have been constructively dismissed. A complaint for
illegal dismissal filed prior to the lapse of said six-month and/or the
actual dismissal of the employee is generally considered as
prematurely filed.

NHPI was acting well within its prerogatives when it eventually


terminated Leynes’ services on the ground of redundancy. One of the
recognized authorized causes for the termination of employment,
redundancy exists when the service capability of the workforce is in
excess of what is reasonably needed to meet the demands of the
business enterprise. It has been held that the exercise of business
judgment to characterize an employee’s service as no longer
necessary or sustainable is not subject to discretionary review where,
as here, it is exercised there is no showing of violation of the law or
arbitrariness or malice on the part of the employer. Considering that
Leynes was terminated from service upon an authorized cause, there
is no illegal or constructive dismissal to speak of. However, NHPI
should be held liable to pay nominal damages in the sum of ₱50,000
for failing to comply with the 30-day minimum requirement for the
required notice of termination thereby effectively violating Leynes’ right
to due process.

998
Page 999

Case Digests by: CALVO, KRISKA ANGELA A.

MAYON HOTEL & RESTAURANT vs. ROLANDO ADANA,


ET AL.
G.R. No. 157634. May 16, 2005
PUNO, J.

DOCTRINE:
INSERT THE TOPIC HERE IN BOLD, CAPITAL LETTER.
Add the doctrine here.

FACTS:
Petitioner Mayon is a hotel and restaurant business which
employs about 16 employees. Due to the expiration and non-renewal
of the lease contract for the rented space occupied by the petitioner,
the hotel operations of the business were suspended on March 31,
1997. The operation of the restaurant was fortunately continued at a
new location while waiting for the construction of the new Mayon Hotel
& Restaurant. Only 9 out of its 16 employees continued working in the
new site.

On various dates of April and May 1997, several employees filed


complaints for illegal dismissal against petitioners. According to the
complainants they were not permitted to work nor were recalled even
after the construction of the new site was finished. The LA rendered a
decision in favor of the employees but the same was reversed by the
NLRC who dismissed all claims. The CA then reversed the NLRC
ruling.

ISSUE:

Whether or not the respondents were illegally dismissed.

RULING:
Affirmative. Article 286 of the Labor Code is clear — there is
termination of employment when an otherwise bona fide suspension of
work exceeds 6 months; employer has the burden of proving that the
termination was for a just or authorized cause. While the closure of the
operations of the petitioner may have been temporary, the lay-off of
respondents was not. What is clear from the records is that since April
1997, when petitioner Mayon Hotel & Restaurant suspended its hotel
operations and transferred its restaurant operations in Elizondo Street,
respondents have not been permitted to work for petitioners. Nor have
they been recalled, even after the construction of the new premises at
Peñaranda Street and the reopening of the hotel operations with the
restaurant in this new site. A total of 1 year has lapsed since the

999
Page 1000

respondents were laid-off by petitioner. The temporary cessation of


employment became a dismissal by operation of law when petitioners
failed to reinstate respondents after the lapse of 6 months, pursuant to
Article 286 of the Labor Code.

1000
Page 1001

Case Digests by: CALVO, KRISKA ANGELA A.

ICT MARKETING SERVICES, INC. vs. MARIPHIL L.


SALES
G.R. No. 202090. September 9, 2015
DEL CASTILLO, J.

DOCTRINE:
MANAGEMENT PREROGATIVE – TRANSFER OF EMPLOYEE
Under the doctrine of management prerogative, every employer
has the inherent right to regulate, according to his own discretion and
judgment, all aspects of employment, including hiring, work
assignments, working methods, the time, place and manner of work,
work supervision, transfer of employees, lay-off of workers, and
discipline, dismissal, and recall of employees. The only limitations to
the exercise of this prerogative are those imposed by labor laws and
the principles of equity and substantial justice.

FACTS:
Petitioner ICT Marketing (now Sykes) is engaged in the BPO
industry. It hired respondent Mariphil L. Sales as its CSR and assigned
her to its Capital One account. She was later assigned to the
Washington Mutual account, where she was awarded with a certificate
for being the "Top Converter/Seller" for the month of April 2007. Sales
wrote to ICT's Vice-President Glen Odom complaining about supposed
irregularities in the handling of funds entrusted to petitioner by
Washington Mutual which were intended for distribution to outstanding
Washington Mutual CSRs and TSRs as prizes and incentives.
However, no action appears to have been taken on her complaint.
Following the said complaint, Sales was transferred to the Bank of
America account however she failed to be certified to take calls as she
was not able to complete the training. As explained by respondent, her
absence was brought about by her being sick hence needing to
undergo a medical check-up on the said date. From then on,
respondent was placed on "floating status" and was not given any work
assignment constraining her to tender her resignation and file a
complaint for constructive dismissal.

Petitioner ICT denied the allegations and contended respondent’s


transfer to another account was done as a valid exercise of
management prerogative, which allows it to regulate all aspects of
employment. Sales' transfer was done in good faith, and without
diminution in rank and salary. The LA ruled in favor of Sales. The said
decision was reinstated by the CA after it was reversed by the NLRC.

ISSUE:

1001
Page 1002

Whether or not Sales was constructively dismissed.

RULING:
Affirmative. Under the doctrine of management prerogative, every
employer has the inherent right to regulate, according to his own
discretion and judgment, all aspects of employment, including hiring,
work assignments, working methods, the time, place and manner of
work, work supervision, transfer of employees, lay-off of workers, and
discipline, dismissal, and recall of employees. The only limitations to
the exercise of this prerogative are those imposed by labor laws and
the principles of equity and substantial justice. Concerning the transfer
of employees, these are the following jurisprudential guidelines: (a) a
transfer is a movement from one position to another of equivalent rank,
level or salary without break in the service or a lateral movement from
one position to another of equivalent rank or salary; (b) the employer
has the inherent right to transfer or reassign an employee for legitimate
business purposes; (c) a transfer becomes unlawful where it is
motivated by discrimination or bad faith or is effected as a form of
punishment or is a demotion without sufficient cause; (d) the employer
must be able to show that the transfer is not unreasonable,
inconvenient, or prejudicial to the employee.

In this case, while the prerogative to transfer respondent to


another account belonged to petitioner, it wielded the same unfairly.
Based on evidence presented, at the time the respondent was
transferred to the Bank of America program, petitioner was hiring
additional CSRs/TSRs. This simply means that if it was then hiring new
CSRs/TSRs, then there should be no need to transfer respondent to
the Bank of America program; it could simply train new hires for that
program. Moreover, even if respondent had attendance and
punctuality issues, her overall performance as a CSR/TSR was
certainly far from mediocre; on the contrary, she proved to be a top
performer. And if it were true that respondent suddenly became lax by
way of attendance in July 2007, it is not entirely her fault. This may be
attributed to petitioner’s failure to properly address her grievances
relative to the supposed irregularities in the handling of funds entrusted
to petitioner by Washington Mutual which were intended for distribution
to outstanding Washington Mutual CSRs and TSRs as prizes and
incentives.

The only conceivable reason why petitioner transferred


respondent to another account is the fact that she openly and bravely
complained about the supposed anomalies in the Washington Mutual
account. Thus, in causing respondent’s transfer, petitioner clearly
acted in bad faith and with discrimination, insensibility and disdain; the
transfer was effected as a form of punishment for her raising a valid
grievance related to her work. This being the case, the transfer
amounted to constructive dismissal.

1002
Page 1003

As to the floating status respondent Sales, in placing her in such


disposition, petitioner further acted arbitrarily and unfairly, making life
unbearable for her. In so doing, it treated respondent as if she were a
new hire; it improperly disregarded her experience, status,
performance, and achievements in the company; and most
importantly, respondent was illegally deprived of her salary and other
emoluments for her single absence during training. In effect,
respondent’s transfer to the Bank of America account was not only
unreasonable, unfair, inconvenient, and prejudicial to her; it was
effectively a demotion in rank and diminution of her salaries, privileges
and other benefits.

1003
Page 1004

Case Digests by: CALVO, KRISKA ANGELA A.

GERARDO A. CARIQUE vs. PHILIPPINE SCOUT


VETERANS SECURITY and INVESTIGATION AGENCY,
INC.
G.R. No. 197484. September 16, 2015
DEL CASTILLO, J.

DOCTRINE:
FLOATING STATUS AS GROUND FOR ILLEGAL DISMISSAL -
PROOF
It is incumbent upon the employee to prove by substantial
evidence the fact that he was indeed illegally dismissed from
employment. Illegal dismissal must be established by positive and
overt acts clearly indicative of a manifest intention to dismiss. This
critical affirmative fact must be proved by the party alleging the same
with substantial evidence as required by the nature of this case. Mere
allegation is neither proof nor evidence.

FACTS:
Petitioner Carique was hired as a security guard by respondent
agency PSVSIA. He was thereafter assigned to respondent agency’s
several clients, the last of which was at National Bookstore - Rosario,
Pasig Branch. On October 28, 2002, petitioner was relieved from his
post and was replaced by another guard pursuant to a rotation policy
being implemented by PSVSIA. On May 6, 2003, petitioner filed an
illegal dismissal case alleging that petitioner refused to furnish him with
new assignment. Respondent denied having dismissed petitioner, let
alone illegally, and alleged that petitioner was relieved from his post
because of a rotation policy being implemented as required by
respondent agency’s clients. In fact, petitioner was offered
assignments twice, the 1st at NBS Bicutan, 5 months after his relief,
and the 2nd at CS Buendia; he refused both.

The LA ruled in favor of petitioner but the same was reversed by


the NLRC, which reversal was affirmed by the CA on appeal.

ISSUE:
Whether or not Carique was illegally dismissed when he was
placed on floating status by PSVSIA.

RULING:
Negative. It is incumbent upon the employee to prove by
substantial evidence the fact that he was indeed illegally dismissed
from employment. Illegal dismissal must be established by positive and
overt acts clearly indicative of a manifest intention to dismiss. This

1004
Page 1005

critical affirmative fact must be proved by the party alleging the same
with substantial evidence as required by the nature of this case. Mere
allegation is neither proof nor evidence. Petitioner anchored his claims
on unfounded and unproven allegations. No positive or direct evidence
was adduced to show that he was indeed illegally dismissed from
employment, either factually or constructively. If anything, the evidence
on record showed that petitioner was relieved from his last assignment
because of the implementation of a rotation policy by respondent
agency which was requested by its clients; and that as correctly found
by the CA, petitioner, from that point on, was considered on floating
status or on temporary off-detail which is not an unusual occurrence
for security guards given that their assignments primarily depend on
the contracts entered into by the agency with third parties. Placing
petitioner on floating or off-detail status for not more than six months is
not prohibited by law and did not amount to dismissal.

Petitioner’s insistence that he was not given any new assignment


after his relief was not corroborated by any evidence. Significantly,
both the NLRC and the CA noted that petitioner never denied or
disputed having received copies of the SSDs directing him to report to
his new assignments. If at all, he simply shrugged off the SSDs,
claiming that these SSDs were fabricated and contained inaccurate
and falsified entries. Confronted with these conflicting claims, this
Court finds no difficulty in upholding the claims of the duty officer and
the investigator general because these claims square with the facts on
record.

1005
Page 1006

Case Digests by: CALVO, KRISKA ANGELA A.

JEROME M. DAABAY vs. COCA-COLA BOTTLERS PHILS., INC.


G.R. No. 199890. August 19, 2013
REYES, J.

DOCTRINE:
GRANT OF RETIREMENT BENEFITS FOR VALIDY TERMINATED
EMPLOYEE
Termination of employment for a just cause does not warrant the
application of Article 287 of the Labor Code nor can the same be made
to operate for the benefit of respondent.

FACTS:
Petitioner Daabay was employed by respondent Coca-Cola as a
Sales Logistics Checker. Following the receipt of information that
petitioner was part of a conspiracy that allowed the pilferage of
company property, Coca-Cola launched an investigation on the matter
which eventually resulted in the termination of Daabay in June 2005.
The latter then filed a case for illegal dismissal. The LA ruled in favor
of petitioner which was reversed by the NLRC. Notwithstanding its
ruling on the legality of the dismissal, the latter awarded retirement
benefits in favor of Daabay. On appeal by Coca-Cola, the CA deleted
the award for retirement benefits for lack of basis considering that
Daabay was dismissed for just cause.

ISSUE:
Whether or not petitioner, who was validly terminated for a just
cause, should be granted retirement benefits.

RULING:
Negative. Daabay was declared by the NLRC to have been
lawfully dismissed by Coca-Cola on the grounds of serious
misconduct, breach of trust and loss of confidence. As held in PAL vs.
NLRC, termination of employment for a just cause does not warrant
the application of Article 287 of the Labor Code nor can the same be
made to operate for the benefit of respondent. Even if respondent was
already qualified for retirement at the time of dismissal, this does not
aid his case because the fact remains that respondent was already
terminated for just cause thereby rendering nugatory any entitlement
to mandatory or optional retirement pay that he might have previously
possessed.

NLRC's justification for the award of retirement benefits, which is


the need to humanize the severe effects of dismissal and tilt the scales
of justice in favor of labor as a measure of equity and compassionate
social justice, is untenable. Being intended as a mere measure of

1006
Page 1007

equity and social justice, the NLRC’s award was then akin to a financial
assistance or separation pay that is granted to a dismissed employee
notwithstanding the legality of his dismissal. As a measure of social
justice, such award is allowed only in instances where the employee is
validly dismissed for causes other than serious misconduct or those
reflecting on his moral character. A contrary rule would have the effect,
of rewarding rather than punishing the erring employee for his offense.

1007
Page 1008

Case Digests by: CALVO, KRISKA ANGELA A.

ROSENDO PIÑERO VS. NLRC


G.R. No. 149610, August 20, 2004
Ynares-Santiago, J.

DOCTRINE:
ENTITLEMENT TO RETIRMENT BENEFITS AFTER VALID
DISMISSAL FROM EMPLOYMENT
An employee who is dismissed for cause is generally not entitled
to any financial assistance. Equity considerations, however, provide an
exception, such as in this case where petitioner has no derogatory
records and has served the company for 29 years.

FACTS:
Private Respondent, Dumaguete Cathedral College, Inc. is an
educational institution and is the employer of the faculty and staff
members comprising the labor union DUCACOFSA-NAFTEU. Upon
the expiration of the implemented CBA, the parties failed to conclude
another CBA which led DUCACOFSA-NAFTEU to file a notice of strike
with the DOLE on the ground of refusal to bargain. DUCACOFSA-
NAFTEU conducted a strike in the premises of private respondent
without submitting to the DOLE the required results of the strike vote
obtained from the members of the union. Consequently, on November
21, 1991, private respondent filed with the DOLE a complaint to
declare the strike illegal and to dismiss the officers of DUCACOFSA-
NAFTEU.

The LA declared the strike illegal; consequently, respondent union


officers lost their employment status effective on the date of the said
decision. This was affirmed by the NLRC who, in addition to the failure
to comply with strike vote requirements, also ruled that the strike was
illegal because DUCACOFSA-NAFTEU is not a legitimate labor
organization therefore it has no personality to hold a strike. Upon
appeal, the CA dismissed the case. The SC ruled that the strike held
by the Union was indeed illegal for non-compliance with the procedural
requirements of Article 263 of the Labor Code. Piñero, who was the
then Union President, was dismissed from service. However, during
the pendency of the case, the latter turned 60 years old and retired on
March 1, 1996 after 29 years of service, rendering his dismissal moot
and academic.

ISSUE:
Whether or not Piñero is entitled to retirement benefits.

1008
Page 1009

RULING:
Affirmative. An employee who is dismissed for cause is generally
not entitled to any financial assistance. Equity considerations,
however, provide an exception. Equity has been defined as justice
outside law, being ethical rather than jural and belonging to the sphere
of morals than of law. It is grounded on the precepts of conscience and
not on any sanction of positive law, for equity finds no room for
application where there is law. Although meriting termination of
employment, Piñero's infraction is not so reprehensible nor
unscrupulous as to warrant complete disregard of his long years of
service. Moreover, he has no previous derogatory records. Weighed
on the scales of justice, conscience and reason tip in favor of granting
financial assistance to support him in the twilight of his life after long
years of service.

Under the circumstances, social and compassionate justice


dictate that petitioner Piñero be awarded financial assistance
equivalent to 1/2 month's pay for every year of service computed from
his date of employment up to October 28, 1994 when he was declared
to have lost his employment status. Indeed, equities of this case should
be accorded due weight because labor law determinations are not only
secundum rationem but also secundum caritatem.

1009
Page 1010

Case Digests by: CALVO, KRISKA ANGELA A.

STA. CATALINA COLLEGE vs. NLRC


G.R. No. 144483, November 19, 2003
Carpio-Morales, J.

DOCTRINE:
GRATUITY PAY SHOULD NOT BE DEDUCTED FROM
RETIREMENT BENEFITS.
Gratuity pay is separate and distinct from retirement benefits. It is
paid purely out of generosity and is not intended to pay a worker for
actual services rendered or for actual performance. It is a money
benefit or bounty given to the worker, the purpose of which is to reward
employees who have rendered satisfactory service to the company.

FACTS:
Private Respondent, Hilaria Tercero, was hired as an elementary
school teacher at the petitioner school in June 1955. In 1970, she
applied for a one-year leave of absence without pay on account of her
mother’s illness. However, after the expiration of her leave of absence,
she had not been heard from by the petitioner school again. In the
meantime, she was hired as a teacher at the San Pedro Parochial
School from 1980-1981 and at the Liceo de San Pedro from 1981-
1982. In 1982, she applied anew at the petitioner school which hired
her. In March 1997, she reached the compulsory retirement age of 65.
Petitioner school asserted that since she abandoned her employment
in 1970, thus, her retirements benefits were computed on the basis of
her fifteen (15) years of services from 1982-1997 and excluded her
service from 1955-1970. From her retirement benefit, the amount
representing reimbursement of the employer’s contribution to her
retirement benefits under the Private Education Retirement Annuity
Association (PERAA) which the private respondent already received.
The gratuity pay which she had also received was likewise deducted.

Private respondent insisted that her retirement benefits should be


on the basis of her thirty (30) years of service from 1955-1997, and that
the gratuity pay that she received should not be deducted. The parties
having failed to agree on how the retirement benefits should be
computed, private respondent filed a complaint before the NLRC
Regional Arbitration against petitioner school for non-payment of
retirement benefits. The LA ruled in favor of the petitioner school, which
was set aside by the NLRC. Petitioner school brought the case on
certiorari to the CA which dismissed the petition holding that petitioner
school failed to prove that private respondent abandoned her
employment in 1970 since it gave her a Plaque of Appreciation for 30
years of service. Furthermore, petitioner school never sent a notice of
dismissal thus it never severed the employer-employee relationship.

1010
Page 1011

ISSUE:
Whether or not the private respondent’s service from 1955-1970
should be included in the computation of her retirement benefits and
whether or not the gratuity pay awarded to the respondent should be
deducted from her retirement benefits.

RULING:
Negative. The Supreme Court ruled that the private respondent
can only be awarded with what she is rightfully entitled to under the
law. After her one year leave of absence expired in 1971, without her
requesting for extension thereof as in fact she had not been heard from
until she reapplied to the petitioner school in 1982, she has been found
to have abandoned her teaching position. Furthermore, she was even
employed in other schools from 1980-1982. Thus, private respondent
cannot be credited for her service in 1955-1970 in the determination of
her retirement benefits. It is error to conclude that since petitioner
school did not award separation pay and Hilaria’s share of her
retirement contributions when she "temporarily" stopped working after
she left her teaching position in 1971, employer-employee relation
between them was not severed. It bears noting that an employee who
is terminated for just cause is generally not entitled to separation pay.
Moreover, the PERAA, petitioner school’s substitute retirement plan,
was only established in 1972, such that when Hilaria abandoned her
work in 1971, there were no retirement contributions to speak of.

As Hilaria was considered a new employee when she rejoined


petitioner school upon re-applying in 1982, her retirement benefits
should thus be computed only on the basis of her years of service from
1982 to 1997. Anent the ruling of the CA affirming that the gratuity pay
earlier awarded to Hilaria should not be deducted from the retirement
benefits due her, the same is in order. Gratuity pay is separate and
distinct from retirement benefits. It is paid purely out of generosity and
is not intended to pay a worker for actual services rendered or for
actual performance. It is a money benefit or bounty given to the worker,
the purpose of which is to reward employees who have rendered
satisfactory service to the company. Retirement benefits, on the other
hand, are intended to help the employee enjoy the remaining years of
his life, releasing him from the burden of worrying for his financial
support, and are a form of reward for his loyalty to the employer.

1011
Page 1012

Case Digests by: CALVO, KRISKA ANGELA A.

PANTRANCO NORTH EXPRESS, INC. vs. NLRC


G.R. No. 114333. January 24, 1996
PUNO, J.

DOCTRINE:
ILLNESS AS A GROUND FOR RETIREMENT
To affirm the legality of retirement due to illness, the grounds
provided under the IRR of the Labor Code should be followed.
Dismissal is the ultimate penalty that can be meted to an employee. It
must, therefore, be based on a clear and not on an ambiguous or
ambivalent ground. Any ambiguity or ambivalence on the ground relied
upon by an employer in terminating the services of an employee denies
the latter his full right to contest its legality.

FACTS:
Private respondent Reynaldo Rueda was employed as a bus
conductor by petitioner Pantranco; he was later promoted to the
position of Line Inspector. When petitioner suffered financial setbacks,
it retrenched some of its employees, including respondent, who it later
re-hired. During this time, respondent got involved in a quarrel with a
co-employee whom he stabbed resulting in criminal and administrative
complaints being lodged against him. Instead of dismissing Rueda,
Pantranco put him on forced retirement on the ground of medical
reasons. The respondent's retirement benefits were computed from
the date of his re-employment instead of the date of his original starting
date, which was contested by Rueda. The latter also appealed his
forced retirement and when his plea was unheard, he filed a complaint
for illegal dismissal. The LA ruled in favor of Rueda but the said
decision was reversed by the NLRC.

ISSUE:
Whether or not the Rueda’s retirement due to illness is proper.

RULING:
Negative. The Labor Code, in addition to Article 282 therein,
considers illness and retrenchment to prevent losses as valid grounds
for termination of employment, subject to the conditions specified
therein. Petitioner contends that the actual ground upon which it
anchors its right to terminate the employment of Rueda is that of
serious misconduct due to the previous stabbing incident. The facts
show, however, that petitioner abandoned serious misconduct as a
ground to dismiss respondent when it opted to retire him due to illness.
In order to affirm the legality of Rueda's retirement due to the

1012
Page 1013

aforementioned cause, the grounds provided under the IRR of the


Labor Code should be followed.

In this case, petitioner did not submit the required certification by


a competent public health authority to show that Rueda's illness could
not be cured within the period specified under the aforequoted rule.
The radiograph report submitted by petitioner merely stated Rueda's
disease. In the absence of such certification, Rueda's retirement due
to illness has no leg to stand on. Dismissal is the ultimate penalty that
can be meted to an employee. It must, therefore, be based on a clear
and not on an ambiguous or ambivalent ground. Any ambiguity or
ambivalence on the ground relied upon by an employer in terminating
the services of an employee denies the latter his full right to contest its
legality.

1013
Page 1014

Case Digests by: CALVO, KRISKA ANGELA A.

R & E TRANSPORT, INC. vs. AVELINA P. LATAG


G.R. No. 155214. February 13, 2004
PANGANIBAN, J.

DOCTRINE:
RETIREMENT – BOUNDARY FEE
It is accepted that taxi drivers do not receive fixed wages, but
retain only those sums in excess of the boundary or fee they pay to the
owners or operators of their vehicles. Thus, the basis for computing
their benefits should be the average daily income.

FACTS:
Pedro Latag was a regular employee of La Mallorca Taxi. When
La Mallorca ceased from business operations, Latag transferred to
petitioner R & E Transport, Inc. He was receiving an average daily
salary of five hundred pesos P500.00. Latag got sick and was forced
to apply for partial disability with the SSS, which was granted. When
he recovered, he reported for work in September 1998 but was no
longer allowed to continue working on account of his old age. Latag
thus asked the administrative officer of petitioner for his retirement pay
but, he was ignored. Thus, on December 21, 1998, he filed a case for
payment of his retirement pay before the NLRC. Latag however died
and was subsequently substituted by his wife.

The LA ruled in favor of Latag. The wife was invited to the office
of petitioner’s counsel and was offered the amount of P38,500.00 she
accepted. She was also asked to sign an already prepared quitclaim
and release and a joint motion to dismiss the case. Petitioners filed the
quitclaim and motion to dismiss.

ISSUE:
Whether or not the quitclaim concerning the retirement benefits
received by Mrs. Latag was valid.

RULING:
Negative; the amount received was not proper. Latag was credited
with 14 years of service with R & E Transport, Inc. Article 287 of the
Labor Code on Retirement provides that in the absence of a retirement
plan or agreement providing for retirement benefits of employees in the
establishment, an employee upon reaching the age of 60 years or
more, but not beyond 65 years, which is hereby declared the
compulsory retirement age, who has served at least 5 years in said
establishment, may retire and shall be entitled to retirement pay
equivalent to at least 1/2 month salary for every year of service, a

1014
Page 1015

fraction of at least 6 months being considered as 1 whole year. Unless


the parties provide for broader inclusions, the term one half-month
salary shall mean 15 days plus 1/12 of the 13th month pay and the
cash equivalent of not more than 5 days of service incentive leaves.

The rules implementing the New Retirement Law similarly provide


the above-mentioned formula for computing the ½ month salary. Since
Pedro was paid according to the boundary system, he is not entitled to
the 13th month and the service incentive pay; hence, his retirement
pay should be computed on the sole basis of his salary. It is accepted
that taxi drivers do not receive fixed wages, but retain only those sums
in excess of the boundary or fee they pay to the owners or operators
of their vehicles. Thus, the basis for computing their benefits should be
the average daily income. In this case, the CA found that Pedro was
earning an average of P500 per day. The court thus computed his
retirement pay as follows: P500 x 15 days x 14 years of service equals
P105,000. Compared with this amount, the P38,850 he received,
which represented just 1/3 of what was legally due him, was
unconscionable.

1015
Page 1016

Case Digest by: Cerera, Florence Diane N.

SERRANO v SEVERINO SANTOS TRANSIT


G.R. No. 187698. 09 August 2010.
Carpio-Morales, J.

DOCTRINE:
SERVICE INCENTIVE LEAVE
For purposes of applying the law on SIL, as well as on retirement, the Court
notes that there is a difference between drivers paid under the boundary system and
conductors who are paid on commission basis.
In practice, taxi drivers do not receive fixed wages. They retain only those sums in
excess of the boundary or fee they pay to the owners or operators of the vehicles.
Conductors, on the other hand, are paid a certain percentage of the bus earnings for
the day.
It bears emphasis that under P.D. 851 or the SIL Law, the
exclusion from its coverage of workers who are paid on a purely
commission basis is only with respect to field personnel.

FACTS:
Rodolfo J. Serrano was hired on September 28, 1992 as bus conductor by
respondent Severino Santos Transit, a bus company owned and operated by its co-
respondent Severino Santos.

After 14 years of service or on July 14, 2006, petitioner applied for optional
retirement from the company whose representative advised him that he must first sign
the already prepared Quitclaim before his retirement pay could be released. As
petitioners request to first go over the computation of his retirement pay was denied,
he signed the Quitclaim on which he wrote U.P. (under protest) after his signature.

Petitioner soon after filed a complaint before the Labor Arbiter, alleging that the
company erred in its computation since under Republic Act No. 7641, otherwise known
as the Retirement Pay Law, his retirement pay should have been computed at 22.5
days per year of service to include the cash equivalent of the 5-day service incentive
leave (SIL) and 1/12 of the 13th month pay which the company did not.

The company maintained, however, that the Quitclaim signed by


petitioner barred his claim and, in any event, its computation was
correct since petitioner was not entitled to the 5-day SIL and pro-rated
13th month pay for, as a bus conductor, he was paid on commission
basis.

Labor Arbiter ruled in favor of petitioner, awarding him P116,135.45 as retirement


pay differential, and 10% of the total monetary award as attorneys fees.

1016
Page 1017

The NLRC reversed the LA’s ruling. NLRC held that since
petitioner was paid on purely commission basis, he was excluded from
the coverage of the laws on 13th month pay and SIL pay, hence, the
1/12 of the 13thmonth pay and the 5-day SIL should not be factored in
the computation of his retirement pay.

ISSUE:
Whether or not petitioner is entitled to the cash equivalent of the
SIL as part of his retirement pay.

RULING:
YES. Republic Act No. 7641 which was enacted on December 9, 1992
amended Article 287 of the Labor Code by providing for retirement pay to qualified
private sector employees in the absence of any retirement plan in the establishment.
The pertinent provision of said law reads:

Section 1. Article 287 of Presidential Decree No. 442, as amended,


otherwise known as the Labor Code of the Philippines, is hereby
amended to read as follows:
xxxx
In the absence of a retirement plan or agreement providing for
retirement benefits of employees in the establishment, an employee
upon reaching the age of sixty (60) years or more, but not beyond sixty-
five (65) years which is hereby declared the compulsory retirement age,
who has served at least five (5) years in the said establishment, may
retire and shall be entitled to retirement pay equivalent to at least one-
half (1/2) month salary for every year of service, a fraction of at least six
(6) months being considered as one whole year.
Unless the parties provide for broader inclusions, the term one-half (1/2)
month salary shall mean fifteen (15) days plus one-twelfth (1/12) of the
13th month pay and the cash equivalent of not more than five (5) days
of service incentive leaves.
Retail, service and agricultural establishments or operations employing
not more than (10) employees or workers are exempted from the
coverage of this provision.

Further, the Implementing Rules of said law provide:

SECTION 5
Retirement Benefits.
5.1 In the absence of an applicable agreement or retirement plan, an
employee who retires pursuant to the Act shall be entitled to retirement
pay equivalent to at least one-half (―) month salary for every year of
service, a fraction of at least six (6) months being considered as one
whole year.

1017
Page 1018

5.2 Components of One-half (―) Month Salary. For the purpose of


determining the minimum retirement pay due an employee under this
Rule, the term one-half month salary shall include all of the following:
(a) Fifteen (15) days salary of the employee based on his latest
salary rate. As used herein, the term salary includes all
remunerations paid by an employer to his employees for services
rendered during normal working days and hours, whether such
payments are fixed or ascertained on a time, task, piece of
commission basis, or other method of calculating the same, and
includes the fair and reasonable value, as determined by the Secretary
of Labor and Employment, of food, lodging or other facilities customarily
furnished by the employer to his employees.
(b) The cash equivalent of not more than five (5) days of service
incentive leave;
(c) One-twelfth of the 13th month pay due the employee.
(d) All other benefits that the employer and employee may agree upon.

Admittedly, petitioner worked for 14 years for the bus company which did not
adopt any retirement scheme. Even if petitioner as bus conductor was paid on
commission basis then, he falls within the coverage of R.A. 7641 and its implementing
rules. As thus correctly ruled by the Labor Arbiter, petitioners retirement pay should
include the cash equivalent of the 5-day SIL and 1/12 of the 13th month pay.

For purposes of applying the law on SIL, as well as on retirement, the Court
notes that there is a difference between drivers paid under the boundary system and
conductors who are paid on commission basis.

In practice, taxi drivers do not receive fixed wages. They retain only those sums
in excess of the boundary or fee they pay to the owners or operators of the vehicles.
Conductors, on the other hand, are paid a certain percentage of the bus earnings for
the day.

It bears emphasis that under P.D. 851 or the SIL Law, the exclusion from its
coverage of workers who are paid on a purely commission basis is only with respect
to field personnel.
According to Article 82 of the Labor Code, field personnel shall refer to non-
agricultural employees who regularly perform their duties away from the principal place
of business or branch office of the employer and whose actual hours of work in the
field cannot be determined with reasonable certainty.

As a general rule, [field personnel] are those whose performance


of their job/service is not supervised by the employer or his
representative, the workplace being away from the principal office and
whose hours and days of work cannot be determined with reasonable
certainty; hence, they are paid specific amount for rendering specific
service or performing specific work. If required to be at specific places
at specific times, employees including drivers cannot be said to be field

1018
Page 1019

personnel despite the fact that they are performing work away from the
principal office of the employee.

1019
Page 1020

Case Digest by: Cerera, Florence Diane N.

OBUSAN v PHILIPPINE NATIONAL BANK


G.R. No. 181178. July 26, 2010.
Nachura, J.

DOCTRINE:
RETIREMENT
Article 287 of the Labor Code, as amended, applies only to a
situation where (1) there is no CBA or other applicable employment
contract providing for retirement benefits for an employee; or (2) there
is a collective bargaining agreement or other applicable employment
contract providing for retirement benefits for an employee, but it is
below the requirement set by law.

FACTS:
Back in 1979, respondent Philippine National Bank (PNB) hired petitioner
Amelia R. Obusan, who eventually became the Manager of the PNB Medical Office.
At that time, PNB was a government-owned or controlled corporation, whose
retirement program for its employees was administered by the GSIS, pursuant to the
Revised Government Service Insurance Act of 1977 (PD No. 1146).

On May 27, 1996, PNB was privatized pursuant to the Revised Charter of the
PNB (EO No. 80).

Consequent to the privatization, all PNB employees, including Obusan, were


deemed retired from the government service. The GSIS confirmed Obusans
retirement from the government service, and accordingly paid her retirement gratuity
in the net amount of P390,633.76. Thereafter, Obusan continued to be an employee
of PNB.

Later, the PNB Board of Directors, through Resolution No. 30 dated December
22, 2000, as amended, approved the PNB Regular Retirement Plan (PNB-RRP).
Section 1, Article VI of which provides
Normal Retirement. The normal retirement date of a Member shall be the day he
attains sixty (60) years of age, regardless of length of service or has rendered thirty
(30) years of service, regardless of age, whichever of the said conditions comes first.
A Member who has reached the normal retirement date shall have to compulsorily
retire and shall be entitled to receive the retirement benefits under the Plan.
PNB informed its officers and employees of the terms and conditions of the PNB-RRP,
along with its implementing guidelines.

Subsequently, the PNB-RRP was registered with the BIR. Later,


the Philnabank Employees Association, the union of PNB rank-and-file

1020
Page 1021

employees, recognized the PNB-RRP in the Collective Bargaining


Agreement (CBA) it entered with PNB.

PNB informed Obusan that her last day of employment would be on March 3,
2002, as she would reach the mandatory retirement age of 60 years on March 4, 2002.
Obusan questioned her compulsory retirement and even threatened to take legal
action against PNB for illegal dismissal and unfair labor practice in the form of union
busting, Obusan being then the President of the PNB Supervisors and Officers
Association.

PNB replied to Obusan, explaining that compulsory retirement under the PNB-
RRP is not contrary to law and does not constitute union busting. Dissatisfied with
PNBs explanation, Obusan filed before the Labor Arbiter a complaint for illegal
dismissal and unfair labor practice, claiming that PNB could not compulsorily retire her
at the age of 60 years, with her having a vested right to be retired only at 65 years old
pursuant to civil service regulations.

The Labor Arbiter dismissed Obusan's complaint as he upheld the validity of the
PNB-RRP and its provisions on compulsory retirement upon reaching the age of 60
years.

The NLRC dismissed Obusan's appeal, and affirmed the assailed decision in
toto. Obusan's motion for reconsideration of this resolution was later denied.

CA dismissed the petition ratiocinating that the PNB-RRPs


lowering the compulsory retirement age to 60 years is not violative of
Article 287 of the Labor Code of the Philippines, as amended, despite
the issuance of the plan years after Obusan was hired. Obusan’s
motion for reconsideration of this Decision was subsequently denied.

ISSUE:
Can PNB unilaterally lower the compulsory retirement age to 60
years without violating Article 287 of the Labor Code and Obusans
alleged right to retire at the age of 65 years?

RULING:
Yes. Article 287 of the Labor Code, as amended by RA No. 7641, which took
effect on January 7, 1993,
provides:

“ART. 287. Retirement. Any employee may be retired upon


reaching the retirement age established
in the collective bargaining agreement or other applicable employment
contract.

In case of retirement, the employee shall be entitled to receive


such retirement benefits as he may have earned under existing laws

1021
Page 1022

and any collective bargaining agreement and other agreements:


Provided, however, That an employees retirement benefits under any
collective bargaining agreement and other agreements shall not be less
than those provided herein.

In the absence of a retirement plan or agreement providing for


retirement benefits of employees in the establishment, an employee
upon reaching the age of sixty (60) years or more, but not beyond
sixty-five (65) years which is hereby declared the compulsory retirement
age, who has served at least five (5) years in the said establishment,
may retire and shall be entitled to retirement pay
equivalent to at least one-half (1/2) month salary for every year of
service, a fraction of at least six (6) months being considered as one
whole year.

Unless the parties provide for broader inclusions, the term one-
half (1/2) month salary shall mean fifteen (15) days plus one-twelfth
(1/12) of the 13th month pay and the cash equivalent of not more than
five (5) days of service incentive leaves.”

Undoubtedly, under this provision, the retirement age is primarily determined by


the existing agreement or employment contract. Absent such an agreement, the
retirement age shall be fixed by law. The above-cited law mandates that the
compulsory retirement age is at 65 years, while the minimum age for optional
retirement is set at 60 years. Moreover, Article 287 of the Labor Code, as amended,
applies only to a situation where (1) there is no CBA or other applicable employment
contract providing for retirement benefits for an employee; or (2) there is a collective
bargaining agreement or other applicable employment contract providing for
retirement benefits for an employee, but it is below the requirement set by law. The
rationale for the first situation is to prevent the absurd situation where an employee,
deserving to receive retirement benefits, is denied through the nefarious scheme of
employers to deprive employees of the benefits due them under existing labor laws.
The rationale for the second situation is to prevent private contracts from derogating
from the public law.

Retirement plans allowing employers to retire employees who


have not yet reached the compulsory retirement age of 65 years are
not per se repugnant to the constitutional guaranty of security of
tenure. By its express language, the Labor Code permits employers
and employees to fix the applicable retirement age at 60 years or
below, provided that the employees retirement benefits under any CBA
and other agreements shall not be less than those provided therein. By
this yardstick, the PNB-RRP complies.

1022
Page 1023

Case Digest by: Cerera, Florence Diane N.

KIMBERLY CLARK PHILS v DIMAYUGA


G.R. No. 177705. September 18, 2009.
Carpio-Morales, J.

DOCTRINE:
RETIREMENT BENEFITS
It is settled that entitlement of employees to retirement benefits
must specifically be granted under existing laws, a collective
bargaining agreement or employment contract, or an established
employer policy. No law or collective bargaining agreement or other
applicable contract, or an established company policy was existing
during respondent’s employment entitling them to the P200,000 lump-
sum retirement pay. Petitioner was not thus obliged to grant them such
pay.

FACTS:
Respondents were employees of Kimberly-Clark Philippines, Inc.
(petitioner). Nora Dimayuga (Nora) was Cost Accounting Supervisor,
Rosemarie Gloria (Rosemarie) was Business Analyst, and Maricar de
Guia (Maricar) was General Accounting Manager.

On September 19, 2002, Nora tendered her resignation effective


October 21, 2002. On October 7, 2002, Rosemarie tendered her
resignation, also effective October 21, 2002.

As petitioner had been experiencing a downward trend in its


sales, it created a tax-free early retirement package for its employees
as a cost-cutting and streamlining measure. Twenty-four of its
employees availed of the offer that was made available from November
10-30, 2002.

Despite their resignation before the early retirement package


was offered, Nora and Rosemarie pleaded with petitioner that they be
retroactively extended the benefits thereunder, to which petitioner
acceded. Hence, Nora received a total of P1,025,113.73 while
Rosemarie received a total of P1,006,493.94, in consideration of which
they executed release and quitclaim deeds dated January 17, 2003
and January 16, 2003, respectively.

On November 4, 2002, Maricar tendered her resignation


effective December 1, 2002, citing career advancement as the reason
therefor. As at the time of her resignation the early retirement package

1023
Page 1024

was still effective, she received a total of P523,540.13 for which she
signed a release and quitclaim.
On November 28, 2002, petitioner announced that in lieu of the
merit increase which it did not give that year, it would provide economic
assistance, to be released the following day, to all monthly-paid
employees on regular status as of November 16, 2002.

Still later or on January 16, 2003, petitioner announced that it


would the grant a lump sum retirement pay in the amount of P200,000,
in addition to the early retirement package benefit, to those who signed
up for early retirement and who would sign up until January 22, 2003.

On May 23, 2003, respondents filed a Complaint before the NLRC Regional
Arbitration Branch No. IV against petitioner and its Finance Manager Fernando B.
Gomez (Gomez) whom respondents alleged to be responsible for the withholding of
[their] additional retirement benefits, claiming entitlement to the P200,000 lump sum
retirement pay. Respondents Nora and Rosemarie additionally claimed entitlement to
the economic assistance.

The Labor Arbiter dismissed the claims of Nora and Rosemarie,


holding that they were not entitled to the P200,000 lump sum
retirement pay, they having ceased to be employees of petitioner at
the time it was offered or made effective on January 16, 2003. He,
however, granted Maricar’s claim for the same pay, holding that she
was entitled to it because at the time she resigned from the company
effective December 1, 2002, such pay was already offered. Besides,
the Labor Arbiter ruled, Maricar had a vested right to it as she was
given a formal notice of her entitlement to it by petitioner, through its
Human Resources Director.

The NLRC modified the Labor Arbiters Decision by ordering


petitioner to pay Nora P200,000 additional bonus and P2,880
economic assistance, and to pay Rosemarie P200,000 additional
bonus and P2,656 economic assistance. It affirmed Maricar’s
entitlement to the lump sum retirement pay.

Applying the ruling in Businessday Information Systems and


Services, Inc. v. NLRC (Businessday), the NLRC ratiocinated that
petitioners refusal to give Nora and Rosemarie the lump sum
retirement pay was an act of discrimination, more so because a certain
Oscar Diokno, another employee who presumably resigned also prior
to January 16, 2003, was given said benefit.

As to the award of economic assistance, the NLRC held that Nora and
Rosemarie were also entitled to it as the same was given in lieu of the annual
performance-based salary increase that was not given in 2002 and, therefore, already
earned by them when they resigned.

1024
Page 1025

The CA affirmed the NLRC Decision.

Respondents argue that since other employees who resigned before the
announcement of the grant of the lump sum retirement pay received the same, they
(respondents) should also receive it, citing the pronouncement in Businessday that:
The law requires an employer to extend equal treatment to its employees. It may not,
in the guise of exercising management prerogatives, grant greater benefits to some
and less to others. Management prerogatives are not absolute prerogatives but are
subject to legal limits, collective bargaining agreements, or general principles of fair
play and justice.

ISSUE:
Whether or not respondents Nora, Rosemarie and Maricar are
entitled to retirement benefits and economic assistance.

RULING:
No. It is settled that entitlement of employees to retirement benefits must
specifically be granted under existing laws, a collective bargaining agreement or
employment contract, or an established employer policy. No law or collective
bargaining agreement or other applicable contract, or an established company policy
was existing during respondents’ employment entitling them to the P200,000 lump-
sum retirement pay. Petitioner was not thus obliged to grant them such pay.

Respondents’ reliance on Businessday is misplaced. The factual


milieu in Businessday is markedly different from that of the present
case. That case involved the retrenched employees’ separation pay to
which they are entitled under Article 283 of the Labor Code. In the
present case, Nora and Rosemarie resigned prior to petitioner’s offer
of the lump sum retirement pay as an incentive to those employees
who would voluntarily avail of its early retirement scheme as a cost-
cutting and streamlining measure. That respondents resigned, and not
retrenched, is clear from their respective letters to petitioner. And
nowhere in the letters is there any allegation that they resigned in view
of the company’s downward trend in sales which necessitated
downsizing or streamlining.

The appellate court’s finding that petitioner’s inclusion of Nora


and Rosemarie in the termination report submitted to the DOLE and its
grant to them of the early retirement benefits made them full retirees to
thus entitle them to the same benefits offered to those who would
voluntarily resign after November 16, 2003, does not lie.

Petitioner’s claim that it allowed Nora and Rosemarie to avail of


the early retirement package despite their previous separation from the
company out of pure generosity is well-taken in light of Noras letter of
September 15, 2002, asking if she could avail of the early retirement
package as it would certainly be of great assistance to us financially. It
is thus absurd to fault petitioner for acceding to such a request out of

1025
Page 1026

compassion by directing it to pay additional benefits to resigned


employees who are not entitled thereto.

Petitioner’s decision to extend the benefit to some former


employees who had already resigned before the offer of the lump sum
pay incentive was thus an act of generosity which it is not obliged to
extend to respondents.

Neither are Nora and Rosemarie entitled to the economic assistance which
petitioner awarded to all monthly employees who are under regular status as of
November 16, 2002, they having resigned earlier or on October 21, 2002.

Again, contrary to the appellate court’s ruling that Nora and


Rosemarie already earned the economic assistance, the same having
been given in lieu of the performance-based annual salary increase,
the Court finds that the economic assistance was a bonus over and
above the employees’ salaries and allowances. A perusal of the
memorandum regarding the grant of economic assistance shows that
it was granted in lieu of salary increase (the grant of which depends on
petitioner’s financial capability) and that it was not intended to be a
counterpart of the Collective Bargaining Agreement grant to members
of the K-CPI union. The grant of economic assistance to all monthly
employees under regular status as of November 16, 2002, was thus
well within petitioners prerogatives.

Moreover, petitioner’s decision to give economic assistance was


arrived at more than a month after respondents’ resignation and,
therefore, it was a benefit not yet existing at the time of their separation.
In any event, assuming that Nora and Rosemarie are entitled to the
economic assistance, they had signed release and quitclaim deeds
upon their resignation in which they waived.

While quitclaims executed by employees are commonly frowned


upon as being contrary to public policy and are ineffective to bar claims
for the full measure of their legal rights, where the person making the
waiver has done so voluntarily, with a full understanding thereof, and
the consideration for the quitclaim is credible and reasonable, the
transaction must be recognized as being a valid and binding
undertaking. In the case at bar, Nora and Rosemarie are accounting
graduates. They have not alleged having been compelled to sign the
quitclaims, nor that the considerations thereof (P1,024,113.73 for Nora
and P682,721.24 for Rosemarie) are unconscionable.

As for Maricar’s claim to the lump sum retirement pay, the Court finds that, like
Nora and Rosemarie, she is not entitled to it. Although the incentive was offered when
she was still connected with petitioner, she resigned from employment, citing career
advancement as the reason therefor. Indubitably, the incentive was addressed to
those employees who, without prior plans of resigning, opted to terminate their

1026
Page 1027

employment in light of the downsizing being undertaken by petitioner. In other words,


Maricar resigned from petitioner in order to find gainful employment elsewhere a
reason which has no bearing on the financial viability of petitioner.

1027
Page 1028

Case Digest by: Cerera, Florence Diane N.

MAGDADARO v PHILIPPINE NATIONAL BANK


G.R. No. 177705. September 18, 2009.
Carpio-Morales, J.

DOCTRINE:
RETIREMENT
Retirement is the result of a bilateral act of the parties, a voluntary
agreement between the employer and the employee whereby the
latter, after reaching a certain age, agrees to sever his or her
employment with the former. Retirement is provided for under Article
287 of the Labor Code, as amended by Republic Act No. 7641, or is
determined by an existing agreement between the employer and the
employee.

FACTS:
Marcelino A. Magdadaro was employed by PNB since 1968. In
1998, Magdadaro filed his application for early retirement under
respondent’s Special Separation Incentive Program (SSIP). Petitioner
stated in his application that 31 December 1999 was his preferred
effective date of retirement.

Respondent approved the application for early retirement but


made it effective on 31 December 1998. Petitioner protested the
acceleration of his retirement. He received, under protest, his
retirement and separation benefits amounting to P908,950.44.
Later, petitioner filed a complaint for illegal dismissal and payment of
moral, exemplary, and actual damages.

The Labor Arbiter ruled that respondent had the discretion and
prerogative to set the effective date of retirement under the SSIP. The
Labor Arbiter ruled that there was no dismissal to speak of because
petitioner voluntarily availed of the SSIP. Still, the Labor Arbiter granted
petitioners preferred date of retirement and awarded him additional
retirement benefits.

The NLRC affirmed the Labor Arbiters Decision. However, the


NLRC considered petitioners retirement on 31 December 1998 as
tantamount to illegal dismissal. The NLRC ruled that while it
recognized respondent’s prerogative to change petitioner’s retirement
date, management prerogative should be exercised with prudence and
without malice.

1028
Page 1029

The Court of Appeals held that petitioner voluntarily applied for


the SSIP. The Court of Appeals ruled that petitioner could not claim to
have been illegally dismissed just because the date of effectivity of his
retirement did not conform to his preferred retirement date.

ISSUE:
Whether or not petitioner was illegally dismissed from
employment.

RULING:
No. Retirement is the result of a bilateral act of the parties, a
voluntary agreement between the employer and the employee
whereby the latter, after reaching a certain age, agrees to sever his or
her employment with the former. Retirement is provided for under
Article 287 of the Labor Code, as amended by Republic Act No. 7641,
or is determined by an existing agreement between the employer and
the employee.

Whether petitioner’s early retirement within the SSIP period will


improve or impair the delivery of bank services is a business decision
properly within the exercise of management prerogative. More
importantly, the SSIP provides:

7. Management shall have the discretion and prerogative in


approving the applications filed under the Plan, as well as in setting
the effectivity dates for separation within the implementation
period of the Plan.

It is clear that it is within respondent’s prerogative to set the date


of effectivity of retirement; and it may not be necessarily what is stated
in the application. We see no grave abuse of discretion on the part of
respondent in the exercise of this management prerogative. The
exercise of management prerogative is valid provided it is not
performed in a malicious, harsh, oppressive, vindictive, or wanton
manner or out of malice or spite. In this case, the NLRCs finding that
petitioner received a rating of 70.5% in his working and business
relations is not enough reason to ascribe bad faith on the part of
respondent in accelerating the date of effectivity of petitioner’s
retirement.

1029
Page 1030

Case Digest by: Cerera, Florence Diane N.

LAYA v COURT OF APPEALS


G.R. No. 205813. January 10, 2018.
Bersamin, J.

DOCTRINE:
RETIREMENT
An employee in the private sector who did not expressly agree to
the terms of an early retirement plan cannot be separated from the
service before he reaches the age of 65 years. The employer who
retires the employee prematurely is guilty of illegal dismissal and is
liable to pay his backwages and to reinstate him without loss of
seniority and other benefits, unless the employee has meanwhile
reached the mandatory retirement age under the Labor Code, in which
case he is entitled to separation pay pursuant to the terms of the plan,
with legal interest on the backwages and separation pay reckoned from
the finality of the decision.

FACTS:
Petitioner Alfredo Laya, Jr. was hired by respondent Philippine
Veterans’ Bank as Chief Legal Counsel with a rank of Vice President.
One of the terms of his appointment include membership in the
Provident Fund Program/Retirement Program established by the
company. Pertinent provisions of the said program are:

ARTICLE IV RETIREMENT DATES


Section 1. Normal Retirement. The normal retirement date
of a Member shall be the first day of the month coincident
with or next following his attainment of age 60.
Section 2. Early Retirement. A Member may, with the
approval of the Board of Directors, retire early on the first
day of any month coincident with or following his
attainment of age 50 and completion of at least 10 years of
Credited Service.
Section 3. Late Retirement. A Member may, with the
approval of the Board of Directors, extend his service
beyond his nonnal retirement date but not beyond age 65.
Such deferred retirement shall be on a case by case and
yearly extension basis.

On June 14, 2007, petitioner was informed thru letter by private


respondent of his retirement effective July 1, 2007. Petitioner wrote to
Col. Emmanuel De Ocampo, Chairman of the respondent bank,
requesting for extension of his tenure by 2 more years pursuant to the

1030
Page 1031

Retirement Plan’s late retirement clause. On June 26, 2008, petitioner


received a memorandum directing him to continue the discharge of his
duties. However, on July 18, 2008, petitioner was informed thru
respondent bank’s president, Ricardo Balbido Jr. that his request for
extension was denied “to avoid precedence”. Petitioner sought for
reconsideration, but respondent bank certified the effectivity of the
former’s resignation, prompting petitioner to file a complaint for illegal
dismissal.

The Labor Arbiter dismissed the complaint; the dispositive


portion of which reads:

WHEREFORE, the charge of illegal dismissal and money


claims raised by the complainant, together with the
counterclaim raised by the respondents are DISMISSED
for lack of merit but by reason of a flaw in the denial of
complainant's application for term extension as discussed
above, the respondent bank is hereby ordered to pay the
complainant the amount of P200,000.00 by way of
reasonble (sic) indemnity.

The NLRC affirmed LA ruling but deleted the award of indemnity.

The CA ruled that the petitioner's acceptance of his appointment


as Chief Legal Officer of PVB signified his conformity to the retirement
program; 15 that he could not have been unaware of the retirement
program which had been in effect since January 1, 1996; 16 that the
lowering of the retirement age through the retirement plan was a
recognized exception under the provisions of Article 287 of the Labor
Code; 17 that considering his failure to adduce evidence showing that
PVB had acted maliciously in applying the provisions.

ISSUE:
Whether or not respondent validly retired by respondent bank at
the age of 60?

RULING:
No. The Court ruled that under Art. 287 (Now Art. 302) of the
Labor Code employers and employees may agree to fix the retirement
age for the latter, and to embody their agreement in either their
collective bargaining agreements (CBAs) or their employment
contracts. Retirement plans allowing employers to retire employees
who have not yet reached the compulsory retirement age of 65 years
are not per se repugnant to the constitutional guaranty of security of
tenure, provided that the retirement benefits are not lower than those
prescribed by law. However, it ruled that the CA erred in finding that

1031
Page 1032

petitioner’s acceptance of his employment carried with it the


agreement with the retirement plan (embodied as one of the terms of
petitioner’s appointment), the mere mention of the retirement plan in
the letter of appointment did not sufficiently inform the petitioner of the
contents or details of the retirement program. To construe from the
petitioner's acceptance of his appointment that he had acquiesced to
be retired earlier than the compulsory age of 65 years would, therefore,
not be warranted. This is because retirement should be the result of
the bilateral act of both the employer and the employee based on their
voluntary agreement that the employee agrees to sever his
employment upon reaching a certain age.

That the petitioner might be well-aware of the existence of the


retirement program at the time of his engagement did not suffice. His
implied knowledge, regardless of duration, did not equate to the
voluntary acceptance required by law in granting an early retirement
age option to the employee. The law demanded more than a passive
acquiescence on the part of the employee, considering that his early
retirement age option involved conceding the constitutional right to
security of tenure. Citing Cercado v. Umiprom, Inc., it stated:

Acceptance by the employees of an early retirement age


option must be explicit, voluntary, free, and uncompelled.
While an employer may unilaterally retire an employee
earlier than the legally permissible ages under the Labor
Code, this prerogative must be exercised pursuant to a
mutually instituted early retirement plan. In other words,
only the implementation and execution of the option may
be unilateral, but not the adoption and institution of the
retirement plan containing such option. For the option to be
valid, the retirement plan containing it must be voluntarily
assented to by the employees or at least by a majority of
them through a bargaining representative.

Further, a perusal of petitioner’s terms of appointment shows that


membership in the retirement plan is automatic upon acceptance of
the employment. Having thus automatically become a member of the
retirement plan through his acceptance of employment as Chief Legal
Officer of PVB, the petitioner could not withdraw from the plan except
upon his termination from employment.

In view of the foregoing, the Court disagrees with the view


tendered by Justice Leonen to the effect that the petitioner, because of
his legal expertise and educational attainment, could not now validly
claim that he was not informed of the provisions of the retirement
program. The pertinent rule on retirement plans does not presume
consent or acquiescence from the high educational attainment or legal
knowledge of the employee. In fact, the rule provides that the

1032
Page 1033

acquiescence by the employee cannot be lightly inferred from his


acceptance of employment.

Moreover, it was incumbent upon PVB to prove that the petitioner


had been fully apprised of the terms of the retirement program at the
time of his acceptance of the offer of employment. PVB did not
discharge its burden, for the petitioner's appointment letter apparently
enumerated only the minimum benefits that he would enjoy during his
employment by PVB and contained no indication of PVB having given
him a copy of the program itself in order to fully apprise him of the
contents and details thereof. Nonetheless, even assuming that he
subsequently obtained information about the program in the course of
his employment, he still could not opt to simply withdraw from the
program due to his membership therein being automatic for the regular
employees of PVB.

To stress, company retirement plans must not only comply with


the standards set by the prevailing labor laws but must also be
accepted by the employees as commensurate to their faithful services
to the employer within the requisite period. Although the employer
could be free to impose a retirement age lower than 65 years for as
long its employees consented, the retirement of the employee whose
intent to retire was not clearly established, or whose retirement was
involuntary is to be treated as a discharge.

With the petitioner having been thus dismissed pursuant to the


retirement provision that he had not knowingly and voluntarily agreed
to, PVB was guilty of illegal dismissal as to him.

1033
Page 1034

Case Digest by: Cerera, Florence Diane N.

TOLENTINO v PHILIPPINE AIRLINES


G.R. No. 218984. January 14, 2018.
Carpio, J.

DOCTRINE:
RETIREMENT
Retirement is the result of a bilateral act of the parties, a voluntary
agreement between the employer and the employee whereby the
latter, after reaching a certain age, agrees to sever his or her
employment with the former. It is clear; therefore, Tolentino had not
retired from Philippine Airlines – it was not a result of a voluntary
agreement. Tolentino lost his employment status because of his own
actions.

FACTS:
Tolentino was hired by respondent Philippine Airlines, Inc. (PAL)
as a flight engineer then was promoted to the rank of A340/A330
Captain. As a pilot, Tolentino was a member of the Airline Pilots
Association of the Philippines (ALPAP), which had a collective
bargaining agreement (CBA) with PAL.

ALPAP members went on strike. The Secretary of Labor issued


an Order requiring all striking officers and members of ALPAP to return
to work within 24 hours from receipt of the Order and requiring PAL
management to accept them under the same terms and conditions of
employment prior to the strike. While the union officers and members
had until 9 June 1998 to comply with the directive of the Secretary of
Labor, some pilots – including Tolentino – continued to participate in
the strike.

When Tolentino and other striking pilots returned to work, PAL


refused to readmit these returning pilots. Thus, they filed a complaint
for illegal lockout against PAL. On 20 July 1998, Tolentino reapplied
for employment with PAL as a newly hired pilot, and thus voluntarily
underwent the six months probationary period. After less than a year,
Tolentino tendered his resignation. Meanwhile, the Secretary of Labor
issued a Resolution declaring the strike conducted by ALPAP illegal
for being procedurally infirm and in open defiance of the return-to-work
order. Members and officers of ALPAP who participated in the strike in
defiance of the return-to-work order were declared to have lost their
employment status.

1034
Page 1035

Tolentino worked for a foreign airline. Upon his return, he


informed PAL of his intention of collecting his separation and/or
retirement benefits under the CBA. PAL refused to pay Tolentino the
separation and/or retirement benefits as stated in the CBA. Tolentino
filed his complaint against PAL for non-payment of holiday pay, rest
day pay, separation pay, and retirement benefits with prayer for the
payment of damages and attorney's fees.

ISSUE:
Whether or not the petitioner-heirs are entitled to receive Capt.
Tolentino’s retirement benefits under the CBA and/or equity in the
retirement fund under the PAL Pilot’s Retirement Benefit Plan.

RULING:
No. An employee who knowingly defies a return-to-work order issued by the
Secretary of Labor is deemed to have committed an illegal act which is a just cause to
dismiss the employee under Article 282 of the Labor Code.

In fact, it has already been settled that those who participated in


the 5 June 1998 strike of ALPAP are deemed to have lost their
employment status with PAL.

Thus, Tolentino, who did not deny his participation in the strike
and his failure to promptly comply with the return-to-work order of the
Secretary of Labor, could not claim any retirement benefits because he
did not retire – he simply lost his employment status.

Retirement is the result of a bilateral act of the parties, a


voluntary agreement between the employer and the employee
whereby the latter, after reaching a certain age, agrees to sever his or
her employment with the former. It is clear; therefore, Tolentino had
not retired from PAL – it was not a result of a voluntary agreement.
Tolentino lost his employment status because of his own actions.

Admittedly, Tolentino was hired again by PAL on 20 July 1998.


This was after he reapplied with the company. He also voluntarily
completed the probationary period of six months. It was made clear to
Tolentino, and he certainly admitted, that he was rehired on the
condition that his employment would be as a new hire. Reemployment,
on the condition that the employee will be treated as a new employee,
is a valid exercise of the employer's prerogative, as long as it is not
done with anti-union motivation. less than one year after he was
rehired as a new pilot, Tolentino resigned from PAL. In this instance,
Tolentino had voluntarily resigned from work. However, the act of
resignation alone does not entitle him to retirement benefits which he
claimed under the PAL-ALPAP Retirement Plan.
Tolentino is not entitled to any of the retirement benefits under the
PAL-ALPAP Retirement Plan. He had not completed even one year of

1035
Page 1036

his new employment with PAL. The Rules and Regulations of the PAL-
ALPAP Retirement Plan provide that the member-pilot must have
completed at least five years of continuous service with PAL to be
entitled to the resignation benefit. His resignation in July 1999, which
was only about a year from when he was rehired by the company, did
not qualify him for such resignation benefit.

For purposes of the retirement plan, the computation of Tolentino's


length of service to the company should be reckoned from the date he
was rehired after his own voluntary application as a new pilot. His
services from October 1971 to June 1998 cannot be tacked to his new
employment starting in July 1998 because the first employment had
already been finally terminated – not due to his voluntary resignation
or retirement, but because of termination due to just causes. The
requirements under the PAL-ALPAP Retirement Plan must be present
at the time the employee resigns or retires from PAL. Unfortunately for
Tolentino, when he finally tendered his resignation with PAL, he was
no longer compliant with the requirements for the retirement benefit –
as a new hire, he only completed less than one year of service.
Therefore, he is not entitled to any retirement or resignation benefits
under the PAL-ALPAP Retirement Plan.

1036
Page 1037

Case Digest by: Cerera, Florence Diane N.

DE LA SALLE ARANETA UNIVERSITY v BERNARDO


G.R. No. 180809. February 13, 2017.
Leonardo-De Castro, J.

DOCTRINE:
PRESCRIPTION
ART 291. MONEY CLAIMS. - All money claims arising
from employer-employee relations accruing during the
effectivity of this Code shall be filed within three (3) years
from the time the cause of action accrued; otherwise they
shall forever be barred.

xxxx

The prescriptive period referred to in Article 291 of the Labor


Code, as amended applies to all kinds of money claims arising from
employer-employee relations including claims for retirement benefits.

The ruling of the Supreme Court in De Guzman v. Court of


Appeals, (G.R. No. 132257, October 12, 1998), squarely applies to the
instant case:

"The language of Article 291 of the Labor Code does not


limit its application only to "money claims specifically
recoverable under said Code, " but covers all money
claims arising from employer-employee relations. Since
petitioners' demand for unpaid retirement/separation
benefits is a money claim arising from their employment by
private respondent, Article 291 of the Labor Code is
applicable. Therefore, petitioners' claim should be filed
within three years from the time their cause of action
accrued, or forever barred by prescription. "

It cannot be denied that the claim for retirement benefits/pay arose out
of employer-employee relations. In line with the decision of the
Supreme Court in De Guzman, it should be treated as a money claim
that must be claimed within three years from the time the cause of
action accrued.

FACTS:
On February 26, 2004, Bernardo filed a complaint against DLS-
AU and its owner/manager, Dr. Oscar Bautista (Dr. Bautista), for the
payment of retirement benefits.

1037
Page 1038

Bernardo alleged that he started working as a part-time


professional lecturer at DLS-AU on June 1, 1974 for an hourly rate of
₱20.00. Bernardo taught for two semesters and the summer for the
school year 1974-1975. Bernardo then took a leave of absence from
June 1, 1975 to October 31, 1977 when he was assigned by the
Philippine Government to work in Papua New Guinea.

When Bernardo came back in 1977, he resumed teaching at


DLS-AU until October 12, 2003, the end of the first semester for school
year 2003-2004. Bernardo's teaching contract was renewed at the start
of every semester and summer. However, on November 8, 2003, DLS-
AU informed Bernardo through a telephone call that he could not teach
at the school anymore as the school was implementing the retirement
age limit for its faculty members.

As he was already 75 years old, Bernardo had no choice but to


retire. At the time of his retirement, Bernardo was being paid ₱246.50
per hour. Bernardo immediately sought advice from the DOLE
regarding his entitlement to retirement benefits after 27 years of
employment.

The DOLE, through its Public Assistance Center and Legal


Service Office, opined that Bernardo was entitled to receive benefits
under Republic Act No. 7641, otherwise known as the "New
Retirement Law," and its Implementing Rules and Regulations. Yet, Dr.
Bautista stated that Bernardo was not entitled to any kind of separation
pay or benefits. Dr. Bautista explained to Bernardo that as mandated
by the DLS-AU's policy and CBA, only full-time permanent faculty of
DLS-AU for at least five years immediately preceding the termination
of their employment could avail themselves of the postemployment
benefits. As part-time faculty member, Bernardo did not acquire
permanent employment under the Manual of Regulations for Private
Schools, in relation to the Labor Code, regardless of his length of
service.

The Labor Arbiter rendered its Decision dismissing Bernardo's


complaint on the ground of prescription.

The NLRC reversed the Labor Arbiter's ruling and found that
Bernardo timely filed his complaint for retirement benefits.

The CA promulgated its Decision, affirming in toto the NLRC judgment.

ISSUE:

Whether or not respondent timely filed his case.

RULING:

1038
Page 1039

Yes. A cause of action has three elements, to wit, (1) a right in favor of the
plaintiff by whatever means and under whatever law it arises or is created; (2) an
obligation on the part of the named defendant to respect or not to violate such right;
and (3) an act or omission on the part of such defendant violative of the right of the
plaintiff or constituting a breach of the obligation of the defendant to the plaintiff.

Bernardo's right to retirement benefits and the obligation of DLS-


AU to pay such benefits are already established under Article 302 [287]
of the Labor Code, as amended by Republic Act No. 7641. However,
there was a violation of Bernardo's right only after DLS-AU informed
him on November 8, 2003 that the university no longer intended to offer
him another contract of employment, and already accepting his
separation from service, Bernardo sought his retirement benefits, but
was denied by DLSAU. Therefore, the cause of action for Bernardo's
retirement benefits only accrued after the refusal of DLS-AU to pay him
the same, clearly expressed in Dr. Bautista's letter dated February 12,
2004. Hence, Bernardo's complaint, filed with the NLRC on February
26, 2004, was filed within the three-year prescriptive period provided
under Article 291 of the Labor Code.

1039
Page 1040

Case Digest by: Cerera, Florence Diane N.

UDMC v BERNADAS
G.R. No. 218984. January 14, 2018.
Carpio, J.

DOCTRINE:
RETIREMENT
An employee who has already qualified for optional retirement but
dies before the option to retire could be exercised is entitled to his or
her optional retirement benefits, which may be claimed by the qualified
employee's beneficiaries on his or her behalf.

FACTS:
Respondent Cesario Bernadas started working as an orderly
under petitioner United Doctor Medical Center’s (UDMC)
housekeeping department on July 17, 1986 and was later on promoted
as utility man. UDMC and its rank-and-file employees had at that time
a Collective Bargaining Agreement providing for optional retirement
benefits. It provides:

ARTICLE XI RETIREMENT AND SEVERANCE PAY


SECTION 1. RETIREMENT AND SEVERANCE PAY.
The CENTER shall grant each employee retirement and
severance pay in accordance with law. It shall also
continue its present policy on optional retirement.

Under the optional retirement policy, employees who rendered at


least 20 years of service is entitled to optionally retire, with the
retirement pay equivalent to a retiree’s salary for 11 days for every year
of service. Insurance (premium paid by UDMC) is also provided to the
employees with their family members as beneficiaries.

On Oct. 20, 2009, Cesario died due to a “freak accident” while


working in a doctor’s residence. He was 53 years old.

Leonila Bernadas, representing her husband, respondent


Cesario, filed a claim for payment of retirement benefits with the NLRC.
However, petitioner contended that respondent’s beneficiaries do not
have legal capacity to apply for the optional retirement benefits since
respondent himself didn’t apply for such during his lifetime. Further, it
claims that even assuming that respondent was already qualified to
apply for optional retirement 3 years prior to his death, he never did.

1040
Page 1041

On the other hand, Leonila counters that had her husband died
"under normal circumstances," he would have applied for optional
retirement benefits. That Cesario was unable to apply before his death
"is a procedural technicality" that should be set aside so that "full
protection to labor" is afforded and "the ends of social and
compassionate justice" are met.

The Labor Arbiter dismissed the complaint. It ruled that Cesario


should have applied for optional retirement benefits during his lifetime,
the benefits being optional. Since he did not apply for it, his
beneficiaries were not entitled to claim his optional retirement benefits.

The NLRC reversed LA ruling. It found that the optional


retirement plan was never presented in this case, casting a doubt on
whether or not the plan required an application for optional retirement
benefits before an employee could become entitled to them.
Considering the "constitutional mandate to afford full protection to
labor," the National Labor Relations Commission resolved the doubt in
favor of Cesario.

The CA affirmed NLRC ruling.

ISSUE:
Whether or not Cesario is entitled to optional retirement benefits.

RULING:
Yes. To begin, the Supreme Court discussed the 3 types of retirement existing
in the country. It stated:

“Within this jurisdiction, there are three (3) types of


retirement plans available to employees. The first is
compulsory and contributory. This type of plan is embodied
in Republic Act No. 828240 for those in the private sector
and Republic Act No. 829141 for those in the government.
These laws require a mandatory contribution from the
employer as well as the employee, which shall become a
pension fund for the employee upon retirement.
Considering that the mandatory employee contribution is
deducted from the employee's monthly income, "retirement
packages are usually crafted as 'forced savings' on the part
of the employee."

Under this type of retirement plan, the pension is not considered


as mere gratuity but actually forms part of the employee's
compensation. An employee acquires a vested right to the benefits that
have become due upon reaching the compulsory age of retirement.
Thus, the beneficiaries of the retired employee are entitled to the
pension even after the retired employee's death. The second and third

1041
Page 1042

types of retirement plans are voluntary. They may not even require the
employee to contribute to a pension fund. The second type of retirement
plan is by agreement between the employer and the employee, usually
embodied in the CBA between them. "The third type is one that is
voluntarily given by the employer, expressly as in an announced
company policy or impliedly as in a failure to contest the employee's
claim for retirement benefits." The rules regarding the second and third
types of retirement plans are provided for in Article 302 [287]49 of the
Labor Code, as amended.”

It held that these types of retirement plans are not meant to be a


replacement to the compulsory retirement scheme under social
security laws but must be understood as a retirement plan in addition
to that provided by law and that Art. 302 allows the employer and
employee to enter into early retirement packages or schemes. Citing
Pantranco North Express v. NLRC, it provided the rationale for early
retirement packages as:

“In almost all countries today, early retirement, i.e., before


age 60, is considered a reward for services rendered since
it enables an employee to reap the fruits of his labor -
particularly retirement benefits, whether lump-sum or
otherwise - at an earlier age, when said employee, in
presumably better physical and mental condition, can
enjoy them better and longer. As a matter of fact, one of
the advantages of early retirement is that the
corresponding retirement benefits, usually consisting of a
substantial cash windfall, can early on be put to productive
and profitable uses by way of income-generating
investments, thereby affording a more significant measure
of financial security and independence for the retiree who,
up till then, had to contend with life's vicissitudes within the
parameters of his fortnightly or weekly wages. Thus, we
are now seeing many CBAs with such early retirement
provisions.”

It further discussed that acceptance by the employees of an early


retirement age option must be explicit, voluntary, free, and
uncompelled. While an employer may unilaterally retire an employee
earlier than the legally permissible ages under the Labor Code, this
prerogative must be exercised pursuant to a mutually instituted early
retirement plan. In other words, only the implementation and execution
of the option may be unilateral, but not the adoption and institution of
the retirement plan containing such option. For the option to be valid,
the retirement plan containing it must be voluntarily assented to by the
employees or .at least by a majority of them through a bargaining
representative.

1042
Page 1043

The Supreme Court then moved on to discuss the entitlement of


respondent Cesario to the optional retirement benefits, examining first
the provision in the UDMC CBA providing for such (reproduced in the
“FACTS”) and stating that the CBA does not provide for the terms and
conditions of the "present policy on optional retirement. It observed that
Leonila merely alleged before the Labor Arbiter that petitioner "grants
an employee a retirement or separation equivalent to eleven (11) days
per year of service after serving for at least twenty (20) years," which
was not disputed by petitioner. Therefore, doubt arises as to what
petitioner's optional retirement package actually entails.

The Court then reiterated that any doubt as to the law must be resolved in favor
of labor and that retirement laws should be liberally construed and administered in
favor of the persons intended to be benefited and all doubts as to the intent of the law
should be resolved in favor of the retiree to achieve its humanitarian purposes.
Retirement encompasses even the concept of death and the Court has considered
death as a form of disability retirement as "there is no more permanent or total physical
disability than death. Compulsory retirement and death both involve events beyond
the employee's control. While the choice to retire before the compulsory age of
retirement was within respondent Cesario's control, his death foreclosed the possibility
of him making that choice.

Petitioner's optional retirement plan is premised on length of service, not upon


reaching a certain age. It rewards loyalty and continued service by granting an
employee an earlier age to claim his or her retirement benefits even if the employee
has not reached his or her twilight years. It would be the height of inequity to withhold
respondent Cesario' s retirement benefits despite being qualified to receive it, simply
because he died before he could apply for it. In any case, the CBA does not mandate
that an application must first be filed by the employee before the right to the optional
retirement benefits may vest. Thus, this ambiguity should be resolved in favor of the
retiree. Retirement benefits are the property interests of the retiree and his or her
beneficiaries. The CBA does not prohibit the employee's beneficiaries from claiming
retirement benefits if the retiree dies before the proceeds could be released. Even
compulsory retirement plans provide mechanisms for a retiree's beneficiaries to claim
any pension due to the retiree Thus, Leonila, being the surviving spouse of respondent
Cesario is entitled to claim the optional retirement benefits on his behalf.

1043
Page 1044

Case Digest by: Cerera, Florence Diane N.

PAL v HASSARAM
G.R. No. 217730. June 5, 2017.
Sereno, CJ.

DOCTRINE:
RETIREMENT
Art. 287 (Now Art. 302) is applicable only to a situation where (l)
there is no CBA or other applicable employment contract providing for
retirement benefits for an employee, or (2) there is a CBA or other
applicable employment contract providing for retirement benefits for an
employee, but it is below the requirement set by law. The rationale for
the first situation is to prevent the absurd situation where an employee,
deserving to receive retirement benefits, is denied them through the
nefarious scheme of employers to deprive employees of the benefits
due them under existing labor laws. On the other hand, the second
situation aims to prevent private contracts from derogating from the
public law.

...Emphasis must be placed on the fact that the purpose of the


amendment is not merely to establish precedence in application or
accord blanket priority to existing CBAs in computing retirement
benefits. The determining factor in choosing which retirement scheme
to apply is still superiority in terms of benefits provided. Thus, even if
there is an existing CBA but the same does not provide for retirement
benefits equal or superior to that which is provided under Article 287 of
the Labor Code, the latter will apply. In this manner, the employee can
be assured of a reasonable amount of retirement pay for his
sustenance.

FACTS:
Respondent Arjan T. Hassaram, a former PAL pilot filed a complaint against
PAL for illegal dismissal and payment of retirement benefits. He averred that he had
applied for retirement in August 2000 after 24 years of service as pilot but was denied.
Instead, PAL informed him that he had lost his employment in the company as of 9
June 1998, in view of his failure to comply with the Return to Work Order issued by
the Secretary of Labor against members of the Airline Pilots Association of the
Philippines (ALPAP) on 7 June 1998. Before the Labor Arbiter (LA), Hassaram argued
that he was not covered by the Secretary's Return to Work Order; hence, PAL had no
valid ground for his dismissal. He asserted that on 9 June 1998, he was already on his
way to Taipei to report for work at Eva Air, pursuant to a four-year contract approved
by PAL itself. Petitioner further claimed that his arrangement with PAL allowed him to
go on leave without pay while working for Eva Air, with the right to accrue seniority and
retire from PAL during the period of his leave.

1044
Page 1045

PAL, on the other hand contended that (a) the LA had no


jurisdiction over the case, which was a mere off-shoot of ALPAP's
strike, a matter over which the Secretary of Labor had already
assumed jurisdiction; (b) the Complaint should be considered barred
by res judicata, forum shopping, and prescription; (c) the case should
be suspended while PAL was under receivership; and (d) if at all,
Hassaram was entitled only to retirement benefits of ₱5,000 for every
year of service pursuant to the Collective Bargaining Agreement (CBA)
between PAL and ALPAP.

The Labor Arbiter ruled in favor of respondent, awarding


retirement benefits and attorney’s fees. It ruled Hassaram did not defy
the Return to Work Order, as he was in fact already on leave when the
order was implemented. As to the computation of benefits, the LA ruled
that Article 287 of the Labor Code should be applied, since the statute
provided better benefits than the PAL-ALPAP CBA.

The NLRC initially affirmed the LA's Decision to award retirement


benefits to respondent Hassaram under Article 287 of the Labor Code.
This affirmation prompted PAL to seek reconsideration of the ruling
citing, for the first time, respondent's purported receipt of retirement
benefits in the amount of ₱4,456,817.75 pursuant to the PAL Pilots’
Retirement Benefit Plan. PAL likewise alleged that, as a consequence
of this newly discovered payment, any claim made by Hassaram for
retirement benefits should be deemed extinguished.

The NLRC granted PAL's Motion for Reconsideration and


reversed the LA ruling. It held that the ₱4,456,817.75 paid to
respondent as retirement benefits under the plan was sufficient to
discharge his claim for retirement pay.

The CA ruled that the funds received under the PAL Pilots’
Retirement Benefit Plan were not the retirement benefits contemplated
by law. Hence, respondent was still entitled to receive retirement
benefits in the amount of ₱2, 111,984.60 pursuant to Article 287 of the
Labor Code.

ISSUE:

(1) Whether or not the amount received by respondent under the


PAL Pilots’ Retirement Benefit Plan be deemed part of his retirement
pay.
(2) Whether or not respondent entitled is to receive retirement
benefits under Art. 287 (Now Art, 302) of the Labor Code.

RULING:

1045
Page 1046

(1) Yes. The Court that under the provisions of the plan that it is the
petitioner that contributes to a "retirement fund" for the account of the pilots. These
contributions comprise the benefits received by the latter upon retirement, separation
from service, or disability. It then cited Philippine Airlines, Inc. v. Airline Pilots
Association of the Phils. explains provisions under the plan:

“The PAL Pilots' Retirement Benefit Plan is a retirement fund


raised from contributions exclusively from PAL of amounts equivalent to
20% of each pilot's gross monthly pay. Upon retirement, each pilot
stands to receive the full amount of the contribution. In sum, therefore,
the pilot gets an amount equivalent to 240% of his gross monthly income
for every year of service he rendered to petitioner. This is in addition to
the amount of not less than ₱100,000.00 that he shall receive under the
1967 Retirement Plan.”

Based on the foregoing characterization, the Court included the amount


received from the Plan in the computation of the retirement pay of the pilot involved in
that case. Considering that the very same retirement plan is involved in the case the
Court ruled that the amount of ₱4,456,8l7.75 received by Hassaram from the PAL
Plan formed part of his retirement pay.

(2) No. The Court held that the benefits due to Hassaram under the PAL
retirement plan are greater than that when the provisions of Art. 287 (Now Art. 302)
of the Labor Code are applied. Citing Elegir v. PAL, it stated:

“It can be clearly inferred from the language of the foregoing


provision that it is applicable only to a situation where (l) there is no CBA
or other applicable employment contract providing for retirement
benefits for an employee, or (2) there is a CBA or other applicable
employment contract providing for retirement benefits for an employee,
but it is below the requirement set by law. The rationale for the first
situation is to prevent the absurd situation where an employee,
deserving to receive retirement benefits, is denied them through the
nefarious scheme of employers to deprive employees of the benefits
due them under existing labor laws. On the other hand, the second
situation aims to prevent private contracts from derogating from the
public law.

Emphasis must be placed on the fact that the purpose of the


amendment is not merely to establish precedence in application or
accord blanket priority to existing CBAs in computing retirement
benefits. The determining factor in choosing which retirement scheme
to apply is still superiority in terms of benefits provided. Thus, even if
there is an existing CBA but the same does not provide for retirement
benefits equal or superior to that which is provided under Article 287 of
the Labor Code, the latter will apply. In this manner, the employee can
be assured of a reasonable amount of retirement pay for his
sustenance.”

1046
Page 1047

All in all, the retirement benefits under Art. 287 is applicable only to a situation
where (l) there is no CBA or other applicable employment contract providing for
retirement benefits for an employee, or (2) there is a CBA or other applicable
employment contract providing for retirement benefits for an employee, but it is below
the requirement set by law. Having established that Hassaram has already received
benefits under PAL retirement’s program, with said benefits being greater than those
afforded under Art. 287, he may no longer claim the benefits under the said provision
of the Labor Code.

1047
Page 1048

Case Digest by: Cerera, Florence Diane N.

CARISSA E. SANTO v UNIVERSITY OF CEBU


G.R. No. 232522. August 28, 2019.
Lazaro-Javier, J.

DOCTRINE:
RETIREMENT
As amended by RA 7641, the provision bears two (2) types of retirements: 1)
optional at age sixty (60); and 2) compulsory at age sixty-five (65). The law does not
make a distinction as to the retirement benefits granted in either case. In both cases,
the retirement benefit is equivalent to 1/2 month salary for every year of service, the
1/2 month being computed at 22.5 days provided the employee has worked with his
or her employer for at least five (5) years prior to retirement.

FACTS:
In May 1997, respondent University of Cebu hired petitioner Carissa E. Santo
as a full-time instructor. During her employment, as such, she studied law and passed
the 2009 Bar Examinations. She continued working for respondent until she got
qualified for optional retirement under respondent's Faculty Manual, viz:

“Optional Retirement

A permanent employee, may, upon reaching his fifty-fifth (55th)


birthday or after having completed at least fifteen (15) years of service,
opt for an early retirement (which is a resignation with separation pay)
considering that separation before reaching 15 years of full-time service
does not entitle an employee to any separation pay, except that which
is contributed by the University to PAG-IBIG), and shall be entitled to
the retirement pay equivalent to a total of fifteen (15) days for every year
of service based on the average monthly salary to the employee
computed for the past three years.”

In April 2013, she applied for optional retirement; she was then only forty-two
(42) years old but had already completed sixteen (16) years of service with
respondent. The latter approved her application and computed her optional retirement
pay at fifteen (15) days for every year of service per provisions of the Faculty Manual.
She asserted, though, that her retirement pay should be equivalent to 22.5 days per
year of service in accordance with Article 287 of the Labor Code. Respondent refused
to accept her computation. Thus, she initiated the complaint below for payment of
retirement benefits under Article 287 of the Labor Code, damages, and attorney's fees
against respondent.

The Labor Arbiter found that respondent’s retirement package was less than
what Article 287 of the Labor Code prescribed i.e. 22.5 days for every year of service.

1048
Page 1049

The NLRC reversed the ruling of the Labor Arbiter. It ruled that Article 287 was
not intended to benefit petitioner who voluntarily resigned not to rest in the twilight
years of her life but to actively engage in the practice of the legal profession.

The CA affirmed the NLRC’s ruling.

ISSUE:

Whether or not petitioner is entitled to retirement benefits under


the Labor Code?

RULING:
Retirement benefits are a form of reward for an employee's loyalty and service
to an employer and are earned under existing laws, Collective Bargaining Agreements
(CBA), employment contracts and company policies. It is the result of a bilateral act of
the parties, a voluntary agreement between the employer and the employee whereby
the latter, after reaching a certain age or length of service, agrees to sever his or her
employment with the former.

The optional retirement benefits granted under respondent’s Faculty Manual


squarely fits the definition.

Clearly, the Faculty Manual intends to grant retirement benefits to qualified


employees. It entitles an employee to retire after fifteen (15) years of service or upon
reaching the age of fifty-five (55) and accordingly collect retirement benefits. It even
mandates compliance with RA 7641 such that when the computation of its retirement
plan is found to be lower than what the law requires, respondent is bound to pay the
deficiency.

Respondent's claim -- that its optional retirement benefit is actually a form of


separation pay to qualified employees who wish to resign is belied by its own company
policy. This benefit clearly falls within the category of "Retirement Pay," specifically
under "Optional Retirement." For sure, respondent is precluded from claiming
otherwise.

The optional retirement under respondent's Faculty Manual, therefore, should


not be taken as anything else but a retirement benefit within the ambit of Article 287 of
the Labor Code.

We are confronted with two (2) retirement schemes here: 1) Article 287 of the Labor
Code; and 2) Respondent's Faculty Manual. The riveting question is "which retirement
scheme applies to petitioner?"

Article 287 of the Labor Code.

As amended by RA 7641, the provision bears two (2) types of retirements: 1)


optional at age sixty (60); and 2) compulsory at age sixty-five (65). The law does not

1049
Page 1050

make a distinction as to the retirement benefits granted in either case. In both cases,
the retirement benefit is equivalent to 1/2 month salary for every year of service, the
1/2 month being computed at 22.5 days provided the employee has worked with his
or her employer for at least five (5) years prior to retirement.

Now, comparing the optional retirement benefits under the two (2) retirement
schemes, it is apparent that fifteen (15) days' worth of salary for every year of service
provided under respondent's Faculty Manual is much less than 22.5 days' worth of
salary for every year of service provided under Article 287 of the Labor Code.
Obviously, it is more beneficial for petitioner if Article 287's retirement plan will be
applied in the computation of' her retirement benefits.

In Beltran v. AMA Computer College-Biñan, the Court ruled that while the
employer is free to grant retirement benefits and impose different age or service
requirements, the benefits should not be lesser than those provided in Article 287 of
the Labor Code.

1050
Page 1051

Case Digest by: Cerera, Florence Diane N.

PADILLO v RURAL BANK OF NABUNTURAN


G.R. No. 199338. January 21, 2013.
Perlas-Bernabe, J.

DOCTRINE:
RETIREMENT
In particular, Article 300 of the Labor Code as amended by
Republic Act Nos. 7641 and 8558. Simply stated, in the absence of any
applicable agreement, an employee must (1) retire when he is at least
sixty (60) years of age and (2) serve at least (5) years in the company
to entitle him/her to a retirement benefit of at least one-half (1/2) month
salary for every year of service, with a fraction of at least six (6) months
being considered as one whole year.

FACTS:
On October 1, 1977, petitioner Padillo, was employed by
respondent Rural Bank of Nabunturan, Inc. (Bank) as its SA
Bookkeeper. Due to liquidity problems which arose sometime in 2003,
the Bank took out retirement/insurance plans with Philippine American
Life and General Insurance Company (Philam Life) for all its
employees in anticipation of its possible closure and the concomitant
severance of its personnel. The Bank procured Philam Life Plan in
favor of Padillo for a benefit amount of ₱100,000.00 and which was set
to mature on July 11, 2009.

On October 14, 2004, respondent Mark S. Oropeza (Oropeza),


the President of the Bank, bought majority shares of stock in the Bank
and took over its management which brought about its gradual
rehabilitation. The Bank’s finances improved and eventually, its
liquidity was regained.

During the latter part of 2007, Padillo suffered a mild stroke due
to hypertension which consequently impaired his ability to effectively
pursue his work. On September 10, 2007, he wrote a letter addressed
to respondent Oropeza expressing his intention to avail of an early
retirement package. Despite several follow-ups, his request remained
unheeded.

On October 3, 2007, Padillo was separated from employment


due to his poor and failing health. Not having received his claimed
retirement benefits, Padillo filed on September 23, 2008 a complaint
for the recovery of unpaid retirement benefits. He asserted, among
others, that the Bank had adopted a policy of granting its aging
employees early retirement packages. The Bank and Oropeza

1051
Page 1052

(respondents) countered that the claim of Padillo for retirement


benefits was not favorably acted upon for lack of any basis to grant the
same.

The Labor Arbiter dismissed Padillo’s complaint but directed the


Bank to pay him the amount of ₱100,000.00 as financial assistance,
treated as an advance from the amounts receivable under the Philam
Life Plan. It found Padillo disqualified to receive any benefits under
Article 300 (formerly, Article 287) of the Labor Code of the Philippines
(Labor Code) as he was only fifty-five (55) years old when he resigned,
while the law specifically provides for an optional retirement age of
sixty (60) and compulsory retirement age of sixty-five (65).

The NLRC ordered respondents to pay Padillo the amount of


₱164,903.70 as separation pay, on top of the ₱100,000.00 Philam Life
Plan benefit. Relying on the case of Abaquin Security and Detective
Agency, Inc. v. Atienza (Abaquin), the NLRC applied the Labor Code
provision on termination on the ground of disease – particularly, Article
297 thereof (formerly, Article 323) – holding that while Padillo did
resign, he did so only because of his poor health condition.

The Court of Appeals reinstated the LA’s March 13, 2009 Decision but with
modification. It directed the respondents to pay Padillo the amount of ₱50,000.00 as
financial assistance exclusive of the ₱100,000.00 Philam Life Plan benefit which
already matured on July 11, 2009. It likewise found the evidence insufficient to prove
that the Bank has an existing company policy of granting retirement benefits to its
aging employees.

ISSUE:

Whether or not Padillo is entitled to early retirement benefits.

RULING:
No. The Labor Code provision on termination on the ground of
disease under Article 297 does not apply in this case, considering that
it was the petitioner and not the Bank who severed the employment
relations. As borne from the records, the clear import of Padillo’s
September 10, 2007 letter and the fact that he stopped working before
the foregoing date and never reported for work even thereafter show
that it was Padillo who voluntarily retired and that he was not
terminated by the Bank.

What remains applicable, however, is the Labor Code provision


on retirement. In particular, Article 300 of the Labor Code as amended
by Republic Act Nos. 7641 and 8558. Simply stated, in the absence of
any applicable agreement, an employee must (1) retire when he is at
least sixty (60) years of age and (2) serve at least (5) years in the
company to entitle him/her to a retirement benefit of at least one-half

1052
Page 1053

(1/2) month salary for every year of service, with a fraction of at least
six (6) months being considered as one whole year.

Notably, these age and tenure requirements are cumulative and


non-compliance with one negates the employee’s entitlement to the
retirement benefits under Article 300 of the Labor Code altogether.
In this case, it is undisputed that there exists no retirement plan,
collective bargaining agreement or any other equivalent contract
between the parties which set out the terms and condition for the
retirement of employees, with the sole exception of the Philam Life
Plan which premiums had already been paid by the Bank. Neither was
it proven that there exists an established company policy of giving early
retirement packages to the Bank’s aging employees.

All told, in the absence of any applicable contract or any evolved


company policy, Padillo should have met the age and tenure
requirements set forth under Article 300 of the Labor Code to be
entitled to the retirement benefits provided therein. Unfortunately, while
Padillo was able to comply with the five (5) year tenure requirement –
as he served for twenty-nine (29) years – he, however, fell short with
respect to the sixty (60) year age requirement given that he was only
fifty-five (55) years old when he retired. Therefore, without prejudice to
the proceeds due under the Philam Life Plan, petitioners’ claim for
retirement benefits must be denied.

Finally, the Court finds no bad faith in any of respondents’ actuations as they
were within their right, absent any proof of its abuse, to ignore Padillo’s misplaced
claim for retirement benefits. Respondents’ obstinate refusal to accede to Padillo’s
request is precisely justified by the fact that there lies no basis under any applicable
agreement or law which accords the latter the right to demand any retirement benefits
from the Bank.

1053
Page 1054

Case Digest by: Cerera, Florence Diane N.

PHILIPPINE JOURNALISTS INC. v DE GUZMAN


G.R. No. 208027. April 01, 2019.
Del Castillo, J.

DOCTRINE:
RETIREMENT BENEFITS
To be considered as a regular company practice, the employee
must prove by substantial evidence that the giving of the benefit is done
over a long period of time, and that it has been made consistently and
deliberately. Jurisprudence has not laid down any hard-and-fast rule
as to the length of time that company practice should have been
exercised in order to constitute voluntary employer practice. The
common denominator in previously decided cases appears to be the
regularity and deliberateness of the grant of benefits over a significant
period of time. It requires an indubitable showing that the employer
agreed to continue giving the benefit knowing fully well that the
employees are not covered by any provision of the law or agreement
requiring payment thereof. In sum, the benefit must be characterized
by regularity, voluntary and deliberate intent of the employer to grant
the benefit over a considerable period of time.

FACTS:
Erika R. [sic] Marie de Guzman and Edna Quirante are both
employees of Philippine Journalists, Inc. ('PJI'). On 28 October 2008
and 23 January 2009 respectively, [respondents], in separate letters,
informed the company of their desire to avail of the company's optional
retirement plan as embodied in the Collective Bargaining Agreement.

Because of PJI's failure and refusal to process the payment of the


optional retirement benefits due them, [respondents] filed a complaint
for unfair labor practice and money claims, nonpayment of optional
retirement benefits and service incentive leave against PJ1 and its
corporate officers.

On 29 April 2010, the Labor Arbiter dismissed the complaint for


lack of merit. According to the Labor Arbiter respondents are not rank
and file employees and therefore not entitled to optional retirement
benefits.

The NLRC decided in favor of respondents.

The CA dismissed Petitioner’s Petition for Certiorari.

ISSUE:

1054
Page 1055

Whether or not respondents are entitled to optional retirement


benefits.

RULING:
Yes. Indeed, PJI appears to discriminate against its core
employees, while it favors those in the upper tier; it had been found
guilty of illegal dismissal based on an illegal retrenchment scheme,
while upper management continued to enjoy its perks and privileges
and refused to tighten its belt in this respect. While respondents are
not considered as belonging to the rank-and-file, they do not belong to
the upper echelon of PJI management either: De Guzman was
Executive Security to the Chairman, while Quirante was HR Supervisor
- not exactly juicy positions that find immediate favor with
management.

Operating under the honest belief that they could avail of an


optional retirement scheme that PJI allowed with respect to other
employees in the past, respondents tendered their respective
resignation letters on the sole ground that they were availing of the
company's optional retirement package. Instead of clarifying the matter
with respondents, petitioners treated the latter's actions with a lack of
understanding and sympathy. If petitioners believed that respondents
were not entitled to avail of the optional retirement scheme which
respondents in good faith thought was available to them, and which
was obviously the sole reason for tendering their resignations, then
petitioners should have at least put their respective resignations on
hold pending clarification of the issues. Instead, petitioners
immediately took a hostile stance, and quickly grabbed the opportunity
to declare respondents separated from PJI by voluntary resignation
with its concomitant effects such as non-payment of benefits,
separation pay, etc. They did not take time to explain, if so, that the
optional retirement program was no longer in effect and give
respondents the opportunity to reconsider their actions. This is
tantamount to bad faith, considering the factual milieu and petitioners'
conduct, where they have consistently shown an interest in dismissing
their employees, yet keeping for themselves their corporate bonuses,
perks, and privileges.

To be considered as a regular company practice, the employee


must prove by substantial evidence that the giving of the benefit is done
over a long period of time, and that it has been made consistently and
deliberately. Jurisprudence has not laid down any hard-and-fast rule
as to the length of time that company practice should have been
exercised in order to constitute voluntary employer practice. The
common denominator in previously decided cases appears to be the
regularity and deliberateness of the grant of benefits over a significant
period of time. It requires an indubitable showing that the employer

1055
Page 1056

agreed to continue giving the benefit knowing fully well that the
employees are not covered by any provision of the law or agreement
requiring payment thereof. In sum, the benefit must be characterized
by regularity, voluntary and deliberate intent of the employer to grant
the benefit over a considerable period of time.

The grant of optional retirement benefits to two management


employees in the past was voluntary, deliberate, and done with
sufficient regularity as would indicate that this had become a company
practice within PJI, which petitioners now refuse to apply in the case of
respondents, on the pretext that the company was losing money at that
time. But PJI was not incurring losses, and was in fact exhibiting
conduct inconsistent with the claim. What is clear is that it engaged in
unfair labor activities and took an anti-labor stance at the expense of
its employees, including respondents. PJI has shown that its
employees' interests take a backseat to the perks and prerogatives of
management. This cannot be countenanced.

1056
Page 1057

Case Digest by: Cerera, Florence Diane N.

PLDT v PINGOL
G.R. No. 182622. September 8, 2010.
Mendoza, J.

DOCTRINE:
PRESCRIPTION
The Labor Code has no specific provision on when a claim for
illegal dismissal or a monetary claim accrues. Thus, the general law on
prescription applies. Article 1150 of the Civil Code states:

Article 1150. The time for prescription for all kinds of


actions, when there is no special provision which ordains
otherwise, shall be counted from the day they may be
brought.
xxxxx

Like other causes of action, the prescriptive period for money


claims is subject to interruption, and in the absence of an equivalent
Labor Code provision for determining whether the said period may be
interrupted, Article 1155 of the Civil Code may be applied, to wit:

ART. 1155. The prescription of actions is interrupted when


they are filed before the Court, when there is a written
extrajudicial demand by the creditors, and when there is
any written acknowledgment of the debt by the debtor.

Thus, the prescription of an action is interrupted by (a) the filing of


an action, (b) a written extrajudicial demand by the creditor, and (c) a
written acknowledgment of the debt by the debtor.

FACTS:
Respondent Roberto Pingol was hired by petitioner PLDT as a
maintenance technician in 1979. On April 13, 1999, respondent was
admitted at The Medical City, Mandaluyong City due to paranoid
personality disorder caused by financial and marital problems. He as
later on discharged but frequently absented himself due to his poor
mental condition.

From September 16, 1999 to December 31, 1999, Pingol was


absent from work without official leave. PLDT claimed that notices
were given to respondent warning him that he would be dismissed from
employment if he continued to be absent without official leave
"pursuant to PLDT Systems Practice A-007 which provides that
‘Absence without authorized leaves for seven (7) consecutive days is

1057
Page 1058

subject to termination from the service.’" On January 1, 2000, PLDT


terminated his services on the grounds of unauthorized absences and
abandonment of office.

On March 29, 2004, four years later, Pingol filed a Complaint for
Constructive Dismissal and Monetary Claims against PLDT. Petitioner
PLDT moved for the dismissal of the complaint arguing that it was filed
beyond the prescriptive period, having been filed 4 years and 3 months
after respondent’s dismissal.

Respondent, on the other hand, countered that in computing the


prescriptive period, the years 2001 to 2003 must not be taken into
account since during that time he was inquiring from PLDT about the
benefits due him as an employee who was no longer allowed to do his
work, with such inquiry to be considered as non-abandonment of his
claim.

The Labor Arbiter dismissed complaint, citing Article 291 of the


Labor Code which provides:

The NLRC reversed the ruling of the Labor Arbiter.

The CA Affirmed NLRC’s ruling.

ISSUE:

Whether or not respondent’s money claim prescribed.

RULING:
Yes. The pivotal question in resolving the issues is the date when
the cause of action of respondent Pingol accrued. It is a settled
jurisprudence that a cause of action has three (3) elements, to wit: (1)
a right in favor of the plaintiff by whatever means and under whatever
law it arises or is created; (2) an obligation on the part of the named
defendant to respect or not to violate such right; and (3) an act or
omission on the part of such defendant violative of the right of the
plaintiff or constituting a breach of the obligation of the defendant to
the plaintiff.

The Labor Code has no specific provision on when a claim for


illegal dismissal or a monetary claim accrues. Thus, the general law on
prescription applies. Article 1150 of the Civil Code states:

Article 1150. The time for prescription for all kinds of


actions, when there is no special provision which ordains
otherwise, shall be counted from the day they may be
brought.

1058
Page 1059

The day the action may be brought is the day a claim starts as a
legal possibility. In the present case, January 1, 2000 was the date that
respondent Pingol was not allowed to perform his usual and regular
job as a maintenance technician. Respondent Pingol cited the same
date of dismissal in his complaint before the LA. As, thus, correctly
ruled by the LA, the complaint filed had already prescribed.

Respondent claims that between 2001 and 2003, he made


follow-ups with PLDT management regarding his benefits. This, to his
mind, tolled the running of the prescriptive period.

The rule in this regard is covered by Article 1155 of the Civil


Code. Its applicability in labor cases was upheld in the case of
International Broadcasting Corporation v. Panganiban where it was
written:

Like other causes of action, the prescriptive period for


money claims is subject to interruption, and in the absence
of an equivalent Labor Code provision for determining
whether the said period may be interrupted, Article 1155 of
the Civil Code may be applied, to wit:

ART. 1155. The prescription of actions is interrupted when


they are filed before the Court, when there is a written
extrajudicial demand by the creditors, and when there is
any written acknowledgment of the debt by the debtor.

Thus, the prescription of an action is interrupted by (a) the filing of


an action, (b) a written extrajudicial demand by the creditor, and (c) a
written acknowledgment of the debt by the debtor.

In this case, respondent Pingol never made any written


extrajudicial demand. Neither did petitioner make any written
acknowledgment of its alleged obligation. Thus, the claimed "follow-
ups" could not have validly tolled the running of the prescriptive period.
It is worthy to note that respondent never presented any proof to
substantiate his allegation of follow-ups.

1059
Page 1060

Case Digest by: Cerera, Florence Diane N.

SERRANO v CA
G.R. No. 182622. August 15, 2001.
Puno, J.

DOCTRINE:
PRESCRIPTION
It is settled jurisprudence that a cause of action has three
elements, to wit, (1) a right in favor of the plaintiff by whatever means
and under whatever law it arises or is created; (2) an obligation on the
part of the named defendant to respect or not to violate such right; and
(3) an act or omission on the part of such defendant violative of the
right of the plaintiff or constituting a breach of the obligation of the
defendant to the plaintiff .Petitioner's cause of action accrued only
upon respondent A.P. Moller's definite denial of his claim in November
1993. Having filed his action five (5) months thereafter or in April 1994,
we hold that it was filed within the three-year (3) prescriptive period
provided in Article 291 of the Labor Code.

FACTS:
From 1974 to 1991, respondent Maersk-Filipinas Crewing, Inc.,
the local agent of respondent foreign corporation A.P. Moller, deployed
petitioner Serrano as a seaman to Liberian, British and Danish ships.
As petitioner was on board a ship most of the time, respondent Maersk
offered to send portions of petitioner's salary to his family in the
Philippines. The amounts would be sent by money order. Petitioner
agreed and from 1977 to 1978 for respondent Maersk to send money
orders to his family. Respondent Maersk deducted the amounts of
these money orders totaling HK$4,600.00 and 1,050.00 Sterling
Pounds from petitioner's salary.

It appears that petitioner's family failed to receive the money


orders petitioner sent through respondent Maersk. Upon learning this
in 1978, petitioner demanded that respondent Maersk pay him the
amounts the latter deducted from his salary. Respondent Maersk
assured him that they would look into the matter, then assigned him
again to board one of their vessels.

Whenever he returned to the Philippines, petitioner would go to


the office of respondent Maersk to follow up his money claims but he
would be told to return after several weeks as respondent Maersk
needed time to verify its records and to bring up the matter with its
principal employer, respondent A.P. Moller. Meantime, respondent
Maersk would hire him again to board another one of their vessels for
about a year.

1060
Page 1061

Finally, in October 1993, petitioner wrote to respondent Maersk


demanding immediate payment to him of the total amount of the money
orders deducted from his salary from 1977 to 1978. On November 11,
1993, respondent A.P. Moller replied to petitioner that they keep
accounting documents only for a certain number of years, thus data on
his money claims from 1977 to 1978 were no longer available.
Likewise, it was claimed that it had no outstanding money orders. A.P.
Moller declined petitioner's demand for payment.
In April 1994, petitioner filed a complaint for collection of the total
amount of the unsent money orders and illegal salary deductions
against the respondent Maersk in the Philippine Overseas
Employment Agency (POEA).

The Labor Arbiter ruled in favor of the petitioner. Regarding to


complainant's claim for payment of and/or refund of seven (7) money
orders for the period covering 1977 to 1978, while the respondents
claim payment of that claim, it failed to present competent evidence of
payment such that this Office is constrained to approve this claim as
warranted. The Labor Arbiter ordered the respondent and/or TICO
Insurance Co., Inc. to refund to complainant his untransmitted money
order payment of HK$4,600 and 1,050 Sterling Pounds.

Respondent Maersk appealed to the NLRC the Labor Arbiter's


grant of the claim for the amount of unsent money orders. The NLRC
reversed and set side Labor Arbiter’s decision and dismissed the case
on the ground of prescription. Under Article 291 of the Labor Code.
Money claims. All money claims arising from employer-employee
relations accruing during the effectivity of this Code shall be filed within
three years from the time the cause of action accrued, otherwise they
shall be forever barred.

The appellate court dismissed his petition for having been filed
out of time.

Petitioner contends that his cause of action accrued only in 1993


when respondent A.P. Moller wrote to him that its accounting records
showed it had no outstanding money orders and that his case was
considered outdated. Thus, the three (3) year prescriptive period
should be counted from 1993 and not 1978 and since his complaint
was filed in 1994, he claims that it has not prescribed.

ISSUE:

Whether or not the claim of the petitioner has prescribed.

RULING:
No. The applicable law is Article 291 of the Labor Code, viz:

1061
Page 1062

"ARTICLE 291. Money claims. All money claims arising


from employer-employee relations accruing during the
effectivity of this Code shall be filed within three years from
the time the cause of action accrued, otherwise they shall
be forever barred."

Petitioner's cause of action accrued in November 1993 upon respondent


Maersk's definite denial of his money claims. Petitioner repeatedly demanded
payment from respondent Maersk but similar to the actuations of Baliwag Transit in
the above cited case, respondent Maersk warded off these demands by saying that it
would look into the matter until years passed by. In October 1993, Serrano finally
demanded in writing payment of the unsent money orders. Then and only then was
the claim categorically denied by respondent A.P. Moller in its letter dated November
22, 1993. It is settled jurisprudence that a cause of action has three elements, to wit,
(1) a right in favor of the plaintiff by whatever means and under whatever law it arises
or is created; (2) an obligation on the part of the named defendant to respect or not to
violate such right; and (3) an act or omission on the part of such defendant violative of
the right of the plaintiff or constituting a breach of the obligation of the defendant to the
plaintiff .Petitioner's cause of action accrued only upon respondent A.P. Moller's
definite denial of his claim in November 1993. Having filed his action five (5) months
thereafter or in April 1994, we hold that it was filed within the three-year (3) prescriptive
period provided in Article 291 of the Labor Code.

1062
Page 1063

Case Digest by: JAZMIN, JENDY F.

IBC v PANGANIBAN
G.R. No. 151407. February 06, 2007.
Austria-Martinez, J.

DOCTRINE:
PRESCRIPTION OF MONEY CLAIMS
Although the commencement of a civil action stops the running
of the statute of prescription or limitations, its dismissal or voluntary
abandonment by plaintiff leaves the parties in exactly the same
position as though no action had been commenced at all.

FACTS:
Ireneo Panganiban was employed as Assistant General Manager
of the Intercontinental Broadcasting Corporation from May 1986 until
his preventive suspension on August 1988. Panganiban resigned from
his employment on September 1988.

On April 1989, he filed with the Regional Trial Court of Quezon


City a civil case against the members of the Board of Administrators of
IBC for the non-payment of his unpaid commissions.

A motion to dismiss was filed by Joselito Santiago on the ground


of lack of jurisdiction, as respondent’s claim was a labor money claim,
but this was denied by the RTC. Thus, Santiago filed a petition for
certiorari with the Court of Appeals, which the latter granted. The CA
ordered petitioner to pay respondent the amount of unpaid
commissions.

Petitioner contended that the respondent’s claim for alleged


unpaid commission has already prescribed because he has not filed
his claim within the three year prescriptive period and before a
competent labor arbiter who ought to hear his alleged money claims
within the proper time.

ISSUE:
Whether or not respondent’s claim for unpaid commission has
already prescribed?

RULING:
YES. The applicable law in this case is Article 291 of the Labor
Code which provides that “all money claims arising from empoyer-
employee relations accruing during the effectivity of this Code shall be

1063
Page 1064

filed within three (3) years from the time the cause of action accrued;
otherwise they shall be forever barred." The term "money claims"
covers all money claims arising from an employer-employee relation.

The prescription of an action is interrupted when they are filed


before the Court, when there is a written acknowledgement of the debt
by the debtor. However, the Court ruled that although the
commencement of a civil action stops the running of the statute of
prescription or limitations, its dismissal or voluntary abandonment by
plaintiff leaves the parties in exactly the same position as though no
action had been commenced at all.

Hence, while the filing of civil case could have interrupted the
running of the three-year prescriptive period, its consequent dismissal
by the CA due to lack of jurisdiction effectively canceled the tolling of
the prescriptive period within which to file his money claim, leaving
respondent in exactly the same position as though no civil case had
been filed at all.

1064
Page 1065

Case Digest by: JAZMIN, JENDY F.

ACCESSORIES SPECIALIST v ALABANZA


G.R. No. 168985. July 23, 2008.
Nachura, J.

DOCTRINE:
PRESCRIPTION OF MONEY CLAIMS
Promissory estoppel may arise from the making of a promise,
even though without consideration, if it was intended that the promise
should be relied upon, as in fact it was relied upon, and if a refusal to
enforce it would virtually sanction the perpetration of fraud or would
result in other injustice.

FACTS:
Erlinda Alabanza, for and in behalf of her husband Jones
Alabanza, filed a complaint against Accessories Specialists, Inc. also
known as ARTS 21 Corporation, and Tadahiko Hashimoto for non-
payment of salaries, separation pay, and 13th month pay.

Erlinda alleged that her husband Jones was the Vice-President,


Manager and Director of ASI. Jones rendered outstanding services for
the petitioners from 1975 to 1997. On October 17, 1997, Jones was
compelled by the owner of ASI, herein Hashimoto, to file his involuntary
resignation on the ground that ASI allegedly suffered losses due to lack
of market and incurred several debts caused by a slam in the market.
At the time of his resignation, Jones had unpaid salaries for 18 months.
He was likewise not paid his separation pay commensurate to his 21
years of service and 13th month pay. Jones demanded payment of his
money claims upon resignation but ASI informed him that it would just
settle first the money claims of the rank-and-file employees, and his
claims will be paid thereafter.

The petitioners contend that Jones voluntarily resigned on


October 31, 1997. Thus, Erlinda’s cause of action has already
prescribed and is forever barred on the ground that under Article 291
of the Labor Code, all money claims arising from an employer-
employee relationship shall be filed within 3 years from the time the
cause of action accrues. Since the complaint was filed only on
September 27, 2002, or almost 5 years from the date of the alleged
illegal dismissal of Jones, Erlinda’s complaint is now barred.

ISSUE:
Whether or not respondent’s money claim already prescribed?

1065
Page 1066

RULING:
NO. Based on the findings of facts of the LA, it was ASI which was
responsible for the delay in the institution of the complaint. When Jones
filed his resignation, he immediately asked for the payment of his
money claims. However, the management of ASI promised him that he
would be paid immediately after the claims of the rank-and-file
employees had been paid. Jones relied on this representation.
Unfortunately, the promise was never fulfilled even until the time of
Jones’ death.

In light of these circumstances, we can apply the principle of


promissory estoppel, which is a recognized exception to the three-year
prescriptive period enunciated in Article 291 of the Labor Code.

Promissory estoppel may arise from the making of a promise,


even though without consideration, if it was intended that the promise
should be relied upon, as in fact it was relied upon, and if a refusal to
enforce it would virtually sanction the perpetration of fraud or would
result in other injustice. Promissory estoppel presupposes the
existence of a promise on the part of one against whom estoppel is
claimed. The promise must be plain and unambiguous and sufficiently
specific so that the court can understand the obligation assumed and
enforce the promise according to its terms.

Jones relied on the promise of ASI that he would be paid as soon


as the claims of all the rank-and-file employees had been paid. If not
for this promise that he had held on to until the time of his death, we
see no reason why he would delay filing the complaint before the LA.
Thus, we find ample justification not to follow the prescriptive period
imposed under Article 291 of the Labor Code. Great injustice will be
committed if we will brush aside the employee's claims on a mere
technicality, especially when it was petitioner's own action that
prevented respondent from interposing the claims within the required
period.

1066
Page 1067

Case Digest by: JAZMIN, JENDY F.

AUTO BUS TRANSPORT SYSTEMS, INC. v BAUTISTA


G.R. No. 156367. May 16, 2005.
Chico-Nazario, J.

DOCTRINE:
PRESCIPTIVE PERIOD FOR CLAIM OF PAYMENT OF SERVICE
INCENTIVE LEAVE
Applying Article 291 of the Labor Code in light of this peculiarity of
the service incentive leave, we can conclude that the 3-year
prescriptive period commences, not at the end of the year when the
employee becomes entitled to the commutation of his service incentive
leave, but from the time when the employer refuses to pay its monetary
equivalent after demand of commutation or upon termination of the
employees services, as the case may be.

FACTS:
Antonio Bautista has been employed by Auto Bus Transport
Systems, Inc. as driver-conductor. Respondent was paid on
commission basis, 7% of the total gross income per travel, on a twice
a month basis.

While Bautista was driving, the bus he was driving accidentally


bumped the rear portion of another Auto Bus, as the latter vehicle
suddenly stopped at a sharp curve without giving any warning.

Bautista averred that the accident happened because he was


compelled by the management to go back to Roxas, Isabela, although
he had not slept for almost 24 hours, as he had just arrived in Manila
from Roxas, Isabela. He further alleged that he was not allowed to work
until he fully paid the cost of repair of the damaged buses. Afterwards,
the management sent him a letter of termination.

Bautista filed a complaint for illegal dismissal with claims for non-
payment of 13th month pay and the service incentive leave. Auto Bus
avers that in the exercise of its management prerogative, Bautista’s
employment was terminated only after the latter was provided with an
opportunity to explain his side regarding the accident.

The NLRC ruled that Auto Bus is liable for the payment of
Bautista’s service incentive leave pay.

ISSUE:

1067
Page 1068

Whether or not the 3 year prescriptive period in Article 291 of the


Labor Code applicable to respondent’s claim of SIL pay?

RULING:
NO. Article 291 of the Labor Code states that all money claims
arising from employer-employee relationship shall be filed within 3
years from the time the cause of action accrued; otherwise, they shall
be forever barred. In the application of this section of the Labor Code,
the pivotal question to be answered is when does the cause of action
for money claims accrue in order to determine the reckoning date of
the three-year prescriptive period.

In cases of nonpayment of allowances and other monetary


benefits, if it is established that the benefits being claimed have been
withheld from the employee for a period longer than three (3) years,
the amount pertaining to the period beyond the three-year prescriptive
period is therefore barred by prescription. The amount that can only be
demanded by the aggrieved employee shall be limited to the amount
of the benefits withheld within three (3) years before the filing of the
complaint.

It can be conscientiously deduced that the cause of action of an


entitled employee to claim his service incentive leave pay accrues from
the moment the employer refuses to remunerate its monetary
equivalent if the employee did not make use of said leave credits but
instead chose to avail of its commutation. Accordingly, if the employee
wishes to accumulate his leave credits and opts for its commutation
upon his resignation or separation from employment, his cause of
action to claim the whole amount of his accumulated service incentive
leave shall arise when the employer fails to pay such amount at the
time of his resignation or separation from employment.

Applying Article 291 of the Labor Code in light of this peculiarity of


the service incentive leave, we can conclude that the three (3)-year
prescriptive period commences, not at the end of the year when the
employee becomes entitled to the commutation of his service incentive
leave, but from the time when the employer refuses to pay its monetary
equivalent after demand of commutation or upon termination of the
employees services, as the case may be.

In the case at bar, respondent had not made use of his service
incentive leave nor demanded for its commutation until his employment
was terminated by petitioner. Neither did petitioner compensate his
accumulated service incentive leave pay at the time of his dismissal. It
was only upon his filing of a complaint for illegal dismissal, one month
from the time of his dismissal, that respondent demanded from his

1068
Page 1069

former employer commutation of his accumulated leave credits. His


cause of action to claim the payment of his accumulated service
incentive leave thus accrued from the time when his employer
dismissed him and failed to pay his accumulated leave credits.

Therefore, the prescriptive period with respect to his claim for


service incentive leave pay only commenced from the time the
employer failed to compensate his accumulated service incentive
leave pay at the time of his dismissal. Since respondent had filed his
money claim after only one month from the time of his dismissal,
necessarily, his money claim was filed within the prescriptive period
provided for by Article 291 of the Labor Code.

1069
Page 1070

Case Digest by: JAZMIN, JENDY F.

MONTERO v TIMES TRANSPORTATION CO., INC.


G.R. No. 190828. March 16, 2015.
Reyes, J.

DOCTRINE:
PRESCIPTIVE PERIOD IN LABOR CASES
Settled is the rule that when one is arbitrarily and unjustly deprived
of his job or means of livelihood, the action instituted to contest the
legality of one’s dismissal from employment constitutes, in essence, an
action predicated upon an injury to the rights of the plaintiff, as
contemplated under Article 1146⁠2 of the New Civil Code, which must
be brought within four years.

FACTS:
Times Transportation Co., Inc., is a company engaged in the
business of land transportation for passengers and goods serving the
Ilocos Region to Metro Manila route. TTCI employed Montero and his
co-petitioners as bus drivers, conductors, mechanics, welders, security
guards and utility personnel. The sale of 25 buses of TCCI, as well as
the Certificates of Public Convenience for the operation of the buses
were approved and subsequently transferred to Mencorp Tranport
Systems, Inc. Several union members received notices that they were
being retrenched.

TEU then declared a strike against TCCI, but the latter merely
reiterated the earlier return-to-work order of the Labor Secretary. For
disregarding the said return-to-work order, respondent Santiago
issued two notices of termination dated October 26, 1997 terminating
some 106 workers and a revised list dated November 24, 1997
increasing the number of dismissed employees to 119, for participating
in the illegal strike.

On May 14, 1998, petitioners filed several complaints against


respondents Times and Mencorp. Said complaints were consolidated
under the case entitled Malana v. TTCI docketed as NLRC RAB-I-01-
1007. However, said case was withdrawn on March 4, 1999 upon
motion by the TEU’s counsel which was given due course on March
22, 1999.

Four years later, several complaints for unfair labor practice, illegal
dismissal with money claims, damages and attorney’s fees were filed
against respondents TTCI, Santiago, MENCORP and its General

1070
Page 1071

Manager Virginia Mendoza, including the latter’s husband Reynaldo


Mendoza (collectively called the respondents), before the LA from
June to July 2002. Accordingly, these complaints were consolidated.

However, respondent TTCI averred that petitioners’ cause of


action had already prescribed (following Art. 291, now Art. 306)
because the complaints were filed only in June 2002 or after almost
five years from the date of their dismissal.

ISSUE:
Whether or not the petitioners’ complaint for illegal dismissal
already prescribed?

RULING:
YES. Settled is the rule that when one is arbitrarily and unjustly
deprived of his job or means of livelihood, the action instituted to
contest the legality of one’s dismissal from employment constitutes, in
essence, an action predicated upon an injury to the rights of the
plaintiff, as contemplated under Article 1146⁠2 of the New Civil Code,
which must be brought within four years.

The petitioners contend that the period when they filed a labor
case on May 14, 1998 but withdrawn on March 22, 1999 should be
excluded from the computation of the four-year prescriptive period for
illegal dismissal cases. However, the Court had already ruled that the
prescriptive period continues even after the withdrawal of the case as
though no action has been filed at all. The applicability of Article 1155
of the Civil Code in labor cases was upheld in the case of
Intercontinental Broadcasting Corporation v. Panganiban where the
Court held that “although the commencement of a civil action stops the
running of the statute of prescription or limitations, its dismissal or
voluntary abandonment by plaintiff leaves the parties in exactly the
same position as though no action had been commenced at all.”

Sadly, the petitioners have no one but themselves to blame for


their own predicament. By their own allegations in their respective
complaints, they have barred their remedy and extinguished their right
of action. Although the Constitution is committed to the policy of social
justice and the protection of the working class, it does not necessary
follow that every labor dispute will be automatically decided in favor of
labor. The management also has its own rights. Out of concern for the
less privileged in life, this Court, has more often than not inclined, to
uphold the cause of the worker in his conflict with the employer. Such
leaning, however, does not blind the Court to the rule that justice is in
every case for the deserving, to be dispensed in the light of the
established facts and applicable law and doctrine.

1071
Page 1072

Case Digest by: JAZMIN, JENDY F.

CALLANTA v CARNATION PHILIPPINES, INC.


G.R. No. 70615. October 28, 1986.
Fernan, J.

DOCTRINE:
PRESCIPTIVE IN LABOR CASES
Settled is the rule that when one is arbitrarily and unjustly deprived
of his job or means of livelihood, the action instituted to contest the
legality of one’s dismissal from employment constitutes, in essence, an
action predicated upon an injury to the rights of the plaintiff, as
contemplated under Article 1146⁠2 of the New Civil Code, which must
be brought within four years.

FACTS:
Virgilio Callanta was employed by Carnation Philippines, Inc. as a
salesman in the Agusan del Sur area. 51 years later, Carnation filed
with the Regional Office of the Ministry Labor and Employment an
application for clearance to terminate the employment of Callanta on
the alleged grounds of serious misconduct and misappropriation of
company funds amounting to P12,000.00. Upon approval of the said
clearance application, Callanta’s employment with Carnation was
terminated.

Callanta filed with the MOLE a complaint for illegal dismissal with
claims for reinstatement, backwages, and damages against Carnation.
In its position paper

ISSUE:
Whether or not the petitioners’ complaint for illegal dismissal
already prescribed?

RULING:
YES. Settled is the rule that when one is arbitrarily and unjustly
deprived of his job or means of livelihood, the action instituted to
contest the legality of one’s dismissal from employment constitutes, in
essence, an action predicated upon an injury to the rights of the
plaintiff, as contemplated under Article 1146⁠2 of the New Civil Code,
which must be brought within four years.

The petitioners contend that the period when they filed a labor
case on May 14, 1998 but withdrawn on March 22, 1999 should be
excluded from the computation of the four-year prescriptive period for

1072
Page 1073

illegal dismissal cases. However, the Court had already ruled that the
prescriptive period continues even after the withdrawal of the case as
though no action has been filed at all. The applicability of Article 1155
of the Civil Code in labor cases was upheld in the case of
Intercontinental Broadcasting Corporation v. Panganiban where the
Court held that “although the commencement of a civil action stops the
running of the statute of prescription or limitations, its dismissal or
voluntary abandonment by plaintiff leaves the parties in exactly the
same position as though no action had been commenced at all.”

Sadly, the petitioners have no one but themselves to blame for


their own predicament. By their own allegations in their respective
complaints, they have barred their remedy and extinguished their right
of action. Although the Constitution is committed to the policy of social
justice and the protection of the working class, it does not necessary
follow that every labor dispute will be automatically decided in favor of
labor. The management also has its own rights. Out of concern for the
less privileged in life, this Court, has more often than not inclined, to
uphold the cause of the worker in his conflict with the employer. Such
leaning, however, does not blind the Court to the rule that justice is in
every case for the deserving, to be dispensed in the light of the
established facts and applicable law and doctrine.

1073
Page 1074

Case Digest by: JAZMIN, JENDY F.

ARRIOLA v PILIPINO STAR NGAYON


G.R. No. 70615. October 28, 1986.
Fernan, J.

DOCTRINE:
PRESCIPTIVE PERIOD FOR ACTION INVOLVING ILLEGAL
DISMISSAL
It is a principle in American jurisprudence which, undoubtedly, is
well-recognized in this jurisdiction that one's employment, profession,
trade or calling is a "property right," and the wrongful interference
therewith is an actionable wrong. The right is considered to be property
within the protection of a constitutional guaranty of due process of law.
Clearly then, when one is arbitrarily and unjustly deprived of his job or
means of livelihood, the action instituted to contest the legality of one's
dismissal from employment constitutes, in essence, an action
predicated "upon an injury to the rights of the plaintiff," as contemplated
under Art. 1146 of the New Civil Code, which must be brought within 4
years.

FACTS:
Virgilio Callanta was employed by Carnation Philippines, Inc. as a
salesman in the Agusan del Sur area. 51 years later, Carnation filed
with the Regional Office of the Ministry Labor and Employment an
application for clearance to terminate the employment of Callanta on
the alleged grounds of serious misconduct and misappropriation of
company funds. Upon approval of the said clearance application,
Callanta’s employment with Carnation was terminated.

Callanta filed with the MOLE a complaint for illegal dismissal with
claims for reinstatement, backwages, and damages against Carnation.
In its position paper. In its position paper, Carnation put in issue the
timeliness of Callanta’s complaint alleging that the same is barred by
prescription for having been filed more than 3 years after the date of
Callanta’s dismissal.

Petitioner contends that since the Labor Code is silent as to the


prescriptive period of an action for illegal dismissal with claims for
reinstatement, backwages and damages, the applicable law, by way of
supplement, is Article 1146 of the New Civil Code which provides a
four [4]-year prescriptive period for an action predicated upon "an injury
to the rights of the plaintiff" considering that an action for illegal
dismissal is neither a "penal offense" nor a mere "money claim," as

1074
Page 1075

contemplated under Articles 291 and 292, respectively, of the Labor


Code.

ISSUE:
Whether or not an action for illegal dismissal prescribes in 3 years
pursuant to Articles 291 and 292 of the Labor Code?

RULING:
NO. The said action will prescribe in 4 years based on Article 1146
of the Civil Code. In the case of illegal dismissal, no penalty of fine nor
imprisonment is imposed on the employer upon a finding of illegality in
the dismissal. By the very nature of the reliefs sought, therefore, an
action for illegal dismissal cannot be generally categorized as an
“offense” as used under Article 291 of the Labor Code, which must be
brought within the period of 3 years from the time the cause of action
accrued.

It is true that the “backwages” sought by an illegally dismissed


employee may be considered, by reason of its practical effect, as a
“money claim.” However, it is not the principal cause of action in an
illegal dismissal case but the unlawful deprivation of the one’s
employment committed by the employer in violation of the right of an
employee. Backwages is merely one of the reliefs which an illegally
dismissed employee prays the LA and the NLRC to render in his favor
as a consequence of the unlawful act committed by the employer. The
award thereof is not private compensation or damages but is in
furtherance and effectuation of the public objectives of the Labor Code
even though the practical effect is the enrichment of the individual, the
award of backwages is not inredness of a private right, but, rather, is
in the nature of a command upon the employer to make public
reparation for his violation of the Labor Code.

It is a principle in American jurisprudence which, undoubtedly, is


well-recognized in this jurisdiction that one's employment, profession,
trade or calling is a "property right," and the wrongful interference
therewith is an actionable wrong. The right is considered to be property
within the protection of a constitutional guaranty of due process of law.
Clearly then, when one is arbitrarily and unjustly deprived of his job or
means of livelihood, the action instituted to contest the legality of one's
dismissal from employment constitutes, in essence, an action
predicated "upon an injury to the rights of the plaintiff," as contemplated
under Art. 1146 of the New Civil Code, which must be brought within 4
years.

1075
Page 1076

Case Digest by: JAZMIN, JENDY F.

CALADIN v POEA
G.R. No. L-104776. December 5, 1994.
Quiason, J.

DOCTRINE:
PRESCIPTIVE PERIOD FOR ACTION INVOLVING FOREIGN LAW
The Labor Code provides that "all money claims arising from employer-
employee relations . . . shall be filed within three years from the time
the cause of action accrued; otherwise they shall be forever barred.”
This three-year prescriptive period shall be the one applied here and
which should be reckoned from the date of repatriation of each
individual complainant, considering the fact that the case is having (sic)
filed in this country.

FACTS:
Cadalin et al. are Filipino workers recruited by Asia International
Builders Co., a domestic recruitment corporation, for employment in
Bahrain to work for Brown & Root International Inc. (BRII) which is a
foreign corporation with headquarters in Texas. Plaintiff instituted a
class suit with the POEA for money claims arising from the unexpired
portion of their employment contract which was prematurely
terminated. They worked in Bahrain for BRII and they filed the suit after
1 year from the termination of their employment contract.

As provided by Art. 156 of the Amiri Decree aka as the Labor Law
of the Private Sector of Bahrain: “a claim arising out of a contract of
employment shall not be actionable after the lapse of 1 year from the
date of the expiry of the contract,” it appears that their suit has
prescribed.

Plaintiff contends that the prescription period should be 10 years


as provided by Art. 1144 of the Civil Code as their claim arise from a
violation of a contract.

The POEA Administrator holds that the 10-year period of


prescription should be applied but the NLRC provides a different view
asserting that Art 291 of the Labor Code of the Philippines with a 3
years prescription period should be applied. The Solicitor General
expressed his personal point of view that the 1-year period provided
by the Amiri Decree should be applied.

1076
Page 1077

ISSUE:
Whether or not the prescriptive period for the filing of the claims of
the complainants was 3 years?

RULING:
YES. The Labor Code provides that "all money claims arising from
employer-employee relations . . . shall be filed within three years from
the time the cause of action accrued; otherwise they shall be forever
barred" (Art. 291, Labor Code, as amended). This three-year
prescriptive period shall be the one applied here and which should be
reckoned from the date of repatriation of each individual complainant,
considering the fact that the case is having (sic) filed in this country.
We do not agree with the POEA Administrator that this three-year
prescriptive period applies only to money claims specifically
recoverable under the Philippine Labor Code. Article 291 gives no such
indication. Likewise, We cannot consider complainants' cause/s of
action to have accrued from a violation of their employment contracts.
There was no violation; the claims arise from the benefits of the law of
the country where they worked.

The contention of the POEA Administrator, that the three-year


prescriptive period under Article 291 of the Labor Code of the
Philippines applies only to money claims specifically recoverable under
said Code, does not find support in the plain language of the provision.
Neither is the contention of the claimants in G.R. Nos. 104911-14 that
said Article refers only to claims "arising from the employer's violation
of the employee's right," as provided by the Labor Code supported by
the facial reading of the provision.

1077

You might also like